@StudyPivot Plancess Maths 11 PDF
@StudyPivot Plancess Maths 11 PDF
op kers
Class 11 T
By E ran culty
-JE Fa r
IIT enior emie .
S fP r es
o titut
Ins
MATHEMATICS
FOR JEE MAIN & ADVANCED
SECOND
EDITION
Exhaustive Theory
(Now Revised)
Formula Sheet
9000+ Problems
based on latest JEE pattern
PlancEssential
Questions recommended for revision
1. B A S I C M AT H E M AT I C S
NUMBER SYSTEM
Imaginary
Complex Number
x 0, y 0
(z = x + iy)
Pure Imaginary
x 0, y 0
Also known as
counting number
Natural
Number (N) Zero is a special number, it doesn’t
All the integers quite obey all the same laws as
excluding zero other numbers i.e. you cannot
divide by zero
Prime Composite
Number Number
1. NUMBER SYSTEM
(a) Natural Numbers: The counting numbers 1, 2, 3, 4, ……. are called natural numbers. The set of natural
numbers is denoted by N.
N = {1, 2, 3, 4, ……..} N is also denoted by I’ or Z’
(b) Whole Numbers: Natural numbers including zero are called whole numbers. The set of whole numbers is
denoted by W.
Thus W = {0, 1, 2, ……}
(c) Integers: The numbers ….. -3, -2, -1, 0, 1, 2, 3 …….. are called integers and the set of integers is denoted by I
or Z.
Thus I (or Z) = {……-3, -2, -1, 0, 1, 2, 3……..}
(a) Set of negative integers is denoted by I¯ and consists of {……., -3, -2, -1}
(b) Set of non-negative integers is denoted by W.
(c) Set of non-positive integers {……., -3, -2, -1, 0}
(d) Even integers: Integers which are divisible by 2 are called even integers. e.g. 0, ±2, ± 4, ………
(e) Odd integers: Integers which are not divisible by 2 are called odd integers. e.g. ±1, ±3, ±5, ±7
(f) Prime numbers: A natural number (except unity) is said to be a prime number if it is exactly divisible by unity
and itself only. e.g. 2, 3, 5, 7, 11, 13, 17, 19, 23, 29, 31, ……
(g) Composite numbers: Natural numbers which are not prime (except unity) are called composite numbers.
(h) Co-prime numbers: Two natural numbers (not necessarily prime) are said to be co-prime, if their H.C.F.
(Highest common factor) is one. e.g. (1, 2), (1, 3), (3, 4), (3, 10), (3, 8), (4, 9), (5, 6), (7, 8) etc. These numbers are
also called as relatively prime numbers.
(i) Twin prime numbers: If the difference between two prime numbers is two, then the numbers are called twin
prime numbers. e.g. {3, 5}, {5, 7}, {11, 13}, {17, 19}, {29, 31}
( j) Rational numbers: All the numbers that can be represented in the form p/q, where p and q are integers and
1 22
q ≠ 0, are called rational numbers and their set is denoted by Q. e.g. , 2, 0, -5, , 2.5, 0.3333 …….. etc. Thus
2 7
p
Q = : p,q ∈ I and q ≠ 0 . It may be noted that every integer is a rational number since it can be written as
q
p/1. The decimal part of rational numbers is either terminating or recurring.
(k) Irrational numbers: There are real numbers which cannot be expressed in p/q form. These numbers are
called irrational numbers and their set is denoted by QC or Q (i.e. complementary set of Q). The decimal part
of irrational numbers is neither terminating nor recurring e.g. 2 , 1 + 3 , π etc.
(l) Real numbers: The complete set of rational and irrational numbers is the set of real numbers and is denoted
by R. Thus R = Q ∪ QC.
(m) Complex numbers: A number of the form a + ib is called a complex number, where a, b ∈ R and i = −1 . A
complex number is usually denoted by ‘z’ and a set of complex numbers is denoted by C.
PLANCESS CONCEPTS
•• Zero is neither positive nor negative but zero is non-negative and non-positive.
•• ‘1’ is neither prime nor composite
•• ‘2’ is the only even prime number
•• ‘4’ is the smallest composite number
M a them a tics | 1.3
•• Two distinct prime numbers are always co-prime but the converse need not be true.
•• Consecutive natural numbers are always co-prime numbers.
e ≈ 2.71 is called Napier’s constant and π ≈ 3.14. And both are irrational.
2.1 Ratio
(a) If A and B are two quantities of the same kind, then their ratio is A : B; which may be denoted by the fraction
A
(this may be an integer or fraction)
B
a ma na
(b) A ratio may be represented in a number of ways e.g.= = = ……… where m, n, …….. are non-zero
b mb nb
numbers.
(c) To compare two or more ratios, reduce them to their common denominator.
2.2 Proportion
a c
When two ratios are equal, then the four quantities composing them are said to be proportional. If = , then it
is written as a : b = c : d or a : b :: c : d b d
(a) ‘a’ and ‘d’ are known as extremes and ‘b’ and ‘c’ are known as means.
(b) An important property of proportion; product of extremes = product of means.
(c) If a: b = c : d, then b : a = d : c (invertendo)
3. DEFINITION OF INDICES
If ‘a’ is any non-zero real or imaginary number and ‘m’ is a positive integer, then am = a.a.a…...a (m times). Here ‘a’
is called the base and m is the index, power, or exponent.
Law of indices:
(a) a0 =1 , (a ≠ 0)
1
(b) a-m = , (a ≠ 0)
am
(c) am+n = am .an , where m and n are real numbers
am
(d) am–n = , where m and n are real numbers
an
1 . 4 | Basic Mathematics
(c) a2 - b2 = (a + b) (a - b)
5. SURDS
3 4 3
Any root of an arithmetical number which cannot be completely found is called surd. E.g. 2, 5, 7 etc. are all
surds.
(a) Pure Surd: A surd which consists of purely an irrational number expressed as n where a ≠ xn (x∈I) is called
3 5
a pure surd. e.g. 7, 5 etc.
3 5
(b) Mixed surd: A pure surd when multiplied with a rational number becomes a mixed surd. e.g. 2 3,4 5,2
3 etc.
3 3 3
A mixed surd can be written as a pure surd. e.g. 2 × 3 = 3× 8 = 24 , 2 5 = 20
4 3 6
(c) Order of Surd: The order of a surd is indicated by the number denoting the roots i.e. 2, 5, 7 are surds
of the 4th, 3rd and 6th order respectively.
5 3 3
(d) Simple Surd: Surds consisting of one term only are called simple surds. E.g. 2, 3, a2bc etc. are simple
surds or Monomial surds.
(e) Compound Surd: An expression consisting of two or more simple surds connected by (+) or (-) sign is called
a compound surd. E.g. 5 2 +4 3, 3 + 2, 3‒ 5.
M a them a tics | 1.5
6 LOGARITHM
6.1 Introduction
( 45.5)
2
2
It is very lengthy and time consuming to find the value of 5 0.0000165 , or finding number of
(3.2) ( 6.5)
2 2
digits in 312, 28. John Napier (1550-1617 AD) invented logarithm (in 1614 AD) to solve such problems. The word
“Logarithm” was formed by two Greek words, ‘logos’ which means ‘ratio’, and ‘arithmos’ meaning ‘number’. Henry
Briggs (1556-1630 AD) introduced common logarithm. He published logarithm in 1624 AD.
In its simplest form, a logarithm answers the question, “How many of one number do we multiply to get another
number?”
Sol: 2 × 2 × 2 = 8, So we needed to multiply 3 of the 2s to get 8. So the logarithm of 8 to the base 2, written as
log2 ( 8 ) is 3.
Base
6.1.2 Exponents
Exponents and Logarithms are related, let’s find out how…
Figure 1.2
?
exponent
2 = 8
3
2 = 8 log2(8) = 3
log2(8) = 3 base
(a) (b)
Figure 1.3
1 . 6 | Basic Mathematics
loga(y) = x
Figure 1.4
Note:
(a) The logarithm of a number is unique i.e. No number can have two different log to a given base.
loga N
(b) From the definition of the logarithm of the number to a given base ‘a’. a = N, a > 0, a ≠ 1 and N > 0 is
known as the fundamental logarithmic identity.
(c) The base of log can be any positive number other than 1, but basically two bases are mostly used. They are
10 and e (= 2.718 approx.)
Logarithm of a number to the base 10 are named as common logarithm, whereas the logarithm of numbers to the
base e are called as Natural or Napierian logarithm.
So make sure that when you read “log” that you know what base they mean.
Note: Since NCERT assumed log x to be loge x, for JEE Main and Advanced this convention is to be used.
⇒ (M × N) = ax + y ⇒ loga (M × N) = x + y
⇒ loga(M × N) = loga M + logaN
1
(d) log x M = loga M (x ≠ 0) (Power rule for base)
a x
Proof: Let log x M = y … (i)
a
1
logaM = y … (ii)
x
1
Using (i) and (ii) log x M = loga M
a x
logc a log a
(e) logb a = (c > 0, c ≠ 1) = (Base changing theorem)
logc b log b
5
Illustration 2: What is logarithm of 32 4 to the base 2 2 (JEE MAIN)
3
4 as 25 41/5 = ( 2 )
5 27/5
Sol: Here we can write 32 and 2 2 as 22 and then by using the formulae logaMx = x loga .M
1
and log x M = loga M we can solve it.
a x
log2 2
325=
4 log
2
3/2
(2
5
)
41/5= log
2
3/2
(2)
27/5
=
2 27
3 5
log2=
18
2 = 3.6
5
4
Sol: By solving we get 1.3 = , and use the formula logaa = 1 .
3
log4/3 1.3 = 1
Let x = 1.333 … … (i)
10x = 13.3333 … ... (ii)
From Equation (i) and (ii), we get
So 9x = 12 ⇒ x = 12/9, x = 4/3;
Now log4/3 1 / 3 = log4/3 (4/3) = 1
Illustration 4: If N = n! (n ∈ N, n ≥ 2) then lim [(log2N)–1 + (log3N)–1 + … + (logn N)–1] is (JEE ADVANCED)
N→ ∞
1
Sol: Here by using logab = we can write given expansion as logN2 + logN3 + …… + logNn and then by using
logb a
loga (M.N) = logaM + logaN and N = n! we can solve this.
(M.N)
Sol: By using loga = logaM + logaN and logaMx = x loga .M we can easily solve above problem.
Clearly x > 0. Then the given equation can be written as 2 log x – log 2 – log x = 3 log 3 – log 2 – log 3
⇒ log x = 2 log 3 ⇒ x = 9
1
Sol: By multiplying and dividing by 2 + 3 to 2 − 3 we will get 2 + 3 = . Therefore by using log1/N N = –1
we can easily prove this. 2 − 3
1
(2 − 3 ) (2 − 3 ) = −1
−1
⇒ log2− 3
⇒ log 2− 3
⇒ −1.log2− 3
2− 3
Sol: Here 5 5 5.......∞ can be represented as y = 5y where y = 5 5 5.......∞ . Hence, by obtaining the value
of y we can prove this.
Let y = 5 5 5.......∞
M a them a tics | 1.9
y= 5y ⇒ y2 = 5y or y2 – 5y = 0
y(y – 5) = 0 ⇒ y = 0, y = 5
y = 0 is not possible because log is not defined for zero.
∴ log55 = 1
x = 0.4444..... … (i)
10x = 4.4444..... … (ii)
Equ (ii) – Equ (i)
So 9x = 4 ⇒ x = 4/9
225 9 4
Also, 2.25
= = ; ( )
log2.25 0.4 = log 9 = −1
100 4 9
4
log6 18 log6 3
Illustration 9: Find the value of 2 .3 (JEE MAIN)
(M.N)
Sol: We can solve above problem by using loga = logaM + logaN and a
loge c loge a
=c step by step.
log6 18 log 3 log (6×3) log 3 1 +log 3 log 3 log6 3 log6 3 loge c loge a
2 (3) 6 = 2=
6 .3 6 2=
6 .3 6 2·2 ·3 ( a =c )
log 2 log 3 log6 2 +log6 3 log (6)
2·(3)
= 6 ·(3) 6 2(3) = 2(3) =
6 2·(3)
= 6
(
Illustration 10: Find the value of, logsec α cos3 α where α ∈ (0, π/2) ) (JEE MAIN)
( )
Sol: Consider logsec α cos3 α =x . Therefore by using formula = y
y loga x ⇔ a= x we can write Cos3α = (sec α)x .
Hence by solving this we will get the value of x.
Let logsec α cos3 α =x
x
1
Cos α = (sec α) ⇒ (cos α) =
3 x
⇒ (cos α) = (cos α) ⇒ x = –3
3 3 –x
cos α
Illustration 11: If k ∈ N, such that log2x + log4x + log8x = logkx and ∀ x ∈ R’ (JEE ADVANCED)
If k = (a)1/b then find the value of a + b; a ∈ N, b ∈ N and b is a prime number.
logc a loga
Sol: By using log
= ba = we can obtain the value of k and then by comparing it to k = (a)1/b we can
logc b logb
obtain value of a + b.
11 1 1 11 log2 11
Also, − =0 ⇒ = ⇒ =logk 2
6 log2 logk 6 logk 6
11 1 1
So 2 = k 6 ; 26 11 =
k ⇒ 26 ( ) 11 k ⇒ ( 64 )11 =
= k
8 ( x + 3) 8 ( x + 3)
⇒ log4 2⇒
= 42 ⇒ x + 3 = 2x – 2 ⇒ x = 5
=
x −1 x −1
Also for x = 5 all terms of the equation are defined.
Sol: Here it’s given that log (–x) = 2 log (x + 1). Therefore by using the formula logaMx = x loga .M . We can evaluate
the value of x.
By definition, x < 0 and x + 1 > 0 ⇒ –1 < x < 0
Now log (–x) = 2 log (x + 1) ⇒ –x = (x + 1)2 ⇒ x2 + 3x + 1 = 0
−3 + 5 −3 − 5 −3 + 5
⇒x= , (rejected). Hence, x = is the only solution.
2 2 2
Illustration 14: Find the number of solutions to the equation log2 (x + 5) = 6–x. (JEE MAIN)
y y = 2c
Sol: By using the formula =y loga x ⇔ a= x , we can write given the equation as y=x+5
x + 5 = 2 . Hence, by checking the number of intersections made by the graph of
6–x 10
y = x + 5 and y = 26 –x we will obtain the number of solutions. 5
Here, x + 5 = 2 6–x
-5 5
Now graph of y = x + 5 and y = 26 –x intersects only once. Hence, there is only one
solution.
PLANCESS CONCEPTS
Always check your answer by putting it back in the equation; sometimes answer might not be in the
domain of logarithm.
Shrikant Nagori (JEE 2009, AIR 7)
M a them a ti cs | 1.11
(i) x (ii) x
(0, 0) (1, 0) (0, 0) (1, 0)
If the number and the base are on the same side of unity, then the logarithm is positive, and if the number and the
base are on different side of unity then the logarithm is negative.
Illustration 15: Which of the following numbers are positive/negative? (JEE MAIN)
(i) log27 (ii) log1/23 (iii) log1/3 (1/5) (iv) log4 3 (v) log2 9
Sol: By observing whether the Number and Base are on the same side of unity or not we can say whether the
numbers are positive or negative.
(i) Let log27 = x (number and base are on the same side of unity) ⇒ x > 0
(ii) Let log1/23 = x (number and base are on the same side of unity) ⇒ x < 0
(iii) Let log1/3(1/5) = x (number and base are on the same side of unity) ⇒ x > 0
(iv) Let log43 = x (number and base are on the same side of unity) ⇒ x > 0
(v) Let (log29) = x (number and base are on the same side of unity) ⇒ x > 0
(b) The mantissa part of the log of a number is always kept non-negative, it ranges from [0, 1]
(c) If the characteristic of log10N is C then the number of digits in N is (C + 1)
(d) If the characteristic of log10N is (–C) then there exist (C – 1) number of zeros after decimal point of N.
Illustration 16: Let x = (0.15)20. Find the characteristic and mantissa of the logarithm of x to the base 10. Assume
log10 2 = 0.301 and log10 3 = 0.477. (JEE ADVANCED)
Sol: Simply by applying log on both sides and using various logarithm formulas we can solve the above illustration.
15 10
logx = log(0.15)20 = 20 log = 20[log 15 – 2] = 20[log3 + log5 – 2] = 20[log3 + 1 – log 2 – 2] log10 5 = log10
100 2
= 20 [–1 + log3 – log2] = – 20 × 0.824 = – 16.48 = 17.52
Hence, characteristic = – 17 and mantissa = 0.52
Illustration 17: Find the number of digits in the following: (i) 2100 (ii) 310 (JEE ADVANCED)
Sol: By considering x = 2100 and 310 respectively and applying log on both sides we can solve the problems given above.
(i) Let, x = 2100
log10X = log102100 = 100 log10 2 = 100 × 0.3010 = 30.10
1 . 1 2 | Basic Mathematics
Illustration 18: Find the number of zeros after decimal before a significant figure in
(i) 3–50 (ii) 2–100 (iii) 7–100 (JEE ADVANCED)
Sol: Similar to the illustration above, we can solve these too.
(i) N = 3–50
log10 N = log10 3–50 = - 50 log103 = – 50 × (0.4771) ⇒ log10 N = – 23.855
Now to find the characteristic and mantissa many would say that (c = –23, m = –0.855) (which is wrong) because
mantissa is always non-negative.
log10N = – 23.855 = –23 –1 + 1 – 0.855 = – 24 + 0.145
C = –24, M = 0.145. Number of zeroes after decimal = |–24| –1 = 23 or |–24 + 1| = 23
(ii) N = 2–100
log10 N = – 100 log2 = –30.10 = –30 –0.10 = –31 + 0.90. Number of zeroes after decimal = |–31| –1 = 30 or |–31 + C| = 30
(iii) N = 7–100
log10N = –100 log 7 = –100 × 0.8451 = –84.51 = – 84–1 + 0.49 = – 85 + 0.49
C = –85, M = 0.49. Number of zeroes after decimal = |–85| –1 = 84 or |–85 + 1| = 84
Illustration 19: Find the number of positive integers which have the characteristic 2, when base of log is 6.
(JEE ADVANCED)
Sol: If any number x has the characteristic a, when base of log is b then x = b . By using the given condition we can
a
Natural numbers ranging from 36 to 215 will give characteristic 2, when taken log with base 6.
Number of positive integers = 215 – 35 = 180
x2 −2x
Illustration 20: Solve (1 / 2 ) <1/4 (JEE MAIN)
x2 −2x
Sol: Here we can write the given equation as (1 / 2 ) < (1 / 2 ) and then by comparing powers on both side we
2
( (
⇒ x− 1+ 3 ) ) ( x − (1 − 3 ) ) > 0 ⇒ x > 1 + 3 or x < 1 – 3 ⇒ x ∈ (–∞, 1 – 3 ) ∪ (1+ 3 , ∞)
1 + 5x
Illustration 21: Solve ≥ 0 . (JEE MAIN)
7− x + 97
( )
Sol: Simply by multiplying 7− x − 72 on both sides and solving we will get the result. - + -
x
g(x) =
1 −5
7− x − 7
( )( )
≤ 0 . Now 1 − 5x 7− x − 7 ≤ 0 ; 5x – 1 = 0 ⇒ x = 0; 7–x –7 = 0 ⇒ x = –1
-1 0
Figure 1.8
g(x) behavior on the number line. Hence, from above, x ∈ (–∞, -1) ∪ [0, ∞)
−a, if a < 0
x
x
Basic properties of modulus O
Figure 1.9
(A) |ab| = |a| |b|
a |a|
(B) = where b ≠ 0
b |b|
1 . 1 4 | Basic Mathematics
Sol: The above illustration can be solved by taking two cases; the first one is by taking x – 2 as greater than 0 and
second one is by taking x - 2.
Case-I: When x – 2 ≥ 0 ⇒ x ≥ 2 ... (i)
Since x – 2 is non negative, the modulus can simply be removed. x – 2 = 3; x = 5
We had taken x ≥ 2 and we got x = 5 hence this result satisfy the initial condition ⇒ x = 5
Case-II: When x – 2 < 0 ⇒ x < 2; Since x – 2 is negative, the modulus will open with a –ve sign.
-(x – 2) = 3; -x + 2 = 3 ⇒ x = – 1 Since x < 2 Hence x = –1, 5
x x 2 x3 xn
((i)i) e=x 1+ + +
1! 2! 3!
+ ....... + .......∞
n!
( −1) xn + .......∞
n
x x 2 x3
(ii) e = 1 –
–x
+ − + ....... (Replace x by –x)
1! 2! 3! n!
1 1 1
(iii) e = 1 + + + + .......∞ (Substituting x = 1 in (i))
1! 2! 3!
1 1 1
(iv) e–1 = 1 – + − + .......∞ (Substituting x = – 1 in (i))
1! 2! 3!
ex + e− x x2 x 4 x6
(v) =1 + + + + .......∞
2 2! 4! 6!
ex − e− x x3 x5
(vi) =x + + + .......∞
2 3! 5!
x2 x3
(vii) ax = 1+ x (ln a) + (ln a)2 + (ln a)3 + ………………; (a > 0) , where ln a = loge(a)
2! 3!
1 . 1 6 | Basic Mathematics
x 2 x3 x 4
(ii) ln (1 – x) = – x – − − + ......∞
2 3 4
1 + x x3 x5
(iii) ln (x + 1) – ln (1 – x) = ln = 2 x + 3 + 5 + .....
1 − x
x2 x 4 x6
(iv) ln (1 + x) + ln (1 – x) = ln (1 – x2) = – 2 + + + ......
2 4 6
6.11 Antilogarithm
The positive number n is called the antilogarithm of a number m if m = log n. If n is the antilogarithm of m, we
write n = antilog m. For example
(i) log (100) = 2 ⇒ antilog 2 = 100
(ii) log (431.5) = 2.6350 ⇒ antilog (2.6350) = 431.5
(iii) log (0.1257) = 1.0993 ⇒ antilog ( 1.0993 ) = 0.1257
Illustration 26: Find the antilogarithm of each of the following: (JEE MAIN)
(i) 2.7523 (ii) 0.7523 (iii) 2.7523 (iv) 3.7523
Sol: By using log table and following the above mentioned steps we can find the algorithms of above values.
(i) The mantissa of 2.7523 is positive and is equal to 0.7523.
Now, look into the row starting 0.75. In this row, look at the number in the column headed by 2. The number
is 5649. Now in the same row move in the column of mean differences and look at the number in the column
headed by 3. The number there is 4. Add this number to 5649 to get 5653. The characteristic is 2. So, the
decimal point is put after 3 digits to get 565.3
(ii) Proceeding as above, we have antilog (0.7523) = 5.653.
M a them a ti cs | 1.17
(iii) In this case, the characteristic is 2 , i.e., – 2. So, we write one zero on the digit side of the decimal point. Hence,
antilog ( 2 .7523) = 0.05653
(iv) Proceeding as above, antilog ( 3 .7523) = 0.005653
PROBLEM-SOLVING TACTICS
(a) The main thing to remember about surds and working them out is that it is about manipulation. Changing
and manipulating the equation so that you get the desired result. Rationalizing the denominator is all about
manipulating the algebra expression.
(b) Strategy for Solving Equations containing Logarithmic and Non-Logarithmic Expressions:
(i) Collect all logarithmic expressions on one side of the equation and all constants on the other side.
(ii) Use the Rules of Logarithms to rewrite the logarithmic expressions as the logarithm of a single quantity
with coefficient of 1.
(iii) Rewrite the logarithmic equation as an equivalent exponential equation.
(iv) Solve for the variable.
(v) Check each solution in the original equation, rejecting apparent solutions that produce any logarithm of
a negative number or the logarithm of 0. Usually, a visual check suffices!
x 2 x3 x 4
(i) ln (1 + x) = x – + − + ......∞
2 3 4
x 2 x3 x 4
(ii) ln (1 – x) = – x – − − + ......∞
2 3 4
1 + x x3 x5
(iii) ln (x + 1) – ln (1 – x) = ln = 2 x + + + ......
1 − x 3 5
x2 x 4 x6
(iv) ln (1 + x) + ln (1 – x) = ln (1 – x2) = – 2 + + + ......
2 4 6
FORMULAE SHEET
(a) Laws of indices
1
(i) a0 =1 , (a ≠ 0) (ii) a-m = , (a ≠ 0)
am
(iii) a
m+n
= am .an , where m and n are real numbers am
(iv) am–n =
an
1 . 1 8 | Basic Mathematics
( )
n q
(v) a
m
= amn (vi) a q
= ap
n
a an
( )
n
(vii) ab = anbn (viii) =
b bn
(iii) a2 - b2 = (a + b) (a - b)
1 1 1
(viii) (a + b + c)2 = a2 + b2 + c2 + 2ab + 2bc + 2ca = a2 + b2 + c2 + 2abc + +
a b c
1
(ix) a2 + b2 + c2 – ab – bc – ca = [(a – b)2 + (b – c)2 + (c – a)2]
2
1
n
(i) a = an (ii) n
ab
= n
a×nb
a na
( a)
n
n
(iii) n = (iv) =a
b nb
( a)
m
(v)
m n a = mn a (vi) n
= am
n
M
(i) loga =
M.N ( ) logaM + logaN (ii) loga = logaM – logaN
N
y
(
(iii) y= loga x ⇔ a = x a, x > 0,a ≠ 1 ) (iv) logaMx = x loga .M
M a them a ti cs | 1.19
1 logc a loga
(v) log x M
=
a x
loga M ( x ≠ 0) (vi) logb a= =
logc b logb
( c > 0, c ≠ 1)
1 (viii) logb a . logcb . logdc = logda
(vii) logab =
logb a
loge c loge a
(ix) a =c (x) logb a . logab = 1
( −1) xn + ......∞
n
x x 2 x3 xn x x 2 x3
(i) ex =1 +
(i) + + + ...... + + ......∞ (ii) e
(ii)
−x
=1 − + − + ...... +
1! 2! 3! n! 1! 2! 3! n!
1 1 1 1 1 1
(iii) e = 1 + + + + .......∞ (iv) e–1 = 1 – + − + .......∞
1! 2! 3! 1! 2! 3!
ex + e− x x2 x 4 x6 ex − e− x x3 x5
(v) =1 + + + + .......∞ (vi) =x + + + .......∞
2 2! 4! 6! 2 3! 5!
x2
(vii) ax =1+x (ln a)+ (ln a)2 +.. (a > 0)
2!
Solved Examples
Let x = 3
72.3 , Then, log x = log (72.3)1/3 = 0.8096 + 2.9919 (using log table)
(
log 72.3 log 0.723 × 102
= ) Example 3: Find minimum value of x satisfying
|x –3| + 2 |x + 1| = 4
2
=⇒ log 0.723 + log 10
= 1.8591 =
+ 2 1.8591 ... (ii)
1 Sol: Similar to illustration 25.
⇒ log10 x = × 1.8591
3 Case-I: When x < –1
⇒ log10 x = 0.6197 ; ⇒ x=antilog (0.6197)
–1 (x –3) –2 (x + 1) = 4
⇒ x = 4.166 (using antilog table)
⇒ – x + 3 – 2x – 2 = 4 ; ⇒ –3x + 1 = 4
⇒ 3x = –3 ; ⇒ x = – 1
1 . 2 0 | Basic Mathematics
=
3+β2
N 5= ,N 53=
)
,54 ,N 125,625 ) log=
10 x 8.1332 ⇒
= x (10 )108 0.1332
x(
log x −2 )
49 125 243 343 625 729 Example 8: Solve x =9
1
Sol: Here, by using log x M = loga M we can solve the
problem above.
a x
x(
log x −2 )
Common part is [243, 343]. So largest integral value = 342 x = 9 ⇒ x2logx (x-2) = 9
2
logx ( x −2 )
(b) Difference of largest and smallest values ⇒ x = 9 where x > 0, x ≠ 1
= 342 – 243 = 99
⇒ ( x − 2) =
2
9 ; ⇒ x − 2 =±3
⇒ x = – 1, x = 5
Example 5: Find the number of zeros in, x = (0.35)12,
Given log10(7) = 0.8451, log10(2) = 0.3010 But x = –1 is rejected as x should be greater than 0.
2
⇒ log1/2 x − 3log1/2 x + 5 =9 ; Hence substitute it in the above equation and solve
using the logarithm formula.
Let log1/2 ( x ) = t ⇒ t2 − 3t − 4 =
0
Given that
(t – 4) (t + 1) = 0 ⇒ t = 4, t = –1 ( )
log3x + 7 9 + 12x + 3x2 + log2x +3 6x2 + 23x + 21 =
4 ( )
⇒ log1/2 x =
4,log1/2 x =
−1
log3x + 7 ( 2x + 3) + log2x +3 ( 2x + 3)( 3x + 7 ) =
2
4 Let
x= 1/16, x = 2
1
log3x + 7 ( 2x + 3) =
A ; 2A + 1 + =4
( )
2
1 − 2 log10 x2 A
Example 10: Solve =1
⇒ 2A2 − 3A + 1 =0 ; 2A2 –2A –A +1 = 0
log10 x − 2 (log10 x )
2
( )
2
1 − 2 log10 x2 1
For A= ,2x + 3= 3x + 7
= 1 , Let log10 x = t 2
log10 x − 2 (log10 x )
2
4 4 2
JEE Advanced/Boards 3 2 5 1
Let log2 x = t ; t +t− =
4 4 2t
Example 1: Solve,
3t3 + 4t2 − 5t =2 ⇒ 3t3 + 4t2 − 5t − 2 =0
(
log3x + 7 9 + 12x + 4x 2
) + log (6x 2
+ 23x + 21 =
4 )
( )
2x +3
⇒ 3t3 + 3t2 − 6t + t2 + t − 2 =0 ⇒ ( 3t + 1 ) t2 + t − 2 =0
2
Sol: Here 6x + 23x + 21 1
⇒ ( 3t + 1 )( t + 2 )( t − 1 ) =
0 ; ⇒ t = 1, −2, −
(
( 2x + 3) (3x + 7 ) and 9 + 12x + 4x2 = ( 2x + 3) .
= ) 2
Putting t = log2 x
3
1 . 2 2 | Basic Mathematics
1 4 ± 16 − 20
log2 x = 1 ⇒ x = 2 ; log2 x =−2 ⇒ x = =⇒x ,x ∉R
4 2
1 / (2)
1/3
log2 x =−1 / 3 ⇒ x = which is not possible
log3 x2 −2logx 9 −6
Case-II: (4 – x) < 0 or x > 4 then (x + 1) = (x – 1) (x – 4)
Example 3: Solve x − 1 ( x − 1)
=
⇒ x + 1 = x2 − 5x + 4 ⇒ x2 − 6x + 3 =0
Sol: As ax is defined for a > 0 so (x –1) > 0. Therefore by
taking log on both side we can solve it. 6 ± 24
⇒ x2 − 6x + 3 =0 ⇒x=
Now taking log on both sides 2
6±2 6
(log x
3
2
)
− 2logx 9 log ( x − 1=
) log ( x − 1) ⇒x=
2
⇒ x =3± 6
( )
2
log3 x = 4 or log3 x = −1 / 2 3 log10 x − mlog10 x − 2 =
( ) 0
(=
3 ) or x ( 3 )
4 −1/2
=x
log10 x
Let e 3 = t then
x = 81, x = 1/ 3
⇒ t − t − 2 =0 ; ⇒ t2 − 2t + t − 2 =
2
0
1
For x = log (x – 1) is not defined, so x = 2 or x = 81.
3 ⇒ t ( t − 2) + 1 ( t − 2) =
0 ⇒ ( t + 1 )( t − 2 ) =
0
;
log10 x
Example 4: Solve, Case-I:
⇒ Case −1 ; ⇒ t =−1 & t =2 ⇒ 3
−I: t = −1
=
Exponential value cannot be negative
( )
log4 x2 − 1 − log4 ( x −
= 1) log4
2
(4 − x)
2
Case-II: t = 2; 3
log10 x
=2
M Taking log3 both side
Sol: By using formula loga = logaM – logaN and
N
log3 ( 3 )
log10 x log3 2
using modulus inequalities we can solve the problem = log3 2 ⇒ log10=
x log3 2 ⇒=
x 10
above.
Comparing by ( a)
logb c
we get
log4
(x 2
−1 ) =log 4 − x x2 =x a = 10 , b = 3, c = 2
4
( x − 1)
2
∴ a + b + c = 10 + 3 + 2 = 15
⇒ log4
( x − 1)( x + 1) =
log4 4 − x
( x − 1)
2
Example 6: Find the number of zeros after decimal
before a significant digit in ( 9 / 8 )
−100
.
So we have
( x + 1=) 4−x −100
( x − 1) 9
Sol: By putting x = and applying log10 on both
8
or (x + 1) = (x – 1) |4 – x| side we will get the result.
−100
Case-I: 4 – x > 0 or x < 4 then (x + 1) = (x – 1) (4 – x) 9
Let x =
⇒ x + 1 = 4x − x2 − 4 + x ⇒ x2 − 4x + 5 = 0 8
M a them a ti cs | 1.23
⇒ log10 x =
−100 log10 9 − log10 8 Example 9: Solve, log ( x / 4 ) = 15
2
x
log2 − 1
⇒ log10 x =
−100 2log30 3 − 3log10 2 8
⇒ log10 x =−100 ( 2 × 0.4771 − 3 × 0.3010 ) Sol: Simply by putting log2 (x) = t and using basic
logarithmic formula we can solve the problem above.
−100 0.9542 − 0.9030 =
= −100 0.0512 =
−5.12
15 15
log30 x = ( −5 − 0.12 ) + 1 − 1 log2 ( x / 4 ) = ⇒ (log2 x − 2 ) =
x
log2 − 1
( 2 − 3) − 1
log x
log10 x = 6.88 ⇒ x = 10-6 ×100.88 8
Let log2 (x) = t
∴ Number of zeros before any significant digits = 5
15
⇒ t−2 = ⇒ t2 - 6t +8 = 15
( (
Example 7: Solve log4 2log3 1 + log2 (1 + 3log2 x ) =
1/2 )) t−4
y
⇒ t2 − 6t − 7 =0 ⇒ (t –7) (t + 1) = 0
Sol: Here by using =
y loga x ⇔ a= x we can solve it.
⇒ t = 7, t = – 1 ⇒ log2x = 7 and log2x = -1
( (
log4 2log3 1 + log3 ( 3log3 x ) )) = 1/2
⇒ x = 27 and x = 2–1
⇒ 2log3 (1 + log2 (1 + 3log3 x ) ) = 2
(1 + 2log2 x ) 2 + 12 log2 x =
3
equation to log2 x .
Example 8: Solve log0.5x x − 7log16x x3 + 40log4x 4 x =
0 2
equation to
log2 x
− +
7log2 x3 40log2 4 x
0 and
=
( )
log2 2x2 log4 (16x ) = log4 x3
( t + 4 )( t + 2 ) − 21 ( t − 1 )( t + 2 ) + 10 ( t − 1 )( t + 4 ) 4 + t 9t2 9t2
⇒ (1 + 2t ) = ⇒ ( 2t + 1 )( t + 4 ) =
⇒ t = 0
( t − 1)( t + 4 )( t + 2) 2 4 2
t2 + 6t + 8 − 21t2 − 21 t + 42 + 10t2 + 30 t − 40 ( )
⇒ 2. 2t2 + 9t + 4 = 9t2
⇒ t = 0
( t − 1)( t + 4 )( t + 2) ⇒ 5t2 − 18t − 8 = 0 ⇒ 5t2 − 20t + 2t − 8 =0
−10 t2 + 15 t + 10 ⇒ 5t ( t − 4 ) + 2 ( t − 4 ) =
0 ; t =
−2 / 5,t =
4
⇒ t =0
( t − 1 )( t + 4 )( t + 2 )
But t ≠ 4 ⇒ x =6 and logx = -2/5 is Not Possible
1 2 2
⇒ t = 0, − , 2 ∴ log2 x = 0 ⇒ x = 1 ∴ t=− ⇒ log2 x =− ∴ x=2−2/5
2 5 5
1 1
log2 x =− ⇒ x = and log2 x = 2 ⇒ x = 4
2 2
1 . 2 4 | Basic Mathematics
JEE Main/Boards
Exercise 1 1 1 1
Q.5 1 − log5
= log + logx + log5
3 2 3
Q.1 Solve
1 1 1 1 1
(i) log16 32 Q.6 logx − log x − =
log x + − log x +
2 2 2 2 8
(ii) log8 16 log10 x + 7
log10 x +1
Q.7 x 4 = 10
(iii) log1/3 (1 / 9 )
log210 x +log10 x2 −2
(iv) log2 3
(1728 ) log10 x
Q.8 = log10 x
(v) log2 cos 45º 2
(ii) log0.125 ( 8 ) = −1 (
Q.12 log4 x2 − 1 − log4 ( x=
− 1)
2
)
log4
(4 − x)
2
( )
(iii) log1.5 0.6 = −1
Q.13 2log3
x −3
+1 =log3
x −3
x−7 x −1
(iv) log2.25 ( 0.4 ) = −1
2log10 x
(v) x = 10.x2 1 1
Q.20 log2 ( x −=
2) − log1/8 3x − 5
log10 x +5 6 3
5 + log10 x
(vi) x 3 = 10
log x Q.21
log10 ( x +1 +1 ) =3
(vii) x 3 =9
log10 ( 3
x − 40 )
M a them a ti cs | 1.25
1 1 log 3 log x
Q.22 1 − log10 ( 2x=
− 1) log ( x − 9 ) Q.4 If 5x 2 + 3 2 = 162 then logarithm of x to the
2 2 10 base 4 has the value equal to
(A) 2 (B) 1 (C) –1 (D) 3/2
( )
Q.23 log10 3x2 + 7 − log10 ( 3x − 2 ) =
1
log2 x +log10 x3 +3 2
Q.5 (x) 10 =
1
Q.24 1 + log10 3 + log10 2= log10 27 − 31/x
2x
( ) 1
–
1
x +1 –1 x +1 +1
1 where x1> x2> x3, then
Q.25 2 + x log10 x ( x + 2 ) + 1
log x + 3log10=
2 10 2
(A) x1 + x3 =
2x2 (B) x1 .x3 = x2
( )
Q.26 log2 4 x + 1 =x + log2 2x +3 − 6 ( ) (C) x2 =
2x1 x2
(D) x1−1 + x1−1 =
x3−1
x1 + x2
Q.27 log 5 (4 x
)
− 6 − log 5 (2 x
−2 =
2 ) Q.6 Let x = 2
log3
and y = 3
log2
where base of the
xlog10 4 logarithm is 10, then which one of the following holds
(
Q.28 log10 3x − 24 − x =
1
2 + log10 16 −
4
) 2
good?
(A) 2x < y (B) 2y < x (C) 3x = 2y (D) y = x
Q.29 log10 (log10 x ) + log10 log10 x − 3 =
0 ( 4
)
Q.7 Number of real solution(s) of the equation
x
Q.30 log3 9= ( )
+ 9 log3 3x 28 − 2.3x ( ) x −3
3x2 −10x +3
1 is-
=
Single Correct Choice Type Q.8 If x1 and x2 are the roots of the equation
log2010 x
2010x = x2 , then find the cyphers at the end
1 1 1 of the product (x1x2)
Q.1 + +
log bc
abc log ac
abc log ab
abc
(A) 1 (B) 3 (C) 2 (D) 4
has the value equal to
Q.9 Let x = 2 or x = 3 satisfy the equation, log4 (x2 + bx
(A) 1/2 (B) 1 (C) 2 (D) 4 + c) = 1. Then find the value of |bc|.
(A) 50 (B) 60 (C) 40 (D) 55
Q.2 The equation, log2 2x2 + log2 x.x ( ) logx (log2 x +1 )
1 −3log1/2 (log2 x )
+ log4 2x 4 + 2 1 has
=
2
(A) Exactly one real solution (B) Two real solutions
(C) 3 Real solutions (D) No solution
JEE Advanced/Boards
Exercise 1 log10 ( x − 3) 1
=
Q.1 Let A denotes the value of
Q.11 (a) Solve for x,
( 2
log10 x − 21 ) 2
4 3
(a) log1/3 729 9−1.27−4/3 (b)
(
log b log b N ) loga z loga (xyz) = 84
log b a
a
Q.13 Let ‘L’ denotes the antilog of 0.4 to the base 1024.
and ‘M’ denotes the nuber of digits in 610 (Given log102 =
Q.3 (a) Which is smaller? 2 or (logπ 2 + log2 π )
0.3010, log103 = 0.4771) and ‘N’ denotes the number of
(b) Prove that log3 5 and log2 7 are both irrational. positive integers which have the characteristic 2, when
base of the logarithm is 6. Find the value of LMN.
Q.4 Find the square of the sum of the roots of the
equation log3x · log4x · log5x = log3x · log4x + log4x · log5x + Q.14 Prove the identity.
log5x · log3x.
logaN . logbN+ logbN . logcN + logcN . loga
2
of N.
log1/5 (1/2 ) 4 1
Q.7 Simplify: 5 + log + log1/2 Q.16 If log102 = 0.0310, log103 = 0.4771. Find the
2
7+ 3 10 + 2 21 number of integers in:
(a) 5200
2
Q.8 Given that log2 a = s, log4 b = s and log 2 8 = 2
2 5 s3 + 1 c (b) 615
a b
Write log2 as function of ‘s’ (a, b, c > 0) (c ≠ 1). (c) The number of zeros after the decimal in 3–100.
C4
log2 24 log2 192
Q.9 Prove that − =3 Q.17 log5120 + (x – 3) –2 log5 (1 – 5x –3) = – log5 (2 – 5x–4)
log96 2 log12 2
Q.20 If ‘x’ and ‘y’ are real numbers such that, 1/3 1/3
log10 ( 2x − 3x=
x log x + log 3 log x3 =
) log10 x + log10 y, find y +
3 3 x
x 3 2
Q.21 If a = log12 18 and b = log24 54 then find the value
of ab + 5 (a – b) Q.31 Let a = (log7 81 )(log6561 625 )(log125 216 )(log1296 2401 )
3
(log9 x )2 − 9 log x + 5 =3 3 Q.1 Number of ordered pair(s) satisfying simultaneously,
9
2 x+ y
the system of equations, 2 = 256 and
Q.25 Let a, b, c, d are positive integers such that loga
log10 xy – log10 1.5 = 1, is:
b = 3/2 and logc d = 5/4. If (a – c) = 9, find the value of
(b – d). (A) Zero (B) Exactly one
(C) Exactly two (D) More than two
Q.26 Find the product of the positive roots of the
( 2008 ) ( x )
log2008 x
equation = x2 Q.2 Let ABC be a triangle right angled at C. The value of
logb + c a + logc −b a
(b + c ≠, c – b ≠ 1) equals
Q.27 Find x satisfying the equation logb + c a.logc −b a
( )
( )
2
3log2 −2log log103 + log log106
1 + 2log3 2
Q.6 Let N = 10 where
Q.14 The number N = + log26 2
(1 + log3 2)
2
base of the logarithm is 10. The characteristics of the
logarithm of N to the base 3, is equal to
when simplified reduces to-
(A) 2 (B) 3 (C) 4 (D) 5
(A) A prime number
(B) An irrational number
10 + 2 10 − 2
Q.7 If x
= = and y , then the value (C) A real number is less than log3π
2 2
(
of log2 x2 + xy + y 2 , is equal to ) (D) A real which is greater than log76
(A) 0 (B) 2 (C) 3 (D) 4 Q.15 The value of x satisfying the equation,
22x ‒ 8.2x = ‒ 12 is
5 3 5 3 log3 1 3
Q.8 The sum + + − is equal to (A) 1 + (B) log6 (C) 1 + log (D) 1
4 2 4 2 log2 2 2
π π π π
(A) tan (B) cot (C) sec (D) sin x x
3 3 3 3
Q.16 If 5 2 − 7 + 6 5 2 + 7 =
7,
Q.9 Suppose that x < 0. Which of the following is equal then the value of x can be equal to-
( x − 2)
2
to 2x − (A) 0 (B) log 36
(5 2 −7 )
(A) x – 2 (B) 3x – 2 (C) 3x + 2 (D) – 3x + 2 −2
(C) (D) log 6
(
log6 5 2 + 7 ) 5 2 −7
Statement-I: If the number N > 0 and the base of the Comprehension Type
logarithm b (greater than zero not equal to 1) both lie
on the same side of unity then logbN > 0 and if they lie Paragraph 1: Equations of the form (i) f (logax) = 0,
on different side of unit then logb N < 0. a > 0, a ≠ 1 and (ii) g (logxA) = 0, A > 0, then Eq. (i) is
equivalent to f(t) = 0, where t = logax. If t1, t2, t3, ……… tk
(
1 log2 ( x − 1 )
Q.18 Statement-I: log2 2 17 − 2x =+ ) are the roots of f(t) = 0, then logax = t, loga x = t2, …… ,
x = tk and eq. (ii) is equivalent to f(y) = 0, where y
has a solution. logx A. If yf , y2, y3, …., yk are the root of f(y) = 0, then
because logx A = y1, logx, A = y2, ……. , logx A = yk.
Statement-II: Change of base in logarithms is possible. On the basis of above information, answer the following
questions.
Q.19 Consider the following statements
Q.20 The number of solution of the equation
log 5 x3 +1
2
Statement-I: The equation 5 1 has two
−x = log3x 10 − 6log2x 10 + 11logx 10 − 6 =0 is:
distinct real solutions.
(A) 0 (B) 1 (C) 2 (D) 3
Because.
loga N
Statement-II: a = N when a > 0, a ≠ 1 and N > 0.
Q.21
Column-I Column-II
(A) The value of x for which the radical product (p) 4
P (x) (q) 7
(B) Let P(x) = x7 – 3x5 + x3 –7x2 + 5 and Q (x) = x – 2. The remainder of is not smaller than
Q(x)
(C) Given a right triangle with side of length a, b and c and area equal to a2 + b2 – c2. The ratio of the (r) 10
larger to the smaller leg of the triangle is
equal to
Q.22
Column I Column II
1 1 1
(C) The expression + − simplifies to (r) A natural
log5 3 log6 3 log10 3
PlancEssential Questions
JEE Main/Boards JEE Advanced/Boards
Exercise 1 Exercise 1
Q.2 Q.3 Q.15 Q.6 Q.12 Q.14
Q.25 Q.26 Q.16 Q.23
Exercise 2 Exercise 2
Q.3 Q.5 Q.9 Q.4 Q.6 Q.11
Q.10 Q.15 Q.17
Answer Key
JEE Main/Boards
Exercise 1
5 4 1 1
Q.1 (i) (ii) (iii) 2 (iv) 6 (v) – (vi) 1 (vii) –
4 3 2 2
24
Q.5 Q.6 1 Q.7 10–4, 10 Q.8 10-3, 10, 102
51/3
loga b,3loga b
Q.9 2, 16 Q.10 1/3, (1/3)4 Q.11 2 Q.12 3+ 6
17 17
–1 + –1–
5 5
Q.13 – 5 Q.14. 3, 1/9 Q.15 2 , 2 Q.16 1/16, 2
41
Q.17 27, 2–1 Q.18 8, – Q.19 4, 1 Q.20 3
5
1 1
Q.21 48 Q.22 13 Q.23 1, 9 Q.24 ,
4 2
Q.25 98 Q.26 0 Q.27 2 Q.28 3
Exercise 2
Single Correct Choice Type
JEE Advanced/Boards
Exercise 1
Q.15 507 Q.16 (a) 140 (b) 12 (c) 47 Q.17 -0.410 Q.18 1
5+3 5
Q.19 1 Q.20 4/9 Q.21 1 Q.22
10
Q.23 5625 Q.24 2196 Q.25 93 Q.26 (2008)2
Exercise 2
Single Correct Choice Type
Q.13 A
Comprehenstion Type
Q.20 D
Q.21 A → q, r, s; B → p, q, r, s; C → p; D → r Q.22 A → p; B → p, r, s; C → p, r; D → p, q, r
1 . 3 2 | Basic Mathematics
Solutions
let 5 5 5..........∞ =x
⇒ 5 x = x ⇒ 5 = x1/2 ⇒ x = 25
JEE Main/Boards log=
5 x log5=
25 log
= 55 1
3
Exercise 1 log20.5 ⇒ log0.5
2
(ii)log=
0.125 8
3
2
⇒ log1/2 ⇒ log2 −1 ⇒ − 1.log22 = −1
( 2)
Sol 1: (i) log1632 = log 25
24
we know log n ym =
m
logx y (iii) log ( 0.6 )
x n
2
5 5 = 0.6 0.6666.............
=
⇒ log 4 25 = log2 2 = 3
2 4 4
( 2) =
log ( 3 )
−1
4 4 4 ⇒ log3/2 3 −1
=
(ii) log816 = log 3 24 = log2 2 = (1) = 2 3/2
2 3 3 3
(iii) log1/3(1/9) = log1/3(1/3)2 = 2 log1/3(1/3) = 2.(1) = 2 ( )
(iv) log2.25 0.4
0.9 =
0.99999 =
1 ⇒ log10 1 =
0
22
⇒ log2 log2
Sol 3: We have to find out no. of digits in
⇒ log2 ( 2 log22 ) (i) 2100 = x (Assume)
⇒ log22 logaa = 1
⇒ log10x = log10 2100 = 100 log10 2 = 100(0.3010) = 30.10
⇒1
= 1030 (10 )
0.103
⇒ x 1030.103
=
(vii) log3 (tan 30°)
Total no. of digit = 30 + 1 = 31
1
tan 30° = (ii) x = 310
3
−1 ⇒ log10x = log10310 = 10 log103 = 10(0.47712) = 4.7712
1
⇒ log3 3
2
⇒ log3
3 ⇒ x = 104.7712 = 104 × 100.7712
−1 Total no. of digits = 4 + 1 = 5
⇒ log33
2
−1 Sol 4: (i) logx–1 3 = 2 ( x ≠ 1, 2)
⇒
2
1
logx–13 = 1 ⇒ logx–131/2 = 1
2
Let
let 5 5 5..........∞ =x (ii) log3(3x – 8) = 2 – x
1/2
⇒ 5 x = x ⇒ 5 = x1/2 ⇒ x = 25 ⇒ (3x – 8) = (3)2–x= 32 . 3–x = 9.3–x ⇒ 3x – 9.3–x = 8
log=
55
x log55=
25 log
= 55
5 1
M a them a ti cs | 1.33
1 1 1
Sol 5: 1 − log10 =
5 log10 + log10 x + log10 5
It can’t be –ve so x ≠ – 4 ⇒ x = 0 3 2 3
(v) x2 logx = 10 x2 1 1
3 (1 − log10 5 ) = log10 + log10 x + log10 5
2 3
Take logarithms is both sides
1
3 − log10 5 = log10 + log10 51/3 + log10 x
log10(x ) = log10 10x
2 logx 2 2
1
2 log10x (log10x) = log1010 + log10x2 ⇒ 3 log10 53 + log10
= + log10 51/3 + log10 x
2
2log10x(log10 x) = 1 + 2log10 x 1
⇒ 3 log10 53 × × 51/3 + log10 x
=
2
Assume log10 x = y ... (i)
3+ 1 1
⇒ 2y(y) = 1 + 2y ⇒ 2y2 = 1 + 2y ⇒ 2y2 – 2y – 1 = 0 ⇒ log10 3 − log10 5 3 ×
x=
2
2 ± 22 – 4(2)(–1)
⇒y=
2(2) log10 x =log10 103 − log10 510/3 × 2−1 ( )
2± 4+8 2±2 3 1± 3
y= = =
4 4 2
1 . 3 4 | Basic Mathematics
103 53 × 23 ⇒ x2 = 4x2 − 2x − 1 ⇒ 3 x2 − 2x − 1 =0
log
= 10 log10
510/32–1 510/3 × 2–1
2 ± 4 + 12 2± 4
⇒x= ⇒x=
9–10 6 6
= log10[ 5 3 23+1 ] = log10 [5–1/3 24] 1 1
x= 1, − x= 1, −
24 24 3 at 3
log
= 10 x log10 = ⇒x
51/3 51/3 2log ( −2 ) =
log ( 4 )
(
⇒ log10 x1/2 ) = log10 x1/2
1
x+1 x2 x +
x2 8 8 ⇒ log102x + log10 x2 – 2 = 1 or log10 x1/2 = 1
⇒ = 1 ⇒ =1
x–1
2 2 1 1
1 x – x + ⇒ log102x + 2log10x – 2 = 1; log10 x = 2 ⇒ x = 102
2 x + 2 4 2
⇒ log2x + 2log x - 2=0
1 1 1 Assume that log x = y
⇒ x2 x + 8 = x2 – x +
4 2 ⇒ y2 + 2y – 2 = 1 ⇒ y2 + 2y – 2 – 1 = 0
x x2 x 1 1 ⇒ (y + 3)(y – 1) = 0
⇒ x3 + = x3 + – –
8 2 4 4 2 y = – 3 or y = 1
2 2
x x x 1 log10 x = – 3 or log10 x = 1
⇒ x3 + = x3 + − −
8 2 4 8
⇒ x = 10–3 or 101 ⇒ x = 10–3, 10, 102
M a them a ti cs | 1.35
⇒ 3x2 + 9 – 3x – 9x = x2 – 14x + 49
2
Sol 12: log4(x – 1) – log4(x – 1) = log4( (4 – x) )
2 2
⇒ 2x2 + 2x – 40 = 0 ⇒ x2 + x – 20 = 0
2 ⇒ (x + 5)(x – 4) = 0
log4 x – 1 = log4( (4 – x)2 )
(x – 1)2 x = – 5 or x = 4
4–3
x2 – 1 At x = 4, equation is 2log3 4 – 3 + 1 = log
= (4 – x) 2 4 –7
4–7
(x – 1)2
4–3 +1
= ⇒ –ve which is not possible
(x – 1)(x + 1) 4–7 –3
⇒ = (4 – x)2 ; x ≠ 1,
(x – 1)2 Hence x ≠ 4, x = – 5
1 . 3 6 | Basic Mathematics
⇒ log2 x = y ⇒ x = 2y = 2° = 1 1
Sol 18: log10(5x – 4) + log10 x + 1 = 2 + log100.18
2
or (y + 4)(y + 2) + 14(y – 1)(y + 2) + 10(y – 1)(y + 4) = 0
⇒ log10(5x – 4) + 2log10 x + 1 = 2[2 + log100.18]
⇒ y2 + 8 + 6y + 14y2 – 28 + 14y + 10y2 – 40 + 30y = 0
⇒ 25y2 + 50y – 60 = 0 ⇒ log10(5x – 4) + log10(x + 1) = 4 + 2log100.18
60 12
⇒ y2 + 2y – = 0 ⇒ y2 + 2y – =0 ⇒ log10[(5x – 4)(x + 1)] = 4 + log10(0.18)2
25 5
M a them a ti cs | 1.37
−41 x=
83 ± = = 48, 35
=
∴x = is rejected 2 (1 ) 2 2
5
Now, for x = 35
Sol 19: log10x2 = log10(5x – 4) log10 35 + 1 + 1
The given equations yields = 3 –5
⇒ x2 = 5x – 4 ⇒ x2 – 5x + 4 = 0 log10 3 35 – 40
⇒ (x – 4)(x – 1) = 0 ⇒ x – 4 = 0or x – 1 = 0 Wich is not a possible solutions
Hence, x = 4, 1 Hence x ≠ 35 and x = 48
1 1 1 1
Sol 20: log2(x – 2)– = log1/8 3x – 5 Sol 22: 1 – log10(2x – 1) = log10(x – 9)
6 3 2 2
⇒ 2 – log10(2x – 1) = log10(x – 9)
1 1
⇒ log2(x – 2)– = log –3 3x – 5
6 3 2 ⇒ log10(x – 9) + log10(2x – 1) = 2
1 1
Sol 24: 1 + log103 + log102 = log10(27 – 31/x) 2X = 1 or 2x = – (not valid)
2x 7
⇒x=
0 and so, x = 0 is only solution.
⇒ log10 3(1+1/2x ) + log102 = log10(27 – 31/x)
⇒ 2 × 31+1/2x = 27 – 31/x 4x – 6 4x – 6
log x =2⇒ x =5
Assume that 3 1/x
=y 5 2 –2 2 –2
⇒ 2×3× y = 27 – y Assume that 2x = y
1 xlog10 4
1 Sol 28: log10(3x – 24–x) = 2 + log1016 –
Sol 25: log10 x + 3 log10=
2 + x log10 x ( x + 2) + 1 4 2
2
1 xlog22
log10 x + 6 log10
= 2 + x 2log10 x ( x + 2) + 2 ⇒ log10(3x–24–x)=log10102+ log1024 –
4 2
⇒ log10 x + log10(2 + x)3 – log10[x(x + 2)] = 2 4 x × 2 log10 2
⇒ log10(3X–24–X)=log10100+ log102–
x(2 + x)3 4 2
⇒ log10 = log102
x(x + 2) ⇒ log10(3x – 24–x) = log10[100 × 2] – log2x
1 1
log10 x = 1 or log10x = – ⇒ 1 + 2log2x + (log2x + 1)(log2x) +
1 4 4
–
⇒ x = 10 or x = 10 4 log2 (log2 x)3
+ log2x + (2) =1
1
−
for x ± 10 4 given log function is not defined. 1
⇒ 1 + 2log2x + (log2x)(log2x + 1) +
4
Hence, x = 10 + log2x + (log2x)3 = 1
1 4 162
3log (log x) 3y = = 27 = 33
+ log44 + log4x + 2 2 2 = 1 6
4 2
1 . 4 0 | Basic Mathematics
Or ⇒ Assume log10x = y
Sol 9: (A) Given that x = 2 or x = 3 satisfy the equation
⇒ y + 3y + 3 = 1 ⇒ y + 3y + 2 = 0
2 2
log4(x2 + bx + c) = 1 = log44
⇒ (y + 2)(y + 1) = 0 ⇒ x2 + bx + c – 4 = 0
y = – 2 or y = – 1 ⇒ b = 2 + 3 = 5 and c – 4 = 2 . 3 ⇒ c = 10
= log1/3
4 3
729 3–2.3–4 ⇒ x = 1 or x = 60
Sum of roots = 1 + 60 = 61
4 4
= log1/3 729.3–2 = log1/3 81 = log1/33 = 1 Square of sum of roots = (61)2=3721
logb (logb N)
logb a 2 3
(b) a = ax say Sol 5: +
6
log4 (2000) log5 (2000)6
logb (logb N)
x= = loga(logb N) 2 3
logb a + 6log (2000)
6log4 (2000) 5
loga (logb N)
So ax = a = logbN
1 2 3
= +
6 log (4 × 5 ) log (5 × 4 )
2 3 3 2
Sol 3: (a) logp2 + log2p 4 5
1 2 3
log2 log π log2 = +
⇒ + Assume that = x (+ve always) 6 log 42 + log 53 log 53 + log 42
logπ log2 logπ 4 4 5 5
1 1 2 3
(2 < π < 10) ⇒ x + = c (Assume) = +
x 6 2 + 3log4 5 3 + 2log5 4
c ± c2 – 4
x2 – cx + 1 = 0 ⇒ x =
2
1 2 3
For x to be real c2 – 4 ≥ 0 = +
6 3log10 5 2log10 4
2 + 3+
c2 ≥ 4 ⇒ c ≥ 2 ⇒ c = 2 ⇒ x = 1 log10 4 log10 5
For all other value c > 2 (Not Possible)
1 2log10 4 3log10 5
Here, logp2 + log2π is greater than 2 = +
6 2log10 4 + 3log10 5 3log10 5 + 2log10 4
(b) For log35 and log27
Assume that log35 is rational ∴ log35= = a ⇒ 5 = 3a 1 2log10 4 + 3log10 5 1
=
6 2log10 4 + 3log10 5 6
This is not possible when a is rational ∴ a is irrational
Similarly, log27 = b assuming b is rational gives 7 = 2b 3
1
Which is not possible, so b is irrational. log5 9 log
6
3 2
81 +3 ( 7 )log25 7 – (125)log25 6
Sol.6
409
Sol 4: log3x . log4x . log5x = log3x.log4x + log4x. log5x +
log5x log3x 9
2log9 5 3log3 6
+3 3
log 6
= ( 7 )2log7 25 – (25) 2 25
Assume that log10x = y 409
log10 x.log10 x.log10 x log9 52 log3 ( 6 )3
⇒ 9 +3 log7 25 log25 63/2
log10 3 log10 4 log10 5 = [7 – 25 ]
409
log10 xlog10 x log10 xlog10 x log10 x.log10 x
= + + ⇒ y3 52 + ( 6 )3 (52 )2 – (63/2 )2
log10 3log10 4 log10 4.log10 5 log10 5.log10 3 = [25 – 63/2] =
409 409
1 . 4 2 | Basic Mathematics
1
4 and y = logb a ⇒ x2 = logab
log1/5
2 + log
1
Sol 7: (5) 2
+ log1/2 1
7+ 3 10 + 2 21 y2 = logba ⇒ y2 = ⇒ x2y2 = 1
2
x
2 2
log 2 4 1 xy = 1 (x, y > 0) now ax – by = (b y )x – (ax )y
=5 5 + log 1 + log2–1
22
7+ 3 10 + 2 21 ⇒ (bxy)y – (axy)x ⇒ by – ax ⇒ ax – by = by – ax = – (ax – by)
4
2 ⇒ ax – by + ax – by = 0 ⇒ 2(ax – by) = 0 ⇒ ax – by = 0
= 2 + log2 + log2 10 + 2 21
7+ 3
log10 (x – 3) 1
2 Sol 11: (a) =
4 16 16 2
log10 (x – 21) 2
= = =
7+ 3 7+3+2 7 3 10 + 2 21
⇒ 2log10(x – 3) = log10(x2 – 21)
16 (x – 3)2
= 2 + log2 (10 + 2 21 ) ⇒ log10(x – 3)2 log10(x2 – 21) = 0 ⇒ log10 =0
10 + 2 21 (x2 – 21)
= 2+ log2 24 = 2 + 4 = 6 (x – 3)2
⇒ = 1 ⇒ x2 + 32 – 2(3)x = x2 – 21
x2 – 21
30
Sol 8: log2 a = s ⇒ a = 2s ⇒ 9 – 6x = – 21 ⇒ 6x = 9 +21 ⇒ x = =5
6
2 2
log4 b = s2 ⇒ b = 4s = (2)2s (b) log(log x) + log(log x3– 2) = 0
( 3
)
⇒ log2 2 × 12 − log2 12 ⇒ 3 + log2 12 − log2 12 = 13
(d) 5
loga x
+ 5x
loga 5
= 3, (a > 0)
M a them a ti cs | 1.43
1 10
⇒a+ = ⇒ 3a2 – 10a+ 3 = 0
Sol 13: Given a 3
1
L = antilog of 0.4 to the base 1024 ⇒ (3a 1)(a 3) = 0 ⇒ a = 3,
3
⇒ L = (1024)0.4 = (210)0.4 = 24 = 16 log12 x
So = 3 ⇒ add + 1 both side
L = 16 log12 y
And M is the number of digits in 610
log12 x logx + log y
⇒ log10 610 = 10log10 6 ⇒ 10[0.7761] = 7.761 +1=3+1=4 ⇒ =4
log12 y log12 y
⇒ 6 = 1010 7.761
= 10 .10 7 0.761
1 . 4 4 | Basic Mathematics
= N 5×5×5 25 × 25 53
2
(x + y)2 Multiply by 56
⇒ = N ⇒ x2 + y2 + 2xy = 4N
22 ⇒ 53 ×120y[2 – y 5–4] = 56 + y2 – 2x53y
⇒ (123/2)2 + (121/2) + 2(144) = 4 N 2
⇒ 53 × 240y – 120y = 56 + y2 – 2 × 53y
⇒ 12 + 12 + 2 × 144 = 4 N
3 5
x–3 ⇒ x2 + x – 6 = – (x + 1)2 ⇒ x2 + x – 6 = – x2 – 1 – 2x
⇒ log5 120 × 5 × (2 – 5x– 4 ) =0
(1 – 5x–3 )2 ⇒ 2x2 + 3x – 5 = 0 ⇒ (2x + 5)(x – 1) = 0
M a them a ti cs | 1.45
⇒ log1010x+log10(1+2x)=log105x+log106
Sol 22: log9 (9x 4 )log3 (3x) = log3 x3
⇒ log10[10 (1 + 2 )] = log10[5 6]
x x x
⇒ (y – 2)(y + 3) = 0 ⇒ y = – 3 or y = 2 5 ± 45 5 ± 32 × 5 5±3 5
y= = =
10 10 10
⇒ 2x = –3 or 2x = 2 ⇒ 2x = - 3 is not possible so, 2x = 2
In equation (i) log3x > 0
Therefore, the real solution ⇒ x = 1
5+3 5
Hence, y =
Sol 20: 2log10(2y – 3x) = log10x + log10y 10
x
We have to find Sol 23: Given that xyz = 1081
y
⇒ log10(2y – 3x)2 = log10(xy) ⇒ 4y2 – 12xy + 9x2 = xy (log10x)(log10yz) + (log10y)(log10z)= 468
3 3
Sol 29: 3
⇒ (log3 x + 1) + (logx 3 + 1) log3 x
4 4 4 4
loga ab +logb ab – loga b/a +logb a/b . loga b 3
2 = 2x
A= (2log3 x + (log3 x)2 + 1)
1
(loga (a × b) + logb (a × b))
4
⇒x= log b A = |log3x + 1|
a
1
– (loga ba–1 + logb ab –1 )
4 x 1 1 3
And B = log3 + logx log3 x3
33 3 x
1 1 + loga b + 1 + logb a
x= loga b
2 3
– –1 + log b – 1 +log a
b ⇒ log x – 1 + logx 3 – 1 log3 x
3 3
a
1 |x – 4| = x – 4
So x = ,3 – {1}
3
Eq. → x + 1 + x – 4 = 2x – 3 = 7
x ≠ 1 because base can’t be 1
7+3 10
⇒ 2x = = 10?2x = 10 ⇒ x = =5
1 2
Sol 31: a = (log781)(log6561625)(log125216)(log12962401) If – 1 < x < 4
4 3 4 |x – 4| → 4 – x
⇒ a = 4(log73) (log35)(log56) log67
8 3 4 ⇒ 1 + x + 4 – x= 5 ≠ 7
2log10 3 log10 5 log10 6 log10 7 So no solution for this region → x = 5 and – 2
⇒ a = 2
log10 7 log10 3 log10 5 log10 6 But – 2 is not natural no.
⇒ and b = sum of roots of the equation So c = 5
x
log2 x
= (2x)
log2 x a+b=2+3=5
5
log2 x log2 x1/2 (a + b) ÷ c = =1
x = (2x) 5
b=3
log10 xy = log10(10 × 1.5) = log1015
and c = sum of all natural solution of equation
⇒ xy = 15 ⇒ xy = 152 = 225
|x + 1| + |x – 4| = 7
I II III | x– y|= ( x + y )2 – 4 xy
-1 4
= 82 – 4 × 15 = 64 – 60
If x < – 1 → |x + 1| = – 1 – x
| x– y|= 4 =2
|x – 4| = 4 – x
x+ y =8
⇒ eq.→ – 1 –x + 4 –x = 3 – 2x = 7
4 ⇒ If x > y ⇒ (x, y) = (25, 9)
⇒ 2x = 3– 7 = – 4 ⇒ x = – =–2
2 ⇒ If x < y ⇒ (x, y) = (9, 25)
If x > 4 → |x + 1| = x + 1
M a them a ti cs | 1.49
⇒ a = 2, b = 16, c = 1
C B
a
a + b + c = 2 + 16 + 1 = 19
⇒c =a +b
2 2 2
⇒c –b =a
2 2 2
1
3x Multiple Correct Choice Type
3 = 27 ≤ =3–3x
3
3
1 + 2log3 2
3 ≤ – 3x ⇒ –x ≥ 1 ⇒ x ≤ –1 Sol 14: (C, D) N = + log26 2
(1 + log3 2)2
x ∈ [–∞, –1]
2
1 + 2log3 2 log3 2
N= +
2x +1
–3
(1 + log3 2)2 log 6
1 1–x 1 3
Sol 11: (B) >
5 5
Assume that log32 = y
2x + 1
<–3
1–x 1 + 2y y2
⇒N= +
2x + 1 < – 3(1 –x) = – 3 + 3 x (if (1 –x) > 0) (1 + y)2 (log3 2 + log3 3)2
⇒ 2x + 1 < – 3 + 3x ⇒ 3x – 2x > 1 + 3 = 4 1 + 2y y2 y 2 + 2y + 1
⇒N= + =
⇒ x > 4 ⇒ x > 4 and x < 1 which implies no solution (1 + y)2 (1 + y)2 (1 + y)2
2x + 1 (1 + y)2
If x > 1 ⇒ 1 – x < 0 ⇒ <–3 ⇒N= =1
1–x
(1 + y)2
2x + 1
⇒ > – 3(1 – x) = 3x – 3
1 And π = 3.147 > 3 and 7 > 6
⇒ 3x – 2x < 1 + 3 = 4 ⇒ x < 4 and x > 1 ⇒ x ∈ (1, 4) So, log3 π > 1 and log7 6 < 1
Assume ln x = y 5 2 +7
5 2 –7 = 5 2 –7×
⇒ y – 3y + 3 < y – 1 ⇒ y – 3y – y + 3 + 1 < 0
2 2
5 2 +7
⇒ y2 – 4y + 4 < 0 ⇒ (y – 2)2 < 0 always false 50 – 49 –1
= = 5 2 + 7
So if ln x < 1 ⇒ x < e and x > 0 5 2 +7
So, x ∈ (0, e) ⇒ y2 + 6 = 7y ⇒ y2 – 7y + 6 = 0
M a them a ti cs | 1.51
⇒ 17 – 2x = (x – 1)2 = x2 – 2x + 1
⇒ x = log 6 = 2log(5 6 = log(5 36
(5 2 –7)1/2 2 –7) 2 –7)
⇒ 17 = x2 + 1 ⇒ x2 = 16⇒ x = ± 4
2 –2 ⇒ x ≠ – 4 does not satisfy equation in statement-I
x= =
log6 (5 2 – 7) log6 (5 2 + 7) So x = 4. x has a solution
Statement-II
f(y) Area = a2 + b2 – c2
Also, we have b2 = a2 + c2
y
1 ac
So area = a2 + (a2 + c2) – c2 = ×a×c=
2 2
ac ac a
⇒ 2a2 = ⇒4= =
2 a2 c
c
⇒ ratio = =4
Match the Columns a
(D) a, b, c ∈ N
Sol 21: A → q, r, s; B → p, q, r, s; C → p; D → r
∴ ((4)1/3 + (2)1/3 – 2)(a(4)1/3+b(2)1/3+c)=20
(A)
= (22/3+21/3–2)(a22/3 + b21/3+c)=20
3 x – 7x + 4x – 1 2x + 4x – 1
⇒ a(24/3 + 2 – 2.22/3) + b[23/3 + 22/3 – 2.21/3]
+ c(22/3+21/3–23/3)=20
3 x + 7x + 4x – 1 = 13
⇒ 21/3(2a – 2b + c) + 23/3(a + b – c)
(3 x – 7x + 4x – 1 )(3 x + 7x + 4x – 1 ) + 22/3(–2a + b + c)= 20
20
⇒a+b–c= = 10
( 2x + 4x – 1 ) 2
2
(3 x ) ( )
2
= – 7x + 4x – 1 2x + 4x – 1
Sol 22: A → p; B → p, r, s; C → p, r; D → p, q, r
(D) N = 2 + 5 – 6 – 3 5 + 14 – 6 5
N = 2 + 5 – 6 – 3 5 + (3 – 5)2
N= 2 + 5 – 6 – 3 5 + (– 5 + 3) = 2+ 5 – 9– 4 5
N= 2 + 5 – ( 5 – 2)2 = 2 + 5 – 5 + 2 = 4 = 2
MATHEMATICS
FOR JEE MAIN & ADVANCED
SECOND
EDITION
Exhaustive Theory
(Now Revised)
Formula Sheet
9000+ Problems
based on latest JEE pattern
1. QUADRATIC POLYNOMIAL
Quadratic polynomial: A Polynomial of degree 2 in one variable of the type f ( x ) = ax2 + bx + c where a, b, c, ∈ R
and a ≠ 0 is called a quadratic polynomial. ‘a’ is called the leading coefficient and ‘c’ is called the absolute term of
f (x). If a = 0, then y = bx + c is called a linear polynomial and if=a 0, b ≠ 0 & c = 0 then y = bx is called an odd
linear polynomial since f ( y ) + f ( − y ) =
0
Standard appearance of a polynomial of degree n is f ( x =
) anXn + an−1 X x −1 + an−2 Xn−2 + .... + a1 X + a0
Where an ≠ 0 & an , an−1 ,....a0 ∈ R ; n = 0, 1, 2…
When the Highest exponent is 3 → It is a cubic polynomial
When the Highest exponent 4 → It is a biquadratic polynomial
For different values of a, b, and c there can be 6 different graphs of y = ax2 + bx + c
y y
y
a>0 a>0
a>0 D<0
D>0 D=0
x1 x2
O x O x1 x2 x O x
y y
x y x
O O x O
x1 x2
a<0 a<0
a<0
D>0 D<0
D=0
2. QUADRATIC EQUATION
A quadratic polynomial expression equated to zero becomes a quadratic equation and the values of x which satisfy
the equation are called roots/ zeros of the Quadratic Equation.
General form: ax2 + bx + c =0
Where a, b, c, ∈ R and a ≠ 0 , the numbers a, b and c are called the coefficients of the equation.
a is called the leading coefficient, b is called the middle coefficient and c is called the constant term.
e.g 3x2 + x + 5 = 0, − x2 + 7x + 5 = 0, x2 + x = 0, x2 = 0
x − 2)3 x2 + 5x − 8
3x2 − 6x
11x − 8
11x − 22
14
( x − 2) q(x) + R .
polynomial as 3x2 + 5x − 8 =
Thus, we can conclude that 3x2 + 5x − 8 = ( x − 2 )( 3x + 11 ) + 14
Where the quotient q ( x=
) 3x + 11 and the remainder R = 14.
M a them a tics | 2.3
Illustration 2: Use the remainder theorem to find the remainder when f ( x ) = 3x2 + 5x − 8 is divided by (x – 2)
(JEE MAIN)
Sol: Use Remainder theorem. Put x = 2 in f(x). Since we are dividing f ( x ) = 3x2 + 5x − 8 by (x – 2), we consider x = 2.
Hence, the remainder R is given by
R= f ( 2=
) 3 ( 2 ) + 5 ( 2 ) − 8= 14
2
Illustration 3: By using the remainder theorem, determine the remainder when 3 ( x ) − x2 − 20x + 5 is divided by
3
(
Illustration 4: Find the remainder R by long division and by the Remainder Theorem 2x 4 − 10x2 + 30x − 60 ÷ ( x + 4 ) . )
(JEE MAIN)
3 2
Sol: We can find the remainder in the given division problem by using 2x - 8x + 22x - 58
4 3 2
the long division method, i.e. similar to number division and also by x+4 2x + 0x - 10x + 30x - 60
the Remainder theorem, i.e. R = f(r). 4 4
-2x - 8x
Now using the Remainder Theorem: f ( x ) = 2x − 10x + 30x − 60
4 2
3 2
-8x - 10x
Remainder = f ( −4 ) = 2 ( −4 ) − 10 ( −4 ) + 30 ( −4 ) − 60 = 172
4 2
3 2
8x + 32x
This is the same answer we achieved by the long division method.
2
22x + 30x
2
-22x - 88x
-58x - 60
58x + 232
172
2 . 4 | Quadratic Equations and Inequalities
= ( )
−4x2 x3 + 4x − 8 + 16x3 + 64x = (
16 x3 + 4x − 8 + 128 =
128 )
3
Alter: x + 4x =
8
x7 + 64x2
Now divide x 7 + 64 x2 by x3 + 4x − 8 ⇒
x 3 +4x−8
Here, after division, the remainder will be the value of the expression x7 + 64x2 .
Thus, after dividing, the value is 128.
Illustration 7: A cubic polynomial P(x) contains only terms of the odd degree. When P (x) is divided by (x – 3), then
( )
the remainder is 6. If P(x) is divided by x2 − 9 , then the remainder is g(x). Find the value of g(2). (JEE MAIN)
Sol: Let p(x) = ax3 + bx , and use Remainder theorem to get the value of g(2).
Let p(x) = ax3 + bx ; By remainder theorem P(3) =6
P(3) = 3(b + 9a) = 6 ; 9a + b = 2 …..(i)
( 2
)
P(x) = x − 9 ax + (b + 9a) x
Given that the remainder is g(x) when P(x) is divided by x2 − 9 ( )
∴ g(x) =+
(b 9a) x
From (i) (b + 9a) = 2 ∴ g(x) =
2x ∴ g ( 2 ) =
4
Algebraic Method
ax2 + bx + c =0 ; Divide by a
2
bx c b b2 c b2 − 4ac
x2 + + =0 ⇒ x + = − =
a a 2a 4a2 a 4a2
M a them a tics | 2.5
b b2 − 4ac −b ± b2 − 4ac
x+ =
± ; ⇒ x =
2a 2a 2a
b2 − 4ac =
D (Discriminant)
−b + D −b − D −b c
α= ,β = ; α + β = , α.β =
2a 2a a a
ax2 + bx + c= 0 ⇒ x2 − ( α + β ) x + ( α.β )= 0
5. NATURE OF ROOTS
Given the Quadratic Equation ax2 + bx + c =0 , where a, b, c, ∈ R and a ≠ 0
Discriminant: D = b2 − 4ac
b c
⇒ = 0 & = 0 ∴b = c = 0 ∴y =ax2
a a
PLANCESS CONCEPTS
π
Illustration 8: Form a quadratic equation with rational coefficients having cos2 as one of its roots. (JEE MAIN)
8
(
Sol: If the coefficients are rational, then the irrational roots occur in conjugate pairs. Hence if one root is α + β )
( )
then other one will be α − β , Therefore, by using the formula x − ( sum of roots ) x + (product of roots ) =
2
0 we
can obtain the required equation.
π 1 π 1 π 1 1
cos2 = × 2cos2 = 1 + cos = 1 +
8 2 8 2 4 2 2
1 1 1
Thus, the other root is 1 − also Sum of roots = 1 and Product of roots =
2 2 8
M a them a tics | 2.7
1
Illustration 9: Find the quadratic equation with rational coefficients when one root is . (JEE MAIN)
( 2+ 5 )
Sol: Similar to Illustration 8.
If the coefficients are rational, then the irrational roots occur in conjugate pairs. Given that if one root is
1 1
α= = 5 − 2 , then the other root is β = = − 5 −2
(2 + 5) (2 − 5)
Sum of the roots α + β = − 4 and product of roots α β = -1. Thus, the required equation is x2 + 4x − 1 =0.
Illustration 10: If cos θ , sin φ , sin θ are in G.P then check the nature of the roots of x2 + 2 cot φ x+1 = 0?
(JEE MAIN)
Illustration 11: Form a quadratic equation with real coefficients when one root is 3 – 2i. (JEE MAIN)
Sol: Since the complex roots always occur in pairs, so the other root is 3 + 2i. Therefore, by obtaining the sum and
the product of the roots, we can form the required quadratic equation.
The sum of the roots is
6 . The product of the root is ( 3 + 2i ) × ( 3 − 2 i ) = 9 − 4 i2 = 9 + 4 = 13
(3 + 2i) + (3 − 2i) =
Hence, the equation is x2 − Sx + P =0
⇒ x2 − 6x + 13 =
0
Illustration 12: If p, q and r are positive rational numbers such that p>q>r and the quadratic equation (p + q – 2r)
x2 + (q + r – 2p)x + (r + p – 2q) = 0 has a root in (-1, 0) then find the nature of the roots of px2 + 2qx + r =0
(JEE ADVANCED)
Sol : In this problem, the sum of all coefficients is zero. Therefore one root is 1 and the other root is .
r + p − 2q r + p − 2q
. which also lies in (-1, 0). Hence, by solving − 1 < < 0 we can obtain the nature of roots of
p + q − 2r p + q − 2r
Illustration 13: Consider the quadratic polynomial f ( x ) =x2 − px + q where f ( x ) = 0 has prime roots. If p + q = 11
and =a p2 + q2 , then find the value of f ( a) where a is an odd positive integer. (JEE ADVANCED)
2
Sol: Here f(x) = x − px + q, hence by considering α and β as its root and using the formulae for sum and product
of roots and the given conditions, we get the values of f(a).
f(x) = x2 − px + q
Given α and β are prime
α + β =p ... (i);
αβ = q ... (ii)
Given p + q = 11 ⇒ α + β + αβ = 11
⇒ ( α + 1 )( β + 1 ) =12 ; α = 2, β= 3 are the only primes that solve this equation.
∴ f(x) = (x – 2)(x – 3) = x2 − 5x + 6
∴ p = 5, q = 6 ⇒ a = p2 + q2 = 25 + 36 = 51; f(51) = (51 – 2)(51 – 3) = 49 × 48 = 3422
Sol: In this problem, the maximum vertical distance d means the value of y.
The vertical distance is given by
y
d = −2x2 + 4x + 3 − ( x − 2 ) =
−2x2 + 3x + 5 ,
4
which is a parabola which opens downwards. 3
2
Its maximum value is the y-coordinate of -0.5 1
O x
-1
−b −3 3 -2 0.5 1 1.5 2 -2.5
the vertex which has x-coordinate equal to
= = .
2a 2 ( −2 ) 4
2
3 3 −9 18 40 49 Figure 2.8
Then y =−2 + 3 + 5 = + + =
4
4
8 8 8 8
Illustration 15: y = ax2 + bx + cC has no real roots. Prove that c(a+b+c) > 0. What can you say about expression
c(a – b + c)? (JEE ADVANCED)
Illustration 16: α , β are roots of the equation f ( x ) = x2 − 2x + 5 = 0 , then form a quadratic equation whose roots
are α3 + α2 − α + 22 & β3 + 4β2 − 7β + 35 . (JEE MAIN)
( ) ( )
Similarly β3 + 4β2 − 7β + 35 = β β2 − 2β + 5 + 6β2 − 12β + 35 = 6 β2 − 2β + 5 + 5 =5
2
D1 : D2 Equation is x − 12x + 35 =
0
dy d2 y
y ax2 + bx + c > 0 ∀ x ∈ R , then prove that polynomial z =y +
Illustration 17: If= + will also be greater
dx dx2
than 0. (JEE ADVANCED)
Sol: In this problem, the given equation =y ax2 + bx + c > 0 ∀ x ∈ R means a > 0 & b2 − 4ac < 0 . Hence, by
dy d2 y
substituting y in z =y + + and solving we will get the result.
dx dx2
Since, y>0 ⇒ a > 0 & b2 − 4ac < 0
Z = ax2 + bx + c + 2ax + b + 2a = ax2 + (b + 2a) x + b + c + 2a
Again, as a > 0 & b2 − 4ac < 0
α (b + 2a) – 4a(b+ c +2a) = b2 − 4ac − 4a2 < 0
2
D=
For the new expression since D < 0 and a > 0, it is always positive.
Sol: By solving these two equations, we get the quadratic equation; and as it has equal roots, hence D = 0.
(mx + c ) 4ax ; m2 x2 + 2 ( cm − 2a) x + c2 =
2
= 0
Given that the roots are equal. So, D = 0 ⇒ 4 ( cm − 2a) ⇒
2
= 4c2m2 ⇒ 4a2= 4acm
a
a=cm ⇒ c = ;
m
This is a condition for the line y = mx + c to be a tangent to the curve y 2 = 4ax .
2c
Illustration 19: Prove that the roots of the equation ax2 + bx + c =0 are given by (JEE MAIN)
−b b2 − 4ac
− b ± b2 − 4ac
Sol: We know that the roots of the quadratic equation ax2 + bx + c =0 are found by xx =
= . Therefore,
2a
in multiplying and dividing by − b b2 − 4ac we can prove the above problem.
ax2 + bx + c =0
2 2
2b c b b c
⇒ x + x += 0 ⇒ x + = −
a a 2a 2a a
b
2
b2 − 4ac b b2 − 4ac
⇒ x + = ⇒ x − = ±
2a 4a2 2a 2a
=⇒x
( −b )
=
2
(
− b2 − 4ac
⇒x
) 2c
2
2a −b b − 4ac −b ± b2 − 4ac
2 . 1 0 | Quadratic Equations and Inequalities
7 7
Illustration 20: Let f ( x ) = ax + bx + a which satisfies the equation f x + = f − x and the equation
2
4 4
f ( x=
) 7x + a has only one solution. Find the value of (a + b). (JEE ADVANCED)
) 7x + a has only one solution, i.e. D = 0 and f x + 74 = f 74 − x . Hence, by solving these two equations
Sol: As f ( x=
simultaneously we will get the values of a and b.
Given
f ( x ) = ax2 + bx + a …(i)
7 7
f x + = f − x …(ii)
4 4
and given that f ( x=
) 7x + a …(iii)
has only one solution. Now using (i) and (ii).
2 2
7 7 7 7 49 7 7 49 7 7
a a − x + b − x + a ⇒ a x2 +
a x + + b x + + = + x + b x + =
a + x2 − x + b − x
4 4 4 4 16 2 4 16 2 4
⇒ 7ax + 2bx = 0 ; (7a + 2b)x = 0 … (iv)
f ( x ) = 7x + a has only one solution, i.e., D is equals to zero.
ax 2 + (b − 7 ) x= 0 ⇒ D = (b− 7)2 − 4a × 0 ⇒ D = x2 (b − 7 ) = 0; b = 7
2
ax2 + bx + a = 7x + a ⇒
Using equation (iv), a = –2, Then a + b = 5
Illustration 21: If the equation 2x2 + 4xy + 7y 2 − 12x − 2y + t =0 where t is a parameter that has exactly one real
solution of the form (x, y). Find the value of (x + y). (JEE ADVANCED)
Sol: As the given equation has exactly one real solution, hence D = 0.
2x2 + 4x ( y − 3) + 7y 2 − 2y + t =0
D=0 (for one solution)
( ) (
⇒ 16 ( y − 3) − 8 7y 2 − 2y + t = 0 ⇒ 2 ( y − 3 ) − 7y 2 − 2y + t = 0
2 2
)
( ) ( )
⇒ 2 y 2 − 6y + 9 − 7y 2 − 2y + t = 0 ⇒ − 5y 2 − 10y + 18 − t = 0
⇒ 5y 2 + 10y + t − 18 =0
6. GRAPHICAL APPROACH
b
2
D
2
Let y = ax + bx + c ; y = a x + − a, b and c are real coefficients. …(i)
2a 4a2
−b −D −b
Equation (i) represents a parabola with vertex , and axis of the parabola is x =
2a 4a 2a
M a them a ti cs | 2.11
If a > 0, the parabola opens upward, while if a < 0, the parabola opens downward.
The parabola intersects the x-axis at points corresponding to the roots of ax2 + bx + c =0 . If this equation has
(a) D > 0 the parabola intersects x – axis at two real and distinct points.
−b
(b) D = 0 the parabola meets x-axis at x =
2a
(c) D < 0 then;
If a > 0, parabola completely lies above x-axis.
If a < 0 parabola completely lies below x-axis.
( )
then f x > 0 ∀ ∈ ( − ∞ , α ) ∪ ( β, ∞ ) a>0
2 D>0
(where α <β , and are the roots of ax + bx + c =0)
x
2.
( ) ( ) (
If a < 0 and D > 0 then f x < 0 ∀ x ∈ − ∞ , α ∪ β, ∞ ) y
where β > α
a<0
D>0
O x
O x
O x
6. y
( )
If a< 0 and D < 0 then f x < 0 ∀ x ∈ R
O
x
a>0
D<0
Illustration 22: The graph of a quadratic polynomial y = ax2 + bx + c is as shown in the figure below. Comment on
the sign of the following quantities. (JEE MAIN)
(A) b – c (B) bc (C) c – a (D) ab2
y
Sol: Here a < 0;
b c
− < 0 ⇒ b < 0; < 0 ⇒ c > 0. As b – c = (–ve) – (+ve); it must be negative;
a a x
O
Also, bc = (–ve)(+ve); this must be negative;
1 Figure 2.15
Then, β + = (–ve) (+ve); the product must be negative; finally,
α
c – a = (+ve) – (–ve), it must be positive.
{
∴ l ( SR ) − l ( SL ) = − (p − 2 ) − − (p − 1 ) =1
}
7. THEORY OF EQUATIONS
Consider α , β, γ the roots of ax3 + bx2 + cx + d =0 ; then
d a ( x − α )( x − β )( x − γ )
ax3 + bx2 + cx +=
( )
+ d a x2 − ( α + β ) ( x + αβ )( x − γ )
ax3 + bx2 + cx=
3
+ d a ( x − x ( α + β + γ ) + x ( αβ + βγ + γα ) − αβγ )
2
ax + bx + cx = 3 2
x + x + x + ≡ ( x − x ( α + β + γ ) + x ( γ + βγ + γα ) − αβγ )
3 b c 2 d 3 2
a a a
−b coefficient of x2 c coefficient of x
Comparing them, α + β + γ = ⇒− , αβ + βγ + γα= ⇒
a coefficient of x 3 a coefficient of x3
M a them a ti cs | 2.13
d constant term x
αβγ = − ⇒ −
a coefficient of x2
PLANCESS CONCEPTS
As a general rule
a0 Xn + a1 Xn−1 + a2 Xn−2 + a3 Xn−3 + .... + an =
0 has roots X1 , X 2 , X3 ......Xn
−a coefficient of Xn−1 a2 coefficient of X n−2
∑ X1 = 1 =
− , ∑ X1 X=
2
=
a0 coefficient of Xn a0 coefficient of X n
a coefficient of Xn−3 constant term an
( −1)
n
∑ X1 X 2 X 3 =
− 3
a
=
−
coefficient of Xn
, X1 X 2 X3 ....xn =
coefficient of X n
= ( −1) n
a0
0
−d c
Now αβγ
= , αβ + βγ + =
λα
a a
Illustration 25: If the two roots of cubic equation x3 + px2 + qx + r = 0 are equal in magnitude but opposite in sign,
find the relation between p, q, and r. (JEE MAIN)
Sol: Considering α, - α and β to be the roots and using the formula for the sum and product of roots, we can solve
above problem.
Let us assume the roots are α, - α and β
Then, α − α + β = −p ⇒ β = -p
−α2 − αp + αp = q ⇒ α2 = −q ; −α2β = −r ⇒ pq =
r
2 1 1
Illustration 27: If the roots of p (q – r) x2 + q(r – p)x + r(p – q) = 0 has equal roots, prove that = +
q p r
(JEE MAIN)
Sol: This problem can be solved in the manner shown in the previous illustration.
One root is 1 ; ∴ other root is 1
rp − rq
∴ Product = 1 = ; ∴ pq – pr = rp – rq
pq − pr
2 p+r 1 1
∴ q(p + r) = 2rp ∴ = = +
q pr r p
Sol: As we know, if α , β and γ are the roots of cubic equation ax3 + bx2 + cx + d =0
−b c −d
then α + β += γ , αβ + βγ + λα
= and αβγ = . Therefore, by using these formulae we can solve the above
illustration. a a a
α + β + γ =0 ; ∑ αβ = q ; αβγ = −r
( α + β + γ ) = 0 ⇒ α2 + β2 + γ2 = −2 ( αβ + βγ + γα ) ⇒ ∑ α2 = −2∑ αβ
2
Illustration 29: Form the cubic equation whose roots are greater by unity than the roots of x3 − 5x2 + 6x − 3 =0
(JEE ADVANCED)
γ 5; ∑ αβ= 6; αβγ= 3
α+β+ =
α1 = α + 1 β1 = β + 1 γ1 = γ + 1
∑ α1 = α + β + γ + 3 = 8
∑ α1β1 = α1β1 + α1 γ1 + β1 γ1 = ( α + 1 )( β + 1 ) + ( β + 1 )( γ + 1 ) + ( γ + 1 )( α + 1 ) = 19
∴ x3 − 8x2 + 19x − 15 =
0
M a them a ti cs | 2.15
Alternate Method
y = x + 1 ⇒ x = y – 1 Put (y – 1) in given equation
⇒ ( y − 1) − 5 ( y − 1) + 6 ( y − 1) − 3 =
3 2
0 ⇒ y 3 − 1 − 3y 2 + 3y − 5y 2 − 5 + 10y + 6y − 6 − 3 = 0 ⇒ y 3 − 8y 2 + 19y − 15 = 0
Illustration 30: Find the sum of the squares and the sum of the cubes of the roots of x3 − ax2 + bx − c =0
(JEE ADVANCED)
Sol: Similar to the previous problem.
∑ α= a ; ∑ αβ
= b; ∑ αβγ= c;
(α + β + γ )
2
∴ α2 + β2 + γ 2 = − 2 ∑ α β = a2 − 2b
= ( α + β + γ ) ( α2 + β2 + γ2 − α β −βγ − γα ) +=
3αβγ ( a) ( a2 − 2b − b=
) + 3c (a) (a2 − 3b) + 3c
π
Illustration 31: If α , β, γ & δ are the roots of equation tan + x =
3tan3 x , then find the value of ∑ tan α
4 (JEE ADVANCED)
1 + tanx 3 tan x − tan3 x
Sol: Here, = 3 , therefore by putting tan x = y and solving we will get the result.
1 − tanx 1 − 3 tan2 x
⇒ 3y 4 − 6y 2 + 8y − 1= 0 ; ∑ y1= 0 ⇒ ∑ tan α= 0
Illustration 32: Find the number of quadratic equations with real roots remain unchanged even after squaring
their roots. (JEE ADVANCED)
Sol: As given α β = α2 β2 and α2 + β2 = α + β , therefore by solving it we will get the values of α and β .
α β = α2 β2 …..(i)
and α2 + β2 = α + β …..(ii)
PLANCESS CONCEPTS
b2 − 4ac ± D
( α − β) = ± ( α + β)
2
• − 4αβ =± =
a a
b2 − 2ac
(α + β)
2
• α2 + β2 = − 2αβ =
a2
b b2 − 4ac
• α2 − β2 = ( α + β )( α − β ) = ±
a2
• α3 + β3 = ( α + β ) − 3αβ ( α + β ) −
2 (
b b2 − 3ac )
a3
(b2 − ac) b2 − 4ac
( α − β) + 3αβ ( α − β ) = ( α − β ) ( α + β ) − 4αβ + 3αβ = ±
3 2
• α3 − β3 =
a3
b2 − 2ac c2
• α 4 + β4 = (α 2
)
+ β2 − 2α2β2 =
a2
−2
a2
• α2 + αβ + β2 = ( α + β)
2
− αβ = (b 2
)
− ac / a2
• α2β + β2 α = αβ ( α + β ) = −bc / a2
2 2
α β α 4 + β4 (α2 + β2 )2 − 2α2β2
• + = = =(b2 − 2ac / ac)2 − 2
β α 2 2 2 2
α β α β
Nitish Jhawar (JEE 2009 AIR 7)
8. TRANSFORMATION OF EQUATIONS
We now list some of the rules to form an equation whose roots are given in terms of the roots of another equation.
Let the given equation be a0 xn + a1 xn−1 + ....an−1 x + an =
0 …. (i)
x
Rule 1: To form an equation whose roots are k(≠0) times the roots of the equation, replace x by .
k
Rule 2: To form an equation whose roots are the negatives of the roots in the equation, replace x by –x.
In rule 1, y = kx Hence x = y/k. Now replace x by y/k and form the equation. We can do the same thing for the
other rules.
Alternatively, change the sign of the coefficients of Xn−1 , Xn−3 , Xn−5 ,.... etc. in (i).
Rule 3: To form an equation whose roots are k more than the roots of the equation, replace x by x – k.
Rule 4: To form an equation whose roots are reciprocals of the roots of the equation, replace x by
1
( x ≠ 0 ) and then multiply both sides by xn .
x
M a them a ti cs | 2.17
Rule 5: To form an equation whose roots are the square of the roots of the equation in (1) proceed as follows:
Step 1 Replace x by x in (1)
Step 2 Collect all the terms involving x on one side.
Step 3 Square both the sides and simplify.
( α + β )( αβ ) ± αβ ( α + β ) − 4 ( αβ )
2 3
For instance, to form an equation whose roots are the squares of the roots of
2
replace x by x to obtain.
x x + 2x − x + 2 =0 ⇒ x ( x − 1 ) =−2 ( x + 1 )
x ( x − 1 ) = 4 ( x + 1 ) or x3 − 6x2 − 7x − 4 =
2 2
Squaring both sides, we get 0
Rule 6: To form an equation whose roots are the cubes of the roots of the equation, proceed as follows:
Step 1 Replace x by x1/3
Step 2 Collect all the terms involving x1/3 and x2/3 on one side.
Step 3 Cube both the sides and simplify.
( )
Illustration 33: If a2 − 1 x2 + ( a − 1 ) x + a2 − 4a + 3 =0 is an identity in x, then find the value of a. (JEE MAIN)
Sol: The given relation is satisfied for all real values of x, so all the coefficients must be zero.
a2 − 1 =0 ⇒ a =±1
a − 1 = 0 ⇒ a = 1 Common value a is 1
a2 − 4a + 3 = 0 ⇒ 1,3
(
Illustration 34: If the equation a ( x − 1 ) + b x2 − 3x + 2 + x − a2 =
2
)
0 is satisfied for all x ∈ R, find all possible
ordered pairs (a, b). (JEE ADVANCED)
Sol: Similar to illustration 33, we can solve this illustration by taking all coefficients to be equal to zero.
( )
a ( x − 1 ) + b x2 − 3x + 2 + x − a2 =
2
0
2 . 1 8 | Quadratic Equations and Inequalities
⇒ ( a + b ) x2 − ( 2a + 3b − 1 ) x + 2b − a2 + a =0
⇒ b=1 ⇒ (a + 1) (a – 2) = 0 ; a = –1, 2
But from (i) a = –b ⇒ only a = –1 is the possible solution. Hence (a, b) = (–1, 1)
10.1 Intervals
Given E(x) = (x – a)(x – b)(x – c)(x – d) ≥ 0
To find the solution set of the above inequality we have to check the intervals in which E(x) is greater/less than zero.
Intervals
(a) Closed Interval: The set of all values of x, which lies between a & b and is also equal to a & b is known as a
closed interval, i.e. if a ≤ x ≤ b then it is denoted by x ∈ [a, b].
(b) Open Interval: The set of all values of x, which lies between a & b but equal to a & b is known as an open
interval, i.e. if a < x < b then it is denoted by a ∈ (a, b)
(c) Open-Closed Interval: The set of all values of x, which lies between a & b, equal to b, but not equal to a is
known as an open-closed interval, i.e. if a < x ≤ b then it is denoted by x ∈ (a, b].
(d) Closed-open Interval: The set of all values of x, which lies between a & b, equal to a but not equal to b is
called a closed-open interval, i.e. if a ≤ x < b, then it is denoted by x ∈ [a, b).
Note: (i) x ≥ a ⇒ [a, ∞ ) (ii) x > a ⇒ (a, ∞ )
(iii) x ≤ a ⇒ (- ∞ , a) (iv) x < a ⇒ (- ∞ , a)
(i)
( x − 2)( x − 3) > 1 ⇒ (x − 3)
> 1 iff x − 2 ≠ 0
( x − 2)( x − 5) (x − 5)
( x − 5) ( x − 8 ) ≥ 0
2
(ii) ⇒ ( x − 5) ( x − 8 ) ≥ 0 iff x − 5 ≠ 0
( x − 5)
( x − a1 ) 1 ( x − a2 ) 2 .... ( x − a) n
k k k
Where k1 , k 2 ……………. kn & r1 , r2 ……….. rn ∈ N and a1 , a2 , ……….. an & b1 , b2 ………. bn are fixed real numbers.
The points where the numerator becomes zero are called zeros or roots of the function and points where the
denominator becomes zero are called poles of the function. Find poles and zeros of the function f(x). The
corresponding zeros are a1 , a2 , ……….. an and poles are b1 , b2 ………. bn . Mark the poles and zeros on the real
numbers line. If there are n poles & n zeros the entire number line is divided into ‘n+1’ intervals. For f(x), a number
line is divided into ‘2n+1’ intervals.
Place a positive sign in the right-most interval and then alternate the sign in the neighboring interval if the pole or
zero dividing the two interval has appeared an odd number of times. If the pole or zero dividing the interval has
appeared an even number of times then retain the sign in the neighboring interval. The solution of f(x) > 0 is the
union of all the intervals in which the plus sign is placed, and the solution of f(x) < 0 is the union of all the intervals
in which minus sign is placed. This method is known as the WAVY CURVE method.
1st condition
( x − 1) > 0 ⇒ x > 1 x ≥ 2
( x − 2 ) > 0 ⇒ x > 2
x − 1 < 0 ⇒ x < 1
2nd condition x ≤1
x − 2<0⇒ x <2
∴ x ∈ ( −∞ ,1] ∪ [2, ∞ )
2 . 2 0 | Quadratic Equations and Inequalities
12
Illustration 35: (x – 3) (x+1) x − < 0, find range of x (JEE MAIN)
7
Sol:Comparing all brackets separately with 0, we can find the range of values for x.
12 12
x < −1 and < x < 3 ; ∴ x ∈ ( −∞ , −1 ) ∪ ,3
7 7
Type II: Inequalities involving repeating linear factors
( x − 1 ) ( x + 2 ) ( x − 3) ≤ 0
2 3
( x + 1) ( x + 2) ( x + 2)( x − 3) ≤ 0
2 2
⇒
( x + 1 ) ( x − 3) ( x − 5 ) ( x − 4 ) ( x − 2 )
101 2 11 200 555
<0 (JEE MAIN)
Sol: Comparing all brackets separately with 0, we can find the greatest integer.
{ } − ( x − 2) − 2 =0
2
The inequality − ( x − 2 )
x = -1, 3 , 5, 4, 2
Illustration 37:
( x − 1)( x + 2) ≥ 0 (JEE MAIN)
( x + 3)( x − 4 )
( x + a)( x + b ) ≥ 0 then x + c x + d ≠ 0 , and x + a x + b =
Sol: If ( )( ) ( )( ) 0
( x + c )( x + d)
Hence, x ≠ − 3, 4 & x = 1, -2 ; x ∈ ( −∞ , −3) U [-2, 1] U (4, ∞ )
x2 (x + 1)
Illustration 38: <0 (JEE MAIN)
(x − 3)3
Sol: Similar to the illustration above.
x +1
< 0 x ≠ 3, -1, 0 ; x ∈ (–1, 0) U (0, 3)
x −3
x2 − 1
Illustration 39: ≥ 1 (JEE MAIN)
x2 − 7x + 12
Sol: First reduce the given inequalities in rational form and then solve it in the manner similar to the illustration
above.
x2 − 1
≥1
( x − 4 )( x − 3)
M a them a ti cs | 2.21
∴
( x + 1)( x − 1) ≥ 1 ⇒ x2 − 1 − 1 ≥ 0
( x − 4 )( x − 3) x2 − 7x + 12
x2 − 1 − x2 + 7x − 12 7x − 13
∴ ≥ 0 ∴ ≥0
( x − 4 )( x − 3) ( x − 4 )( x − 3)
13
∴ x ≠ 3, 4 ; x ∈ ,3 ∪ (4, ∞ )
7
Type IV: Double inequality
3x2 − 7x + 8
Illustration 40: 1 < ≤2 (JEE ADVANCED)
x2 + 1
Sol: Here 3x2 − 7x + 8 > x2 + 1 therefore if D < 0 & if a > 0, then f(x) > 0 and always positive for all real x.
3x2 − 7x + 8 > x2 + 1 ⇒ 2x2 − 7x + 7 > 0 ;
D= b2 − 4ac = 49 – 56 = -7
∴ D < 0 & a > 0 ∴ always positive for all real x
3x2 − 7x + 8 ≤ 2x2 + 2 ⇒ x2 − 7x + 6 ≤ 0 ⇒ ( x − 1 )( x − 6 ) ≤ 0
x ∈ [1, 6] ; x ∈ [1, 6] ∩ R
Illustration 41: (x 2
)(
+ 3x + 1 x2 + 3x − 3 ≥ 5 ) (JEE ADVANCED)
Sol: Using x2 + 3x =
y ,we can solve this problem
Let x2 + 3x =
y ∴( y + 1 )( y − 3 ) ≥ 5
y 2 − 2y − 8 ≥ 0 ∴ (y – 4) (y + 2) ≥ 0
α2 α 1
By cross multiplication method,
= =
b1c2 − b2c1 − ( a1c2 − c1a2 ) a1b2 − b1a2
α2 α 1
= =
b1c2 − b2c1 c1a2 − a1c2 a1b2 − b1a2
b c − b2c1
α2 = 1 2 ... (i)
a1b2 − b1a2
2 . 2 2 | Quadratic Equations and Inequalities
c1a2 − a1c2
α=
a1b2 − b1a2 … (ii)
Divide (1)/(2)
b1c2 − b2c1
α= … (iii)
c1a2 − c2a1
equating (i) and (ii) ; ( c1 a2 − c2 a1 ) =( a1 b2 − b1 a2 ) (b1 c2 − b2 c1 ) is the condition for a common root.
2
a1 b1 c1
(ii) If both roots are common, then = =
a2 b2 c2
Illustration 42: Determine the values of m for which the equations 3x2 + 4mx + 2 =0 and 2x2 + 3x − 2 =0 may
have a common root. (JEE MAIN)
Sol: Consider α to be the common root of the given equations. Then, α must satisfy both the equations. Therefore,
by using a multiplication method we can solve this problem.
3α2 + 4mα + 2 = 0 ; 2α2 + 3α − 2 = 0
Using the cross multiplication method, we have
(–6 – 4)2 = (9 – 8m)(– 8m – 6)
⇒ 50 = (8m – 9)(4m +3) ⇒ 32m2 − 12m − 77 =
0
⇒ 32m2 − 56m + 44 =
0 ⇒ 8m(4m – 7)+11(4m – 7) = 0
11 7
⇒ (8m + 11)(4m – 7) = 0 ⇒ m= − ,
8 4
Illustration 43: The equation ax2 + bx + c and y ≥ 0 have two roots common, Find the value of (a + b).
(JEE ADVANCED)
Sol: We can reduce x3 − 2x2 + 2x − 1 =
2
(
0 to (x – 1) x2 − x + 1 = )
0 as the given equations have two common roots,
therefore − ω and − ω are the common roots (as both roots of a quadratic equation are either real or non-real).
( )
We have x3 − 2x2 + 2x − 1 = 0 ⇒ ( x − 1 ) x2 − x + 1 = 0
−1 + 3i
⇒ x = 1 or x = − ω , −ω2 , where ω =
2
Since ax2 + bx + a = 0 and x3 − 2x2 + 2x − 1 =0 have two roots in common, therefore −ω and −ω2 are the common
roots (as both roots of a quadratic equation are either real or non-real), also −ω is a root of ax2 + bx + a =0 . Hence.
( )
a 1 + ω2 − b ω = 0 ⇒ a ( −ω) − b ω = 0 (as 1 + w+ ω2 = 0)
⇒a+b=0
a<0
a<0
-b
2a
x
O -b x
2a -D
4a
Figure 2.17
Figure 2.18
PLANCESS CONCEPTS
If α is a repeated root, i.e., the two roots are α , α of the equation f(x) = 0, then α will be a root of the
df
derived equation f’(x) = 0 where f’(x) =
dx
If α is a repeated root common in f(x) = 0 and φ (x) = 0, then α is a common root both in f’(x) = 0 and
φ ‘(x) = 0.
Shrikant Nagori (JEE 2009 AIR 30)
x
Illustration 44: Find the range of y = 2
(JEE ADVANCED)
x − 5x + 9
Sol: Here as x ∈ R therefore D ≥ 0 . Hence, by solving these inequalities we can find the required range.
x = yx2 − 5yx + 9y; yx2 − 5yx + 9y − x =.
0
yx2 − (5y + 1 ) x + 9y =
0; x ∈ R D≥0
∴ (5y + 1 ) − 36 y ≥ 0 ;
2 2
∴ 25 y + 1 + 10y − 36 y 2 ≥ 0
2
(11y +1)(y – 1) ≤ 0 ; −1
y ∈ ,1
11
x
∴ ( x − 3) = 0
2
Putting the end points in the eq. 1= ; x2 − 6x + 9= 0
2
x − 5x + 9
−1 x
If D < 0, then 1 would be open, i.e. excluded; =
11 x2 − 5x + 9
( )
⇒ − x2 − 5x + 9= 11x ; 9 0 ; ∴ ( x + 3)= 0 ; ∴
∴ x2 + 6x +=
2 −1
11
remains closed
x 1
Alternative Solution:
= y =
x2 − 5x + 9 9
x −5 +
x
Apply the concept of Arithmetic mean > Geometric mean for the values for x and 9/x
2 . 2 4 | Quadratic Equations and Inequalities
9
x +
x 9
We have ≥ x*
2 x
9 9
Thus x + ≥ 6 for x ≥ 0 and x + ≤ 6 for x < 0
x x
Since the term is in the denominator if we consider its maximum value, we will get the minimum value of y and
vice versa.
1
The maximum value of y will be = 1 and
6 −5
1 −1
The minimum value of will be = .
−6 −5 11
−1
Thus the range of y is ,1
11
x2 + 2x − 3
Illustration 45: Find range of y = (JEE ADVANCED)
x2 + 2x − 8
Sol: Similar to the preceding problem, by taking b2 ‒ 4ac ≥ 0 we can solve it.
x2 + 2x ‒ 3 = yx2 + 2xy ‒ 8y
(y ‒ 1)x2 + (2y ‒ 2)x ‒ (8y ‒ 3) = 0 ; b2 ‒ 4ac ≥ 0
∴ (2y ‒ 2)2 + (4)(8y ‒ 3)(y ‒1) ≥ 0 ⇒ 4y2 + 4 ‒ 8y + 4(8y2 ‒ 3y ‒ 8y + 3) ≥ 0
4y2 + 4 ‒ 8y ‒ 44y ‒ 32y2 + 12 ≥ 0 ⇒ 36y2 ‒ 52y + 16 ≥ 0
4
∴ 9y2 ‒ 13y + 4 ≥ 0 ⇒ (y – 1) (9y – 4) ≥ 0 ; ∴ y ∈ − ∞ , ∪ (1, ∞ )
9
To verify if the bracket is open or closed, apply the end points in the equation,
4 4 x2 + 2x − 3
Check for y = ; =
9 9 x2 + 2x − 8
ax2 − 7x + 5
Illustration 46: Find the limits of ‘a’ such that y = is capable of all the values of ‘x’ being a real
quantity. 5x2 − 7x + a (JEE ADVANCED)
(5y − a) x2 − 7x ( y − 1) + ay − 5 =0 ;
( )
D = 49 y 2 + 1 − 2y − 4 ( ay − 5 )(5y − a) ; D≥0
M a them a ti cs | 2.25
a>0
D>0
d -b x-axis
b
2a a<0
2a D>0
d X-axis
Figure 2.20
Figure 2.19
d b x-axis a>0
2a D=0
a<0 -b
D=0 2a
d = x-axis
Figure 2.22
Figure 2.21
−b
(i) D ≥ 0 (ii) f(x) > 0 (iii) >d
2a
Type II: If both the roots are less than a specified number, say ‘d’, then
a>0
D>0
b d x-axis
b 2a a<0
2a D>0
d x-axis
Figure 2.24
Figure 2.23
2 . 2 6 | Quadratic Equations and Inequalities
=
-b d x=axis
2a a>0
D=0
a<0 -b
D=0 2a
= d x-axis
−b
(i) D ≥ 0 (ii) f(x) > 0 (iii) <d
2a
2 2
Illustration 47: If both the roots of the quadratic equation x + x 4 − 2k + k − 3k − 1 =( )
0 are less than 3, then find
the range of values of k. (JEE MAIN)
−b
Sol: Here both the roots of the given equation is less than 3, hence, D ≥ 0 , < 3 and f (3) > 0.
2a
() (
The equation is f x = x2 + x 4 − 2k + k 2 − 3k − 1= 0 )
D ≥ 0 …(i)
3
−b
d
< 3 ….(ii)
2a
f (3) > 0 ….(iii)
⇒ (k2 ‒ 4k + 4) ‒(k2 ‒ 3k ‒ 1) ≥ 0 5
⇒ –k + 5 ≥ 0 ⇒ k–5 ≤ 0 ;
(
k ∈ −∞ ,5
4 5
−4 ( 4 − 2k )
(ii) < 3 ; k – 2 < 3; k < 5
2
()
(iii) f 3 > 0 ⇒ 9 + 3(4 – 2k) + k2 – 3k – 1 > 0
a>0
a<0
b
2a d
d -b x-axis x-axis
2a
Type IV: Exactly one root lies in the interval (d, e) when d < e, then f(d)⋅f(e)<0
a>0
D>0
b d b e x-axis
2a e 2a
d x-axis a<0
D>0
Figure 2.29
Figure 2.30
a>0
D>0 e
b
2a d b d x-axis
e x-axis 2a a<0
D>0
Figure 2.31
Figure 2.32
()
Type V: If both the roots of f x = 0 are confined between real numbers‘d’ and ‘e’, where d < e. Then
b
(i) D ≥ 0, (ii) f(d)f(e) >0, (iii) d < − < e.
2a
d = e
d e
b x-axis
2a b
a<0 2a a<0
D=0 D<0
a>0
D>0
b a>0
2a D=0
d e x-axis -b
2a
= d x-axis
Figure 2.35
Figure 2.36
Type VI: One root is greater than e and the other root is less than ‘d’.
a>0
D>0
b d -b e x-axis
d 2a e 2a a<0
x-axis D>0
∴x =
−2 (hy + g) ± 4 (hy + g) − 4a by 2 + 2fy + c
2
⇒x=
( )
− (hy + g) ± h2 y 2 + g2 + 2ghy − 2afy − ac − aby 2
2a a
⇒ ax + hy + g =± h2 y 2 + g2 + 2ghy − aby 2 − 2afy − ac …..(ii)
At this point, the expression (i) can be resolved into two linear factors if
(h2
)
− ab y 2 + 2 ( gh − af ) y + g2 − ac is a perfect square and h2 ‒ ab > 0.
( )
But h2 − ab y 2 + 2 ( gh − af ) y + g2 − ac will be a perfect square if D = 0
Illustration 48: If the equation x2 + 16y2 ‒ 3x + 2 = 0 is satisfied by real values of x & y, then prove that
−1 1
x ∈[1, 2], y ∈ , (JEE MAIN)
8 8
Sol: For real values of x and y, D ≥ 0 . Solve this by taking the x term and the y term constant one by one.
x2 − 3x + 16y 2 + 2 =
0 ; D ≥ 0 as x ∈ R
(
⇒ 9 − 4 16y 2 + 2 ≥ 0 ; ) ⇒ 9 – 64y2 – 8 ≥ 0
∴ 64y 2 − 1 ≤ 0
−1 1
⇒ (8y – 1)(8y + 1) ≤ 0 ∴ y∈ ,
8 8
(
Hence, -64 x2 − 3x + 2 ≥ 0 )
Solving this, we get x ∈ [1, 2]
M a them a ti cs | 2.29
Illustration 49: Show that in the equation x2 ‒ 3xy + 2y2 ‒ 2x ‒ 3y ‒ 35 ‒ 0, for every real value of x there is a real
value of y. (JEE MAIN)
(
x2 − x ( 3y + 2 ) + 2y 2 − 3y − 35 =
0 )
2
Now, x = 3y + 2 + 2 quadratic in y . As the quadratic equation in y is a perfect square y + 12 .
( )
∴ The relation between x & y is a linear equation which is a straight line.
∴∀ x ∈ R , y is a real value.
( ) (
Illustration 50: If a1 x2 + b1 x + c1 y + a2 x2 + b2 x + c2 = )
0 find the condition that x is a rational function of y
(JEE ADVANCED)
Sol: For x is a rational function of y, its discriminant will be greater than or equal to zero, i.e. D ≥ 0.
x − ( a1 y + a2 ) + x (b1 y + b2 ) + ( c1 y + c2 ) =
2
0
− (b1 y + b2 ) ±
(b1 y + b2 ) − 4 ( a 1 y + a2 )( c1 y + c2 )
2
x=
2 ( a 1 y + a2 )
( ) (
⇒ b12 − 4a1c1 y 2 + 2 (b1b2 − 2a1c2 − 2a2c1 ) y + b22 − 4a2c2 ≥ 0 )
⇒ b12 − 4a1c1 > 0 and D ≤ 0
Solving this will result in a relation for which x is a rational function of y.
A(a, f(a))
A(a,f(a))
P (b,0) (b,0)
(a,0) (a,0)
B(b,f(B))
B(b,f(b))
One real root Odd number of real roots
Figure 2.39
2 . 3 0 | Quadratic Equations and Inequalities
But if f(a) and f(b) are of the same sign, then either f(x) = 0 have one real root or an even number of real roots
in (a, b)
B(b,f(b))
A(a,f(a)) B(b,f(b))
A(a,f(a))
P R
Q S
(a,0) (b,0)
(c) If the equation f(x) = 0 has two real roots a and b, then f’(x) = 0 will have at least one real root lying between
a and b (using Rolle’s theorem).
PLANCESS CONCEPTS
f ( x )= a0 xn + a1 xn−1 + a2 xn−2 + .... + an−1 x + an= 0 is the number of changes of the sign of coefficients from
positive to negative and negative to positive. For instance, in the equation x3 + 3x2 + 7x − 11 =
0 the
sign of the coefficients are +++- as there is just one change of sign, the number of positive roots of
x3 + 3x2 + 7x − 11 = 0 is at most 1.
Rule 2 : The maximum number of negative roots of the polynomial equation f(x) = 0 is the number of
changes from positive to negative and negative to positive in the sign of the coefficient of the equation
f(–x) = 0.
Shivam Agarwal (JEE 2009 AIR 27)
PROBLEM-SOLVING TACTICS
( )( ) (
(c) An equation of the form ax2 + bx + c1 ax2 + bx + c2 ..... ax2 + bx + cn = )
A . Where c1, c2, ......cn, A ∈ R , can be
solved by putting ax + bx = y.
2
M a them a ti cs | 2.31
(d) An equation of the form (x – a)(x – b)(x – c)(x – d) = ⇒ Awhere ab = cd, can be reduced to a product of two
ab
quadratic polynomials by putting y = x + .
2
(e) An equation of the form (x – a) (x – b)(x – c)(x – d) = A where a < b < c < d, b – a = d – c can be solved by a
( x − a) + ( x − b ) + ( x − c ) + ( x − d) = x − 1
change of variable y =
4 4
( a + b + c + d)
(f) A polynomial f(x, y) is said to be symmetric if f(x, y) = f(y, x) ∀ x, y. A symmetric polynomial can be represented
as a function of x + y and xy.
b
(c) To solve an equation of the type ap x + () p(x)
+c =0 where p(x) is an expression of x, put p(x)= y
1
(d) To solve an equation of the form a x2 + 1 + b x + 1 + c =
0 , put x + =y
2
x x x
1 1 1
and to solve a x2 + + b x − + c =0 , put x − =y
2 x x
x
(f) To solve an equation of the type (x + a)(x + b)(x + c)(x + d) + k = 0 where a+b =c+d, put x2 +(a+ b)x = y
(g) To solve an equation of the type ax + b = cx + d or ax2 + bx + c = dx + e , square both the sides.
Step 1: Transfer one of the radical to the other side and square both the sides.
Step 2: Keep the expression with radical sign on one side and transfer the remaining expression on the other side
Step 3: Now solve as in 7 above.
FORMULAE SHEET
(c) Nature of roots: (i) D > 0 ⇒ roots are real and distinct (unequal)
( ) ( ) ( )( )
(d) The roots α + iβ , α − iβ and α + β , α − β are the conjugate pair of each other.
(e) Sum and Product of roots : If α and β are the roots of a quadratic equation, then
(
(f) Equation in the form of roots: x2 − α + β x + α. β = 0 ) ( )
(g) In equation ax2 + bx + c= 0, a ≠ 0 If
(i) b = 0 ⇒ roots are of equal magnitude but of opposite sign.
(vi) a > 0, b > 0, c > 0 or a < 0, b < 0, c < 0 ⇒ both roots are –ve.
(vii) a > 0, b < 0, c > 0 or a < 0,b > 0, c < 0 ⇒ both roots are +ve.
2 2
(h) The equations a1 x + b1 x + c1 =
0 and a2 x + b2 x + c2 =
0 have
1 a b1 c1
(ii) Both roots common if = =
a2 b2 c2
b
2
D
2
(i) In equation ax + bx +=
c a x + −
2a 4a2
4ac − b2 −b
(i) If a>0, the equation has minimum value = at x= and there is no maximum value.
4a 2a
4ac − b2 −b
(ii) If a < 0, the equation has maximum value at x = and there is no minimum value.
4a 2a
( j) For cubic equation ax3 + bx2 + cx + d =0,
−b
(i) α+β+ γ =
a
c
(ii) αβ + βγ + λα =
a
−d
(iii) αβγ = … where α , β, γ are its roots.
a
M a them a ti cs | 2.33
Solved Examples
JEE Main/Boards 2− 5
=
Example 1: For what values of ‘m’ does the quadratic (2 + 5 )(2 − 5 )
equation (1 + m) x2 – 2(1 + 3m)x + (1 + 8m) = 0 have
equal roots? = 5 −2
Then the other root, x2 + px + q = x will be −2 − 5 ,
Sol: The roots are equal if discriminant (D)= 0.
α + β = ‒4 and αβ = ‒1
(
4(1 + 3m)2 – 4(1 + m)(1 + 8m) = 0 ⇒ 4 m2 − 3m =
0 )
Thus, the required quadratic equation is :
⇒ m = 0, 3
x2 − ( α + β ) x + αβ = 0 Or , x2 + 4x − 1 =0
Example 2: When pr = 2(q + s), where p, q, r, s are real
numbers, show that at least one of the equations x2 + Example 4: The quadratic equations x2 ‒ ax + b = 0 and
px + q and x2 + rx +s = 0 has real roots.
x2 ‒ px + q = 0 have a common root and the second
ap
Sol: For at least one of the given equation has equal roots, show that b + q = .
2
equations to have real roots means one of
Sol: By considering α and β to be the roots of eq. (i) and
their discriminant must be non negative. α to be the common root, we can solve the problem by
using the sum and product of roots formulae.
The given equations are
The given quadratic equations are
f(α ) =0 + px + q = 0 … (i)
x2 − ax + b =0 ... (i)
f(α ) =0 + rx + s = 0 … (ii)
x2 − px + q =0 ... (ii)
consider D1 and D2 be the discriminants of equations (i)
and (ii) respectively, Consider α and β to be the roots of eq. (i) and α to be
the common root.
D1 + D2 = p2 ‒ 4q + r2 ‒4s
From (i) α + β = a, α = b
= p2 + r2 ‒ 4(q + s)
From (ii) 2α = p, α2 = q
= p + r ‒ 2pr
2 2
ap
∴ b + q = αβ + α2 = α ( α + β ) =
= (p ‒ r) ≥ 0 [ p and r are real]
2
2
At least one of D1 and D2 must be non negative.
Example 5: If α and αn are the roots of the
Hence, at least one of the given equation has real roots. quadratic equation ax2 + bx + c = 0, then show that
1 1
1
c n+1 c n+1 −b
⇒ + =
1
∴α +β =
−1
2
3 2
and 4α2 + 2α − 1 = 04α − 3α= 4α + 2α − 1 ( ) α − 12
a a a
1 1 ( 1
) 1
4α3 − 3α= 4α2 + 2α − 1 α − − α + = β
Or ( ) can n+1
( )
+ cna n+1 +b =0. 2 2
Hence 1 3
− α + 4α= β− 3α is the other root.
2 2
Example 6: x2 + ax + bc = 0 and x + bx + ca = 0 have
a non zero common root and a ≠ b, then show that the 1 1 1
Example 9: the roots of + = are equal in
other roots are roots of the equation, x2 + cx + ab = 0, x +p x +q r
c ≠ 0. magnitude, but opposite in sign, show that p+ q = 2r
p2 + q2
Sol: By considering α to be the common root of and the product of the roots = −
2
the equations and β , γ to be the other roots of the
equations respectively, and then by using the sum and Sol: By considering α and − α as the roots of the given
product of roots formulae we can prove this. equation and then by using the sum and product of
roots formulae we can solve it.
Further, α + β = −a and αβ = bc ;
1 1 1
+ = ….(i)
α + γ = −b , α. γ = ca x +p x +q r
2α + β + γ = − ( a + b ) and α2βγ = abc2 … (i) ⇒ (x + q + x +p)r = x2 + (p + q)x + pq
∴ β + γ = c − 2c = −c (ii) … (ii) ⇒ x2 + (p + q – 2r)x + pq – r(p + q) = 0
and c2βγ = c2ba Since, its roots are equal in magnitude but opposite in
sign
∴ βγ = ab … (iii)
consider roots are α , − α.
From equation (ii) and (iii),
∴ α − α= p + q − 2r
β and γ are the roots of the equation x2 + cx + ab =
0
⇒ p+q=2r
Product of roots = pq – r(p + q)
Example 7: Solve for x when
(p + q)
2
p2 + q2
log10= (
log10 x log ) 2
x
x: x >1 = pq –
2
= −
2
(
i.e., qα2 + 2q − p2 αβ + qβ2 = 0 ) Sol: Consider y ∈ R and also that given as x ∈ R. Hence,
ax2 + x − 2
(
i.e., q α2 + 2αβ + β2 − p2 αβ = 0 ) the discriminant of y =
than or equal to zero. a + x − 2x2
must be greater
i.e., q ( α + β ) − p αβ = 0
2 2
Let y ∈ R; then,
2 2
i.e., p q − p q =
0 which is obviously true. ax2 + x − 2
y= for some x ∈ R
a + x − 2x2
Example 11: Find the value of ‘a’ for which (a + 2y) x2 + (1 – y)x – 2 – ay = 0
(
3x2 + 2 a2 + 1 x + a2 − 3a + 2 = )
0 possesses roots with
∴ (1 – y)2 + 4(a +2y)(2 + ay) ≥ 0 ; ∀y ∈R
opposite signs.
a2 − 3a + 2 1
∴
3
<0 Or a > −
8
( )
and a2 − 4a + 3 ( a + 2 ) ≤ 0
= (a – 2) (a – 1) < 0 and a ∈ (1, 2) And D > 0 1
Or a > − and (a – 3)(a – 1) < 0
( 2
)
4 a + 1 − 4.3 a − 3a + 2 > 0 ( 3
) 8
i.e. a ∈ [1, 3]
This equation will always hold true for a ∈ (1, 2)
– 2y γ – 2y + 40 y3 0 −1 − 3
⇒x=
2
⇒ y 2 + y − 20 ≤ 0
⇒ ≤ ⇒ ≤ ≤ Example 3: Solve the equation 2|x +1| − 2x = 2x − 1 + 1
(y + 5)(y – 4) 0 –5 y 4
Sol: By taking the conditions as x ≥ 0 and x ≤ 0 we can
solve this problem.
JEE Advanced/Boards
2x − 1 if x ≥ 0
x
2 −1 =
Example 1: Prove that y =
2
ax + x − 2
2
x
− 2 − 1 ( ) if x < 0
a + x − 2x
Case-I x ≥ 0
takes all real values for x ∈ R only if a ∈ [1, 3]
2 . 3 6 | Quadratic Equations and Inequalities
2|x +1| − 2x = 2x − 1 + 1 3 3
Case II : Let < x < 1 ; 2x – < x2
8 4
This is true ∀ x ≥ 0
Or 4 x2 – 8x + 3 > 0
Case-II x < 0 ; 2|x +1| − 2x =1 − 2x + 1
3 1
(2x – 3)(2x – 1)> 0; ∴ x ∈ ,
2|x +1| = 2 ; |x + 1| = 1 ; x = –2
8 2
Example 4: For what values of a are the roots of the Example 6: Solve the equation
(
2
)
equation a + 1 x − 3ax + 4a (
= 0 a ≠ − 1 real and less ) (2x 2
− 3x + 1 ) (2x 2
)9x2
+ 5x + 1 =
than 1?
Sol: Here the roots of the given equation have to be Sol: This problem is solved by dividing both sides by x2
real and less than 1, therefore D ≥ 0 ; f(1).(a + 1) > 0 1
and taking y = 2x +
and the x-coordinate of the vertex < 1 . x
Let f(x) = (a + 1) x2 − 3ax + 4a ( )(
2x2 − 3x + 1 2x2 + 5x + 1 = 9x2 ) ... (i)
Clearly, x = 0 does not satisfy (i), Therefore, we can
D ≥ 0 ; f(1).(a + 1) > 0 and x-coordinate of vertex < 1
rewrite equation (i) as
16
D ≥ 0 ⇒− ≤ a ≤ 0 … (i) 1 1
9 ... (ii)
7 2x − 3 + 2x + 5 + =
x x
(a + 1)f(1) > 0 ⇒ (2a + 1) (a + 1) > 0 1
∴ (y – 3)(y + 5) = 9 where y = 2x +
1 x
⇒ a < –1 or a > − ... (ii)
2 Or y2 + 2y ‒ 24 = 0
16 −1 ⇒ (y + 6)(y – 4) = 0 ⇒ y = 4, –6
By (i) & (ii) a ∈ − , −1 ∪ ,0 … (iii)
7 2
1
Since x coordinate of vertex x < 1, we have When y = –6, 2x + = –6
x
−1 ⇒ 2 x2 + 6x + 1 = 0
Combined with (iii) we get: a ∈ ,0
2
−6 ± 36 − 8 −3 ± 7
⇒ x= =
Example 5: Find all the values of x satisfying the 4 2
3 1
inequality ⇒ 2x − > 2 .
When y = 4, 2x + =4
4 x
Sol: First, we can reduce the given inequality as ⇒ 2 x2 – 4x + 1 =
3
logx 2x − > logx x2 . Then, by applying each case of ⇒ x=
4 ± 16 − 8 −2 ± 7
=
4 4 2
3 −−33±± 77 −−22±± 22
x > 1 and < x < 1 we can solve this problem. Thus, the solutions are x = , .
8 22 22
3 3
logx 2x − > 2 (∴ x ≠ 1 and x > ) Example 7: If α and β are the roots of the equation a
4 8
x2 + bx + c = 0, then find the equation whose roots are,
⇒ logx 2x − 3 > logx x2 … (i) α2 +
α
2
+ββ2,
2 1
+
1
?
4 α2 β2
3
Case I : Let x > 1 ; 2x − > x2
4 Sol: Using the sum and product of roots formulae, we
can get the value of α and β and then by using
Or 4 ⇒ – 8x + 3 < 0
x2 − ( sum of roots ) x + (product of roots ) =
0
1 3 3
Or 4 x − x − < 0 ∴ x ∈ 1,
2 2 2 we can arrive at the required equation.
α2 + β2 ,
1
+
1 a2 a
2 2 Hence – 11 + a = 0, a = 0 or a = 24
α β 9 3
Since a ≠ 0, a = 24
α2 + β2 b2 − 2ac b2 − 2ac
2 2
As S = α + β += (
a2
+
c2
) x2 − 11x + 24 =
∴ the common factor of
0
( αβ )
2
2
x − 14x + 48 =0
a2 + c2
2
(
= b − 2ac 2 2
ac
) is clearly x – 8 or the common root is x = 8.
Note: A shorter method is in eliminating a from both
Now the product of the roots will be expressions
( )( ) ( )
2
( acx )
2
− b2 − 2ac a2 + c2 x + b2 − 2ac 0
= Example 10: α and β are the roots of
a x2 + bx+ c= 0 and γ , δ be the roots of
Example 8: If an are the roots of a x2 + bx + c = 0 p x2 +qx + r = 0; . If α , β, γ , δ are
2
and γ , δ the roots of x + mx + n = 0 , then find the
equation whose roots are α γ + β δ and α δ + β γ ? in A.P., then find the ratio of their Discriminants.
= (α 2
)
+ β2 γδ + αβ γ 2 + δ2 ( ) … (ii) ⇒β − α = δ − γ ; ( β − α ) = ( δ − γ )
2 2
(β + α ) ( γ + δ)
2 2
= b2 n + m2ac − 4 acn / a2 2 − 4αβ = − 4 γδ
b2 4c q2 4r
Hence, from x2 – Sx + P = 0 ⇒ − = −
a2 a p2 p
bm
2 b2n + m2ac − 4acn
x − x+ 0
=
a a2 2 b2 − 4ac q2 − 4qr
⇒ =
a2 p2
Example 9: The expression x2 ‒ 11x + a = 0 and
D1 D2 D1 a2
x2 ‒ 14x + 2a = 0 must have a common factor and a ≠ = ⇒ =
0, Find the common factor and then the common root. a2 p2 D2 p2
p (a+ b) x + c(b− a) −b ± D
⇒ = Therefore by putting x2 + 2x = y and using x =
2x x 2 − c2 2a
we can solve this.
⇒ p(x2‒ c2) = 2(a + b)x2 ‒ 2c(a ‒ b)x
Put x2 + 2x = y … (i)
⇒ (2a + 2b – p) x2 – 2c(a – b)x + p c2 = 0
(y – 80)(y – 24) = 660
For this equation to have equal roots
4c2 ( a − b ) − 4pc2 ( 2a + 2b − p ) =
2
0 ⇒ y2 ‒ 104y + 1920 ‒ 660 = 0
⇒ y2 ‒ 104y + 1920 = 0
⇒ ( a − b ) − 2p ( a + b ) + p2 =
2
0
⇒ (y – 90)(y – 14) = 0 ⇒ y = 90 or 14
⇒ p22 − 2p ( a + b ) =− (a − b)
2 2
− 2 ± 42 − 4 × ( − 90 )
[p ‒ (a + b)] = 4ab 2
x= =−1 ± 94
2
⇒ p – (a + b) = ± 2 ab
When y = 14, (i) gives x2 + 2x – 14 = 0
( )
2
⇒ p=a+b±2 ab = a± b
− 2 ± 42 − 4 × ( − 14 )
x= =−1 ± 3 2
2
Example 12: Solve (x + 10)(x – 4)(x – 8)(x + 6)= 660
x
The solutions are: −1 ± 3 2 & −1 ± 94
2
Sol: By multiplying (x + 10)(x – 8)(x – 4)(x + 6) we get x − 5x + 9
( x2 + 2x – 80)( x2 + 2x – 24) = 660.
JEE Main/Boards
Q.12 If a and b are integers and the roots of equation Q.24 Let α be a root of the equation a x2 + bx + c = 0
x2 + ax + b =0 are rational, show that they will be and β be a root of the equation – ax2 + bx + c = 0. Show
integers. a
that there exists a root of the equation x2 + bx + c = 0
2
Q.13 For what values of m, can the following expression
be split as product of two linear factors?
(
that lies between α and β α , β ≠ 0 . )
(i) 3x2 − xy − 2y 2 + mx + y + 1 Q.25 Let a, b and c be integers with a > 1, and let p be a
prime number. Show that if ax2 + bx + c is equal to p for
(ii) 6 x2 − 7xy − 3y 2 + mx + 17y − 20 two distinct integral values of x, then it cannot be equal
to “2p” for any integral value of x. (a ≠ p).
x2 − 2x + 4
Q.14 Prove that the expression lies between Q.26 For a ≤ 0, determine all real roots of the equation:
x2 + 2x + 4
1
and 3 for all real values of x. x2 − 2a x − a − 3a2 =0.
3
Q.15 Find all the values of a for which the roots of the Q.27 Find the values of a for which the inequality
equation (1 + a) x+ ‒ 3ax + 4a = 0 exceed unity. x2 + ax + a2 + 6a < 0 is satisfied for all x ∈ (1, 2).
Q.16 If P(x) = a x2 + bx + c and Q(x) = ‒ax2 + bx + c Q.28 If the roots of 2x3 + x2 − 7 = 0 are α , β and
where ac ≠ 0, show that the equation P(x). Q(x) = 0 has f(x) = x + x(4 − 2k) + k − 3k − 1 = 0 ,
2 2
Q.4 Let a > 0, b > 0 & c > 0. Then both the roots of the
equation a x2 + bx + c = 0
( ) ( ) (
(A) − ∞ , 0 ∪ 6, ∞ (B) − ∞ , 0 ∪ 6, ∞ ( )
(A) Are real & negative (C) ( − ∞ , 0 ∪ 6, ∞ ) (D) (0, 6)
(B) Have negative real parts
Q.12 If α , β are roots of the equation
(C) Are rational numbers
x2 − 2mx + m2 − 1 =0 then the number of integral
(D) None values of m for which α , β ∈ (–2, 4) is
(A) 0 (B) 1 (C) 2 (D) All of these
Q.5 The equation x2 + bx + c = 0 has distinct roots. If 2
is subtracted from each root, the results are reciprocals
(
of the original roots. The value of b2 + c2 + bc equals ) Q.13 If x be the real number such that x3 + 4x =
the value of the expression x7 + 64x2 is
8 then
(A) x ≤ –2 or x ≥ 3 (B) x ≤ 2 or x ≥ 3
Q.3 Let a, b, c be real numbers with a ≠ 0 and let α , β be
(C) x ≤ –3 or x ≥ 2 (D) –3 ≤x≤ 2 the roots of the equation ax2 + bx + c = 0 . Express the
roots of a3 x2 + abcx + c3 =0 in terms of α , β . (2001)
Q.11 If exactly one root of the quadratic equation
()
f x = 0 – (a + 1)x + 2a = 0 lies in the interval (0, 3) Q.4 If α , β are the roots of ax2 + bx + c = 0 , (a ≠ 0)
then the set of values ‘a’ is given by and α + δ, β + δ are the roots of Ax + Bx + C =
2
0 , (A ≠
0) for some constant δ , then
M a them a ti cs | 2.41
a>0
3
( ) 2
(A) p − 3 3p − 1 q + q =0
(B) p − q ( 3p + 1 ) + q =
3 2
2
y=ax +bx+c 0
x
(C) p + q ( 3p − 1 ) + q =
3 2
-1 0 1 0
(D) p + q ( 3p + 1 ) + q =
3 2
0
Q.6 Let a, b, c, p, q be the real numbers. Suppose f k 2 ( ) Q.13 The number of solutions of log4 x −= (
1 log2 x − 3 ) ( )
are the roots of the equation is (2001)
Q.16 The quadratic equations x2 – 6x + a = 0 and x2 – cx Q.19 Let α and β be the roots of equation px2 + qx + r = 0,
+ 6 = 0 have one root in common. The other roots of 1 1
the first and second equations are integers in the ratio p ≠ 0. If p, q, r are in A.P. and + =4 , then the value
α β
4 : 3. Then the common root is (2008) of |α – β| is (2014)
JEE Advanced/Boards
Exercise 1 2
Q.4 When y + my + 2 is divided by (y – 1) then
the quotient is f(y) and the remainder is R1. When
Q.1 A quadratic polynomial y 2 + my + 2 is divided by (y + 1) then quotient is
g(y) and the remainder is R2. If R1 = R2, find the value
f ( x ) = x2 + ax + b is formed with one of its zeros of m.
4+3 3
being where a and b are integers Also, Q.5 Find the value of m for which the quadratic
2+ 3
0 and x2 − 14x + 2m =
equations x2 − 11x + m = 0 may
g ( x ) = x 4 + 2x3 − 10 x2 + 4x − 10 is a biquadrate have common root.
4+3 3
polynomial such that g = c 3 + d where c Q.6 The quadratic polynomial P(x) = ax2 + bx + C
2+ 3
has two different zeroes including –2. The quadratic
polynomial Q ( x ) = ax2 + cx + b has two different
and d are also integers. Find the values of a, b, c and d.
zeroes including 3. If α and β be the other zeroes of P(x)
α
and Q(x) respectively then find the value of .
Q.2 Find the range of values of a, such β
Instructions for Q.7 and Q.8
ax2 + 2 ( a + 1 ) x + 9a + 4
that f(x) = is always negative. Let α , β , γ be distinct real numbers such
x2 − 8x + 32
2
that a α + b α + c= ( sin θ) α2 + ( cos θ) α
Q.3 Let a, b, be arbitrary real numbers. Find the smallest
natural number ‘b’ for which the equation
a β2 + b β + c= ( sin θ) β2 + ( cos θ) β
x2 + 2 ( a + b ) x + ( a − b + 8 ) =0 has unequal real roots
a γ 2 + b γ + c= ( sin θ) γ2 + ( cos θ) γ
for all a ∈ R. (where a, b, c, ∈ R.)
M a them a ti cs | 2.43
( )
Q.7 log|x + 6| 2 .log2 x2 − x − 2 ≥ 1 ( ) Q.14 If the range of m, so that the equations
( x + 2mx + 7m − 12) = 0
2
Q.8 If
V1= sin θ ˆi + cos θ ˆj makes an angle π /3 with ( 4x − 4mx + 5m − 6 ) =
2
0
V2 = ˆi + ˆj + 2 kˆ then find the number of values of have two distinct real roots, is (a, b) then find (a + b).
θ ∈ 0, 2 π .
Q.15 Match the column
(α )( α )&α
3 2
2
− β2 3
− β3 β − α2 β3 . (C) Positive integral values of x (r) 2
satisfying
x +1 x +5
Q.10 Find the product of the real roots of the ≥ , is (are)
x −1 x +1
equation x2 + 18x + 30= 2 x2 + 18 x + 45
(D) The value of expression (s) 3
x2 + ax + 4 2π 4π 4π 4
Q.11 Let f(x) = is defined for all real, then sin sin + sin
x2 + bx + 4
7 7 7
, is
find the number of possible ordered pairs
8π 8 π 2π
sin + sin sin
7 7 7
(a – b) (where a, b, ∈ I).
Q.12 If the equation 9x2 − 12ax + 4 − a2 =0 has a unique Q.16 Find the product of uncommon real roots of the
root in (0, 1) then find the number of integers in the two polynomials
range of a.
P ( x ) = x 4 + 2x3 − 8x2 − 6x + 15 and
(d) 2( x + 2) − 2x +1 − 1= 2x +1 + 1
2 . 4 4 | Quadratic Equations and Inequalities
(e) For a ≤ 0, determine all real roots of the equation Q.25 Find the complete set of real values of ‘a’
2
x − 2a x − a − 3a =
0. 2 for which both roots of the quadratic equation
(a2
) (
0 lie on
− 6a + 5 x2 − a2 + 2a x + 6a − a2 − 8 = )
Q.18 (a) Let α , β and γ are the roots of the cubic either side of the origin.
x3 − 3x2 + 1 =0 . Find a cubic whose
Solve the inequality.
α β γ
roots are , and . 2
α−2 β−2 γ −2 x5
( )
4
Q.26 log2 x log1 − 20 log2 x + 148 < 0
Hence or otherwise find the value of ( α − 2 ) ( β − 2 ) ( γ − 2 ) . 4
2
(b) If α , β , γ are roots of the cubic 2011
Q.27 (log 100 x ) + (log 10 x ) + log x ≤ 14
2 2
x3 + 2x2 + 1 =0 , then find
(i) ( α β )
−1
+ (β γ )
−1
+ (γ α)
−1
(ii) α −2 + β−2 + γ −1
( )
Q.28 log1/2 x + 1 > log2 2 − x ( )
Q.19 If the range of parameter t in the interval (0, 2π),
satisfying (
Q.29 log1/5 2x2 + 5x + 1 < 0 )
( −2x 2
+ 5x − 10 )
( sin t ) x2 + 2 (1 + sint ) x + 9 sin t + 4
Exercise 2
for all real value of x is (a, b), then a + b =
kπ . ( )
Find the value of k. Single Correct Choice Type
() 2
Q.21 Let P x = x + bx + c where b and c are
integers. If P(x) is a factor of both x 4 + 6x2 + 25 and Q.2 For every x ∈ R , the polynomial x8 − x5 + x2 − x + 1 is
3x 4 + 4x2 + 28x + 5 . Find the value of P(1). (A) Positive
(B) Never positive
Q.22 If α , β are the roots of the equation,
(C) Positive as well as negative
x2 − 2x − a2 + 1 =0 and γ , δ are the roots of the
(D) Negative
2
(
equation, x − 2 a + 1 x + a a − 1 = )
0 such that ( )
α , β ∈( γ , δ ) then find the value of ‘a’.
Q.3 If the equation a(x ‒ 1)2 + b(x2 ‒3x + 2) + x ‒ a2 = 0 is
satisfied for all x ∈ R then the number of ordered pairs
Q.23 Let A denotes the set of values of x for which
of (a, b) can be
x+2
≤ 0 and B denotes the set of values of x for (A) 0 (B) 1 (C) 2 (D) Infinite
x−4
2
which x − ax − 4 ≤ 0 . If B is the subset of A, then find
the number of possible integral values of a. Q.4 The inequality The inequality y(‒1)≥ ‒4, y(1) ≤ 0 and
y(3)≥5 are known to hold for y = ax2+ bx+ c then the
least value of ‘a’ is :
Q.24 The quadratic ax2 + bx − c = 0 has two
different roots including the root ‒2. The equation (A) – 1/4 (B) –1/3 (C) 1/4 (D) 1/8
ax2 + cx + b =0 has two different roots including the
root 3. The absolute value of the product of the four
roots of the equation expressed in lowest rational is
p
. Find (p+ q).
q
M a them a ti cs | 2.45
(A) 125/64 (B) –27/8 (C) –125/8 (D) 27/8 f ( x ) = ax2 + ax + ( a + b ) and g(x) = ax2 + 3ax + 3a + b,
where a and b non-zero real numbers having same sign.
Q.8 For the quadratic polynomial f(x) = 4x ‒ 8kx + k, 2
Statement-I: Graphs of the both y = f(x) and y = g(x)
the statements which hold good are either completely lie above x-axis or lie completely
(A) There is only one integral k for which f(x) is non below x-axis ∀ x ∈R .because
negative ∀ x ∈ R Statement-II: If discriminant of f(x), D < 0, then y = f(x)
(B) for k < 0 the number zero lies between the zeros of ∀x ∈ R is of same sign and f(x+1) will also be of same
the polynomial. sign as that of f(x) . ∀x ∈ R
(C) f(x) = 0 has two distinct solution in (0, 1) for k ∈
(1/4, 4/7) Match the Columns
(D) Minimum value of y ∀ k ∈R is k (1+ 12k) ( )
Q.14 It is given that α , β β ≥ α are the roots of the
() 2
equation if f x = ax + bx + c . Also a f(t) > 0.
Q.9 The roots of the quadratic equation x2 ‒ 30x + b = 0
are positive and one of them is the square of the other. Match the condition given in column I with their
If the roots are r and s with r > s then corresponding conclusions given in column II.
Consider the polynomia (C) a < 0 and b² > 4ac (r) α<t<β
(D) a < 0 and b² = 4ac (s) t < α or r > β
P ( x ) = ( x − cos 36° )( x − cos 84° )( x − cos156° )
Q.15 Match the conditions on column I with the (C) Suppose a cubic polynomial f(x) = (r) 277
intervals in column II. x3+px2+qx+72 is divisible by both
() 2 2
Let f x = x − 2px + p − 1 , then x2+ax+b and x2+bx+a (where a, b, p,
q are cubic polynomial and a ≠ b).
The sum of the squares of the roots
Column I Column II
of the cubic polynomial, is
(A) Both the roots of f(x) = 0 are (p) (–1, ∞)
(s) 298
less than 4, if p ∈ R
(B) Both the roots of f(x) = 0 are (q) (–∞, 3)
greater than –2 if p ∈ R
Previous Years’ Questions
(C) exactly one root of f(x) = 0 (r) (0, 2)
lie in (–2, 4), if p ∈ R Q.1 Let (x, y, z) be points with integer coordinates
(D) 1 lies between the roots of (s) (–3, –1) U (3,5) satisfying the system of homogeneous equations
f(x) = 0, if p ∈ R 3x − y − z = 0, − 3x + z = 0, − 3x + 2y + z = 0 . Then the
number of such points for which x2 + y2 + z2 ≤ 100 is
Q.16 ……… (2009)
(B) The integral values of the parameters c (q) 4 Q.4 Let -1 ≤ p < 1. Show that the equation
for which the inequality 4x3 ‒ 3x ‒ p=0 has a unique root in the interval [1/2, 1]
and identify it, (2001)
7
1 + log2 2x2 + 2x + ≥ log2 (cx2 + c)
2
has at least one solution is
2
()
Q.5 Let f x = Ax + Bx + C where, A, B, C, are real
numbers. Prove that if f(x) is an integer whenever x is an
(C) Let P(x) = x2+bx+c, where b and c (r) 6 integer, then the numbers 2A, A + B and C are all integers.
are integers. If P(x) is a factor of both Conversely prove that if the numbers 2A, A + B and C are
x4+6x2+25 and 3x4+4x2+28x5, then the all integers, then f(x) is an integer whenever x is an integer.
value of P(1) equals (1998)
(s) 8
Q.6 Find the set of all solution of the equation
y
2 − 2y −1 − 1= 2y −1 + 1 (1997)
Q.17
Column I Column II
logequation
Q.7 Solve x in the following
2
(
( 2x +3) 6x + 23x + 21 )
(A) α, β are the roots of the equation K (p) 146 (
log( 2x +3) 6x2 + 23x + 21 = ) (
4 − log(3x + 7 ) 4x2 + 12x + 9 )
(x2-x) + x + 5 = 0. If K1 & K2 are the
two values of K for which the roots
=
(
4 − log(3x + 7 ) 4x2 + 12x + 9 ) (1987)
Then the equation f(x) = 0 has a root in R. Consider Q.15 Let p and q be real numbers such that p ≠ 0, p³ ≠
()
f=x kex − x for all real x where k is real constant. (2007) q and p³ ≠ –q. If α and β are non-zero complex numbers
3 3
satisfying α +β = –p and α + β = q , then a quadratic
x
Q.8 The line y = x meets y = ke for k ≤ 0 at α β
equation having and as its roots is (2010)
(A) No point (B) One point β α
(A)
1
(B) 1 (C) e (D) loge 2 ( ) ( ) (
(C) p3 − q x2 − 5p3 − 2q x + p3 − q =
0 )
e
Q.21 Let a, b, c, p, q be real numbers. Suppose α, β are Q.22 Let b = 6, with a and c satisfying (E). If α and β are
1 the roots of the quadratic equation ax2 + bx + c = 0,
the roots of the equation x2 + 2px + q = 0 and α, are ∞ n
β 1 1
the roots of the equation ax2 + 2bx + c = 0, where β2 ∉
then ∑ α + β is (2011)
n= 0
{−1, 0, 1}.
6
(A) 6 (B) 7 (C) (D) ∞
Statement-I: (p2 − q) (b2 − ac) ≥ 0 and 7
Statement-II: b ≠ pa or c ≠ qa (2008)
Q.23 A value of b for which the equations
(A) Statement-I is True, statement-II is True; statement-II x2 + bx - 1 = 0
is a correct explanation for statement-I
x2 + x + b = 0,
(B) Statement-I is True, statement-II is True; statement-II
is NOT a correct explanation for statement-I. have one root in common is (2011)
PlancEssential Questions
JEE Main/Boards JEE Advanced/Boards
Exercise 1 Exercise 1
Q. 12 Q. 15 Q. 18 Q. 7 Q. 10 Q. 13
Q. 20 Q. 22 Q. 24 Q. 17 Q. 19 Q. 22
Q. 25 Q. 30
Exercise 2
Exercise 2
Q. 3 Q. 8 Q. 9
Q. 5 Q. 9 Q. 12
Q. 11 Q. 14 Q. 17
Q. 16
Previous Years’ Questions
Previous Years’ Questions
Q. 2 Q. 5 Q. 6
Q. 5 Q. 6 Q. 8
Q. 15
Q. 13
M a them a ti cs | 2.49
Answer Key
JEE Main/Boards
Exercise 1
−16
−6 −8 −13 Q.15 a ∈ , − 1
Q.3 (i)m = , 1 (ii) m = (iii) m = 7
5 9 11
Q.4 K = 5 Q.18 x =± 1, ± 1 + 2 ( )
−11 7
Q.6 m = , Q.20 0
8 4
Q.7 qx2 − p ( q + 1 ) x + ( q + 1 ) =
2
0 (
Q.26 −a 1 + 6 ,a 1 + 2) ( )
Q.8 1 ≤ x ≤ 3
Q.27 −7 − 3 5 ≤ a ≤ − 4 + 2 3
Q.10 −1 ≤ x < 1 or 2 < x ≤ 4 2
Q.28 –3
Q.11 α2 β, α β2
1
−7 98 Q.29 k ∈ 0,
Q.13 (i) 4, (ii) 7, 3
2 3
Exercise 2
Q.20 A
JEE Advanced/Boards
Exercise 1
Q.2 a∈ − ∞ , − 1 ;
Q.1 a = 2, b = –11, c = 4, d = –1 Q.3 5
2
Q.4 0 Q.5 0 or 24 Q.6 11
Q.9 (a) (ii) and (iv); (b) x2 − p(p 4 − 5p2q+ 5q2 )x + p2 q2 (p2 − 4q)(p2 − q) =
0 Q.10 20
5 +1
Q.11 135 Q.12 10 Q.13 (a) x = ; (b) (a) ymin = 6
2
Q.14 6 Q.15 (A) S; (B) Q,R,S,T (C) R, S ; (D) P Q.16 6
Q.18 (a) 3y
3
− 9y 2 − 3y + 1 =0 ; ( α − 2 )( α − 2 )( γ − 2 ) =3 ; (b) (i) 2 ; (ii) – 4 Q.19 3
Q.22 a∈ − 1 , 1
a= 1 − 2 or 5 + 10
Q.20 Q.21 P(1) = 4
4
( ) (
Q.29 − ∞ , − 2.5 ∪ 0 , ∞ )
Exercise 2
Single Correct Choice Type
Comprehenstion Type
Q.10 A Q.11 B
Q.12 D Q.13 A
Q.16 A → r; B → p, q, r, s; C → q Q.17 A → q; B → r; C → p
Solutions
Exercise 1 −8
⇒m=
9
Sol 1: Equation px2 + qx + r =0.
−q (iii) Sum of roots is 6
The sum of roots of a quadratic equation is: .
p −b (4 + 2m)
c −q ⇒ =6⇒ =6 ⇒ 22m =−26
Let roots be = 1 ⇒ r1 + r2 = a 5 + 4m
a p
−13
⇒m=
Given that:- r12 + r22 = r1 + r2 ⇒ (r1 + r2 )2 − 2r1r2 = r1 + r2 11
⇒ (5 + 4m)x2 − (4 + 2m)x + (2 − m) =
0 ⇒ 3α2 + 4mα + 2 = 0 … (i)
and 2α2 + 3α − 2 = 0 … (ii)
(i) If the eq. has equal roots then b2 − 4ac =
0
Solving equation (ii) we get
⇒ [ −(4 + 2m)]2 − 4(5 + 4m)(2 − m) =0
2α2 + 4α − α − 2= 0 ⇒ (2α − 1)(α + 2)= 0
2
⇒ 4m + 16m + 16 − 4( −4m + 3m + 10) =
0 2 1
∴α = or α = − 2
2
20m2 + 4m − 24 =
0
3 1
5m2 + m − 6 =0 + 2m + 2 =0 (Putting α = )
4 2
11
0 ⇒ m = 1 or m = −6 / 5 .
⇒ (m − 1)(5m + 6) = m= −
8
(ii) Product of roots is 2 and 3 × 4 − 8m + 2 =0 (Putting α = −2 )
c (2 − m) 14 7
⇒ 2
=⇒ =2 ⇒ 9m =−8 ⇒m= =
a 5 + 4m 8 4
2 . 5 2 | Quadratic Equations and Inequalities
From the first equation, we can write (x − 1)(x − 2) > 0 (i) The expression is 3x2 − xy − 2y 2 + mx + y + 1
+ – + −1 m 1
h= , a = 3, b = –2, g = , f = and c = 1
1 2 2 2 2
M a them a ti cs | 2.53
−7
⇒ ( 2m + 7 )(m − 4 ) =
0, ⇒ m = ,4 The conditions are
2
(ii) 6x2 − 7xy − 3y 2 + mx + 17y − 20 (i) D ≥ 0
P(x).Q(x) = 0 x2 − 2 x =x2 − 2x
Clearly both cannot be less than zero at the same time. When x < 0
−2b c −2 − 2 2
⇒ α=
+β ,=
αβ ⇒x= or x = −1
a a 2
2
For equation Ax + 2Bx + c =0 ( ) (
The values of x are −1, −1 − 2 ,1, 1 + 2 )
−2B b B Sol 19:
⇒ (α + β) + 2k = , ⇒k = −
A a A LHS = (x − 2)(x − 3) − 8(x − 1)(x − 3) + 9(x − 1)(x − 2)
C C
Also, ( α + k )( β + k ) = , ⇒ k 2 + (α + β)k + αβ = = x2 − 5x + 6 − 8x2 + 32x − 24 + 9x2 − 27x + 18 = 2x2
A A
2
b B −2b b B c C Which is always equal to RHS no matter what the value
⇒ − + − + =
a A a a A a A of x
∴ y ∈ [0, 1]
( ) ( )
t t
Sol 18: We have 15 + 4 14 + 15 − 4 14 30
=
∴ The given equation becomes (1 + a)y 2 − 3a(y) + 4a =
0
t x2 − 2 x
and =
where the roots of equation should be between (0 & 1)
( )
t
Let 15 + 4 14 y
=
0 1
1 or
⇒y+ =30 0 1
y
30 ± 896
⇒ y 2 − 30y + 1 =0 ⇒y= These conditions should be satisfied
2
2 (i) D≥0
y 15 ±
⇒= 224 ⇒ =
y 15 ± 4 14
2 ∴ 9a2 − 16a(a + 1) ≥ 0 ⇒ 7a2 + 16a ≤ 0
(15 + 4 14 )
t
15 ± 4 14
= −16
∴a∈ ,0
∴ t =1 or t =−1 7
M a them a ti cs | 2.55
af(d) > 0 & af(e) > 0 Sol 22: From condition of common root
( −b / a) ( −b1 / a1 )
2 2
2
Sol 21: (l − m)x + lx + 1 =0 ⇒ =
c/a c1 / a1
Let one root be α: other root = 2α
2
l l b2 b12 b ca
2α
⇒ α += ⇒=
α ⇒ = ⇒ =
m−l 3(m − l) ac a1c1 b1 c1a1
l
2α2 = (Product of roots) Sol 24: α is root of equation ax2 + bc + c =0
l−m
2
⇒ aα 2 + bα + c = 0
2l 1
⇒ =
9(m − l) 2 (l − m) Similarly −aβ2 + bβ + c = 0
2 ax2
⇒ 2l= 9(l − m) Let f(x) = + bx + c
2
2
⇒ 2l − (9)l + 9m =
0
aα 2 −aα2
f(=
α) + bα =
+c
2 2
The roots are real ⇒ b2 − 4ac ≥ 0
aβ2 3aβ2
9 f(=
β) + bβ +=
c
81 − 8 × 9m ≥ 0 ⇒ m ≤ 2 2
8
−3a2 2 2
∴ f(α ). f(
= β) α β <0
4
2 . 5 6 | Quadratic Equations and Inequalities
∴ By mean value theorem, there exists a root of f(x) Sol 27: x2 + ax + a2 + 6a < 0 is satisfied for all x ∈ (1, 2)
between α and β
∴ (1 and 2 exists between the roots)
2
Sol 25: Given, ax + bx + c − p =0 for two distinct α & β ∴ Bring condition for the given case.
−b c −p −7 − 3 5
∴α + β = & αβ = Solving we get ≤ a ≤ −4 + 2 3
a a 2
To prove ax2 + bx + c − 2p ≠ 0 for any integral value of x,
let us assume these exist integer R satisfying Sol 28: Given that, 2x3 + x2 − 7 =0
−1 −7
ax2 + bx + c − 2p =0 For roots, α + β=
+γ γα 0 & αβγ =
αβ + βα +=
2 2
⇒ ak 2 + bk + c − 2p =0 α β
For ∑ +
k 2 + bk c − p p β α
or + =
a a a α α β β γ γ
= + + 1 + + + 1 + + + 1 − 3
p β γ α γ α β
or (k − α )(k − β) ==an integer
a
p
Since p is a prime number ⇒ is an integer if a=p or
∑ αβ ∑ αβ ∑ αβ
a = + + − 3 =0 − 3 =−3
a=1 but a > 1 ∴ a = p βγ αγ γβ
⇒ (k – α) (k – β) = 1
Sol 29: (x − 3k)(x − (k + 3)) < 0
∴ either k – α = –1 and k – β = –1
⇒ α = β (not possible)
1 3
∴ There is contradiction
k+3 3k
For x < a
Exercise 2
2 2
Eqn becomes x + 2ax − 5a =
0
Single Correct Choice Type
−2a ± 24a2
⇒ x= =−a±a 6
2 Sol 1: (C) Given that a2 + b2 + c2 = 1
But since x < a We know that
∴a ( )
6 − 1 is the only possible solution. (a + b +c)2 = a2 + b2 + c2 + 2 (ab + bc + ca)
⇒ (a +b +c)2 = 1 + 2 (ab + bc + ca)
M a them a ti cs | 2.57
(a + b + c )2 − 1 1
⇒ ab + bc + ca = …(i) Sol 5: (A) α − 2 =
2 α
Also, 2(a2 + b2 + c2) – 2ab – 2bc –2ac ⇒ α2 − 2α − 1 = 0 ⇒ b = −2 , c = −1
∴ At least one of P and Q is real. Sol 7: (D) One root is α ∴ The other root = −α
Let third root = β
∴ P(x) & Q(x) = 0 has atleast 2 real roots
⇒ α − α + β= 2 ⇒ β= 2
2
Sol 3: (B) Given that x + 3x + 1 =0
Putting this value in the given equation
For roots, α + β = −3 αβ = 1
23 − 23 + 2a + 10 =0 ⇒ a =−5
2 2
α β [α(α + 1)]2 + [β(β + 1)]2 ∴ a ∈ ( −10,0)
+ = 2
1 + β α +1 (α + 1) (β + 1)2
( (α ) Sol 8: (B) x=
+ 1 log2 (2x + 3)2 − 2log(1980 − 2− x )
2
2
+ α ) + (β2 + β) − 2αβ(α + 1)(β + 1)
=
(αβ + α + β + 1)2 (2x + 3)2
⇒ 2x +1 =
1980 − 2− x
2
(α + β)2 − 2αβ + (α + β) − 2αβ(αβ + α + β + 1)
= ⇒ 1980 × 2 × 2x − 2= (2x + 3)2
(αβ + α + β + 1)2
Let 2x = t
[9 − 2 − 3]2 − 2 × 1( −3 + 2) ⇒ t2 + 6t + 11 + 1980 × 2t =
0
= = 16 + 2 = 18
2
(1 − 3 + 1)
Now 2α × 2β =
11
⇒ 2α+β= 11 ⇒ α + β= log2 11
Sol 4: (B) ax2 + bx + c =0
a > 0, b > 0 & c > 0 Sol 9: (B) Product of H.C.F. & L.C.M. of two numbers =
product of the nos
−b ± b2 − 4ac
⇒α=
2a ∴ 16 × LCM =
295680
Sol 10: (A) Given that 4y 2 + 4xy + x + 6 =0 Sol 13: (C) (x3 + 4x)2 = 82
Sol 11: (B) If exactly one root lies in (0, 3) (as interval ⇒ x7 + 8x2(x3 + 4x) = 16(x3 + 4x);
is open) ⇒ x7 + 8x2 × 8 = 16 × 8;
⇒ f(0)f(3) < 0 ⇒ x7 + 64x2 = 128
∴ 2a(6 − a) < 0
Sol 14: (D) Given equations have real roots so,
⇒ a ∈ ( −∞ ,0) ∪ (6, ∞ )
a2 − 8b ≥ 0 ⇒ a2 ≥ 8b and 4b2 − 4a ≥ 0 ∴ ⇒ b2 ≥ a
Now we check at boundaries
⇒ b 4 ≥ a2 ≥ 8b
At a = 0 ⇒ x – x = 0
2
∴ a ∈ ( −∞ ,0] ∪ (6, ∞ ) D= a2 + 36
2
D2 is always > 0
Sol 12: (D) x2 − 2mx + m2 − 1 =0
∴ The equation has atleast two real roots.
Since both roots lies between (–2, 4)
−b
⇒ D ≥ 0 af(d) > 0 & af(e) > 0 and d < <e Sol 16: (B) f(x) = x2 + ax + b
2a
2 2
(i) ⇒ 4m − 4(m − 1) ≥ 0 For X ∈ [0, 2]
⇒ (16 − 8m + m2 − 1) > 0
Previous Years’ Questions
⇒ m ∈ ( −∞ ,3) ∪ (5, ∞ )
⇒− 2 <
−b
< 4 ⇒ −2 < m < 4
(
Sol 1: (i) Given x2 − 8kx + 16 k 2 − k + 1 =)
0
2a
D
Now, = 64 {k − (k − k + 1=
2 2
)} 64 (k − 1 ) > 0
Combining all the above three conditions, we get
∴ m ∈ ( −1,3) ∴k > 1
b 8k
∴ Integral values of m are 0, 1, 2 (ii) − >4⇒ > 4⇒k >1
2a 2
M a them a ti cs | 2.59
(iii) f(4) ≥ 0
( )
⇒ x x − α2β − αβ2 x + α2β = 0 ( )
( 2
)
16 – 32k + 16 k − k + 1 ≥ 0 ⇒ k − 3k + 2 ≥ 0 2
a3 x2 + abcx + c3 =
0
⇒ (k – 2)(k – 1) ≥ 0
⇒ x = α2β, α β2 which is the required answer.
⇒ k ≤ 1 or k ≥ 2 Hence, k = 2.
b c
2x 1 Sol 4: Since α + β = − , αβ = and
Sol 2: Given > a a
2x2 + 5x − 2 x +1
B C
α+δ+β+δ = − , ( α + δ )( β + δ ) =
2x 1 A A
⇒ − >0;
( 2x + 1 ) ( x + 2) ( x + 1 ) Now, α − β = ( α + δ ) − (β + δ )
( ) (
a1 x2 + b1 x + c1 y + a2 x2 + b2 x + c2 =
0 ) 2
⇒ ( α − β ) = ( α + δ ) − ( β + δ )
2
x ( a1 y + a2 ) + x (b1 y + b2 ) + ( c1 y + c2 ) =
2
0
2
⇒ ( α + β ) − 4αβ= ( α + δ ) − ( β + δ ) − 4 ( α + δ ) . ( β + δ )
2
− ( 3x + 2 )
⇒ >0
( 2x + 1)( x + 1)( x + 2) b 4c B
2
4C
2
⇒ − − = − −
a a A A
Using number line rule
b2 4c B2 4C b2 − 4ac B2 − 4AC
2 1 ⇒ − = − ⇒ =
∴ x ∈ ( −2, − 1 ) ∪ − , − a2 a A2 A a2 A2
3 2
Sol 5: From figure it is clear that if a > 0, then f(– 1) < 0
Sol 3: Since ax2 + bx + c =0 in terms of α , β . and f(1) < 0, if a < 0, f(–1) > 0 and f(1) > 0. In both cases,
⇒ α + β = −b / a and α β =c / a af(–1)< 0 and af(–1) < 0
⇒ a (a – b + c) < 0 and a (a + b + c) < 0
Now, a3 x2 + abc x + c3 =
0 ………(i)
On dividing by a2, we get
On dividing the equation by c2 , we get
b c b c
a3 2 abcx c3 1− + < 0 and 1 + + < 0
x + + 0
= a a a a
c2 c2 c2
On combining both, we get
2
ax ax
⇒ a + b + c =0 y
c c
a<0
2
y=ax +bx+c
ax
⇒ = α , β are the roots -1 1
c 0
x
y
c c a>0
⇒ x = α , β are the roots
a a 2
y=ax +bx+c
∴ α + β = − 2p … (i)
⇒ x − α βx − αβ x + ( αβ ) = 0
2 2 2 3
αβ = q … (ii)
2 . 6 0 | Quadratic Equations and Inequalities
(
Now, p2 − q b2 − ac )( ) Alternate solution
1
2 2
α + β 2
α+ β Given x − 2 + x − 2 − 2 =0
α
− a2
= − αβ
−2
−2
β
( ) (
⇒ x − 2 + 2 + x − 2 −1 =
0 )
∴ x − 2 =−2, 1 (neglecting – 2)
α2 2 1
2 ⇒ x−2 =
1
=
16
( α − β) α − ≥ 0
β
⇒ x = 3, 1
∴ Statement-I is true. ⇒ Sum of roots = 4
a
Again now pa = − (α + β) Sol 8: (B) If a1 x2 + b1 x + c1 =
0 and a2 x2 + b2 x + c2 =
0
2
Have a common real root, then
a 1
and b = − α + Since, pa ≠ b
⇒ ( a1c2 − a2c1 ) = (b1c2 − b2c1 )( a1b2 − a2b1 )
2
2 β
1 x2 + bx − 1 =0
⇒ α + ≠ α + β ⇒ β2 ≠ 1, β ≠ {−1, 0, 1} ∴
2
have a common root.
β x + x +b = 0
{ }
⇒ α3 α3 + 1 + 3 α ( α + 1 ) = −p3 (cubing both sides) ⇒ x = 5 [∴ x = 2 make log (x – 3) undefined].
Hence, one solution exits.
⇒ q (q + 1 – 3p) = −p3
⇒ p3 − ( 3p − 1 ) q + q2 =
0 Sol 14: (B) Given c < 0 < b
Since α + β = −b ... (i)
Sol 11: (B) As we know ax2 + bx + c > 0 for all x ∈R , if
a > 0 and D < 0 and α β =c ... (ii)
(
⇒ 4 a2 + 3a − 10 < 0 ) From Eq. (i), b > 0 ⇒ − b < 0 ⇒ α + β < 0
Sol 13: (B) Given Sol 16: (D) Let α and 4β be roots of x2 – 6x + a = 0 and
α, 3β be the roots of x2 – cx + 6 = 0, then
log4 ( x −=
1 ) log2 ( x −=
3) log ( x − 3)
41/2
α + 4β = 6 and 4αβ = a
⇒ log4 ( x=
− 1 ) 2log4 ( x − 3)
α + 3β = c and 3αβ = 6.
⇒ log4 ( x −=
1 ) log4 ( x − 3)
2
We get αβ = 2 ⇒ a = 8
⇒ ( x − 3) =x − 1 ⇒ x2 − 7x + 10 =
2
0 So the first equation is x2 – 6x + 8 = 0 ⇒ x = 2, 4
⇒ ( x − 2) ( x − 5) =
0 ⇒ x = 2 or x = 5 If α = 2 and 4β = 4 then 3β = 3
2 . 6 2 | Quadratic Equations and Inequalities
Now | α − β=| (α + β)2 − 4αβ ax2 + 2(a + 1)x + (9a + 4) < 0 for all x
2
∴ 2(a + 1) − 4a(9a + 4) < 0 Q(x) = ax2 + cx + b ( β & 3 are roots)
x2 − x − 2 ≤ | x + 6 |
Sol 4: When y 2 + my + 2 is divided by (y − 1) the
x2 − 2x − 8 ≤ 0 when x ∈ ( −5, −6)
remainder = f(1) = 1 + m + 2 = 3 + m
Similarly R 2 =g( −1) =3 − m ⇒ x ∈ ( −2, +4) ⇒ no possible value of x
x2 − x − 2 ≤ −x − 6 ⇒ x2 + 4 ≤ 0 ⇒ not possible
Sol 5: x2 − 11x + m =
0 and x2 − 14x + 2m =
0
∴ x ∈ ( −7, −∞ ) ∪ ( −5, −2) ∪ (4, ∞ )
Let α be the common root
2 2 Sol 8: V1= sin θˆi + cos θˆj
Let α − 11α + m = 0 and α − 14α + 2m = 0
∴ 3α − m = 0 ⇒ α =
m
. Substituting V2 = ˆi + ˆj + 2 kˆ
3
angel between V1 & V2 =α = π / 3
2 m2 8m
⇒ m − 11m + m =
0 ⇒ − 0
=
9 3 9 3 sin θ + cos θ
cos α =
for m = 0, 24 the equations have common roots.
1×2
⇒ sin θ + cos θ = 1
2
Sol 6: p(x) = ax + bx + c ( α & − 2 are roots) 1×2 2
2 . 6 4 | Quadratic Equations and Inequalities
Let x2 + 18x + 30 =
t ∴ (m− 1)2 =0 ⇒ m =1
⇒= t2 4(t + 15)
t 2 t + 15 ⇒ = 1 1± 5
∴x− = 1 ⇒ x2 − x − 1 = 0 ⇒ x =
x 2
⇒ t2 − 4t − 60 =
0 ⇒ (t − 10)(t + 6) =
0 now in equation (i) LHS >0 ∴ x > 0
∴ t = 10 or t = −6 1+ 5
∴ x= only possible solution
2
⇒ x2 + 18x + 20= 0 ; d > 0
3
1 1
or x2 + 18x + 36= 0 ; d > 0 (b) Let x + = m & x3 += n
x x3
But also x2 + 18x + 45 > 0
M a them a ti cs | 2.65
6 2
1 3 1 5p 5p + 1
⇒ α(1 + α ) =
x + − x + 3 4 & 2a +1 = 4
x x m2 − n2
⇒ =
3
1 3 1 m+n 5p − 3
or α =
x + + x + 3 8
x x
5p − 3 5(p + 1) 5
3 ⇒ × = p
1 1 1 8 8 4
= m − n = x + − x3 + = 3 x +
x
3
x x
5p2 − 14p − 3 =0
1
The minimum value of x + 2
= 5p2 − 15p + p − 3 =0
x
−1
∴ f(x)min = 3 × 2 = 6 ⇒ p 3=
= or p
5
p = 3 is the integral value
Sol 14: Given that
X2 + 2mx + 7m – 12 = 0 …. (i) x2 − 3x + 4
(b) y =
4x2 – 4mx + 5m – 6 = 0 x2 + 3x + 4
2
For equation (i), D > 0 ⇒ x (y − 1) + 3x(y + 1) + 4(y − 1) =
0
Maximum value of m = 4 + - + -
19 -1 1 3
Then, a + b= +4
8
x ≠ 1 as x − 1 is in denominator the positive integral
19 + 32 51 values of x are 2 & 3 Ans (R ) (S)
= =
8 8
2π 4π 4π 8π 8π 2π
(d) sin sin + sin sin + sin sin f ( say )
=
2
Sol 15: (a) 4x − (5p + 1)x + 5p = 4 7 7 7 7 7
0
2π 4π 8π
β= 1 + α Let =A = B and =C
7 7 7
2 . 6 6 | Quadratic Equations and Inequalities
2π 4 π 6π −4 π
cos − − cos + cos −x2 + 4x − 3 + 2x + 5 =0 ⇒ x2 − 6x − 2 =0
1 7 7 7 7
f= 6 ± 40
2 12π 6π 10π ⇒ x= = 3 ± 10
− cos + cos − cos 2
7 7 7
which doesn’t belong to (1,3)
1 2π 4π 12π 10π ∴ x=
1 is the only solution
cos + cos − cos − cos
2 7 7 7 7 (b) 3 |x2+4x+2|=5x−4
4π 4π 10π ⇒ 3x2+12x+6=5x − 4
cos = cos 2π − = cos
7 7 7 ⇒ 3x2+7x+10=0
∴ f =0 −7 ± 49 − 120
⇒ x⇒=
6
Sol 16: x 4 + 2x3 − 8x2 − 6x + 15 =
p(x) Which indicates x is imaginary here. So, this is not
acceptable.
Q(x) = x3 + 4x2 − x − 10 Case II: x2+4x+2 < 0
By trial one root of Q(x) = −2 −3(x2+4x+2) = 5x − 4
= x2 (x2 + 2x − 5) − 3(x2 + 2x − 5) = (x2 − 3)(x2 + 2x − 5) x will be the union of Eq. (i) and Eq. (ii)
(c) For x ≥ −1
The uncommon real roots are
x=
3 ,x =
− 3 &x =
−2 x3 + x 2 − x − 1 =0
(x + 1)(x2 − 0x − 1)
2 2
Sol 17: (a) (x − 1) x − 4x + 3 + 2x + 3x − 5 =0
(x + 1)(x2 + 0x − 1) ⇒ (x +1)2 (x – 1) = 0
2 2
(x − 1) x − 4x + 3 + 2x + 5x − 2x − 5 =0
for x < ‒ 1
(x − 1) x2 − 4x + 3 + (x − 1)(2x + 5) =
0 ∴ –x3 – 1 + x2 – x – 2 = 0
x = ‒ 1 is only solution
x2 − 2x + 8 =0
D < 0 so not possible x = –1, 1
Sol 18: Given that x3 − 3x2 + 1 =0 Sol 20: Minimum value of quadratic occurs at
⇒ α + β + γ = 3 , αβ + βγ + γα = 0 , αβγ = −1 −b 4p p
x
= = =
2a 8 2
Now we have (α − 2)(β − 2)( γ − 2)
When xmin ∈ [0,2]
= (αβ − 2α − 2β + 4)( γ − 2)
∴ f(x)
= min f(x
= min ) 3
= αβγ − 2(αβ + βγ + γα ) + 4(α + β + γ ) − 8
⇒ p ∈ [0, 4]
=−1 + 12 − 8 =3
⇒ p2 − 2p2 + p2 − 2p + 2 =3 ⇒ 2p = −1
Similarly we can find
−1
α β γ α β α ⇒ p= not true
, ∑ × , ∑ 2
α − 2 β − 2 λ − 2 α − 2 β − 2 α −2
when xmin < 0 ⇒ p<0
α β λ αβγ −1
= =
α − 2 β − 2 λ − 2 (α − 2)(β − 2)( γ − 2) 3
α β 3αβγ − 2(αβ + βγ + γα ) −3
∑α−2×β−2 =(α − 2)(β − 2)( γ − 2)
=
3
= −1 Xmin
Sol 21: Since p(x) is a factor of q(x) =x 4 + 6x2 + 25 and For product of root for (i) and (ii), we can
c b
r(x) = 3x 4 + 4x2 + 28x + 5 , then p(x) will also be a factor –2α = , 3β =
a a
of its linear combination.
We can write here
2
Now, r(x) - 3q(x) = x − 2x + 5
α – 2 = ‒3β and β + 3 = 2α
2
∴ p(x) = x − 2x + 5
Solving these two equations
α – 2 = ‒3 (2α - 3)
Sol 22: f(x) = x2 − 2x − a2 + 1 =(x − 1)2 − a2
⇒ α – 2 = ‒6α + 9
= (x − 1 − a)(x − 1 + a) 11
⇒ 7α = 11 ⇒α=
7
∴ α = a + 1 &β = 1 − a
11
Now g(α ) < 0 & g(β) < 0 Therefore, for β, β = 2 -3
7
∴ (a + 1)2 − 2(a + 1)(a + 1) + a2 − a < 0
22 1
= − 3=
−1 7 7
⇒ −a2 − 2a − 1 + a2 − a < 0 ⇒ a >
3 Absolute product of four roots
2 2
and (1 − a) − 2(a + 1)(1 − a) + a − a < 0
1 11 3 −2 66
= =
2
∴ 4a − 3a − 1 < 0 7 7 1 1 49
1 Therefore, (p + q) = 66 + 49 = 115
⇒ (4a + 1)(a − 1) < 0 ⇒ a ∈ − ,1
4
Sol 25: For origin to lie between the roots.
x+2
Sol 23: ≤ 0 ⇒ x ∈ [ −2, 4)
x−4 af(0) < 0
1 − 5 1 + 5 ⇒ (a + b)x2 + (1 − 2a − 3b)x + a + 2b − a2 =0
x ∈ −∞ , ∪ ,∞
2 2
Since this is satisfied by all x
Also x + 1 > 0 ⇒ x > –1 and x < 2 ⇒ a+b =0 , 2a + 3b =
1
⇒ b = 1 & a = −1
1 − 5 1 + 5
∴ x ∈ −1, ∪ ,2
2 2 also a + 2b − a2 =
0
Which is satisfied by (-1, 1)
2
Sol 29: log1/5 (2x + 5x + 1) < 0
4λ 2 − 2λ2
5: (A) x
Sol = = ,y
1 + λ2 1 + λ2
Let =
λ tan θ 8k ± 64k 2 − 16k k
α= k
=± k2 −
8 4
⇒ x = 2sin2θ & y = 2cos2θ
k k
f = x2 − xy + y 2 α = k − k2 − , β = k + k2 −
4 4
4 4 sin2θ cos2θ =
=− 4 − 2sin 4θ
2 k
∴ f lies between 2 and 6 or f ∈ [2,6] If k < 0 then α < 0, β > 0 k − > −k
4
∴a = 2 & b = 6 ∴a+b = 8
p
Let −k =p ⇒ p2 + > p
Multiple Correct Choice Type 4
−b
⇒ α3 =p … (ii) at n= = k − 4k 2
2a
From equation (i)
Sol 9: (A, D) α , α2 (α > 0) are roots of x2 − 30x + b =0
4α2 + 4α − 15 = 0 ⇒ 4α2 + 10α − 6α − 15 = 0
−5 3 α + α2 =30 ; α3 =b
⇒=
α ,=
α
2 2
α2 + α − 30 = 0
−125 27
=⇒p = or p (α + 6)(α − 5) = 0 ⇒ α = −6 , α = 5
8 8
α 5 ( α > 0)
=
Sol 8: (A, B, C) f(n) = 4n2 − 8kn + k , f(n) ≥ 0
α2 =25
=r 25,
= s 5,=
b 125
b+r −s =145
b+r +s =155
2
⇒ 4n − 8kn + k ≥ 0 ⇒∆ ≤ 0 b − r − s =95
⇒ 64k 2 − 16k ≤ 0 ⇒ (2k + 1)(2k − 1) ≤ 0 b −r + s =105
−1 1
⇒ k ∈ ,
2 2 Comprehension Type
∴ k=0 is the only integral solution (x − cos360 )(x − cos840 )(x − cos1560 )
Sol 10: (A) p(x) =
(b) Roots of the equation f(n) = 0 are co efficient of x2 is −(cos360 + cos840 + cos1560 )
M a them a ti cs | 2.71
–3 –1 3 5
2 . 7 2 | Quadratic Equations and Inequalities
1
3
1 ⇒ K 2 − 16K + 1 =0
Let x + =
m & x3 + = n
x x3 K1 K 2 (K1 + K 2 )2 − 2K1K 2 (16)2 − 2 × 1
∴= + = = 254
m2 − n2 1
3
1 K 2 K1 K1K 2 1
= = m − n = x + − x3 + 3
m+n x x x2 + ax + b
(B) y =
1 1 x2 + 2x + 3
= 3x + 3 × = 3 x +
x x
(y − 1)x2 + (2y − a)x + (3y − b) =
0
x +1
The minimum value of =2 (2y − a)2 − 4(y − 1)(3y − b) ≥ 0
x
∴ f(x)min = 6 ⇒ 4y 2 + a2 − 4ay − 4(3y 2 − (b + 3)y + b) ≥ 0
Sum of squares of roots = a2 + b2 + (1)2 Now, to find values of ‘a’ for which equation has unequal
real roots for all values of b.
= 81 + 1 + 64 = 146
i.e, above equation is true for all b.
∴ Discriminate, D < 0
Sol 1: Given 3x − y − z =0 … (i)
−3x + 2y + z =0 … (ii)
(
⇒ ( 4 − 2a) − 4 a2 + 4a − 4 < 0
2
2 2
)
⇒ 16 − 16 a + 4a − 4a − 16a + 16 < 0
and −3x + z =0 … (iii)
⇒ − 32a + 32 < 0 ⇒ a > 1
On adding Eqs. (i) and (ii), we get y = 0 So,
⇒ x2 + ( 3x ) + 0 ≤ 100
2 3
1 1 1
Now, f = 4 − 3 − p
2
2
2
⇒ 10 x2 ≤ 100; ⇒ x2 ≤ 10
4 3
X = –3, – 2, – 1, 0, 1, 2, 3 = − − p =−1 (1 + p )
8 2
So, number of such 7 points are possible
f(1) = 4(1)³ – 3(1) – p = 1 – p
⇒ 3x + 7 = 2x + 3 or ( 3x + 7 ) = ( 2x + 3)
2
⇒ C, A + B, A – B are integers.
n (n − 1 ) ∴ x =−2, − 4, − 1 / 4 … (i)
f (n) = An2 =
+ Bn + C 2A + ( A + B)n + C
2 But log exists only when, 6x2 + 23x + 21 > 0 ,
( )
∴ 2− y + 2y −1 − 1 = 2y −1 + 1
Sol 8: (B) Let y = x intersect the curve y = kex at exactly
one point when k ≤ 0.
⇒ 2− y = 2 ⇒ y = −1 ∈ ( − ∞ ,0 … (i)
Sol 9: (A) Let f (=
x ) kex − x
Case II when y ∈ ( 0, 1
f ' ( x ) =kex − 1 =0 ; ⇒ x =−ln k
( )
∴ 2y + 2y −1 − 1 = 2y −1 + 1
f '' ( x ) =
kex ; ∴ f " ( x ) 1>0
=
⇒ 2 =2 ⇒ y =1 ∈ ( 0, 1
y
… (ii) x = −1n k
Hence, f(–ln k ) = 1 + ln k
Case III when y ∈ (1, ∞ )
For one root of given equation
M a them a ti cs | 2.75
1 2 2
(∴ α is root of x − 6x − 2 = 0 ⇒ α − 2 = 6α )
1 + ln k = 0 ; ⇒ k =
e
2 2
( Also, β is root of x − 6x − 2 = 0 ⇒β − 2 = 6β )
Sol 10: (A) For two distinct roots, 1 + ln k < 0 (k > 0)
1 1
a10 − 2a10
=
α
8
( 6α=
) − β8 ( 6β ) (
= 3
9
6 α −β
9
)
ln k < –1 , k < ; Hence, k ∈ 0,
e e
2a9 (
2 α 9 − β9 ) (
9 α 9 − β9 )
Sol 11: (C) Given f ( x ) = 4x2 + 3x3 + 2x + 1
α2 + β2
Sol 15: (B) Sum of roots = and product = 1
( )
f ' ( x ) =2 6x2 + 3x + 1 ; D =9 − 24 < 0 αβ
Given, α + β = −p and α3 + β3 = q
Hence, f(x) = 0 has only one real root.
1 3 4
f − = 1 −1 + − > 0
(
⇒ ( α + β ) α2 − αβ + β2 = q )
2 4 8
−q
∴ α2 + β2 − αβ = .…(i)
3 6 27 108 p
f − =1 − + −
and ( α + β ) = p2
2
4 4 16 64
(p − 2q) + 1 =0
1/2 t 3/ 4
f ( x ) dx < ∫ f ( x ) dx < ∫ f ( x ) dx
3
Sol 12: (A) ∫ ∴ Required equation x 2
−
0 0 0
(p + q)
3
∫ f ( x ) dx = ∫ (1 + 2x + 3x )
2 3
Now, + 4x dx
2
=x + x + x + x ; 3 4 ( ) (
⇒ p3 + q x2 − p3 − 2q x + p3 + q =
0 ) ( )
1/2 3/ 4
15 3 530 Sol 16: (A) Since, roots are real therefore D ≥ 0
⇒ ∫ f ( x ) dx = >
16 4 ∫ f ( x=
) dx 256
<3
0
⇒ 4 ( a + b + c ) − 12 λ ( ab + bc + ca) ≥ 0
0 2
⇒ ( a + b + c ) − 3 λ ( ab + bc + ca) ≥ 0
2
Sol 13: (B) Figure is self explanatory
⇒ a2 + b2 + c2 ≥ ( ab + bc + ca) ( 3λ − 2 )
a2 + b2 + c2
⇒ 3λ − 2 ≤ … (i)
ab + bc + ca
-3 -1 1 3
4 2 2 4 b2 + c2 − a2
Also
= , cos A <1
2bc
⇒ b2 + c2 − a2 < 2bc
a10 − 2a8 α10 − β10 − 2α8 + 2β8
Sol 14: (C) =
2a9 (
2 α9 − β9 ) Similarly, c2 + a2 − b2 < 2c a and a2 + b2 − c2 < 2 a b
=
(
α8 α2 − 2 − β8 β2 − 2)
( ) ⇒ a2 + b2 − c2 < 2 ( ab + bc + ca)
2(α − β )
9 9
a2 + b2 + c2
⇒ <2 … (ii)
ab + bc + ca
2 . 7 6 | Quadratic Equations and Inequalities
∴ From Eqs. (i) and (ii), we get 3 λ − 2 < 2 From Eqs. (i), (ii), (iii) and (iv), we get
⇒λ<
4 a ∈ ( −4, 2 )
3
Note: There is correction in answer a < 2 should be –4
< a < 2.
(b + 2a) − 4a ( c + b + 2a)
2
a Whose discriminant =
b
1 =b + 4a + 4ab − 4ac − 4ab − 8a2 =b2 − 4a2 − 4ac
2 2
⇒ –4 < a < 3 ... (ii) Sol 21: (B) Suppose roots are imaginary then β = α
1 1
Again, D =B2 − 4AC ≥ 0 and = α ⇒ β = not possible
β β
(
⇒ ( −2a) − 4.1 a2 + a − 3 ≥ 0
2
) ⇒ Roots are real ⇒ (p2 − q) (b2 − ac) ≥ 0
∞ ∞ n n
1 1 1 1
∑ α + β = ∑ 1 − 7 = 7
=
n 0= n 0
(b + 1) + b + 1 + b = 0 ⇒ b =
2
⇒ 3i − 3i,0
(b − 1) b − 1
2
2017-18 100 &
op kers
Class 11 T
By E ran culty
-JE Fa r
IIT enior emie .
S fP r es
o titut
Ins
MATHEMATICS
FOR JEE MAIN & ADVANCED
SECOND
EDITION
Exhaustive Theory
(Now Revised)
Formula Sheet
9000+ Problems
based on latest JEE pattern
1. SEQUENCE
1.1 Introduction
A sequence can be defined as an ordered collection of things (usually numbers) or a set of numbers arranged one
after another. Sometimes, sequence is also referred as progression. The numbers a1, a2 ,a3 .....an are known as terms
or elements of the sequence. The subscript is the set of positive integers 1, 2, 3..... that indicates the position of the
term in the sequence. Tn is used to denote the nth term.
Some examples of a sequence are as follows:
0, 7, 26......................., 1, 4, 7, 10......................., 2, 4, 6, 8…………………..
Sequence
3, 5, 7, 9, .....
1.3 Rule
A sequence usually has a rule, on the basis of which the terms in the sequence are built up. With the help of this
rule, we can find any term involved in the sequence. For example, the sequence {3, 5, 7, 9} starts at the number 3
and jumps 2 every time.
+2 +2 +2 +2
0 1 2 3 4 5 6 7 8 9 9
Figure 3.2
As a Formula:
Saying ‘start at the number 3 and jump 2 every time’ is fine, but it does not help to calculate the 10th term or 100th
term or nth term. Hence, we want a formula for the sequence with “n” in it (where n is any term number). What
would the rule for {3, 5, 7, 9.......} be? First, we can see the sequence goes up 2 every time; hence, we can guess that
the rule will be something like ‘2 times n’ (where ‘n’ is the term number). Let us test it out.
n Test Rule Term
1 2n = 2 × 1 = 2 3
2 2n = 2 × 2 = 4 5
3 2n = 2 × 3 = 6 7
That nearly worked! But it is less by 1 every time. Let us try changing it to 2n+1.
n Test Rule Term
1 2n + 1 = 2 × 1 + 1 = 3 3
2 2n + 1 = 2 × 2 + 1 = 5 5
3 2n + 1 = 2 × 3 + 1 = 7 7
That Works: Therefore, instead of saying ‘starts at the number 3 and jumps 2 every time,’ we write the expression
2n + 1. We can now calculate, e.g. the 100th term as 2 × 100 + 1 = 201.
1.4 Notation
The notation Tn is used to represent the general term of the sequence. Here, the position of the term in the
sequence is represented by n. To mention for the ‘5’th term, just write T5.
Thus, the rule for {3, 5, 7, 9…} can be written as the following equation: Tn = 2n + 1.
To calculate the 10th term, we can write T10 = 2n + 1 = 2 × 10 + 1 = 21
Illustration 1: Find out the first 4 terms of the sequence, {Tn} = {–1/n}n. (JEE MAIN)
Sol: By substituting n = 1, 2, 3 and 4 in{Tn} = {–1/n}n, we will get the first 4 terms of given sequence.
T1 = (–1/1)1 = –1
T2 = (–1/2)2 = 1/4
T3 = (–1/3)3 = –1/27
T4 = (–1/4)4 = 1/256
⇒ {Tn} = {–1, ¼,–1/27, 1/256 ...}
M a them a tics | 3.3
Illustration 2: Write the sequence whose nth term is (i) 2n and (ii) log(nx). (JEE MAIN)
2. SERIES
Series is something that we get from a given sequence by adding all the terms. If we have a sequence as
T1, T2, …. ,Tn, then the series that we get from this sequence is T1 + T2 +….+Tn. Sn is used to represent the sum of n
terms. Hence, Sn = T1 + T2 +…. +Tn
n=1 r =1
5
i+1 1+1 2+1 3+1 4 +1 5+1
(iii) ∑ 2i + 4 = 2 × 1 + 4 + 2 × 2 + 4 + 2 × 3 + 4 + 2 × 4 + 4 + 2 × 5 + 4
i=l
Properties of Σ (Sigma)
k k k
(i) ∑ a = a + a + a… (k times) = ka, where a is a constant. (ii) ∑ ai = a∑ i , where a is a constant.
i=l =i l =i l
n n n in jn jn in
(iii) ∑ (ar ± br =) ∑ ar ± ∑ br (iv) ∑ ∑ aia j = ∑ ∑ aia j
=r l =r l =r l =i i0=j j0 =j j0=i i0
3.2 Pi Notation
The symbol ∏ denotes the product of similar terms. For example:
6 k
(i) ∏ n = 1 × 2 × 3 × 4 × 5 × 6 (ii) ∏ nm = 1m × 2m × 3m × 4m × ............. × km
n=1 n=1
k
(iii) ∏ n = 1 × 2 × 3 × ........ × k = k!
n=1
4. ARITHMETIC PROGRESSION
The sequence in which the successive terms maintain a constant difference is known as an arithmetic progression
(AP). Consider the following sequences:
a, a + d, a + 2d, a + 3d
T1, T2, T3, T4
3 . 4 | Sequences and Series
PLANCESS CONCEPTS
• If the mth term is n and the nth term is m, then the (m + n)th term is 0.
• If m times the mth term is equal to n times the nth term, then the (m + n)th term is 0.
Vaibhav Krishnan (JEE 2009, AIR 54)
Illustration 3: If the 5th term of an AP is 17 and its 7th term is 15, then find the 22th term. (JEE MAIN)
Illustration 4: If 11 times the 11th term of an AP is equal to 9 times the 9th term, then find the 20th term. (JEE MAIN)
Illustration 5: Check whether the sequences given below are AP or not. (JEE MAIN)
(i) Tn = n2 (ii) Tn = an + b
Sol: By taking the difference of two consecutive terms, we can check whether the sequences are in AP or not.
(i) Tn = n2; Tn–1 = (n – 1)2
M a them a tics | 3.5
3 1 1
Illustration 6: The 2nd, 31st and the last term of an AP are given as 7 , and −6 , respectively. Find the first term
4 2 2
and the number of terms. (JEE MAIN)
Sol: Using Tn = a + (n – 1) d, we can get the first term and common difference. Suppose a be the first term and d
be the common difference of the AP.
3 31
Given, T2 = 7 ⇒ a+ d = .... (i)
4 4
1 1
T31 = ⇒ a + 30d = .... (ii)
2 2
1 31 29 −1
Subtracting (i) from (ii), we get 29d = – =– ⇒ d=
2 4 4 4
1 31 31 1 32
Putting the value of d in (i), we get a – = ⇒ a= + = = 8
4 4 4 4 4
13
Suppose the number of terms be n, so that Tn = –
2
13 1 13
i.e. a + (n – 1) d = – ⇒ 8 + (n – 1) – = –
2 4 2
⇒ 32 – n + 1 = – 26 ⇒ n = 59
Hence, the first term = 8 and the number of terms = 59.
Illustration 7: Prove that the square roots of three unequal prime numbers cannot be three terms of an AP.
(JEE ADVANCED)
Sol: Here by Considering p, q, r to be the lth, µth and vth terms of an AP and solving them using Tn = a + (n – 1)
d, we prove the problem.
If possible let p , q, r be the three terms of an AP. a, a +d, a + 2d........., where p ≠ q ≠ r and they are prime numbers.
Let them be the l , µ and vth terms, respectively.
th th
∴ p = a + (λ – 1) d
q = a + (µ – 1) d
r = a + (ν – 1) d
∴ p – r = (λ – µ) d
Also, q – r = (µ – v) d
∴
p– q
=
λ–µ
or
( p – q )( q+ r ) = λ–µ
q– r µ−v
( q – r )( q+ r) µ− v
λ–µ λ–µ
or pq + pr – q – qr = (q – r) or pq + pr – qr =
q+ (q − r) = rational number
µ–v µ−v
3 . 6 | Sequences and Series
Since p, q, r are unequal primes, pq, pr and qr are unequal pure irrational numbers. Thus, LHS is irrational,
but irrational ≠ rational.
Hence, the problem is proved.
Illustration 8: If x, y and z are real numbers satisfying the equation 25 (9x2 + y2) + 9z2 – 15 (5xy + yz + 3zx) = 0, then
prove that x, y and z are in AP. (JEE ADVANCED)
Sol: By solving the equation 25 (9x2 + y2) + 9z2 – 15 (5xy + yz + 3zx) = 0, we can prove that x, y and z are in AP.
We have
(15x)2 + (5y)2 + (3z)2 – (15x) (5y) – (5y) (3z) – (3z) (15x) = 0
⇒ (15x – 5y)2 + (5y – 3z)2 + (3z – 15x)2 =0
⇒ 15x – 5y = 0, 5y – 3z = 0, 3z – 15x = 0
x y y
⇒ 15x = 5y = 3z ⇒ = = (= k say)
1 3 5
∴ x = k, y = 3k, z = 5k
Thus, x, y and z are in AP.
Illustration 9: Let a1, a2, a3, ...., an be in AP, where a1 = 0 and the common difference ≠ 0. Show that
a3 a4 a5 an 1 1 1 a a
+ + + ........ – a2 +
a
+ ....... + = n–1 + 2 (JEE ADVANCED)
a2 a3 a4 an–1 2 a3 an–2 a2 an–1
a3 – a2 a4 – a2 a5 – a2 an–1 – a2 an
LHS
= + + + ... + +
a2 a3 a4 an–2 an–1
a2 a3 a4 an–2 an
= + + + ...... + +
a2 a3 a4 an–2 an–1
an a1 + (n – 1)d n–1
= (n–3) + = (n – 3) + (n – 3) +
= {a1 =
0}
an–1 a1 + (n – 2)d n–2
a1 + (n – 2)d a2 1 1 n–1
RHS = + = (n – 2) + = (n – 3) + 1 + = (n – 3) +
a2 a1 + (n – 2)d n–2 n–2 n–2
∴LHS = RHS
PLANCESS CONCEPTS
A sequence obtained by multiplication or division of corresponding terms of two APs may not be in AP
For example, let the first AP be 2, 4, 6, 8,................ and the second AP be 1, 2, 3, 4, 5.......
Multiplying these two, we get 2,8,18,32, ........., which is clearly not an AP
Vaibhav Gupta (JEE 2009, AIR 54)
M a them a tics | 3.7
4.2 Series of an AP
Series of an AP can be obtained as
Sn = a + (a + d) + (a + 2d) ...... [a+(n – 1)d]
Sn = [a + (n – 1)d] + [a+(n – 2)d] ...... + a (writing in the reverse order)
∴ 2Sn = n(2a + (n –1) d)
n n
∴ Sum to n terms, Sn =(2a + (n – 1)d) =( T1 + Tn )
2 2
Illustration 10: Find the sum of the first 19 terms of an AP when a4 + a8 + a12 + a16 = 224. (JEE MAIN)
19
Sol: We need to find out the sum of the first 19 terms of an AP, i.e. (2a + 18d), and we can represent the given
2
equation as (a + 3d) + (a + 7d) + (a + 11d) + (a + 15d) = 224.
Given (a + 3d) + (a + 7d) + (a + 11d) + (a +15d) = 224
⇒ 4a + 36d = 224 ⇒ a + 9d = 56
19 19
Sum of the first 19 terms ⇒ S = (2a + 18d) = × 2 × 56 =1064
2 2
7n + 1
Illustration 11: The sum of n terms of two arithmetic progressions is in the ratio of . Find ratio of the 11th
4n + 27
terms? (JEE MAIN)
n
Sol: Since we know the sum of n terms, i.e. Sn = 2a + (n – 1 ) d , we can write the equation as
2
n
(2a1 + (n – 1)d1 )
7n + 1
= 2 . Hence, by putting n = 11 in this equation, we can obtain the ratio of the 11th terms.
4n + 27 n
(2a + (n – 1)d2 )
2 2
n−1
a1 + d
7n + 1 2 1
=
4n + 27 n−1
a1 + d2
2
a1 + 10d1 n–1
We want the ratio of . Hence, = 10 ⇒ n = 21
a2 + 10d2 2
a1 + 10d2 148
⇒ =
a2 + 10d2 111
Illustration 12: In an AP of n terms, prove that the sum of the kth term from the beginning and the kth term from
the end is independent of k and equal to the sum of the first and last terms. (JEE MAIN)
Sol: Using the formula Tk = a + (k – 1)d and Tn−k +1= [a + (n – k)d] , we can obtain the kth term from the beginning
and end, respectively, and after that by adding these values we can prove the given problem.
Suppose a be the first term and d be the common difference of the AP.
The kth term from the end of the given AP is the (n – k + 1)th term from the beginning.
Illustration 13: If a1, a2, a3 ,..., an is an AP of non-zero terms , then prove that (JEE ADVANCED)
1 1 1 n–1
+ + ..... + =
a1a2 a2a3 an–1an a1an
Sol: By considering a as the first term and d as the common difference, we can write an as a + (n – 1) d, where n =
1, 2, 3,… n.
1 1 1 1 1 1
+ + ..... + = + + ......
a1a2 a2a3 an–1an a(a + d) (a + d)(a + 2d) [a + (n – 2)d][a + (n − 1)d]
1 1 1 1 1 1 1 1 1
= – + – + ... + –
d a a + d d a + d a + 2d d a + (n – 2)d a + (n – 1)d
PLANCESS CONCEPTS
Facts:
• If each term of an AP is increased, decreased, multiplied or divided by the same non-zero number, the
resulting sequence is also an AP.
• The sum of the two terms of an AP equidistant from the beginning and end is constant and is equal
to the sum of the first and last terms.
a1 + an = a2 + an – 1 = a3 + an – 2 = ....
Vaibhav Krishnan (JEE 2009, AIR 54
Illustration 14: Split 69 into three parts such that they are in AP and the product of the two smaller parts is 483.
(JEE MAIN)
Sol: By considering the three parts as a – d, a, and a + d and using the given conditions, we can solve the given
problem.
Sum of the three terms = 69 ⇒ (a – d) + a + (a + d) = 69
⇒ 3a = 69 ⇒ a = 23 ..... (i)
M a them a tics | 3.9
Illustration 15: Divide 32 into four parts that are in AP such that the ratio of the product of extremes to the product
of mean is 7: 15. (JEE MAIN)
Sol: We can consider the four parts as (a – 3d), (a – d), (a + d) and (a + 3d).
Sum of the four parts = 32
⇒ (a – 3d) + (a – d) + (a + d) + (a + 3d) = 32 ⇒ 4a = 32 ⇒ a = 8
⇒ 128 d2 = 512 ⇒ d2 = 4 ⇒ d = ± 2
b +c c+a a+b
Adding 1 to each term, find that + 1 , + 1 , + 1 are in AP
a b c
a+b+c c+a+b a+b+c
i.e. , , are in AP
a b c
1 1 1
Dividing each term by a + b + c, we find that , , are in AP
a b c
1 1 1 1 1 1
Illustration 17: If a + ,b + ,c + are in AP, then prove that a, b, c are in AP. (JEE ADVANCED)
b c a c a b
abc
Sol: By adding 1 and then multiplying by to each term, we will get the result.
ab + bc + ac
1 1 1 1 1 1 b + c a+ c b + a
a + , b + , c + are in AP ⇒ a , b , c are in AP
b c a c a b bc ac ab
ab + ac ab + bc bc + ac
Adding 1, we find that + 1, + 1, + 1 are in AP
bc ac ab
ab + ac + bc ab + bc + ac bc + ac + ab
⇒ , , are in AP
bc ac ab
abc
Multiplying by to all the terms, we find that a, b, c are in AP
ab + bc + ac
3 . 1 0 | Sequences and Series
Note: The sum of the n AMs inserted between a and b is equal to n times A.M. between them.
Illustration 18: Insert 20 AMs between the numbers 4 and 67. (JEE MAIN)
b–a
Sol: Given, a = 4 and b = 67; therefore by using the formula d= , we can solve it.
n+1
67 – 4
d= =3
20 + 1
A1 = a + d ⇒ A1 = 7
A2 = a + 2d ⇒ A2 = 10
A3 = a + 3d ⇒ A3 = 13
A20 = a + 20 d ⇒ A20 = 63
Thus, between 4 and 67, 20 AMs are 7, 10, 13, 16, ...., 63.
an + bn
Illustration 19: If is the A.M. between a and b, find the value of n. (JEE ADVANCED)
an–1 + bn–1
a+b an + bn
Sol: Since the A.M. between a and b = , we can obtain the value of n by equating this to .
2 an–1 + bn–1
an + bn a+b
⇒ = [Given] ⇒ 2an + 2bn = an + abn–1 + an–1 b + bn
a n–1
+b n–1 2
a
n–1
a
0 a 0
⇒ a n–1
=b n–1
[ a ≠ b] ⇒ = 1= = 1
b b b
⇒ n – 1 = 0 ⇒ n = 1
M a them a ti cs | 3.11
Illustration 20: Between 1 and 31, m arithmetic means are inserted in such a way that the ratio of the 7th and
(m – 1)th means is 5: 9. Calculate the value of m. (JEE MAIN)
Sol: AMs inserted between 1 and 31 are in AP. Thus, by considering d to be the common difference of AP and
obtaining the 7th and (m – 1)th means we can solve the problem.
Suppose A1, A2, A3, A4,...... Am be the m AMs between 1 and 31.
Thus, 1, A1, A2,....Am, 31 are in AP
The total number of terms is m + 2 and Tm+2 = 31
30
1 + (m + 2 – 1) d = 31 ⇒(m + 1) d = 30 ⇒ d =
m+1
30 m + 1 + 210 m + 211
A7 = T8 = a + 7d = 1 +=
7× =
m+1 m+1 m+1
30 m + 1 + 30m – 30 31m – 29
Am–1 = Tm = 1 + (m –1) d = 1 + (m – 1) × = =
m+1 m+1 m+1
A7 (m + 211) / (m + 1) m + 211 5
= = = [Given]
Am–1 (31m − 29) / (m + 1) 31m – 29 9
m + 211 5
⇒ = ⇒ 9m +1899 = 155m – 145
31m – 29 9
2044
⇒ 146m = 2044 ⇒ m= = 14 ;Thus, m = 14
146
Illustration 21: Gate receipts at the show of “Baghbaan” amounted to Rs 9500 on the first night and showed a
drop of Rs 250 every succeeding night. If the operational expenses of the show are Rs 2000 a day, find out on which
night the show ceases to be profitable? (JEE MAIN)
Sol: Here, a = 9500 and d = –250. The show ceases to be profitable on the night when the receipts are just Rs 2000.
Thus, by considering that it will happen at nth night and using Tn = a + (n – 1) d, we can solve this problem.
We have the cost of gate receipt on the first night (a) = 9500
Common difference (d) = –250
Suppose, it happens on the nth night, then
2000 = 9500 + (n – 1) (–250) ⇒ 2000 – 9500 = – 250n + 250
7750
⇒ –7500 – 250 = – 250n ⇒ –7750 = – 250n ⇒ n= = 31
250
PLANCESS CONCEPTS
(a) If the sum of n terms Sn is given, then the general term Tn = Sn – Sn–1, where Sn–1 is sum of (n – 1) terms of AP.
(b) In a series, if Sn is a quadratic function of n or Tn is a linear function of n, the series is an AP.
(i) If Tn = an + b, the series so formed is an AP and its common difference is a.
(ii) If Sn = an2 + bn + c, the series so formed is an AP and its common difference is 2a.
3 . 1 2 | Sequences and Series
PLANCESS CONCEPTS
(c) If in a finite AP, the number of terms is odd, then its middle term is the A.M. between the first term and
last term and its sum is equal to the product of the middle term and number of terms.
(d) It is found that the sum of infinite terms of an AP is ∞, if d > 0 and –∞, if d < 0.
(e) If for an AP, the pth term is q and the qth term is p, then the mth term is = p + q – m.
(f) If for an AP, the sum of p terms is q and sum of q terms is p, then the sum of (p + q) terms is – (p + q).
(g) If for an AP, the sum of p terms is equal to the sum of q terms, then the sum of (p +q) terms is zero.
Sn fn Tn f(2n – 1)
(h) If for different APs, = , then = .
S'n φn T'n φ(2n – 1)
n+1
A +B
TnAn + B S 2
(i) If for two APs, = , then we find that n = .
T'n Cn + D S'n n+1
C + D
2
Shrikant Nagori (JEE 2009, AIR 54)
An Important Property of AP: A sequence is said to be an AP if the sum of its n terms is of the form An2 + Bn,
where A and B are constants. Thus, the common difference of the AP is 2A.
Proof: Suppose, a and d be the first term and common difference of AP, respectively, and Sn be the sum of n terms.
n
Sn
= [2a + (n – 1)d]
2
n2 n d 2 d
⇒ Sn =
an + d – d =
n + a – n
2 2 2
2
d d
⇒ Sn = An2 + Bn, where A = and B = a –
2 2
5. GEOMETRIC PROGRESSION
A sequence of non-zero numbers is called a geometric progression (GP) if the ratio of successive terms is constant.
In general, G.P. is written in the following form: a, ar, ar2, ....., arn–1,....
where a is the first term and r is the common ratio
Illustration 22: The 5th, 8th and 11th terms of a G.P. are given as p, q and, s respectively. Prove that q2 = ps.
(JEE MAIN)
Sol: By using Tn = arn – 1 and solving it, we can prove the problem.
Given, T5 = p, T8 = q, T11 = s ....(i)
Now, T5 = ar5–1 = ar4 ⇒ ar4 = p .... (ii) [Using (i)]
T8 = ar 8–1
= ar ⇒
7
ar =q
7
... (iii) [Using (i)]
T11 = ar11–1 = ar10 ⇒ ar10 = s ..... (iv)
On squaring (iii), we get
q2 = a2 r14 = a⋅a.r4⋅r10 = (ar4) (ar10)
⇒ q2 = ps [Using (ii) and (iv)] proved.
5.2 Series of GP
Let us suppose Sn = a + ar + ar2 + ...... +arn–1 ... (i)
Multiplying ‘r’ on both the sides of (i) and shifting the RHS terms by one place, we get
Snr = 0 + ar + ar2 + ...... +arn ... (ii)
By subtracting (ii) from (i), we get
Sn (1 – r) = a – arn = a (1 – rn)
a(1 – rn )
Sn = , where r ≠ 1
1–r
a(rn – 1) Tn+1 – a
by ⇒ Sn =
Thus, the sum of the first n terms of a G.P. is given=
r – 1 r – 1
And Sn = na, when r = 1
Note: If r = 1, then the sequence is of both AP and GP, and its sum is equal to na, i.e. Sn = na.
If |r| < 1, the nth term of G.P. converges to zero and the sum becomes finite.
a(rn – 1)
The sum to infinite terms of G.P. = lim Sn = lim
n→∞ n→∞ r – 1
a
As |r| < 1 rn → 0 as n → ∞ ∴ S∞ =
1–r
Note: If a and b are two number of opposite signs, then the G.M. between them does not exist.
To Insert ‘n’ GMs Between a and b: If a and b are two positive numbers and we have to insert n GMs, G1, G2, ………,
Gn, between the two numbers ‘a’ and ‘b’ then a, G1, G2, ……….., Gn, b will be in GP. The series consists of (n + 2) terms
and the last term is b and the first term is a.
1
b n+1
Thus, b = arn + 2 – 1 ⇒ b = arn + 1 ⇒ r =
a
⇒ G1 = ar, G2 = ar …….Gn = ar or Gn = ayn+1 . byn+1 = (ab)yn+1
2 n
Note: The product of n GMs inserted between ‘a’ and ‘b’ is equal to the nth power of the single G.M. between ‘a’
and ‘b,’ i.e.
n
∏ Gr = (G)n, where G = ab (GM between a and b)
r =1
( ) ≥0⇒a–2 a+b
2
Now, we can write a– b ab + b ≥ 0 ⇒ a + b ≥ 2 ab ⇒ ≥ ab ⇒ A.M. ≥ GM
2
Note: (i) Equality for AM, G.M. (i.e. A.M. = GM) exists when a = b.
(ii) Since A.M. ≥ GM; (AM)min = GM; (GM)max = AM
x y z
Illustration 23: If x, y and z have the same sign, then prove that + + ≥ 3 . (JEE ADVANCED)
y z x
x y z
Sol: As we know that A.M. ≥ G.M., therefore by obtaining A.M. and G.M. of , and we can prove the problem.
y z x
x y z
Let = x1; = x2; = x3
y z x
x y z
+ +
x1 + x2 + x3 y z x
∴ ≥ (x1 x2 x3 )1/3 ⇒ ≥1
3 3
Hence proved.
an+1 + bn+1
Illustration 24: Calculate the values of n so that may be the G.M. between a and b.(JEE ADVANCED)
an + bn
an+1 + bn+1
Sol: We know that the G.M. between a and b = ab , but here G.M. between a and b is .
an + bn
an+1 + bn+1
⇒ = ab ⇒ an + 1 + bn + 1 = (an + bn) (ab)1/2
an + bn
1 1 1 1 1 1 1 1
n+ n+ n+ n+
⇒ an + 1 + bn + 1 = a 2 ⋅ b 2 + a2 ⋅ b 2 ⇒ an+1 – a 2 ⋅ b 2 = a2 ⋅ b 2 – bn+1
n+
1 1 1
n+
1 1 1
n+
1
n+
1
⇒ a 2 a2 – b 2 = b 2 a2 – b 2 ⇒ a 2 = b 2
1
n+ 0
a 2 a 1 1
⇒ = 1 = ⇒ n+ = 0 ⇒ n= –
b b 2 2
M a them a ti cs | 3.15
Illustration 25: Find the sum to n terms for the series 9 + 99 + 999 …… n. (JEE ADVANCED)
Sol: The given series can be written as S = (10 – 1)+ (102 – 1) + (103 – 1) ….. +(10n − 1).
a(1 – rn )
Thus, by using Sn = , we can find out the required sum.
1–r
10(1 – 10n )
∴ S = (10 + 102 + 103 +….+ 10n) – n; S = –n
1 – 10
Illustration 26: If a1, a2 and a3 are in G.P. with a common ratio r (r > 0 and a > 0), then values of r for which inequality
9a1 + 5a3 > 14a2 hold good are? (JEE ADVANCED)
a
Sol: Since a1 = a = a, a3 = ar, by substituting these values to the given inequality we will get the result.
r 2
a 9a
a1 = , a = a, a3 = ar Now, + 5ar > 14a
r 2 r
⇒ 5ar2 – 14ar + 9a > 0 ⇒ 5r2 – 14r + 9 > 0
⇒ 5r2 – 5r – 9r + 9 > 0 ⇒ 5r(r – 1) – 9 (r – 1) > 0
5
⇒ (5r – 9) (r – 1) > 0 ⇒ r ∈ R − 1,
9
PLANCESS CONCEPTS
Illustration 27: On a certain date, the height of a plant is 1.6 m. If the height increases by 5 cm in the following year and
if the increase in each year is half of that in the preceding years, show that the height of the plant will never be 1.7 m.
(JEE MAIN)
Sol: Here, the sum of the increases in the height of the plant in the first, second, third, … year is equal to (1.7 – 1.6) m
= 0.1 m = 10 cm.
5 5
According to the question, increases in the height of the plant in the first, second, third, … year are 5, , , … cm,
respectively. 2 4
Let it reach the height of 1.7 m (i.e. increases [1.7 – 1.6] m = 0.1 m = 10 cm).
5 5
Therefore, the sum of 5, , , …to n terms = 10
2 4
1
5 1 – n
2 a(1 – rn )
⇒ = 10 [ a = 5, r = ½, Sn = ]
1 1–r
1–
2
1 1 1
⇒ 10 1 – = 10 ⇒ 1 – =1⇒ = 0 , which does not hold for any n. Thus, the plant will never reach the
n
n
2 2 2n
height of 1.7 m.
3 . 1 6 | Sequences and Series
Illustration 28: A manufacturer reckons that the value of a machine (price = Rs 15,625) will depreciate each year
by 20%. Calculate the estimated value at the end of 5 years. (JEE MAIN)
Sol: Here the value of the machine after 5 years= ar5, where a= 15,625. We will obtain the value of r using the given
condition.
The present value of the machine = Rs 15,625
80
The value of the machine in the next year = Rs 15,625 ×
100
80 80
The value of the machine after 2 years = Rs 15,625 × ×
100 100
The values of the machine in the present year, after 1 year and after 2 years are
80 80 80
Rs 15,625, Rs 15,625 × and Rs 15,625 × × , respectively
100 100 100
These values form a GP.
80 4
Here, the first term is Rs 15,625 and the common ratio is ,i.e. .
100 5
5
4 15625 × 1024
Thus, thee value of the machine after 5 years = ar = Rs 15,625 × =
5
= 1024 × 5 = Rs 5120
5 625 × 5
5.5 Properties of GP
(a) If each term of a G.P. is multiplied or divided by the same non-zero quantity, then the resulting sequence is
also a GP.
(b) If in a finite GP, the number of terms is odd, then its middle term is the G.M. of the first and last terms.
b c
(c) If a, b and c are in GP, then = ⇒ b2 =ac (which is the condition of GP).
a b
1
(d) The reciprocals of the terms of a given G.P. also give a G.P. with a common ratio of .
r
Proof: Let a1, a2, a3, a4,....., an,...... be the terms of a G.P. with the common ratio r.
an+1
Then, = r for all n ∈ N ... (i)
an
The sequence formed by the reciprocals of the terms of the above G.P. is given by
1 1 1 1
, , ,....., ,.....
a1 a2 a3 an
1 / an+1 an 1
Now, = = [Using (i)]
1 / an an+1 r
Hence, the new sequence is also a G.P. with the common ratio 1/r.
(e) If each term of a G.P. is raised to the same power (say k), then the resulting sequence also forms a G.P. with
the common ratio as rk.
Proof: Let a1, a2, a3, a4 ,...., an.... be the terms of a G.P. with the common ratio r.
an+1
Then, = r for all n ∈ N (i)
an
Let k be a non-zero real number. Consider the sequence. a1k ,ak2 ,ak3 ,.......,ank ,....
M a them a ti cs | 3.17
k
akn+1 an+1
Here,
= = rk for all n ∈ d [Using (i)]
akn a
n
Thus, a1k ,ak2 ,ak3 ,......,ank ,..... is a G.P. with a common ratio rk.
(f) In a GP, the product of the terms equidistant from the beginning and the end is always the same and it is equal
to the product of the first and last terms (only for finite GP).
Proof: Let a1, a2, a3 ,....., an be a finite G.P. with the common ratio r. Then,
kth term from the beginning = ak = a1rk–1
kth term from the end = (n – k + 1)th term from the beginning
an–k +1 = a1rn–k
=
∴ (kth term from the beginning) (kth term from the end)= akan–k+1
= a1rk-1a1rn–k =a12 rn–1 =a1.a1rn–1= a1an for all k = 2,3,....,n –1
Thus, the product of the terms equidistant from the beginning and the end is always the same and is equal to
the product of the first and last terms.
(g) If the terms of a G.P. are chosen at regular intervals, the new sequence so formed also forms a G.P. with the
common ratio as rp , where p is the size of interval.
For example:
2, 4, 8, 16, 32, 64, 128,..... (GP, where r = 2)
4, 16, 64 ..... (also a GP, where r = 4)
(h) If a1, a2, a3 ,...., ar..... is a G.P. of non-zero, non-negative terms, then log a1, log a2,...., log an,..... is an AP and vice
versa.
(i) If T1, T2, T3... and t1, t2, t3 are two GPs, T1t1, T2t2, T3t3.... is also in GP.
Proof: Let the two GPs be T1, T2, ...., Tn,... with the common ratio R
Tn+1
⇒ = R ... (i)
Tn
tn+1
⇒ =r .... (ii)
tn
T tn–1
Multiplying each term of the sequence (i) by the corresponding term of (ii), we get n+1
= Rr
Tn tn
Thus, the resulting sequence is also in G.P. with the common ratio Rr.
( j) The resulting sequence thus formed by dividing the terms of a G.P. by the corresponding terms of another
G.P. is also a GP.
Proof: Let the two GPs be T1,T2,....,Tn,.... with the common ratio R
Tn+1
⇒ = R ... (i)
Tn
tn+1
⇒ = r ...(ii)
tn
Dividing each term of the sequence (i) by the corresponding term of (ii), we get
3 . 1 8 | Sequences and Series
Tn+1
tn+1 R
= [Using (i) and (ii)]
Tn r
tn
R
Thus, the resulting sequence is also in G.P. with the common ratio .
r
Illustration 29: If sum of infinite terms of G.P. is 15 and sum of squares of infinite terms of G.P. is 45, then find GP.
(JEE MAIN)
a
Sol: As the sum of infinite terms S∞ = , therefore by using this formula we can obtain the value of the common
ratio. 1–r
a
= 15
1–r
a2 225(1 – r)(1 – r)
Now, a2, a2r2, a2r4,……; = 45 ∴ = 45
1–r 2 (1 – r)(1 + r)
Illustration 30: If x = 1 + a + a2 + .... ∞, y = 1 + b + b2+.... ∞ and |a| < 1, |b| < 1, then prove that
xy
1 + ab + a2b2 + .... = (JEE MAIN)
x + y –1
a
Sol: By using the formula S∞ = , we can solve problem.
1–r
1
x = 1 + a + a2 + .... to ∞ = ( |a| < 1)
1–a
1 1 x –1
⇒ 1–a= ⇒ a=1– ⇒ a= ... (i)
x x x
1
Also, y = 1 + b + b2 +...... to ∞ = ( |b| < 1)
1–b
1 1 y –1
⇒ 1–b= ⇒ b =1– ⇒ b = ... (ii)
y y y
1
∴ 1 + ab + a2b2 + ..... to ∞ = ( |a| < 1, |b| < 1 ⇒ |ab| < 1)
1 – ab
1 xy xy
= [Using (i) and (ii)] = =
x –1 y –1 xy – xy + x + y – 1 x + y – 1
1– .
x y
Hence proved.
Illustration 31: If S1, S2, S3, ……., SP denote the sum of an infinite G.P. whose first terms are 1, 2, 3, …… , p, respectively
1 1 1 1 p(p + 3)
and whose common ratios are , , ,……., , respectively, show that S1 + S2 + S3 + ……..+ Sp = .
2 3 4 (p + 1) 2
(JEE ADVANCED)
a
Sol: By using S∞ = we can obtain S1, S2, S3, ……., SP and after that by adding them we can prove the given
equation. 1–r
M a them a ti cs | 3.19
1 1
For S1, we have a = 1, r= ∴ S1 = =2
2 1
1–
2
1 2
For S2, we have a = 2, r= ∴ S2 = =3
3 1
1–
3
1 3
For S3, we have a = 3, r= ∴ S3 = =4
4 1
1–
4
1 p
For Sp, we have a = p, r= ∴ Sp = =p+1
p +1 1
1–
p +1
p p p(p + 3)
= [2 + (p + 1)] = [p + 3] =
2 2 2
Hence proved.
dbr(1 – rn–1 )
Sn(1 – r) = ab + − (a + (n – 1)d)brn
1–r
ab dbr
⇒ S∞ = +
1 – r (1 – r)2
3 . 2 0 | Sequences and Series
Illustration 32: If |x| < 1, then find the sum S = 1 + 2x + 3x2 + 4x3 ….. + ∞. (JEE MAIN)
1 1
S(1 – x) = 1+ x + x2 + x3 + ….. +∞; S(1 – x) = ⇒S =
(1 – x) (1 – x)2
Illustration 33: If |x| < 1, then find the sum S = 1 + 3x + 6x2 + 10x3 + ……∞. (JEE ADVANCED)
S (1–x)2 = 1+ x + x2 + …… ∞
1 1
S(1 – x)2 = S=
1–x (1 – x)3
7. MISCELLANEOUS SEQUENCES
3n(n + 1)
⇒ 3Σn2 = n3 + 1 + 3n2 + 3n – 1 – –n
2
3n(n + 1)
⇒ 3Σn2 = n3 + 3n2 + 2n –
2
2n3 + 6n2 + 4n – 3n2 – 3n 2n3 + 3n2 + n
⇒ 3Σn2 = ⇒ 3Σn2 =
2 2
2n3 + 3n2 + n2 + n 2n2 (n + 1) + n(n + 1)
⇒ 3Σn2 = ⇒ 3Σn2 =
2 2
r =1 2
Proof: Consider (x + 1)4 – x4 = 4x3 + 6x2 + 4x + 1
Put x = 1, 2, 3…… n
24 – 14 = 4⋅13 + 6⋅12 + 4⋅1 + 1
34 – 24 = 4⋅23 + 6⋅22 + 4⋅2 + 1
44 – 34 = 4⋅32 + 6⋅22 + 4⋅3 + 1
:
(n + 1)4 – n4 = 4⋅n3 + 6⋅n2 + 4⋅n + 1
Adding all, we get
(n + 1)4 – 14 = 4(13 + 23 + ….. + n3) + 6(12 + 22+ ….+ n2) + 4(1 + 2 + 3….. +n) +n
n(n – 1)(2n + 1) n + 1
= 4Σn3 + 6 + 4 n + n
6 2
On simplification, we get
2
n + 1
Σn = n
3
2
(vi) Sum of the fourth powers of the first n natural numbers (Σn4)
[The result can be proved in the same manner as done for Σn3]
n i j
Illustration 34: Find the value of ∑∑ ∑ (1) . (JEE MAIN)
=i 1 =j 1 =
k 1
n n
n(n + 1) n(n + 1)(2n + 1)
Sol: Using the formula ∑r = 2
and ∑ r2 = 6
, we can solve the problem.
r =1 r =1
n i j
Let S = ∑∑ ∑ (1)
=i 1 =j 1 =
k 1
n i n
i(i + 1) 1 1 n(n + 1)(2n + 1) n(n + 1) n(n + 1)(2n + 4) n(n + 1)(n + 2)
S= ∑∑ ( j)= ∑ 2
==
2
[Σn2 + Σn]
2
6
= +
2
=
12 6
=i 1 =j 1 =i 1
3 . 2 2 | Sequences and Series
Illustration 35: Find the sum of 1.2.3 + 2.3.4 + 3.4.5…… n terms. (JEE MAIN)
n 2 n n
n+1 n(n + 1)(2n + 1) n(n + 1)
Therefore, by using ∑ r3 = n2 2 ∑ r2 = 6
and ∑r = 2
, we can solve the problem.
r =1 r =1 r =1
Tn = n(n + 1) (n + 2) = n (n2 + 3n + 2)
Tn = n3 + 3n2 + 2n
2 n+1
3n (n + 1 ) (2n + 1)
2
Type 2: Using Method of Difference: If T1, T2, T3, T2, T4 T5…. is a sequence whose terms are sometimes in AP and
sometimes in GP, then for such series we first compute their nth term and then compute the sum to n terms using
sigma notation.
Sol: By calculating [Sn + (– Sn)], we will get Tn. After that we will obtain Σ Tn and thus we will get the result.
Σn = 6 + 13 + 22 ……… Tn
– Σn = – 6 – 13 ……… – Tn–1 – Tn
⇒ 0 = 6 + (7 + 9 +11 ….. (Tn – Tn–1)) – Tn
⇒ Tn = 6 + (7 + 9 + 11…. (Tn – Tn–1)) = 6 + (n – 1) (7 + n – 2) = 6 + (n – 1) (n + 5)
⇒ Tn = 6 +n2 + 4n – 5 = n2 + 4n + 1
n(n + 1)(n + 1)
Σn = Σn2 + 4Σn + n = + 2n (n +1) + n
6
1 1 1
Illustration 37: Find S = 1 + 1 + + 1 + + + ...... n terms. (JEE ADVANCED)
3 3 32
1 1 1
Sol: Given, Tn = 1 + + + ..... + ; therefore by obtaining ΣTn, we will get the result.
3 32 3n−1
1 1 1
S = 1 + 1 + + 1 + + .......
3 3 9
1
31 – n
1 1 1 3
Tn = 1 + + + ..... + =
3 32
3 −1
n 2
3n 3 1 3n 3 1 1 1 3 3 1
ΣTn = – =
Σ – + ...... = n – 1 –
2 2 3n 2 23 3 2 n
3 2 2 3n
Type 3: Splitting the nth term as a difference of two: Here, S is a series in which each term is composed of the
reciprocal of the product of r factors in an AP.
M a them a ti cs | 3.23
1 1 1
Illustration 38: Find the sum of n terms of the series + + + ..... (JEE ADVANCED)
1 ⋅ 2 ⋅ 3 ⋅ 4 2 ⋅ 3 ⋅ 4 ⋅5 3 ⋅ 4 ⋅5 ⋅ 6
1
Sol: Here nth term of the series will be Tn = .
n(n + 1)(n + 2)(n + 3)
1
By considering Sn = c – λ, where λ = , we will get the result.
3(n + 1)(n + 2)(n + 3)
1
First calculate the nth term, Tn =
n(n + 1)(n + 2)(n + 3)
1 1
Remark ⇒ If we want to calculate S∞, then n → ∞, → 0 ⇒ S∞ =
(n + 1)(n + 2)(n + 3) 18
Note: The above method is applicable only when the series looks like as follows:
1 1 1
+ + + ....
a(a + d)(a + 2d) (a + d)(a + 2d)(a+ 3d) (a + 2d)(a + 3d)(a + 4d)
Type 4: Vn Method: This is method of resolving the nth term into partial fraction and summation by telescopic
cancellation. First, find the nth term of the series and try to create a denominator part in the numerator by using
partial fraction whenever the series is in the form of fraction or Tn is in the form of fraction.
For example, let us suppose a summation where the nth term is like the following:
2
Tn =
2
n –1
1 1
Using the partial fraction, we can write the nth term as Tn = –
n–1 n+1
Now, when we find the summation, there will be telescopic cancellation and thus we will get the sum of the given
series.
Type 5: Dealing with Sn4: This technique is valid for Σn2 and Σn3. In this type, there is a series in which each term
is composed of factors in an AP, i.e. factors of several terms being in AP.
3 . 2 4 | Sequences and Series
1 1
Tn = [(n + 1)(n + 2)(n + 3)(n + 4)[n– (n – 1)]] = (n(n + 1)(n + 2)(n+ 3)(n + 4) – (n – 1)(n + 1)(n + 2)(n + 3)(n + 4))
5 5
1
T1 = (1 ⋅ 2 ⋅ 3 ⋅ 4 ⋅ 5 − 0)
5
1
T2 = (2 ⋅ 3 ⋅ 4 ⋅ 5 ⋅ 6 – 1 ⋅ 2 ⋅ 3 ⋅ 4 ⋅ 5)
5
1
T3 = (3 ⋅ 4 ⋅ 5 ⋅ 6 ⋅ 7 – 2 ⋅ 3 ⋅ 4 ⋅ 5 ⋅ 6)
5
1
Tn = = (n(n + 1)(n + 2)(n+ 3)(n + 4) – (n – 1)(n + 1)(n + 2)(n + 3)(n + 4))
5
1
Sn =
5
(n(n + 1)(n + 2)(n + 3)(n + 4))
Note: This method will be applicable only when the series looks like the following:
a(a + d) ( a+ 2d) + (a + d) (a + 2d) (a + 3d) + (a + 2d) (a + 3d) (a + 4d)+….+ up to n term, where a = first term and
d = common difference
PLANCESS CONCEPTS
1 1 1 1 n
• + + + .... + =
1.2 2.3 3.4 n(n + 1) n + 1
1 1 1 1 1
+ + .... + =–
1.2.3 2.3.4 n(n + 1)(n + 2) 4 2(n + 1)(n + 2)
1 1 1 1 1 1
+ + ..... + = –
a1a2 ....ar a2a3 ....ar +1 anan+1 ....an+r –1 (r – 1)(a2 – a1 ) a1a2 ....ar –1 an+1an+ 2 ....an+r –1
1
• a1a2….ar + a2a3….ar+1 +…..+ anan+1….an+r–1 = [ a a ….a – a0a1a2….an]
(r + 1)(a2 – a1 ) n n+1 n+r
8. HARMONIC PROGRESSION
A sequence will be in harmonic progression (HP) if the reciprocals of its terms are in AP, e.g. if a1, a2, a3, ……….
1 1 1
are in HP, then , , ……. are in AP. For every AP, there will be a corresponding HP, and the standard H.P. is
a1 a2 a3
1 1 1 1
, , + ............ + .
a a + d a + 2d a + (n – 1)d
Note:
(i) 0 cannot be a term of H.P. because ∞ is not a term of AP, but ∞ can be a term of HP.
(ii) There is no general formula for finding the sum to n terms of HP.
1 1 1
(iii) If a, b and c are in HP, then , , are in AP.
a b c
2 1 1 2ac
∴ = + ⇒ b=
b a c a+c
⇒ a, b and c are in HP
1 1 3
Illustration 39: If the 3rd, 6th and last terms of a H.P. are , , , then find the number of terms. (JEE MAIN)
3 5 203
1
Sol: If nth term of a H.P. is , then the nth term of the corresponding AP will be a. Thus, by using
a
Tn = a + (n – 1) d, we will get the result.
Let a be the first term and d be the common difference of the corresponding AP.
1
If the 3rd term of H.P. = ; then the 3rd term of the corresponding AP = 3
3
⇒ a + 2d = 3 ….. (i)
1
If the 6th term of H.P. = ; then the 6th term of the corresponding AP = 5
5
⇒ a + 5d = 5 ….. (ii)
2 5
From (i) and (ii), we get d = ⇒ a =
3 3
3 203
If the nth term of H.P. = ; then nth term of AP =
203 3
203 5 2 203
a + (n – 1) d = ; + (n – 1) =
3 3 3 3
5 + 2n – 2 = 203; n = 100
Illustration 40: If a1, a2, ……….an are in H.P. then the expression a1a2 + a2a3 + …….+ an – 1an is equal to.
(JEE ADVANCED)
1 1 1 1 1 1 1 1 1
Sol: As , ,...... are in AP, taking = – = – ... – = d, we can obtain the values of a1a2, a2a3 and
a1 a2 an a2 a1 a3 a2 an an–1
so on.
a1, a2, ……. an are in HP
1 1 1
, ,...... are in AP
a1 a2 an
1 1 1 1 1 1
⇒ = – = – ... – = d (say)
a2 a1 a3 a2 an an–1
1 1 1
⇒ a1a2 = (a1 – a2 ) , a2a3 = (a2 – a3 ) ,……., an–1an = (an – 1 – an)
d d d
3 . 2 6 | Sequences and Series
1 1
Hence, a1a2 + a2a3 + ….+an – 1an = [a1 – a2 + a2 – a3 + …..+an – 1 – an] = (a1 – an)
d d
1 1 a1 – an
But = + (n – 1) d ⇒ = (n – 1) d
an a1 ana1
1 1 1 1 a–b
⇒ = + (n + 1)d; – = (n + 1) d; d =
b a b a ab(n + 1)
1 1 1 1
⇒ = + d ⇒ = + 2d
H1 a H2 a
1 1 1 1
⇒ = + 3d ⇒ = + nd
H3 a Hn a
Note: The sum of the reciprocals of all the n HMs between a and b is equal to n times the reciprocal of the single
HM between a and b.
100
1 1
For example, between 1 and if 100 HMs are inserted, then ∑ = 5050.
100 i=1 Hi
n(a + b)
The sum of reciprocal of n harmonic means =
2ab
1 1 1 1 1 1
, , , ........... , → A.P.
a H1 H2 H3 Hn b
1 1 a–b
= + (n + 1) d ⇒ (n + 1) d =
b a ab
M a them a ti cs | 3.27
a–b
d=
(n + 1)ab
1 1 1 1
Illustration 41: Find the sum of + + ......... .
H1 H2 H3 Hn
n
1 1 1 1 1
Sol: Using =
Hn a
+ nd , we can obtain the values of ,
H1 H2
and so on. Then, by obtaining the value of ∑H , we
n=1 n
will get the result.
1 1 1 1
= + d = + 2d
H1 a H2 a
n
1 1 1 n n(n + 1)
=
Hn a
+ nd ⇒ ∑H= a
+
2
d
n=1 i
n n(n + 1) × (a – b) n (2b + a – b) n
= + = = (a+ b)
a 2(n + 1)ab a 2b 2ab
(i) For 3 numbers a, b and c, HM is defined as the reciprocals of the mean of the reciprocals of a, b and c, i.e. means
11 1 1 3
of reciprocal = + + ; HM =
3a b c 1 1 1
+ +
a b c
(ii) If a1, a2, a3, ………., an are n numbers, then
a + a2 + a3 + a3 + ........ + an
AM = | 1 |
n
GM = (a1 a2 a3 …….an)1/n
n
HM =
1 1 1 1
+ + + .......
a1 a2 a2 an
Illustration 42: If a2, b2, c2 are in AP, then show that b + c, c + a, a + b are in HP.
Sol: Given that a2, b2 and c2 are in AP. Thus, by adding ab + ac + bc to each term and then dividing each term by
(a + b)(b + c)(c + a), we will get the result.
By adding ab + ac + bc to each term, we find that a2 + ab + ac + bc, b2 + ba + bc + ac, c2 + ca + cb + ab are in AP, i.e.
(a + b)(a + c), (b + c)(b + a), (c + a)(c + b) are in AP
∴ Dividing each terms by (a + b)(b + c)(c + a), we find that
1 1 1
, , are in AP, i.e.
b+c c+a a+b
b + c, c + a, a + b are in HP
H1 + a Hn + b
Illustration 43: If H1, H2, ……., Hn are n harmonic means between a and b (≠ a), then find the value of + .
H1 – a Hn – b
1 1 1 1 1
Sol: As a, H1, H2, ....., Hn, b are in HP, , , ....... , are in AP. By considering d as the common difference of this
a H1 H2 Hn b
3 . 2 8 | Sequences and Series
1 1 1 1
= + (n + 1)d and – = (n – 1)d
b a Hn H1
H1 + a 1 / a + 1 / H1 1 / a + 1 / H1
Now, = =
H1 – a 1 / a – 1 / H1 –d
Hn + b 1 / b + 1 / Hn 1 / b + 1 / Hn
and = =
Hn – b 1 / b – 1 / Hn d
H1 + a Hn + b 1 / a + 1 / H1 1 / b + 1 / Hn 1 1 1 1 1
∴ + = + = – + – = 2n
H1 – a Hn – b –d d d b a Hn H1
If a and b are two positive numbers, then it can be shown that A ≥ G ≥ H and A, G, H are in GP, i.e. G2 = AH.
a+b 2ab
Proof: Given that, A = , G = ab and H =
2 a+b
a+b
∴ A–G= – ab
2
( a – b)2
⇒ A–G= ≥0
2
⇒ A ≥ G ……(i)
2ab
G–H= ab –
a+b
a + b – 2 ab ab
⇒ G–H= ab ⇒ G–H= ( a – b )2 ≥ 0
a+b a+b
⇒ G ≥ H ……(ii)
Using (i) and (ii), we find that
A ≥ G ≥ H
Please note that the equality holds only when a = b.
Proof of G2 = AH
a+ b 2ab
Proof: A = ,G= ab and H =
2 a+b
Now, AH = ab = G2 ⇒ A, G & H are in G.P.
A G
Moreover, = ; ∴ A ≥ G ⇒ G ≥ H
G H
Therefore, A ≥ G ≥ H; in fact, RMS ≥ A.M. ≥ G.M. ≥ HM (where RMS is root mean square).
M a them a ti cs | 3.29
PLANCESS CONCEPTS
• If a and b are two positive quantities, then AM, G.M. and HM are always in GP, i.e. only for two numbers.
• If there are three numbers. then AM, G.M. and HM are in G.P. only when the three numbers. are in GP.
For example, 2, 4, 8 → GP
14 24
GM = 4; A.M. = ; HM =
3 7
• For two positive numbers, it has been shown that A ≥ G ≥ H, equality holding for equal numbers.
• For n non-zero positive numbers, it has been shown that A ≥ G ≥ H, equality holding when all the
numbers are equal.
Girish Chandani (JEE 2009, AIR 54)
Illustration 44: If a, b and c are unequal positive numbers in HP, then prove that
a+b c+b
+ > 4. (JEE ADVANCED)
2a – b 2c – b
2 1 1
Sol: As a, b and c are in HP, therefore = + . Thus, by substituting this to LHS, we can prove the given problem.
b a c
1 1 1 1 1 1 1 1
+ + + +
LHS = b a + b c = b a + b c , using (i)
2 1 2 1 1 1
– –
b a b c c a
c c a a a+c a c
=+ + += + + .
b a b c b c a
a c
+
a c a c
Now, A.M. > G.M. ⇒ c a > . or + > 2.
2 c a c a
(=
a + c) (a − c)
2 2
a+c a+c
= = +2
b 2ac 2ac 2ac
a+c
a+ca c
∴ LHS = + > 2 + 2 = 4
b c a
1 1 1
a+b+c + +
1
Proof: As given, A = , G = (abc)1/3 and = a b c
3 H 3
3G3
⇒ a + b + c = 3A, abc = G3 and = ab + bc + ca
H
The equation having a, b and c as its roots is x3 – (a + b + c) x2 + (ab +bc +ca) x – abc = 0
3G3
⇒ x3 – 3Ax2 + x – G3 = 0
H
Illustration 45: The harmonic means between two numbers is given as 4, their A.M. is A, and G.M. is G, satisfy the
relation 2A + G2 = 27. Determine the two numbers. (JEE ADVANCED)
Sol: Let a and b be the two numbers and H = 4 be the harmonic mean between them. Therefore, by using A.M. =
a+b
and G.M. = ab , we can obtain the values of a and b.
2
H = 4 (given)
As A, G and H are in GP, therefore G2 = AH ⇒ G2 = 4A
Also, 2A + G2 = 27 (given; ∴G2 = 4A)
∴ 6A = 27
9 a+b 9
⇒ A= ⇒ = ⇒ a+b=9
2 2 2
We have, G2 = 4A and A = 9/2 ⇒ G2 = 18 ⇒ ab = 18
The quadratic equation having a and b as its roots is x2 – (a + b) x + ab = 0 or, x2 – 9x +18 = 0
⇒ x = 3, 6
Thus, the two numbers are 3 and 6.
Illustration 46: If 2a + b + 3c = 1 and a > 0, b > 0, c > 0, then find the greatest value of a4b2c2 and obtain the
corresponding values of a, b and c. (JEE ADVANCED)
Sol: Since there is a4,. take four equal parts of 2a; as there is b2, take two equal parts of b; as there is c2, take two
equal parts of 3c. Since A.M. ≥ G.M., obtaining A.M. and G.M. of these numbers will help in solving this illustration.
2a 2a 2a 2a b b 3c 3c
Let us consider the positive numbers , , , , , , , .
4 4 4 4 2 2 2 2
2a 2a 2a 2a b b 3c 3c
+ + + + + + +
For the numbers, A = 4 4 4 4 2 2 2 2 = 2a + b + 3c = 1
4+2+2 8 8
(∴ 2a + b + 3c = 1)
1 1
2a 2a 2a 2a b b 3c 3c 8 1 1 1 8
G = . . . . . . . = . . .32 a4b2c2
4 4 4 4 2 2 2 2 24 22 22
1
1 32 4 2 2 8
∴ A ≥ G ⇒ ≥ a b c
8 28
1 32 28 1
or ≥ a4b2c2 or ≥ a4b2c2 or ≥ a4 b2 c2 .
8 8 2 8 8
8 2 3 .8 9.4
M a them a ti cs | 3.31
1
Hence, the greatest value of a4b2c2 =
9.48
It has been found that when the equality holds, the greatest value takes place.
We know that A = G when all the numbers are equal, i.e.
2a b 3c
= = ⇒ a = b = 3c
4 2 2
a b c
∴ = = = k ∴ a = 3k , b = 3k, c = k
3 3 1
∴ 2a + b + 3c = 1 ⇒ 6k + 3k + 3k = 1
1 3 3 1 1 1 1
∴ k= ∴ a= ,b= , c = , i.e. a = , b = , c =
12 12 12 12 4 4 12
PROBLEM-SOLVING TACTICS
(a) When looking for a pattern in a sequence or series, writing out several terms will help you see the pattern, do
not simplify directly. If you do this way, it is often easier to spot the pattern (if you leave terms as products,
sums, etc.).
(b) If each term of an AP is multiplied by (or divided by a non-zero) fixed constant C, the resulting sequence is
1
also an AP, with a common difference C times or times the previous.
c
(c) Tips for AP problems
(i) When the number of terms are three, then we take the terms as a – d, a, a + d;
Five terms as a - 2d, a - d, a, a + d, a + 2d
Here, we take the middle term as ‘a’ and common difference as ‘d’.
(ii) When the number of terms is even, then we take:
Four terms as a – 3d, a – d, a + d, a + 3d;
Six terms as a – 5d, a – 3d, a – d, a + d, a + 3d, a + 5d
Here, we take ‘a – d’ and ‘a + d’ as the middle terms and common difference as ‘2d’.
(iii) If the number of terms in an AP is even, then take the number of terms as 2n and if odd then take it as
(2n + 1).
3 . 3 2 | Sequences and Series
a a a a a
(ii) When the number of terms is even, then we take four terms as, ,ar,ar3; six terms as , ,ar,ar3 ,ar5.
r 3 r
r5 r 3 r
'a'
Here, we take and ‘ar’ as the middle terms and common ratio as ‘r2.’
r
1 1 1 1
For four terms, we take as , , ,
a – 3d a – d a + d a + 3d
1 1 1 1 1
For five terms, we take as , , , ,
a – 2d a – d a a + d a + 2d
FORMULAE SHEET
Arithmetic Progression: Here, a, d, A and Sn represent the first term, common difference, A.M. and sum of the
numbers, respectively, and Tn stands for the nth term.
n
1. Tn = a + (n – 1) d 4. Sn = 2a + (n – 1 ) d
2
2. Tn =
Tn–1 + Tn+1
5. A =
( a1 + a2 + ….. + an )
2 n
Insertion of n arithmetic means between a and b is An
n
3.
Sn =
2
( a + Tn ) 6. n (b – a)
=a+
n+1
Geometric Progression: Here, a, r, Sn and G represent the first term, common ratio, sum of the terms and
G.M., respectively, and Tn stands for the nth term.
a(rn – 1)
1. Tn = a.rn–1 4. Sn =
r– 1
Tn = Tn−1 . Tn+1 a
2. 5. S∞ = (for – 1 < r < 1)
1–r
Arithmetic Geometric Progression: Here, a = the first term of AP, b = the first term of GP, d = common dif-
ference and r = common ratio of GP.
3. ab dbr
S∞ = + (for – 1 < r < 1)
1 – r (1 – r)2
Harmonic Progression
1 1 1 1
1. an = , where a = and d = −
a + (n− 1)d a1 a2 a1
1 1 1 1 1
2. = + + ... +
H n a1 a2 an
1 1 2(a − b) 1 1 n(a − b)
= + and so on ⇒ = +
H2 a (n+ 1)ab Hn a (n+ 1)ab
n
n(n + 1)
1. The sum of n natural numbers
∑r = 2
r =1
n
2. The sum of n odd natural numbers
∑ (2r – 1) = n2
r =1
n
3. The sum of n even natural numbers
∑ 2r
= n(n + 1)
r =1
n
n(n + 1)(2n + 1)
4. The sum of squares of n natural numbers
∑ r2 = 6
r =1
n 2
n(n + 1)
5. The sum of cubes of n natural numbers ∑ r3 = 2
r =1
3 . 3 4 | Sequences and Series
Solved Examples
Example 2: Find out the number of terms in a given AP Example 5: If the sum of four numbers in AP is 50 and
20, 25, 30, 35, …. 100. the greatest of them is four times the least, then find
the numbers.
Sol: We know that Tn = a + (n – 1) d.
Sol: Let the four numbers in AP be a, a +d, a + 2d, a
Given, a = 20, d = 5 and Tn = 100. Therefore, by solving +3d with d > 0.
the equation, we will get the number of terms.
As given, sum of the numbers is 50.
Let the number of terms be n.
a + (a + d) + (a + 2d) + (a + 3d) = 50
Given, Tn = 100, a = 20, d = 5
∴ 4a + 6d = 50
Tn = a + (n – 1) d
⇒ 2a + 3d =
25 ...(i) and
⇒ 100 = 20 + (n – 1) 5 ⇒ 80 = (n – 1) 5
a + 3d = 4a
⇒ 16 = (n – 1) ⇒ n = 17
⇒ 3d = 3a
∴d = a
Example 3: Solve the following series:
∴ Equation (i) becomes 5a = 25
99 + 95 + 91 +87 + …. to 20 terms
Thus, a = 5 = d
n
Sol: Using Sn = 2a + (n – 1 ) d , we can solve the given Therefore, the four number are 5, 10, 15 and 20.
problem. 2
We know that the terms of the given series are in AP. Example 6: If S1, S2, S3,….., SP are the sum of p infinite
Given, geometric progression whose first terms are 1, 2, 3,….,
D = – 4, a = 99 and n = 20 1 1 1 1
p and whose common ratios are , , ,.... ,
2 3 4 p +1
M a them a ti cs | 3.35
p(p + 3) n–1
respectively, then prove that S1 + S2 +….+ Sp = . a1 + d
2 ⇒ 2 1 = 5n + 4 … (i)
a n–1 9n + 6
Sol: As we know S∞ = , therefore by using this a2 + d2
1–r 2
formula we can obtain the value of S1, S2 , ….Sp. a + 12d1
The ratio of the 13th terms is 1 [which is
obtained from (i) with n = 25] a2 + 12d2
a
We know that S∞ =
1–r
a1 + 12d1 129
1 2 ∴ =
∴ S1 = = 2 ; S2 = =3 a2 + 12d2 231
1 1
1– 1–
2 3
p Example 9: If the 7th and 8th terms of an H.P. are 8 and
Sp = = p +1
1 7, respectively, then find its 15th term.
1–
p +1 1
p p Sol: We know that tn = . Therefore, by using
S1 + S2 +….+Sp = [2 × 2 + (p – 1)1] = [p + 3] a + (n− 1)d
2 2
this formula we can solve the given problem.
Example 7: Solve the series 1 +2⋅2+ 3⋅22 + 4⋅23 + ….. Given, T7 = 8 =and T8 = 7
+100⋅299. 1
∴ = 8 ⇒ 8 a + 48 d − 1 = 0 ..(i)
Sol: Let S = 1 + 2⋅2 + 3⋅2 + 4⋅2 + ….+ 100⋅2 .
2 3 99
a + 6d
Therefore, by multiplying 2 on both the sides and then 1
= 7 ⇒ 7 a + 49 d − 1 = 0 ..(ii)
taking the difference, we can solve the given problem. a + 7d
Given,
By solving these two equations, we find that d = a
S = 1 + 2⋅2 + 3⋅22 + 4⋅23 + ….+ 100⋅299 … (i)
1
Multiplying 2 on both the sides, ∴ From eq.(i), we get 8
=
7a
2S= 1⋅2+2⋅22+3⋅23+…..+99⋅299+100⋅2100 … (ii) 1
⇒a=d=
56
Subtracting (ii) from (i), we get
1 56
–S =1 + 1⋅2 + 1⋅22 +1⋅23+……+1⋅299 - 100⋅2100 ∴ T15 = =
a + 14d 15
1 – 2100
–S= − 100 ⋅ 2100 ;
1–2 Example 10: Suppose x y and z are positive real
numbers, which are different from 1.
⇒ S = 99⋅2100 + 1
If x18 = y21 = z28, then show that 3, 3logy(x). 3logz(y) and
Example 8: If (5n + 4) : (9n + 6) is the ratio of the sums 7 logx(z) are in AP.
of the nth terms of two APs, then find out the ratio of
Sol: By applying log on x18 = y21 = z28, we can find the
their 13th terms.
values of logy x , logz y and logx z .
Sol: Let a1 and a2 be the first terms of the two APs and Given, x18 = y21 = z28
d1 and d2 be their respective common difference.
Taking log, we find that
n
Applying Sn = 2a + (n – 1 ) d , we can solve the given
2 18 log x = 21 log y = 28 log z
problem.
logx 7
Given, log
= y x =
log y 6
n 7
[2a + (n – 1)d1 ] ⇒ 3logyx = … (i)
2 1 5n + 4 2
=
n 9n + 6 log y 4
[2a2 + (n – 1)d2 ] logzy = = ; 3logzy = 4 … (ii)
2 logz 3
3 . 3 6 | Sequences and Series
Suppose d be the common difference of the given AP, From (i), (ii), (iii)
then a – b = (p - q) d1
logyx = 1 + d ⇒ x = y 1+d
… (i) b – c = (q - r) d1
logzy = 1 + 2d ⇒ y = z1+2d … (ii) c – a = (r - p) d1
15logxz =– (1 + 3d) Therefore, ab–c. bc–a. ca–b
1 +3d
= (a1r1p–1)b–c (a1r1q–1)c–a (a1r1r–1)a–b
⇒ z=x –15 … (iii)
Elimination y and z from equations (i), (ii) and (iii), we = a1b–c+c-a+a–b.r1(p–1)(b–c)+(q–1)(c–a)+(r–1)(a-b)
get (p −1)( q−r )d1 +( q−1)(r −p )d1 +(r −1)(p −q)d1
= a10 . r1
(1–d)(1 + 2d)(1 +3d)
x= x –15 0 0
= a=
1 .r1 1
= ( a )( a – a ) = d
n n
= [2 × 1 + (n – 1) 1] = [1 + n] an + n–1 n n–1
2 2
= ( a – a ),
n n 1 1 1
S2 = [2a + (n – 1)d2] = [2 × 1 + (n – 1) 2] = n2 ⇒ 2 1
2 2 a2 + a d a + a2
1 3
n n
S3 = [2a + (n – 1) d3] = [2 × 1 + (n – 1) 3]
2 2 a3 – a2
= , ………
n d
= [3n – 1]
2
1 an – an–1
n =
S1 + S3 = [1 + n + 3n – 1] = 2n2 = 2S an + an–1 d
2
1 1
Example 2: Calculate the sum to n terms of the series: LHS= + +…………+
a1 + a2 a2 + a3
8 +88 + 888 + ……..
8 8 8
= [10 + 100 + 1000 + … + to n terms] – n= Example 4: A series of natural numbers is divided into
9 9 9
groups: (1); (2, 3, 4); (5, 6, 7, 8, 9) and so on. Prove that
(10n – 1) 8n the sum of the numbers in the nth group is (n – 1)3 + n3.
–
9 9
8 Sol: In this problem, the last term of each group is the
= [10n+1 – 9n – 10] square of the corresponding number of the group.
81
Thus, the first term of the nth group is (n – 1)2 + 1 = n2
n
Example 3: If a1, a2, a3, ……., an are in AP, where ai > 0 for – 2n + 2. Hence, by using Sn = 2a + (n – 1 ) d , we can
all I, then show that 2
solve the problem.
1 1
+ + ........ +
a1 + a2 a2 + a3 The number of terms in the first group = 1
The number of terms in the second group = 3
1 n–1
= The number of terms in the third group = 5
an–1 + an a1 + an
∴ The number of terms in the nth group = 2n–1
3 . 3 8 | Sequences and Series
JEE Main/Boards
Q.13 Find the sum of first 2n terms of the series: Q.24 Natural numbers have been grouped in the
12+ 2 + 32 + 4 + 52 + 6 + …. following way 1 ; (2, 3) ; (4, 5, 6); (7, 8, 9, 10) ; ……
Show that the sum of the numbers in the nth group is
Q.14 The H.M of two numbers is 4 and their A.M. (A)
n(n2 + 1)
and G.M. (G) satisfy the relation 2A + G2 = 27. Find the .
numbers. 2
Q.8 Let s1, s2, s3 ….. and t1,t2,t3 …. are two arithmetic
Exercise 2 10 15
sequence such that s1 = t1 ≠ 0; s2 = 2t2 and ∑ si = ∑ ti .
=i 1 =i 1
Single Correct Choice Type s2 – s1
Then the value of is
t2 – t1
Q.1 If a, b, c are distinct positive real in H.P., then the
(A) 8/3 (B) 3/2 (C) 19/8 (D) 2
b+a b+c
value of the expression, + is equal to
b–a b–c
Q.9 Let an, n ∈ I be the nth term an A.P. with common
(A) 1 (B) 2 (C) 3 (D) 4 difference ‘d’ and all whose terms are non-zero. If n
approaches infinity, then the sum
Q.2 The sum of infinity of the series 1 1 1
+ + ..... + will approach
1 1 1 a1a2 a2a3 anan+1
+ + + ........ is equal to
1 1+2 1+2+3
1 2 1
(A) (B) (C) (D) a1d
(A) 2 (B) 5/2 (C) 3 (D) None a1d a1d 2a1d
Q.3 Along a road lies an odd number of stones placed Q.10 The sum of the first three terms of an increasing
at intervals of 10 m. These stones have to be assembled G.P. is 21 and the sum of their squares is 189. Then the
around the middle stone. A person can carry only one sum of its first n term is
stone at a time. A man carried out the job starting with
1
the stone in the middle, carrying stones in succession, (A) 3(2n – 1) (B) 12 1 –
thereby covering a distance of 4.8 km. Then the number 2n
of stones is 1
(C) 6 1 – (D) 6(2n – 1)
(A) 15 (B) 29 (C) 31 (D) 35 2n
∞
n
Q.4 If S = 12 +32 +52 + …. + (99)2 then the value of the Q.11 The sum ∑ n4 + 4 is equal to
sum 22 + 42 +62 +….. + (100)2 is n=1
(A) S + 2550 (B) 2S (C) 4S (D) S +5050 (A) 1/4 (B) 1/3 (C) 3/8 (D) 1/2
Q.5 In an A.P. with first term and the common difference Q.12 If a ≠ 1 and (ln a2) + (ln a2)2 + (ln a2)3 + ….. = 3 [lna
d (a, d ≠ 0), the ratio ‘S’ of the sum of the first n terms to + (ln a)2 + (ln a)3 + (ln a)4 + ……], then ‘a’ is equal to
sum of n terms succeeding them does not depend on
n. Then the ratio "a/d" and the ratio ‘ρ’, respectively are (A) e1/5 (B) e1/2 (C) 3e1/2 (D) e1/4
M a them a ti cs | 3.41
Previous Years’ Questions Q.8 If the sum of first n terms of an AP is cn2, then the
sum of squares of these n terms is (2009)
Q.1 If a, b, c d and p are distinct real numbers such that n(4n2 – 1)c2 n(4n2 + 1)c2
(A) (B)
(a2 + b2 + c2)p2 – 2(ab + bc + cd)p + (b2 + c2 + d2) ≤ 0, 6 3
then a, b, c, d (1987) n(4n2 – 1)c2 n(4n2 + 1)c2
(C) (D)
(A) Are in A.P. (B) Are in G.P. 3 6
(A) AP (B) H.P. (C) G.P. (D) None Q.11 If m is the A. M. of two distinct real numbers l
and n(l, n > 1) and G1, G2 and G3 are three geometric
means between l and n, then G14 + 2G24 + G34 equals:
Q.4 If a, b, c, d are positive real number such that
(2015)
a + b + c + d = 2, then M = (a + b) (c +d) satisfies the
relation (2000) (A) 4 l2 mm (B) 4 lm2 n (C) 4 lmn2 (D) 4 l2m2n2
(A) 0 < M ≤ 1 (B) 1 ≤ M ≤ 2
Q.12 The sum of first 9 terms of the series is
(C) 2 ≤ M ≤ 3 (D) 3 ≤ M ≤ 4
13 13 + 23 13 + 23 + 33
+ + + ....... (2015)
Q.5 Let the positive numbers a, b, c, d be in A.P. then 1 1+ 3 1+ 3 + 5
abc, abd, acd, bcd are (2001) (A) 71 (B) 96 (C) 142 (D) 192
(A) not in AP/GP/HP (B) in AP
Q.13 If the 2nd, 5th and 9th terms of a non-constant
(C) in GP (D) in HP
A.P. are in G.P., then the common ratio of this G.P. is:
(2016)
Q.6 Suppose a, b, c are in AP and a2, b2, c2 are in G.P. 4 7 8
3 (A) (B) 1 (C) (D)
If a < b < c and a + b + c = , then the value of a is 3 3 5
2 (2002)
1 1 Q.14 If the sum of the first ten terms of the series is 16 m
(A) (B) 5
2 2 2 3 2
3 2 1
2 2 2
2 4
then m is equal to 1 + 2 + 3 + 4 + 4 + .......
1 1 1 1 5 5 5 5
(C) – (D) – (2016)
2 3 2 2
(A) 101 (B) 100 (C) 99 (D) 102
Q.7 An infinite G.P. has first term x and sum 5, then x
belongs to (2004) Q.15 Three positive numbers form an increasing G.P. If
the middle term in this G.P. is doubled, the new numbers
(A) x < – 10 (B) – 10 < x < 0
are in A.P. Then the common ratio of the G.P. is (2014)
(C) 0 < x < 10 (D) x > 10
(A) 2 − 3 (B) 2 + 3 (C) 2 + 3 (D) 3 + 2
3 . 4 2 | Sequences and Series
Assertion Reasoning Type Q.19 The real number k for which the equation, 2x3 +
3x + k = 0 has two distinct real roots in [0, 1] (2013)
Q.16 Statement-I: The sum of the series 1 + (1 + 2 + 4) + (A) Lies between 1 and 2
(4 + 6 + 9) + (9 + 12 + 16) + ...... + (361 + 380 + 400) is 8000.
n (B) Lies between 2 and 3
Statement-II: ∑ (k3 − (k − 1)3 ) =
n3 for any natural
(C) Lies between -1 and 0
number n. k =1 (2012)
(A) Statement-I is false, statement-II is true (D) Does not exist.
JEE Advanced/Boards
Q.3 If sinx, sin22x and cosx.sin4x form an increasing Q.6 Two distinct, real infinite geometric series each
geometric sequence, then find the numerical value have a sum of 1 and have the same second term. The
of cos2x. Also find the common ratio of geometric third term of one of the series is 18. If the second term
sequence. m–n
of both the series can be written in the form ,
p
Q.4 If a, b, c, d, e be 5 numbers such that a, b, c are in where m, n and p are positive integers, and m is not
AP; b, c, d are in G.P. & c, d, e are in H.P. then, divisible by the square of any prime, find the value of
100m + 10n+p.
M a them a ti cs | 3.43
99
5100 a– b
Q.7 Let S = ∑ (25)n + 5100 . Find [s]. where Tn is the nth term of series then T22 =
4
(a,
n=1 b ∈ N). Find the value of (a + b).
Where [y] denotes largest integer less than or equal to y.
Q.17 Given a three digit number whose digits are
Q.8 Given that the cubic ax – ax + 9bx – b = 0 (a ≠ 0)
3 2 three successive terms of a G.P. If we subtract 792
has all three positive roots. Find the harmonic mean of form it, we get a number written by the same digits
the roots independent of a and b, hence deduce that in the reverse order. Now if we subtract four from the
the root are all equal. Find also the minimum value of hundred’s digit of the initial number and leave the
(a + b), if a and b ∈ N. other digits unchanged, we get a number whose digits
are successive terms of an A.P. Find the number.
Q.9 A computer solved several problems in succession.
π
The time it took the computer to solve each successive Q.18 For 0 < θ < , let S(θ) = 1 + (1 + sinθ) cos θ +
4
problem was the same number of times smaller than
(1 + sinθ + sin2θ) cos2θ + ….. ∞.
the time it took to solve the preceding problem. How
many problems were suggested to the computer if it 2 –1 π
Then find the value of S .
spent 63.5 min to solve all the problems except for the 2 4
first, 127 min to solve all the problems except for the
last one, an 31.5 min to solve all the problems except π π x
for the first two? Q.19 If tan – x ,tan ,tan + x in order are
12 12 12
three consecutive terms of a G.P., then sum of all the
Q.10 The sequence a1, a2, a3, …… a98 satisfies the relation
solutions in [0, 314] is kπ. Find the value of k.
an+1 = an +1 for 1,2,3,…. 97 and has the sum equal to
49
4949. Evaluate ∑ a2k . Exercise 2
k =1
Q.11 Let a and b be positive integers. The value of xyz Single Correct Choice Type
343
is 55 or , according as a, x, y, z, b are in arithmetic
55 Q.1 The arithmetic mean of the nine numbers in the
progression or harmonic progression resp.. Find the give set {9, 99,999,……999999999} is a 9 digit number
value of (a2 +b2). N, all whose digits are distinct. The number N does not
contain the digit
Q.12 If the roots of 10x3 – cx2 – 54x – 27= 0 are in (A) 0 (B) 2 (C) 5 (D) 9
harmonic progression, then find c and all the roots.
360
1
Q.13 If a, b, c be in G.P. & logca, logbc, logab be in AP,
Q.2 ∑ is the ratio of two relative
k =1 k k + 1 + (k + 1 k )
then find the common difference of the AP if logac = 4.
prime positive integers m and n. The value of (m + n)
is equal to
Q.14 The first term of a geometric progression is equal
to b – 2, then third term is b + 6, and the arithmetic (A) 43 (B) 41 (C) 39 (D) 37
mean of the first and third term to the second term is in 100
k
the ratio 5: 3. Find the positive integral value of b. Q.3 The sum ∑ k 4 + k2 + 1 is equal to
k =1
Q.15 In a G.P. the ratio of the sum of the first eleven 4950
(A) (B) 5050
terms to the sum of the last eleven terms is 1/8 and the 10101 10101
ratio of the sum of all the terms without the first nine to
the sum of all the terms with out the last nine is 2. Find 5151
(C) (D) None of these
the number of terms in the GP. 10101
these square were also connected by segments so that Q.9 If the roots of the equation x3 + px2 + qx – 1 = 0
a new square was obtained and so on, then the radius form an increasing GP, where p and q are real, then
of the circle inscribed in the nth square is
(A) p + q = 0
1–n 3–3n (B) p ∈ (–3, ∞)
(A) 2 2 r (B) 2 2 r
(C) One of the roots is unity
(A) Statement-I is true, statement-II is true and (A) 18 (a + b + c)2 = 18(a2 + b2 +c2) + ab
statment-II is correct explanation for statment-I. (B) a, b, c are in GP
(B) Statement-I is true, statement-II is true and (C) a, 2b, 2c are in HP
statment-II is NOT the correct explanation for statment-I.
(D) a, b, c can be the lengths of the sides of a triangle.
(C) Statement-I is true, statement -II is false.
(Assume all logarithmic terms to be defined)
(D) Statement-I is false, statement-II is true
Q.4 Let a1, a2, a3,….., a11 be real numbers satisfying Q.9 The sum V1 + V2 + ...... + Vn is
(B) H1 < H2 < H3 < ...... Statement-I: The numbers b1, b2, b3, b4 are neither in
A.P. Nor in G.P.
(C) H1 > H3 > H5 >...... and H2 < H4 < H6 <......
Statement-II: The numbers b1, b2, b3, b4 are in H.P.
(D) H1 < H3 < H5 <...... and H2 > H4 > H6 >......
(A) Statement-I is true, statement-II is true; statement-II
is a correct explanation for statement-I
Paragraph 2: Let Vr denote the sum of the first ‘r’ terms
of an arithmetic progression (A.P.), whose first term is ‘r’ (B) Statement-I is true, statement-II is true; statement-II
and the common difference is (2r – 1). Let Tr = Vr + 1 – is not a correct explanation for statement-I.
Vr – 2 and Qr = Tr + 1 – Tr for r = 1, 2, ... (2007) (C) Statement-I is true, statement-II is false
(D) Statement-I is false, statement-II is true
3 . 4 6 | Sequences and Series
Q.14 If the sum of first n terms of an A.P. is cn2, then the Q.18 Let a1, a2, a3, ….. be in harmonic progression with
sum of squares of these n terms is (2009) a1 = 5 and a20 = 25. The least positive integer n for
2 2 2 2 which an< 0 is (2012)
(A) n(4n − 1)c (B) n(4n + 1)c
6 3 (A) 22 (B) 23 (C) 24 (D) 25
4n k(k +1)
n(4n2 − 1)c2 n(4n2 + 1)c2
(C)
3
(D)
6
Q.19 Let S=
n ∑ ( −1) 2 k2. Then Sn can take value(s)
k =1 (2013)
Q.15 Let Sk, k = 1, 2,...., 100, denote the sum of the (A) 1056 (B) 1088 (C) 1120 (D) 1332
k −1
infinite geometric series whose first term is b
1 k! Q.20 Let a, b, c be positive integers such that is an
a
and the common ratio is . Then the value of
2 100 k integer. If a, b, c are in geometric progression and the
100
+ ∑ (k 2 − 3k + 1)Sk is (2010) arithmetic mean of a, b, c is b + 2, then the value of
100! k =1
a2 + a − 14 is ________ (2014)
a +1
Q.16 Let a1, a2, a3, …, a11 be real numbers satisfying
a1 = 15, 27 − 2a2 > 0 and ak = 2ak−1 − ak−2 for k
a12 + a22 + .... + a11
2
Q.21 Suppose that all the terms of an arithmetic
= 3, 4, …,11. If = 90 , then the value of
11 progression (A.P.) are natural numbers. If the ratio
a1 + a2 + .... + a11 of the sum of the first seven terms to the sum of the
is equal to (2010)
11 first eleven terms is 6 : 11 and the seventh term lies in
between 130 and 140, then the common difference of
Q.17 Let b = 6, with a and c satisfying (E). If α and β are this A.P. is (2015)
the roots of the quadratic equation ax2 + bx + c = 0, then
n
∞
1 1 Q.22 Let bi > 1 for i = 1, 2, …, 101. Suppose loge b1 loge
∑ α + β is (2011) b2, …, loge b101 are in Arithmetic Progression (A.P.) with
n= 0 the common difference loge 2. Suppose a1, a2, …, a101
6
(A) 6 (B) 7 (C) (D) ∞ are in A.P. such that a1 = b1 and a51 = b51. If t = b1 + b2 +
7
…+ b51 and s = a1 + a2 + … + a51, then (2016)
(A) s > t and a101 > b101 (B) s > t and a101 < b101
(C) s < t and a101 > b101 (D) s < t and a101 < b101
M a them a ti cs | 3.47
PlancEssential Questions
JEE Main/Boards JEE Advanced/Boards
Exercise 1 Exercise 1
Q.3 Q.11 Q.14 Q.6 Q.9 Q.12
Q.17 Q.21 Q.25 Q.15 Q.17
Q.27
Exercise 2 Exercise 2
Q.2 Q.4 Q.10 Q.1 Q.4 Q.5 Q.12
Q.13
Previous Years’ Questions
Previous Years’ Questions Q.1 Q.3 Q.4
Q.2 Q.5 Q.8 Q.6 Q.7 Q.8
Answer Key
7n 7 1 n Q.16 188090
Q.3 (a) – 1– ; (b) 2 [10n+1 – 9n – 10]
9 81 10 27 Q.17 a = 5, b = 8, c = 12.
Q.5 (2, 4, 6) or (6, 4, 2) 1
Q.20 –5050, [(5050)2 – 338350]
2
Q.6 (i) 6633 (ii) 2842 (iii) 945
Q.21 9
Q.7 69: 128
Q.22 469
35 3 (3n− 2)
Q.10 − − Q.23 3370 m
16 16(5 ) 4(5n−1 )
n − 2
1
Q.26
Q.11 c = 6, b = 4, d = 9; b = – 2, c = – 6, d = – 18 a1 (a2 – a1 )
3 . 4 8 | Sequences and Series
Exercise 2
Single Correct Choice Type
JEE Advanced/Boards
Exercise 1
5 –1
Q.1 (i) 6, 3 ; (ii) 120, 30 Q.2 1 Q.3 ; 2
2
Q.4 (iii) b = 4, c = 6, d = 9 or b = – 2, c = – 6, d = – 18 Q.5 12 Q.6 518 Q.7 49
Exercise 2
Single Correct Choice Type
Q.5 D
Solutions
given r = 3, a = 1 7n 7 1
n
= – 1–
(3n – 1) 9 81 10
⇒ = 364 ⇒3n – 1 = 728
(3 – 1)
(b) 6 + 66 + 666 = 6[1 + 11 + 111 …]
⇒3n = 729
6 2
= [9 + 99 + 999 + ……] = [10 – 1 + 100 – 1 + 1000 – 1 … ]
⇒ n=6 9 3
2 10(10n – 1) 2 10 2n
a = – n = × (10n – 1) –
Sol 2: Sum of infinite G.P. is 2 ⇒ =2 3 10 – 1 3 9 3
–r + 1
⇒a = – 2(r – 1)
Series is a, ar, ar2 ……. (|r| < 1) =
2
27
(
10n+1 – 10 –)2n × 9
3× 9
=
2 n+1
27
10 – 9n – 10
⇒a3, (ar)3, (ar2)3 …………….. …(2)
First term of this infinite series is a3 and ratio is r3 Sol 4: a, b, c are in AP
3
a (i) b + c, c + a, a + b are also in AP
Hence sum of this infinite series is 3
–r + 1
a, b, c are in AP ⇒ 2b = a + c
a3
Given = 24 ⇒b–a=c–b
–r 3 + 1
8(r − 1)3 (r – 1)2 ⇒ a–b =b–c
= 24⇒ =3
( – r + 1)
3 2
(r + r + 1)
Difference between term of given AP = a – b, b – c
which are equal by equation (i)
⇒r2 + 1 – 2r = 3r2 + 3r + 3
Hence b + c, c + a, a + b is an AP
⇒2r2 + 5r + 2 = 0 ⇒2r2 + 4r + r + 2 = 0
1 1 1
⇒(2r + 1) (r + 2) = 0 (ii) , , are also in AP
bc ac ab
1 (b – a) (c – b)
⇒ r = – 2, r = – Common difference = ,
2 cab abc
1 By equation (i) b – a = c – b ie difference between terms
|r| < 1 ⇒ r = – ⇒ a = +3
2 is same
3 3 3
Series is 3, − , , − ,..... Hence the given series is in AP
2 4 8
(iii) a2(b + c), b2(c + a), c2(a + b)
Sol 3: (a) Sum upto n terms Difference = b 2
c + b2a – a2b – a2c , c
2
a + c2b – b2c – b2a
Sn = 0.7 + 0.77 + 0.777 ……….… n terms d1 d2
3 . 5 0 | Sequences and Series
Hence given series is an AP Sol 7: Sum of first n terms of 2 AP’s are in ratio
1 1 1 1 1 1 3n – 3
(iv) a + , b + , c + =
b c c a a b 5n + 21
(4 + 2, 4, 4 – 2) for (r = – 2) p+q
= [2a + (p + q – 1)d] …(iii)
2
⇒ (6, 4, 2)
Subtracting (ii) from (i)
Sol 6: (i) Sum of integers between 1 & 200 which are (p – q)d = x – y
multiple of 3 x–y
⇒d= and putting this value in equation (i)
3, 6, 9, … 198 ⇒ n = 66 p–q
n (p – 1)(x – y)
Hence sum = [a + l] a+ =x
2 p–q
(66)
= [3 + 198] = 33 [201] = 6633 (px – py – x + y) –qx + py + x – y
2 ⇒a= x– =
p–q p–q
(ii) Multiple of 7
7, 14, 21 … 196 ⇒ n = 28 Putting values of a and d in equation (iii)
p + q x – y – qx + py + px – qy 4sn 3n – 2 15 (5n–1 – 1)
= =1– n +
2 p–q 5 5 4 5n
p+q (p – q) (x – y) p + q x – y 5 (3n – 2) 75 1 1
= (x + y) + = x + y + Sn = – + –
2 (p – q) p – q 2 p – q 4 4.5n–1 16 5 5n
5× 4 15 3 –(3n – 2)
Sol 9: a, b, c are in HP = + - n–2
4×4 16 16 × 5 4 × 5n–1
2 1 1 2ac
i.e = + ⇒b= …(i)
b a c a+c 35 3 –(3n – 2)
= –
2a1c1 16 16(5n–2 ) 4(5n–1 )
[where a1 & c1 are 1st & 3rd terms of given series]
a1 + c1
2ac Sol 11: a, b, c are in AP
(a + b)(b + c) 2ac 2ac ⇒ 2b = a + c …(i)
= = =
a c a2
+ ab + c 2
+ bc a2
+ c 2
+ b(a + c)
+ b, c, d in GP
b+c a+b
c2 = bd …(ii)
2ac
= (from equation (i) c, d, e are in HP
a2 + c2 + (2ac)
2ce
d= …(ii)
2ac b c + e
= = = b1
(a + c)(a + c) a+c
Given that a = 2, e = 18
2a1c1
Middle term of given series, hence = b1 ie given (2b – 2)2
a1 + c1 We have (2b – 2) = c from (i) and = d from (ii)
Series is H.P b
= n(n +1) +
( 4n – 1) n = n [4n +3n + 2]
2
2
Sol 17: a, b, c ∈ (2, 18)
a + b + c = 25 …(i)
3 3
2a = 2 + b …(ii)
2ab c2 = 18 b …(iii)
Sol 14: HM of 2 numbers is 4 ie =4
a+b
b = 2a – 2
ab
= 2a + 2b …(i)
c = 25 – a – 2a + 2 = 27 – 3a
a+b ⇒ (27 – 3a)2 = 18(2a – 2)
AM = and G.M. = ab
2
⇒ (9 – a)2 = 4(a – 1) ⇒ a2 + 81 – 18a = 4a – 4
We have 2A + G2 = 27 ⇒ a + b + ab = 27
⇒ a2 – 22a + 85 = 0
a + b + 2a + 2b = 27 [from (i)]
a = 17, 5
⇒ a + b = 9 and ab = 18
b = 32, 8
[a = 6, b = 3]; [a = 3, b = 6]
c = 24, 12
Sol 15: (33 – 23) + (53 – 43) + (73 – 63) + … Numbers are (5, 8, 12)
= 33 + 53 + 73 + … + 213
M a them a ti cs | 3.53
Sol 18: a > 0, b > 0, c > 0 Sol 21: Given that,for a polygon of "n" sides, we have
1 1 1 α = 120º; d = 5
To prove (a + b + c) + + ≥ 9
a b c Sum of interior angle
We know that A.M. ≥ G.M. ≥ HM (n – 2) 180º = a + (n – 1)d
Therefore, A.M. ≥ HM n n
= 2(120) + (n – 1)5 = 5n + 235
For 3 numbers a, b, c 2 2
G1 G2 = ab …(ii) ⇒ 7b = 5c – 3
2{
1
(5050 ) − 338350}
2
=
3 . 5 4 | Sequences and Series
1 1 1 1 1 1
Sol 26: S = + … ⇒ + + ≥9
a1a2 a2a3 anan+1 x y z
1 a2 – a1 a3 – a2 Therefore, xy + yz + zx – 9xyz ≥ 0
= + ... Hence proved.
a2 – a1 a1a2 a2a3
Exercise 2 n
S2 = [2a + 2nd + (n – 1)d]
2
Single Correct Choice Type s1 2a + (n – 1)d
=
s2 2a + 2nd + (n – 1)d
2ac
Sol 1: (B) b = [a, b, c are in HP] …(i) d
a+c 2 + (n – 1)
⇒ a =s
2ac 2ac d
+a +c 2 + (3n – 1)
b+a b+c a+c
= + = + +c
a a
b–a b–c 2ac 2ac
–c –c d d
a+c a+c ⇒ 2 + (n – 1) = 2s + (3n – 1) s
a a
3ac + a2 3ac + c2 3c + a (3a + c) d
= + = + ⇒ 2s – 2 = [n – 3ns + s – 1]
ac – a 2
ac – c 2 c–a a–c a
a
3c + a (3a + c) 2c – 2a (2s – 2) = n (1 – 3s) + s – 1
= – = =2 d
c–a c–a c–a
This is independent of n ie coefficient of n will be zero
1 1 1 1 a 1
Sol 2: (A) Given summation is, + + ⇒s= ; ⇒ =
1 1+2 1+2+3 3 d 2
2 1 1
…
Tn = ⇒ Tn = 2 – Sol 6: (D) a = 51+x + 51 –x + 25x + 25–x
n(n + 1) n n + 1
5
1 1 1 1 1 1 ⇒ a = 5.5x + + (5x)2 + (5–x)2
Sn = STn = 2 – + – + ... = 2 5x
1 2 2 3 3 ∞ 5 1
Let 5x = t ⇒ a = 5t + + t2 + 2
t t
5 2 1
5t + t + 2
Sol 3: (C) t t ≥ 1 hence a ≥ 10 + 2
≥ 5;
• • • • • • • 2 2
n stones
Sol 7: (A) S11 = S19
⇒ 2[20 + 40 + 60 …n terms] = 4800
11 19
[2a + 10d] = [2a + 18d]
⇒ 120 = 1 + 2 … n terms 2 2
n(n + 1) a –29
⇒ 120 = ⇒ n2 + n = 240 16a = – 232 d ⇒ =
2 d 2
⇒ n = 15 30 29d
S30 = [2a + 29d] = 30 a + =0
Total no of stone = 2n+1 = 31 [C] 2 2
Sol 10: (A) a + ar + ar2 = 21 …(i) ⇒ (a2p2 – 2abp + b2) + (b2p2 – 2bcp + c2) +
(c2p2 – 2cdp + d2) ≤ 0
a2 + a2r2 + a2r4 = 189 …(ii)
⇒ (ap – b)2 + (bp – c)2 + (cp – d)2 ≤ 0
Squaring equation (i) & then dividing by (ii)
(Since, sum of squares is never less than zero )
2 2 2
a (1 + r + r ) 441 ⇒ Each of the squares is zero
⇒ =
2 2
a (1 + r + r ) 4 189
∴(ap – b)2 = (bp – c)2 = (cp – d)2 = 0
(1 + r + r 2 )(1 + r + r 2 ) 441 b c d
⇒ = ⇒p= = =
2
(1 + r + r )(1 – r + r ) 2 189 a b c
\a, b, c are in G.P.
⇒ 2r2 – 5r + 2 = 0
1 Sol 2: (C) Sum of the n terms of the series
⇒ r = 2, ⇒ a = 3, 12
2
1 3 7 15
(2n – 1) + + + + .... upto n terms can be written as
GP is 3, 6, 12 … Sn = 3 = 3(2n – 1) Hence, (A) is 2 4 8 16
the correct choice 2 – 1
1 1 1 1
1 – + 1 – + 1 – + 1 – .... upto n terms
2 4 8 16
∞
n 1
Sol 11: (C) ∑ n4 + 4 =
2 4 1 1 1
n=1
nn + 2 + 4 – 4 = n – + + + .... + nterms
n 2 4 8
1 1
1 1 1 – n
=∑ = ∑ 2 2 2 2
2
2 = n– = n + 2–n – 1
n n + – 4 n n + – 2 n + + 2 1
n n n 1–
2
n Sol 3: (B) Let the common ratio of the G.P. be r. Then,
= ∑ (n2 + 2 – 2n)(n2 + 2n + 2)
Y = xr and z = xr2
1 1 1 ⇒ ln y = lnx + ln r and ln z = ln x + 2 ln r
= ∑ –
4 n2 – 2n + 2 n2 + 2n + 2 Let A = 1 + ln x, D = ln r
1 1
1 ∞ 1 1 Then, = ,
= ∑ –
4 n=1 (n − 1) + 1 (n + 1) + 1
2 2 1 + lnx A
1 1 1
= =
1 1 ∞ 1 1 3 1 + ln y 1 + lnx + lnr A + D
= 1 + + ∑ – =
4 2 n=1 (n + 1) + 1 (n + 1) + 1 8
2 2
1 1 1
and
= =
1 + lnz 1 + lnx + 2lnr A + 2D
Sol 12: (D) ln a2 + (ln a2)2 + (ln a2)3 …..
1 1 1
Therefore, , , are in H.P.
= 3 {ln a + (ln a)2 + (ln a)3 + …. } 1 + lnx 1 + ln y 1 + lnz
2lna 3lna
⇒ =
1 – 2lna 1 – lna Sol 4: (A) Since A.M. ≥ G.M., then
⇒ 2 – 2 ln a = 3 – 6 ln a
(a + b) + (c + d)
≥ (a + b)(c + d)
⇒ 1 = 4 ln a 2
⇒ a = e1/4 ⇒M ≤ 1
M a them a ti cs | 3.57
Leta = A – D, b = A, c = A + D
Sol 9: (B) Let a, ar, ar2, …
3
Given, a + b + c = a + ar = 12 … (i)
2
3 3 1 ar + ar = 48
2 3
… (ii)
⇒ (A – D) + A + (A + D) = ⇒ 3A = ⇒ A = Dividing (ii) by (i), we have
2 2 2
1 1 1 ar 2 (1 + r)
∴ The numbers are – D, , + D =4
2 2 2 a(r + 1)
2 2
1 1 1 ⇒ r2 = 4 if r ≠ – 1
Also, – D , , + D are in GP.
2 4 2 ∴r=–2
Also, a = – 12 (using (i)).
2 2 2 2
1 1 1 1 1
=
⇒ – D +D ⇒ = – D2 2 6 10 14
4 2 2 16 4 Sol 10: (B) Let S =1 + + + + + .......
...(1)
... (i)
3 32 33 34
1 1 1 1 1 1 2 6 10
⇒ – D2 = ± ⇒ D2 = ⇒ D =± S= + + + + ....... ...(2)... (ii)
4 4 2 2 3 3 32 33 34
1 1 3
n 4 n 2 n 4
=G1 =
G2 =
G3 Sol 15: (B) a, ar, ar2 → G.P.
a, 2ar, ar2 → A.P.
G14 + 2G24 + G34
2 × 2ar = a + ar2
2 3
n n n
= 4 × + 2 4 × + 4 × 4r = 1 + r2
2
3
⇒ r2 – 4r + 1 = 0
=3n + 22n2 + n3
4 ± 16 − 4
= n(2 + 2n + n2 ) r= = 2± 3
2
= n( + n)2
= 4m2n r= 2 + 3
n2 (n + 1)2 r = 2 − 3 is rejected
7 1 1 1 G2
= 1 − + 1 − 2 + 1 − 3 + ....... + up to 20 terms HM =
AM
(we know this)
9 10 10 10
⇒ (A – 27) (A) = (A – 15)2
20
1 1 ⇒ 27A = 225 – 30 A
1−
7 10 10 20
A = 75 …(i)
= 20 − = = 7 20 − 1 1 − 1
9 1 9 9 10 G.M. = 60 …(ii)
1 −
10
H.M. = 48
7 179 1 1 7
20 a + b = 150 using (i) and (ii)
= + = 179 + (10)−20
9 9 9 10 81 ab = 3600
a = 120
b = 30
Sol 21: (A) 2y = x + z
2 tan-1 y = tan-1 x + tan-1 (z) Sol 2: H10 = 21, H21 = 10
2y x+z 1 1 1
tan−1 = tan−1 = + 9d =
1− y2 1 − xz H10 H1 21
x+z x+z 1 1 1
= = + 20d =
1− y 2 1 − xz H10 H1 10
11 1
⇒ 11d = ⇒d=
⇒ y2 = xz or x + z = 0 ⇒ x = y = z 210 210
1 9 10
+ =
H1 210 210
JEE Advanced/Boards 1 1
=
H1 210
Exercise 1 1 1 1 209 210
= + 209d = + = =1
H210 H1 210 210 210
Sol 1: (i) Let 2 numbers be a, b
2ab ab Sol 3: sin x, sin22x, cos x sin 4x are in GP.
Given H.M. = =4⇒a+b=
a+b 2
sin42x = sin x cos x sin 4x
a+b
We have A.M. = and G.M. = ab (2 sin x cos x)4= sin x cos x 2 sin 2x cos 2x
2
(G.M.)2 = ab … (i) 16 sin4 x cos4 x = 4 sin2 x cos2 x cos 2x
2A + G2 = 27 Sin22x = cos 2x
If r1 = ½ then r1 = r2
Sol 4: a, b, c, d, e be 5 numbers ⇒ 4r12 – 2r1 – 1 = 0
a b c in AP, b c d in GP, c d e in HP
1± 5
2ce
⇒ r1 =
2b = a + c ,c2 = bd, d = , Let b be b 4
c+e
c be br, d be br2
1− 5
If r1 = then, r2 > 1.
2bre br 2 4
br2 = ⇒ br2 + er = 2e ⇒ e =
br + e 2–r 1+ 5
⇒ r1 =
a = 2b – br = b(2 – r) 4
1 + 5 1 + 5
ae = b2r2 = c2 hence a,c,e are in GP …(i) ∴ a1r1 = (1 – r1)r1 = 1 −
4 4
(2b – a)2 c2 b2r 2 br 2
(ii) ⇒ = = = =e
a a b(2 – r) 2–r 5 −1 m −n
= 3 − 5 1 + 5 = =
Hence proved. 4 4 8 p
b1 127 S
αβγ = 127 – =S–
a1 r r n–1
1 1 63.5
H.M .= 127 1 – = 32r – r – 31.5
3 r r
α+β+ γ
≥ H.M.(abc) from (v)
3
α+β+ γ 1 127
≥ 127 – = 63.5 r – 63.5
3 3 r
⇒r=2
α + β + γ ≥1 …(i)
∴ s = 64
Its given α + β + γ =1
s 1
Equality of equation (i) holds only if α = β = γ =
r n–1 2
1
i.e all the roots are
3 2n–2 = 64
3
b 1 ⇒n=8
= α3 =
a 3
b = 27a Sol 10: an+1 = an +1 for n = 1 … 97
b + a = 28a ⇒ a2 = a1 + 1
∴ a is an integer, min (a + b) = 28 ⇒ a3 = a2 + 1 = a1 + 2
⇒ a4 = a1 + 3
Sol 9: Let time taken to solve 1st problem be S time to an = a1 + (n – 1)
S 98
solve second problem will be ⇒ a1 + a2 … a98 = 4949 = [2a1 + 97.1]
r 2
S S
+… + = 63.5 …(i) 101 = 2a1 + 97 ⇒ a1 = 2
r rn–1
s s Now, we can write here Sa2k
Sn = 127 = S + … …(ii)
r r n–2
= a2 + a4 + a6 … a98 = a1 + 1 + a1 + 3 … a1 + 97
s s 49
31.5 = +… …(iii) = [2a1 + 2 + 48 × 2] = 49[a1 + 49] = 49 × 51
2 n–1
r r 2
S = 2499
= 32 …(iv)
r
S 343
127 + = 63.5 + S Sol 11: xyz = 55 or acc to a, x, y, z, b in AP/HP
rn–1 55
For a, x, y, z, b in AP
S
63.5 + =S x = a + d; y = a + 2d z = a + 3d
rn–1
b–a
S S 1 b = a + 4d ⇒ d =
– = 63.5 1 – 4
r r n
r
(a + d) (a + 2d) (a + 3d) = 55 …(i)
s 63.5 For a, x, y, z, b in HP
32 – = 63.5 –
rn r
1 1 1 1 1 1 1
63.5 S = + dH ; = + 4dH ⇒ dH = –
= 31.5 …(v) x a b a 4 b a
r rn
1 1
1 = + 2dH
S 1 – n–1 y a
r
127 =
1 1 1
1 – = + 3dH
r z a
3 . 6 2 | Sequences and Series
loga ac − logc a
(3a + b)(2a + 2b)(a + 3b) =
= 55 2
64
1 + 3 loga c
Equation (ii) can be written as =
4
3 1 1 1 1 3 55 Given that logac = 4
+ + + =
4a 4b 2a 2b 4a 4b 343
1+ 3× 4 13
∴d = =
(3b + a)(2b + 2a)(b + 3a) 55 4 4
=
2 2
64a b ab 343
Sol 14: a = b – 2
⇒ a3b3 = 343
ar2 = b + 6
⇒ ab = 7
a & b are integers a + ar 2 5
=
2ar 3
i.e a = 1 , b = 7or a = 7, b = 1
2b + 4 5
i.e a2 + b2 = 50 =
2ar 3
3
Sol 12: 10x3 – cx2 – 54x – 27 = 0 (b + 2) = ar
5
Let α, b, g be the roots
9
(b + 2)2 = (b + 6) (b – 2)
c 25
α+β+g= …(i)
10
54 ⇒ 9(b2 + 4 + 4b) = 25 (b2 + 4b – 12)
aβ + by + gα = – …(ii)
10 16b2 + 64b – 336 = 0
27
abg = …(iii) b2 + 4b – 21 = 0
10
α β & g are in harmonic progression b2 + 7b – 3b – 21 = 0
2αγ b = 7, 3 ⇒ +ve integral value of b is 3.
i.e β =
α+γ
bα + bg = 2αg ….(iv) S1–11 1
Sol 15: = … (i)
Sn–10–n 8
Putting this in equation (iii)
S10–n
β = – 3/2 this in equation (iv) =2 … (ii)
S(n–8 ) –n
–3 –3.6
(α + γ ) =
2
10
S1 – 11 = a
(r 11
–1 )S = ar 9
(r n–9
–1 )
10 – n
12
r –1 r –1
α+ γ =
5
–3 S(n – 10) – n = ar n–11
(r 11
–1 )s =a
(r n–9
–1 )
⇒ α = 3; γ = (n–8)–n
5 r –1 r –1
–3 –3
The 3 roots are 3, , Putting these values in equation (i) and equation (ii)
2 5
1 1
C 12 3 9 =
=α+β+y= − = ⇒C=9 r n–11 8
10 5 2 10
M a them a ti cs | 3.63
1 ⇒ a(a – 8) = (a – 6)2 ⇒ a = 9, r = 3
r =2⇒r=
9
29
So the number is 931
⇒ rn – 11 = 23
n–11 Sol 18: S(θ) = 1 + (1 + sin θ) cos θ
⇒ 2 9 = 23
+ (1 + sin θ + sin2θ) cos2 θ … ∞
⇒ n = 11 + 27 = 38 = 1 + cos θ + cos2 θ ...
+ sin θ (cos θ + cos2 θ ...) + sin2 θ
Sol 16: Sn = (1 + 2Tn) (1 – Tn)
1 sin θ sin2 θ cos2 θ
n = + cos θ +
Sn = [2a + (n – 1)d] 1 – cos θ 1 – cos θ 1 – cos θ
2
= [1 + 2a + (n – 1) 2d] [1 – a – (n – 1)d] 1
= [1+ sin θ cos θ + sin2θ cos2θ …]
1 – cos θ
Tn = a + (n – 1)d
1
Sn = 1 + Tn – 2Tn2 S(θ) =
(1 – sin θ cos θ)(1 – cos θ)
S1 = 1 + T1 – 2T12 = T1
1 1 1 2 2
S π = 1 – =
T1 =
2
4
1 –
1
2 ( 2 –1 )
2
S2 = 1 + T2 – 2T2 = T1 + T2 2
1
1– = 2T22 π π π
2 Sol 19: tan – x , tan , tan + x are in GP
12 12 12
2 –1 2– 2 4– 2
= T22 = = π π π
2 2 4 4 tan2 = tan – x tan + x
12 12 12
⇒ a = 4 and b = 2
π π π
sin + x sin – x cos2x – cos
a+b=6 12 12 6 π
= ⇒ = tan2
π π π 12
cos + x cos – x cos2x + cos
Sol 17: Let number be abc (a > b > c) 12 12 6
a a
a = a, b = ,c= π π π π π
r r2 cos tan2 + cos cos tan2 + 1
6 12 6 = 6 12
10a a a 10a − a cos 2x =
⇒ 100a + + – 100 – = 792 2 π 2 π
r 2 2 r 1 – tan 1 − tan
r r 12 12
99a a
⇒ 99a – = 792 ⇒ a – =8 2 π 2 π
r 2
r2 sin + cos
π 12
12
3 1
cos =1
∴ New number = 100 ( a − 4 ) + 10b + c =
6 2 π 2 π
= 2 π
cos − sin cos
2a a 12 12 6
⇒ 2b = a – 4 + c ⇒ =a–4+
r r2
2a 3 2
⇒ =a–4+a–8 = × 1
=
r 2 3
2a a a ∴ cos 2x = 1
⇒ 2a – 12 = ⇒a= +6= +8
r r r2
a
⇒r=
a–6 π/2
⇒
a
2 O π
a
⇒ =
a – 6 a – 8
3 . 6 4 | Sequences and Series
K = 4950
Sol 4: (A)
r
a
Exercise 2
a/2
Single Correct Choice Type
Circle inscribed in 1st circle = r = a/2
Sol 1: (A) A.M. = 9 + 99 … 999999999/9
a
⇒ 9[1 + 11 + 111 + …. 111111111]/9 In 2nd circle = r1 =
2 2
= 123456789 a 1
In 3rd circle = this is G.P. with common ratio
This does not contain 0 2×2 2
n–1 1–n
n–1
a 1 1 2
360
1 m ar = = r = r2 2 [A]
Sol 2: (D) Given ∑ =
n
2 2 2
k =1 k k + 1 + (k + 1) k
360
1 1 k +1 – k Assertion Reasoning Type
∑ =
k =1 k + 1 k k + k + 1 k + 1 – k
Sol 5: (D) Statement-I: If (a + b + c)3 ≤ 27 abc
360
k +1 – k 3a + 4b + 5c = 12
∑ =
k =1 k +1 k
Statement-II: ⇒ A.M. ≥ G.M. (True)
360 1 1 We Know A.M. ≥ G.M.
∑ = ∑ –
k =1 k k + 1 For three numbers a + b + c
360
1 1 1 1 1 a+b+c
∴ ∑ Tk =–
2
+
2
–
3
…- ⇒
3
≥ (abc)1/3
k =1 1 361
1 18 m ⇒ (a + b + c)3 ≥ 27 abc
=1– = = (given)
19 19 n Given (a + b + c)3 ≤ 27 abc
⇒ m + n = 18 + 19 = 37 ⇒ a = b = c & (a + b + c)3 = 27 abc
3a + 4a + 5a = 12 ⇒ a = b = c = 1
100
k k
Sol 3: (B) ∑ 4
= ∑ (k 2 + 1)2 – k 2 1
+
1
+
1
=
1
+
1
+
1
= 3 ≠ 10
k =1 k + k2 + 1 a 2 3
b b 5 1 1 1
k
= ∑ (k 2 + 1 + k)(k 2 + 1– k) So statement-II is false.
Hence, correct a3 = 1 ⇒ a = 1
r2 = 9 4b2 3c
=
r=±3 9c 2 2b
a = 3, for r = 3 b 3 3c
= ⇒b=
ar = 9 c 2 2
ar2 = 27 3b 9c
a= =
2 4
a=3
15c
sum = 39 [A] a+b= >c
4
–3 13c 3c
for r = – 3; a = a+c= 4 > 2 (b)
2
9 –27 5c 9c
ar = ; ar2 = b+c= > (a)
2 2 2 4
–3 9 27 27 –21 Hence a, b, c can form ∆
∴ sum = + – =3– =
2 2 2 2 2 log b – log a = log c – log b
2log b = log a + log c
Sol 9: (A, C, D) x3 + px2 + qx – 1
b2 = ac ie a, b, c are in GP[B]
Roots form increasing GP
a = a, b = ar, c = ar2
a
Roots be , a, ar 18 (a + b + c)2 – 18a2 – 18b2 – 18c2
r
a = 18 (2ab + 2bc + 2ac)
+ a + ar = – p
r = 36 (ab + bc +ac) > ab so A is incorrect
3 . 6 6 | Sequences and Series
An–1 + Hn–1 1 dx π
=An =
2
,Gn An–1Hn–1 , = ∫=
0
1 + x + x2 3 3
2An–1Hn–1
Hn =
An–1 + Hn–1 π n−1 n
Now, Tn > as h f(kh) > 1f(x)dx > h f(kh)
3 3 0 ∑ ∫ ∑
Clearly, G1 = G2 = G3 = ….. = = k 0= k 1
ab
Sol 7: (A) A2 is A.M. of A1 and H1 and A1 > H1 Sol 13: (C) b1 = a1, b2 = a1 + a2, b3 = a1 + a2 + a3,
b4 = a1 + a2 + a3 + a4
⇒ A1 > A2 > H1
Hence b1, b2, b3, b4 are neither in A.P. nor in G.P. nor in
A3 is A.M. of A2 and H2 and A2 > H2
H.P.
⇒ A2 > A3 > A4
::: Sol 14: (C) tn = c {n2 − (n − 1)2} = c (2n − 1)
∴ A1 > A2 > A3 >……. tn2 c2 (4n2 − 4n + 1)
⇒=
n
Sol 8: (B) As above A1 > H2 > H1, A2 > H3 > H2 4n(n + 1)(2n + 1) 4n(n + 1)
⇒ ∑ tn2 c2
= − + n
∴ H1 < H2 < H2 < …… n=1 6 2
c2n
r 1 3 2
Sol 9: (B) Here, Vr= [2r + (r – 1)(2r – 1)] =(2r – r + r)=
6
{4(n + 1)(2n + 1) − 12(n + 1) + 6}
2 2
1 c2n c2
∴ ΣV
=r
2
[2Σr3 – Σr 2 + Σr] =
3
{
4n2 + 6n + 2 − 6n − 6=
+3
3
n(4n2 − 1) }
1 n(n + 1) n(n + 1)(2n + 1) n(n + 1)
2
2 – + 100
2 2 6 2
Sol 15: (3) ∑ (k 2 − 3k + 1)Sk
k =2
n(n + 1)
= [3n(n + 1) – (2n + 1) + 3] for k = 2 |k2 - 3k + 1) Sk| = 1
12
1
= n(n + 1)(3n2 + n + 2) 100
k −1 k −1 +1
12 ∑ (k − 2)! − (k − 1)!
k =3
1 1
Sol 10: (D) Vr +1 – Vr = (r + 1)3 – r3 – [(r + 1)2 – r 2 ] + (1) 100
2 2 1 1 1 1
= 3r2 + 2r – 1
∑ (k − 3)! + (k − 2)! − (k − 2)! − (k − 1)!
k =3
∴ Tr = 3r + 2r – 1= (r + 1) (3r – 1)
2
100
1 1
Which is a composite number. ∑ (k − 3)! − (k − 1)!
k =3
3 . 6 8 | Sequences and Series
1 1 1 1 1 1 1 1 1 (n−1)
S =+
1 1 − + − + − + − + − + =
2! 1! 3! 2! 4! 3! 5! 4! 6!
∑ ((4r + 4)2 + (4r + 3)2 − (4r + 2)2 − (4r + 1)2 )
r =0
1 1 1 1 1 1 1 1
.... − + − + − + − (n−1) (n−1)
94! 96! 95! 97! 96! 98! 97! 99!
=
∑ (2(8r + 6) + 2(8r=
+ 4)) ∑ (32r + 20)
1 1 r =0 r =0
2
=− −
98! 99!
1056 for n = 8
= 16(n – 1)n + 20n = 4n(4n + 1) =
1002 1 1 1332 for n = 9
E
∴= +3− −
100! 98! 99.98!
b c
1002 100 1002 100 Sol 20: (6) = = (int eger)
= +3− = +3− = 3 a b
100! 99! 100.99! 99!
b2
b2 = ac ⇒ c =
Sol 16: (0) ak = 2ak−1 − ak−2 ⇒ a1, a2, …, a11 are in A.P. a
a+b+c
a2 + a22 + .... + a11
2
11a2 + 35 × 11d2 + 10ad = b+2
∴ 1 = = 90 3
11 11
a + b + c = 3b + 6 ⇒ a − 2b + c = 6
⇒ 225 + 35d2 + 150d = 90
35d2 + 150d + 135 = 0 ⇒ d = − 3, − 9/7 b2 2b b2 6
a − 2b + =6 ⇒ 1 − + =
27 a a a2 a
Given a2 < ∴ d = − 3 and d ≠ − 9/7 ⇒
2 2
b 6
a1 + a2 + .... + a11 11 − 1 = ⇒ a = 6only
= 30 − 10 × 3
= 0
11 2 a a
Sol 17: (B) ax2 + bx + c = 0 ⇒ x2 + 6x – 7 = 0 Sol 21: (9) Let seventh term be ‘a’ and common
difference be ‘d’
⇒ α = 1, β = – 7
n n
S7 6
∞ ∞ Given, = ⇒ a = 15d
1 1 1 1 S11 11
∑ α + β = ∑ 1− 7 = 7
=
n 0= n 0 Hence, 130 < 15d < 140
⇒ d=9
Sol 18: (D) Corresponding A.P.
Sol 22: (B) log (b2) - log (b1) = log (2)
1 1
,.......................... (20th term) b2
5 25 ⇒ =2 ⇒ b1, b2, …. are in G.P. with common ratio 2
b1
1 1 1 −4 4
= + 19d ⇒d= = − ∴ t = b1 + 2b1 + …… + 250 b1 = b1 (251 - 1)
25 5 19 25 19 × 25
S = a1 + a2 + ….. + a51
an < 0
51 51 51
= (a + a ) = (b + b ) = b (1 + 250 )
1 4 2 1 51 2 1 2 2 1
− × (n − 1) < 0
5 19 × 25 51 53
− t b1 + 51 × 249 − 251 =
S= + 1 b1 + 249 × 47 ⇒ S > t
19 × 5 2 2
< n−1
4
⇒S>t
n > 24.75 b101 = 2100 b1
4n k(k +1) a101 = a1 + 100 d = 2 (a1 + 50d) - a1
Sol 19: (A, D) S
=n ∑ (−1) 2 k2
= 2a51 - a1 = 2b51 - b1 = (2 × 251 - 1) b1 = (251 - 1) b1
k =1
∴ b101 > a101
2017-18 100 &
op kers
Class 11 T
By E ran culty
-JE Fa r
IIT enior emie .
S fP r es
o titut
Ins
MATHEMATICS
FOR JEE MAIN & ADVANCED
SECOND
EDITION
Exhaustive Theory
(Now Revised)
Formula Sheet
9000+ Problems
based on latest JEE pattern
PlancEssential
Questions recommended for revision
4. BINOMIAL THEOREM
MATHEMATICAL INDUCTION
The technique of Induction is used to prove mathematical theorems. A variety of statements can be proved using
this method. Mathematically, if we show that a statement is true for some integer value, say n = 0, and then we
prove that the statement is true for some integer k+1 if it is true for the integer k (k is greater than or equal to 0),
then we can conclude that it is true for all integers greater than or equal to 0.
The solution in mathematical induction consists of the following steps:
Step 1: Write the statement to be proved as P(n) where n is the variable.
Step 2: Show that P(n) is true for the starting value of n equal to 0(say).
Step 3: Assuming that P(k) is true for some k greater than the starting value of n, prove that P(k+1) is also true.
Step 4: Once P(k+1) has been proved to be true, we say that the statement is true for all values of the variable.
The following illustrations will help to understand the technique better.
Sol: Clearly, the statement P(n) is true for n = 1. Assuming P(k) to be true, add (k+1) on both sides of the statement.
P(n):1+2+3+...+n=n(n+1)/2
Clearly, P(1) is true as 1=1.2/2.
Let P(k) be true. That is, let 1+2+3+...+k be equal to k(k+1)/2
Now, we have to show that P(k+1) is true, or that
1+2+3+...+(k+1)=(k+1)(k+2)/2.
L.H.S = 1+2+3+...+(k+1)
= 1+2+3+...+k+(k+1) = k(k+1)/2 + (k+1) (As P(k) is true)
= (k+1) (k/2+1) = (k+1)(k+2)/2
= R.H.S
Illustration 2: Prove that (n+1)! >2n for all n>1. (JEE MAIN)
Sol: For n = 2, the given statement is true. Now assume the statement to be true for n = m and multiply (m+2) on
both sides.
4 . 2 | Binomial Theorem
Illustration 3: Prove that n2 + n is even for all natural numbers n. (JEE MAIN)
Sol: Consider P(n) = n2 + n. It can written as a product of two consecutive natural numbers. Use this fact to prove
the question.
Consider that P(n) n2 + n is even, P(1) is true as 12 + 1= 2 is an even number.
Consider P(k) be true,
To prove : P(k + 1) is true.
P(k + 1) states that (k + 1)2 + (k + 1) is even.
Now, (k + 1)2 + (k + 1) = k2 + 2k + 1 + k + 1 = k2 + k + 2k + 2
As P(k) is true, hence k2 + k is an even number and can be written as 2 λ , where λ is sum of natural number.
∴ 2 λ + 2k + 2 ⇒ 2( λ + k + 1) = a multiple of 2.
Thus, (k + 1)2 + (k + 1) is an even number.
Hence, P(n) is true for all n, where n is a natural number.
Illustration 4: Prove that exactly one among n+10, n+12 and n+14 is divisible by 3, considering n is always an
natural number. (JEE MAIN)
Without the loss of generality consider for n=m, m+10 was divisible by 3
Therefore, m+10 = 3k
m+12 = 3k+2
m+14 = 3k+4
We need to prove that for n=m+1 , exactly one among them is divisible by 3. Putting m+1 in place of n, we get
(m+1)+10 = m+11 = 3k + 1 (not divisible by 3)
(m+1)+12 = m+13 = 3k+3 = 3(k+1) (divisible by 3)
(m+1)+14 = m+15 = 3k+5 (not divisible by 3)
Therefore, for n=m+1 also exactly one among the three, n+10, n+12 and n+14 is divisible by 3.
Similarly we can prove that exactly one among three of these is divisible by 3 by considering cases when n+12 = 3k
and n+14 = 3k.
BINOMIAL THEOREM
1.1 Introduction
Consider two numbers a and b, then
(a + b)
2
=a2 + 2ab + b2
(a + b)
3
=( a + b )( a + b )
2
(
=( a + b ) a2 + 2ab + b2 ) =a3 + 3a2b + 3ab2 + b3
(a + b) =( a + b ) ( a + b ) ( )( )
4 2 2
= a2 + 2ab + b2 a2 + 2ab + b2 =a4 + 4a3b + 6a2b2 + 4ab3 + b 4
As the power increases, the expansion becomes lengthy, difficult to remember and tedious to calculate. A binomial
expression that has been raised to a very large power (or degree), can be easily calculated with the help of Binomial
Theorem.
( x +=
a)
n n
C0 xna0 + n C1 xn−1a1 + n C2 xn−2a2 + ...... + n Cr xn−r ar + ...... + n Cn−1 x1an−1 + n Cn x0 an
n
n!
i.e. ( x + a) =
n
∑ n Cr xn−r ar where n Cr = r! (n − r ) !
0
and the co-efficients nC0, nC1, nC2, ……………… and nCn are known as binomial coefficient.
4 . 4 | Binomial Theorem
PLANCESS CONCEPTS
n
(a) The total number of terms in the expansion of (x + a)n = ∑ n Cr xn−r ar , is (n + 1).
r =0
(b) The sum of the indices of x and a in each term is n.
(c) n
C0, nC1, nC2, . . . , nCn are called binomial coefficients and also represented by C0, C1, C2 and so on.
n n
(i) Cx= C y ⇒ x= y or x + y= n (ii) n Cr = n Cn−r
n +1
(iii) n Cr + n Cr −1 = n
Cr (iv) = Cr n / (n − r ) . n−1 Cr
( x − 2) = 5 C0 x5 + 5 C1 x 4 ( −2 ) + 5 C2 x3 ( −2 ) + 5 C3 x2 ( −2 ) + 5 C 4 x ( −2 ) + 5 C5 ( −2 )
5 1 2 3 4 5
(i)
( x=
− a) C0 xna0 − n C1 xn−1a1 + n C2 xn−2 .a2 − ..... + ( −1 ) Cr xn−r ar + ...... + ( −1 )
n n r n n n
Cn x0 an
n
i.e. ( x − a) = ∑ ( −1 )
n r n
Cr xn−r ar
r =0
Therefore, the terms in (x – a)n are alternatively positive and negative, and the sign of the last term is positive or
negative depending on whether n is even or odd.
(1 + x ) =
n n
C0 + n C1 x + n C2 x2 + ...... + n Cr xr + .... + n Cn xn
n
⇒ (1 + x ) =
n
∑ n Cr xr
r =0
(1 − x )= C0 − n C1 x + n C2 x2 − n C3 x3 + ..... + ( −1 ) Cr xr + ..... + ( −1 )
n n r n n n
Cn xn
D-5 From the above expansions, we can also deduce the following
( x + a) + (=
x − a)
n n
2 n C0 xna0 + n C2 xn−2a2 + .....
and ( x + a) −=
( x − a) 2 n C1 xn−1a1 + n C3 xn−3 a3 + .....
n n
PLANCESS CONCEPTS
(1 + x )
n n
= C0 + n C1 x + n C2 x2 + ....... + n Cr xr + ...... + n Cn xn
Also, (1 + x )=
n n
C0 xn + n C1 xn−1 + n C2 xn−2 + ....... + n Cr xn−r + ...... + n Cn−1 x + n Cn
Let us represent the binomial coefficients n C0 , n C1 , n C2 ,......., n Cn−1 , n Cn by C0 ,C1 ,C2 ,.......,Cn−1 ,Cn respectively. Then
the above expansions become
n
(1 + x ) =C0 + C1 x + C2 x2 + ...... + Cn xn i.e. (1 + x ) =∑ Cr xr
n n
r =0
n
Also, (1 + x ) = C0 xn + C1 xn−1 + C2 xn−2 + ...... + Cr xn−r + ...... + Cn−1 x + Cn i.e. (1 + x ) =
n n
∑ Cr xn−r
r =0
The binomial coefficients C0, C1, C2,……..Cn–1, and Cn posses the following properties:
Property-I In the expansion of (1 + x)n, the coefficients of terms equidistant from the beginning and the end
are equal.
Property-II The sum of the binomial coefficients in the expansion of (1 + x)n is 2n.
n
i.e. C0 + C1 + C2 + ……. + Cn = 2n or, ∑ n Cr = 2n .
r =0
4 . 6 | Binomial Theorem
Property-III The sum of the coefficient of the odd terms in the expansion of (1 + x)n is equal to the sum of the
coefficient of the even terms and each is equal to 2n–1.
i.e. C0 + C2 + C 4 + ........ = C1 + C3 + C5 + ....... = 2n−1
n n n n − 1 n− 2
Property-IV Cr =⋅ n−1 Cr −1 =⋅ ⋅ Cr −2 and so on.
r r r −1
C0 − C1 + C2 − C3 + C 4 − ...... + ( −1 ) Cn =0
n
Property-V
n
∑ ( −1 )
r n
i.e. Cr =
0
r =0
PLANCESS CONCEPTS
n n+1
Cr Cr +1
(a) (n+1)
Cr = Cr + Cr – 1
n n
(b) r Cr = n
n n –1
Cr – 1 (c) r + 1 =
n+1
(
(d) When n is even, ( x + a) + ( x − a) = 2 xn + n C2 xn−2a2 + n C 4 xn− 4 a4 + ..... + n Cnan
n n
)
(
When n is odd, ( x + a) + ( x − a) = 2 xn + n C2 xn−2a2 + ..... + n Cn−1 x an−1
n n
)
When n is even ( x + a) − (=x − a) 2( C x )
n n n n−1
1 a + n C3 xn−3a3 + ..... + n Cn−1 x an−1
C1 C2 C3 Cn
1
(iii) C0 − + − + ......... + ( −1)n =
2 3 4 n+1 n+1
Sol: By using properties of binomial coefficients and methods of summation, differentiation, and integration we
can easily prove given equations.
(i) (1 + x)n = C0 + C1 x + C2 x2 + .......... + Cn xn
Method 1: By Summations
rth term in the series is given by (r + 1 ) . n Cr
n
Therefore, L.H.S = n C0 + 2.n C1 + 3.n C2 + ........ + (n + 1). n C
=n ∑ (r + 1). n Cr
r =0
M a them a tics | 4.7
n n n n
= ∑ r. n Cr + ∑ n Cr = n∑ n−1 Cr −1 + ∑ n Cr = n.2n−1 + 2=
n
2n−1 (n + 2) = R.H.S
=r 0=r 0 =r 0 =r 0
Method 2: By Differentiation
(1 + x)n = C0 + C1 x + C2 x2 + ....... + Cn xn
3
Multiplying x on both sides, x(1 + x)n = C0 x + C1 x2 + C2 x + ....... + Cn xn+1
C1 C2 C3 Cn 1
(iii) C0 − + − + ......... + ( −1)n =
2 3 4 n+1 n+1
Method 1: By Summations
n
Cr
rth term in the series is given by ( −1 ) .
r
r +1
n
C1 C2 C3 Cn n C
+ ...... + ( −1 ) .
n
∑ ( −1 ) . r + 1r
r
Therefore, L.H.S. = C0 − + − =
2 3 4 n+1 r =0
1 n n+1 n 1 n+1
C3 − ........ + ( −1 ) . n+1 Cn+1
n
( −1 ) n+1 Cr +1
r
= ∑ using
n+1
=
. Cr = Cr +1 C1 − n+1 C2 + n+1
n + 1 r =0 r +1 n + 1
1 n+1
C1 − n+1 C2 + ........ + ( −1 ) . n+1 Cn+1 + C0
n+1 n n+1
= − C0 +
n + 1
1
as − n+1 C0 + C1 − n+1 C2 + ........ + ( −1 ) . n+1 Cn+1 =0 = R.H.S.
n+1 n
=
n+1
Method 2: By Integration
(1 + x)n = C0 + C1x + C2x2 + ……… + Cnxn.
On integrating both sides within the limits –1 to 0, we have
0 0
∫ (1 ) dx ∫ ( C0 + )
n
+ x= C1 x + C2 x2 + ……… + Cn xn dx
−1 −1
0
1 + x n+1
( ) = x2 x3 xn+1
0
⇒ C0 x + C1 + C2 + ...... + Cn
n+1 2 3 n + 1
−1 −1
1 C C n+1 Cn C1 C2 n Cn 1
⇒ − 0 = 0 − −C0 + 1 − 2 + ...... + ( −1 ) ⇒ C0 − + + ...... + ( −1 ) =
n+1 2 3 n + 1 2 3 n+1 n+1
2n 2n
(ii) C0 C2 + C1C3 + C2C 4 + ...... + Cn−2Cn = Cn−2 or Cn+ 2
Sol: In the expansion of (1+x)2n, (i) and (ii) can be proved by comparing the coefficients of xn and xn-2 respectively.
The third equation can be proved by two methods - the method of summation and the methods of differentiation.
(1 + x )
2n
(
= C0 + C1 x + C2 x2 + ...... + Cn xn ) (C x 0
n
+ C1 xn−1 + ....... + Cn x0 ) …. (iii)
Method 1: By Summation
rth term in the series is given by ( 2r + 1 ) n Cr2
n
L.H.S. = 1.C20 + 3.C12 + 5.C22 + ...... + ( 2n + 1 ) Cn2
= ∑ ( 2r + 1 ) n Cr2
r =0
n n n
∑ 2.r. ( n Cr ) + ∑ ( n Cr )= 2∑ .n. n−1 Cr −1 n Cr + 2n Cn
2 2
=
=r 0 =r 0=r 1
(1 + x )
n n
= C0 + n C1 x + n C2 x2 + ....... + n Cn xn …. (i)
( x + 1=
)
n−1 n−1
C0 xn−1 + n−1
C1 xn−2 + ....... + n−1
Cn−1 x0 …. (ii)
Method 2: By Differentiation
(1 + x )
n
2
= C0 + C1 x2 + C2 x 4 + C3 x6 + ...... + Cn x2n
( )
n
x 1 + x2 = C0 x + C1 x3 + C2 x5 + ...... + Cn x2n+1
M a them a tics | 4.9
( ) ( )
n−1 n
x.n 1 + x2 .2x + 1 + x2 = C0 + 3.C1 x2 + 5.C2 x 4 + ...... + ( 2n + 1 ) Cn x2n …. (i)
(x )
n
2
+ 1 = C0 x2n + C1 x2n−2 + C2 x2n− 4 + ....... + Cn …. (ii)
( ) ( ) = (C )( )
2n−1 2n
2nx2 1 + x2 + 1 + x2 0 + 3C1 x2 + 5C2 x 4 + ..... + ( 2n + 1 ) Cn x2n C0 x2n + C1 x2n−2 + ...... + Cn
Illustration 8: If (1 + x ) = C0 + C1 x + C2 x2 + ...... + Cn xn ,
n
2n!
Prove that C0 Cr + C1Cr +1 + C2Cr + 2 + ........ + Cn−r Cn = (JEE MAIN)
(n − r ) ! (n + r ) !
Sol: Clearly the differences of lower suffixes of binomial coefficients in each term is r.
By using properties of binomial coefficients we can easily prove given equations.
(1 + x )
n
Given =C0 + C1 x + C2 x2 + ........ + Cn−r xn−r + ......... + Cn xn …. (i)
( x + 1)
2n
= (C 0 + C1 x + C2 x2 + ..... + Cn−r xn−r + ......... + Cn xn )
(
× C0 xn + C1 xn−1 + C2 xn−2 + ..... + Cr xn−r + Cr +1 xn−r −1 + Cr + 2 xn−r −2 + ...... + Cn ) …. (iii)
2n 2n!
Now coefficient of xn–r on L.H.S. of (iii) = Cn−r =
(n − r ) ! (n + r ) !
and coefficient of xn–r on R.H.S. of (iii) = C0 Cr + C1Cr +1 + C2Cr + 2 + ........ + Cn−r Cn
Hence, Proved.
n
1 + x 2 − 1= n C 1 + x 2n − n C 1 + x 2n−2 + n C 1 + x 2n− 4 + ...
( ) 0( ) 1( ) 2( ) …. (i)
∴ Coefficient of xr in xn (2 + x)n
= Coefficient of xr–n in (2 + x)n = nCr–n 22n–r if r > n
= 0 if r < n (Since lower suffix cannot be negative)
But (i) is an identity, therefore coefficient of xr in R.H.S. = coefficient of xr in L.H.S.
= 0 if r < n.
n
= Coefficient of xn in (1 + x ) (1 + x ) − 1
n
= Coefficient of xn in (1 + x ) ( x )
n n
We have, ( x +=
a)
n n
C0 xna0 + n C1 xn−1 a1 + n C2 xn−2 x2 + ...... + n Cr xn−r ar + ...... + n Cn x0 an
Illustration 11: The number of dissimilar terms in the expansion of (1 – 3x + 3x2 – x3)20 is (JEE MAIN)
Sol: As we know that number of dissimilar terms in the expansion of (1 – x)n is n+1. Rewrite the given expression
in the form of (1 – x)n.
M a them a ti cs | 4.11
9
4x 5
Illustration 12: Find (i) 28 term of (5x + 8y) (ii) 7 term of
th
−
30 th
(JEE MAIN)
5 2x
Sol: Here in this problem, by using Tr +1 = n Cr xn−r ar we can easily obtain (r+1)th term of given expansion.
(i) 28th term of (5x + 8y)30
30!
( 5x ) . ( 8y )
3 27
T28 = T27 + 1 = 30C27 (5x)30–27 (8y)27 =
3!.27!
9
4x 5
(ii) 7th term of −
5 2x
9 −6 6 3 6
4x 5 9! 4x 5 10500
T7 = T6+1 = 9 C6 = − =
5
2x 3!6! 5
2x x3
Illustration 13: Find the number of rational terms in the expansion of (91/4 + 81/6)1000. (JEE ADVANCED)
Sol: In this problem, by using Tr +1 = n Cr xn−r ar we can easily obtain (r+1)th term of given expansion and after that by
using the conditions of rational number we can obtain number of rational terms.
The general term in the expansion of (91/4 + 81/6)1000 is
1000 −r
1 1000 −r r
( )
r
1000
Tr +1 Cr 9 4
= 81/6 1000
Cr 3 2 22
1000 − r r
Tr +1 will be rational if the power of 3 and 2 are integers. It means and must be integers.
2 2
Therefore the possible set of values of r is {0, 2, 4... … 1000}. Hence, number of rational terms is 501.
(a) If n is even, then the number of terms in the expansion i.e. (n + 1) is odd, therefore, there will be only one
th th
n+2 n
middle term which is term i.e. + 1 term.
2 2
th n n
n
So middle term = + 1 term i.e. T n
= n C n x 2 a2
2 +1
2
2
(b) If n is odd, then the number of terms in the expansion i.e. (n+1) is even, therefore there will be two middle
terms which are
th th n+1 n−1 n−1 n+1
n+1 n+3
= and term i.e. T n+1 = n C n−1 x 2 a 2 and T n+3 = n C n+1 x 2 a 2
2 2
2 2 2 2
4 . 1 2 | Binomial Theorem
PLANCESS CONCEPTS
•• When there are two middle terms in the expansion then their binomial coefficients are equal.
•• Binomial coefficient of middle term is the greatest Binomial coefficient.
Neeraj Toshniwal (JEE 2009, AIR 21)
14 9
x2 a3
Illustration 14: Find the middle term(s) in the expansion of (i) 1 − (ii) 3a − (JEE MAIN)
2 6
th th th
n n+1 n+3
Sol: By using appropriate formula of finding middle term(s) i.e. + 1 when n is even and and
2 2 2
when n is odd, we can obtain the middle terms of given expansion.
14 th
x2 14
(i) 1 − Since, n is even, therefore middle term is + 1 term.
2 2
7
14
x2 429 14
∴ T8 = Cc7 − =
14
− x
2 16
9
a3
(ii) 3a −
6
th th
9 +1 9 +1
Since, n is odd therefore, the middle terms are and + 1 .
2 2
4 5
a3 189 17 a3 21
C 4 ( 3a) C5 ( 3a)
9 9−4 9 9 −5
∴ T5
= −= a and T6 = − = − a19 .
6 8 6 16
9
4 3
Illustration 15: The term independent of x in the expansion of x2 − is (JEE MAIN)
3 2x
nα
Sol: By using the result proved above i.e. r = , we can obtain the term independent of x. Here, α and β are
α+β
n
1
obtained by comparing given expansion to x α ± .
xβ
9 n
4 3 1
On comparing x2 − with x α ± , we get α= 2, β= 1, n= 9
3 2x xβ
M a them a ti cs | 4.13
9 (2)
i.e. r = = 6 ∴ (6 + 1) = 7th term is independent of x.
2+1
15
2
Illustration 16: The ratio of the coefficient of x15 to the term independent of x in x2 + is (JEE MAIN)
x
Sol: Here in this problem, by using standard formulas of finding general term and term independent of x we can
obtain the required ratio.
r
( ) 2
15 −r
15 2 15 30 −3r r
General term in the expansion is Tr +1 = Cr x i.e., Cr x .2
x
For x15, 30 – 3r = 15 ⇒ 3r = 15 ⇒ r=5
5
( ) 2
15 −5
15
∴ T6 = T5+1 = C5 x2 15 15 5
i.e., C5 x .2
x
∴ Coefficient of x15 is 15C5 25 (r = 5)
( ) 2
15 −10
15
T11 T=
∴= 10 +1 C10 x2 15
i.e., C10 2
10
x
∴ Coefficient of constant term is 15C10210.
( ) 1
9 −r −
= 9 Cr ( −1 ) x
r
Cr ( −1 ) x
9 6 9 r 6
=Tr +1 Cr x − 3 6 3=
x
9 − 3r
⇒ 0 ⇒r =
= 3 ∴ T4 =T3+1= – 9 C3
6
Illustration 18: If the second, third and fourth terms in the expansion of (b+a)n are 135, 30 and 10/3 respectively,
then n is equal to (JEE MAIN)
Sol: In this problem, by using the formula of finding general term we will get the equation of given terms and by
taking ratios of these terms we can get the value of n.
T2 = nC1 abn–1 = 135 ...(i)
T3 = nC2 a2bn–2 = 30 ...(ii)
10
T4 = nC3 a3bn–3 = ...(iii)
3
On dividing (i) by (ii), we get
n
C1abn−1 135 n b 9
= ⇒ = …(iv)
n 2 n− 2
C2 a b 30 n(n − 1) a 2
2
b 9
∴ =
a 4
(n − 1) ...(v)
4 . 1 4 | Binomial Theorem
Illustration 19: If a, b, c and d are the coefficients of any four consecutive terms in the expansion of (1+x)n, n being
a c 2b
positive integer, show that + = (JEE MAIN)
a + b c + d b + c
∴
a
=
n
Cr −2
=
n!
×
(r − 1)! (n − r + 2)=! r − 1
a+b n+1
Cr −1 (r − 2 ) ! (n − r + 2 ) ! (n + 1 ) ! n+1
b
n
Cr −1 n! r! (n − r + 1 ) ! r
= = × =
b+c n +1
Cr (r − 1 ) ! (n − r + 2 ) ! (n + 1 ) ! n + 1
c
n
=r =
C n!
×
(r + 1 ) ! (n − r ) ! =
r +1
c + d n+1 C
r +1
n! ( )
n − r ! ( )
n + 1 ! n +1
a c r −1 r +1 2r r 2b
∴ + = + = = 2 =
a+b c + d n+1 n+1 n+1 n+1 b + c
∴ Tr (E ) =
Tn−r + 2 (B )
Illustration 20: The 4th term from the end in the expansion of (2x – 1/x2)10 is (JEE MAIN)
Sol: By using Tr (E) = Tn-r+2(B) we will get the fourth term from the end in the given expansion.
7
3 1
Required term = T10 − 4 + 2 = T8 = 10
C7 ( 2x ) − = – 960 x–11
2
x
M a them a ti cs | 4.15
Tr = n Cr −1 xn−r +1ar −1 .
∴
Tr +1
= r
n
C xn−r ar
=
n!
x
(r − 1 ) ! (n − r + 1 ) ! . a =
n−r +1 a
.
Tr n
Cr −1 xn−r +1ar −1 (n − r ) !r! n! x r x
Tr +1 n−r +1 a n + 1 a
Now, Tr +1 > = < Tr ⇒ > = < 1 ⇒ . >=< 1 ⇒ − 1 > = < 1
Tr r x r
x
n+1 n+1
Case-I: When is an integer Let = m , Then, from (i), we have
x x
1+ 1+
a a
Tr +1 > Tr , for =
r 1,2,3,.....(m − 1) ….(ii)
Tr +1 T=
= r , for r m …(iii)
Tm + 1 = Tm [From (iii)]
and, Tm+ 2 < Tm+1 , Tm+3 < Tm+ 2 , Tn+1 < Tn [From (iv)]
⇒ T1 < T2 < ..... < Tm−1 < Tm= Tm+1 > Tm+ 2 .... > Tn
This shows that mth and (m + 1)th terms are greatest terms.
n + 1
Case-II: When = m . Then, from (i), we have
1 + ax
Tr +1 > Tr for r =
1,2,.....,m ….(v)
and, Tm+ 2 < Tm+1 , Tm+3 < Tm+ 2 ,......, Tn+1 < Tn [From (vi)]
⇒ T1 < T2 < T3 < ..... < Tm < Tm+1 > Tm+ 2 > Tm+3 ...... < Tn+1
4 . 1 6 | Binomial Theorem
Following algorithm may be used to find the greatest term in a binomial expansion.
Step IV: Simplify the inequality obtained in step III, and write it in the form of either r < m or r > m.
Step V: If m is an integer, then mth and (m+1)th terms are the greatest terms and they are equal.
If m is not an integer, then ([m]+1)th term is the greatest term, where [m] means the integral part of m.
nC
r = ×
n! (r + 1 ) ! (n − r − 1 ) ! =
r +1
….(i)
nC (n − r )!r! n! n−r
r +1
n n n
Now, for 0 ≤ r ≤ − 1 ⇒ 1 ≤ r + 1 ≤ and + 1 < n − r ≤ n
2 2 2
n
r +1 Cr
⇒ < 1 [Using (i)] ⇒ < 1 ⇒ nCr < n Cr +1
n−r n
Cr +1
n
Putting
= r 0,1,2,......, − 1 , we get n C0 < n C1 , n C1 < n C2 , n C2 < n C3 ..... < n C n < n C n
2 2
−1
2
∴ n=
C0 n
Cn , n=
C1 n
Cn−1 , n=
C2 n
Cn−2 ,......, n C n < nCn
−1
2 2
From (ii) and (iii), we refer that the maximum value of nCr is nCn/2.
n
r +1 Cr n
⇒ <1⇒ < 1 [Using (i)] ⇒ Cr < nCr +1
n−1 n
Cr +1
Putting r = 0, 1, 2, ……., n − 3
2
We get n C0 < n C1 , n C1 < n C2 , n C2 < n C3 ,....., < n C n−3 < n C n−1 =
n
C n+1
2 2 2
n n n n n n n
⇒ C0 < C1 < C2 < C3 < ..... < C n−3 < C n−1 =C n−1
2 2 2 ...(ii)
Since nCn–r = nCr. Therefore,
∴ n=
C0 n
Cn , n=
C1 n
Cn−1 , n=
C2 n
Cn−2 ,....., n C n=
−1
n
C n+1 …(iii)
2 2
From (ii) and (iii), it follows that the maximum value of nCr is n C n−1 = n C n+1
2 2
1
Illustration 21: Find the numerically greatest term in the expansion of (3 – 4x)15, when x = . (JEE MAIN)
4
Sol: Follow the algorithm mentioned above.
Let rth and (r + 1) th be two consecutive terms in the expansion of (3 – 4x)15
Tr +1 > Tr
15 −(r −1 )
( ) ( −4x )
15 r r −1
Cr 315−r −4x > 15
Cr −1 3
⇒ 4r < 16 ⇒r<4
Illustration 22: Show that 72n + 7 is divisible by 8, where n is a positive integer. (JEE MAIN)
Sol: Write 72n + 7 in the form of 8λ + c, where c is a constant. If c = 0 then we can conclude that 72n + 7 is divisible
by 8.
2n
72n + 7 = (8 - 1)2n + 7 = C0 82n − 2n
C1 .82n−1 + 2n
C2 .82n−2 − ...... + 2n
C2n + 7
Illustration 23: Prove that 1399 ‒ 1957 is divisible by 162. (JEE ADVANCED)
Sol: Reduce 1399 − 1957 into the form of 162λ + C using binomial expansion and If C = 0 then 1399 − 1957 is
divisible by 162.
Let the given number be called S. Hence, S = 1399 –1957 = (1 + 3×4)99 – (1 + 9×2)57
{
S = 1+ 99
C1 . ( 3 × 4 ) + 99
C2 . ( 3 × 4 ) +
2 99
C3 . ( 3 × 4 ) + ...... +
3 99
C99 . ( 3 × 4 )
99
}
– {1 + C1 . ( 9 × 2 ) + 57 C2 . ( 9 × 2 ) + 57 C3 . ( 9 × 2 ) + ...... + 57 C57 . ( 9 × 2 ) }
57 2 3 57
S = 1+ { 99
( ) } {
C1 . ( 3 × 4 ) + 34 × 2 k1 − 1 + 57 C1 . ( 9 × 2 ) + 34 × 2 k 2 ( ) }
All terms like { 99
C1 . ( 3 × 4 ) , 99 C2 . ( 3 × 4 ) ,......, 99 C99 . ( 3 × 4 )
2 3 99
} and
{ 57
C2 . ( 9 × 2 ) , 57 C3 . ( 9 × 2 ) ,......, 57 C57 . ( 9 × 2 )
2 3 57
} have a common factor of (3 .2 = 162) . 4
Hence they can be written as (34.2) k1 and (34.2) k2 respectively, where k1 and k2 are integers.
Therefore, S = 1 + 99
C0 . ( 3 × 4 ) − 1 − 57 C1 . ( 9 × 2 ) + (162 )(k1 − k 2 )
{
= (1188 − 1026 ) + 162 × (k1 − k 2 ) = (162 × some integer) }
Hence the given number S is exactly divisible by 162.
Illustration 24: What is the remainder when 52015 is divisible by 13. (JEE MAIN)
Sol: In this problem, we can obtain required remainder by reducing 52015 into the form of 13 λ +a, where λ and a
are integers.
52015 = 5.52014 = 5.(25)1007
= 13 (k ) + 52 + 8 = 13 × (some integer) + 8.
Illustration 25: Find the last two digits of the number (13)10. (JEE MAIN)
Sol: Write (13)10 in the form of ( x − 1 ) , such that x is a multiple of 10. Then using expansion formula we will get
n
(170 −=
1) C0 (170 ) − 5 C1 . (170 ) + ....... + 5 C 4 (170 ) − 5 C5 (170 )
5 5 5 4 1 0
(13)10 = (169)
= 5
M a them a ti cs | 4.19
Illustration 26: Find the last three digits of 13256. (JEE MAIN)
(13 =
)
128
(170 − 1)
128
132
Now, = 2
C0 (170 ) − 128 C1 . (170 ) + 128 C2 . (170 ) − ... + 128 C126 (170 ) − 128 C127 (170 ) + 1
128 128 127 126 2
=
Sol: By reducing (1.01)1000000 into the form of (1 + 0.01 ) and solve it by using expansion formula we can obtain the
n
value of (1.01)1000000 .
(1.01)1000000 = (1 + 0.01)1000000
1 + 1000000 C1 ( 0.01 ) + 1000000 C2 ( 0.01 ) + 1000000 C3 ( 0.01 ) + ...
2 3
=
1 + 1000000 × (0.01) + some positive terms
=
= 1 + 10000 + some positive terms
Hence 10,000 < (1.01)1000000.
5. MULTINOMIAL THEOREM
Using binomial theorem, we have
n
( x + a)
n
= ∑ n Cr xn−r ar , n∈N
r =0
n
n! n! s r
= ∑ (n − r ) !r! xn−r ar = ∑ xa, wheres= n − r
=r 0 =r +s n r!s!
n n k
( x1 =
+ x 2 + x3 ) ( x2 + x3 ) ∑ (n− n! k!
n k
∑ n Ck x1n−k= k)! k!
x1n−k ∑
xk −p xp
(k − p)! p! 2 3
=k 0 =k 0=p 0
n k
n! n!
= ∑ ∑ xn−k xk −p xp
(n− k)! (k − p)! p! 1 2 3
= ∑ xr xq xp where, k - p = q, n - k = r.
r! q! p! 1 2 3
=k 0=p 0 p + q+r =n
4 . 2 0 | Binomial Theorem
n!
Therefore, general term in the expansion of ( x1 + x2 + ..... + xk ) is
n r r r r
x 1 x 2 x 3 .......xkk
r1 !r2 !r3 !.....rk ! 1 2 3
The number of terms is equal to the number of non-negative integral solution of the equation r1 + r2 +………..+ rk = n,
because each solution of this equation gives a term in the above expansion. The number of such solutions is n + k –1Ck –1.
PLANCESS CONCEPTS
PLANCESS CONCEPTS
Illustration 28: If x is very large and n is a negative integer or a proper fraction, then an approximate value of
n
1 + x
is equal to___________ (JEE MAIN)
x
1
Sol: Since x is very large therefore will be very small. Neglect the terms containing three and higher powers of
x n
1 1 + x
in the expansion to obtain the approximate value of .
x x
M a them a ti cs | 4.21
n n (n − 1 ) 1
n 2
1
1 + = 1 + + + ...... Since x is very large, we can ignore terms after the 2nd term.
x x 1.2 x
(1 − 3x ) + (1 − x )
1/2 5/3
Illustration 29: If is approximately equal to a + bx for small values of x, then (a, b) is equals
to. 4−x (JEE MAIN)
(1 − 3x ) + (1 − x )
1/2 5/3
1 1 11 5 521
1 + ( −3x ) + − ( −3x ) + ...... + 1 + ( −x ) + ( −x ) + ......
2 2
(1 − 3x ) + (1 − x )
1/2 5/3
2 2 2 2 3 3 3 2
=
1/2 2
x 1 x 1 11 x
2 1 − 2 1 + − + − − + ........
4 2 4 2 22 4
19 53 2
1 − 12 x + 144 x − ..........
= 35
= 1− x + .......
x 1 2 24
1 − 8 − 8 x − .......
35 35
Neglecting the higher powers of x, ⇒ a + bx = 1 − x ⇒ a =1,b =−
24 24
Illustration 30: Find the coefficient of a3b2c4d in the expansion of (a – b – c + d)10 (JEE ADVANCED)
Sol: Expand (a – b – c + d)10 using multinomial theorem and by using coefficient property we can obtain the
required result.
Using multinomial theorem, we have
(10 ) !
( a − b − c + d) ( a) 1 ( −b ) 2 ( −c ) 3 ( d) 4
10 r r r r
= ∑
10 r1 !r2 !r3 !r4
r1 +r2 +r3 +r4 = !
11
5
Illustration 31: In the expansion of 1 + x + find the term independent of x. (JEE ADVANCED)
x
11
5
Sol: By expanding 1 + x + using multinomial theorem and obtaining the coefficient of x0 we will get the term
independent of x. x
5
11
(11) ! r 5
3 r
∑ r !r !r ! (1) 1 ( x ) 2 x
r
1 + x + =
x r1 + r2 + r3 =
11 1 2 3
The exponent 11 is to be divided in such a way that we get x0. Therefore, possible set of values of (r1, r2, r3) are
(11, 0, 0), (9, 1, 1), (7, 2, 2) (5, 3, 3), (3, 4, 4), (1, 5, 5) Hence the required term is
4 . 2 2 | Binomial Theorem
1+
=
(11) ! . 2! 51 + (11) ! . 4! 52 + (11) ! . 6! 53 + (11) ! . 8! 54 + (11) ! . (10 ) ! 55
9!2! 1!1! 7! 4! 2!2! 5!6! 3!3! 3!8! 4! 4! 1!10! 5!5!
1 + 11 C2 × 2 C1 × 51 + 11 C 4 × 4 C2 × 52 + 11 C6 × 6 C3 × 53 + 11 C8 × 8 C 4 × 54 + 11 C10 × 10 C5 × 55
=
5
1 + ∑ 11 C2r . 2r Cr × 5r
=
r =1
PROBLEM-SOLVING TACTICS
Summation of series involving binomial coefficients
For (1 + x ) = n C0 + n C1 x + n C2 x2 + ...... + n Cn xn , the binomial coefficients are n C0 , n C1 , n C2 ,... , n Cn . A number of
n
series may be formed with these coefficients figuring in the terms of a series.
Some standard series of the binomial coefficients are as follows:
n
(a) By putting x = 1, we get C0 + n C1 + n C2 + ...... + n Cn =
2n …(i)
C0 − n C1 + n C2 − ...... + ( −1 ) . n Cn =0 ...(ii)
n n
(b) By putting x =-1, we get
n
(c) On adding (i) and (ii), we get C0 + n C2 + n C 4 + ...... =
2n−1 …(iii)
(e) 2n C0 + 2n
C1 + 2n
C2 + ...... + 2n
Cn−1 + 2n
22n−1
Cn =
⇒2 ( 2n
C0 + 2n
C1 + 2n
C2 + ....... + 2n
Cn−1 +) 2n
Cn =2n
2= [ 2n C0 2n
C2n , 2n C1
= 2n
C2n−1 and so on. ]
(g) Sum of the first half of n C0 + n C1 + ... + n Cn = Sum of the last half of n C0 + n C1 + ... + n Cn = 2n–1
(By using r. C n
r = nn−1Cr −1 )
( )
n
(ii) By differentiating the expansion of xd 1 + xd .
M a them a ti cs | 4.23
n n n n
C0 C1 C2 Cn
(j) Bino-harmonic series: + + + ...... +
a a+d a + 2d a + nd
1 1 1 1
Consider an HP - , , ,... ,
a a + d a + 2d a + nd
It is obtained by the sum of the products of corresponding terms of the sequences. Such series are calculated
in two ways :
(i) By elimination of r in the multiplier of binomial coefficient from the (r + 1)th term of the series
1 n 1 n+1
By using Cr = Cr +1
r + 1 n + 1
As the name suggests such series are obtained by multiplying two binomial expansion, one involving the first
factors as coefficient and the other involving the second factors as coefficient. They can be calculated by equating
coefficients of a suitable power on both sides.
For explanation see illustration 4
FORMULAE SHEET
For the sake of convenience the coefficients n C0 , n C1 , n C2 ...... n Cr ....... n Cn are usually denoted by C0 ,C1 ,......Cr .......Cn
respectively.
2n
C0 + C1 + C2 + ....... + Cn =
C0 − C1 + C2 − C3 + ....... + Cn =
0
n n n−1 n n − 1 n− 2
Cr= Cr −1= ⋅ Cr −2 and so on........
r r r −1
2n 2n!
Cn+r =
(n − r ) ! (n + r ) !
n
Cr + n Cr −1 =
n+1
Cr
0, if n is odd
C20 − C12 + C22 − C32 + ..... = n/2 n
( −1) Cn/2 , if n is even
Note: 2n+1
C0 + 2n+1
C1 + ....... + 2n+1
Cn
= 2n+1
Cn+1 + 2n+1
Cn+ 2 + ...... + 2n+1
C2n
= +1 22n
; C − C1 + C2 − C3 ...... + ( ) n =
C1 C2 Cn 2n+1 − 1 −1 n C 1
C0 + + + ...... + =
2 3 n+1 n+1 0
2 3 4 n+1 n+1
(a) Greatest term:
(i) If
(n + 1 ) a ∈ Z
(integer) then the expansion has two greatest terms. These are kth and (k + 1)th where x
x+a
and a are +ve real numbers.
(ii) If
(n + 1 ) a ∉ Z (n + 1 ) a
then the expansion has only one greatest term. This is (K + 1)th term k =
x+a x + a
denotes greatest integer less than or equal to x}
Solved Examples
( )
7 −3 1
y2 y2 two middle terms. 4th term = 7 C3 2x2 −
x
Sol: By using formula of finding general term we can 7!
= − 16x8 −3 =−560x5
1 3! 4!
easily get coefficient of . 4
( )
7 − 4 −1
y2 7 2 2
and 5 term = C 4 2x = 280x
th
5
( )
=(10 ) + 5 103 + 10 (10 ) ( 0.01 ) + 10 (10 )
3 2 Solving both we get n = 7 & r = 3
( 0.001) + 5 (10 )( 0.0001) + ( 0.00001) Example 5: Find the remainder when 2710 + 751 is
divided by 10
= 100000 + 5000 + 100 + 1 +0.005 + 0.00001
Sol: We can obtain the remainder by reducing 2710 +
= 105101.00501 751 into the form of 10 λ +a, where λ is any integer and
a is an integer less than 10.
Example 3: Find the middle term(s) in the expansion of We have 2710 = 330 = 915 = (10 – 1)15
7
2 1 751 = 7.750 = 7.(49)25 = 7 (50 –1)25
2x − .
x 2710 = 10m1 …(i)
Sol: Since n = 7 is a odd number. Therefore, find the 751= 7(50 –1)25 = 10m2 – 7 ...(ii)
Example 6: If A be the sum of odd numbered terms and Sol: By putting x = 1, -1, and ω, ω2
B the sum of even numbered terms in the expansion of Respectively in the expansion of (1 + x + x2)n we will get
(x + a)n prove that A2 – B2 = (x2 – a2)n the result.
( x + a) =
n n
C0 xn + n C1 xn−1a =a0 + a1 x + a2 x2 + ...... + a2n x2n ….(i)
Example 9: If (1 + x ) = C 0 + C1 x +
n
We know that (1 + x ) =
n n
C0 + n C1 x
C2 x2 + C3 x3 + ....... + Cn xn
+ n C2 x2 + .... + n Cn−1 xn−1 + n Cn xn
then prove that C12 + 2C22 + 3C32 + ...... + nCn2 =
( 2n − 1)!
( x + 1) =
n n n n n−1
C0 x + C1 x
( )
2
(n − 1 ) !
n n− 2 n n
+ C2 x + ..... + Cn−1 x + Cn
Sol: Expanding (1 + x ) and ( x + 1 ) and multiplying
n n
Multiplying these equations side by side, we get these two expansion and comparing the coefficient of
xn–1 we will prove above equation.
(1 + x ) ( x + 1 )
n n
= (C 0 + C1 x + C2 x2 + .... + Cn−1 xn−1 + Cn xn ) Given (1 + x ) =C0 + C1 x +
n
(
× C0 xn + C1 xn−1 + C2 xn−2 + .... + Cn−1 x + n Cn )
C2 x2 + C3 x3 + ..... + Cn xn
Coefficient of xn on R.H.S. is equal to Differentiating both sides w. r. t. to x, we get
C20 + C12 + C22 + .... + Cn2−1 + Cn2
n (1 + x )
n−1
=0 + C1 + 2C2 x +3C3 x2 + ..... + nCn xn−1
2n 2n!
Coefficient of xn in L.H.S. is Cn = . ⇒ n (1 + x )
n−1
=C1 + 2C2 x
n!n!
This proves the required identity. +3C2 x2 + ..... + nCn xn−1
….(i)
( )
n
and ( x + 1 ) = C0 xn + C1 xn−1 + C2 xn−2
n
Example 8: If 1 + x + x2 = a0 + a1 x + a2 x2 +...... + a2n x2n
show that
n
+C3 xn−3 + C 4 xn− 4 + ....... + Cn .…(ii)
(i) a0 + a1 + a2 + ...... + a2n =
3
M a them a ti cs | 4.27
n (1 + x )
2n−1
= (C 1 + 2C2 x + 3C3 x2 + ..... + nCn xn−1 ) 3n−1
∴ a0 + a3 + a6 + ...... =
(
× C0 xn + C1 xn−1 + C2 xn−2 + C3 xn−3 + .... + Cn .. ( iii) .…(iii) ) From (i) + (ii) × w2 (iii) × w,+we get,
3n + 0 × ω2 + 0 × ω
Now, coefficient of xn–1 on R.H.S.
= n=
( 2n − 1) ! ( 2n − 1)! =
( 2n − 1)! ∴ 3n= 3 ( a1 + a4 + a7 + ...... )
(n − 1) !n! (n − 1 ) ! (n − 1 ) !
( (n − 1 ) !)
2
Because coefficient of each is
Sol: Follow the algorithm for the greatest term. Example 12: Sum the series
C1 C2 Cn
Using standard notations w.r.t. (x + a)n C0 + + + ...... +
2 3 n+1
n+1 16
= =4
Sol: Expanding (1 + x ) integrating it from 0 to 1 or by
n
x 3
1+
a 1 + ( −1 ) using summation method we will get result.
n+1 C n+1
1
Sum = ∑ r −1
= ∑ n + 1 . n+1 Cr
T4 and T5 are numerically equal to each other and are r
=r 1 = r 1
greater than any other term.
=
1
n+1
( n+1
C0 + n+1 C1 + ...... + n+1 Cr +1 − n+1 C0 )
Example 11: If (1 + x + x ) = a0 + a1x + a2x + a3x ... + a2nx
2 2 3 2n
(1 + x )
n
equation and adding these values we can prove it. = C0 + C1 + C2 x2 + ..... + Cn xn
(
3n 3 ( a0 ) + a1 1 + ω + ω2 + a2
= )
(1 + ω 2
+ω 4
) + a (1 + ω
3
3
+ ω6 )
4 . 2 8 | Binomial Theorem
−..... − 13
C10 ( 730 ) −
3 13
C12 ( 730 )
2 (
(a) 6 C2 × 24 × x2 (1 − 3x ) = 15 × 16x2 1 − 6x + 9x2
2
)
∴ The term in x4 is (15) (16) (9x4)
− 13 C12 ( 730 ) + 1 (b) − 6 C3 23 × x3 (1 − 3x )
3
(c) 6 C 4 22 x 4 (1 − 3x ) = 15 × 4x 4 (1 − 4 × 3x + ....... )
4
= 1000 m + 15288 – 9490 + 1
= 1000 m + 5799
∴ The term in x4 is 15 × 4 × x 4
Thus, the last three digits of 17 256
are 799.
∴ The total term in x4 is
=+(1 C x + C x + C x + C x
11
1
11
2
2 11
3
3 11
4
4
+ ....... ) We have
(1 + x )
n
(1 + C x + C x + ......)
11
1
2 11
2
4 =C0 + C1 x + C2 x + ....... + Cn xn ….(i)
To find term in x4 from the product of two brackets on Differentiating both side w.r.t x, we get
the right-hand-side, consider the following products
n (1 + x )
n−1
= C1 + 2C2 x + 3C3 x2 + .... + n Cn xn … (ii)
terms as
(i) can also the be written as
1 × 11 C2 x 4 + 11 C2 x2 × 11 C1 x2 + 11 C 4 x 4
(1 + x ) =( x + 1 ) =C0 xn
n n
= 11 C2 + 11
C2 × 11
C1 + 11
C4 x 4
+C1 xn−1 + C2 xn−2 + ...... +Cn−1 x + Cn ....(iii)
55 + 605 + 330 x 4 =
990x 4
Differentiating both sides w.r.t. x, we get
∴ The coefficient of x4 is 990.
n (1 + x=
) nC0 xn−1 + (n − 1)
n−1
( )
6 6
(ii) 2 − x + 3x2 = 2 − x (1 − 3x )
C1 xn−2 + (n − 2 ) C2 xn−3 + ..... + Cn−1 ….(iv)
M a them a ti cs | 4.29
1
= [C1 + 2C2 + 3C3 x2 + ..... + r Cn xn−1 ] 2
+ C3 x3 + ....Cn xn ) dx
∫ (C0 + C1 x + C2 x + C2 x
0
x [nC0 xn−1 + (n − 1)C1 xn−2 + (n − 2)
1
1 + x n+1
C2 x n−3
+ ...... + Cn−2 x + Cn−1 ] .…(v) ( ) =C x + C x2 +
n + 1 0 1
2
0
The coefficient of xn–2 on the LHS of (v) is
1
x3 xn+1
n . 2 2n−2
Cn−2 = n . 2 2n−2
Cn C2 + ....... + Cn
3 n + 1
0
The coefficient of xn–2 on the RHS of (v) is 2 n+1
−1 C C C
= C0 + 1 + 2 + ....... + n
1. (n − 1 ) C12 + 2. (n − 2 ) C22 + ..... + (n − 1 ) .1Cn2−1 n+1 2 3 n+1
∫ (1 + x ) = ∫ ( C0 + C1 x + C2 x )
n n
( )
2
+ C3 x3 + .....Cn xn dx
Hence, r n − r Cr2 ∑( ) n
= 2 2n−2
Cn
0 0
r =0
2
1 + x n+1
or ( )
= C0 x + C1
x2
+ C2
x3
+ .... + Cn
xn+1
2
∫ (1 + x ) ∫ ( C0 + C1 x + C2 x )
n 2
1 1 1 dx= + C3 x3 + .....Cn xn dx
(iii) C0 − C1 + C2 − C3 + ......
2 3 4 −1 −1
n Cn 1 0
+ ( −1 ) 1 + x n+1
( )
= 2 3 n+1
n+1 n+1 = C x + C x + C x + ....... + C x |0
n + 1 0 1
2 2
3 n
n + 1 −1
(iv) C0 C1 C2 −1
+ + + ......
1.2 2.3 3.4 1 C C C
Cn 2n+ 2 − n − 3 − 0 = 0 − −C0 + 1 − 2 + ..... + ( −1)n+1 n
+ = n+1 2 3 n + 1
(n + 1) . (n + 2) (n + 1)(n + 2)
1 C C n Cn
C2 C4 2n = C0 − 1 + 2 + ....... + ( −1 )
(v) C0 + + + ...... = n+1 2 3 n+1
3 5 n+1
n
Ck
(iv) General term of L.H.S =
Sol: Expand (1 + x ) and integrate it within the limit 0
n
(k + 1)(k + 2)
to 1, 0 to 2, -1 to 0 and -1 to 1 respectively to prove
these equations n+1
Ck +1 nC n−1
Cr −1 n+ 2
Ck + 2
= = = r =
(1 + x )
n
=C0 + C1 x + C2 x2 + C3 x3 + ....... + Cn xn ….(i)
(n + 1) (k + 2) n
r (n + 1 )(n + 2 )
∴ The sum of terms on L.H.S.
(i) Integrating both sides of equation (i) within limits 0
n+ 2 n
to 1, we get n C 1
= ∑ (n + 1)(kn++2 2 )
= ⋅ ∑ n+ 2 Ck + 2
k =0 (n + 1)(n + 2) k =0
4 . 3 0 | Binomial Theorem
1 2n+ 2 − n+ 2 C − n+ 2 C k. n C = n. n−1 C
= k k +1
(n + 1)(n + 2) 0 1
–1 to 1, we get
(ii) To prove ( C0 + C1 )( C1 + C2 )( C2 + C3 ) ......
1
∫ (1 + x )
n
dx =
C0 C1 ......Cn−1 (n + 1 )
n
−1
( Cn−1 + Cn ) =
n!
1
∫ ( C0 + C1 x + C2 x )
2
+ C3 x3 + ...... + C n xn dx Multiply and divide L.H.S. by C0 C1C2 .......Cn−1 ; then,
−1
1 C
1 + x n+1 L.H.S. = C0 C1C2 .......Cn−1 1 + 1
( ) 1
2 3 n+1
= C x + C x + C x + ....... + C x C0
n + 1 0 1
2 2
3 n
n+1
−1 −1 C2 Cn
1 + ........ 1 +
2n+1 − 0 C C C C C C1 Cn−1
= C0 + 1 + 2 + ....... + n − −C0 + 1 − 2 + ......
n+1 2 3 n+1 2 3
n
Cr n−r +1
On using = we have,
2 n+1 C C n
Cr −1 r
= 2 C0 + 2 + 4 + .......
n+1 3 5
L.H.S. = C0 C1C2 .....Cn−1
2n C C
⇒ = C0 + 2 + 4 + .....
n+1 3 5 C1 C2 Cn
1 + 1 + ...... 1 +
C0 C1 Cn−1
Example 4: Prove, by binomial expansion, that
1 + n 1 + n n + 1
n = C0 C1C2 ...Cn−1 (1 + n) ...
(i) ∑k 2
⋅ Ck = n (n + 1 ) 2
n n− 2
2 3 n
k =1
C0 C1 ...Cn−1 (1 + n) C0 C1 .......Cn−1 (n + 1 )
n n
C0 C1 ........Cn−1 (n + 1 )
n
n
== = R.H.S.
(ii) ∏ ( Ck −1 + Ck ) =
n!
1.2.3........n n!
k =1
Differentiating twice w.r.t. x, we get Sol: By using summation and coefficients properties
we can prove given equations.
n (n − 1 )(1 + x )
n− 2
= 2.C2 + 3.2.C3 x
∑ ∑ ( Ci + C j )
2
+4.3C 4 x + ...... + n (n − 1 ) Cn x
2 n− 2
0 ≤i < j≤n
= ( C0 + C1 ) + ( C0 + C2 ) + ...... +
2 2
Substituting x = 1, we get
n
n (n − 1 ) 2n−2 = ∑ k (k − 1 )( Ck ) ( C0 + Cn ) + ( C1 + C2 )
2 2
k =1
+ ..... + ..... +
n
∴ ∑ k2
k =1
( )( C ) =
n (n − 1 ) 2
n
k
n− 2
+ n.2n−1
( C1 + Cn ) + ( C2 + C3 )
2 2
+ .... + .... + ( C2 + Cn )
2
M a them a ti cs | 4.31
∴ 2∑ ∑ Ci ⋅ C j 3n n!
In =
0 ≤i < j≤n
(3n + 1)(3n − 2)(3n − 5) ......7.4
= ( C0 + C1 + C2 + C3 + ...... + ) − C20 + C12 + ..... + Cn2
2
( ) Using (i)
(2 )
2
= n
− 2n
Cn C0
−
C1
+
C2
−
C3
+...... +
( −1) n Cn =
3n n!
1 4 7 10 3n + 1 1.4.7.....(3n + 1)
∑ ( Ci + C j=
)
2
∴∑ n. 2n C + 22n − 2n
Cn ,
n
0 ≤i < j≤n
Example 7: Prove that
(n − 1) 2n Cn + 22n
=
n (n − 1 ) n (n − 1 ) ......2.1
1 n
C0 + C1 C2 +..... + C
m! (m + 1 ) ! (m − 2 ) ! (m + n) ! n
Example 6: Show that
C0 C1 C2 C3 ( −1 ) n Cn 3n n! =
(m + n + 1)(m + n + 2) ...... (m + 2n)
1
−
4
+
7
−
10
+ ..... + 3
2n + 1
=
1.4.7....... ( 3n + 1 ) (m + n) !
(1 − x ) 3
n
=C0 − C1 x + C2 x 3 6
⇒ ( m+n
C0 + m+n
C1 x + m+n
C2 x2 + .... + m +n
Cm+n xm+n )
−C3 x9 + C 4 x12 + .... + ( −1 ) Cn x3n
n
(
× C0 + C1 x + C2 x2 + ..... + Cn xn = ) (1 + x )
m+ 2n
Integrating both sides between limits 0 and 1, we get Equating the coefficients of xn on both sides, we find
1
( ) −1 n
Cn m+n
Cn .C0 + m+n
Cn−1 .C1 + m+n
Cn−2 .C2
∫ (1 − x ) dx =
C 3 C C n
1 2 3
C − + − + ..... + 0 …(i)
0
4 7 10 3n + 1
m+n m+ 2n
+..... + C0 .Cn = Cn
1
∫ (1 − x ) dx
n
(m + n) ! C (m + n) ! C
3
In
Also =
0 ⇒ +
m!n! 0
(n − 1 ) ! (m + 1 ) ! 1
1 1
( ) ( ) ( −3x ) .x dx
n n−1
(m + n) ! C + .... + (m + n) ! C =(m + 2n) !
2
= x 1 − x3 − ∫ n 1 − x3 ;
0 +
(n − 2 ) ! (m + 2 ) ! 2 (m + n) ! n (m + n) !n!
0
1
( )
n−1
= 3n∫ x3 1 − x3 dx
0
Dividing both sides by (m + n)!/n! we find
1 n (n − 1 ) n (n − 1 ) .....2.1
( )( ) 1 n
n−1
= 3n∫ x3 − 1 + 1 1 − x3 dx C0 + C1 + C2 +...... + C
0
m! (m + 1 ) ! (m + 2 ) ! (m + n) ! n
3n
= 3nIn−1 − 3nIn ; (1 + 3n) In =
3nIn−1 ∴ In = I
3n + 1 n−1
4 . 3 2 | Binomial Theorem
=
(m + 2n) ! (m + n + 1)(m + n + 2) ..... (m + 2n) = Coefficient of xn in
(m + n) ! (m + n) ! (m + n) ! 1 + x n + nx 1 + x n−1 x + 1 n
( ) ( ) ( )
= Coefficient of xn in
Example 8: Find the sum of the following series
1 + x 2n + nx 1 + x 2n=
( ) ( )
−1 2n
1 + 2 2 1 + 2 + 3 2 …Upto n term Cn + n. 2n−1 Cn−1
C12 +
S= C + C3 +
2 2 3
Sol: In this problem, first obtain the rth term and then = ( ) 1 + n
2n ! 2n n
= Cn 1 +
by using binomial expansion and coefficient property n!n! 2 2
we can get required sum.
The rth term of the given series ⇒ 2C12 + 3C22 + 4C32 + ...... + (n + 1 ) Cn2
1 + 2 + ...... + r 2 r (r + 1) 2 1 n
=
r
Cr =
2r
C=
r
2
(r + 1) Cr2 = 2n
Cn 1 + − 1
2
C0 = 1
1 1 1
∴ S = (1 + 1 ) C12 + ( 2 + 1 ) C22 + ⇒S
1 2n n
2 2 2 = Cn 1 + − 1
2 2
( ) is an odd integer. If f be
n
We know that
the integral part I of 5 + 2 6
C0 + C1 x + C2 x2 + .... + Cn xn = (1 + x )
n
Differentiating both sides w.r.t. x we get Sol: By using expansion formula we can expand the
given binomial and separating its integral and fractional
C0 + 2C1 x + 3C2 x2 + 4C3 x3 part we can prove given equations.
( )
n
+.... + (n + 1 ) Cn xn = (1 + x )
n
Let P =
5+2 6 I+f
=
+nx (1 + x ) ( )
n−1
…(i) Or f 5n + C1 5n−1 2 6
I +=
Also
+C 5 ( 2 6 ) + ...... + C ( 2 6 )
2 n
n− 2
2 3 2 n ...(i)
1 1 1
C0 + C1 + C2 + C3
0 < 5 − 2 6 < 1 ⇒ 0 < (5 − 2 6 ) < 1
x x x n
n n
1 1
Let (5 − 2 6 ) = f’, where 0 < f’ < 1.
n
+.... + Cn = 1 + …(ii)
x
x
∴ f + f ' =1 and ∴ I is an odd integer Again, from (i) + (ii) w + (iii) × w3, we get
(5 + 2 6 ) , ( ) ( )
n
Now I + f = = 3n= a0 1 + ω + ω2 + a1 1 + ω2 + ω4
(5 − 2 6 ) ( )
n
1 − f ⇒ ( I + f )(1 − f ) =
f' =
= 1 ...... 3 ( a2 + a5 + a8 + ........ )
+a2 1 + ω3 + ω3 + =
1 1
∴ (I + f ) = ∴=I −f Example 11: Find the
1−f 1−f
(i) Last digit
Example 10: If 1 + x + x ( 2
)= a0 + a1 x (ii) Last two digits and
+a2 x2 + a3 x3 .... + a2n x2n (iii) Last three digits of 17256.
3n = a0 + a1 + a2 + a3 + a4 + ...... …(i)
+ 128 C126 ( 290 ) − 128 C127 ( 290 ) + 1
2
2 3 4
0= a0 + a1 ω + a2 ω + a3 ω + a4 ω + ...... ...(ii)
0 = a0 + a1 ω2 + a2 ω4 + a3 ω6 + a4 ω8 + ...... …(iii)
+ 128 C2 ( 290 ) − ...... − 128 C125 ( 290 ) ]
126 3
Adding these
+ 128 C126 ( 290 ) − 128 C127 ( 290 ) + 1
2
(
3n 3 ( a0 ) + a1 1 + ω + ω2 + a2
= )
(1 + ω 2
)
+ ω4 + a3 1 + ω3 + ω6 ( ) 1000m + 128 C2 ( 290 ) − 128 C1 ( 290 ) + 1 (m ∈ I + )
=
2
+ a (1 + ω
4
6
+ ω ) + ...
9
32
Example 12: If 3232 is divided by 7, then find the
∴ 3 = 3 ( a1 + a4 + a7 + ...... )
n
remainder
Because coefficient of each is Sol: Here in this problem, we can obtain required
2 2 32
1 +=
ω+ω 0, using
= ω 1 remainder by reducing 3232 into the form of 7 λ +a,
3n−1
∴ a1 + a4 + a7 + ........ = where λ is any integer and a is an integer less than 7.
4 . 3 4 | Binomial Theorem
( )
32
∴ ( 32 ) = 2160 ; ( 32 ) = (3 − 1)
32 32 160
= 25
+ 5m+1 C2 ( 7 )
5m−1
+ .... +
160 160 160 159 160 160
= C0 3 − C1 3 + .... + C159 3 + C160 + 1 5m+1
C5m 7 + 5m+1 C5m+1 )
= 3 3 ( 159
− 160
C1 3 158
+ .... − 160
)
C159 + 1
= 4[7{ 5m+1 C0 − 75m + 5m+1 C1 75m−1
+
= 3m + 1, m ∈ I
+ 5m+1 C2 75m−2 + .... + 5m+1 C5m } + 1]
3232 3m+1 5( 3m+1 ) 15m+5
Now, 32
= 32
= 2= 2
4 7n + 1 ,
= n ∈ I+ = 28 n + 4
2( ).22 = 4. 8
32
( )
3 5m+1 5m+1
∴ 3232 = 32
This show that where 3232 is divided by 7, then
= 4. ( 7 + 1 )
5m+1 remainder is 4.
JEE Main/Boards
Exercise 1
n
1
Q.9 Given that the 4th term in the expansion of px +
x
5
( )
5
Q.1 Expand x2 + 2a by binomial theorem. is , find n and p.
2
(1 − 2x + x ) .
n
2 Q.15 If 3 consecutive coefficients in the expansion of
(1 + x)n are in the ratio 6 : 33 : 110, find n and r.
(1 + 2x + x )
27
2
Q.7 If (11)27 + (21)27 when divided by 16 leaves the
remainder
9
3 1
Q.27 Find the term independent of x in x2 − . (A) 0 (B) 1 (C) 2 (D) 14
2 3x
Q.28 If (1 + ax)n = 1 + 8x + 24x2 + ….. Find a and n. Q.8 Last three digits of the number N = 7100 – 3100 are
(A) 100 (B) 300 (C) 500 (D) 000
Exercise 2 Q.9 The last two digits of the number 3400 are:
(A) 81 (B) 43 (C) 29 (D) 01
Single Correct Choice Type
( )
25
( )
15
Q.1 Given that the term of the expansion x1/3 − x −1/2 Q.10 If 1 + x + x2 = a0 + a1 x + a2 x2 + .. + a50 .x50
which does not contain x is 5 m where m ∈ N , then m = then a0 + a2 + a4 + .... + a50 is:
(A) 1100 (B) 1010 (C) 1001 (D) None (A) Even
(B) Odd and of the form 3n
(C) Odd and of the form (3n –1)
(D) Odd and of the form (3n+1)
4 . 3 6 | Binomial Theorem
x
2.P2 +3.P3 + ..... + n.Pn =
(A) 5 (B) 8 (C) 10 (D) 100
(A) (n + 1)! –1 (B) (n + 1)! + 1
(C) (n + 1)! (D) None of these
(
Q.20 (1 + x ) 1 + x + x2 ) (1 + x + x 2
)
+ x3 .....
( )
100
2+43 is-
(
Q.15 If (1 + x ) 1 + x + x2 )
(A) 25 (B) 26 (C) 27 (D) 28
(1 + x + x 2
+ x ) ..... (1 + x + x
3 2
+ x3 + .... + xn )
Q.23 The greatest value of the term independent of x in
= a0 + a1 x + a2 x2 + a3 x3 + .... + am xm 10
cos θ
m the expansion of x sin θ + is
Then ∑ ar has the value equal to x
10
r =0
10 10 C5
(A) C5 (B) 25 (C) 25. C5 (D)
(A) n! (B) (n + 1)! 25
(C) (n –1)! (D) None of these
(1 + x − 3x )
2145
Q.24 If 2
=a0 + a1 x + a2 x2 + .... then
Q.16 In the expansion of (1 + x) if the coefficient of 43 a0 − a1 + a2 − a3 + .... end with
the (2r + 1)th and the (r + 2)th terms are equal, the value
of r is : (A) 1 (B) 3 (C) 7 (D) 9
( )
Q.27 Given 1 − 2x + 5x2 − 10x3 (1 + x ) = 1 + a1x + a2x2
n
30
Q.7 C030 C10 − 30
C1 30C11 + ...... 30
C20 30C30 is equal to
Previous Years’ Questions (2005)
30 60 30 65
Q.1 Given positive integers r > 1, n > 2 and the (A) C11 (B) C10 (C) C10 (D) C55
coefficient of (3r)th and (r + 2)th terms in the binomial
expansion of (1 + x)2n are equal. Then (1980) Q.8 For r = 0, 1, ....., let Ar, Br and Cr denote, respectively, the
(A) n = 2r (B) n = 2r + 1 coefficient of xr in the expansions of (1 + x)10, (1 + x)20 and
10
(C) n = 3r (D) None of these (1 + x)30. Then ∑ Ar (B10Br − C10 Ar ) is equal to (2010)
r =1
Q.2 If Cr stands for nCr, then the sum of the series (A) B10 – C10 2
(B) A10 B10 (
− C10 A10 )
n n (C) 0 (D) C10 − B10
2 ! !
2 2 . C20 − 2C12 + 3C22 − .... + ( −1 ) (n + 1 ) Cn2
n
n!
Q.9 If the coefficients of x3 and x 4 in the expansion
Where n is an even positive integer, is equal to (1986)
( )
of 1 + ax + bx2 (1 − 2x ) in powers of x are both zero,
18
(A) ( −1 ) (n + 2 ) (B) ( −1 ) (n + 1 )
n/2 n
then (a, b) is equal to: (2014)
251 251
(C) ( −1 ) (n + 1 )
n/2
(D) None of these (A) 16, (B) 14,
3 3
( )
50
the binomial expansion of 1 − 2 x is: (2015) Q.11 If the number of terms in the expansion of
n
2 4
(A)
2
(
1 50
3 + 1 (B)
2
)
1 50
3 ( ) 1 − + 2 , x ≠ 0 , is 28, then the sum of the
x x
coefficients of all the terms in this expansion, is:(2016)
(C) (
1 50
2
3 − 1 (D))1 50
2
2 +1 ( )
(A) 64 (B) 2187 (C) 243 (D) 729
JEE Advanced/Boards
Q.11 For which positive values of x is the fourth term in Paragraph for questions. 22 and 23
the expansion of (5 + 3x)10 is the greatest.
A path of length n is a sequence of points
x 2
n
( x1 , y1 ) , ( x2 , y 2 ) ,....., ( xn , yn ) with integer coordinates
Q.12 Find the index n of the binomial + if the
5 5 such that for all i between 1 and n – 1 both inclusive,
9 term of the expansion has numerically the greatest
th either xi+1= xi + 1 and y i+1 = y i (in which case we say
coefficient (n ∈ N) . the ith step is rightward) or xi+1= xi and y i+1= y i + 1 (in
which case we say that the ith step is upward).
Q.13 Find the number of divisors of the number This path is said to start at (x1, y1) and end at ( xn , yn ) .
2000 Let P (a, b), for a and b non negative integers, denotes
N
= C1 + 2. 2000 C2 + 3. 2000 C3 + .... + 2000. 2000 C2000
the number of paths that start at (0, 0) and end at (a, b)
10
Q.14 Find number of different dissimilar terms in the
sum
Q.22 The value of ∑ P (i,10 − i) , is
i=0
( ) ( )
2011 2010
(1 + x )
2012 2 3
+ 1+x + 1+x (A) 1024 (B) 512 (C) 256 (D) 128
Q.15 Find the term independent of x in the expansion Q.23 Number of ordered pairs (i, j) where i ≠ j for which
9 P (i, 100 – i) = P( j, 100 – j), is
3x2 1
(
of 1 + x + 2x3
2
− .)
3x (A) 50 (B) 99 (C) 100 (D) 101
( )
2n+1
k n n Q.24 If 6 6 + 14 = N & F be the fractional part of
Q.16 Let f(n) = ∑∑ . Find the total number of
k rr
=r 0= N, prove that NF = 20 2n+1
(n ∈ N) .
divisors of f (11).
( )
5
Q.25 Let P = 2 + 3 and f = P – [P], where [P] denotes
i 11 11
Q.17 Find the sum ∑∑ . f2
=j 0=i j j
the greatest integer function. Find the value of .
1 − f
n
[Note : = n Cr ]
r Q.26 If C0, C1, C2, …. , Cn are the combinatorial
coefficients in the expansion of (1 + x)n , n ∈ N then
n+ 4
( ) . (1 + x ) prove the following:
2 n
Q.18 Let 1 + x2 ∑ ak .xk . If a1, a2 and a3 are
=
k =0 n.2n−1
(a) C1 + 2C2 + 3C3 + .... + n.Cn =
in AP, find n.
(b) C0 + 2C1 + 3C2 + .... + (n + 1)Cn =(n + 2) 2n−1
n
Q.19 Prove that ∑ n Ck sink x.cos (n − k ) x =
2n−1 sinnx . (c) C0 + 3C1 + 5C2 + .... + (2n + 1)C =
n (n + 1) 2n
K =0
(d) ( C0 + C1 )( C1 + C2 )( C2 + C3 ) ....(Cn−1 + Cn )
Q.20 Find the sum of the roots (real or complex) of the
C0 .C1 .C2 ....Cn−1 (n + 1 )
n
2001
1
equation x2001 + − x =0. =
2 n!
(n + 1)( 2n)!
2n
(e) 1.C20 + 3.C12 + 5.C22 + .... + ( 2n + 1) Cn2 =n!n!
Q.21 If for n ∈ N, ∑ (−1)k (2n Ck )2 =
A , then what will
k =0
2n
Q.27 Let I denotes the integral part and F the proper
(3 + 5 )
n
be the value of ∑ (−1) k
(k − 2n) ( Ck ) ? 2n 2
fractional part of where n ∈ N and if ρ
k =0
denotes the rational part and σ the irrational part of
the same, show that
4 . 4 0 | Binomial Theorem
1 1 100 100
=
ρ
2
(I + 1) and =
σ
2
(I + 2F − 1) Q.31 Let ∑ ∑ ( C12 + Cs2 + C=
r Cs ) m ( 2n
)
Cn + 2p
=r 0=s 0
Q.30 If a0, a1, a2,..... be the coefficients in the expansion Q.2 The remainder, when (1523 + 2323) is divided by 19, is
of (1 + x + x2)n in ascending powers of x, then prove
(A) 4 (B) 15 (C) 0 (D) 18
that :
(i) a0 a1 − a1a2 + a2a3 − .... =
0 Q.3 The value of 4 {Cn
1 + 4. n C2 + 42. n C3 + .... + 4n−1 is}
(ii) a0 a2 − a1a3 + a2a4 − .... + a2n−2a2n = an+1 or an−1
(A) 0 (B) 5n + 1 (C) 5n (D) 5n – 1
(iii) E=
1 E=
2 E=
3 3n−1 ;
Q.4 If n be a positive integer such that n ≥ 3 ,then the
Where E1 = a0 + a3 + a6 + ....;E2 = a1 + a4 + a7
value of the sum to n terms of the series
+..... & E3 = a2 + a5 + a8 + .... (n − 1 ) (n − 1)(n − 2)
1.n −
1!
(n − 1 ) + 2!
M a them a ti cs | 4.41
8
1 2 Q.11 Value of the expression C20 + C12 + C22 + .... + Cn2 is
is
8/3 + x log10 x is 5600, then x equals to-
x (A) 22n ‒ 1 (B) 2n (2nCn)
(A) 5 (B) 8 (C) 10 (D) 100 (C) 2nCn (D) None of these
(α + p) ( α + q) + ..... + ( α + q)
m −3 2 m−1
where Cr denotes combinatorial coefficient in the
expansion of (1 + x)n, n ∈ N
Where α ≠ −q and p ≠ q is:
(A) (a + 2nb)2n (B) (2a + nb)2n
(A)
m
(
Ct pt − qt ) (B)
m
(
Ct pm−t − qm−t ) (C) (a + nb)2n–1 (D) (2a + nb)2n–1
p−q p−q
(C)
m
(
Ct pt + qt ) (D)
m
(
Ct pm−t + qm−t ) Previous Years’ Questions
p−q p−q
( 1) n.Cn
n
Q.1 Prove that C12 − 2.C22 + C32 − ..... − 2n.C22n =−
( )
2145
Q.7 If 1 + x − 3x2 = a0 + a1 x + a2 x2 + ... (1979)
then a0 − a1 + a − a3 + .... end with Q.2 Given,
2
(A) 1 (B) 3 (C) 7 (D) 9 Sn = 1 + q + q2 + .... + qn
2 n
Q.8 Coefficient of x6 in the binomial expansion q+1 q+1 q+1
Sn =
1+ + +.... + ,q ≠ 1
9 2 2 2
4x2 3
− is n+1 n+1 n+1
3 2x Prove that C1 + C2S1 + C3S2
n+1 n
(A) 2438 (B) 2688 +.... + Cn+1Sn =
2 Sn (1984)
Q.5 Let n be a positive integer and Q.8 The coefficients of three consecutive terms of
(1 + x + x ) (1 + x )
n n+5
2
= a0 + a1 x + .... + a2n x2n . are in the ratio 5 : 10 : 14. Then n =__(2013)
(1 + x ) (1 + x ) (1 + x )
4 7 12
2 3 4
is (2014)
Q.6 Prove that (2003)
2k nC0nCk − 2k −1 nC1n−1Ck −1 + 2k −2 (A) 1051 (B) 1106 (C) 1113 (D) 1120
n
C2n−2Ck −2 − ...... + ( −1)k nCkn−k C0 =nCk
Q.10 The coefficient of x9 in the expansion of
−z r z 2s
r =1
Let P =
( ) and I be the identity matrix of
is equal to (2010)
z 2s zr
(A) B10-C10 2
(
(B) A10 B10 − C10 A10 ) order 2. Then the total number of ordered pairs (r, s) for
(C) 0 (D) C10-B10 which P2 = −I is (2016)
PlancEssential Questions
JEE Main/Boards JEE Advanced/Boards
Exercise 1 Exercise 1
Q. 3 Q. 16 Q. 19 Q. 23 Q. 14 Q. 23 Q. 26 Q. 31
Q. 28 Q. 32 Q. 34 Q. 34 Q. 35
Exercise 2 Exercise 2
Q. 7 Q. 13 Q. 15 Q. 21 Q. 2 Q. 4 Q. 12
Q. 22 Q. 25 Q. 29
Previous Years’ Questions
Previous Years’ Questions Q. 3 Q. 4
Q. 2 Q. 3 Q. 5 Q. 6
Q. 8
M a them a ti cs | 4.43
Answer Key
JEE Main/Boards
Exercise 1
Q.1 x10 + 10x8 a + 40x6 a2 + 80x 4 a3 + 80x2a4 + 32a5 Q.2 4ab 3a4 + 10a2b2 + 3b 4⇒
, 140 2
2n!
( −1 ) xn
n
Q.4 100 Q.5 41 Q.6 280x2 Q.7
n!n!
1
Q.9 n = 6, p = Q.10 m = 12 Q.15 n = 12, r = 1 Q.20 34, 23
2
Q.23 232 Q.24 k = ± 3
Q.21 11, T3+1, T3+2, T3+3 Q.26 54C14
7
Q.27 Q.28 n = 4, a = 2
18
Exercise 2
JEE Advanced/Boards
Exercise 1
5
Q.1 816 Q.2 (a) (i) (ii) T6 = 7, (b) x = 0 or 1
12
11 a6 11 a5
Q.3 (i) C5 (ii) C6 (iii) ab = 1 Q.4 (a) r = 5 or 9
b5 b6
Q.5 0 Q.7 (a) – 1260 a2b3c4; (b) – 12600
7.313
Q.9 (i) T7 = (ii) 455 × 312 Q.10 (i) 3n (ii) 1, (iii) an
3
4 . 4 4 | Binomial Theorem
5 20
Q.11 <x< Q.12 n = 12 Q.13 8016 Q.14 4023
8 21
17
Q.15 Q.16 24 Q.17 4095 Q.18 n = 2 or 3 or 4
54
Q.20 500 Q.22 A Q.23 C Q.25 722
n2 + n + 2 n(n + 1) (n + 1)!
Q.31 502 Q.32 (a) , (b) a0 = a , (c) Q.33 66
2 2 2
Exercise 2
2mn − 1
Q.3 Q.9 C
(
2mn 2n − 1 )
Solutions
C0a6+6C1a5b+6C2a4b2+6C3a3b3
6 = 2[50C1(100)49 + 50C3(100)47]+ 50C5(100)45+
+ 6C4a2b4+6C5ab5+6C6b6 ……+50C49100]
(x )
1 log10 x
2 2n! 2n!
T3 T=
= 5
C2 = 1000 = =
2 +1
x n!(2n − n)! n!n!
⇒ 62n–35n–1 = (62 )n − 35n − 1 = (36)n − 35n − 1 n!(r + 1)!(n− r)! n!(r + 3)!(n− r − 2)!
= +
(n+ 1)!(r)!(n− r)! (n+ 1)!(r + 2)!(n− r − 2)!
M a them a ti cs | 4.47
= (1–x)4+4x2(1–x)3+6(1+x2–2x)x4+4x6(1–x)+x8
(r + 1)(r + 2) 33 3 r+2 3
And = = ⇒ =
(r + 1)(n − r − 1) 110 10 n − r − 1 10 = (4C0–4C1x+4C2x2–4C3x3+4C4x4)
Subtracting equation (i) from (ii), we get n = 12 +(6 + 6x2 − 12x)x 4 + 4x6 − 4x7 + x8
Putting n = 12 in equation (i) = 1 − 4x + 6x2 − 4x3 + x 4 + 4x2 − 12x3
13r = 2n – 11 = 2(12) – 11 = 24 – 11 = 13 ⇒ r = 1
+12x 4 − 4x5 + 6x 4 + 6x6 − 12x5 + 4x6 − 4x7 + x8
So terms are Tr +1 , Tr + 2 , Tr +3
=1 − 4x + 10x2 − 16x3 + 19x 4 − 16x5 + 10x6 − 4x7 + x8
n! n! n! So greatest is 4+n.
+ 2
= 3
13!(n − 13)! 15!(n − 15)! 14!(n − 14)! 11 2
| T4 | | T=
= 3+1 | C3 (7)11−3 5 ×
3
1 1 2
+ =
(n − 13)(n − 14) 15 × 14 14(n − 14) 11 × 10 × 9 8 3 23 11 3 4 8
= ×7 5 × = 25 7
1.2.3 33 9
15 × 14 + (n − 13)(n − 14) 2
=
15 × 14(n − 13)(n − 14) 14(n − 14) 8
2 14
Sol 23: (1 + 3x + x ) 1 +
210 + n2 + 182 – n(13 + 14) = 2 × 15(n – 13) x
n2 + 392 – 27n = 30n – 390 For Coefficient of x −1
n – 57n + 782 = 0
8 1 1 1
= (1) C1 + 3x2 8 C3 + x 4 8 C5
2
57 ± 57 − 4(1)(782) 57 ± 121 57 ± 11 x
x 3
x5
n = =
2(1) 2 2
8 3 × 8 × 7 × 6 −1 8 × 7 × 6 −1
n = 34, 23 = + ×x + x
x 1×2×3 1.2.3
Last three digits = 000 When 1 + 1P1 + 2P2 + 3P3 +.... + nPn = (n + 1)!
= −1 + 1(n + 1)!− 1
Sol 9: (D) 3400 = (32)200
= (n + 1)!− 1
(9)200
= (10 − 1)200
200
= C0 10200 − 200 C1 10199 +.... 200 C199 101 + 200 C200 .1 Sol 13: (D)
= 10m + 1 (m∈N)
7
7
1 1 + 4x + 1 − 1 − 4x + 1
Last 2 digits are 01 4x + 1 2 2
1.2
Sol 10: (A) (1+x+x2)25 = a0 + a1x + …. + a50x50 = 7 C 4x + 1 + 7 C ( 4x + 1)3
27
4x + 1 1 3
x=1
+...... + 7 C7 ( 4x + 1)7
325
= a0 + a1 + a2 + ..... + a50 …(i)
= 2−6 7 C1 + 7 C3 (4x + 1) + 7 C5
x = –1
( )
m
8×7×6 3
∑ ar = a0 + a1 + a2 + ...... + am ⇒
1.2.3
× x −8/3 x10 (log10 x)5 =
5600
r =0
10
C3a3 = 10 C1a F + f = 2I − p = Integer
0 < F + f < 2 ⇒ F + f =⇒
1 F=1–f
10 × 9 × 8 2
a = 10
1.2.3 (F)(f + p) = (5 − 2 6)n (5 + 2 6)n = – 1
1 1 1
a2 = ⇒a
= =
12 12 2 3
4 . 5 2 | Binomial Theorem
10 Coefficient of x=a1=nC1–2=n–2
cos θ
Sol 23: (D) x sin θ + n(n − 1)
x Co-efficient of x2=a2=5+nC2–2nC1= 5 + − 2n
2
r
10 cos θ n2 − n − 4n n2 − 5n
Tr +1
= Cr (x sin θ)10 −r = 5+ 5+
=
x 2 2
Power of x = 10 − r + r( −1) = 10 − 2r =
0 (given) 10 + n2 − 5n
a12 = 2a2 ⇒ (n − 2)2 =
2
⇒r=5 2
⇒ n2+4–4n=10+n2–5n ⇒ n=6
Tr +1= 10
C5 (sin θ)5 (cos θ)5 = 10 C5 (sin θ cos θ)5
5
10 sin2θ Sol 28: (D) aC0+(a+b)C1+(a+2b)C2+…. +(a+nb)Cn
= C5 . Max value when sin2θ =1
2 a(C0+C1+C2+….. +Cn)
10
C5 +b(C1+2C2+……+nCn)
∴ Max. value =
25
= a2n+b[n2n–1] = 2n–1[2a+nb]
Sol 24: (B) (1+x–3x2)2145=a0+a1x+a2x2+
At x= – 1 Previous Years’ Questions
(1 − 1 − 3)2145 =−(3)2145
Sol 1: (A) In the expansion
= a0 − a1 + a2 − a3 + .......
(1 + x ) C3r −1 ( x )
2n 2n 3r −1
,t3r =
2145 2144 1072
L. H. S. = 3 = 3.3 = 3[9]
Cr +1 ( x )
2n r +1
Even power of 9 ends with 1. Hence 32145 ends with 3. tr + 2 =
4x2
9 −r r ⇒ 3r − 1 = r + 1 or 2n = (3r − 1) + (r + 1)
9 3
=Tr +1 Cr −
3 2x 2r 2 or=
⇒= 2n 4r
⇒ 3r = 18 − 6 = 12 ⇒ r = 4 But r > 1,
4
9− 4
−3
4
9×8×7×6 4 3
5 4 ∴ We take n = 2r
9
Coefficient C4 =
3 2
1.2.3.4 3 2
M a them a ti cs | 4.53
+.... + ( −1 ) (n + 1 ) Cn2
n
∴ n+1Cr +n+1 Cr −1 = n+ 2Cr
n n n n
n! n
−1 n n! =
+ + +
( −1) − ( −1 ) 2
n/2
= ⋅
2 n n r r − 1 r − 1 r − 2
n n
! ! ! !
2 2 2 2 n + 1 n + 1 n + 2
= + =
r r −1 r
( −1) n n! n 1 + n2
n/2
=
According to given condition, Tn = n C3
! !
2
2
and Tn+1 − Tn =
21
n n
2 ! ! n+1
2 2 ⇒ C3 − n C3 =
21
∴
n! 1 1
⇒
6
( n + 1 )(n)(n − 1 ) − n (n − 1 )(n − 2 ) =
6
21
{C20 − 2C12 + 3C22 − .... + ( −1 ) (n + 1 ) Cn2 }
r
n (n − 1 )
( n + 1 ) − ( n − 2 ) =
n n
2 ! !
⇒
6 21
2 2 −1 n/2 n! (n + 2 ) =
( ) ( −1 ) (n + 2 ) n (n − 1)
n/2
=
n! n n 2 ⇒ 21 ⇒ n (n − 1 ) =
= 42
! ! 6
2 2
⇒n=7
5 2 3 5 4 5 5 5 5 5 4
⇒ n−1
(
Cr =k 2 − 3 ) r +n 1 n−1
Cr
+ C3a b + C 4 ab + C5b + C0 a − C1a b
r +1
⇒ k2 − 3 =
n
+ 5 C2a3b2 − 5 C3a2b3 + 5 C 4 ab 4 − 5 C5b5
r +1
(Since,n ≥ r ⇒ ≤ 1 and n,r > 0)
n
=2 a5 + 10a3b2 + 5ab 4
⇒ 0 < k2 − 3 ≤ 1
5 5
( ) ( )
1/2 1/2
∴ x + x3 − 1 3
+ x − x − 1 ⇒ 3 < k2 ≤ 4
(
2
= 2 x5 + 10x3 x3 − 1 + 5x x3 − 1 ) ( )
⇒ k ∈ −2, − 3 ∪
) ( 3,2
30 30 30 30
Therefore, the given expression is a polynomial of Sol 7: (C) Let −
0 10 1 11
degree 7.
30 30 30 30
Sol 4: (D) n Cr + 2 nCr −1 +n Cr −2 + − .... +
2 12 20 30
(
= n Cr +n Cr −1 + ) ( n
Cr −1 +n Cr −2 ) ∴A
= 30
C0 . 30 C10 − 30
C1 . 30 C11
We know that
+ 30 C2 . 30 C12 − .... + 30
C20 . 30 C30
4 . 5 4 | Binomial Theorem
10 10 (i) + (ii)
∑ 10
Cr 20 C10 20 Cr − ∑ 10 Cr 30 C10 10 Cr
r 1 =r 1 350 + 1= 2 1 +50 C2 .22 +50 C 4 .24 + .... +50 C50 .250
10 10
350 + 1
= ∑ 10 C10 −r ⋅ 20 C10 20 Cr − ∑ 10 C10 −r 30 C10 10 Cr ∴ =1 +50 C2 .22 +50 C 4 .24 + .... +50 C50 .250
r 1 =r 1 2
10 10
20
C10 ∑ 10 C10 −r . 20 Cr − 30 C10 ∑ 10 C10 −r 10 Cr
(=
n + 1 )(n + 2 )
28
=r 1 =r 1 Sol 11: (D) Number
= of terms
2
= 20
C10 ( 30
)
C10 − 1 − 30 C10 ( 20
C10 − 1 ) ⇒n=
6
n
30 20 a1 a2 a2n 2 4
= C10 − C10 =C10 − B10 ∴ a0 + + + .... + = 1 − +
x x2 x2n x x2
Put x = 1, n = 6,
Sol 9: (D) 1 + ax + bx2( ) a0 + a1 + a2 + .... + a2n = 36 = 729
1 −18 C 2x +18 C 2x −18 C 2x +18 C 2x 4
2( ) 3( ) 4( )
2 3
1
Coefficient of x3 is
JEE Advanced/Boards
−18 C3 23 + a ( ) ( 18
C2 × 4 − b ) ( 18
C1 × 2 =
0 ) …(i)
Coefficient of x4 is
Exercise 1
18
( ) (
C 4 24 + a −18 C3 × 23 +18 C2b22 =
0 ) ….(ii) Sol 1: f(x)=1–x+x2–x3+……x16–x17
(1 − 2 x ) =C − C (2 x ) ( )
50 1 2
50 50
+50 C2 2 x 2 – 6x + … = 2a2 + 6a3(1 + x) + …
0 1
− C (2 x ) + C (2 x )
50
3
50
4 Putting x = –1
3 4
M a them a ti cs | 4.55
8×7×6
(2(16) + 1)16(16 + 1) 16(16 + 1) = × (4 x + 44)(2x −1 + 7)=
−1
336
= + = 1632 1.2.3
6 2
4 x + 44 336
⇒ a2 = 816 ⇒ = = 6
2 x −1
+ 7 8x7
10
x 3 ⇒ 4 x + 44 = 6 × 2x −1 + 6 × 7 = 3.2x + 42
Sol 2: (a) (i) +
3 2x2
(2x)2–3(2)x+44–42=0
10 −r r
10
x 3 Assume 2x = y
Tr +1 = Cr 2
3 2x
y2 – 3y + 2 = 0 ⇒ (y – 2)(y – 1) = 0
10 −r 10 −r r
1
10
−2r 3 ⇒ y = 1 or y = 2 ⇒ x = 0 or x = 1
= Cr x 2
3 2
11
2 1
For term independent of x- Sol 3: ax +
bx
10 − r r
0 ⇒ 10 − r − 4r =0 ⇒ r = 2
− 2r = 1
2 Tr +1 = 11Cr (ax2 )11−r
10 −2 2 bx
10 1 3
So=
T3 T= C2 x 2(11 − r) + r( −1)
Power of = = 7 (given)
2 +1
3 2
⇒ 22–2r–r=7 ⇒ r = 5
10 × 9 1 3 5
= × × = 1
5
2 3 4 4 12 Coefficient T5+1 = C5 (a) 11 11 −5 11
C5a6b−5
=
b
8
1 11
(ii) x1/3 + x −1/5 1
2 (ii) ax −
bx2
8 −r
1 r
Tr +1 = Cr x1/3
8
(x −1/5 )r 11 11 −r 1
2 =Tr +1 Cr (ax) − 2
bx
8 −r r
Power of x = − =0 for independence Power of x = 11 – r – 2r = –7 (given)
3 5
⇒ 5(8 – r) – 3r = 0 ⇒ 40 – 5r – 3r = 0 ⇒ r = 5 ⇒ r=6
6
8 −5 1 11
8 1 8×7×6 1
3 Coefficient T=
r +1 T=
6 +1
11 11 −6
C6 a − = C6 a5b−6
T5+=
1 T=
6 C5 (x1/3 ) .=
(x )–1/5 5
× = 7 b
2 1.2.3 2
(iii) Given that both coefficient are equal
8
2 log 4 x + 44
5 1 ⇒ 11
C5a6b−5 = 11C6 a5b−6 ⇒ ab = 1
(b) 55 +
3
log5 2x −1 + 7
5
Sol 4: (a) (1+x)14
= (a1+a2)8 assume
Coefficients Tr=T(r-1)+1= 14Cr–1
8
T4 T=
= 3+1 C3 (a1 )8 −3 (a2 )3
4 . 5 6 | Binomial Theorem
1 1 2 0 ⇒ r =5
⇒ 10 − 2r =
⇒ + =
(15 − r)(14 − r) (r + 1)r r(14 − r)
10
Coefficient = C5 ( −2)5 =
− 10 C5 25
r(r + 1) + (15 − r)(14 − r) 2
⇒ = 10
C5 1 1
r(r + 1)(14 − r)(15 − r) r(14 − r) Ratio of both coefficients = = =
10
C5 25
25 32
⇒ r2+r+210–14r–15r+r2=2(r+1)(15–r)
Sol 8: sn= 1 + q + q2 + …… + qn =
Sol 5: a = Coefficient of x3 in (1+x+2x2+3x3)4
2 n
b = Coefficient of x3 in (1+x+2x2+3x3+4x4)4 q + 1 (q + 1) q+1
Sn =
1+ + +... + ,q ≠ 1
4x4 has no effect on the coefficient of x3.
2 2 2
Constant term
Sol 6: (1–x2)10
T= 10
Cr ( −x2 )r
n+1
C1+n+1C2+……+n+1Cn+1 = 2n+1–1
r +1
2 n
1 1 1 Compare x → −x in
In Sn constant term = 1 + + + .....
2 2 2 (x2+x+1) → (x2–x+1) a0 – a1x + a2x2 – a3x3 + ….. + a2x x2x
n+1
1
1− n+1 [(1+x+x2)(x2–x+1)]n
2 1 (2n+1 − 1)
= = 2 1 − =
1 2 2n = a20 x2n − a12 x2n − a3 x2n + a4 ⋅ x2n − ..... + a2n x2n
1−
2
(2x +1 − 1) ∴ For x = 1
So (2n+1 − 1) =
(2n ),
2n a20 − a12 − a32 + .... + a2n
2
3n
=
n+1
C1 + n+1C2S1 + n+1C3Sn =
2n Sn
Sol 11: (5+3x)10
We can prove this with other terms also.
T4=10C3(5)10–3(3x)3
3
Sol 9: (i) (2 + 3x)9 , x = . Now we have =10C35733x3
10
C3 5733 x3 is the greatest term
2
n +1 9 +1 10 n +1 10 + 1
= = So, =
a 2 × 2 4 a 5
1+ 1+ 1+ 1+ 1+
x 3×3 9 x 3x
n n
Sol 16: f (n) = ∑∑ k Cr x3 ⇒ 2 nC1 + nC3 =
a3
r −0 k −r
n(n − 1)(n − 2)
11 11
2n + a3
= …. (iii)
1.2.3
For f (11 ) = ∑∑ Cr k
+ 1C1 + 0 C0 n
11 10 9 1 0
Sol 19: ∑ nCk sin kx. cos (n-k) x = 2n−1 sin nx
= 2 +2 + 2 + ....... + 2 + 2 k =0
n
=
211+1 − 1
= 4095 = 4095 = 51.32 71. 131
L. H. S. = ∑ nCk sinkx cos(n − k)x
2 −1 k =0
No. of divisors = (1+1). (2+1). (1+1)(1+1) We know that 2 sin A cos B = sin(A+B) +sin (A–B)
= 2×3×4 = 24 A + B = kx + (n – k)x = kx + nx – kx = nx
A – B = kx – (n – k )k = kx – nx + kx = 2kx – nx
11 11
Sol 17: ∑∑ i C j So, ∑
n
1n
Ck [sinnx + sin(2kx − nx)]
j−0 i− j
k =0 2
= 0
C0 + ( 1
C0 +1 C1 + ) ( 2
C0 + .... +2 C2 ) =∑
n
1n n
1
Ck sinnx + ∑ nCk sin(2kx − nx)
+ ( 3
C0 + .... +3 C3 + ..... +) ( 11
)
C0 +11 C1 + .... +11 C11=k 2
0= k 02
n
= 20 + 21 + .... + 211 1 1
= sinnx
2
∑ nCk + 2
k =0
= 212 − 1
[ nC0 sin( −nx) + ...... + nCn sin(nx)]
n+ 4
∑ ak .xk
Sol 18: (1 + x2 ).(1 + x)n =
1
k =0 = sinnx.2n + 0 = (sinnx)2n−1 = 2n−1 sinnx
a1, a2 and a3 are in A.P 2
2001
(1+x4+2x2)(nC0+nC1x+nC2x2+nC3x3+……nCnxn) 2001 1
Sol 20: x + − x 0
=
= a0+a1x+a2x2+a3x3+…. 2
k =0 - (6 6 − 14)2n+1
⇒ S = ‒ nA
= 2( nC1 (6 6 )2n+1 14
10
+ nC3 (6 6 )2n−11−3 143 + ......)
Sol 22: (A) ( ∑ P(i,10 − i)
i=0
= 2K (K is const. integer)
y
0 ≤F−e <1
F – e=2K–I= Integer
F – e = 0 = e = F = F= e= (6 6 − 14)2n+1
(20)2n+1
F=
(6 6 + 14)2n+1
x
(I + F)F= (6 6 + 14)2n+1
P(0,10)+P(1,9)+……+P(10,0) 202n+1
10 × 9 (6 6 + 14)2n+1
= 1 + (9 + 1)(1) + + ....
2
202n+1
(I + F)F =
10
= C0 − 10 C1 + 10 C2 + 10 C3 + ...... + 10 C10
= 210 = 1024
M a them a ti cs | 4.61
n
C0 xn + C1 xn−1 + ...... + Cn =(x + 1)n Cr n−r +1
We know that =
n
Cr −1 r
= and compare xn and coefficient
n
r Cr
C0 + 3C12 + 5C22 + ...... + (2n + 1)Cn2 ⇒ = (n − r + 1)
n
Cr −1
= Coefficient of xn in (n+1) (1+x)n+n
L. H. S. = (n − 1 + 1) + (n − 2 + 1)
= (n+1)2nCn =
+(n − 3 + 1) + .......(n − n + 1)
L. H. S. = R. H. S.
n2 + n − (1 + 2 + 3 + ....n)
0<F+F<2 (0)n+1 − 1 C C
= C0 ( −1) + 1 − 2
n+1 2 3
F+F is 1 only integer between0and 2
1 Cn C1 C2
I + 1 = 2F ==
P (I + 1) +...... + ( −1)n+1 = C0 − + + .......
2 n+1 2 3
I + F − F = (3 + 5)n − (3 − 5)n Cn 1
+( −1)n =
n+1 n+1
I +F +F −F −F =2( nC1 3n−1
Sol 29: (a) In equation (ii) compare coefficient of xn−1
( 5)1 + nC3 3n−3 ( 5)3 + ........)
2n
I + 2F − (F + F) =2σ Cn−1 = C0 C1 + C1C2 + ....... + Cn−1Cn
I + 2F − 1 = 2σ 2n 2n!
Cn−1 =
(n − 1)!(n + 1)!
1
=
σ (I + 2F − 1)
2 2n–(n–1) = n+1
L. H. S. = R. H. S.
M a them a ti cs | 4.63
(d) 100
C10 + 5. 100 C11 + 10. 100 C12 (1 + x2 +=
x 4 )n a0 a2 x2n−2 − a1a3 x2n−2 + ....
+10. 100 C13 + 5. 100 C14 + 100 C18 Compare x2n−2 coefficient
an+1 = an−1 = a0 a2 − a1a3 + ...
105 105 105 105!
= C90
= C105
= −90 C15
=
90!15! ( in (1 + x + x2 )n x → x2
2(n−1)
100! 5100! 10 × 100! So Coefficient of x = a= n−1 an+1 )
L. H. S. = + +
90!10! 11!89! 12!88! (iii) (1 + x + x2 )n = a0 + a1 x + ..... + a2n x2n
15 × 14 × 13 × 12 × 11 x=w
100! +
90!15! 0= a0 + a1 ω + a2 ω2 + a3 + ....a2nω2n . . 2
5 × 90 × 15 × 14 × 13 × 12 10 × 90 × 89 × 15 × 14 × 13 x = ω2
+
15!90 15!90!
0= a0 + a1 ω2 + a2 ω + .... + a2nω4n …3
10 × 15 × 14 × 90 × 89 × 88 5.90 × 89.8887
+ + A +B + C =
70!15! 15!90!
3n = 3(a0 + a3 + a6 + ....)
90.89.88.87.86
+
90!15! a0 + a3 + a6 + ......3n−1 … (i)
x(1 + x + x2 )n = a0 x + a1 x2 + ...... + a2n x2n+1 …. A
100 × 101 × 102 × 103 × 104 × 105 = 105!
= 90!15!
90!15! x = ω = 0= a0 ω + a1 ω2 + a2 + ..... + a2nω2n+1 …B
= 105
C15
= 105
C90 x = ω2 = 0= a0 ω2 + a1 ω + a2 + ..... + a2nω4n+ 2 . . C
r =0
Assume P = Q
+Cr (C0 + C1 + ..... + C100 )]
(n + 1)!
P + Q = 2P = 2Q = (n + i)! =P = Q =
2
= 101 ∑ Cr2 + 101( 2nCn ) + ∑ Cr(2n )
Sol 33: S=
1 ∑< ∑ CiC j.
= 101 2nCn + 101 2nCn + 2n (2n ) 0 ≤i j≤100
= 202 2nCn+2100+100 S=
2 ∑ < ∑ CiC j
0≤ j i≤100
2n 200 2n P
202 Cn + 2 = m( Cn ) + 2
= S=
3 ∑
= ∑ CiC j
0 ≤i j≤100
n=100, m=202, P=200
(1 + x)100 ⇒ n = 100
Hence, n+m+p = 200+100+202=502
ab
S1 + S2 + S3 =
+( −1)n−1 n−1Cn−1
8
Sol 1: (B) Given binomial is (21/3+3–1/3)n Sol 5: (C) t6 in x −8/3 + x2 log10
x
5600
=
∴ T7 = T6+1= nC6(21/3)n–6(3–1/3)6
( ) ( x log ) = 5600
3
8
C5 x −8/3 2 x
10
n −1/3 n−6 1/3 6
T7 ' from end = Cn−6 (3 ) (2 )
(log ) =
5
1T7n
C6 2n/32−23−2 (2.3)n/3 ⇒ x2 x
10 100
⇒ = = = = (6)(n/3)− 4
T7 ' 6 n
C6 3−n/33222 (6)2+ 2 ⇒x=
10
1 n
6(n/3)− 4 = ⇒ − 4 =−1 ⇒n=9
6 3
Sol 6: (B) (α + p)m−1 + (α + p)m−2 (α + q)
Sol 2: (C) We have 1523+2323 = (19–4)23+(19+4)23 +(α + p)3 (α + q)2 + ...... + (α + q)m−1
Coefficient of t
= 2 23C0 1923 + 23C2 1921 + ... + 23C22 19
m−1
α + q α + q
= 2. 19K always divisible by 19
+ ...... + α + p
m−1
= (α + p) 1 +
α +p
So the remainder is zero
m
1 − α + q
n n 2 n n−1
Sol 3: (D) 4{ C1 + 4 C2 + 4 . C3 + ...... + 4 }
= (α + p)m−1 α +p
= {4 nC1 + 42 nC2 + 43 nC3 + ..... + 4n nCn }
1 − α + q
α + p
= (1 + x)n = C0 + C1 x + C2 x2 + ..... + xn nCn
At x = 4 (α + q)
n 2 1 − α + p
5 =
1 + 4C1 + 4 C2 + ... (α + p)
= (α + p)m−1
4n
α +p−α −q
So C1 + 42 nC2 + 43 nC3 + ..... + 4n nCn = 5n − 1
m
1 − α + q
Sol 4: (A) n ≥ 3
α + p = (α + p) − (α + q)
m m
= (α + p)m
(n − 1) (n − 1)(n − 2) p−q p−q
n− (n − 1) + (n − 2)
1! 2!
m
(n − 1)(2 − n)(n − 3) Ct [pm−t − qm−t ]
− (n − 3) + ...... Coefficient of α t =
3! p−q
At n = 3
( )
2145
1.3 − (3 − 1) (3 − 1)(3 − 2) Sol 7: (B) 1 + x − 3x2 = a0 + a1 x + a2 x2 + ...
= (3 − 1) + (3 − 2) − 0
1 2!
Put x = - 1
4 . 6 6 | Binomial Theorem
9
4x2 3
Sol 8: (B) − Sol 12: (D) aC0 + ( a + b ) C1 + .... + ( a + nb ) Cn
3 2x
9 −r
= C0 + C1 + .... + Cn
r
4x2 3 = C0 + C1 + .... + Cn
=Tr +1 9 Cr − +b 0 × C0 + 1 × C1 + 2 × C2 + .... + nCn
3 2x +b 0 × C0 + 1 × C1 + 2 × C2 + .... + nCn
Power of x= 2(9 − r) + ( −1)r = a2n + bn2n−1
⇒ 18 − 2r − r = 18 − 3r =
6 (given)
= ( 2a + nb ) 2n−1
⇒ r 12
⇒ 3r = 18 − 6 = 12= = /3 4
9− 4 4 5 4
9 4 −3 9×8×7×6 4 3
Coefficient C4 = Previous Years’ Questions
3 2 1.2.3.4 3 2
( )
n 2n−1
∑
= (n Cr ) (n Cn − r ) [ nCr nCn − r ] = Coefficient of x2n−1 in 2n 1 − x2 (1 − x )
r =0
M a them a ti cs | 4.67
n−1 ( 2n−1) n
= 2n ( −1)
∑ ( −1)
r n
. Cn−1 Sol 3: Cr
r =0
= ( −1 ) ( 2n)
n ( 2n − 1) !
1 3r 7r 15r
(n − 1) !n! r + 2r + 3r + 4r + ....upto mterms
2 2 2 2
(2n)!
= − ( −1 ) n ⋅
n
⋅n … (iii) n r
(n!)2 1
∑ ( −1 )
r n
Cr +
r =0 2
= −( −1)n n ⋅ Cn r r
n n
3 7
( −1 ) Cr + ∑ ( −1 ) n Cr + ...
r r n
1 ∑ 4
Again, the coefficient of on the RHS =r 0= r 0 8
x
Upto m terms
(
= − C12 − 2.C22 + 3.C32 + .... + −2nC22n
… (iv)
n
) n
using∑ ( −1 ) n Cr xr =( 1 − x)
r
From Eqs. (iii) and (iv), we get
r =0
( −1) n.Cn
n
C12 − 2.C22 + 3.C32 − .... − 2n.C22n = 1 3
n
7
n n
= 1 − + 1 − + 1 − + .....
2 4 8
Sol 2: n+1 C1 + n+1
C2s1 + n+1
C3s2
Upto m terms
n+1 n n n
1 1 1
+.... + n+1
Cn+1sn =∑ n+1 Cr sr −1 = + + + .....
r =1 2 4 8
1 − qn+1
Where sn = 1 + q + q2 + .... + qn = Upto m terms
1−q
1
m
n+1 1 − qr 1 − n
∴ ∑ n+1 Cr 1
n
2 2mn − 1
1−q = =
( )
r =1
2 1 − 1 2mn 2n − 1
1 n+1 n+1 n+1 2n
∑ Cr − ∑ Cr q
n+1 r
1 − q r 1=r 1
=
Sol 4: Let y = (x – a)m, where m is a positive integer,
1 r ≤m,
= (1 + 1 )n+1 − (1 + q)n+1
1−q
dy
Now, = m ( x − a)
m−1
dx
1 n+1
2 − (1 + q )
n+1
= … (i)
1 − q d2 y
= m (m − 1 )( x − a)
m− 2
⇒
2
dx
2 n
q+1 q+1 q+1 d4 y
Also,=
Sn 1 + + ..... + ⇒ = m (m − 1 )(m − 2 )(m − 3)( x − a)
m− 4
2 2 2 dx 4
q+1
n+1 …………………………………..
1−
2 2n+1 − (q + 1)n+1 On differentiating r times, we get
= … (ii)
q+1 2n (1 − q)
1− dr y
= m (m − 1 ) .... (m − r + 1 )( x − a)
m−r
2 r
dx
From eqs. (i) and (ii), we get
n+1
Cr + n+1
C2s1 + n+1
C3s2 +.... + n+1
Cn+1sn =n
2 sn
=
m!
(m − r ) !
( x − a)
=
m−r
r! ( C ) ( x − a)
m
r
m−r
dr y
And for r > m, =0
dxr
4 . 6 8 | Binomial Theorem
(1 + x )
Now, n
2
+ x4
2n 2n =
∑ ar ( x − 2=
) ∑ br ( x − 3)
r r
(given) x2n
=r 0=r 0
Thus, a20 − a12 + a22 − a32 + .... + a2n
2
On differentiating both sides n times w.r.t. x, we get
= Coefficient of the term independent of x in
2n
∑ ar (n!) Cn ( x − 2 )
r r −n
1
(1 + x )
n
2
r =n + x4
2n
x
( )
2n n
∑ br (n!) Cn ( x − 3)
r r −n
= Coefficient of x2n in 1 + x2 + x 4
r =n
( )
n
On putting x = 3, we get = Coefficient of tn in 1 + t + t2 =an
2n
∑ ar (n!) Cn = (bn ) n!
r
Sol 6: To show that
r =n
n! r =n
( −1)
k n
= 2n + 1
Cn Ck n−k C0 =
n
Ck
= 2n + 1
Cn + 1 Taking LHS
( )
n
Sol 5: 1 + x + x2 =a0 +a1 x + .... + a2n x2n … (i)
k
∑ ( −1)
r
Replacing x by –1/x, we get = .2k −r . n Cr . n−r Ck −r
r =0
n
1 1
1 − + 2
k
n! (n − r ) !
∑ ( −1 )
r
x x = .2k −r . .
r =0 r!(n − r)! (k − r ) ! (n − k ) !
a1 a2 a3 a2n
= a0 − + − + .... + … (ii) k
n! k!
∑ ( −1)
r
x x2 x3 x2n = .2k −r . .
r =0 (n − k )!k! r! (k − r )!
Now, a20 − a12 + a22 − a32 + .... + a22n = coefficient of the
k
∑ ( −1)
r
term independent of x in = .2k −r . n Ck . k Cr
r =0
a + a x + a x2 + .... + a x2n
0 1 2 2n
k r 1
a a a = 2k . n Ck ∑ ( −1 ) . . k Cr
× a0 − 1 + 2 − .... + 2n r =0 2r
x x 2
x2n
k
1
= Coefficient of the term independent of x in = 2k . n Ck 1 − = n Ck = RHS
2
n
( ) 1 2
n
1 + x + x2 1 − + 2
10
x x Sol 7: Let y = ∑ Ar (B10 Br − C10 Ar )
r =1
n
( ) 1 1
n 10
2
Now, RHS = 1 + x + x 1 − + 2 ∑ ArBr = coefficient of x 20
in ((1 + x)10 (x + 1)20) – 1
x x
r =1
(1 + x + x ) ( x )
n n
2 2
− x +1 10 2
= = C20 – 1 = C10 – 1 and ∑ ( Ar ) = coefficient of x10 in ((1
x2n + x)10 (x + 1)10) – 1 = B10 – 1r =1
n
2
( 2
)
2
x +1 − x ⇒ y = B10(C10 – 1) – C10(B10 – 1) = C10 – B10.
(1 + 2x2 + x 4 − x2 )n
= =
2n 2n
x x
M a them a ti cs | 4.69
MATHEMATICS
FOR JEE MAIN & ADVANCED
SECOND
EDITION
Exhaustive Theory
(Now Revised)
Formula Sheet
9000+ Problems
based on latest JEE pattern
1. INTRODUCTION
The main subject of this chapter is counting. Given a set of objects the problem is to arrange some or all of them
according to some order or to select some or all of them according to some specification.
Illustration 1: There are three stations, A. B and C. Five routes for going from station A to station B and four routes
for going from station B to station C. Find the number of different ways through which a person can go from A to
C via B. (JEE MAIN)
Sol: This problem is an application of the Fundamental Principle of Counting. The rule of product can be used to
solve this question easily.
Given there are five routes for going from A to B and four routes for going from B to C.
A C
B
Figure 5.1
Hence, by the fundamental principle of multiplication the total number of different ways
= 5 x 4 (i.e., A to B and then B to C) = 20 ways
Illustration 2: A hall has 12 gates. In how many ways, can a man enter the hall through one gate and come out
through a different gate. (JEE MAIN)
There are 12 ways of entering the hall. After entering the hall the man can come out through any of 11 different
gates.
Hence, by the fundamental principle of multiplication, the total number of ways are 12 x 11 = 132 ways.
Illustration 3: How many numbers between 10 and 10,000 can be formed by using the digits 1, 2, 3, 4, 5 if
(i) no digit is repeated in any number. (ii) digits can be repeated. (JEE MAIN)
Sol: The numbers between 10 and 10,000 can be either two digit, three digit or four digit numbers. We consider
each of these cases and try to find the number of possibilities using 1, 2, 3, 4 and 5. Finally, we add them up to get
the desired result.
(i) Number of two digit numbers = 5 x 4 = 20
Number of three digit numbers = 5 x 4 x 3 = 60
Number of four digit numbers = 5 x 4 x 3 x 2 = 120
Total number of numbers = 20 + 60 +120 = 200
(ii) Number of two digit numbers = 5 x 5 = 25
Number of three digit numbers = 5 x 5 x 5 = 125
Number of four digit numbers = 5 × 5 × 5 × 5 = 625
Total number of numbers = 25 +125 +625 = 775
3. FACTORIAL NOTATION
An efficient way of writing a product of several consecutive integers is the factorial notation. The number n! (read
as “n-factorial”) is defined as follows :
For any positive integer n; n! = n(n – 1)(n – 2) …….. (3)(2)(1); For instance, 4! = 4.3.2.1 = 24
Note: (i) n! = n(n – 1)(n – 2) ………… 3.2.1 ; n! = n.(n – 1)! ; 0! = 1! = 1; (2n)! = 2n.n![1.3.5.7 ……… (2n – 1)]
(ii) n! = n(n – 1)! = n(n – 1)(n – 2)! = n(n – 1)(n – 2) (n – 3)!
n!
(iii) n(n – 1) …….. (n – r + 1) =
(n − r)!
Illustration 4: Find the sum of n terms of the series whose nth term is n!×n. (JEE ADVANCED)
Sol: Represent the general term in this question as a difference of two terms and then add them up to find the
answer.
The required sum = (1)! + 2(2)! + 3(3)! + ……. + n(n!) = (2 – 1)! + (3 – 1)2! + (4 – 1)3! + …….. + [(n + 1) – 1]n!
= (2! – 1!) + (3! – 2!) + (4! – 3!) + …….. + [(n + 1)! – n!] = (n + 1)! – 1
4. PERMUTATION
Each of the different arrangements, which can be made by taking some or all of a number of objects is called
permutation. The number of permutations of n different objects taken r at a time is represented as
n!
n
Pr = = n(n – 1)(n – 2) ……… (n – r + 1) (where, 0 ≤ r ≤ n)
(n − r)!
Note: (i) In permutation, the order of the items plays an important role.
(ii) The number of all permutations of n distinct objects taken all at a time is n!
M a them a tics | 5.3
∴ 41
P2 = 41 x 40 = 1640
Illustration 6: Three men have 4 coats, 5 waist coats and 6 caps. In how many ways can they wear them?
(JEE ADVANCED)
Sol: Use the concept and understanding of Permutation, i.e. arrangement to find the answer.
The total number of ways in which three men can wear 4 coats is the number of arrangements of 4 different coats
taken 3 at a time. So, three men can wear 4 coats in 4P3 ways. Similarly, 5 waist coats and 6 caps can be worn by
three men in 5P3 and 6P3 ways respectively. Hence, the required no. of ways = 4P3 × 5P3 × 6P3 = (4!) × (5 × 4 × 3) ×
(6 × 5 × 4) = 172800.
Illustration 7: Suppose 8 people enter an event in a swim meet. In how many ways could the gold, silver, and
bronze prizes be awarded? (JEE ADVANCED)
Sol: Use the formula for nPr. The required number of ways is an arrangement of 3 people out of 8 i.e.
8! 8.7.6.5. 4 .3.2.1
8
P3 = = = 8 . 7 . 6 = 336.
5! 5. 4 .3.2.1
PLANCESS CONCEPTS
The following two steps are involved in the solution of a permutation problem:
Step 1: Recognizing the objects and the places involved in the problem.
Step 2: Checking whether the repetition of the objects is allowed or not.
Uday Kiran G (JEE 2012, AIR 102)
Illustration 8: A student appears in an objective test which contains 10 multiple choice questions. Each question
has four choices in which one is the correct option. What maximum number of different answers can the student
give? How will the answer change if each question may have more than one correct answers? (JEE ADVANCED)
PLANCESS CONCEPTS
The mn or nm dilemma
Let us start with an example.
Q. There are 7 letters and 5 letter-boxes. In how many ways can you put the letters in the boxes?
Sol: This is the typically confusing question asked frequently from the P & C area. Let’s see how you can
solve this type of question.
The Exhaustive Approach: One way to solve this question is through (what we will call) The Exhaustive
Approach. While solving such problems, first decide which of the items (letters and letter-boxes here) is
exhaustive. Exhaustive here means the entity which is sure to be used up completely.
In this example, all the “letters” are sure to be placed in the boxes, whereas there is no such constraint as
regards the “letter-boxes”. Some boxes could go empty. Having decided this, just go by the options we
have for all the instances of the exhaustive item and you have your answer.
As you can see here, every letter has 5 boxes to choose from. Thus the total would
be (5 x 5 x........7 times) = (57)
A similar question could be: In how many ways can 10 rings be worn on 5 fingers? Try it yourself.
Chinmay S Purandare (JEE 2012, AIR 698)
Illustration 9: Determine the number of permutations of the letters of the word ‘SIMPLETION’ taken all at a time.
(JEE MAIN)
Sol: In the given word the letter I occurs twice and the remaining letters occur only once. So, the concept of
Permutation of Alike Objects is used to find out the answer.
There are 10 letters in the word ‘SIMPLETION’ and out of these 10 letters two are identical. So, just selecting all 10
objects at a time will give twice the actual result. Hence, the number of permutations of taking all the letters at a
time = 10P10 / 2! = 10! / 2! = 181440.
(a) First of all, put the m objects for which there is no restriction, in a line. These m objects can be arranged in
m! ways.
(b) Then count the number of gaps between every two m objects for which there is no restriction, including the
end positions. Number of such gaps will be (m + 1).
(c) If m is the number of objects for which there is no restriction and n is the number of objects, two of which are
not allowed to occur together, then the required number of ways = m! × m+1Cn × n!
The number of permutations when two types of objects are to be arranged alternately
(a) If their numbers differ by 1 put the object whose number is greater in the first, third, fifth.... places, etc. and
the other object in the second, fourth, sixth.... places.
(b) If the number of two types of objects is same, consider two cases separately keeping the first type of object
in the first, third, fifth places, etc. and the second type of object in the first, third, fifth places.... and then add.
Sol: The numbers between 300 and 3000 can either be a three digit number or a four digit number. The solution is
divided into these two different cases and their sum will give us the desired result.
Any number between 300 and 3000 must be of three or four digits.
Case I: When number is of three digits: The hundreds place can be filled by any one of the three digits 3, 4 and 5
in 3 ways. The remaining two places can be filled by the remaining five digits in 5P2, ways.
5!
∴ The number of numbers formed in this case = 3 × 5P2 = 3 × =60
3!
Case II: When number is of four digits: The thousands place can be filled by any one of the two digits 1 and 2 in 2
ways and the remaining three places can be filled by the remaining five digits in 5P3 ways.
5!
∴ The number of numbers formed in this case = 2 × 5P3 = 2 × = 120
2!
∴ Total numbers = 60 + 120 = 180
Illustration 11: How many words can be formed from the letters of the word ARTICLE, so that vowels occupy the
even places? (JEE MAIN)
Sol: Clearly, this is an example of Permutation under Restriction. We identify the even places and the odd places
and try to find the number of ways in which the vowels and consonants can fill the spaces.
There are seven places: 3 even and 4 odd in which we have to fill 3 vowels and 4 consonants.
∴ The number of words = 3P3 . 4P4 = 3! × 4! = 6 × 24 = 144.
Illustration 12: How many different words can be formed with the letters of the word ORDINATE so that
(a) Four vowels occupy the odd places (b) Beginning with O (c) Beginning with O and ending with E. (JEE MAIN)
Sol: The concept of Permutation under Restriction can be used to solve this problem.
There are 4 vowels and 4 consonants. Total 8 letters.
(a) No. of words = 4! × 4! = 24 × 24 = 576. Because 4 vowels are to be adjusted in 4 odd place and the 4 consonants
in the remaining 4 even places.
5 . 6 | Permutations and Combinations
Illustration 13: Find the number of ways in which 5 boys and 5 girls can be seated in a row so that
(a) No two girls may sit together.
(b) All the girls sit together and all the boys sit together
(c) All the girls are never together. (JEE ADVANCED)
Sol: Since the number of girls and the number of boys are equal they have to sit alternately. This can be used to
solve (a). For (b), we keep the girls together and arrange the boys in five places. Also, the girls can be arranged
amongst themselves in 5! ways. This gives us the number of arrangements. Use the answer of the second part to
find (c).
(a) 5 boys can be seated in a row in 5P5 = 5! ways. Now, in the 6 gaps between 5 boys, the 5 girls can be arranged
in 6P5 ways. Hence, the number of ways in which no two girls sit together = 5! × 5P5 = 5! × 6!
(b) The two groups of girls and boys can be arranged in 2! ways. 5 girls can be arranged among themselves in
5! ways. Similarly, 5 boys can be arranged among themselves in 5! ways. Hence, by the fundamental principle of
counting, the total number of requisite seating arrangements = 2!(5! × 5!) = 2(5!)2.
(c) The total number of ways in which all the girls are never together = Total number of arrangements – Total
number of arrangements in which all the girls are always together = 10! – 5! × 6!
Illustration 14: The letters of the word OUGHT are written in all possible orders and these words are written out
as in a dictionary. Find the rank of the word TOUGH in this dictionary. (JEE MAIN)
Sol: The word TOUGH will appear after all the words that start with G, H and O. Then we look at the second letter
of the words starting with T and then third. Hence, the rank of the word TOUGH will be one more than the sum of
all the possibilities just mentioned.
Total number of letters in the word OUGHT is 5 and all the five letters are different, the alphabetical order of these
letters is G, H, O, T, U.
Number of words beginning with G = 4! = 24
Number of words beginning with H = 4! = 24
Number of words beginning with O = 4! = 24
Number of words beginning with TG = 3! = 6
Number of words beginning with TH = 3! = 6
Number of words beginning with TOG = 2! = 2
Number of words beginning with TOH = 2! = 2
Next come the words beginning with TOU and TOUH is the first word beginning with TOU.
∴ Rank of ‘TOUGH’ in the dictionary = 24 + 24 + 24 + 6 + 6 + 2 + 2 + 1 = 89
Illustration 15: There are 21 balls which are either white or black and the balls of same color are alike. Find the
number of white balls so that, the number of arrangements of these balls in a row is maximum.(JEE ADVANCED)
Sol: The property of a binomial coefficient can be used to solve this question.
Let there be r white balls so that the number of arrangements of these balls in a row be maximum. Number of
arrangements of these balls is
21! 21 + 1 21 – 1
A= A will be maximum when r = or i.e. 10 or 11
r!(21 − r)! 2 2
M a them a tics | 5.7
Illustration 16: The number plates of cars must contain 3 letters of the alphabet denoting the place and area to
which its owner belongs. This is to be followed by a three-digit number. How many different number plates can be
formed if:
(i) Repetition of letters and digits is not allowed. (ii) Repetition of letters and digits is allowed. (JEE ADVANCED)
PLANCESS CONCEPTS
5. COMBINATION
Each of the different groups or selection which can be made by some or all of a number of given objects without
reference to the order of the objects in each group is called a combination.
n!
The number of all combinations of n objects, taken r at a time is generally denoted by C(n, r) or nCr =
n
pr r!(n – r)!
(0 ≤ r ≤ n) =
r!
Note:
(a) The number of ways of selecting r objects out of n objects, is the same as the number of ways in which the
remaining (n - r) can be selected and rejected.
(b) The combination notation also represents the binomial coefficient. That is, the binomial coefficient nCr is the
combination of n elements chosen r at a time.
5 . 8 | Permutations and Combinations
Permutations Combinations
Different orderings or arrangement of the r objects are Each choice or subset of r object give one combination. Order
different permutations within the group of r objects does not matter.
n! n!
Pr =
n
Cr =
n
(n – r)! (n – r)!r!
Clue words: arrangement, schedule, order Clue words: group, committee, sample, selection, subset.
Illustration 17: A basketball coach must select two attackers and two defenders from among three attackers and
five defenders. How many different combinations of attackers and defenders can he select? (JEE MAIN)
Sol: The number of ways two attackers and two defenders can be selected is 3C2 and 5C2 respectively.
5. 4 .3.2.1
∴ He can select in 3C2 × 5C2 = = 30 different combinations.
2.2
Illustration 18: A soccer team of 11 players is to be chosen from 30 boys, of whom 4 can play only in goal, 12 can
play only as forwards and the remaining 14 in any of the other positions. If the team is to include five forwards and
of course, one goalkeeper, in how many ways can it be made up? (JEE MAIN)
Consider the objects A1, A2, A3,.....,Ap, Ap+1, …........, An. If the p objects A1, A2......Ap are to be excluded then we will
have to select r objects from the remaining n – p objects (Ap+1, Ap+2, …….. An).
Hence the required number of ways = (n–p)Cr
(b) Number of ways of choosing r objects out of n different objects if p particular objects must be included
(p ≤ r).
Consider the objects A1, A2, A3 ....... Ap, Ap+1, ……. An. If the p particular objects A1, A2, ...... Ap (say) must be
included in the selection then to complete the selection, we must select (r – p) more objects to complete the
selection. These objects are to be selected from the remaining n – p objects.
Hence, the required number of ways = n–pCr–p
(c) The total number of combinations of n different objects taken one or more at a time = 2n - 1.
M a them a tics | 5.9
Illustration 19: There are 4 oranges, 5 apples and 6 mangoes in a fruit basket. In how many ways can a person
select fruits from among the fruits in the basket? (JEE MAIN)
Illustration 20: How many four digit numbers are there whose decimal notation contains not more than two
distinct digits? (JEE MAIN)
Sol: A four digit number can consist of either only one digit or two digits as per the question. Clearly, there are nine
four digit numbers with the same digit. Similarly, calculate the number of four digit numbers with two distinct digits
and hence the sum gives us the desired result.
Evidently any number so formed of four digits contains
(i) Only one digit (like 1111, 2222,...) and there are 9 numbers. (ii) Two digits
(a) if zero is one of the two, then the one more can be anyone of the nine, and these two digits can be arranged
in 9C1 [3C1 + 3C2 + 3C2 + 3C3] = 63.
(b) if zero is not one of them, then two of the digits have to be selected from 9, and these two can be arranged
in 9C2[4C1 + 4C2 + 4C3] = 504
Hence, the total number of required numbers = 567.
Illustration 21: In how many ways can a cricket team of eleven players be chosen out of a batch of 15 players, if
(a) there is no restriction on the selection
(b) a particular player is always chosen
(c) a particular player is never chosen (JEE MAIN)
5 . 1 0 | Permutations and Combinations
Sol: Using the concept of combination of alike objects we can get the answer.
15 × 14 × 13 × 12
(a) The total number of ways of selecting 11 players out of 15 is = 15C11 = 15C15–11 = 15C4 = = 1365
4 ×3× 2×1
(b) A particular player is always chosen. This means that 10 players are selected out of the remaining 14 players.
∴ The required number of ways = 14C10 = 14C14–10 = 14C4 = 1001
Illustration 22: In a plane there are 37 straight lines, of which 13 pass through the point A and 11 pass through
the point B. Moreover, no three lines pass through one point, no line passes both points A and B. and no two are
parallel. Find the number of points of intersection of the straight lines. (JEE MAIN)
Sol: Two non parallel straight lines give us a point of intersection. Using this idea we find the total number of points
of intersection. Care should be taken that a point is not counted more than once.
The number of points of intersection of 37 straight lines is 37C2. But 13 straight lines out of the given 37 straight
lines pass through the same point A. Therefore, instead of getting 13C2 points, we get merely one point A. Similarly,
11 straight lines out of the given 37 straight lines intersect at point B. Therefore, instead of getting 11C2 points, we
get only point B. Hence, the number of intersection points of the lines in 37C2 – 13C2 – 11C2 + 2 = 535.
Illustration 23: How many five-digit numbers can be made having exactly two identical digits? (JEE MAIN)
Sol: Note that zero cannot occupy the first place. So we divide the solution into two cases when the common digit
is 0 and otherwise. Calculate the number of possibilities in these two cases and their sum gives us the desired result.
Case I: Two identical digits are 0, 0.
The number of ways to select three more digits is 9C3. The number of arrangements of these five digits is
(5! 2!) − 4! = 60 − 24 = 36 .
Hence, the number of such numbers is 9C3 × 36 = 3024 … (i)
Case II: Two identical digits are (1, 1) or (2, 2) or.... or (9, 9).
If 0 is included, then number of ways of selection of two more digits is 8C2.
The number of ways of arrangements of these five digits is 5! / 2! -4! / 2! =48.
Therefore, the number of such numbers is 8C2 × 48.
If 0 is not included, then selection of three more digits is 8C3.
Therefore, the number of such numbers is 8C3 × 5! / 2! = 8C3 × 60.
Hence, the total number of five-digit numbers with identical digits (1.1)......(9.9) is
9 × (8C2 × 48 + 8C3 × 60) = 42336 ... (ii)
From Eqs. (i) and (ii), the required number of numbers is 3024 + 42336 = 45360.
Illustration 24: How many words can be made with letters of the word “INTERMEDIATE” if
(i) The words neither begin with I nor end with E,
(ii) The vowels and consonants alternate in the words,
(iii) The vowels are always consecutive,
(iv) The relative order of vowels and consonants does not change,
(v) No vowel is in between two consonants,
(vi) The order of vowels does not change? (JEE MAIN)
M a them a ti cs | 5.11
Sol: This is an application of Permutation under restriction and Permutation of Alike objects. Proceed according to
the given conditions.
(i) The required number of words = (the number of words without restriction) – (the number of words beginning
with I) – (the number of word ending with E) + (the number of words beginning with I and ending with E)
(because words beginning with I as well as words ending with E contain some words beginning with I and
ending with E).
12!
The number of words without restriction =
2! 3! 2!
( There are 12 letters in which there are two I’s, three E’s and two T’s).
11!
The number of words beginning with I =
2!3!
( With E in the extreme left place we are left to arrange 11 letters INTERMEDIATE in which there are two T’s
and three E’s).
11!
The number of words ending with E =
2!2!2!
( With E in the extreme right place we are left to arrange 11 letters INTERMEDIATE in which there are two
I’s. two E’s and two T’s.)
10!
The number of words beginning with I and ending with E =
2!2!
( With I in the extreme left and E in the extreme right places we are left to arrange 10 letters INTERMEDIATE
in the which there are two T’s and Two E’s).
12! 11! 11! 10! 10!
∴ the required number or words = – – + = (12×11 – 11×2 – 11×3 +6) = 83 × 10!
2!3!2! 2!3! 2!2!2! 2!2! 2!3!2! 24
(ii) There are 6 vowels and 6 consonants. So, the number of words in which vowels and consonants alternate =
(the number of words in which vowels occupy odd places and consonants occupy even places) + (the number
of words in which consonants occupy odd places and vowels occupy even places)
6! 6! 6! 6! 6! 6!
= × + × = 2. . = 43200
2!3! 2! 2! 2!3! 2!3! 2!
7!
(iii) Considering the 6 vowels IEEIAE as one object, the number of arrangements of this with 6 consonants =
( there are two T’s in the consonants). 2!
For each of these arrangements, the 6 consecutive vowels can be arranged among themselves in 6! .
7! 6! 2!3!
∴ The required number of words = × (as above) = 151200
2! 2!3!
(iv) The relative order of vowels and consonants will not change if in the arrangements of letters, the vowels
occupy places of vowels, i.e.. 1st, 4th, 7th, 9th, 10th, 12th places and consonants occupy their places, i.e., 2nd. 3rd.
5th. 6th. 8th. 11th places, the required number of words
6! 6!
× = 21600
2!3! 2!
(v) No vowel will be between two consonants if all the consonants become consecutive
∴ the required number of words = the number of arrangements when all the consonants are consecutive
7! 6!
= × (as above) = 151200
2!3! 2!
(vi) The order of vowels will not change if no two vowels interchange places, i.e., in the arrangement all the vowels
are treated as identical.
(For example LATE. ATLE. TLAE. etc.. have the same order of vowels A. E. But LETA, ETLA.
TLEA. etc.. have changed order of vowels A. E. So. LATE is counted but LETA is not.
If A, E, are taken as identical say V then LVTV does not give a new arrangement by interchanging V, V.
5 . 1 2 | Permutations and Combinations
Illustration 25: India and South Africa play a one day international series until one team wins 4 matches. No match
ends in a draw. Find, in how many ways can the series can be won. (JEE ADVANCED)
Sol: The team who wins the series is the team with more number of wins. The losing team wins either 0 or 1 or 2
or 3 matches. Using this we find the number of ways in which a team can win.
Let I for India and S for South Africa. We can arrange I and S to show the wins for India and South Africa respectively
Suppose India wins the series, then the last match is always won by India.
Wins of S Wins of I No. of ways
(i) 0 4 1
(ii) 1 4 4! / 3! = 4
5!
(iii) 2 4 =10
2!3!
6!
(iv) 3 4 = 20
3!3!
∴ Total no. of ways = 35
In the same number of ways South Africa can win the series
∴ Total no. of ways in which the series can be won = 35 × 2 = 70
Illustration 26: There are p intermediate railway stations on a railway line from one terminal to another. In how
many ways can a train stop at three of these intermediate stations, if no two of these stations (where it stops) are
to be consecutive? (JEE ADVANCED)
Sol: The train stops only at three intermediate stations implies that the train does not stop at (p – 3) stations. Using
this idea we proceed further to get the answer.
The problem then reduces to the following:
In how many ways can three objects be placed among (p – 3) objects in a row such that no two of them are next
to each other (at most 1 object is to be placed between any two of these (p – 3) objects). Since there are (p – 2)
positions to place the three objects, the required number of ways = p–2C3.
Illustration 27: Five balls are to be placed in three boxes. Each can hold all the five balls. In how many different
ways can we place the balls so that no box remains empty if
(i) balls and boxes are all different
(ii) balls are identical but boxes are different
(iii) balls are different but boxes are identical
(iv) balls as well as boxes are identical
(v) balls as well as boxes are identical but boxes are kept in a row? (JEE MAIN)
Sol: Use the different cases of combination to solve the question according to the given conditions
As no box is to remain empty, boxes can have balls in the following numbers:
Possibilities 1, 1, 3 or 1, 2, 2
M a them a ti cs | 5.13
(i) The number of ways to distribute the balls in groups of 1, 1, 3 = 5C1 × 4C1 × 3C3.
But the boxes can interchange their content, no exchange gives a new way when boxes containing balls in equal
number interchange.
3!
∴ the total number of ways to distribute 1, 1, 3 balls to the boxes = 5C1 × 4C1 × 3C3 ×
2! 3!
Similarly, the total number of ways to distribute 1, 2, 2 balls to the boxes = 5C1 × 4C2 × 2C2 ×
3! 5 3! 2!
∴ the required number of ways = 5C1 × 4C1 × 3C3 × + C 1 × 4C 1 × 2C 2 × = 5 × 4 × 3 + 5 × 6 × 3 = 60 + 90 = 150
2! 2!
Note: Writing the whole answer in tabular form.
Possibilities Combinations Permutations
3!
1, 1, 3 5
C1 × 4C1 × 3C3 5
C 1 × 4C 1 × 3C 3 × = 5 × 4 × 3 = 60
2!
3!
1, 2, 2 5
C1 × 4C2 × 2C2 5
C 1 × 4C 2 × 2C 2 × = 5 × 6 × 3 = 90
2!
∴ the required number of ways = 60 + 90 = 150.
(ii) When balls are identical but boxes are different the number of combinations will be 1 in each case.
3! 3!
∴ the required number of ways = 1 × +1 × =3+3=6
2! 2!
(iii) When the balls are different and boxes are identical, the number of arrangements will be 1 in each case.
∴ the required number of ways = 5C1 × 4C1 × 3C3 + 5C1 × 4C2 × 2C2 = 5 × 4 + 5 × 6 = 20 + 30 = 50
(iv) When balls as well as boxes are identical, the number of combinations and arrangements will be 1 each in
both cases.
∴ the required number of ways = 1 × 1 + 1 × 1 = 2
(v) When boxes are kept in a row, they will be treated as different. So, in this case the number of ways will be the
same as in (ii).
Illustration 28: There are m points on one straight line AB and n points on another straight line AC, none of them
being A. How many triangles can be formed with these points as vertices? How many can be formed if point A is
also included? (JEE MAIN)
Sol: A triangle has three vertices, so we select two points on one line and one on the other and vice versa. Also,
consider the case when one point of the triangle is the intersection of the two lines.
To get a triangle, we either take two points on AB and one point on AC. or one point on AB and two points on AC.
Therefore, the number of triangles, we obtain
m(m – 1) n(n – 1) 1 1
= (mC2)(nC1) + (mC1)(nC2) = n+m = mn(m – 1 + n – 1) = mn(m + n – 2)
2 2 2 2
If the point A is included, we get m n additional triangles. Thus, in this case we get
mn mn(m + n)
= (m + n – 2) + mn = triangles.
2 2
(a) The number of ways in which (m + n) different objects can be divided into two unequal groups containing m
(m + n)!
and n objects respectively is .
m!n!
(2n)!
If m = n, the groups are equal and in this case the number of division is ; as it is possible to interchange the
n!n!2!
two groups without obtaining a new distribution.
5 . 1 4 | Permutations and Combinations
(b) However, if 2n objects are to be divided equally between two persons then the number of ways
(2n)! (2n)!
= 2! =
n!n!2! n!n!
(c) The number of ways in which (m + n + p) different objects can be divided into three unequal groups containing
(m + n + p)
m , n and p objects respectively is = ,m≠n≠p
m!n!p!
(3n)!
If m = n = P then the number of groups = . However, if 3n objects are to be divided equally among three
n!n!n!3!
(3n)! (3n)!
persons then the number of ways = 3! =
n!n!n!3! (n!)3
15!
For example, the number of ways in which 15 recruits can be divided into three equal groups is and the
5!5!5!3!
15!
number of ways in which they can be drafted into three different regiments, five in each, is
5!5!5!
(d) The number of ways in which mn different objects can be divided equally into m groups if order of groups is
mn!
not important is
(n!)m m!
(e) The number of ways in which mn different objects can be divided equally into m groups if the order of groups
mn! (mn)!
is important is × m! =
m
(n!) m! (n!)m
Illustration 29: In how many ways can 12 balls be divided between 2 boys, one receiving 5 and the other 7 balls?
(JEE MAIN)
Sol: Simple application of division of objects into groups. Since the order is important, the number of ways in which
12 different balls can be divided between two boys who each get 5 and 7 balls respectively, is
12! 12.11.10.9.8.7!
= × 2! = × 2 1584
5!7! (5.4.3.2.1)7!
Alternative:
The first boy can be given 5 balls out of 12 balls in 12C5 ways. The second boy can be given the remaining 7 balls in
one way. But the order is important (the boys can interchange 2 ways).
12! 12.11.10.9.8.7!.2
Thus, the required number of ways = 12C5 × 1 × 2! = ×2= = 1584
5!7! 5.4.3.2.1.7!
Illustration 30: Find the number of ways in which 9 different toys can be distributed among 4 children belonging
to different age groups in such a way that the distribution among the 3 elder children is even and the youngest
one is to receive one toy more. (JEE ADVANCED)
Sol: Using the concept of division of objects into groups we can solve this problem very easily.
The distribution should be 2, 2, 2 and 3 to the youngest. Now, 3 toys for the youngest can be selected in 9C3 ways,
the remaining 6 toys can be divided into three equal groups in
6!
ways and can be distributed in 3 ! ways.
(2!)3 .3!
6! 9!
Thus, the required number of ways = 9C3. 3! =
3
(2!) 3! 3!(2!)3
M a them a ti cs | 5.15
Illustration 31: Divide 50 objects in 5 groups of size 10, 10, 10, 15 and 5 objects. Also find the number of
distributions? (JEE MAIN)
50!
Sol: Same as the above question. Number of ways of dividing 50 objects into 5 groups as given = 3
(10!) (15)!(5)!(3)!
50!
Number of ways of distributing 50 objects into above formed groups = × 5!
(10)!3 .(15)!(5)!3!
PLANCESS CONCEPTS
6. CIRCULAR PERMUTATION
Let us consider that persons A, B, C and D are sitting around a round table. If all of them (A, B, C, D) are shifted one
place in an anticlockwise order, then we will get fig.(b) from fig.(a). Now, if we shift A, B, C, D in anticlockwise order
again, we will get fig. (c). We shift them once more and we will get fig.(d); and in the next time fig(a).
C B A D
D B C A B D A C
A D C B
A B C D D A B C C D A B B C D A
a b c d
Figure 5.2
Thus, we see that if 4 persons are sitting at a round table, they can be shifted four times and the four different
arrangements thus obtained will be same, because the anticlockwise order of A, B, C, D does not change. But if A,
B, C and D are sitting in a row and they are shifted in such an order that the last occupies the place of first, then the
four arrangements will be different.
Thus, if there are 4 objects, then for each circular arrangement number of linear arrangements is 4.
Similarly, if n different objects are arranged along a circle, for each circular arrangement the number of linear
arrangements is n.
Therefore, the number of circular arrangements of n different objects = the number of linear arrangements of n
different objects / n = n!/(n) = (n – 1)!
5 . 1 6 | Permutations and Combinations
D B B D B G G B
A A R R
Figure 5.3
Therefore, when clockwise and anticlockwise arrangements are not different, i.e. when observations can be made
from both sides, the number of circular arrangements of n different objects is (n – 1)!/2
Consider five persons A, B, C, D, E on the circumference of a circular table in an order which has no head. Now,
shifting A, B, C, D, E one position in anticlockwise direction we will get arrangements as follows:
We see, that arrangements in all figures are different.
∴ The number of circular permutation of n different objects taken all at a time is (n – 1)!, if clockwise and anticlockwise
orders are taken as different.
Note:
(a) The number of circular permutations of n different objects taken r at a time
n
Pr/r, when clockwise and anticlockwise orders are treated as different.
n
Pr/2r, when clockwise and anticlockwise orders are treated as same.
(b) The number of circular permutations of n different objects altogether
n
Pn/n = (n – 1)!, when clockwise and anticlockwise order are treated as different,
n
Pn/2n = 1/2(n – 1)!, when the above two orders are treated as same.
Illustration 32: In how many ways can 5 Indians and 4 Englishmen be seated at a round table if
(a) There is no restriction, (b) All the four Englishmen sit together,
(c) All four Englishmen don’t sit together, (d) No two Englishmen sit together. (JEE MAIN)
Sol: Clearly, this is a case of Circular Permutation. Using the formula (n – 1)!, we can find the answer according to
the given cases.
(a) Total number of persons = 5 + 4 = 9. These 9 persons can be seated at the round table in 8! Ways.
∴ Required number of ways = 8!
(b) Regarding 4 Englishmen as one person, we have only 5 + 1 i.e. 6 persons.
These 6 persons can be seated at the round table in 5! ways. Also, the 4 Englishmen can be arranged among
themselves in 4! ways.
∴ the required number of ways = 5! 4!
(c) The total number of arrangements when there is no restriction = 8!; the number of arrangements when all the
four English men sit together = 5! 4!
∴ The number of arrangements when all the four Englishmen don’t sit together = 8! – 5! 4!
M a them a ti cs | 5.17
Illustration 33: Consider 21 different pearls on a necklace. How many ways can the pearls be placed in this
necklace such that 3 specific pearls always remains together? (JEE MAIN)
Sol: This is the case of circular permutation when there is no distinction between clockwise and anticlockwise
arrangements.
After fixing the places of three pearls. Treating 3 specific pearls = 1 unit. So we have now 18 pearls + 1 unit = 19
and the number of arrangements will be (19 – 1)! = 18!. Also the number of ways 3 pearls can be arranged between
themselves is 3! = 6. As there is no distinction between the clockwise and anticlockwise arrangements, the required
1
number of arrangements = 18!. 6 = 3 (18!).
2
Illustration 34: Six persons A, B, C, D, E and F are to be seated at a circular table. Find the number of ways this can
be done if A must have either B or C on his right and B must have either C or D on his right. (JEE MAIN)
Sol: Fix the position of some of the persons relative to each other as per the question and arrange the remaining
in the seats available.
When A has B or C to his right we have either AB or AC
When B has C or D to his right we have BC or BD.
Thus, we must have ABC or ABD or AC and BD.
For ABC, D, E, F in a circular number of ways = 3! = 6
For ABD, C, E, F in a circular number of ways = 3! = 6
For AC, BD E, F the number of ways = 3! = 6
Hence, the required number of ways = 18
Coefficient Method
(a) The number of non-negative integral solutions of equation x1 + x2 + … + xr = n
= The number of ways of distributing n identical objects among r persons when each person can get zero, one or
more objects = coeff. of xn in [(1 + x + x2 + … + xn)(1 + x + x2 + … + xn) (1 + x + x2 + … + xn)…upto r factors]
= coeff. of xn in (1 + x + x2 + … + xn)r
r
1 – xn+1
= coeff. of x in
n = coeff. of xn in (1 – xn+1)r (1 – x)–r = coeff. of xn in (1 – x)–r
1–x
[leaving terms containing powers of x greater than n] = n+r–1Cr–1
5 . 1 8 | Permutations and Combinations
Illustration 35: How many integral solutions are there to x + y + z + w = 29, when x ≥ 1, y ≥ 2, z ≥ 3 and w ≥ 0?
(JEE ADVANCED)
Sol: Application of multinomial theorem.
x + y + z + w = 29 .…(i)
x ≥ 1, y ≥ 2, z ≥ 3, w ≥ 0 ⇒ x – 1 ≥ 0, y – 2 ≥ 0, z – 3 ≥ 0, w ≥ 0
Let x1 = x – 1, x2 = y – 2, x3 = z – 3
⇒ x = x1 + 1, y = x2 + 2, z = x3 + 3 and then x1 ≥ 0, x2 ≥ 0, x3 ≥ 0, w ≥ 0
From (i), x1 + 1 + x2 + 2 + x3 + 3 + w = 29
⇒ x1 + x2 + x3 + w = 23
Illustration 36: Find the number of non-negative integral solution 3x + y + z = 24. (JEE MAIN)
Illustration 37: Find the number of solutions of the equation x + y + z = 6, where x, y, z ∈ W. (JEE MAIN)
Illustration 38: How many integers are there between 1 and 1000000 having the sum of the digits as 18?
(JEE ADVANCED)
Sol: Let the digits be a1, …, a6 and use multinomial theorem we get the answer.
Any number between 1 and 1000000 must be of less than seven digits. Therefore, it must be of the form a1 a2 a3
a4 a5 a6
Where a1. a2.a3.a4.a5.a6 ∈ {0, 1, 2, … 9}
23.22.21.20.19 13.12.11.10.9
= – 6. = 33649 – 7722 = 25927
120 120
PLANCESS CONCEPTS
•• m different white balls and n different red balls are to be arranged in a line such that the balls of the
same colour are always together = (m! n! 2!)
•• m different white balls and n different red balls are to be arranged in a line such that all the red balls
are together = ((m + 1)! n!)
•• m different white balls and n different red balls are to be arranged in a line such that no two red balls
are together (m ≥ n – 1) = (m+1Cn m! n!)
•• m different white balls and m different red balls are to be arranged in a line such that colour of the
balls is alternating = (2 × (m!)2)
•• m identical white balls and n different red balls are to be arranged in a line such that no two red balls
are together (m ≥ n – 1) = (m+1Cn)
•• m identical white balls and n different red balls are to be arranged in a line such that no two red balls
are together (m ≥ n – 1) = (m+1Cn n!)
•• If n objects are arranged in a line the number of selections of r objects (n ≥ 2r – 1) such that no two
objects are adjacent is same number of ways of arranging n – r identical white balls and r identical red
ball in a line such that no two balls are together = (n–r+1Cr). e.g. suppose there are n stations on trains’s
route and a train has to stop at r stations such that no two stations are adjacent. The number of ways
must be n–r+1Cr.
•• suppose there are N seats in a particular row of a theatre. The number of ways of making n people
sit (N ≥ 2n – 1) such that no two people sit side by side is same as number of ways of arranging N – n
identical white balls (empty seats) and n different red balls (n people) such that no two red balls are
together. The required number of ways are N–n+1Cn × n!.
Nitish Jhawar (JEE 2009, AIR 7)
5 . 2 0 | Permutations and Combinations
8. DIVISIBILITY OF NUMBERS
The following table shows the conditions of divisibility of some numbers
Divisible by Condition
2 Whose last digit is even
3 sum of whose digits is divisible by 3
4 whose last two digits number is divisible by 4
5 whose last digit is either 0 or 5
6 which is divisible by both 2 and 3
7 If you double the last digit and subtract it from the rest of the number, answer is a multiple of 7
8 whose last three digits number is divisible by 8
9 sum of whose digits is divisible by 9
10 Whose last digit is 0
11 If you sum every second digit and then subtract sum of all other digits, answer is a multiple of 11
25 whose last two digits are divisible by 25
Illustration 39: How many four digit numbers can be made with the digits 0, 1, 2, 3, 4, 5 which are divisible by 3
(digits being unrepeated in the same number)? How many of these will be divisible by 6? (JEE ADVANCED)
Sol: A number is divisible by 3 if the sum of the digits is divisible by 3. This reduces the problem to the number of
non-negative integral solutions of equation x1 + x2 + … + xr = n.
Here, 0 + 1 + 2 + 3 + 4 + 5 = 15; so two digits are to be omitted whose sum is 3 or 6 or 9.
Hence, the number of four digits can be made by either
1, 2, 4, 5 or 0, 3, 4, 5 (omitting two digits whose sum is 3)
0, 1, 3, 5 or 0, 2, 3, 4 (omitting two digits whose sum 6)
0, 1, 2, 3 (omitting two digits whose sum is 9)
The number of 4-digit numbers that can be made with 1, 2, 4, 5 = 4P4 = 4!
The number of 4-digit numbers that can be made by the digits in any one of remaining four groups (each containing
0) = 4! – 3!
∴ The required number of 4-digit numbers divisible by 3 = 4! + 4(4! – 3!) = 24 + 4(24 – 6) = 96
Now, a number is divisible by 6 if it is even as well as divisible by 3.
So, the number of 4-digit numbers divisible by 6 that can be made with 1, 2, 4, 5 = 2 × 3! (∵ the number should
have an even digit in the units places).
The number of numbers of 4 digits, divisible by 6, that can be made with 0, 3, 4, 5 = (3! – 2!) + 3!
( The number should have 4 or 0 in units place and 0 should not come in thousands place).
Similarly, the number of numbers of 4 digits, divisible by 6, that can be made with 0, 1, 2, 3 =
(3! – 2!) + 3!
The number of 4-digit numbers divisible by 6 that can be made with the digits 0, 1, 3 = 3!
The number of numbers of 4 digits, divisible by 6, that can be made with 0, 2, 3, 4 = (3! – 2!) +
(3! – 2!) + 3!
( The number should have 4 or 2 or 0 in units place and 0 should not come in thousands place)
M a them a ti cs | 5.21
9. SUM OF NUMBERS
(a) For given n different digits a1, a2, a3 … an the sum of the digits in the units place of all numbers formed (if
numbers are not repeated) is
(a1 + a2 + a3 + … + an) (n – 1)! i.e. (sum of the digits) (n – 1)!
(b) Sum of the total numbers which can be formed with given different digits a1, a2, a3…… an is
(a + a2 + a3 + … + an) (n –1)! (111. ……….. n times)
Illustration 40: Find the sum of all 4 digit numbers formed using the digits 1, 2, 4 and 6. (JEE MAIN)
Sol: Use formula, Sum = (a1 + a2 + a3 + … + an) (n− 1)! (111 …. N times)
Using formula, Sum = (1 + 2 + 4 + 6) 3! (1111) = 13 × 6 × 1111 = 86658
Alternate:
Here, the total 4-digit numbers will be 4! = 24. So, every digit will occur 6 times at every one of the four places.
Since the sum of the given digits = 1 + 2 + 4 + 6 = 13. So, the sum of all the digits at every place of all the 24
numbers = 13 × 6 = 78.
The sum of the values of all the digits
At first place = 78
At the tens place = 780
At the hundreds place = 7800
At the thousands place = 78000
∴ The required sum 78 + 780 + 7800 + 78000 = 86658
(d) The number of ways in which a composite number N can be resolved into two factors which are relatively
prime (or coprime) to each other is equal to 2n–1 where n is the number of different prime factors in N
Illustration 41: Find the number of factors of the number 38808 (excluding 1 and the number itself). Find also the
sum of these divisors. (JEE MAIN)
Sol: Factorise 38808 into its product of primes and then use the concept of combination to find the answer.
38808 = 23 . 32 . 72 . 11
5 . 2 2 | Permutations and Combinations
Illustration 42: In how many ways can the number 10800 be resolved as a product of two factors? (JEE MAIN)
Sol: Check whether the number is a perfect square or not and accordingly use the formula to find the desired result.
10800 = 24 . 33 . 52
Here 10800 is not a perfect square ( power of 3 is odd).
1
Hence, the number of ways = (4 + 1) (3 + 1) (2 + 1) = 30.
2
Illustration 43: Find the number of positive integral solutions of x1.x2.x3 = 30. (JEE ADVANCED)
Sol: Factorise 30 into primes and then use combination to get the desired result.
x1x2x3 = 2 × 3 × 5. If we treat 2, 3, 5 as objects and x1, x2, x3 as distinct boxes then finding the number of positive
integer solution is the same as finding the number of ways of distributing 3 distinct objects in 3 distinct boxes.
Thus, the required number of solutions is 35 = 27
(For example, if all the objects are held by x1 the corresponding solution is x1 = 30. x2 = 1, x3 = 1, if 2 and 3 are held
by x1 and 5 by x3 then x1 = 6, x2 = 1, x3 = 5 etc)
n
The number of multiples of p in n! is given by . Even if a number k between 1 and n has two factors of p, this
p
n
formula counts it as only one. Hence we need to evaluate also. Similarly, for three and four to infinity. Hence,
2
p
∞
n
the exponent of prime p in n! is given by ep(n) = ∑ ep is called Legendre’s function.
i
p
i=1
Even though this is an infinite sum result, it is finite since for all pi greater than n, step function becomes zero.
Let’s understand this method by an example of finding exponent of 2 in 100!
PLANCESS CONCEPTS
Following method involves part of an advanced topic in mathematics called Modular Arithmetic.
Legendre’s function also has another result, which is
∞ n n – Sp (n)
ep(n) = ∑ pi = p –1
i=1
Illustration 45: Find all positive integers of n such that n! ends in exactly 1000 zeros. (JEE ADVANCED)
Sol: 10 is a multiple of 2 and 5. In order to get 1000 zeroes we must have 1000 as the exponent of 5 in n!. Now use
the definition of the GIF to find the range of numbers satisfying the given condition.
n n
There are clearly more 2’s than 5’s in the prime factorization of n!, hence it suffices to solve the equation +
+ … = 1000. 5 52
n n n n n 1 n 1 n
But + + … < + + … = 1 + + ... (as [x] < x) = . = .
5 2
5 5 52 5 5 5 1 4
1–
5
Hence, n > 4000.
5
On the other hand, using the inequality [x] > x – 1, we have 1
1–
n n n n
1000 > – 1 + – 1 + – 1 +
n n 1 1
– 1 + – 1 = 1 + +
1 1 n 5
+ + –5= . – 5.
5 5 2
53
5 4
5 5
5 5 52
53 4
5 5 1
1–
5
1005 ⋅ 4 ⋅ 3125
So, N < < 4022.
3124
We narrowed n down to {4001, 4002, … , 4021}. Using Legendre’s formula we find that 4005 is the first positive
integer with the desired property and that 4009 is the last. Hence, n = 4005, 4006, 4007, 4008, 4009.
Second solution: It suffices to solve the equation e5(n) = 1000. Using the second form of Legendre’s formula, this
becomes n – s5(n) = 4000. Hence n > 4000. We work our way upward from 4000 looking for a solution. Since e5(n)
can change only at multiples of 5 (why?), we step up 5 each time:
4000 – 4
e5(4000) = = 999.
5–1
4005 – 5
e5(4005) = = 1000.
5–1
4010 – 6
e5(4010) = = 1001.
5–1
Any n > 4010 will clearly have e5(n) ≥ e5(4010) = 1001. Hence the only solutions are n = 4005, 4006, 4007, 4008, 4009.
n n
=
This can be compactly written as Ai ∑ (–1)k +1 ∑ | Ai ∩ ... ∩ Ai |
1 k
i==
1 k 1 1≤i1 <........<ik ≤n
In words, to count the number of elements in a finite union of finite sets, first sum the cardinalities of the individual
sets, then subtract the number of elements which appear in more than one set, then add back the number of
elements which appear in more than two sets, then subtract the number of elements which appear in more than
three sets, and so on. This process naturally ends since there can be no elements which appear in more than the
number of sets in the union.
M a them a ti cs | 5.25
In applications it is common to see the principle expressed in its complementary form. That is, taking S to be a
finite universal set containing all of the Ai and letting Ai denote the complement of Ai in S. By De Morgan’s laws.
n n n
We have, Ai = S – Ai = | S |= ∑ | Ai | + ∑ | Ai ∩ A j | –... + (–1)n | A1 ∩ ... ∩ An | .
=i 1 =i 1 =i 1 1≤i< j≤n
Illustration 46: 105 students take an examination of whom 80 students pass in English. 75 students pass in
Mathematics and 60 students pass in both subjects. How many students fail in both subjects? (JEE MAIN)
Illustration 47: Find the number of permutations of the 8 letters AABBCCDD, taken all at a time, such that no two
adjacent letters are alike. (JEE ADVANCED)
Sol: Divide the question into cases when A’s are adjacent, B’s are adjacent and so on. Similarly proceed to find the
number of ways in which two alike objects are adjacent and so on. Then use Inclusion-Exclusion Principle to find
the result.
First disregard the restriction that no two adjacent letters be alike.
8!
The total number of permutation is then N = = 2250
2!2!2!2!
Now, apply the inclusion exclusion principle. Where a permutation has property α in case the A’s are adjacent,
property β in case the B’s are adjacent, etc. It can be calculated that
7! 6!
N(α) = = 630. N(α, β) = = 180
2!2!2! 2!2!
N(α, β, γ) = 60. N(α, β, γ, δ) = 24.
Hence, the answer is N – 4N (α) + 6N(α, β) – 4N(α, β, γ) + N(α, β, γ, δ) = 864.
n! n! 1 1 (–1)n
= n! – (n – 1)! (n – 2)!+ ... + (–1)n–1 = n! 1 – + + ... +
1!(n – 1) 2!(n – 2)! 1! 2! n!
Remark: If r objects go to wrong place out of n object then (n – r) objects goes to original place.
1 1 1 1
A’r = r!(1 – + – + … + (–1)r )
1! 2! 3! r!
PLANCESS CONCEPTS
n
(–1)i 1
Number of derangements Dn = n! ∑ ; Interestingly, as n → ∞, Dn =
i=0 i! e
n!
This results in an interesting relating Dn = .Where [x] is the nearest integer function.
e
Use this formula only if the given options are wide apart from one another.
ExampleYou have 6 ball in 6 different colors, and for every ball you have a box of the same color. How
many derangements do you have, if no ball is in a box of the same color?
Sol: We know that e = 2.71828. To make division simple let’s round it to 2.7. You have to keep in mind that
we have reduced the value of e, so the result which we get is greater than the actual result.
6! 720 800
∴ Dn = = = = [266.66] = 267
e
2.7 3
Hence, the result will be close to 266.
This is a pretty good approximation as the actual answer is 265.
But, if the given options are all close to 266, then it is advised to calculate using the original formula or
by rounding the value of e to the number of significant digits equal to that of n! (numerator).
So if we use e = 2.72 we get 34
6! 720 4500
Dn ==
= = [264.70] = 265
e 2.72 17
Vaibhav Krishnan (JEE 2009, AIR 22)
Illustration 48: A person writes letters to six friends and addresses the corresponding envelopes. In how many
ways can the letters be placed in the envelopes so that (i) atleast two of them are in the wrong envelopes. (ii) All
the letters are in the wrong envelopes. (JEE MAIN)
1 1 1 1 1 1 1 1 1
= 6C4.2! 1 – + + 6 C 3 .3! 1 – + + + 6 C2 .4! 1 – + – +
1! 2! 1! 2! 3! 1! 2! 3! 4!
M a them a ti cs | 5.27
1 1 1 1 1 1 1 1 1 1 1
+6 C1.5! 1– + – + + + 6 C 0 .6! 1– + – + – +
1! 2! 3! 4! 5! 1! 2! 3! 4! 5! 6!
(ii) The number of ways in which all letters be placed in wrong envelopes
1 1 1 1 1 1 1 1 1 1 1
= 6! 1 – + – + – + ; = 720 – + – + ; = 360 – 120 + 30 – 6 + 1 = 265
1! 2! 3! 4! 5! 6! 2 6 24 120 720
x x2 x f x x2 xm x x2 xn
r! 1 + + + ..... 1 + + + ... 1 + + + ...
1! 2!
l ! 1! 2!
m! 1! 2! n!
Illustration 49: In an examination, the maximum marks for each of three papers is n and that for the fourth paper
is 2n. Prove that the number of ways in which candidate can get 3n marks is
1
(n + 1) (5n2 + 10n + 6). (JEE ADVANCED)
6
Sol: The maximum marks in the four papers are n, n, n and 2n. Consider a polynomial (1 + x + x2 + … + xn)3 (1 +
x + … + x2n). The number of ways of securing a total of 3n is equal to the co-efficient of the term containing x3n.
The number of ways of getting 3n marks
= coefficient of x3n in (1 + x + x2 + … + xn)3 (1 + x + … + x2n)
= coefficient of x3n in (1 – xn+1)3 (1 –x2n+1) (1 – x)–4
= coefficient of x3n in (1 – 3xn+1 + 3x2n+2 – x3n+3) (1 – x2n+1) × (1 + 4C1 x + 5C2x2 + 6C3 x3 + ….)
= coefficient of x3n in (1 – 3xn+1 – x2n+1 + 3x2n+2) ( 1 + 4C1 x + 5C2 x2 …)
= 3n+3C3n – 3.2n+2C2n–1 + 3 . n+1Cn–2 – n+2Cn–1
= 1/6 (n + 1) (27n2 + 27n + 6 – 24n2 – 12n + 3n2 – 3n – n2 – 2n) = 1/6 (n + 1) (5n2 + 10n + 6)
PROBLEM-SOLVING TACTICS
In any given problem, first try to understand whether it is a problem of permutations or combinations. Now, think
if repetition is allowed and then try solving problem.
5 . 2 8 | Permutations and Combinations
A simple method to solve these problems where repetition is not allowed is as follows -
First draw series of dashes representing the number of places you want to fill or number of items you want to select.
Now start filling dashes by the number of objects available to choose from and multiply the numbers. This is the
final answer for a permutations problem.
If it is a combination problem then divide the answer with the factorial or number of items.
This calculation becomes complex if repetition is allowed.
FORMULAE SHEET
(a) Permutation (Arrangement of Objects): Each of the different arrangement, which can be made by taking
some or all of a number of objects is called permutation.
n!
(i) The number of permutations of n different objects taken r at a time is nPr = .
(n − r)!
(ii) The number of all permutations of n distinct objects taken all at a time is n!.
Permutation with Repetition: The number of permutations of n different objects taken r at a time when each
object may be repeated any number of times is nr.
Permutation of Alike Objects: The number of permutations of n objects taken all at a time in which, p are
alike objects of one kind, q are alike objects of second kind & r are alike objects of a third kind and the rest
n!
(n – (p + q + r)) are all different, is .
p!q!r!
Permutation under Restriction: The number of permutations of n different objects, taken all at a time, when m
specified objects always come together is m! × (n – m + 1)!.
(b) Combination (Selection of Objects): Each of the different groups or selection which can be made by some
or all of a number of given objects without reference to the order of the objects in each group is called a
combination.
n!
The number of all combinations of n objects, taken r at a time is generally denoted by C(n, r) or nCr =
n r!(n – r)!
Pr
(0 ≤ r ≤ n) =
r!
Note:
(a) The number of ways of selecting r objects out of n objects, is the same as the number of ways in which the
remaining (n - r) can be selected and rejected.
(b) The combination notation also represents the binomial coefficient. That is, the binomial coefficient nCr is the
combination of n elements chosen r at a time.
(c) (i) nCr = nCn–r
(ii) n Cr +n Cr −1 =
n+1
Cr
(iii) nCx = nCy ⇒ x = y or x + y = n
(iv) If n is even, then the greatest value of nCr is nCn/2
n
(v) If n is odd, then the greatest value of nCr is C(n+1)/2
(vi) C0 + Cr +................+ Cn = 2
n n n n
(b) However, if 2n objects are to be divided equally between two persons then the number of ways
(2n)! (2n)!
= 2! =
n!n!2! n!n!
(c) The number of ways in which (m + n + p) different objects can be divided into three unequal groups containing
(m + n + p)
m , n and p objects respectively is = ,m≠n≠p
m!n!p!
(3n)!
If m = n = P then the number of groups = . However, if 3n objects are to be divided equally among
n!n!n!3!
(3n)! (3n)!
three persons then the number of ways = 3! =
n!n!n!3! (n!)3
(d) The number of ways in which mn different objects can be divided equally into m groups if the order of groups
mn!
is not important is
(n!)m m!
(e) The number of ways in which mn different objects can be divided equally into m groups if the order of groups
mn! (mn)!
is important is × m! =
m
(n!) m! (n!)m
Circular Permutation
Pn/2n = 1/2(n – 1)!, when above two orders are treated as same
n
(d) The number of ways in which a composite number N can be resolved into two factors which are relatively
prime (or coprime) to each other is equal to 2n–1 where n is the number of different prime factors in N
∞ n
Exponent of a Prime P in N! = ∑ pi
i=1
Inclusion-Exclusion Principle: The principle of inclusion-exclusion states that for finite sets A1,…An. One has the
identity
n n
Ai ∑ Ai
= − ∑ | Ai ∩ Aj | + ∑ | Ai ∩ Aj ∩ Ak | – ... + (–1)n–1 | A1 ∩ ... ∩ An | .
i=1=i 1 1≤i< j≤n 1≤i< j<k ≤n
n n
=
This can be compactly written as Ai ∑ (–1)k +1 ∑ | Ai ∩ ... ∩ Ai |
1 k
i==
1 k 1 1≤i1 <........<ik ≤n
Derangements Theorem: The number of ways in which letters n can be placed in n envelopes (one letter in each
1 1 (–1)n
envelope) so that no letter is placed in the correct envelope is n! 1 – + + ... +
1! 2! n!
If n objects are arranged at n places then the number of ways to rearrange exactly r objects at right places is =
n! 1 1 1 1 n–r 1
1 – + – + – ... + (–1)
r 1! 2! 3! 4! (n – r)!
M a them a ti cs | 5.31
Solved Examples
JEE Main/Boards Each of the above pairs can be associated with a third
station in (n – 2) ways. Thus, choosing a pair of stations
Example 1: Find the number of ways in which 5 identical and any third station can be done in (n – 1) (n – 2) ways.
balls can be distributed among 10 different boxes, if The above count also includes the case of three
exactly one ball goes into a box. consecutive stations. However, we can see that each
such case has been counted twice. For example, the pair
Sol: It is same as selecting 5 boxes from 10 boxes and S4S5 combined with S6 and the pair S5S6 combined with
distributing the balls in those 5 boxes. S4 are identical.
Number of boxes = 10 and Number of balls = 5. Hence, subtracting the excess counting, the number of
∴ Possible number of ways = C5 10 ways in which three stations can be chosen so that at
least two of them are consecutive
Example 2: There are n intermediate stations on a = (n – 1) (n – 2) – (n – 2) = (n – 2)2
railway line from one terminal to another. In how many
ways can the train stop at 3 of these intermediate Example 3: How many ways are there to invite 1 of 3
stations if friends for dinner on 6 successive nights such that no
friend is invited more than 3 times?
(i) all the three stations are consecutive.
(ii) at least two of the stations are consecutive. Sol: Divide the solution in different possible cases. 6 can
be partitioned in the following ways
Sol: The first part is very trivial. For the second part
1+2+3
consider a pair of consecutive stations and then select
a station such that it is not consecutive. Check for 0+3+3
multiple counting. 2+2+2
Let the intermediate stations be S1, S2,……, Sn Using this we can form different possibilities and
(i) The number of triplets of consecutive stations, as calculate the number of ways the friends can be invited.
S1S2S3, S2S3S4, S3S4S5, …. Sn–2Sn–1Sn, is (n – 2). Let x, y, z be the friends and let (a, b, c) denote the case
(ii) The total number of consecutive pairs of stations, as where x is invited a times, y, b times and z, c times. For
S1S2, S2S3,…………, Sn–1Sn is (n – 1). example, one possible arrangement corresponding to
5 . 3 2 | Permutations and Combinations
the triplet (3, 2, 1) is x, x, y, x, y, z 2 alike of one kind and 3 alike of different kind and so
on. Count the number of words in these cases and their
Then we have the following possibilities:
sum gives us the answer.
(i) (a, b, c) = (1, 2, 3); (1, 3, 2); (2, 3, 1); (2, 1, 3):
Proposition contains 11 letters PP, R, OOO, S, II, T, N.
(ii) (a, b, c) = (3, 3,0); (3,0,3) ; (0, 3, 3).
Following table given the number of words.
(iii) (a, b, c) = (2, 2, 2). So the total number of ways is 6 ×
Repeated letters: O(2), P(2), I(2)
6!/1! 2! 3! + 3 × 6!/3!3! + 6!/2!2!2!
Different letters: R, S, T, N
Note: We can also solve this problem using linear
equations. Letters No. of Words Total
A 5 Distinct 7
C5.5! 2520
Example 4: There are 2n guests at a dinner party.
Supposing that the master and mistress of the B 3 Alike 1.2C1.(51/3!2!) 20
house have fixed seats opposite one another. And
2 Alike
that there are two specified guests who must not be
placed next to one another. Show that the number C 3 Alike 1.6C2.(5!/3!) 300
of ways in which the company can be placed is 2 Different
(2n – 2)! (4n2 – 6n + 4)
D 2 Alike 3
C2.5C1.(5!/2!2!) 450
Sol: This is an application of division into groups. Find 2Other Alike
the total number of ways of arrangement of the guests
and then subtract the number of ways in which the two 1 Different
mentioned guests are together. E 2 Alike 3
C1.6C3.(5!/2!) 3600
Excluding the two specified guests, 2n – 2 persons can be
3 Different
divided into two groups, one containing n and the other
(2n – 2)! Total no. of words = 6890
(n – 2) in and can sit on either side of mister
n!(n – 2)!
and mistress in 2! ways and can arrange themselves in Example 6: There are 10 points in a plane where no
n!(n – 2)! three points are collinear except for 4 points which are
M collinear. Find the number of triangles formed by the
n-2 points as vertices.
4
n G1 G2 Sol: A triangle is formed from three non-collinear
3 points. Select 3 points from 10 points in 10C3 ways and
m 2
subtract the cases when the points are collinear, as they
1 would not form a triangle.
Now, the two specified guests where (n – 2) guests are Let us suppose that the 10 points are such that no three
seated will have (n – 1) gaps and can arrange themselves of them are collinear. Now, a triangle will be formed by
in 2! Ways. Number of ways when G1 G2 will always be any three of these ten points. Thus forming a triangle
together amount to selecting any three of the 10 points.
(2n – 2)! Now 3 points can be selected out of 10 point in 10
C3
= 2! n! (n – 2) ! (n – 1) 2! = (2n – 2)! 4(n – 1)
n!(n – 2)! ways.
Hence, the number of ways when G1 G2 are never ∴ Number of triangles formed by 10 points when no
together three of them are collinear = 10C3.
2!
= 2! n! n! – 4(n – 1) (2n – 2)! Similarly, the number of triangles formed by 4 points
n! n! 2!
then no 3 of them are collinear = 4C3
= (2n – 2)! [2n(2n – 1) – 4 (n – 1)] = (2n – 2)! [4n2 – 6n + 4]
∴ Required number of triangle formed = 10
C 3 – 4C 3 =
120 – 4 = 116.
Example 5: Find the number of words of 5 letters that
can be formed with the letters of the word Proposition. Example 7: From 6 gentlemen and 4 ladies, a committee
of 5 is to be formed. In how many ways can this be done
Sol: Divide the cases into words having 5 distinct letters, if the committee is to include at least one lady?
M a them a ti cs | 5.33
No. of Ladies No. of Gentlemen No. of Committees (c) Each card can be given to any of the 3 servants.
1 4 4
C 1 6C 4 ∴ No. of ways = 3 × 3 × 3 × 3 × 3 × 3 = 36 = 729.
2 3 4
C 2 6C 3 (d) There are ten letters in the word BENEVOLENT of
which three are E and two are N, and the rest five are
3 2 4
C 3 6C 2 different.
4 1 4
C 4 6C 1 10!
∴ Total number of arrangements =
3! 2!
Total number of committees
Example 9: n1 and n2 are five-digit numbers. Find the
= 4C1 6C4 + 4C2 6C3 + 4C3 6C2 + 4C4 6C1 = 246 total number of ways of forming n1 and n2, so that n2 can
be subtracted from n1 without borrowing at any stage.
Example 8: (a) In how many ways can the following
diagram be coloured, subject to two conditions: Each Sol: Two numbers can be subtracted without borrowing
of the smaller triangle is to be painted with one of three if all the digits in n1 is greater than all the corresponding
colours: red, blue, green and no two adjacent regions digits in the number n2. Using this information, find the
should have the same color? number of ways for different possible cases and add
them up to get the answer.
(b)How many numbers of four digits can be formed
with the digits 1, 2, 3, 4 and 5? Let n1 = x1 x2 x3 x4 x5 and n2 = y1 y2 y3 y4 y5 be two numbers.
n1 and n2 can be subtracted without borrowing at any
stage if xi ≥ yi.
Here, xi and yi denotes the digits at various places in the
number n1 and n2 respectively.
Value of x5 Value of ys
9 0,1,2,…9
8 0,1,2,…8
7 0,1,2,…7
(c) A gentleman has 6 friends to invite. In how many
ways can he send invitation cards to them if he has 3 6 0,1,2,3,4,5,6
servants to carry the cards? 5 0,1,2,3,4,5
(d) Find the number of arrangements of the letters of 4 0,1,2,3,4
the word ‘BENEVOLENT’.
3 0,1,2,3
Sol: For each of the parts (a), (b), (c) and (d), identify 2 0,1
the number of ways a particular cell can be coloured or
filled in and then use permutation / combination to get 1 0
the result. 0
(a) These conditions are satisfied if we proceed as Thus, x5 and y5 can be selected collectively by 10 + 9 +
follows: Just color the central triangle by one color, 8 + … 1 = 55 ways. Similarly, each pair (x4, y4), (x3, y3), (x2,
this can be done in three ways. Next paint other three y2) can be selected in 55 ways. But, pair (x1, y1) can be
triangles with remaining 2 colors. By the fundamental selected in 1 + 2 + 3 + … + 9 = 45 ways as in this pair
principle of counting. This can be done in 3 × 2 × 2 × 2 we cannot have 0.
= 24 ways.
Therefore total number of ways = 45(55)4.
5 . 3 4 | Permutations and Combinations
Example 10: Prove that the product of r consecutive Sol: List down different ways in which we get the sum of
positive integers is divisible by r!. 5 and 4 and get the answer.
Event E (the first throw resulting in 5) can happen in one
Sol: Simple application of the definition of nPr.
of four ways as:
Let P be the product of r consecutive positive integers
3 + 2; 4 + 1; 2 + 3; 1 + 4.
ending with n; then
Event F (the second throw resulting in 4) can happen in
P = n(n – 1) … (n – r + 1)
one of three ways as:
P n(n – 1)...(n – r + 1)
= 2 + 2; 1 + 3; 3 + 1.
r! r!
The two events can together happen in 4 × 3 = 12 ways.
[n(n – 1)(n – 2)...(n – r + 1)][(n – r)...3.2.1]
r!(n – r)...3.2.1
n! Example 3: An eight-oared boat is to be manned by
= = nCr = an integer a crew chosen from 11 men of whom 3 can steer but
r! n – r!
cannot row and the rest cannot steer. In how many ways
∴ P is divisible by r!. can the crew be arranged if two of the men can only row
in bow side?
JEE Advanced/Boards
Bow side
= 9 × 8 × 8 × 8 × … to n factors = 9 × 8n–1.
Example 4: The members of a chess club took part in
a round robin competition in which each plays every
Example 2: A dice is a six-faced cube, with the faces one else once. All members scored the same number of
reading 1, 2, 3, 4, 5 and 6. When two dice are thrown, points, except four juniors whose total score were 17.5.
we add the digits they show on top and take that sum as How many members were there in the club? (Assume
the result of the throw. In how many different ways the
that for each win a player scores 1 point, for draw 1
first throw of the 2 dice shows a total of 5, and second 2
point and zero for losing.)
throw of the 2 dice shows a total of 4?
M a them a ti cs | 5.35
Sol: Form an equation of the total number of points As the man wants to travel by one of the many possible
scored using the given information. Solve the equation shortest paths, he will never turn to the right or turn
to find the answer. downward. So a travel by one of the shortest paths is to
take 4 horizontal pieces and 4 vertical pieces of roads.
Let the number of members be n.
∴ A shortest path is an arrangement of eight objects
Total number of point = nC2.
L1, L2, L3, L4, U1, U2, U3, U4 so that the order of L’s and U’s
∴ nC2 – 17.5 = (n – 4) x (where x is the number of point do not change.
scored by each player)
( Clearly L2 cannot be taken without taking L1, U2
n (n – 1) – 35 = 2 (n – 4)x cannot be taken without taking U1, etc.)
n(n – 1) – 35 Hence, the number of shortest paths
2x = (where x takes the values 0.5, 1, 1.5 etc.)
n– 4
= The number of arrangements of L1, L2, L3, L4, U1, U2,
n2 – n – 35 U3, U4 where the order of Ls as well as the order of Us
= (must be an integer)
n– 4 do not change
n(n – 4) + 3(n – 4) – 23 23 = The number of arrangement treating Ls as identical
= = (n + 3) –
n– 4 n– 4 and Us as identical
23
⇒ must be an integer 8! 8.7.6.5
n– 4 = = = 2.7.5 = 70.
4! 4! 24
⇒ n = 27 is the only possibility.
Example 7: A condolence meeting being held in a hall
Example 5: If p, q, r, s, t are prime numbers. Find the which has 7 doors, by which mourners enter the hall.
number of ways in which the product, pq2r3st can be One can use any of the 7 doors to enter and can come
expressed as product of two factors, excluding 1 as a at any time during the meeting. At each door, a register
factor. is kept in which mourner has to affix his signature while
entering the hall. If 200 people attend the meeting,
Sol: Use the standard result to find the answer. how many different sequences of 7 lists of signatures
Total factors = 2 × 3 × 4 × 2 × 2 = 96 can arise?
96
Hence, the total ways = = 48. but this includes 1
2 Sol: Clearly, the total number of people is 200, hence the
and the number itself also. Hence, the required number sum of the entries is 200. Apply Multinomial theorem
of ways = 48 – 1 = 47 to find the total number of ways list can be made and
hence the answer.
Example 6: In the given figure you have the road plan There are 7 lists, say 1, 2,…. 7. Suppose, that list i has xi
of a city. A man standing at X wants to reach the cinema names; then,
hall at Y by the shortest path. What is the number of
different paths that he can take? x1 + ….. + x7 = 200 where xi ≥ 0 is an integer.
JEE Main/Boards
Q.20Find the value of the expression Q.1 If the letters of the word “VARUN” are written in all
5 possible ways and then are arranged as in a dictionary,
47
C4 + ∑ 52– j C3 . then rank of the word VARUN is:
j=1
(A) 98 (B) 99 (C) 100 (D) 101
Q.21 Prove that the product of r consecutive integers
is divisible by r!. Q.2 Number of natural numbers between 100 and 1000
such that at least one of their digits is 7, is
Q.22 From a class of 25 students, 10 are to be chosen
(A) 225 (B) 243 (C) 252 (D) none
for a field trip. There are 3 students who decide that
either all of them will join or none of them will join. In
Q.3 The 120 permutations of MAHES are arranged in
how many ways can the field trip members be chosen?
dictionary order, as if each were an ordinary five-letter
word. The last letter of the 86th word in the list is
Q.23 There are ten points in a plane. Of these ten
points, four points are in a straight line and with the (A) A (B) H (C) S (D) E
exception of these four points, no three points are in
the same straight line. Find- Q.4 A new flag is to be designed with six vertical strips
using some or all of the colors yellow green, blue and
(i) The number of triangles formed.
red. Then the number of ways this can be done such
(ii) The number of straight lines formed that no two adjacent strips have the same color is
(iii) The number of quadrilaterals formed, by joining (A) 12 × 81 (B) 16 × 192 (C) 20 × 125 (D) 24 × 216
these ten points.
Q.5 The number of 10-digit numbers such that the
Q.24 In an examination a minimum of is to be secured product of any two consecutive digits in the number is
in each of 5 subjects for a pass. In how many ways can a prime number, is
a student fail?
(A) 1024 (B) 2048 (C) 512 (D) 64
Q.25 In how many ways 50 different objects can be
Q.6 Consider the five points comprising of the vertices
divided in 5 sets three of them having 12 objects each
of a square and the intersection point of its diagonals.
and two of them having 7 objects each.
How many triangles can be formed using these points?
Q.26 Six “X”s (crosses) have to be placed in the squares (A) 4 (B) 6 (C) 8 (D) 10
of the figure given below, such that each row contains
at least one X. In how many different ways can this be Q.7 How many of the 900 three digit numbers have at
done? least one even digit?
(A) 775 (B) 875 (C) 100 (D) 101
Q.10 Out of seven consonants and four vowels, the Q.20 A students have to answer 10 out of 13 questions
number of words of six letters, formed by taking four in an examination. The number of ways in which he
consonants and two vowels is (Assume that each can answer if he must answer at least 3 of the first five
ordered group of letter is a word): questions is
(A) 210 (B) 462 (C) 151200 (D) 332640 (A) 276 (B) 267 (C) 80 (D) 1200
Q.11 All possible three digits even numbers which can Q.21 The number of three digit numbers having only
be formed with the condition that if 5 is one of the two consecutive digits identical is:
digit, then 7 is the next digit is:
(A) 153 (B) 162 (C) 180 (D) 161
(A) 5 (B) 325 (C) 345 (D) 365
Q.22 The interior angles of a regular polygon measure
Q.12 Number of 5 digit numbers which are divisible by 150º each. The number of diagonals of the polygon is
5 and each number containing the digit 5, digits being
(A) 35 (B) 44 (C) 54 (D) 78
all different is equal to k(4!), the value of k is
(A) 84 (B) 168 (C) 188 (D) 208 Q.23 The number of n digit numbers which consists of
the digits 1 & 2 only if each digit is to be used at least
Q.13 The number of six digit numbers that can be once, is equal to 510 then n is equal to:
formed from the digits 1, 2, 3, 4, 5, 6, & 7 so that digits
(A) 7 (B) 8 (C) 9 (D) 10
do not repeat and the terminal digit are even is:
(A) 144 (B) 72 (C) 288 (D) 720 Q.24 Number of four digit numbers with all digits
different and containing the digit 7 is
Q.14 The number of natural numbers from 1000 to
(A) 2016 (B) 1828 (C) 1848 (D) 1884
9999 (both inclusive) that do not have all 4 different
digits is
Q.25 An English school and a Vernacular school are
(A) 4048 (B) 4464 (C) 4518 (D) 4536 both under one superintendent. Suppose that the
superintendentship, the four teachership of English
Q.15 Number of positive integers which have no two and Vernacular school each, are vacant, if there be
digits having the same value with sum of their digits altogether 11 candidates for the appointments, 3 of
being 45, is whom apply exclusively for the superintendentship and
(A) 10! (B) 9! (C) 9.9! (D) 17.8! 2 exclusively for the appointment in the English school,
the number of ways in which the different appointment
Q.16 Number of 3 digit number in which the digit at can be disposed of is :
hundredth’s place is greater than the other two digit is (A) 4320 (B) 268 (C) 1080 (D) 25920
(A) 285 (B) 281 (C) 240 (D) 204
Q.26 A committee of 5 is to be chosen from a group of
Q.17 Number of permutation of 1, 2, 3, 4, 5, 6, 7, 8 and 9 people. Number of ways in which it can be formed if
9 taken all at a time, such that the digit 1 appearing two particular persons either serve together or not at
somewhere to the left of 2, 3 appearing to somewhere all and two other particular persons refuse to serve with
the left of 4 and 5 somewhere to the left of 6, is (e.g. each other, is
815723946 would be one such permutation) (A) 41 (B) 36 (C) 47 (D) 76
(A) 9.7! (B) 8! (C) 5!.4! (D) 8!.4!
Q.27 A question paper on mathematics consists of
Q.18 Number of odd integers between 1000 and 8000 twelve questions divided into three parts A, B and C,
which have none of their digit repeated, is each containing four questions, in how many ways can
an examinee answer five questions, selecting at least
(A) 1014 (B) 810 (C) 690 (D) 1736
one from each part?
Q.19 The number of ways in which 5 different books (A) 624 (B) 208 (C) 1248 (D) 2304
can be distributed among 10 people if each person can
get at most one book is:
(A) 252 (B) 105 (C) 510 (D) 10C5.5!
M a them a ti cs | 5.39
Q.28 Number of ways in which 7 green bottles and 8 Q.36 Number of different words that can be formed
blue bottles can be arranged in a row if exactly 1 pair of using all the letters of the word “DEEPMALA” if two
green bottles is side by side, is (Assume all bottles to be vowels are together and the other two are also together
alike except for the color). but separated from the first two is
(A) 84 (B) 360 (C) 504 (D) None (A) 960 (B) 1200 (C) 2160 (D) 1440
Q.29 The kindergarden teacher has 25 kids in her class. Q.37 In a unique hockey series between India &
She takes 5 of them at a time, to zoological garden as Pakistan, they decide to play on till a team wins 5
often as she can, without taking the same 5 kids more matches. The number of ways in which the series can
than once. Then the number of visits, the teacher makes be won by India, if no match ends in a draw is:
to the garden exceeds that of a kid by: (A) 126 (B) 252 (C) 225 (D) None
(A) C5 – C5 (B) C5
25 24 24
(C) C4
24
(D) None
Q.38 Sameer has to make a telephone call to his friend
Harish, Unfortunately he does not remember the 7 digit
Q.30 A rack has 5 different pairs of shoes. The number phone number. But he remembers that the first three
of ways in which 4 shoes can be chosen from it so that digits are 635 or 674, the number is odd and there is
there will be no complete pair is: exactly one 9 in the number. The maximum number of
(A) 1920 (B) 200 (C) 110 (D) 80 trials that Sameer has to make to be successful is
(A) 10,000 (B) 3402 (C) 3200 (D) 5000
Q.31 Number of ways in which 9 different toys can be
distributed among 4 children belonging to different Q.39 There are 12 guests at a dinner party. Supposing
age groups in such a way that distribution among the that the master and mistress of the house have fixed
3 elder children is even and the youngest one is to seats opposite one another, and that there are two
receive one toy more, is: specified guests who must always, be placed next to
(5!)
2
9! 9! one another; the number of ways in which the company
(A) (B) (C) (D) None
8 2 3!(2!)3 can be placed is :
(A) 20 . 10 ! (B) 22 . 10 ! (C) 44 . 10 ! (D) None
Q.32 There are 10 red balls of different shades & 9
green balls of identical shades. Then the number of Q.40 In a conference 10 speakers are present. If S1
such arrangements such that no two green balls are wants to speak before S2 and S2 wants to speak after
together in the row is: S3, then the number of ways all the 10 speakers can
(A) (10!).11P9 (B) (10!).11C9 (C) 10! (D) 10! 9! give their speeches with the above restriction if the
remaining seven speakers have no objection to speak
Q.33 A shelf contains 20 different books of which 4 are at any number is
10!
in single volume and the others form sets of 8, 5 and (A) 10C3 (B) 10P8 (C) 10P3 (D)
3
3 volumes respectively. Number of ways in which the
books may be arranged on the shelf, if the volumes of
Q.41 The number of all possible selection of one or
each set are together and in their due order is
more questions from 10 given questions, each question
20! having an alternative is:
(A) (B) 7! (C) 8! (D) 7.8!
8!5!3! (A) 310 (B) 210–1 (C) 310 – 1 (D) 210
Q.44 Product of all the even divisors of N = 1000, is Q.52 The number 916238457 is an example of nine
digit number which contains each of the digit 1 to 9
(A) 32 . 102 (B) 64 . 214 (C) 64 . 1018 (D) 128 . 106
exactly once. It also has the property that the digits 1
to 5 occur in their natural order, while the digits 1 to 6
Q.45 A lift with 7 people stops at 10 floors. People
do not. Number of such numbers are
varying from zero to seven go out at each floor. The
number of ways in which the lift can get emptied, (A) 2268 (B) 2520 (C) 2975 (D) 1560
assuming each way only differs by the number of
people leaving at each floor, is Q.53 Number of functions defined from f : {1, 2, 3, 4, 5,
6} → {7, 8, 9, 10} such that the sum f(1) + f(2) + f(3) +
(A) 16C6 (B) 17C7 (C) 16C7 (D) None
f(4) +f(5) + f(6) is odd, is
Q.46 You are given an unlimited supply of each of the (A) 210 (B) 211 (C) 212 (D) 212 – 1
digits 1, 2, 3 or 4. Using only these four digits, you
construct n digit numbers. Such n digit numbers will Multiple Correct Choice Type
be called LEGITIMATE if it contains the digit 1 either
an even number times or not at all. Number of n digit Q.54 The continued product, 2.6.10.14… to n factors is
legitimate numbers are equal to:
(A) 2n + 1 (B) 2n+1 + 2 (C) 2n+2 + 4 (D) 2n–1(2n + 1) (A) 2nCn (B) 2nPn
Q.47 Distinct 3 digit numbers are formed using only the (C) 2n+1Cn (D) None
digits 1, 2, 3 and 4 with each digit used at most once
in each number thus formed. The sum of all possible Q.55 The maximum number of permutations of 2n
numbers so formed is letters in which there are only a’s & b’s, taken all at a
time is given by :
(A) 6660 (B) 3330 (C) 2220 (D) None
(A) 2nCn
Q.48 An ice cream parlor has ice creams in eight 2 6 10 4n – 6 4n – 2
(B) . . … .
different varieties. Number of ways of choosing 3 ice 1 2 3 n–1 n
creams taking at least two ice creams of the same
n + 1 n + 2 n + 3 n + 4 2n – 1 2n
variety, is (Assume that ice creams of the same variety (C) . . . … .
1 2 3 4 n–1 n
to be identical & available in unlimited supply)
2n 1.3.5...(2n – 3)(2n – 1)
(A) 56 (B) 64 (C) 100 (D) None (D)
n!
Q.49 There are 12 books on Algebra and Calculus (E) All of the above
in our library, the books of the same subject being
different. If the number of selection each of which
consists of 3 books on each topic is greatest then the Q.56 Number of ways in which 3 numbers in A.P. can be
number of books of Algebra and Calculus in the library selected from 1, 2, 3,… n is :
are respectively: 2
n–1 n(n – 2)
(A) if n is even (B) if n is odd
(A) 3 and 9 (B) 4 and 8 (C) 5 and 7 (D) 6 and 6 2 4
(A) 8C2 . 6C2 (B) 8C2 . 6C2 . 2 (C) 8C4 . 3 (D) None (C) 52C4 (D) None of these
M a them a ti cs | 5.41
Q.2 Eight chairs are numbered 1 to 8. Two women and Q.5 If r, s, t are prime numbers and p, q are the positive
three men wish to occupy one chair each. First the integers such that LCM of p, q is r2s4t2, then the number
women choose the chairs from amongst the chairs of ordered pairs (p, q) is (2006)
marked 1 to 4, and then the men select the chairs
(A) 252 (B) 254 (C) 225 (D) 224
from amongst the remaining. The number of possible
arrangements is (1982)
Q.6 The letters of the word COCHIN are permuted and
(A) 6C3 × 4C2 (B) 4P2 × 4P3 all the permutations are arranged in an alphabetical
(C) 4C2 + 4P3 (D) None order as in an English dictionary. The number of words
that appear before the word COCHIN is (2007)
Q.3 A five digits number divisible by 3 is to be formed (A) 360 (B) 192 (C) 96 (D) 48
using the numbers 0, 1, 2, 3, 4 and 5, without repetition.
The total number of ways this can be done, is (1989) Q.7 The number of seven digit integers, with sum of the
(A) 216 (B) 240 (C) 600 (D) 3125 digits equal to 10 and formed by using the digits 1, 2
and 3 only, is (2009)
Q.4 Number of divisors of the form (4n + 2), n ≥ 0 of (A) 55 (B) 66 (C) 77 (D) 88
integer 240 is (1998)
(A) 4 (B) 8 (C) 10 (D) 3
JEE Advanced/Boards
Exercise 1
Q.6 In an election for the managing committee of a
Q.1 Consider all the six digit numbers that can be reputed club, the number of candidates contesting
formed using the digits 1, 2, 3, 4, 5 and 6, each digit elections exceeds the number of members to be
being used exactly once. Each of such six digit numbers elected by r(r > 0). If a voter can vote in 967 different
have the property that for each digit, not more than ways to elect managing committee by voting at least
two digits, smaller than that digit, appear to the right 1 of them & can vote in 55 different ways to elect (r –
of that digit. Find the number of such six digit numbers 1) candidates by voting in the same manner. Find the
having the desired property number of candidates contesting the election & the
number of candidates losing the elections.
Q.2 Find the number of five digit number that can be
formed using the digits 1, 2, 3, 4, 5, 5, 7, 9 in which Paragraph for question nos. 7 to 9:
one digit appears once and two digits appear twice (e.g 2 American men; 2 British men; 2 Chinese men and one
41174 is one such number but 75355 is not.) each of Dutch, Egyptian, French and German persons
are to be seated for a round table conference.
Q.3 Find the number of ways in which 3 distinct
numbers can be selected from the set {31, 32, 33, ….. 3100, Q.7 If the number of ways in which they can be seated
3101} so that they form a G.P. if exactly to pairs of persons of same nationality are
together is p(6!), then find p.
Q.4 Find the number of odd numbers between 3000 to
6300 that have all different digits. Q.8 If the number of ways in which only American pair
is adjacent is equal to q(6!), then find q.
Q.5 A man has 3 friend. In how many ways he can invite
one friend every day for dinner on 6 successive nights Q.9 If the number of ways in which no two people of
so that no friend is invited more than 3 times. the same nationality are together given by r (6!), find r.
5 . 4 2 | Permutations and Combinations
Q.10 For each positive integer k, let Sk denote the Q.18 (a) Find the number of non-empty subsets S of {1,
increasing arithmetic sequence of integers whose 2, 3, 4, 5, 6, 7, 8, 9, 10, 11, 12} such that if, S contains k
first term is 1 and whose common difference is k. For elements, then S contains no number less than k.
example, S3 is the sequence 1, 4, 7, 10 ….. Find the
(b) If the number of ordered pairs (S, T) of subsets of
number of values of k for which Sk contain the term 36!
{1, 2, 3, 4, 5, 6} are such that S ∪ T contains exactly three
elements 10λ, then find the value of λ.
Q.11 A shop sells 6 different flavors of ice-cream. In
how many ways can a customer choose 4 ice-cream
Q.19 Find the number of permutation of the digits 1,
cones if
2, 3, 4 and 5 taken all at a time so that the sum of the
(i) They are all of different flavors digits at the first two places is smaller than the sum of
the digit at the last two places.
(ii) They are not necessarily of different flavors
(iii) They contain only 3 different flavors Q.20 In a league of 8 teams, each team played every
(iv) They contain only 2 or 3 different flavors? other team 10 times. The number of wins of the 8 teams
formed an arithmetic sequence. Find the least possible
Q.12 (a) How many divisors are there of the number number of games won by the champion.
21600. Find also the sum of these divisors.
Q.21 Find the sum of all numbers greater than 10000
(b) In how many ways the number 7056 can be resolved formed by using the digits 0, 1, 2, 4, 5 no. digit being
as a product of 2 factors. repeated in any number.
(c) Find the number of ways in which the number
300300 can be split into 2 factors which are relatively Q.22 There are 3 cars of different make available to
prime. transport 3 girls and 5 boys on a field trip. Each car can
hold up to 3 children. Find
(d) Find the number of positive integers that are divisors
of at least one of the number 1010; 157; 1811. (a) the number of ways in which they can be
accommodated.
Q.13 How many 15 letter arrangement of 5A’s, 5 B’s and (b) the numbers of ways in which they can be
5 C’s have no A’s in the first 5 letters, no B’s in the next accommodated if 2 or 3 girls are assigned to one of
5 letters, and to C’s in the last 5 letters. the cars.
Q.14 Determine the number of paths from the origin to In both the cases internal arrangement of children
the point (9, 0) in the Cartesian plane which never pass inside the car is considered to be immaterial.
through (5, 5) in paths consisting only of steps going 1
unit North and 1 unit East. Q.23 Find the number of three elements sets of positive
integers {a, b, c} such that a × b × c = 2310.
Q.15 There are n triangles of positive area that have
one vertex A(0, 0) and the other two vertices whose Q.24 Find the number of integer between 1 and 10000
coordinates are drawn independently with replacement with a least one 8 nd at least one 9 as digits
from the set {0, 1, 2, 3, 4} e.g. (1. 2), (0, 1) (2, 2) etc. Find
the value of n. Q.25 Let N be the number of ordered pairs of non-
empty sets A and B that have the following properties:
Q.16 How many different ways can 15 Candy bars be (a) A ∪ B = {1, 2, 3, 4, 5, 6, 7, 8, 9, 10}
distributed between Ram, Shyam, Ghanshyam and
Balram, if Ram cannot have more than 5 candy bars (b) A ∩ B = φ
and Shyam must have at least two. Assume all Candy (c) The number of elements of A is not the element of B.
bars to be alike
(d) The number of elements of B is not an element of A.
Q.17 Find the number of three digits number from 100 Find N.
to 999 inclusive which have any one digit that is the
average of the other two. Q.26 In how many other ways can be letters of the word
MULTIPLE be arranged:
(i) Without changing the order of the vowels
M a them a ti cs | 5.43
(ii) Keeping the position of each vowel fixed and Q.5 Number of different natural numbers which are
without changing the relative order/position or vowels smaller than two hundred million and use only the
& consonants. digits 1 or 2 is
(A) (3) . 28 – 2 (B) (3) . 28 – 1
Q.27 Let N denotes the number of all 9 digits numbers if
(C) 2 (29 – 1) (D) None
(a) The digit of each number are all from the set {5, 6,
7, 8, 9} and
Q.6 There are counters available in x different colors.
(b) Any digit that appears in the number, repeats at The counters are all alike except for the color. The total
least three times. Find the value of N/5. number of arrangements consisting of y counters,
assuming sufficient number of counters of each color,
Q.28 How many integers between 1000 and 9999 have if no arrangement consists of all counters of the same
exactly one pair of equal digit such as 4049 or 9902 but color is:
not 4449 or 4040? (A) xy – x (B) xy – y (C) yx – x (D) yx – y
Q.29 How many 6 digits odd numbers greater than Q.7 If m denotes the number of 5 digit numbers of
60,000 can be formed from the digits 5, 6, 7, 8, 9, 0 if each successive digits are in their descending order
(i) Repetitions are not allowed magnitude and n is the corresponding figure, when the
digits are in their ascending order of magnitude then
(ii) Repetitions are allowed.
(m – n) has the value
(A) 10C4 (B) 9C5 (C) 10C3 (D) 9C3
Exercise 2
Q.8 There are m points on straight line AB & n points on
Single Correct Choice Type the line AC none of them being the point A. Triangles
are formed with these points as vertices, when
Q.1 An eight digit number divisible by 9 is to be formed
(i) A is excluded
by using 8 digits out of the digits 0, 1, 2, 3, 4, 5, 6, 7 8, 9
without replacement. The number of ways in which this (ii) A is included. The ration of number of triangles in
can be done is the two cases is:
(A) 9! (B) 2(7!) (C) 4(7!) (D) (36) (7!) m+n–2 m+n–2
(A) (B)
m+n m+n–1
Q.2 Number of 4 digit numbers of the form N = abcd m+n–2 n(n – 1)
(C) (D
which satisfy following three conditions m+n+2 (m + 1)(n + 1)
(i) 4000 ≤ N < 6000
Q.9 The number of 5 digit numbers such that the sum
(ii) N is a multiple of 5
of their digits is even is
(iii) 3 ≤ b < c ≤ 6 is equal to
(A) 50000 (B) 45000 (C) 60000 (D) None
(A) 12 (B) 18 (C) 24 (D) 48
Q.10 Number of ways in which 8 people can be
Q.3 5 Indian & 5 American couples meet at a party arranged in a line if A and B must be next each other
and shake hands. If no wife shakes hands with her own and C must be somewhere behind D, is equal to
husband and no Indian wife shakes hands with a male
(A) 10080 (B) 5040 (C) 5050 (D) 10100
then the number of handshakes that takes place in the
party is
Q.11 Seven different coins are to be divided amongst
(A) 95 (B) 110 (C) 135 (D) 150 three persons. If no two of the persons receive the same
number of coins but each receives at least one coin &
Q. 4 The 9 horizontal and 9 vertical lines on an 8 × 8 none is left over, then the number of ways in which the
chessboard form ‘r’ rectangles and ‘s’ squares, The ratio division may be made is
s/r in its lowest terms is
(A) 420 (B) 630 (C) 710 (D) None
1 17 4
(A) (B) (C) (D) None
6 108 27
5 . 4 4 | Permutations and Combinations
Q.12 Let there be 9 fixed point on the circumference of Q.19 Number of rectangles in the grid shown which are
a circle. Each of these points is joined to every one of not squares is
the remaining 8 points by a straight line and the points
are so positioned on the circumference that at most 2
straight lines meet in any interior point of the circle. The
number of such interior intersection points is:
(A) 126 (B) 351 (C) 756 (D) None
(A) 160 (B) 162 (C) 170 (D) 185
Q.13 The number of ways in which 8 distinguishable
apples can be distributed among 3 boys such that every Q.20 All the five digit numbers in which each successive
boy should get at least 1 apple & at most 4 apples is K. digit exceeds its predecessor are arranged in the
7
P3 where K has the value equal to increasing order of their magnitude. The 97th number in
(A) 14 (B) 66 (C) 44 (D) 22 the list does not contains the digit
(A) 4 (B) 5 (C) 7 (D) 8
Q.14 There are five different peaches and three different
apples. Number of ways they can be divided into two Q.21 There are n identical red balls & m identical green
packs of four fruits if each pack must contain at least balls. The number of different linear arrangements
one apple, is consisting of n red ball but not necessarily all the green
(A) 95 (B) 65 (C) 60 (D) 30 balls’ is xCy then
(A) x = m + n, y = m
Q.15 Let Pn denote the number of ways in which three
people can be selected out of ‘n’ people sitting in a row, (B) x = m + n + 1, y = ,m
if no two of them are consecutive. If Pn+1 – Pn = 15 then (C) x = m + n + 1, y = m + 1
the value of ‘n’ is
(D) x = m + n, y = n
(A) 7 (B) 8 (C) 9 (D) 10
Q.22 A gentleman invites a party of m + n (m ≠ n) friends
Q.16 The number of positive integers not greater than to a dinner & places m at one table T1 and n at another
100, which are not divisible by 2, 3 or 5 is table T2, the table being round, if not all people shall have
(A) 26 (B) 18 (C) 31 (D) None the same neighbor in any two arrangement, then the
number of ways in which he can arrange the guests, is
Q.17 There are six periods in each working day of a (m + n)! 1 (m + n)!
(A) (B)
school. Number of ways in which 5 subjects can be 4mn 2 mn
arranged if each subject is allotted at least one period
(m + n)!
and no period remains vacant is (C) 2 (D) None
mn
(A) 210 (B) 1800 (C) 360 (D) 3600
Q.23 Consider a determinant of order 3 all whose
Q.18 An old man while dialing a 7 digit telephone entries are either 0 or 1. Five of these entries are 1 and
number remembers that the first four digits consists of four of them are ‘0’. Also aij = aji ∀ 1 ≤ i, j ≤ 3. The
one 1’s, one 2’s and two 3’s. He also remembers that number of such determinants, is equal to
the fifth digit is either a 4 or 5 while has no memorizing
(A) 6 (B) 8 (C) 9 (D) 12
of the sixth digit, he remembers that the seventh digit
is 9 minus the sixth digit. Maximum number of distinct
Q.24 A team of 8 students goes on an excursion, in
trials he has to try to make sure that he dials the correct
two cars, of which one can seat 5 and the other only 4.
telephone number, is
If internal arrangement inside the car does not matter
(A) 360 (B) 240 (C) 216 (D) None then the number of ways in which they can travel, is
(A) 91 (B) 182 (C) 126 (D) 3920
M a them a ti cs | 5.45
Q.25 One hundred management students who read at Q.33 The number of ways of choosing a committee
least one of the three business magazines are surveyed of 2 women & 6 men, if Mr. A refuses to serve on the
to study the readership pattern. It is found that 80 committee if Mr. B is a member & Mr. B can only serve,
read Business India, 50 read Business world, and 30 if Miss C is the member of the committee, is
read Business Today. Five students read all the three
(A) 60 (B) 84 (C) 124 (D) None
magazines. How many read exactly two magazines?
(A) 50 (B) 10 (C) 95 (D) 45 Q.34 Six person A, B, C, D, E and F are to be seated at a
circular table. The number of ways this can be done if
Q.26 Six people are going to sit in a row on a bench. A A must have either B or C on his right and B must have
and B are adjacent, C does not want to sit adjacent to either C or D on his right is:
D. E and F can sit anywhere. Number of ways in which
(A) 36 (B) 12 (C) 24 (D) 18
these six people can be seated, is
(A) 200 (B) 144 (C) 120 (D) 56 Q.35 There are 100 different books in a shelf. Number
of ways in which 3 books can be selected so that no two
Q.27 Number of cyphers at the end of 2002C1001 is of which are adjacent is
(A) 0 (B) 1 (C) 2 (D) 200 (A) 100C3 – 98 (B) 97C3 (C) 96C3 (D) 98C3
(A) Heptagon (B) Octagon (A) 42 (B) 100 (C) 150 (D) 216
(C) Nonagon (D) Decagon Q.37 A 3 digit palindrome is a 3 digit number (not
starting with zero) which reads the same backwards as
Q.29 Given 11 points, of which 5 lie on one circle, other forwards. For example 171. The sum of all even 3 digit
than these 5, no 4 lie on one circle. Then the maximum palindromes, is
number of circles that can be drawn so that each
contains at least three of the given points is: (A) 22380 (B) 25700 (C) 22000 (D) 22400
(A) 216 (B) 156 (C) 172 (D) None Q.38 Two classrooms A and B having capacity of 25
and (n–25) seats respectively An denotes the number
Q.30 Number of 5 digit numbers divisible by 25 that of possible seating arrangements of room ‘A’, when
can be formed using only the digits 1, 2, 3, 4, 5, & 0 ‘n’ students are to be seated in these rooms, starting
taken five at a time is from room ‘A’ which is to be filled up full to its capacity.
(A) 2 (B) 32 (C) 42 (D) 52 If An – An–1 = 25! (49C25) then ‘n’ equals
(A) 50 (B) 48 (C) 49 (D) 51
Q.31 Let Pn denotes the number of ways of selecting
3 people out of ‘n’ sitting in a row, if no two of them Q.39 Number of positive integral solution satisfying
are consecutive and Qn is the corresponding figure the equation (x1 + x2 + x3) (y1 + y2) = 77, is
when they are in a circle. If Pn – Qn = 6, then ‘n’ is
equal to (A) 150 (B) 270 (C) 420 (D) 1024
(A) 8 (B) 9 (C) 10 (D) 12 Q.40 There are counters available in 3 different colors (at
least four of each color). Counters are all alike except for the
Q.32 Let m denote the number of ways in which 4 color. If ‘m’ denotes the number of arrangements of four
different books are distributed among 10 persons, counters if no arrangement consists of counters of same
each receiving none or one only and let n denote the color and ‘n’ denotes the corresponding figure when every
number of ways of distribution if the books are all alike. arrangement consists of counters of each color, then:
Then:
(A) m = 2 n (B) 6 m = 13 n
(A) m = 4n (B) n = 4m (C) m = 24 n (D) none
(C) 3 m = 5 n (D) 5 m = 3 n
5 . 4 6 | Permutations and Combinations
Q.41 Three digit numbers in which the middle one is a Multiple Correct Choice Type
perfect square are formed using the digits 1 to 9. Their
sum is: Q.46 The combinatorial coefficient C(n, r) is equal to
(A) 134055 (B) 270540 (A) number of possible subsets of r members from a set
of n distinct members.
(C) 170055 (D) none
(B) number of possible binary messages of length n
Q.42 A guardian with 6 wards wishes every one of with exactly r l’s.
them to study either Law of Medicine or Engineering. (C) number of non-decreasing 2-D paths from the
Number of ways in which he can make up his mind with lattice point (0, 0) to (r, n).
regard to the education of his wards if every one of
them be fit for any of the branches to study, and at least (D) number of ways of selecting r objects out of n
one child is to be sent in each discipline is: different objects when a particular object is always
included plus the number of ways of selecting ‘r’
(A) 120 (B) 216 (C) 729 (D) 540 objects out of n, when a particular object out of n, when
a particular object is always excluded.
Q.43 There are (p + q) different books on different
topics in Mathematics (p ≠ q) Q.47 There are 10 questions, each question is either
If L = The number of ways in which these books are True or False. Number of different sequences of
distributed between two students X and Y such that X incorrect answers is also equal to
get p books and Y gets q books. (A) Number of ways in which a normal coin tossed 10
M = The number of ways in which these books are times would fall in a definite order if both Heads and
distributed between two students X and Y such that Tails are present.
one of them gets p books and another gets q books. (B) Number of ways in which a multiple choice question
N = The number of ways in which these books are containing 10 alternatives with one or more than one
divided into two groups of p books and q books then, correct alternatives, can be answered.
Q.44 Number of ways in which 5A’ and 6B’s can be (D) Number of different selection of 10 indistinguishable
arranged in a row which reads the same backwards and objects taken some or all at a time.
forwards, is
Q.48 The number of ways in which five different books
(A) 6 (B) 8 (C) 10 (D) 12 can be distributed among 3 persons so that each
person gets at least one book, is equal to the number
Q.45 Coefficient of x2 y3 z4 in the expansion of (x + y + of ways in which
z)9 is equal to
(A) 5 persons are allotted 3 different residential flats so
(A) The number of ways in which 9 objects of which 2 that and each person is allotted at most one flat and no
alike of one kind, 3 alike of 2nd kind, and 4 alike of 3rd two persons are allotted the same flat.
kind can be arranged.
(B) Number of parallelograms (some of which may
(B) The number of ways in which 9 identical objects can be overlapping) formed by one set of 6 parallel lines
be distributed in 3 persons each receiving at least two and other set of 5 parallel lines that goes in other
objects. direction.
(C) The number of ways in which 9 identical objects can (C) 5 different toys are to be distributed among 3
be distributed in 3 persons each receiving none one or children, so that each child gets at least one toy.
more.
(D) 3 mathematics professors are assigned five different
(D) The number of ways in which 9 different books can lecturers to be delivered, so that each professor gets at
be tied up in to three bundles one containing 2, other 3 least one lecturer.
and third containing 4 books.
M a them a ti cs | 5.47
Q.49 If k is odd then kCr is maximum for r equal to Q.54 If the number of arrangements of the letters of
the word W if all the S’s and P’s are separated is (k)
1 1
=
(A) r (k − 1) =
(B) r (k + 1) 10!
2 2
then k equals
4!.4!
(C) k ‒ 1 (D) k
6 4 3
(A) (B) 1 (C) (D)
Q.50 Which of the following statements are correct? 5 3 2
(A) Number of words that can be formed with 6 only
of the letters of the word “CENTRIFUGAL’ if each word Paragraph 2: 16 players P1, P2, P3, ….. P16 take part in
must contain all the vowels is 3 . 7! a tennis tournament. Lower suffix player is better than
any higher suffix player. These players are to be divided
(B) There are 15 balls of which some are white and the into 4 groups each comprising of 4 players and the best
rest black. If the number of ways in which the balls con from each group is selected for semifinals.
be arranged in a row, is maximum then the number of
white balls must be equal to 7 or 8. Assume balls of the Q.55 Number of ways in which 16 players can be
same color to be alike. divided into four equal groups, is
(C) There are 12 objects, 4 alike of one kind, 5 alike and 35 8 35 8
of another kind and the rest are all different. The total (A) ∏ (2r – 1)
27 r =1
(B) ∏ (2r – 1)
24 r =1
number of combinations in 240
(D) Number of selections that can be made of 6 letters 35 8 35 8
from the word “COMMITTEE” is 35.
(C) ∏ (2r – 1)
52 r =1
(D) ∏ (2r – 1)
6 r =1
Q.51 Number of ways in which the letters of the word Q.56 Number of ways in which they can be divided into
‘B U L B U L’ can be arranged in a line is a definite order 4 equal groups if the players P1, P2, P3 and P4 are in
is also equal to the different groups, is:
(A) Number of ways in which 2 alike Apples and 4 alike (11)! (11)! (11)! (11)!
(A) (B) (C) (D)
Mangoes can be distributed in 3 children so that each 36 72 108 216
child receives any number of fruits.
(B) Number of ways in which 6 different books can be Match the Columns
tied up into 3 bundles, if each bundle is to have equal Q.57
number of books.
Column-I Column-II
(C) Coefficient of x2y2z2 in the expansion of (x + y + z)6.
(A) Number of increasing permutations (p) nm
(D) Number of ways in which 6 different prizes can be
of m symbols are there from the n set
distributed equally in three children.
numbers {a1, a2, …, an} where the order
among the number is given by a1 < a2 < a3
Comprehension Type < … an–1 < an is
Paragraph 1: Consider the word W = MISSISSIPPI (B) There are m men and n monkeys. (q) mCn
Number of ways in which every monkey
Q.52 If N denotes the number of different selections has a master, if a man can have any number
of 5 letters from the word W = MISSISSIPPI then N of monkeys
belongs to the set (C) Number of ways in which n red balls are (r) nCm
(A) {15, 16, 17, 18, 19} (B) {20, 21, 22, 23, 24} (m – 1) green balls can be arranged in a
line, so that no two red balls are together, is
(C) {25, 26, 27, 28, 29} (D) {30, 31, 32, 33, 34} (balls of the same color are alike)
Q.53 Number of ways in which the letters of the word (D) Number of ways in which ‘m’ different (s) mn
W can be arranged if at least one vowel is separated toys can be distributed in ‘n’ children if every
from rest of the vowels child may receive any number of toys, is
Column I Column II
Previous Years’ Questions (A) The number of permutations containing (p) 5!
the word ENDEA. is
Q.1 Five balls of different colors are to be placed in
three boxes of different sizes. Each box can hold all five. (B) The number of permutations in which (q) 2 × 5!
In how many different ways can we place the balls so the letter E occurs in the first and the last
that no box remains empty? (1981) positions, is
(C) The number of permutations in which (r) 7 × 5!
none of the letters D, L, N occurs in the last
five positions, is
(D) The number of permutations in which the (s) 21×5!
letters A, E, O occur only in odd positions, is
M a them a ti cs | 5.49
PlancEssential Questions
JEE Main/Boards JEE Advanced/Boards
Exercise 1 Exercise 1
Q.6 Q.10 Q.13 Q.16 Q.22 Q.5 Q.6 Q.12 Q.13 Q.16
Q.26 Q.27 Q.20 Q.22 Q.28 Q.30
Exercise 2 Exercise 2
Q.3 Q.13 Q.15 Q.18 Q.25 Q.1 Q.3 Q.8 Q.13 Q.20
Q.33 Q.43 Q.46 Q.47 Q.50 Q.26 Q.32 Q.39 Q.42 Q.43
Q.52 Q.49 Q.55 Q.58
Answer Key
25 = 2 + 4 + 8 + 16 + 32 = 62
Q.26 26
Q.11 34650 – 840 = 33810
Q.27 150
Q.12 4! 3!
Q.28 2454
5 . 5 0 | Permutations and Combinations
Exercise 2
JEE Advanced/Boards
Exercise 1
Q.1 162 Q.2 7560 Q.3 2500 Q.4 826 Q.5 510 Q.6 10, 3
Q.7 60 Q.8 64 Q.9 244 Q.10 24 Q.11 (i)15, (ii) 126, (iii) 60 (iv) 105
Q.12 (a) 72; 78120; (b) 23 (c) 32; (d) 435 Q.13 2252 Q.14 30980 Q.15 276
Q.16 440 Q.17 121 Q.18 (a) 128; (b) 54 Q.19 48 Q.20 42
Q.21 3119976 Q.22 (a) 1680; (b) 1140 Q.23 40 Q.24 974 Q.25 186
Q.26 (i) 3359; (ii) 59; (iii) 359 Q.27 4201 Q.28 3888 Q.29 (i) 240, (ii) 15552
Exercise 2
Comprehension Type
Q.52 C Q.53 B Q.54 B Q.55 A Q.56 C
Solutions
n!
Sol 5: nP4 = 360; = 360
Sol 1: (n – 4)!
n(n – 1)(n – 2)(n – 3) = 360 ⇒ 6 × 5 × 4 × 3 = 360
0, 1, 4, 7 1 or 7
n=6
Total numbers = 4 × 4 × 2 = 32
Sol 6: Case-I: 4 digits
10r2
Sol 2:
× ×
8 9 9
× × × × × × × ×
× × Number of numbers with at least
8
C6 – 2 = 26 One digit 7 = 900 – 8 × 9 × 9 = 252
Sol 4: (A) Y, G, B, R
5
8 9
choices
4 3 3 3 3 3 choices choices
4 × 35 = 12 × 81 8 × 9 × 5 = 360
Total ways = 5 + 360 = 365
Sol 5: (B) Prime number in 0 – 9
2, 3, 5, 7 Sol 12: (B) 5
1 1 1 1 1 8 8 7 6
4 4 4 4 4 5 4 3 2 0
2 × 45 = 2048 4
C1 × 8 × 7 × 6
↓↓ 9
9 × 10 × 19
5! 4.4! ∑ n2 = 6
= 285
n=1
Total = 9.4!
= 9 × 24 = 216 Sol 17: (A) Consider 1 & 2, 3 & 4, 5 & 6 to be identical
9!
Permutations = = 9.7!
Sol 9: (A) C2 × 2 = 672
7 5
(2!)3
5 choices
M a them a ti cs | 5.55
Sol 19: (D) Sol 28: (C) Combine those 2 green bottles
↓ ↓ ↓ ↓ ↓ ↓ ↓ ↓ ↓ ↓ ××××××××
× × × × × × × × × × 9 spaces
10 persons 6 × 9C6 = 84 × 6 = 504
10
C5 . 5!
Sol 29: (B) 25C5 – 24C4 = 24C5
Choose 5 persons from 10 getting books & distribute
books to then 5! ways .
Sol 30: (D) 5C4 × 24 = 80
Sol 20: (A) 5C3 × 8C7 + 5C4 × 8C6 + 5C5 × 8C5 Select 4 pairs out of 5 different pairs. Now in each pair
you can choose 2 different shoes.
= 80 + 5 × 28 + 56 = 276
Total = 162
Sol 32: (B) × × × × × × × × × ×
(n – 2) 10 red balls
Sol 22: (C) × 180 = 150
n
11 spaces
6n – 12 = 5n ⇒ n = 12
choose 9 spaces to fill green balls. 10! ways to arrange
Diagonals = 12C2 – 12 = 54
red balls.
11
C9 × 10!
Sol 23: (C) 2n – 2 = 510
2n = 512
Sol 33: (C) 7! × 23
n=9
7! ways to arrange
2 order (ascending/descending)
Sol 24: (C) 7
7
9 8 7 8 8 7 6!
Sol 34: (D)
2! 2! 2!
9 × 8 × 7 + 8 × 8 × 7 × 3= 33 × 8 × 7= 1848
Consider man and wife to be identical and arrange them.
Sol 25: (D) Total appointment = 11 Similar concept as used in Q. 18
11 → 2 3 6 7! 7! 7! 7!
Sol 35: (C) + + .......
7! 0! 6! 1! 5! 2! 2! 5!
total ways to disposed = (2! 3! 6!) × 3 = 8640 × 3= 25920
7
C0 + 7C1 + 7C2 ....... 7C5 = 27 – 7C6 – 7C7 = 128 – 7 – 1 = 120
Sol 26: (A) Illegal ways = 2 × 7C4 + 7C3 – 2 × 5C2 = 85
No. of possible ways = 9C5 – 85= 126 – 85 = 41 Sol 36: (D)
× × × ×
Sol 27: (A) 1 1 3 EE AA
122 EAEA
Sol 37: (A) The last match has to be won by India Total ways = 7 + 10–1C10–1= 16C9 = 16C7
1 + 5C1 + 6C2 + 7C3 + 8C4= 126
Sol 46: (D) Total ways = 3n+ nC2 .3n–2+nC4.3n–4+......
Sol 38: (B) (1 + 3)n 2n
= + = 22n–1 + 2n–1 = 2n–1(2n + 1)
2 2
635/674
Sol 47: (A) Each digit will be present at unit’s ten’s and
2(93 × 1 + 3 × 4 × 92) = 2(1701) = 3402 hundred’s place 6 times.
(1 + 2 + 3 + 4) × 6 = 60
Sol 39: (A)
60
Master
× × 60 ×
× × 60 × ×
× × Sum of digits =
× × 6660
× ×
× × Sol 48: (B) 8C1 + 8C1 × 7C1 = 64
Mistress
n–1 (n – 1)2
Total A.P.S. = 0 + 1.....+ +....+1+ 0 = Pq Number of ways
2 4
R0r2 1 way
n is even
R1r21 way
n n
1, 2, ....... , + 1, ....... n
2 2 R2r0, r1, r23 ways
Total A.P.S ∴ Total number of ways to select r = 5.
n n(n – 2) Selection of s as under
= 2 × 0 + 1 + ....... – 1 =
2 4
s0s41 way
s1s41 way
Previous Years’ Questions s2s41 way
5 s3s41 way
Sol 1: (C) Here, 47C4 + ∑ 52– j
C3
s4s41 way
j=1
= (47C4 + 47C3) + 48C3 + 49C3 + 50C3 + 51C3 Similarly, the number of ways to select t = 5.
∴ Total number of ways = 5 × 9 × 5 = 2Sol.25
(using nCr + nCr–1 = n+1Cr)
= (48C4 + 48C3) + 49C3 + 50C3 + 51C3 Sol 6: (C) Arrange the letters of the word COCHIN as in
the order of dictionary CCHINO.
= (49C4 + 49C3) + 50C43 + 51C3
Consider the words starting from C.
= (50C4 + 50C3) + 51C3 = 51C4 + 51C3 = 52C4
There are 5! Such words. Number of words with the two
C’ s occupying first and second place = 4!.
Sol 2: (D) Since, the first 2 women select the chairs
amongst 1 to 4 in 4P2 ways Sol 7: (C) There are two possible cases
Now, from the remaining 6 chairs, three men could be
arranged in 6P3. Case-I Five 1’s , one 2’s, one 3’s
7!
∴ Total number of arrangements=4P2×6P3. Number of numbers = = 42
5!
Sol 3: (A) Since, a five digits number is formed using Case-II Four 1’s three 2’s
the digits {0, 1, 2, 3, 4 and 5} divisible by 3 ie, only 7!
possible when sum of the digits is multiple of three. Number of numbers = = 35
4! 3!
Case I : Using digits 0, 1, 2, 4, 5 Total number of numbers = 42 + 35 = 77
Number of ways = 4 × 4 × 3 × 2 × 1 = 96
Case II : Using digits 1, 2, 3, 4, 5 JEE Advanced/Boards
Number of ways = 5×4×3×2×1 = 120
∴ Total number formed =120+96=216 Exercise 1
3
× 3
× 3 ×3 × 2 = 81 × 2 = 162
Sol 5: (C) Since, r, s, t are prime numbers. Filling first last two places
4 places
∴ Selection of p and q are as under
5 . 5 8 | Permutations and Combinations
2 8 7 4 8 7 4 Total factors = 4 × 3 × 2= 24
2 × 8 × 7 × 4 =448 8 × 7 × 5 = 280
Sol 11: (i) 6C4 = 15
0/2
(ii) All, 4 diff. 6C4 = 15
6 1 6
2diff, 2Alike 6C2 × 4C1 = 60
7 4 2 7 5 2 alike, 2alike6C2 = 15
1 diff, 3alike 2 × 6C2 = 30
7 × 4 = 282 × 7 × 5 = 70
4 alike 6C1 = 6
Total = 826
126
Sol 5: Case-I: 2 2 2 (iii)3 different flavours = 60
6! (iv) 2 or 3 different flavours =60 +15 + 30 = 105
= 90
(2!)3
Sol 12: (a) x = 21600
Case-II: 1 2 3
6! x = 63 × 100 = 25 33 52
× 3! = 360
1! 2! 3! No. of divisors = 6 × 4 × 3 = 72
HCF of 157 & 1811 = 37 Sol 20: Total wins = Total matches = 10 × 8C2
Total divisors = (11×11+8×8+12×23) – (8+11+8)+1=435 Wins of champion = A + 7d
5 8
[2A + 7d] = 280
Sol 13: ∑ (5 Cx )3 = 1 + 5 3
+ 103 + 103 + 53 + 1 = 2252 2
x =0
A = 70 – 7d
2
Sol 14: 18C9 – 10C5 × 8C4= 48620 – 252 × 70 = 30980 Now d ≠ 0 d = 2
A = 28
Sol 15: 52 – 1 = 24 ∴ 24C2 triangles
Wins of champion = 28 + 7 × 2 = 42
Sol 16: x + y + 2 + w = 13
S1 S2 S3 S4 S5
x ≤ 5y ≥ 2 Sol 21:
10
C2 + C2 +.... C2= 45 + 55 + 66 + 78 + 91 + 105 = 440
11 15
S1 = (1 + 2 + 4 + 5) × 4! = 288
S2 = S3 = S4 = S5 =(1 + 2 + 4 + 5) × 3 × 3! = 216
Sol 17: 2 digits should be odd or
Sum = S1 × 104 + S2(103 + 102 + 10 + 1)
2 digits should be even for average to be integer
= 288 × 104 + 216(1111) = 3119976
111 222 ......... 999 each repeats 3 times
C2× 5 C1 ×5 C1 +3C2× 4 C1 ×4 C1 + 2 C1 × 4 × 3 – 2
3
× 9 = 121 Sol 22: (a) 8C3 × 5C3 × 3! = 1680
repetition
odd even/0 zero 5! 5!
(b) 3C1 2 + 3C 2 × 3 × 2 × × 2!
3! 2! 1! 2! 2!
Sol 18: (a) 1 element 12C1 = 12 = 60 + 1080 = 1140
2 elements C2 = 45 10
a, b, c distinct
3 elements 8C3 = 56
Sol 23: {1, x, x} (5C1 + 5C2) = 15
4 elements 6C4 = 15
5! 5!
5 elements 0 {x, x, x} + = 25
1! 1! 3! 2! 1! 2! 2! 2!
Total = 128
Total = 15 + 25 = 40
(b) Select 3 elements
Each is this element is either present in S, T or both Sol 24: No. of integers
S and T
2 digit no. 89, 98 2
∴ Total = 6C3 × 33 = 540
3 digit no.
λ = 54
Zero included 890, 809 4
N1 m+n–2
=
N2 m+n
Exercise 2
Single Correct Choice Type Sol 9: (B) 10000 to 99999
90000
Number of numbers = = 45000
Sol 1: (D) We have 0 + 1 + 2 + 3 … + 8 + 9 = 45 2
To obtain an eight digit number exactly divisible by 9,
we must not use either (0, 9) or (2, 7) or (3, 6) or (4, 5). Sol 10: (B) AB D C E F G H
[Sum of the remaining eight digits is 36 which is exactly 7!
divisible by 9.] 2× = 5040
2!
When, we do not use (0, 9), then the number of required
8 digit number is 8!. 7!
Sol 11: (B) × 3! = 630
1! 2! 4!
When, one of (1, 8) or (2, 7) or (3, 6) or (4, 5) is not used,
the remaining digits can be arranged in 8! – 7! ways as
Sol 12: (A) 9C4 = 126
0 cannot be at extreme left.
Hence, there are 8! + 4(8! – 7!) = (36) (7!) numbers in
the desired category.
M a them a ti cs | 5.61
Sol 14: (D) Pack 1 Pack 2 Sol 21: (B) nCn + n+1Cn + n+2Cn + ........ n+mCn
3122
1 1 m+n
Sol 22: (A) × × Cm(m – 1)(n – 1)!
5
C3 × 3C1 = 30 2 2
1 (m + n)!
Sol 15: (D) = ×
4 mn
× × × ×
n–3
1 x x
(n – 2) choices. select 3 Sol 23: (D) x 1 x 3
C1 = 3
Pn = C3
n–2
x x 1
Pn+1 – Pn = n–1C3 – n–2C3 = n–2C2 = 15 1 x x
n–2=6⇒n=8 x 0 x 3C 1 × 3C 2 = 9
x x 0
Sol 16: (A) n(2) = 50n(2 ∩ 3) = 16
Total = 12
n(3) = 33 n(3 ∩ 5)=6 n(2 ∩ 3 ∩ 5) =5
n(5) = 20n(2 ∩ 5) = 10 Sol 24: (C) 5 3 or 4 4
n(2 ∪ 3 ∪ 5)=(50 + 33 + 20) – (16 + 6 + 10)+3 8
C5 + 8C4 = 126
n( 2 ∪ 3 ∪ 5 ) = 100 – 74 = 26
Sol 25: (A) 100 = (80 + 50 + 30) – n + 5
6! n = 65
Sol 17: (B) 5C1 × = 1800
2! People reading exactly 2 magazines
Choose subject with two periods and then arrange.
= 65 – 3 × 5 = 50
Sol 28: (C) No. of triangles = nC3 – [(n(n –4)) + n] Sol 37: (C) S1 S2 S3
n(n – 1)(n – 2)
= – [(n2 – 3n)] = 30 ⇒ n = 9
3! S1 = S3 = (2 + 4 + 6 + 8) × 10 = 200
S2 = (0 + 1 + 2 + ....... 9) × 4 = 180
Sol 29: (B) 11C3 – 5C3 + 1 = 156
Sum = 200 + 180 × 10 + 200 × 100 = 22000
Sol 30: (C)
Sol 38: (A) An = nC25 × 25!
2 5 5 10
An – An–1 = 25! (nC25 – n–1C25) = 25!(n–1C24)
3! + 3 × 2 × 2! 4C3 × 3! = 18 = 24 n = 50
Total no.s = 18 + 24 = 42
Sol 39: (C) (x1 + x2 + x3) (y1 + y2) = 77 = 7 × 11
6!
(D) × 3! = 90
Sol 44: (C) There are 11 position (2!)3 3!
At the 6th position A should be present. In the 5 positions
left to 6th positions 2positions will have A. Comprehension Type
5
C2 ways Sol 52: (C) M I(4) S(4)P(2)
(i) 2C1 × 3C1 = 6
Sol 45: (D) 9C2 × 7C3 × 4C4
(ii) 2C1 × 2C1 = 4
9!
(A)
2! 3! 4! (iii) 2C1 × 3C1 = 6
9! (iv) 3C2 × 2 = 6
(D)
2! 3! 4!
(v) 3C1 = 3
Adding all these, we get = 25
Multiple Correct Choice Type
Sol 53: (B) Total ways in which all vowels are together
Sol 46: (A, B, D) n–1Cr–1 + n–1Cr = nCr
11! 8! 8! 165 8!.161
= – = – 1 =
Sol 47: (B, C) 2 – 1 10 4! 4! 2! 4! 2! 4! 2! 4 4 4! 2!
(D) 35
Match the Columns
Sol 1: Since, each box can hold five balls. Sol 5: There are 9 women and 8 men. A committee of
∴ Number of ways in which balls could be distributed 12, consisting of at least 5 women, can be formed by
so that none is empty are (2, 21) or (3, 1, 1). choosing:
ie, (5C2 3C2 1C1 + 5C3 2C1 1C1) × 3! (i) 5 women and 7 men
(ii) 6 women and 6 men
= (30 + 20) × 6 = 300
(iii) 7 women and 5 men
Sol 2: The possible cases are (iv) 8 women and 4 men
Case-I: A man invites 3 ladies and women invites 3 (v) 9 women and 3 men
gentlemen ∴ Total number of ways forming the committee
Number of ways = 4C3 . 4C3 = 16 = 9 C 5 × 8C 7 + 9C 6 × 8C 6 + 9C 7 × 8C 5 + 9C 8 × 8C 4 + 9C 9 × 8C 3
Case-II: A man invites (2 ladies, 1 gentlemen) and = 126 × 8 + 84 × 28 + 36 × 56 + 9 × 70 + 1 × 56 = 6062
women invites (2 gentlemen, 1 lady). (i) Clearly, women are in majority in (iii), (iv) and (v)
Number of ways cases as discussed above.
= (4C2.3C1) (3C1.4C1) = 324 (ii) So, total number of committees in which women are
in majority
Case-III: A man invites (1 lady, 2 gentlemen) and
= 9C 7 × 8C 5 + 9C 8 × 8C 4 + 9C 9 × 8C 3
women invites (2 ladies, 1 gentleman).
= 36 × 56 + 9 × 70 + 1 × 56 = 2702
Number of ways
Clearly, men are in majority in only (i) case as discussed
= (4C1.3C2).(3C2.4C1)= 144 above.
Case-IV: A man invites (3 gentlemen) and women So, total number of committees in which men are in
invites (3 ladies). majority
= 9C5 × 8C7 = 126 × 8 = 1008
M a them a ti cs | 5.65
Sol 6: A → p; B → s; C → q; D → q
MATHEMATICS
FOR JEE MAIN & ADVANCED
SECOND
EDITION
Exhaustive Theory
(Now Revised)
Formula Sheet
9000+ Problems
based on latest JEE pattern
PlancEssential
Questions recommended for revision
6. COMPLEX NUMBER
1. INTRODUCTION
The number system can be briefly summarized as N ⊂ W ⊂ I ⊂ Q ⊂ R ⊂ C , where N, W, I, Q, R and C are the
standard notations for the various subsets of the numbers belong to it.
N - Natural numbers = {1, 2, 3 …. n}
W - Whole numbers = {0, 1, 2, 3 …. n}
I - Integers = {….2, -1, 0, 1, 2 …..}
1 3
Q – Rational numbers = , .....
2 5
IR – Irrational numbers = { 2, 3, π}
C – Complex numbers
A complex number is generally represented by the letter “z”. Every complex number z, can be written as, z= x + i y
where x, y ∈ R and i= −1 .
x is called the real part of complex number, and
y is the imaginary part of complex number.
Note that the sign + does not indicate addition as normally understood, nor does the symbol “i” denote a number.
These are parts of the scheme used to express numbers of a new class and they signify the pair of real numbers
(x, y) to form a single complex number. Y
Imaginary axis P(x,y)
X’ X
O Real axis
Y’
Figure 6.1: Representation of a complex number on a plane
Swiss-born mathematician Jean Robert Argand, after a systematic study on complex numbers, represented every
complex number as a set of ordered pair (x, y) on a plane called complex plane.
All complex numbers lying on the real axis were called purely real and those lying on imaginary axis as purely
imaginary.
Hence, the complex number 0 + 0i is purely real as well as purely imaginary but it is not imaginary.
6 . 2 | Complex Number
Note
Purely real if y = 0
x0
Complex number if
y0
(a) The symbol i combines itself with real number as per the rule of algebra together with
PLANCESS CONCEPTS
Complex number as an ordered pair: A complex number may also be defined as an ordered pair of real
numbers and may be denoted by the symbol (a, b) . For a complex number to be uniquely specified, we
need two real numbers in a particular order.
Vaibhav Gupta (JEE 2009, AIR 54)
(g) In real number system if p2 + q2 = 0 implies, p= 0= q . But if z1 and z 2 are complex numbers then z12 + z 22 =
0
does not imply z=1 z=2 0 . For e.g. z1 = i and z 2 = 1 .
However if the product of two complex numbers is zero then at least one of them must be zero, same as in
case of real numbers.
x, if x ≥ 0
(h) In case x is real, then | x | = but in case of complex number z, | z | means the distance of the
point z from the origin. −x, if x < 0
M a them a tics | 6.3
PLANCESS CONCEPTS
z
• is always a uni-modular complex number if z ≠ 0 .
|z|
Vaibhav Krishnan (JEE 2009, AIR 22)
Sol: z= 5 + 12i
Let the square root of the given complex number be a + ib. Use algebra to simplify and get the value of a and b.
Let its square root = a + ib ⇒ 5 + 12i = a2 − b2 + 2abi
⇒ a2 − b2 =
5 … (i)
12
⇒ 2ab = … (ii)
⇒ (a2 + b2 ) 2 = (a2 − b2 ) 2 + 4a2b2 ⇒ (a2 + b2 ) 2 = 25 + 144 =169 ⇒ a2 + b2 =
13 … (iii)
2 2
(i) + (iii) ⇒ 2a =
18 ⇒ a =
9 ⇒ a =± 3
If a = 3 ⇒ b =
2 If a = −3 ⇒ b =−2
∴ Square root = 3 + 2i, −3 − 2i ∴ Combined form ± (3 + 2i)
⇒ x =( −1)(1 + 2i) ± 8 + 6i = −1 − 2i ± (3 + i) ; x = 2 – i, –4 – 3i
6 . 4 | Complex Number
Sol: Let x = 3+2i, and square it to form a quadratic equation. Then try to represent f(x) in terms of this quadratic.
x= 3 + 2i
⇒ (x − 3) 2 =−4 ⇒ x2 − 6x + 13 =
0
x 4 − 4x3 + 4x2 + 8x + 44
= x2 (x2 − 6x + 13) + 2x3 − 9x2 + 8x + 44
= x2 (x2 − 6x + 13) + 2(x3 − 6x2 + 13x) + 3(x2 − 6x + 13) + 5
⇒ f(x) ⇒ f(x) =
5
(i) Amplitude (Principal value of argument): The unique value of θ such that −π < θ ≤ π is called
principal value of argument. Unless otherwise stated, amp z refers to the principal value of argument.
(ii) Least positive argument: The value of θ such that 0 < θ ≤ 2π is called the least positive argument.
y
If φ =tan−1 .
x
M a them a tics | 6.5
PLANCESS CONCEPTS
y y
• If x > 0, y > 0 (i.e. z is in first quadrant), then arg z = θ = tan−1 .
x -
y
• If x < 0, y > 0 (i.e. z is in 2nd quadrant, then arg z = θ = π − tan−1
x
y x
• If x < 0, y < 0 (i.e. z is in 3rd quadrant), then arg z = θ = − π + tan−1 O
x -+ -
y
• If x > 0, y < 0 (i.e. z is in 4th quadrant), then arg z = θ = − tan−1
x Figure 6.5
0, if x > 0
• If y = 0 (i.e. z is on the X-axis), then arg (x + i0) =
π, if x < 0
π
, if y > 0
• If x = 0 (i.e. z is on the Y-axis), then arg (0 + iy) = 2
3π , if y < 0
2
Illustration 5: For what real values of x and y, are −3 + ix2 y and x2 + y + 4i complex conjugate to each other?
(JEE MAIN)
Sol: As −3 + ix2 y and x2 + y + 4i are complex conjugate of each other. Therefore −3 + ix2 y = x2 + y + 4i .
−3 + ix2 y = x2 + y − 4i
Equating real and imaginary parts of the above question, we get
– 3 = x2 + y ⇒ y =−3 − x2 … (i)
and x2 y = – 4 … (ii)
2
Putting the value of y = −3 − x from (i) in (ii), we get
−3 ± 9 + 16 −3 ± 5 2 −8
x2 ( −3 − x2 ) = – 4 0 ⇒ x2 =
⇒ x 4 + 3x2 − 4 = = = , =1, − 4
2 2 2 2
∴ x2 =1 ⇒ x =±1
Putting value of x = ± 1 in (i), we get y = – 3 – (1)2 = – 3 – 1 = – 4
Hence, x = ± 1 and y = – 4.
1+i 1−i
Illustration 6: Find the modulus of − . (JEE MAIN)
1−i 1+i
Sol: Simply substituting z = x + iy and by using formula | z | = x2 + y 2 we will get the result.
Let z = x + iy
| x + iy – 3 | = 3 | x + iy + 3 | | (x – 3) + iy | = 3 | (x + 3) + iy |
(x − 3) 2 + y 2 = 3 (x + 3) 2 + y 2 ; (x − 3) 2 + y 2 = 9(x + 3) 2 + 9y 2 .
β−α
Illustration 8: If α and β are different complex numbers with | β | = 1, then find . (JEE ADVANCED)
1 − αβ
β−α
Sol: By using modulus and conjugate property, we can find out the value of .
1 − αβ
We have, | β | = 1 ⇒ | β | 2 = 1 ⇒ ββ = 1
Now,
β−α
1 − αβ
=
β−α
ββ − αβ
=
β−α
β (β − α )
=
|β−α|
|β||β−α |
=
1
|β|
= 1. {as | x + iy | =| x + iy |}
Illustration 9: Find the number of non-zero integral solution of the equation | 1 − i | x =.
2x (JEE ADVANCED)
We have, | 1 − i | x =
2x
x x
( ) x
x
⇒ 12 + 12 =2 x ⇒ = 2x 2 2x
⇒ 22 = ⇒ 0 ⇒ x =.
= 0
2
∴ The number of non zero integral solution is zero.
a + ib a2 + b2
Illustration 10: If = p + iq. Prove that = p2 + q2 . (JEE MAIN)
c + id c + d2
2
a + ib
Sol: Simply by obtaining modulus of both side of = p + iq .
c + id
a + ib
We have, = p + iq
c + id
a + ib a2 + b2 a + ib a2 + b2
= ⇒ p + iq = p2 + q2 ; = | p + iq | ⇒ p2 + q2 .
=
c + id c2 + d2 c + id c2 + d2
x y
Illustration 11: If (x + iy) 1/3 = a + ib. Prove that + = 4 (a2 − b2 ) . (JEE ADVANCED)
a b
Sol: By using algebra of complex number. We have, (x + iy) 1/3 = a + ib
x + iy = (a + ib) 3 =a3 + i3b3 + 3a2ib + 3a(ib)2 = a3 − b3i + 3a2bi − 3ab2
x + iy = (a3 − 3ab2 ) + (3a2b − b3 )i ; x = a3 − 3ab2 = a (a2 − 3b2 ) ; y = 3a2b − b3
x y
+ = 4 (a2 − b2 ) .
a b
M a them a tics | 6.7
lm(z) > 0
as Argand Diagram.
lm(z) < 0
Generally this form is useful in solving equations and in problems
involving locus.
z | z | ( cos θ + i sin=
θ ) r cis θ
r
⇒
= r sin
Note: (a) (cis α ) (cis β=
) cis(α + β) X
O r cos
(b) (cis α ) (cis( −β=
)) cis(α − β)
1 Figure 6.7: Polar form
−1
(c) = (cis α ) = cis( −α )
(cis α )
PLANCESS CONCEPTS
The unique value of θ such that −π < θ ≤ π for which=x r cos θ &=
y r sin θ is r (cos θ + i sin θ)
known as the principal value of the argument.
The general value of argument is (2nπ + θ) , where n is an integer and θ is the
principal value of arg (z). While reducing a complex number to polar form, we r c i s θ
always take the principal value. Figure 6.8
The complex number z = r (cos θ + isin θ) can also be written as r cis θ .
Nitish Jhawar (JEE 2009, AIR 7)
6 . 8 | Complex Number
1 1
Illustration 12: Find locus represented by Re < . (JEE MAIN)
x + iy 2
Sol: Multiplying numerator and denominator by x − iy.
1 1 x − iy 1
We have, Re < Re <
x + iy 2 x2 + y 2 2
x 1
⇒ < ⇒ x2 + y 2 − 2x > 0
2
x +y 2 2
Locus is the exterior of the circle with centre (1, 0) and radius = 1.
6π 6π
Illustration 13: If z = 1 + cos + isin . Find r and amp z. (JEE MAIN)
5 5
Sol: By using trigonometric formula we can reduce given equation in the form of
= z r ( cosθ + i sinθ ) .
3π 3π 3π 3π 3π 3π
z = 2cos2 + 2isin cos = 2cos cos + isin
5 5 5 5 5 5
2π 2π 2π 2π 2π 2π 2π 2π
= −2cos − cos + isin = 2cos cos − isin Hence, | z | = 2cos ; amp z = −
5 5 5 5 5 5 5 5
M a them a tics | 6.9
1
Illustration 14: Show that the locus of the point P(ω)denoting the complex number z + on the complex plane is
a standard ellipse where | z | = a, where a ≠ 0, 1. z (JEE ADVANCED)
Sol: Here consider w = x + iy and z = α + iβ and then solve this by using algebra of complex number.
1
Let w = z + where z = α + iβ , α2 + β2 = a2 (as | z | = a)
z
1 α − iβ α β 1 1
x + iy = α + iβ + = α + iβ + = α + + i β − ∴ x =α 1 + ; y =β 1 −
α + iβ 2
α +β 2
2
a 2
a 2
a a2
x2 y2 x2 y2
∴ + = α2 + β2 = a2 ; ∴ + = 1.
2 2 2 2
1 1 1 1
1 + 2 1 − 2 a + a −
a a a a
(iv) z1 + z 2 = z1 + z2
(v) z1 − z 2 = z1 − z2
z1 z 2 = z1 · z2
(vi)
z1 z1
(vii) = ; z2 ≠ 0
z 2 z2
z1 | z1 |
(iv) = , z2 ≠ 0
z1 | z2 |
n n
(v) | z | = | z |
2
(vi) | z1 + z 2 | + | z1 − z=
2 |
2
2 | z1 | 2 + | z 2 | 2
(vii) | z1 | − | z 2 | ≤ | z1 + z 2 | ≤ | z1 | + | z 2 | [Triangle Inequality]
6 . 1 0 | Complex Number
z1
(ii) amp = amp z 1 − amp z 2 + 2kπ, k ∈ I
z2
(iii) amp
= (zn ) n amp(z) + 2kπ , where the value of k should be such that RHS lies in ( −π, π]
•• | Re(z) | + | Im(z) | ≤ 2 | z |
1 a + a2 + 4 −a + a2 + 4
•• If z + = a, the greatest and least values of |z| are respectively and .
z 2 2
•• | z1 + z12 − z 22 | + | z 2 − z12 − z 22 | = | z1 + z 2 | + | z1 − z 2 |
z z
Illustration 15: If z1= 3 + 5i and z 2= 2 − 3i , then verify that 1 = 1 (JEE MAIN)
z z
2 2
z 9 19 9 19
L.H.S. = 1=
z − 13 + 13 i =
− − i
13 13
2
z1 3 + 5i 3 − 5i (3 − 5i) (2 − 3i)
R.H.S. = = = = ×
z2 2 − 3i 2 + 3i (2 + 3i) (2 − 3i)
Illustration 16: If z be a non-zero complex number, then show that (z −1 ) = (z)−1 . (JEE MAIN)
Sol: By considering z = a + ib and using properties of conjugate we can prove given equation.
z −1=
1
=
1
=
1
×
a − ib
z a + ib a + ib a − ib
=
2
a
a +b 2
−
2
ib
a +b 2
z −1
⇒=
2
a
a +b 2
+ ( )ib
a + b2
2
… (i)
1 1 1 1 a + ib a b
and (z)−1 = = = = × = +i … (ii)
z a + ib a − ib a − ib a + ib 2
a +b 2
a + b2
2
(a + i) 2 (a2 + 1) 2
Illustration 17: If = p + iq, then show that p2 + q2 = . (JEE MAIN)
2a − i 4a2 + 1
(a + i)2 z1 z1 (a − i) 2
⇒ p − iq = = ⇒ p − iq = … (ii) using (z
= 2
) z= · z (z) 2
· z z=
(2a − i)
z 2 z2 (2a + i)
(a + i) 2 (a − i) 2
Multiplying (i) and (ii), we get (p + iq) (p – iq) =
2a − i 2a + i
(a2 − i2 ) 2 (a2 + 1) 2
⇒ p2 − i2q2 = ⇒ p2 + q2 = .
4a2 − i2 4a2 + 1
Illustration 18: Let z1 , z 2 , z3 , ……. zn are the complex numbers such that | z1 | = | z 2 | = ……. = | zn | = 1 . If z =
n n 1
∑ zk ∑ then prove that
= k 1= k 1 zk
(i) z is a real number (ii) 0 < z ≤ n2 (JEE ADVANCED)
1 n n 1
Sol: Here | z1 | = | z 2 | = ……. = | zn | = 1, therefore zz = 1 ⇒ z = . Hence by substituting this to z = ∑ zk ∑ ,
z z
we can solve above problem. = k 1= k 1 k
1 1 1
Now, z = (z1 + z 2 + z3 + ……. + zn ) + + ……. +
z1 z 2 zn
≤ ( | z1 | + | z 2 | + | z3 | +…….+ | zn | ) = n2 ∴
2
0 < z ≤ n2 .
6 . 1 2 | Complex Number
Illustration 19: Let x1, x2 are the roots of the quadratic equation x2 + ax + b = 0 where a, b are complex numbers and
y1, y2 are the roots of the quadratic equation y 2 + | a | y + | b | =
0 . If | =
x1 | | =
x2 | 1 , then prove that | = y2 | 1 .
y1 | | =
(JEE ADVANCED)
Sol: Solve by using modulus properties of complex number.
∴ | a | ≤ | x1 | + | x2 | or |a | ≤ 2
y1
Now consider y 2 + | a | y + | b | =
0 , where y1 and y 2 are complex numbers
y2
− | a | ± 4 − | a |2 i
− | a | ± | a |2 − 4 | b | | a | 2 + 4− | a | 2
y1, 2 = = ∴ | y1, 2 | = =1
2 2 2
Hence, | = y2 | 1 .
y1 | | =
z1 + z 2 + z3
(a) Centroid: The centroid ‘G’ is given by . Refer to Fig 6.10.
3
A(z1)
B(z2) D C(z3)
a z1 + b z 2 + c z3
(b) Incentre: The incentre ‘I’ is given by . Refer to Fig 6.11.
a+b+c
A(z1)
B(z2) C(z3)
n
E
H
B(z2) p D q C(z3)
p z3 + q z 2
Proof: From section formula, we have zD =
a
In ΔABD and ΔACD, p = c cos B and q = b cos C. Refer to Fig 6.12.
bcosC z 2 + c cosB z3
Therefore, zD =
a
Now, AE = c cos A;= = AEcosec C = c cos A cosec C
n AH
⇒n=2R cos A [Using Sine Rule]
and m = c cos B cot C or, m = 2R cos B cos C [Using Sine Rule]
mz1 + n ZD
Hence, zH = .
m+n
bcosC z 2 + c cosB z3
2R cosBcosC z1 + 2R cos A
a
=
2R (cos A + cosBcosC)
(d) Circumcentre:
Let R be the circumradius and the complex number z0 represent the circumcentre of the triangle as shown in Fig 6.11.
∴ | z1 − z 0 | = | z 2 − z 0 | = | z3 − z 0 |
Consider, | z1 − z 0 | 2 =| z 2 − z 0 | 2
z1 (z1 − z 0 ) − z2 (z 2 − z=
0) z 0 (z1 − z 0 ) − (z 2 − z 0 )
) z 0 (z1 − z 2 )
z1 (z1 − z 0 ) − z2 (z 2 − z 0= … (i)
PLANCESS CONCEPTS
1
• The area of the triangle whose vertices are z, iz and z + iz is | z |2 .
2
3
• The area of the triangle with vertices z, ω z and z + ω z is | z |2 .
4
• If z1 , z 2 , z3 be the vertices of an equilateral triangle and z 0 be the circumcentre, then z12 + z 22 + z32 =
3z 02 .
• If z1 , z 2 , z3 ,…….zn be the vertices of a regular polygon of n sides and z 0 be its centroid, then
z12 + z 22 + ……. + zn2 = nz 02 .
• If z1, z2 , z3 be the vertices of a triangle, then the triangle is equilateral if (z1 ‒ z2)2 + (z2 ‒z3)2 + (z3 ‒ z1)2= 0
1 1 1
or z12 + z 22 + z32= z1 z 2 + z 2 z3 + z3 z1 or + + = 0.
z1 − z 2 z 2 − z3 z3 − z1
• If z1, z2 , z3 are the vertices of an isosceles triangle, right angled at z2 then z12 + 2z 22 + =
z32 2z 2 (z1 + z 3 ) .
2
• If z1, z2 , z3 are the vertices of a right-angled isosceles triangle, then (z1 − z 2 ) = 2 (z1 − z3 ) (z3 − z 2 ) .
• If z1, z2 , z3 be the affixes of the vertices A, B, C respectively of a triangle ABC, then its orthocentere
a (sec A) z1 + b(secB) z 2 + c(sec C) z3
is .
asec A + bsecB + c sec C
Shivam Agarwal (JEE 2009, AIR 27)
M a them a ti cs | 6.15
z3
Illustration 20: If z1, z2 , z3 are the vertices of an isosceles triangle right angled
at z 2 then prove that z12 + 2z 22 + =
z32 2z 2 (z1 + z 3 ) (JEE MAIN)
iπ
Sol: Here (z1 − z 2 ) = (z3 − z 2 )e 2 . Hence by squaring both side we will get the result.
z1 z2
⇒ (z1 − z 2 )2 = i2 (z3 − z 2 )2
Figure 6.14
⇒ z32 + z 22 − 2z3 z 2 =−z12 − z 22 + 2z1 z 2 ⇒ z12 + 2z 22 + =
z32 2z 2 (z1 + z 3 ) .
Illustration 21: A, B, C are the points representing the complex numbers z1, z2 , z3 respectively and the circumcentre of
the triangle ABC lies at the origin. If the altitudes of the triangle through the opposite vertices meets the circumcircle at
D, E, F respectively. Find the complex numbers corresponding to the points D, E, F in terms of z1, z2 , z3. (JEE MAIN)
A(z1)
Sol: Here the ∠BOD = π − 2B , hence OD = OB ei( π−2B) .
(/2)-B E()
i( π−2B)
From Fig 6.13, we have OD = OB e ; 2B
O
i ( π−2B) −i2B
α = z2 e = −z2 e … (i) B(z2) C(z3)
-2B
also, z1 = z3 ei 2B … (ii) D()
Figure 6.15
− z 2 z3
∴ αz1 =−z 2 z3 ⇒α=
z1
−z3 z1 −z z
Similarly, β = and γ = 1 2 .
z2 z3
Illustration 22: If zr (r = 1, 2, …,6) are the vertices of a regular hexagon then E(z5) D(z4)
6
prove that ∑ zr2 = 6z02 , where z 0 is the circumcentre of the regular hexagon.
r =1
(JEE MAIN)
F(z6) z0 C(z3)
Illustration 23: If z1, z2 , z3 are the vertices of an equilateral triangle then prove that z12 + z 22 + z32= z1 z 2 + z 2 z3 + z3 z1
and if z0 is its circumcentre then 3z 20 = z12 + z 22 + z32 . (JEE ADVANCED)
⇒ ∑ z12 + 2∑ z1 z 2 =
9z 02 ∴ 3∑ z12 =
9z 02 .
Illustration 24: Prove that the triangle whose vertices are the points z1, z2 , z3 on the Argand plane is an equilateral
1 1 1
triangle if and only if + + = 0. (JEE ADVANCED)
z 2 − z3 z3 − z1 z1 − z 2
Sol: Consider ABC is the equilateral triangle with vertices z1 , z 2 and z3 respectively.
A(z1)
Therefore | z 2 − z3 | = | z3 − z1 | = | z1 − z 2 | .
Let ABC be a triangle such that the vertices A, B and C are z1, z2 and z3 respectively. 60
⇒ | α |2 = | β | | γ | ⇒ | α |3 = | α | | β | | γ |
Similarly, ⇒ | β |3 = | α | | β | | γ | and | γ |3 =| α | | β | | γ |
∴ |α|=|β|=| γ |
⇒ | z 2 − z3 | = | z3 − z1 | = | z1 − z 2 | ⇒ BC = CA = AB
Hence, the given triangle is an equilateral triangle.
M a them a ti cs | 6.17
1 1 1
Illustration 25: Prove that the roots of the equation + + 0 (where z , z , z are pair wise distinct
=
z − z1 z − z 2 z − z3 1 2 3
complex numbers) correspond to points on a complex plane, which lie inside a triangle with vertices z1, z2 , z3 excluding
its boundaries. (JEE ADVANCED)
z − z1 z − z2 z − z3
Sol: By using modulus and conjugate properties we can reduce given expression as + +
| z − z1 | 2 | z − z2 | 2 | z − z3 | 2
1
= 0. Therefore by putting | z − zi | = , where i = 1, 2 and 3, we will get the result.
2
ti A(z1)
1
t1 (z − z1 ) + t2 (z − z2 ) + t3 (z − z3 ) = 0 where | z − z1 | 2 =etc and t1 , t2 , t3 ∈ R +
t1 t2
t1 (z − z1 ) + t2 (z − z 2 ) + t3 (z − z3 ) = 0 z’ t3
t z +t z +t z t1 z t1 + t2
(t1 + t2 + t3 ) z= t1 z1 + t2 z 2 + t3 z3 ⇒ z =1 1 2 2 3 3
t1 + t2 + t3
B (z2) C (z3)
t1 z1 + t2 z 2 t1 + t2 t3 z 3 t1 + t2 t3 z 3
⇒z · + = z' + Figure 6.19
t1 + t2 t1 + t2 + t3 t1 + t2 + t3 t1 + t2 + t3 t1 + t2 + t3
(t + t )z'+ t3 z3
⇒ z =1 2 ⇒ z lies inside the ∆z1 z 2 z3
t1 + t2 + t3
If t=
1 t=
2 t3 ⇒ z is the centroid of the triangle.
Also, it implies | z − z1 | =| z − z 2 | =| z − z3 | ⇒ z is the circumcentre .
Illustration 26: Let z1 and z 2 be roots of the equation z 2 + pz + q = 0, where the coefficients p and q may be
complex numbers. Let A and B represent z1 and z 2 in the complex plane. If ∠AOB = α ≠ 0 and OA = OB, where O
α
is the origin, prove that p2 = 4qcos2 . (JEE ADVANCED)
2
Sol: Here OB = OAeiα . Therefore by using formula of sum and product of roots of quadratic equation we can prove
this problem.
Y
Since z1 and z 2 are roots of the equation z 2 + pz + q = 0
−p and z z = q B(z2)
z +z =
1 2 1 2 (1)
A(z1)
Since OA = OB. So OB is obtained by rotating OA in anticlockwise direction through X’ X
angle α. O
z z2
∴ ∴ OB = OAeiα ⇒ z2 = z1eiα ⇒ 2 = eiα ⇒ = cos α + isin α
Y’
z1 z1 Figure 6.20
iα
z2 z 2 + z1 α α α α
⇒ + 1 = 1 + cos α + isin α ⇒ = 2cos cos + isin
= 2cos e 2
z1 z1 2 2 2 2
iα 2
z 2 + z1 α z + z1 α
⇒ 2cos e 2 ⇒ 2
= 4 cos2 eiα
=
z1 2 z1 2
2
z + z1 α z2 α
⇒ ( z 2 + z1 ) =
2
⇒ 2 = 4 cos2 4z1 z 2 cos2
z1 2 z1 2
α α
⇒ ( − p) 2 = 4qcos2 ⇒ p 2 = 4qcos2 .
2 2
6 . 1 8 | Complex Number
Illustration 27: On the Argand plane z1, z2 and z3 are respectively the vertices of an isosceles triangle ABC with
AC = BC and equal angles are θ. If z4 is the incentre of the triangle then prove that (z2 ‒ z1) (z3 ‒ z1) = (1 + sec θ)
(z4 ‒ z1)2 (JEE ADVANCED)
Sol: Here by using angle rotation formula we can solve this problem. From Fig 6.21, we have
z 2 − z1 z 4 − z1
= eiθ /2 … (i) (clockwise) C(z3)
| z 2 − z1 | | z 4 − z1 |
z3 − z1 z 4 − z1
and = eiθ /2 … (ii) (anticlockwise)
| z3 − z1 | | z 4 − z1 |
I(z4)
Multiplying (i) and (ii) /2
/2
(z 2 − z1 ) (z3 − z1 ) | (z 2 − z1 ) | | (z3 − z1 ) | AB| AC | 2(AD) (AC) 2(AD)2 AC A(z1) D B(z2)
= = = = ·
(z 4 − z1 ) 2 | z 4 − z1 | 2 (AI) 2
(AI) 2
(AI) 2 AD
Figure 6.21
θ
= 2cos2 sec θ = (1+ cos θ)sec θ .
2
(1,2)
Radius = 3
X’ X
Y’
Figure 6.22: Circle on a complex plane
(b) | z – 1 | = | z – i | denotes the equation of the perpendicular bisector of join of (1, 0) and (0, 1) on the Argand
plane (see Fig 6.24).
Y
B(0,1)
O Locus of z
X’ X
A(0,1)
Y’
(c) | z – 4i | + | z + 4i | = 10 denotes an ellipse with foci at (0, 4) and (0, – 4); major axis 10; minor axis 6 with
4
e= (see Fig 6.24). Y
5
(0,4)
X’ X
(0,-4)
Y’
Figure 6.24: Ellipse on a complex plane
M a them a ti cs | 6.19
2 2
2 36 64 4 x + y =1
e = 1− = ⇒ e= 9 25
100 100 5
(1,0)
X’ X
Y’
π
(g) 0 ≤ arg z ≤ (z ≠ 0) where z is defined by positive real axis and the part of the line x = y in the first quadrant.
4
It includes the boundary but not the origin. Refer to Fig 6.26.
y=x
X’ X
O
Y’
Figure 6.26
(h) Re (z 2 ) > 0 denotes the area between the lines x = y and x = – y which includes the x-axis.
0 ⇒ x2 − y 2 > 0
Hint: (x2 − y 2 ) + 2xyi = ⇒ (x – y) (x + y) > 0.
Let z = x + iy ⇒ (x + iy) 2 + x2 + y 2 =
0 ⇒ x2 − y 2 + x2 + y 2 + (2ixy) =
0
⇒ Either x =0 or y =0 ; x = 0 ⇒ − y 2 + | y | =
0 ⇒ y= 0, 1, − 1 ∴ z= 0, i, − i
2
0
and, y = 0 ⇒ x + | x | = 0
⇒ x= ∴z =0
Therefore, z = 0, z = i, z = – i.
Illustration 29: If the complex number z is to satisfy |z| = 3, z − {a(1 + i) − i} ≤ 3 and | z + 2a – (a + 1) i | > 3
simultaneously for at least one z then find all a ∈ R . (JEE ADVANCED)
All z at a time lie on a circle | z | = 3 but inside and outside the circles | z – {a (1 + i) – i} | = 3 and | z + 2a – (a + 1)
i | = 3, respectively.
Let z = x + iy then equation of circles are x2 + y 2 =
9 … (i)
2 2
(x − a) + (y − a + 1) =9 … (ii)
and (x + 2a)2 + (y − a − 1)2 =9 … (iii)
Circles (i) and (ii) should cut or touch then distance between their centres ≤ sum of their radii.
+ +
⇒ (a − 0) 2 + (a − 1 − 0)2 ≤ 3 + 3 ⇒ a2 + (a − 1)2 ≤ 36
-
1-71 1+71
35
⇒ 2a2 − 2a − 35 ≤ 0 ⇒ a2 − a − ≤0 2 2
2
2 Figure 6.27
1 71 1 − 71 1 + 71
⇒ a − ≤ ∴ ≤a≤ … (iv)
2 4 2 2
Again circles (i) and (iii) should not cut or touch then distance between their centres > sum of the radii
2a
⇒ 5a 2 + 2a − 35 > 0 ⇒ a2 + −7 > 0 +
5 +
-
-1-411 -1+411
−1 − 4 11 −1 + 4 11
Then a − a − >0 5 5
5 5
Figure 6.28
−1 − 4 11 −1 + 4 11
∴ a ∈ −∞ , ∪ , ∞ … (v)
5 5
The common values of a satisfying (iv) and v are
1 − 71 −1 − 4 11 −1 + 4 11 1 + 71
a∈ , ∪ ,
2 5 5 2
8. DEMOIVRE’S THEOREM
Statement: (cos nθ + isin nθ) is the value or one of the values of (cos θ + isin θ) n , ∀ n ∈ Q . Value if n is an integer.
One of the values if n is rational which is not integer, the theorem is very useful in determining the roots of any
complex quantity.
Note: We use the theory of equations to find the continued product of the roots of a complex number.
PLANCESS CONCEPTS
The theorem is not directly applicable to (sin θ + icos θ) n , rather
n
n π π π π
(sin θ + icos θ) = cos − θ + isin − θ = cos n − θ + isin n − θ
2 2 2 2
M a them a ti cs | 6.21
8.1 Application
Cube root of unity
−1 + i 3 −1 − i 3
(a) The cube roots of unity are 1, ,
2 2
[Note that 1 – i 3 = – 2 and 1 + i 3 = –2 ω2 ]
(ii) x2 + x + 1= (x − ω) (x − ω2 )
nth roots of unity: If 1, α1 , α2 , α3 , ……., αn−1 are the n, nth roots of unity then
2π
i
2π 2π
(i) They are in G.P. with common ratio e n
= cos + isin
n n
(ii) 1p + α1p + αp2 + ……. + αnp−1 = 0 if p is not an integral multiple of n
1 1 1
2mπ 2mπ
Explanation: Let z = 1n = (cos 0 + isin 0)n = ( cos 2mπ + isin 2mπ ) n = cos + isin
n n
Put m = 0, 1, 2, 3, ……. (n – 1), we get
1 − ( αp ) n 1 − ( αn ) p
Now, S = 1p + αp + α2p + α3p + ……. + α(n−1)p = =
1 − αp 1 − αp
0
n p = 0, if p is not an integral multiple of n
1 − (α ) non zero
= =
1 − αp 0
= indeterminant, if p is an integral multiple of n
0
Again, if x is one of the nth root of unity then xn – 1 = (x – 1) (x − α1 )(x − α2 ) ……. (x − αn−1 )
xn − 1
1 + x + x2 + …… + xn−1 = ≡ (x − α1 )(x − α2 ) ……. (x − αn−1 )
x −1
Put x = 1, to get (1 − α1 ) (1 − α2 ) ……. (1 − α n−1 ) =n
PLANCESS CONCEPTS
| z | +a | z | −a
Square roots of z = a + ib are ± +i for b > 0.
2 2
Illustration 30: If x= a + b , y = aω + bω2 and z = aω2 + bω , then prove that x3 + y 3 + z3= 3 (a3 + b3 ) (JEE MAIN)
Sol: Here x + y + z =.
0 Take cube on both side.
x + y + z =
0 ⇒ x3 + y 3 + z 3 =
3xyz ∴ LHS = 3xyz
= 3(a + b)(aω + bω2 )(aω2 + bω) = 3(a + b)(aω + bω2 )(aω2 + bω.ω3 ) = 3ω3 (a + b)(a + bω)(a + b=
ω2 ) 3 (a3 + b3 )
Illustration 31: The value of expression 1(2 − ω)(2 − ω2 ) + 2 (3 − ω) (3 − ω2 ) + ... + (n − 1)(n − ω)(n − ω2 ) .
(JEE ADVANCED)
nθ
sin
(a) cos θ + cos2θ + cos3θ + …… + cosnθ = 2 cos n + 1 θ
θ
sin 2
2
nθ
sin
(b) sin θ + sin2θ + sin3θ + …… + sinnθ = 2 sin n + 1 θ
θ
sin 2
2
2π
Note: If θ = , then the sum of the above series vanishes.
n
Illustration 32: If 1, ω, ω2 , ……., ωn−1 are nth roots of unity, then the value of (5 − ω) (5 − ω2 ) ……. (5 − ωn−1 ) is equal
to (JEE MAIN)
1
Sol: Here consider x = (1)n , therefore xn − 1 =0 (has n roots i.e. 1, ω, ω2 , ……., ωn−1 ).
xn − 1
⇒ xn − 1= (x − 1)(x − ω) (x − ω2 ) …….(x − ωn−1 ) ⇒ = (x − ω) (x − ω2 ) …….(x − ωn−1 )
x −1
5n − 1
⇒ Putting x = 5 in both sides, we get ∴ (5 − ω) (5 − ω2 ) ……. (5 − ωn−1 ) = .
4
A(z1)
B(z2)
X’ X
O
Y’
Figure 6.29
6 . 2 4 | Complex Number
The point P(z) which divides the join of A(z1) and B(z2) in the ratio m: n is m P(z)
mz 2 + nz1
given by z = . Refer Fig 6.30. A(z1)
m+n
Figure 6.30
1 1
A(z1) M(z) B(z2)
(c) Square:
B(z2) C(z3)
(i) The diagonals AC and BD bisect each other
Figure 6.33
⇔ z1 + z 3 = z 2 + z 4
(ii) A pair of adjacent sides are equal; for instance, AD = AB A(z1) D(z4)
⇔ | z 4 − z 1 | = | z2 − z 1 |
(iii) The two diagonals are equal, that is AC = BD
⇔ | z3 − z 1 | = | z 4 − z 2 |
B(z2) C(z3)
Figure 6.34
(d) Rectangle:
(i) The diagonals AC and BD bisect each other A(z1) D(z4)
⇔ z1 + z 3 = z 2 + z 4
10.5 Triangle
In a triangle ABC, let the vertices A, B and C be represented by the complex numbers z1, z2, and z3 respectively. Then
(a) Centroid: The centroid (G), is the point of intersection of medians of ∆ABC . It is given by the formula
1
z= (z + z 2 + z3 )
3 1
A(z1)
B(z2) C(z3)
(b) Incentre: The incentre (I) of ∆ABC is the point of intersection of internal angular bisectors of angles of
∆ABC . It is given by the formula
az1 + bz 2 + cz3
z= ,
a+b+c A(z1)
B(z2) C(z3)
(c) Circumcentre: The circumcentre (S) of ∆ABC is the point of intersection of perpendicular bisectors of sides
of ∆ABC . It is given by the formula
| z1 | 2 z1 1 A(z1)
| z2 | 2 z2 1
2
2 2 2
| z1 | (z 2 − z3 )+ | z 2 | (z3 − z1 )+ | z3 | (z1 − z 2 ) | z3 | z3 1
z= =
z1 (z 2 − z3 ) + z2 (z3 − z1 ) + z3 (z1 − z 2 ) z1 z1 1 S
z2 z2 1
B(z2)
z3 z3 1 C(z3)
H 2 G 1 S
(Orthocentre) (Centroid) (Circumcentre)
Figure 6.37
(d) Euler’s Line: The orthocenter H, the centroid G and the circumcentre S of a triangle which is not equilateral
lies on a straight line. In case of an equilateral triangle these points coincide.
G divides the join of H and S in the ratio 2 : 1 (see Fig 6.37).
1
Thus,=
zG (z + 2zS )
3 H
6 . 2 6 | Complex Number
Figure 6.38
10.7 Conditions for Triangle to be Equilateral
The triangle ABC with vertices A(z1 ), B(z 2 ) and C(z3 ) is equilateral
1 1 1
iff + + =0
z 2 − z3 z3 − z1 z1 − z 2
A(z1)
⇔ z12 + z 22 + z32= z 2 z3 + z3 z1 + z1 z 2 ⇔ z1 z2 = z 2 z3 = z3=
z1 ⇔ z12 z 2 z3 =
and z 22 z1 z3
60
1 z2 z3
z − z1 z3 − z 2
⇔ 1 z3 0 ⇔ 2
z1 = =
z3 − z 2 z1 − z 2
1 z1 z2
60 60
1 1 1 1
⇔ + + = 0 where z= (z1 + z 2 + z3 ) . B(z2) C(z3)
z − z1 z − z 2 z − z3 3 Figure 6.39
(a) Non-parametric form: An equation of a straight line joining the two points A(z1 ) and B(z 2 ) is
z z 1 C(z3)
z − z1
Arg = 0 z1 z1 1 = 0
z 2 − z1 z2 z2 1
B(z2)
z − z1 z − z1
or =
z 2 − z1 z2 − z1 A(z1)
or z (z1 − z2 ) − z(z1 − z 2 ) + z1 z2 − z 2 z1 =
0 Figure 6.40
(b) Parametric form: An equation of the line segment between the points A(z1 ) and B(z 2 ) is
z = tz1 + (1 − t)z 2 , t ( 0,1 ) where t is a real parameter.
(c) General equation of a straight line: The general equation of a straight line is az + az + b =0 where, a is
non-zero complex number and b is a real number.
Where a is a complex number and b is a real number such that aa − b ≥ 0 . Centre of (i) is – a and its radius is
aa − b
(c) Diameter form of a circle: An equation of the circle one of whose diameter is the segment joining A(z1) and
B(z2) is (z − z1 ) (z − z2 ) + (z − z1 )(z − z 2 ) =
0
P(z)
(d) An equation of the circle passing through two points A(z1) and B(z2)
B(z2)
Centre
z z 1
A(z1)
is (z − z1 ) (z − z2 ) + (z − z1 )(z − z 2 ) + i k z1 0 where k is a real parameter.
z1 1 =
z2 z2 1 Figure 6.43
z − z2 z − z2 z3 − z 2 z − z1
⇒ arg 3 − arg = 0 or, arg + arg =
π
z − z
3 1 z − z1 z3 − z1 z − z2
z − z 2 z − z1
⇒ arg 3 0
= P(z)
z3 − z1 z − z 2 C(z3)
z − z 2 z − z1
or, arg 3 = π
z3 − z1 z − z 2
B(z2)
A(z1) -
P(z)
(z − z1 ) (z3 − z 2 )
In any case, we get is purely real. Figure 6.44
(z − z 2 )(z3 − z1 )
PLANCESS CONCEPTS
z1 z1 1
Three points z 1 , z 2 and z3 are collinear if z 2 z2 1 = 0.
z3 z3 1
If three points A(z1 ), B(z 2 ) and C(z3 ) are collinear then slope of AB = slope of BC = slope of AC
z1 − z 2 z 2 − z3 z1 − z3
⇒ = =
z1 − z2 z2 − z3 z1 − z3
2z + 1
Illustration 33: If the imaginary part of is – 4, then the locus of the point representing z in the complex
plane is iz + 1
(a) A straight line (b) A parabola (c) A circle (d) An ellipse (JEE MAIN)
2z + 1 2y (1 − y) − x(2x + 1)
As Im = – 4, we get =–4
iz + 1 x2 + (1 − y)2
⇒ 2x2 + 2y 2 + x − 2y= 4x2 + 4(y 2 − 2y + 1) ⇒ 2x2 + 2y 2 − x − 6y + 4 =0 . It represents a circle.
Illustration 34: The roots of z5= (z − 1) 5 are represented in the argand plane by the points that are
(a) Collinear (b) Concyclic
(c) Vertices of a parallelogram (d) None of these (JEE MAIN)
z1
Sol: Here consider z = and z1 and z 2 are represented by A and B respectively and O be the origin.
z2
z1 1
Let z = , then z + =1 ⇒ z2 − z + 1 =0
z2 z
1 ± 3i z 1 ± 3i
⇒ z= ⇒ 1 =
2 z2 2
M a them a ti cs | 6.29
ΟA |z | 1 ± 3i 1 3
= 1 = = + = 1 ⇒ OA =
OB
OB | z2 | 2 4 4
AB | z − z1 | z 1 3 1 3 1 3
Also, = 2 = 1− 1 = 1− ± i = i = + =1
| z2 | z2 2 2 4 4
OB 2 2
OB
⇒ AB = Thus, OA = OB = AB ∴ ∆OAB is an equilateral triangle.
Illustration 36: If z1 , z 2 , z3 are the vertices of an isosceles triangle, right angled at the vertex z 2 , then the value
of (z1 − z 2 ) 2 + (z 2 − z3 ) 2 is
(a) -1 (b) 0 (c) (z1 − z3 ) 2 (d) None of these (JEE ADVANCED)
Sol: Here use distance and argument formula of complex number to solve this problem.
As ABC is an isosceles right angled triangle with right angle at B, C(z3)
z −z π
BA = BC and ∠ABC = 90° ⇒ | z1 − z 2 | = | z3 − z 2 | and arg 3 2 =
z1 − z 2 2
z3 − z 2 | z3 − z 2 | π π
⇒= cos + isin = i
z1 − z 2 | z1 − z 2 | 2
2 B(z2) A(z1)
Figure 6.45
⇒ (z3 − z 2 )2 =
−(z1 − z 2 )2 ⇒ (z1 − z 2 )2 + (z 2 − z3 )2 =
0.
Corollary: If A(z1 ) and B(z 2 ) are two complex number such that
| z2 |
θ , then z 2 =
∠AOB = z1eiθ (see Fig 6.47). Y
| z1 |
B(z2)
Let z1 = r1eiα and z 2 = r2eiβ where = , | z 2 | r2 .
| z1 | r1=
r2
iβ A(z1)
z 2 r2e r r1
Then= = 2 ei(β−α )
z1 r eiα r1 X
1 O
z2 r2 | z2 |
Thus, = eiθ (∵ β − α = θ ) ⇒ z2 = z eiθ Figure 6.47
z1 r1 | z1 | 1
6 . 3 0 | Complex Number
PLANCESS CONCEPTS
π π π π
Let z = r (cos θ + isin θ) and i = cos + isin , then iz = r cos + θ + isin + θ .
2 2 2 2
Hence, iz can be obtained by rotating the vector z by right angle in the positive sense. And so on, to
multiply a vector by – 1 is to turn it through two right angles.
Thus, multiplying a vector by (cos θ + isin θ) is to turn it through the angle θ in the positive sense.
Anvit Tawar (JEE 2009, AIR 9)
Illustration 37: Suppose A( z1 ), B( z 2 ) and C( z3 ) are the vertices of an equilateral triangle inscribed in the circle
| z | = 2. If z1= 1 + 3 i , then z 2 and z3 are respectively.
Sol: As we know x + iy = reiθ . Hence by using this formula we can obtain z 2 and z3 .
iπ
z1= 1 + 3 i = 2e 3
2 πi 2 πi Y
2π 2π
Since, ∠AOC =and ∠BOC = , z 2 = z1e 3 and z3 = z 2e 3 A(z1)
3 3
5 πi
B(z2) /3
⇒ z3 = 2eπi = 2(cos π + isin π) = −2 and z3 = 2e 3
O
X
π π
= 2 cos 2π − + isin 2π − C(z3)
3 3 Figure 6.48
π π 1 3
= 2 cos − isin = 2 − i = 1 − 3 i .
3 3 2 2
PROBLEM-SOLVING TACTICS
(c) For questions related to locus and for equations, use the algebraic form of the complex number.
(d) Polar form of a complex number is particularly useful in multiplication and division of complex numbers. It
directly gives the modulus and the argument of the complex number.
(f) Multiplying by cos θ corresponds to rotation by angle θ about O in the positive sense.
M a them a ti cs | 6.31
a + ib
(g) To put the complex number in the form A + iB we should multiply the numerator and the denominator
c + id
by the conjugate of the denominator.
(h) Care should be taken while calculating the argument of a complex number. If z = a + ib, then arg(z) is not
b
always equal to tan−1 . To find the argument of a complex number, first determine the quadrant in which
a
it lies, and then proceed to find the angle it makes with the positive x-axis.
b π −3π
For example, if z = – 1 – i, the formula tan−1 gives the argument as , while the actual argument is .
a
4 4
FORMULAE SHEET
−1 y
tan x > 0, y > 0
x
y
π − tan−1 x < 0, y > 0
x
(d) Argument of z, i.e. θ =
−π + tan−1 y
x < 0, y < 0
x
−1 y
− tan x
x > 0, y < 0
y -1 y
where, =tan x
=- =
x
=-+ O =-
0, if x > 0
(e) If y=0, then argument of z, i.e. θ =
π, if x < 0
π
, if y > 0
(f) If x=0, then argument of z, i.e. θ = 2
3 π , if y < 0
2
(v) z1 + z 2 = z1 + z2
(vi) z1 − z 2 = z1 − z2
(vii) z1 z 2 = z1 z2
z1 z1
(viii)
= z z if z 2 ≠ 0
2 2
(i) | z | = 0 ⇔ z =
0
(ii) | z | = | z | =| −z | =| − z |
(iii) − | z | ≤ Re(z) ≤ | z |
(iv) − | z | ≤ Im(z) ≤ | z |
(v) zz = | z | 2
If z1 , z 2 are two complex numbers, then
(i) | z1 z 2 | = | z1 | | z 2 |
(ii) z1 | z1 | , if z ≠ 0
= 2
z2 | z2 |
2 2 2 2 2
(iii) | z1 + z 2 | = | z1 | + | z 2 | + z1 z 2 + z1 z2 = | z1 | + | z 2 | +2Re (z1 z2 )
2 2 2
(iv) | z1 − z 2 | = | z1 | + | z 2 | − z1 z 2 − z1 z2 = | z1 | 2 + | z 2 | 2 −2Re (z1 z2 )
In fact arg(z1=
z 2 ) arg(z1 ) + arg(z 2 ) + 2kπ
0, if − π < arg(z1 ) + arg(z 2 ) ≤ π
where,k= 1, if − 2π < arg(z1 ) + arg(z 2 ) ≤ −π
−1, if π < arg(z1 ) + arg(z 2 ) ≤ 2π
(iii) arg(z
= 1 z2 ) arg(z1 ) − arg(z 2 )
z1
(iv) arg=
arg(z1 ) − arg(z 2 )
z2
M a them a ti cs | 6.33
π
(v) | z1 + z 2 | = | z1 − z 2 | ⇔ arg(z1 ) − arg(z 2 ) =
2
(vi) | z1 + z 2 | = | z1 | + | z 2 | ⇔ arg(z1 ) =
arg(z 2 )
If=
z1 r1 (cos θ1 + isin θ1 ) and=
z 2 r2 (cos θ2 + isin θ2 ) , then
z1 + z 2 + z3
(i) Centroid (G), zG =
3
a z1 + b z 2 + c z3
(ii) Incentre (I), zI =
a+b+c
z1 tanA + z 2 tanB + z3 tanC
(iii) Orthocentre (H), zH
∑ tanA
z1 (sin2A) + z 2 (sin2B) + z3 (sin2C)
(iv) Circumcentre (S), zS
sin2A + sin2B + sin2C
( cos θ + i sin θ=
)
n
(n) cosnθ + i sin nθ
z +x z −x
(o) z =x + i y =± +i for y > 0
2 2
(p) Distance between A (z1 ) and B(z 2 ) is given by | z 2 − z1 |
(q) Section formula: The point P (z) which divides the join of the segment AB in the ratio m : n
mz 2 + nz1
is given by z = .
m+n
1
(r) Midpoint formula: z = (z + z 2 ) .
2 1
(s) Equation of a straight line
(i) Non-parametric form: z (z1 − z2 ) − z(z1 − z 2 ) + z1 z2 − z 2 z1 =
0
(ii) Parametric form: z = tz1 + (1 − t)z 2
(iii) General equation of straight line: az + az + b =0
z1 − z 2
(t) Complex slope of a line, µ = . Two lines with complex slopes µ1 and µ2 are
z1 − z2
(i) Parallel, if µ1 =µ2
(ii) Perpendicular, if µ1 + µ2 = 0
(u) Equation of a circle: | z − z 0 | =
r
6 . 3 4 | Complex Number
Solved Examples
2 + 6i 1 − i 2 + 6i − 2i + 6
= × = = 4 + 2i Sol: Consider z = x + iy and then by taking modulus we
1+i 1−i 2 will get the result.
z z
2.
∴ Im 1 2 = Let z = x + iy
z1
z −3 x − 3 + iy
∴ 2 ⇒
= 2
=
Example 2: Find the square root of z = – 7 – 24i. z+3 x + 3 + iy
∴ x2 + y 2 =
25 … (iii) Sol: | z j | = 1 ⇒ z j z j= 1 ∀ j= 1, …… , n
∴ z 0 =±(3 − 4i) . 1 1 1
| z1 + z 2 + ……. + z=
n | + + ……. + =
z1 z2 zn
Example 3: If n is a positive integer and ω be an
imaginary cube root of unity, prove that 1 1 1 1
+ + + ……. +
z1 z 2 z3 zn
3, when n is a mulitpleof 3
1 + ωn + ω2n
0, when n is not a mulitpleof 3 1 1 1 1
= + + + ……. + = R.H.S.
z1 z 2 z3 zn
Sol: Case I: n = 3m; m ∈ I
∴ 1 + ωn + ω2n = 1 + ω3m + ω6m
Example 6: If | z1 + z 2 | = | z1 − z 2 | , prove that
3
= 1 + 1 + 1 [ ω =1] =3 π
argz1 − argz 2 = odd multiple of .
2
Case II: n = 3m + 1 or 3m + 2; m ∈ I
Sol: As we know | z | = z.z . Apply this formula and
(a) Let n = 3m + 1
M a them a ti cs | 6.35
consider
= z r ( cosθ + i sinθ ) . Equating real and imaginary parts.
n
| z1 + z 2 | 2 =| z1 − z 2 | 2 nπ
2 2 cos = a0 – a2 + a4 + …..
4
⇒ (z1 + z 2 ) (z1 + z2 ) = (z1 − z 2 ) (z1 − z2 ) or n
nπ
2 2 sin = a1 – a3 + a5 + …..
z1 z1 + z 2 z2 + z 2 z1 + z1 z2 = z1 z1 + z 2 z2 − z 2 z1 − z1 z2 4
or 0 ; Re (z1 z2 ) = 0
2(z 2 z1 + z1 z2 ) = Example 9: Solve the equation zn–1 = z :n∈N
Let=
z1 r1 (cos θ1 + isin θ1 ) and=
z 2 r2 (cos θ2 + isin θ2 );
Sol: Apply modulus on both side.
then
= z1 z2 r1r2 ( cos(θ1 − θ2 ) + isin(θ1 − θ2 ) )
Zn–1 = z ; |z|n–1 = | z | = |z|
∴ ∴= ( as Re(z1 z2 ) 0 )
cos (θ1 − θ2 ) 0=
∴ |z| = 0 or |z| = 1 If |z| = 0 then z = 0,
π
θ1 − θ2 = odd multiple of .
2 Let |z| = 1; then, zn = z z = 1
2mπ 2mπ
Example 7: If | z – 1 | < 3, prove that | iz + 3 − 5i |< 8 . ∴ z cos
= + isin : m = 0, 1, …., n – 1
n n
Sol: Here we have to reduce iz + 3 – 5i as the sum
1 − iz
of two complex numbers containing z – 1. because we Example 10: If z = x + iy and ω =
z −i
have to use
with |w| =1, show that, z lies on the real axis.
| z – 1 | < 3.
Sol: Substitute value of ω in |w| =1.
| iz + 3 – 5i | = | iz – i + 3 – 4i |
1 − iz
| ω |= = 1 ⇒ |1 – iz| = |z – i|
= | 3 – 4i + i (z – 1) | ≤ | 3 − 4i | + | i (z − 1) | z −i
(by triangle inequality) < 5 + 1 · 3 = 8 or, |1 – ix + y| = |x + i(y – 1)|
or, (1 + y)2 + x2 = x2 + (y – 1)2 or, 4y = 0
Example 8: If (1 + x) = a0 + a1x + a2x +……….…+ anx ,
n 2 n
Hence z lies on the real axis.
then show that
n
nπ Example 11: If a complex number z lies in the interior
(a) a0 – a2 + a4 + ….. = 2 2 cos
4 or on the boundary of a circle of radius as 3 and centre
n
nπ at (0, – 4) then greatest and least value of |z + 1| are-
(b) a1 – a3 + a5 + … = 2 2 sin
4
(A) 3 + 17 , 17 − 3 (B) 6, 1
Sol: Simply put x = i in the given expansion and then
by using formula (C) 17 , 1 (D) 3, 1
∴ Greatest distance = 3 + 12 + 42 = 3 + 17 equilateral triangle. Also show that |z1| = |z2| = |z3|.
Example 2: Find the sixth roots of z = 64i. Re(z) > 3 (see the Figure above)
π π (b) Let z = x + iy, then, x > 0 and y > 0
Sol: Here i = cos + isin and sixth root of z
2 2
i.e. zr = z1/6. y π y π
arg z = tan−1 tan ≤ ≤ tan
x 6 x 4
π π
=z 64 cos + isin ∴ zr = z1/6 Y
2 2
y=x
π π
2rπ + 2rπ +
= 2 cos 2 + isin 2
6 6 arg(z)=6
X
Where r = 0, 1, 2, 3, 4, 5 O
=
9π 9π
z 2 2 cos + isin Example 4: If z12 + z22 – 2z1z2 cos θ = 0, show that the
12 12 points z1, z2 and the origin, in the argand plane, are the
vertices of an isosceles triangle.
13π 13π π π
=z3 2 cos + isin −2 cos + isin
=
12 12 12 12 Sol: By using formula of roots of quadratic equation we
can solve it.
17π 17π 5π 5π
=z 4 2 cos + isin =−2 cos + isin z12 + z22 - 2z1z2 cos θ = 0
12 12 12 12
2
z z
21π 21π 9π 9π ⇒ 1 − 2 1 cos θ + 1 = 0
=z5 2 cos + isin −2 cos + isin
=
12 12 12 12 z2 z2
z 2cos θ ± 4 cos2 θ − 4
⇒ 1 =
Example 3: Locate the region in the Argand plane for
z2 2
the complex number z satisfying
π π
(a) |z – 4| < |z – 2| (b) ≤ arg z ≤ B(z)
6 4
Sol: Consider z = x + iy and solve by using properties B(z1)
of modulus and argument.
(a) Let z = x + iy O
y’ x=3
6 . 3 8 | Complex Number
deduce that c + c2 − d2 + c − c2 − d2 = |c + d| + |c
C(z3) B(z2)
– d| ; all numbers involved being complex
or
z32 + z22 – 2z2z3 + z12 + z32 – 2z1z3 = 0 Sol: By using algebra of complex number and modulus
property we can prove given expresson. And then by
Add and subtract 2z1z2, we get using Appolonius theorem we can interpret the result
geometrically.
z12 + z22 – 2z1z2 + 2z32 – 2z2z3 – 2z1z3 + 2z1z2 = 0, or
S = |a + b|2 + |a – b|2
(z1 – z2) + 2[z3(z3 – z2) – z1(z3 – z2)] = 0 or
2
Example 6: If A, B, C be the angles of triangle then This is nothing but Appolonius theorem. In DOAB, M
is midpoint of AB on applying Appolonius theorem we
e2iA e−iC e−iB get
prove that e−iC e2iB e−iA is purely real. (b)
−iB −iA 2iC B(a+b)
e e e
2
2
z1 a 1
c+d c−d c+d c−d
L.H.S.
= + + − we get z 2 b 1 =0
2 2 2 2
z3 c 1
On simplifying we get
Example 9: If b1 + b2 + b3 + b4 = 0 where b1 etc. are
L.H.S. = c + c2 − d2 + c − c2 − d2 non-zero real numbers, sum of no two being zero,
and b1z1 + b2z2 + b3z3 + b4z4 = 0 where no three of the
points z1, z2, z3, z4 are collinear then prove that the four
Example 8: Show that the triangles whose vertices are
point concyclic if b1b2 |z1 – z2|2 = b3b4 |z3 – z4|2.
z1, z2, z3 and a, b, c are
Example 10: Show that all the roots of the equation 2i sinnθ
= = itannθ . Taking negative sign,
z cos q0 + z cos q1 + z cos q2+......... + z cos qn–1 +
n n–1 n–2
2cosnθ
1 z 2n − 1 −2isin nθ
cos qn = 2 lie outside the circle | z |= where q0, q1 etc. similarly we get = = – i tan nθ
2 z 2n + 1 2cosnθ
are real.
z 2n − 1
∴ = | ± i tan nq| = |tan nq|,
Sol: By using triangle inequality. z 2n + 1
Here |zn cos q0 + zn–1 cos q1 + zn–2 cos q2 + …… + z For |± i| = 1.
cos qn–1 + cos qn| = 2 …. (i)
By triangle inequality. Example 12: Find the complex number z which satisfies
the condition |z – 2 + 2i| = 1 and has the least absolute
2 = |zn cos θ n + 2n–1 cos q1 + 2n–1 cos q2 + ….. + z
value.
cos qn–1 + cos qn | ≤ | zn cos qn| + |zn–1 cos q1|+
Sol: Here z – 2 + 2i = cos θ + i sin θ, therefore by obtaing
|zn–2 cos q2| + …..++ |z cos qn–1| + |cos qn| modulus of z we can solve above problem.
= |zn| |cos qn| + |zn–1| |cos qn| + ……..+ |z| |cos qn–1| + |z – 2 + 2i| = 1
π
8c ± 64c2 − 12(4c2 − 1) z3 − z1 z3 − z1 i 4 π π
∴ x= = = e 2 cos + isin
8(4c2 − 1) z 2 − z1 z 2 − z1 4 4
Example 14: Consider a square ABCD such that z1, z2, z3, /4
and z4 represent its vertices A, B, C and D respectively.
Express ‘z3’ and ‘z4’ in terms of z1 & z2.
JEE Main/Boards
Exercise 1
3π
Q.10 If z = 1 + i tan α, where π < α < . find the value
Q.1 Find all non-zero complex numbers z satisfying of |z| cos α. 2
z = iz 2 .
Q.11 If 1, ω, w2 be the cube roots of unity, find the roots
1 + 2i + 3i2
Q.2 Express in the form A + iB. of the equation (x – 1)3 + 8 = 0.
1 − 2i + 3i2
Q.3 Find x and y if (x + iy) (2 – 3i) =(4 + i) Q.12 If |z| < 4, prove that |i z + 3 – 4i| < 9.
(1 + i) x − 2i (2 − 3i)y + i
Q.4 Find x and y if + i
= Q.13 2 + i 3 is a vertex of square inscribed in circle |z –
3+i 3−i
1| =2. Find other vertices.
Q.5 If x = a + b, y = aα + bβ and z = aβ +bα, where α and
β are complex cube roots of unity, show that xyz = a3 + b3. Q.14 Find the centre and radius of the circle formed
by the points represented by z = x +iy satisfying the
1 + 7i |z−α|
Q.6 in the polar form. relation = k(k ≠ 1) where α & β are constant
(2 − i)2 | z −β|
Q.7 Find the square root of – 8 – 6i. complex number’s given by α =α1 + iα2 & β =β1 + iβ2
Q.15 Prove that there exists no complex number z such
Q.8 Find the value of smallest positive integer n, for 1 a
Q.17 Show that the area of the triangle on the Argand Q.28 If |z| ≤ 1, |w| ≤ 1, show that
diagram formed by the complex numbers: z, iz and
|z – w|2 ≤ (|z| – |w|)2 + (arg z – arg w)2.
1
z + iz is: | z |2 .
2
Q.29 Let A and B be two complex numbers such
Q.18 If iz3 + z2 – z + i = 0 then show that |z| =1. A B
that + = 1, prove that the origin and the points
B A
Q.19 Find the value of the expression represented by A and B form the vertices of an
equilateral triangle.
1(2 – ω) (2 – w2) + 2(3 – ω) (3 – w2) + ….
+ (n – 1) (n – ω) (n – w2) where ω is an imaginary cube Q.30 Let z1, z2, z3 be three complex numbers and a, b, c
root of unity. be real number not all zero, such that a + b + c = 0 and
az1 + bz2 + cz3 = 0.
Q.20 If x =
1
2
( )
5 − 3i , then find the value of Show that z1, z2, z3 are collinear.
x4 – x3 – 12x2 + 23x + 12.
Q.31 If |z – 4 + 3i | ≤ 2, find the least and the greatest
Q.21 Let the complex numbers z1, z2 and z3 be the vertices values of |z| and hence find the limits between which
of an equilateral triangle. Let z0 be the circumcentre of |z| lies.
the triangle. Then prove that: z12 + z22 + z32 = 3z02. z1 − z 2
Q.32 If |z1| < 1 and < 1, then show that |z2| < 1.
1 − z1 z 2
Q.22 If z1, z2, z3 are the vertices of an isosceles triangle,
right angled at z2, prove that z12 + 2z22 + z32 = 2z2 (z1 + z3). | z |2 − | z | + 1
Q.33 Find the locus of points z if log < 2.
3 2+ | z |
Q.23 Show that the equation
2 2 2
Q.34 For complex numbers z and ω, prove that |z|2ω – |
2
A B C H ω |2 z = z – ω if and only if z = w or zω =1 .
+ + + ...... = x+,
x −a x −b x −c x −h
Where A, B, C, …., a, b, c, …. and are real, cannot have
imaginary roots. Exercise 2
Single Correct Choice Type
Q.24 Find the common roots of the equation
z3 + 2z2 + 2z + 1 = 0 and z1985 + z100 + 1 = 0. Q.1 |z + 4| ≤ 3, Z∈ C: then the greatest and least value
of |z + 1| are:
Q.25 If n is an odd integer greater than 3 but not a (A) (7, 1) (B) (6, 1) (C) (6, 0) (D) None
multiple of 3, prove that [(x + y)n – xn – yn] is divisible by
xy (x + y) (x2 + xy + y2). Q.2 The maximum & minimum values of |z +1| when |z
+ 3| ≤ 3 are
Q.26 If α and β are any two complex numbers, (A) (5, 0) (B) (6, 0) (C) (7, 1) (D) (5, 1)
show that α + α2 − β2 + α − α2 − β2
Q.3 The points z1 = 3 + 3 i and z2 = 2 3 + 6i are given
= |α + β| + |α – β| on a complex plane. The complex number lying on the
bisector of the angle formed by the vectors z1 and z2 is:
Q.4 If z1, z2, z3, z4 are the vertices of a square in that Q.9 If q1, q2, q3 are the roots of the equation, x3 + 64 = 0,
order, then which of the following do(es) not hold q1 q2 q3
good?
then the value of the determinant q2 q3 q1 is:
z1 − z 2
(A) is purely imaginary q3 q1 q2
z3 − z 2 (A) 1 (B) 4
z1 − z3 (C) 10 (D) none of these
(B) is purely imaginary
z2 − z 4
Q.5 Let z1, z2 and z3 be the complex numbers Q.11 On the complex plane triangles OAP & OQR are
representing the vertices of a triangle ABC respectively similar and (OA) = 1. If the points P and Q denotes the
and a, b, c are lengths of BC, CA, AB. If P is a point complex numbers z1 & z2 then the complex number ‘z’
representing the complex number z0 satisfying: denoted by the point R is given by:
a(z1 – z0) + b(z2 – z0) + c(z3 – z0) = 0, then w.r.t. the z1 z2 z1 + z 2
triangle ABC, the point P is its: (A) z1z2 (B) (C) (D)
z2 z1 z2
(A) Centroid (B) Orthocentre
Q.12 If A and B be two complex numbers satisfying
(C) Circumcentre (D) Incentre
A B
+ = 1 . Then the two points represented by A and B
B A
Q.6 Three complex numbers α, β & γ are represented
in the Argand diagram by the three points A, B, C and the origin form the vertices of
respectively. The complex number represented by D (A) An equilateral triangle
where A, B, C, D form a parallelogram with BD on a
(B) An isosceles triangle which is not equilateral
diagonal is:
(C) An isosceles triangle which is not right angled
(A) α – β + g (B) – α + β + g
(D) A right angled triangle
(C) α + β – g (D) α – β – g
Q.16 Let z to be complex number having the argument Previous Years’ Questions
π
θ, 0 < θ < and satisfying the equality |z – 3i| = 3.
2 Q.1 The smallest positive integer n for which
6 n
Then cot θ – is equal to: 1 + i
z = 1 , is (1980)
1 −i
(A) 1 (B) – 1 (C) i (D) – i
(A) 8 (B) 16
Q.17 The locus represented by the equation, |z – 1| + (C) 12 (D) None of these
|z + 1| = 2 is:
(A) An ellipse with foci (1, 0): (– 1, 0) Q.2 The complex numbers z = x + iy which satisfy the
(B) One of the family of circles passing through the z − 5i
equation = 1 lie on (1981)
points of intersection of the circles |z + 1| = 1 z + 5i
(C) The radical axis of the circles |z – 1| = 1 and |z + 1| = 1 (A) The x-axis
(D) The portion of the real axis between the points (B) The straight line y = 5
(1, 0) and (– 1, 0) (C) A circle passing through the origin
(D) None of these
Q.18 Let P denotes a complex number z on the
Argand’s plane, and Q denotes a complex number
Q.3 If z = x + iy and w = (1 – iz) / (z – i), then |w| = 1
π
2 | z |2 cos + θ where θ = amp z if ‘O’ is the origin, implies that, in the complex plane (1983)
4
(A) z lies on the imaginary axis
then the DOPQ is:
(B) z lies on the real axis
(A) Isosceles but not right angled
(C) z lies on the unit circle
(B) Right angled but not isosceles
(D) None of these
(C) Right isoscles
(D) Equilateral Q.4 The points z1, z2, z3, z4 in the complex plane are the
vertices of a parallelogram taken in order, if and only if
Q.19 Let z1, z2, z3 be three distinct complex numbers (1983)
satisfying |z1 – 1| = |z2 –1| = |z3 – 1|. (A) z1 + z4 = z2 + z3 (B) z1 + z3 = z2 + z4
If z1 + z2 + z3 = 3 then z1, z2, z3 must represent the (C) z1 + z2 = z3 + z4 (D) None of these
vertices of:
(A) An equilateral triangle Q.5 If z1 and z2 are two non-zero complex numbers
such that |z1 + z2| = |z1| + |z2|, then arg (z1) – arg (z2) is
(B) An isoseles triangles which is not equilateral
equal to (1987)
(C) A right triangle π π
(A) – π (B) – (C) 0 (D)
(D) Nothing definite can be said 2 2
Q.6 The complex numbers sin x + i cos 2x and cos x – i
Q.20 If p = a + bω + cw ; q = b + cω + aw ; and
2 2 sin 2x are conjugate to each other, for (1988)
r = c + aω + bw2 where a, b, c ≠ 0 and ω is the complex (A) x = nπ (B) x = 0
cube root of unity, then:
1
(C) x = n + π (D) No value of x
(A) p + q + r = a + b + c 2
(B) p2 + q2 + r2 = a2 + b2 + c2
Q.7 If ω (≠ 1) is a cube root of unity and (1 +ω)7 = A +
(C) p2 + q2 + r2 = 2(pq + qr + rp)
Bω, then A and B are respectively (1995)
(D) None of these
(A) 0, 1 (B) 1, 1 (C) 1, 0 (D) – 1, 1
M a them a ti cs | 6.45
JEE Advanced/Boards
(i) If the equation has one real root then q′2 – pp′ q′ +
Q.9 Let z1 & z2 be any two arbitrary complex numbers
qp′2 = 0
then prove that:
1 z z
(ii) If the equation has two equal roots then p2 – q′2 = | z1 | + | z 2 | ≥ (| z1 | + | z 2 |) 1 + 2 .
4q and pp′ = 2q ′. 2 | z1 | | z 2 |
state whether these equal roots are real or complex. Q.10 Let zi (i = 1, 2, 3, 4) represent the vertices of a
square all of which lie on the sides of the triangle with
Q.2 Let z = 18 + 26i where z0 = x0 + iy0 (x0, y0∈ R) is the vertices (0, 0), (2, 1) and (3, 0). If z1 and z2 are purely
cube roots of z having least positive argument. Find the real, then area of triangle formed by z3, z4 and origin
value of x0 y 0 (x0 + y 0 ) . is m (where m and n are in their lowest form). Find the
value of (m + n).
Q.3 Show that the locus formed by z in the equation
z3 + iz =1 never crosses the coordinate axes in the
Q.11 (i) Let Cr ’s denotes the combinatorial coefficients
Argand’s plane.
in the expansion of (1 + x)n, n ∈ N. If the integers
− Im(z)
Further show that | z |= an = C0 + C3 + C6 + C9 + ……
2Re(z)Im(z) + 1
bn = C1 + C4 + C7 + C10 + ……
Q.4 Consider the diagonal matrix An = dia (d1, d2, d3, …. and cn = C2 + C3 + C8 + C11+…..
dn) of order where
i2 π
then prove that
Di =ai–1, 1 ≤ i ≤ n and α =e n ; i= −1 , is the nth root
of unity. (a) an3 + bn3 + cn3 – 3anbncn = 2n.
Let L: represent the value of Tr. (A7)7. (b) (an – bn)2 + (bn – cn)2 +(cn – an)2=2
M: denotes the value of det (A2n+1) + det (A2n). (ii) Prove the identity:
(C0 – C2 + C4 – C6 + ……… )2
Find the value of (L + M).
+ (C1 – C3 + C5 – C7 + ….. )2 = 2n.
[Note: Tr(A) denotes trace of square matrix A]
z7, find the value of Q.13 A function f is defined on the complex number by
7 7 f(z) = (a + bi)z, where ‘a’ and ‘b’ are positive numbers.
(a) ∑ Re(Zr ) and ∑ Im(Zr ) This function has the property that the image of each
r =1 r =1
point in the complex plane is equidistant from that
point and the origin. Given that |a + bi| = 8 and that
Q.7 If z is one of the imaginary 7th roots of unity, then
find the equation whose roots are (z + z4 + z2) and u
b2 = where u and v are co-primes. Find the value
(z6 + z3 + z5). v
of (u + v).
M a them a ti cs | 6.47
Q.15 Let f(x) = ax3 + bx2 + cx + d be a cubic polynomial Q.23 If the area of the polygon whose vertices are the
with real coefficients satisfying f(i)= 0 and f(1 + i) = 5. solutions (in the complex plane) of the equation
Find the value of a2 + b2 + c2 + d2.
x7 + x6 + x5 + x4 + x3 + x2 + x + 1 = 0
a b +c
Q.16 Let w1, w2, w3, …. wn be the complex numbers. A can be expressed in the simplest form as , find
the value of (a + b + c + d). d
line L on the complex plane is called a mean line for the
points w1, w2, w3, …. wn if L contains the points (complex
n Q.24 If a and b are positive integer such that
numbers) z1, z2, z3, …. zn such that ∑ (zk − ωk ) = 0 . N = (a + ib)3 – 107 i is a positive integer.
k =1
Q.17 A particle start to travel from a point P on the Q.26 Resolve z5 + 1 into linear and quadratic
curve C1: |z – 3 – 4i| = 5, where |z| is maximum. From
factors with real coefficients. Deduce that:
3
P, the particle moves through an angle tan−1 in π π
4 4 sin cos = 1 .
anticlockwise direction on |z – 3 – 4i| = 5 and reaches 10 5
at point Q. From Q, it comes down parallel to imaginary
axis by 2 units and reaches at point R. Find the complex Q.27 If x = 1 + i 3 : y = 1 – i 3 & z = 2,
number corresponding to point R in the Argand plane.
prove that xp + yp = zp for every prime p > 3.
p
32 10 2qπ 2qπ
Q.18 Evaluate: ∑ (3p − 2) ∑ sin − icos Q.28 Dividing f(z) by z – i, we get the remainder i and
q 1
= p 1= 11 11 dividing it by z + i we get the remainder 1 + i. Find the
remainder upon the division of f(z) by z2 + 1.
Q.19 Let a, b, c be distinct complex numbers
Q.29 (a) Let z = a + b be a complex number, where x
a b c
such that = = = k. and y are real numbers. Let A and B be the sets defined
1−b 1−c 1−a by
Find the value of k.
A = {z| |z| ≤ 2} and
Q.8 Which of the following represents a point in an Q.15 If Ar (r = 1, 2, 3, ….., n) are the vertices of a regular
argand’s plane, equidistant from the roots of equation polygon inscribed in a circle of radius R, then
(z + 1)4 = 16z4?
(A1A2)2 + (A1A3)2 + (A1A4)2 + …+ (A1An)2 =
1 1 2
(A) (0, 0) (B) − ,0 (C) ,0 (D) 0, nR 2
3 3 5 (A) (B) 2nR2
2
π
Q.9 The equation of the radical axis of the two circles (C) 4R2 cot (D) (2n – 1) R2
presented by the equations. 2n
|z – 2| =3 and |z – 2 – 3i| = 4 on the complex plane is: Q.16 If the equation z4 + a1z3 + a2z2 + a3z + a4 = 0,
(A) 3y + 1 = 0 (B) 3y – 1 = 0 where a1, a2, a3, a4 are real coefficients different from
zero has a pure imaginary root then the expression
(C) 2y – 1 = 0 (D) None of these a3 aa
+ 1 4 has the value equal to
a1a2 a2a3
Q.10 Number of real solution of the equation, z3 + iz – 1
= 0 is (A) 0 (B) 1 (C) – 2 (D) 2
(A) Zero (B) One (C) Two (D) Three
Q.17 All roots of the equation (1 + z)6 + z6 = 0
Q.11 A point ‘z’ moves on the curve |z – 4 – 3i| = 2 in (A) Lie on a unit circle with centre at the origin
an argand plane. The maximum and minimum values
(B) Lie on a unit circle with centre at (– 1, 0)
of |z| are:
(C) Lie on the vertices of a regular polygon with centre
(A) 2, 1 (B) 6, 5 (C) 4, 3 (D) 7, 3
at the origin
π (D) Are collinear
Q.12 Let z = 1 – sin α + i cos α where α ∈ 0, , then
2
the modulus and the principle value of the argument of Q.18 Number of roots of the equation z10 – z5 – 992 = 0
z are respectively: with real part negative is:
π α π α (A) 3 (B) 4 (C) 5 (D) 6
(A) 2(1 − sin α ), + (B) 2(1 − sin α ), −
4 2 4 2
Q.19 z1 and z2 are two distinct points in argand plane.
π α π α
(C) 2(1 + sin α ), + (D) 2(1 + sin α ), − az1 bz 2
4 2 4 2 If a |z1| = b|z2|, then the point + is a point on
bz 2 az1
the (a, b ∈ R)
Q.13 z1 and z2 are complex numbers. Then
(A) Line segment [– 2, 2] of the real axis
(B) (1 – ix) (1 + ix)–1 = a – ib ⇒ a2 + b2 = 1 (A) α moves on the circle, centre at (– 2, 0) and radius 3
(C) (a + ib) (a – ib)–1 = x – iy ⇒ |x + iy| = 1 (B) α and β describe the same locus
Q.36 Identify the correct statements(s). Read the following passage and answer the following
questions.
(A) No non zero complex number z satisfies the
equation, z = −4z Let A, B, C be three sets of complex number as defined
below
(B) z = z implies that z is purely real
(C) z = −z implies that z is purely imaginary A = {z: |Im z ≥ 1|}
Q.37 If the complex numbers z1, z2, z3 & z1′, z2′ and z3′ Q.3 The number of elements in the set A ∩ B ∩ C is
are representing the vertices of two triangles such that (A) 0 (B) 1 (C) 2 (D) ∞
z3 = (1 – z0) z1 + z0z2 and z3 ‘ = (1 – z0) z1’ + z0z2’ where
z0 is also a complex number then:
6 . 5 2 | Complex Number
Q.7 The inequality |z – 4| < |z – 2| represents the region Q.15 Let z1 and z2 be two distinct complex numbers
given by (1982) and let z = (1 – t) z1 + tz2 for some real number t with 0
< t < 1. If arg (w) denotes the principal argument of a
(A) Re (z) ≥ 0 (B) Re (z) < 0 non-zero complex number w, then (2010)
(C) Re (z) > 0 (D) None of these (A) |z – z1| + |z – z2| = |z1 – z2|
Q.8 If a, b, c and u, v, w are the complex numbers (B) arg (z – z1) = arg (z – z2)
representing the vertices of two triangles such that z − z1 z − z1
c= (1–r) a+rb and w = (1 – r) u + rv, where r is a complex (C) =0
z 2 − z1 z2 − z1
number, then the two triangles (1985)
(A) Have the same area (B) Are similar (D) arg (z – z1) = arg (z2 – z1)
Q.10 Let z and w be two complex numbers such that Column I Column II
|z| ≤ 1, |w| ≤ 1 and |z + iw| = |z – iw | = 2, then z equals (A) The set of points z (p) An ellipse with
(1995) satisfying |z-i|z|| = |z + i|z|| 4
(A) 1 or i (B) i or – i (C) 1 or – 1 (D) i or –1 eccentricity
Is contained in or equal to 5
Q.11 For positive integers n1, n2 the value of expression (B) The set of points z (q) The set of points z
n n n2 n
satisfying |z+4|+|z-4|=10 is satisfying Im = z = 0
(1 + i) 1 + (1 + i3 ) 1 + (1 + i5 ) + (1 + i7 ) 2 ,Here i= −1 contained in or equal to
is a real number, if and only if (1996) (C) If |w|=2, then the set (r) The set of points z
(A) n1 = n2 + 1 (B) n1 = n2 – 1 1 satisfying | Im z | ≤ 1
of points z= w − is
w
(C) n1 = n2 (D) n1 > 0, n2 > 0 contained in or equal to
M a them a ti cs | 6.53
1 1 1 1
(A) (B) (C) (D)
Q.18 Let z1 and z 2 be two distinct complex numbers 2 2 7 3
and let z = (1 − t ) z1 + t z2 for some real number t with
3 +i
0 < t < 1. If Arg (w) denotes the principal argument of a
nonzero complex number w, then (2010)
Q.24 Let ω =
2
and P =ωn : n = {
1,2,3,..... . Further }
1 −1
z ∈ C : Rez > and H2 =
H1 = z ∈ C : Rez < ,
(A) | z − z1 | + | z − z 2 |= | z1 − z 2 | 2 2
where C is the set of all complex numbers. If
(B) Arg ( z − z1=
) Arg ( z − z2 )
z1 ∈ P ∩ H1 , z 2 ∈ P ∩ H2 and O represents the origin,
z − z1 z − z1
(C) =0
z 2 − z1 z2 − z1 then ∠ z1 Oz 2 = (2013)
π π 2π 5π
(D) Arg ( z − z=
1) Arg ( z 2 − z1 ) (A) (B) (C) (D)
2 6 3 6
(s) 0 1 1
1/2 (A) The circle with radius and centre , 0 for
1 + x 2a 2a
∫ cos 2x.log dx a > 0, b ≠ 0
1 − x
(iv) −1/2
equals 1 1
1/2 1 + x (B) The circle with radius − and centre − , 0 for
∫ cos 2x.log dx a < 0, b ≠ 0 2a 2a
1 − x
0
(C) The x-axis for a ≠ 0, b = 0
Codes:
(D) The y-axis for a = 0, b ≠ 0
(i) (ii) (iii) (iv)
(A) r q s p
(B) q r s p
(C) r q p s
(D) q r p s
M a them a ti cs | 6.55
PlancEssential Questions
JEE Main/Boards JEE Advanced/Boards
Exercise 1 Exercise 1
Q.6 Q.9 Q.15 Q.7 Q.11 Q.13
Exercise 2 Exercise 2
Q.2 Q.8 Q.10 Q.2 Q.6 Q.9
Q. 25 Q.27 Q.31
Previous Years’ Questions
Q. 33 Q. 36 Q. 39
Q.2 Q.4 Q.7
Answer Key
JEE Main/Boards
Exercise 1
3 i 5 14
Q.1 z = 0, i, ± – Q.2 – i Q.3
= x = ,y
2 2 13 13
3π 3π
Q.4 x = 3, y – 1 Q.6 2 cos + isin Q.7 ± (1 – 3i)
4 4
Q.8 n = 4 Q.10 – 1 Q.11 – 1, 1 – 2ω, 1 – 2ω2
α − K 2β α + β −K (n − 1)n 2
Q.14 Centre , radius Q.19 [n + 3n + 4]
2
1 − K 1 − K2 4
6 . 5 6 | Complex Number
Q.20 5 Q.24 ω, w2
Exercise 2
JEE Advanced/Boards
Exercise 1
7
Q.2 12 Q.4 7 Q.5 10 Q.6 (a) − , (b) zero Q.7 x2 + x + 2 = 0 Q.8 4
2
Q.10 41 Q.13 259 Q.15 26 Q.16 163 Q.17 (3 + 7i) Q.18 48 (1 – i)
2
Q.19 – ω or –w2 Q.20 k > α − β
Q.21 If (z) is maximum when z = ω, when ω is the cube root unity v and If (z) = 13
iz 1
Q.26 (z + 1) (z2 – 2z cos 36° + 1) (z2 – 2z cos 108° + 1) Q.28 + + i
2 2
1
Q.29 (a) π – 2; (b) Q.30 A → s; B → q; C → p
2
Exercise 2
Solutions
⇒ab = – 3 …… (ii)
M a them a ti cs | 6.59
n
1
Sol 15: |z| <
3
and ∑ ar zr = 1 … (i)
r =1
Here one vertex is A (2, 3) and equation of circle is ⇒ 1 ≤ |a1z| + |a2z2| + |a3z3| …..|anzn|
(x − 1)2 + y 2 =
4
⇒ |a1||z| + |a2||z|2 + |a3||z|3 …..|an||z|n ≥ 1
Radius of the circle is 2. Hence side of the square will
1
be 2 2 . ⇒ |z| + |z|2 …….|z|n ≥
2
Points that lie on the circle and are at a distance 2 2 ⇒ (Limiting case n → ∞)
from A are B (1 − 3,1) and D (1 + 3, −1) . z 1 1
≥ ⇒ z ≥ − (2)
The point C will be the other end of diameter of A. 1− z 2 3
Hence C (0, − 3) . From (i) and (ii), we can say that there is no ‘z’ satisfying
both conditions.
Hence the four vertices are
1 2 1 2 = (x – 3)[x3 + 2x2 – 6x + 5] + 27
⇒ |D| = (x + y2) = z
2 2
= (x – 3)[(x – 3)(x2 + 5x + 9) + 32] + 27
If z 2 =⇒
i | z |2 =1 ⇒ | z |=1 = w + w2 + 32w + 32w2 + 38
Hence, |z| = 1 = –1 – 32 + 38 = 5
Sol 19: 1(2 – w) (2 – w2) + 2(3 – w) (3 – w2) + …… (n – 1) Sol 21: z2 – z1 = (z3 – z1)e–iπ/3
(n – w) (n – w2)
z3 – z2 = (z1 – z2) e–iπ/3
Tn = n(n + 1 – w) (n + 1 – w ) 2
z3 − z1
z 2 − z1
=
= (n + n – nw) (n + 1 – w )
2 2 z3 − z 2 z1 − z 2
1 1
Sol 20: x = (5 − i 3) =(6 − 1 − i 3) =
3+w
2 2
x4 – x3 – 12x2 + 23x + 12
O z3-z2
= x3(x – 3) + 2x2(x – 3) – 6x(x – 3) (z1 – z2) = (z3 – z2)e iπ/2
α + α2 − β2
⇒ z12 + 2z22 + z32 = 2z2(z1 + z3) = α + α2 + β2 α + α2 − β2 +
A2 B2 H2 2 2
α − α − β + 2|α – a + b |
2 2 2
Sol 23: + .......... x + Let x = p + iq
=
x −a x −b x −H
2 2 2
2
A2 A (p− a) − iq = 2 α + α2 − β2 + 2 β
(p + ) + iq = ∑ =∑
(p − a) + iq (p − a)2 + q2
2 2
= 2 α + 2 β + 2 α2 − β2
A2 (p− a) A2q
= ∑ (p − a)2 + q2 − i∑ (p − a)2 + q2 2
= 2 α +2 β +2 α +β α −β
2
Sol 26: T = α + α2 − β2 + α − α2 − β2 2 2
= z + w − z w − zw + 2 z w − 2 z w
( )
2 2 2
T2 = α + α2 − β2 + α − α2 − β2 + = z −w − z w − zw + 2 z w
z1 – z2 = k1(z3 – z2)
(z − w)
2
= − 2r1r1 cos ( θ1 − θ2 ) + 2r1r2
⇒ z1, z2 and z3 are collinear (by vector)
θ − θ
( )
2
=z − w + 2r1r1 2sin2 1 2
2 Sol 31: |z – 4 + 3i| ≤ 2
2
θ − θ2
( )
2
=z − w + 4r1r1 sin 1
2
( ) + ( θ1 − θ2 )2
2
≤ z −w
( ) + (arg z − arg w )
2 2
≤ z − w
The shaded area show z
Min and max value = Distance of centre from origin ±
A B radius
Sol 29: + = 1
B A
= 5 ± 2 = 3, 7
A2 + B2 = AB
⇒ 3 ≤ |z| ≤ 7
A B
= 1−
B A
R z1 − z 2
A A −B Sol 32: <1
= 1 − z1 z 2
B A
2 2
lA-Bl z1 − z 2 < 1 − z1 z 2
lRl
A2
= A −B (z1 – z2) (z1 − z2 ) < (1 − z1 z 2 )(1 − z1 z2 )
B
lAl ⇒ |z1|2 + |z2|2 – z1 z2 – z1 Z2
2 O A
B
= B−A < – z1 z2 – z2 z1 + |z1|2|z2|2 + 1
A
⇒ |z1|2 – 1 < (|z1|2 – 1) |z2|2
2 2
A B
A −B = B − A ⇒ = 2
z1 − 1
B A
|z2| <
2
2
⇒|A | = |B |3 3 z1 − 1
⇒ |z| + 1 ≥ 0 z1
So (|z| – 5) < 0 z3
Interior of circle x2 + y2 = 25
z2
Sol 34: |z| ω – | ω 2|z = z – ω
2
zˆ 3 = zˆ 2e ; zˆ 1 = zˆ 3eiθ
iθ
⇒ zzω − ωωz = z − ω
24i
zˆ 32 = zˆ 1 zˆ 2 =
⇒ z ω (z − ω) = z – ω 24
Exercise 2
z1(x+iy)
z4(-x+iy)
(x = y as it is aqueous)
Sol 2: (D) |z + 3| ≤ 3 Least & greatest value of |z + 1|
ie its minimum and maximum distance from (–1,0) is 1 2ixy
= (Purely Imaginary)
and 5. x + y2
2
az1 + bz 2 + cz3
Sol 5: (D) = z0
(-3,0)(-1,0) a+b+c
A(z1)
Sol 3: (B) z1 = 3 + i 3
c b
z2 = 2 3 + 6i
B(z2) a C(z3)
z0 is incentre.
6 . 6 4 | Complex Number
A B
Sol 12: (A) + =1
B A
A
C() B() Let y =
B
1
Sol 7: (B) z ≥ 3 y+ 1
=
y
1 1 1 8
z+ ≥ z − =3– = ⇒ y2 − y + 1 =0
z z 3 3
1+ i 3
⇒y=
2
Sol 8: (C)
iπ iπ A 1+ i 3
⇒ =
± z3
± z2
z 2 = z1e 4
z3 = z 2e 4 B 2
/4 z1
(1 ± i) A
z3 – z2 = (z2 – z1) ⇒ 1
= A
2 B
(1 ± i) From Rotation Theorem
z3 = z2 + (z2 – z1)
2 A A iθ B
= e
B B O
Sol 9: (D) x3 = (4)3 (–1)1/3
1+ i 3
x = –4, –4w, –4w2 ⇒ eiθ =
2
q1 q2 q3 −4 −4ω −4ω2 ⇒ θ =60o
2
q2 q3 q1 = −4ω −4ω −4
and A = B ⇒ ∠OAB = ∠OBA = 60o
q3 q1 q2 −4ω2 −4 −4ω
⇒ AOB is equilateral triangle
2
1 ω ω 0 0 0
2 2
= –64 ω ω 1 = –64 ω ω 1 =0 Sol 13: (D) |z|2 – (z + z ) + i(z – z ) + 2 = 0
2 2
ω 1 ω ω 1 ω
z z – (z + z ) + i(z – z ) + 2 = 0
Let z = a + ib
Sol 10: (D) z = (3 + 7i) (p + iq) is purely imaginary.
a2 + b2 – 2a + i(i2b) + 2 = 0
⇒ 3p – 7q = 0
a2 – 2a + 1 + b2 – 2b + 1 = 0
7q
⇒p= (a – 1)2 + (b – 1)2 = 0
3
2
|z|2 = |3 + 7i|2 |p + iq|2 = 58 p2 + q2 ⇒a=b=1
for minimum |z|, q = 3, p = 7 ⇒z=1+i
|z|2 = 58(49+9) = 3364
Sol 14: (D) z1 = –3 + 5i
3 2
Sol 19: (A) |z1 – 1| = |z2 – 1| = |z3 – 1|
2
z 1 + z 2 + z3 = 3
Centroid is at z = 1
3
Radical axis is x =
2
z1
z2
x +y =4
2 2
9 9
⇒ + y2 =
4 ⇒ y2 = 4 –
4 4
7 ± 7
⇒ y2 = ⇒y=
4 2 Distance of vertical z1 , z 2 and z3 from centroid is same,
3 i 7 which mean centroid coincides with circumcenter.
Complex no. is ±
2 2 Therefore, ∆ is equilateral.
Sol 16: (C) |z – 3i| = 3 ⇒ (x)2 + (y – 3)2 = 9 Sol 20: (C) p = a + bw + cw2
y q = b + cw + aw2
⇒ tanθ =
x
6 x 6(x − iy) r = c + aw + bw2
x 6(x − iy)
cotθ = – = −
z y (x + iy)(x − iy) y x2 + y 2 p + q + r = a(1 + w + w2) + b(1 + w + w2)
x2 + y2 = 6y + c(1 + w + w2)
x 6(x − iy) = (a + b + c) (1 + w + w2) = 0
⇒ − =i
y 6y
p2+q2+r2=(p + q + r)2 – 2pq – 2qr – 2rp
The portion of real axis between (–1,0) & (1,0) as the = 2ab + c2 + 2acw + (b2 + 2bc)w2 + a2w + 2bc + a2 +
distance between both the point is 2 2baw + (b2 + 2ac)w2 + c2w + 2ac + b2 + 2bcw + (c2 +
a2 + 2ab)w2
2 π
Sol 18: (C) Q = 2 z cis + θ = (2ab + 2bc + 2ac) (1 + w + w2)
4
+ (c2 + b2 + a2) (1 + w2 + w)
π π
= |z| 2 cos + θ + isin + θ
4 4 p2 + q2 + r2 = 0 = 2(pq + qr + rp)
6 . 6 6 | Complex Number
∴ z re
= =( ) rei π .e−i θ
π−θ
(∵ |1 – iz| = |– i| |z + i| = |z + i|)
∴ It is a perpendicular bisector of (0, 1) and (0, –1) = – re–iθ = – ω
i.e., x-axis ∴ z lies on the perpendicular bisector of the line joining
Thus, z lies on real axis. –iw and – iω is the mirror image of – iw in the x-axis,
the locus of z is the x-axis
Sol 4: (B) Since, z1, z2, z3, z4 are the vertices of parallelogram.
Letz = x + iy and y = 0
D(z4) C(z3)
Now,|z| ≤ 1 ⇒ x2 + 02 ≤ 1
⇒–1≤x≤1 ….. (i)
∴ z may take values given in (i).
A(z1) B(z2)
Sol 9: (D)
∴ Mid-point of AC = mid-point of BD (1 + ω − ω2 )7 = ( −ω2 − ω2 )7 ( 1 + ω + ω2 = 0)
=( −2ω2 )7 =( −2)7 ω14 =−128ω2
M a them a ti cs | 6.67
6i 1 1 z − 3eiπ / 4 4 −iπ /2 4i
= e = −
⇒ − 3i 4 −1 −1 = x + iy 0 − 3e iπ / 4 3 3
20 i i ⇒ 3z − 9eiπ / 4 =
12ieiπ / 4
⇒ x + iy =
0 ( C2 and C3 are identical) ⇒ z =(3 + 4i)eiπ / 4
⇒ x=
0, y =
0 y
p
b 0 or a2 + b2 −=
⇒= 2a 0 2 + 4i sin θ + 3 i sin θ − 6 sin2 θ
=
1 + 4 sin2 θ
⇒ Either real axis or circle passing through origin.
2 − 6 sin2 θ
Given =0
1+z 1 + 4 sin2 θ
Sol 17: (B) Let θ =arg
1 + z 1
⇒ sin2 θ =
1+z 3
⇒ θ = arg {| z |= 1 ⇒ z z = 1} 1
1 + 1 ⇒ sin θ = ±
3
z
1 −1 1
⇒ θ =arg ( z ) =⇒ θ sin−1 , − sin
3 3
1
Sol 18: (B) Given: The expression z + and z ≥ 2
2
Using triangle in equality JEE Advanced/Boards
z1 + z 2 ≥ | z1 | − | z 2 |
Exercise 1
1 1
⇒ z+ ≥ | z |− Sol 1: z2 + (p + ip’)z + (q + iq’) = 0
2 2
One real root
1 1
⇒ z+ ≥ z−
2 2 (z2 + pz + q) + i(p’z + q’) = 0
If z is real
1 3
⇒ z+ ≥ lies in (1, 2) −q'
2 2 P’z = –q’ ⇒ z =
p'
z2 + pz + q = 0
Sol 19: (B) z1 − 2z 2 = 1
2 − z1 z2 q'2 pq'
− +q=0
⇒ z1 − 2z 2 =2 − z1 z2 p' 2 p'
q'2 − pp'q'+ qp'2 =
0
⇒ ( z1 − 2z 2 )( z1 − 2 z2 ) =( 2 − z1 z2 )( 2 − z1 z 2 )
If eqn. has 2 equal roots
2 2
⇒ z1 − 2 z1 z2 − 2 z1 z 2 + 4 z 2
(p + ip’)2 = 4(q + iq’)
2 2
4 − 2 z1 z 2 − 2z1 z2 + z1
= z2 2
p= − p'2 4q and
= p'2 4q2
2 2 2 2
⇒ z1 + 4 z 2 4 z2
=+ z1 −(p + ip')
The roots are i.e. roots (equal) are imaginary.
2
⇒ z1 + 1 − z 2 − 4 1 − z 2 =
2 2 2
0
Sol 2: z = 18 + 26i
⇒ 1 − z 2 z1 − 4 =
2 2
0 z0 = x0 + iy0 is cube root of z
z0 = (r{cosθ + isinq})1/3 = r1/3 eiθ/3
⇒ z1 = 4
2
{ z2 ≠1 }
θ θ
⇒ z1 =
2 = r1/3 cos + isin
3 3
2 + 3i sin θ r = (1000)1/2
Sol 20: (C)
1 − 2i sin θ r1/3 = (103/2)1/3 = 101/2
=
( 2 + 3i sin θ )(1 + 2i sin θ ) cosθ =
18
sinθ =
26
θ
cos = 0.95
3 πi 4n2 −2n
π
n 2 i(2n2 −n)
θ = e = en = eiπ(2n – 1)
sin = 0.3162
3 det A2n+1 = ei2pn
z= 10 (0.95 + i 0.3162) = 3 + i det A2n + det A2n+1 = eiπ(2n – 1) + ei2pn
x0 = 3, y0 = 1 =cos(2n–1)π + isin(2n–1)π + cos2nπ + isin2np
x0y0(x0 + y0) = 3.4 = 12 M=0⇒L+M=7
z1 − z 2 iπ /2
Sol 12: = e= i
z4
(2,1)
z3 z3 − z 2
z1 − iz3 z1 (1 + i) + z3 (1 − i)
(0,0) z1 z2 (3,0) = z 2 ⇒ Z2 =
1−i 2
z1 − z 4
z1 = (a, 0), z2 = (b, 0), z3 = (b, c), z4 = (a, c) e−iπ /2 = −i ⇒ z1 – z4 = –i(z3 – z4)
z3 − z 4
b – a = c(square)
M a them a ti cs | 6.71
z1 (1 − i) + z3 (1 + i) a (1 + i ) + b (1 + i ) + c (1 + i ) + d =
3 2
⇒ z4 = 5
2
( ) ( )
⇒ a 1 + i3 + 3i2 + 3i + b 1 + i2 + 2i + c (1 + i ) + d =
5
Sol 13: f(z) = (a + ib)(z) = c + id
⇒ a (1 − i − 3 + 3i) + b (1 − 1 + 2i) + c (1 + i) + d =5
z = (x + iy)
⇒ a ( −2 + 2i) + b ( 2i) + c (1 + i) + d =5
Image = c – id
(c – x)2 + (y + d)2 = c2 + d2 ⇒ − 2a + c + d = 5 and 2a + 2b + c =0
x2 + y 2 − 2cx + 2dy =
0 From (i), we have
a2 + b2 = 64 d – a = 5 and 3a + 2d = 0
c = ax – by ⇒ a =c =−2 and b= d= 3
( −2) + 32 + ( −2) + 32
2 2
d = bx – ay ∴ a2 + b2 + c2 + d2 =
1 1 255 u
Sol 16: ∑ ( zk − wk ) =
0
k =1
⇒a= ⇒ b2 = 64 – = =
2 4 4 v ⇒ z1 + z2 + z3 + z4 + z5
⇒ u + v = 259 ⇒ Z1 + Z 2 + Z3 + Z 4 + Z5
= 32 + 170i − 7 + 64i − 9 + 200i + 1 + 27i − 14 + 43i
Sol 14: = 3 + 504i
cosx + nC1cos2x + nC2cos3x + .......... + nCncos(n + 1)x If y-intercept is 3, then eq. of line
p
32 10 2qπ 2qπ 2k + 2α1β1 + 2α2β2 − α12 − α22 − β12 − β22
Sol 18: ∑ ( 3p + 2 ) ∑ sin − icos Radius =
= p 1= q 1 11 11 4
p
10 2qπi Sol 21: f(z) = |z3 – z + 2 |z| = 1
= ∑ ( 3p + 2 ) −i ∑ e 11 − 1
q=0
(f(z))2 = |z3 – z + 2|2=(z3 – z + 2)( z 3 – z + 2)
= 1–z2+2z3– z 2 +z z –2z+2 z 3 –2 z +4
32 32 32 p
= ∑ (3p + 2 ) ip = ∑ (3p ) ip 2 ∑ i = 0 = 6 – (z2 – z 2 ) –2(z + z ) + 2(Z3 + z 3 )
p =1 p =1 1
= 6–2(a2–b2)–4(a)+2(z+ z )(z2+ z 2 – z z )
= 3i(1 − 3 + − 31) − 3(2 − 4 + − 32) =48(1 − i)
= 6–2(a2–b2)–4(a)+4a(2a2–2b2–1)
a b c = 6–8a–2(2a2–1) + 8a(2a2–1)
Sol 19: = = = k
1−b 1−c 1−a f(z) = 16a3 – 4a2 – 16a + 8
a = k – kb f’(z) = 48a2 – 8a – 16 = 8(6a2 – a – 2)
b = k – kc = 8(6a2–4a + 3a – 2)
c = k – ka = 8(2a(3a – 2) + 1(3a – 1))
a = k – k2 + k2(k – ka) = 8(2a + 1) (3a – 2)
a = k – k + k –k a)
2 3 3
−1 2
a= a=
k −k +k 2 3 2 3
a= = b= c
1 + k3 2
For a =
3
but a ≠ b ≠ c 8 4 2
f(z) = 16 × –4× – 16 × +8
i.e. k = –1 ⇒ k = –w, –w
3 2 27 9 3
128 – 48 − 288 + 202
= <0
Sol 20: |z – a|2 + |z – b|2 = k 27
Locus of z is a circle −1
For a = ; f (z) = –2 – 1 + 8 + 8 = 13
(x – a1)2+(x – b1)2+(y – a2)2+(y – b2)2 = k 2
Maximum value of f(z) = 13
⇒ 2x2 + 2y2 – 2x(a1 + b1) – 2y2(a2 + b2)
α + β1 α2 + β2 = 2|w2 – w| = 2|(– 3 )| = 2 3
Centre = 1 ,
2 2 = 12 = n1/4 ⇒ n = 144
M a them a ti cs | 6.73
x8 − 1
=0 Sol 26: z5 + 1
x −1 2
3 1 = (z3 + 1) z2 + (1 – z2)
4 = (z + 1) (z2 – z + 1) z2 + (1 – z2)
= (1 + z)[(z2 – z + 1) z2 + 1 – z]
5 7 = (1 + z)[z4 – z3 + z2 – z + 1]
6
= (1 + z)[z2 + az + 1] [z2 + bz + 1]
Total area = unshaded area + shaded area
⇒ b + a = –1
ba + 2 = 1
0 0 1
Unshaded area = 6 × 1 0 1 1 ab = –1
1 1 1
1 ⇒a– = – 1 ⇒ a2 + a – 1 = 0
2 2 a
6 1 6 −1 ± 5 −1 + 5
= × = ⇒a= ⇒a= = –2cos108°
2 2 2 2 2 2
−1 − 5
1 1 1 1 And b = = –2cos36°
Shaded area = 2 × × × = 2
2 2 2 2
∴ Factors are (1 + z) (z2 – 2zcos36° + 1) (z2 – 2cos108° + 1)
6 1 6+ 2 6 2 +2 Since ab = –1
Total area = + = =
2 2 2 2 2 4 π π
⇒ 4cos36°cos108° = -1 ⇒ 4 cos cos = 1
5 10
3 2 +1 a b +c
= =
2 d Sol 27: x = 1 + i 3
a+b+c+d=3+2+1+2=8 y=1–i 3
z=2
Sol 24: N = (a + ib)3 – 107i x = –2wy = –2w2z = 2
= a3 – ib3 + 3a2ib – 3ab2 – 107i xp + ypp > 3 prime (P is odd)
N = (–b + 3a b – 107)i + a – 3ab
3 2 3 2
= –2pwp – 2p w2p = –2p(wp + w2p) = 2p = zp
⇒ 3a b – b = 107 ⇒ b(3a – b ) = 107
2 3 2 2
⇒ N = a3 – 3ab2 = 216 – 18 = 198 f(z) = b(z + i) +(1+ i) ⇒ f(–i) = 1+i ... (ii)
f(z) = c(z2 + 1) + k1z + k2 … (iii)
Sol 25: x + ax + bx + cx + d has 4 non-real roots.
4 3 2
Substituting values from (i) and (ii) in (iii)
α + β = 3 + 4i ⇒ i = k1i + k2 and i + 1 = –k1i + k2
gδ = 13 + i(γ = α , δ = β ) 1 i
⇒ k2 = i + and k1 =
= 13 + i aβ = 13 – i
⇒ αβ 2 2
,
α + β = 3 + 4i iz 1
∴ Remainder = + i +
2 2
⇒ aβ + bγ + gδ + dα + aγ + bδ = b
⇒ aβ + β α + αβ + α α + β β + α β = b
⇒ 13 – i + 13 + i +( α + β )(α + β) = b
= 26+ (3 + 4i) (3 – 4i) = b
6 . 7 4 | Complex Number
-z -
z
M a them a ti cs | 6.75
⇒ ( z1 − z 2 ) = z 2 and ( z1 − z 2 ) =
2
−z 22 =z1 − z 2 = iz 2 z + z2 π
tan–1 1 =
and z1 − z 2 2
⇒e
i∠oz2 z1
= i ⇒ ∠oz 2 z1 = 90o Sol 7: (D) z1
Sol 4: (B) 2 1
23 O G C
-2 z2 z3
4
23 z1 + z 2 + z3
= Centroid,
= zc 0
3
π
|Z| ≤ 4 and Arg ( z ) =
3 z1 + z 2 + z3 2z C + z O
= ⇒ zO = z1 + z2 + z3
3 3
x2 + y2 ≤ 16 and y = 3 x
6 . 7 6 | Complex Number
cos α 1
arg z = tan–1 = tan−1
1 − sin α sec α − tan α
2
α α
sin + cos
1 + sin α 2 2
= tan–1 = tan–1
cos α
2
2 α 2 α
cos − sin
2 2
|z – 2 – 3i| = 4 α α α
sin + cos 1 + tan
⇒S1 = (x – 2) + y = 9
2 2
= tan−1 2 2 = tan–1 2
⇒S2 = (x – 2)2 + (y – 3)2 = 16 cos α − sin α 1 − tan α
2 2 2
Both circles are intersecting. So, radical axis will be
π α
S1 – S2 = 0 ⇒ 9 – 6y = 7 ⇒ 3y – 1 = 0 = tan–1 tan π + α = +
4 2 4 2
k3 + ik ‒ 1 = 0 and k = 0 z1 + z 2 + 2 z1 z 2 + z1 + z 2 − 2 z1 z 2
=
If k = 0, then 0 ‒ 1 = 0 2
( ) +( )
2 2
Not possible z1 + z 2 z1 − z 2
=
Therefore z ≠ k (real) 2
Hence, no real solution
2 ( z1 ) + 2 ( z 2 )
2 2 2 2
= = z1 + z2
2
M a them a ti cs | 6.77
a z2 az bz az1 a + ib
= a − ib
= T = 1 + 2 let =z x + iy
b z1 bz 2 az1 bz 2
a2 − b2 + 2abi
⇒ x + iy =
1z z a2 + b2
T=z+ =z+
zz | z |2 2
a −b 2
2ab
⇒x= ,y=
|z| = 1 ⇒ T = z + z = 2Re(z) 2
a +b 2
a + b2
2
⇒ T ∈ [–2, 2] 1 − ix
(b) = a − ib
1 + ix
Sol 20: (B) (p+q)3 + (pw+qw2)3 + (pw2+qw)3 1 − x2 − 2ix
⇒ = a − ib
= p3+q2+3p2q+3pq2+p3+q3+3pq2w5 1 + x2
1 − x2 2x
⇒a= ,b=
+3p2qw4+p3+q3+3p2qw5+3pq2w4 1+x 2
1 + x2
= 3(p3+q3) ⇒a2 + b2 = 1 B is correct
x
1 + i 3
( )
x a + ib
Sol 21: (A) 1 + i 3 x
2 ⇒
= =1 (c) = x − iy
2 a − ib
⇒ (–w)x = 1 (a + ib)2
⇒ = x − iy
⇒ x = 6, 12, 18, ……... a2 + b2
a2 − b2 + 2iab
It forms an AP ⇒ x + iy =
a2 + b2
a2 − b2 − 2iab
⇒ x + iy = ⇒ |x + iy| = 1
Multiple Correct Choice Type a2 + b2
y − ix
Sol 22: (B, C) x2+(p+iq)x + 3i = 0 (d) = y + ix
a + ib
a2 + b2 = 8 y 2 − x2 − 2ixy
⇒ = a + ib
(α + β) = –(p + iq) y 2 + x2
aβ = 3i y 2 − x2 + 2ixy
⇒a – ib = ⇒ |a – ib| =1
x2 + y 2
⇒ (α + β)2 = 8 + 6i = p2 – q2 + 2ipq
p2 – q2 = 8 Sol 25: (A, D) z = x + iy = r(cosθ + isinθ)
2pq = 6
z
= x + iy
= r(cos θ + isin θ)
⇒p = 3, q = +1 or p = –3, q = –1
θ θ
r eiθ /2
== r cos + isin for y > 0
2 2
Sol 23: (A, D) |z1| = |z2| ⇒ a2 + b2 = c2 + d2
z1 = a + ib, a > 0 1 + cos θ 1 − cos θ
= r +i
z2 = c + id, d < 0 2 2
z1 + z 2 (a + c) + i(b + d) r r +x +i r −x r +x +i r −x
= = = [A]
z1 − z 2 (a − c) + i(b − d)
2 r 2
(a + c) & (b + d) can be zero, so value can be zero Similarly
(a – c) & (b – d) can be simultaneously zero ⇒ purely 1
imaginary. = r + x − i r − x for y < 0
2
M a them a ti cs | 6.79
α – β = 6Z D is correct
⇒ arg z1 = arg z2
z1 z2 ⇒ α – β and z, both move on same circle
or z1 z2 = z1 z 2 ⇒ =
z1 z2
a + ib c + id Sol 30: (B, D) (in + i–n) is
⇒ = ⇒ –ad + bc = 0
a − ib c − id
22n + 22n 2.2n 2n+1
(A) = = (1 + i)2n =
2n (1 − i)2n (1 − i)2n
2 2n
Sol 27: (A, B, C)
(2i)n
= 2in= in + in
n−1
2
|a|2 + |b|2 – α β – β α =1 – α β – β α + |a|2 |b|2 ⇒ (x2 – y2)2 + (2xy – 4)2 = (x2 – y2)2 + (2xy +4)2
x + 1 + 2i − 2
⇒ log2 ≤1 Sol 37: (A, C)
2 −1 z2
⇒ |x + 1 + 2i| – 2 ≤ 2( 2 − 1 )
⇒ |x + 1 + 2i| ≤ 2 2 ⇒ (x + 1)2 + 4 ≤ 8
⇒ (x + 1)2 ≤ 4 ⇒ –2 ≤ x + 1 ≤ 2 z1 z3
n−1 z=0±i
Sol 35: (A, B, D) ∏ ( ω − zr )
r =1
Sol 39: (A, B, C, D) Z = a + bi =
(1 − ix ) (1 − ix )
ωn − 1 (1 + ix ) (1 − ix )
(w – z1) (w – z2) (w – z3)….. (w – zn-1) =
ω−1 1 − x2 − 2ix
a + bi =
If n is a multiple of 3, value can be zero as z1 can be w 1 + x2
or w2 ⇒ |z| = 1
If n = 3k + 1 value is 1 2x
Arg z = tan–1 2
If n = 3k + 2 value is 1 + w x −1
Arg (z) ≡ ( −π, π
Sol 36: (A, B, C) z = −4z
x – iy= –4x – 4iy
5x + i(3y) = 0
x = y = 0 (A is true)
M a them a ti cs | 6.81
i.e., | z1 | − | z 2 | ≤ z1 + z2
5
∴ z −3+ i
2
5 9
= z − 3 − 2i + 2i + i = (z − 3 − 2i) + i
2 2
9
≥ | z − 3 − 2i | − From (i), (ii) and (iii) A ∩ B ∩ C has only one point P
2
shown in the figure.
9 5
≥ 2− ≥
2 2 Sol 4: (C) z + 1 − i + z − 5 − i
2 2
5 5
⇒ z − 3 + i ≥ or |2z – 6 + 5i| ≥ 5 2 2
2 2 = z − ( −1 + i ) + z − (5 + i )
Sol 3: (B)
=A {z : Im ( z ) ≥ 1} A (-1,1)
( AB )
2
AP2 + BP2 =
B
= {z : z − 2 −=i 3}
{z : Re (1 − i) z + 2 }
2 2
=C ⇒ z +1 −i + z −5 −i
(=
6)
2
Taking z − 2 − i =3 = 36
Let z= z + iy
x + iy − 2 − i =3 Sol 5: (D) w − 2 − i < 3
| z − w | = length of diameter
6 . 8 2 | Complex Number
3 −i −1 − i 3 6
2kπ 2kπ
And = i = iω2 Sol 9: (D) ∑ sin − icos
2 2 k =1 7 7
6 i2kπ
∴ z = (– iω)5 + (iw2)5 = – iw2 + iω 6
2kπ 2kπ = −i e 7
∑ i cos 7 + isin 7 ∑
( ) k =1 k =1
= i (ω – w2) = i i 3 = – 3
(-z)
y’ r -
x
O -
triangles. (z)
z − z1 z − z1 z − z1 z − z1
=
⇒ or= 0
z 2 − z1 z 2 − z1 z 2 − z1 z 2 − z1
⇒ 2i | z | ( z − z ) =
0 4
⇒e=
5
| z | 0 or z=
⇒= −z 0
From figure, we can conclude that
⇒ Im ( z ) =
0
Re ( z ) ≤ 2
Also Im ( z ) ≤ 1
A → q, r And z ≤ 3
(B) |z + 4| + |z - 4| = 0
C → p, s, t
Its an equation of ellipse having
M a them a ti cs | 6.85
1 w w z − z1 z 2 − z1
(D ) z= w + w
=+ w
=+ w w
=+ =
w ww | w |2 z − z1 z2 − z1
Let w = a + i b z − z1 z − z1
⇒ 0
=
z = a + i b + a – i b = 2a z 2 − z1 z2 − z1
z ω ω2
z z + ω2 1 =0
z 1 z+ω
2π 2π )3, -5 )
=ω cos + i sin 2
3 3
1 ω ω2 5
∴ Minimum value =
2
1 z + ω2 1 =0
1 1 z+ω 5
2z − 6 + 5i = 2× =5
min 2
Expanding the determinant, we get
Sol 20: Given: a + b + c = x
(
z z + ω2 ) ( z + ω) − 1 − ω ( z + ω − 1) + ω (1 − z − ω ) =0
2 2
a + b ω + c ω2 = y
⇒ z z 2 + z ω + zω2 + ω2 − 1 − zω − ω2 + ω + ω2 − ω2 z − ω4 =0
a + b ω2 + c ω = z
⇒ z z2 =
0 | x |2 + | y |2 + | z |2 xx + yy+zz
=
2 2 2
|a| + |b| + |c| | a |2 + | b |2 + | c |2
⇒z=0
From fig. i π /3
ω =e
| z − z1 | + | z − z 2 |= | z1 − z 2 | −i π /3
ω =e
π
ω2 = e2i 3 = − ω
( )
= ( a + b + c ) a + b + c + ( a + b ω− ω c ) a + b ω + c ω2 ( )
Arg ( z − z=
1) Arg ( z 2 − z1 ) (
+ ( a − b ω + c ω) a + b ω + c ω 2
)
| a | + | b | + | c |2
2 2
6 . 8 6 | Complex Number
(
3 | a |2 + | b |2 + | c |2
= 3
) 1 1
4r 2
⇒ − z 0 − z0 =
α α
| a |2 + | b |2 + | c |2
1 z z
⇒ − 0 − 0 + | z 0 |2 =
4r 2
α α α α
1 a b
| α |2
Sol 21: (A) Given ω 1 c 2
α α = | α | ⇒ α =
ω2 α
ω 1
1 z0 α z0 α
1 a b ⇒ − − + | z 0 |2 =
4r 2
2 2 2
|α| |α| |α|
Determinant (D )= ω 1 c
ω2 ω 1
⇒ 1 − z 0 α − z0 α+ | α |2 | z 0 |2 = 4r 2 | z |2 …(ii)
|= (
D | 1 (1 − c ω ) − a ω − c ω 2
) + b (ω2
−ω 2
) Subtracting (ii) from (i), we get
⇒ | D | = 1 − ( a + c ) ω + acω 2
1− | α |2 + | α |2 | z 0 |2 − | z 0=
|2 r 2 4 | α |2 −1 ( )
| D | ≠ 0 , only when a = ω and c = ω
r2 + 2
(
∴ ( a,b,c ) ≡ ( ω, ω, ω) or ω, ω2 , ω ) 1− | α |2 +
2
(
| α |2 −= )
1 r 2 4 | α |2 −1 ( )
∴ Two non-singular matrices are possible. r2 + 2 2
(
⇒ | α |2 −1
2
)
−1
=
2
r 4 | α | −1 ( )
Sol 22: (D) a = z 2 + z + 1 = 0 r2 2
⇒ z2 − z + 1 − a =0 (
⇒ | α |2 −1 =
2
) 2
r 4 | α | −1 ( )
Discriminant < 0 {Since imaginary part of z us not zero}
⇒ | α |2 −1 = 8 | α |2 −2
1 – 4 (1 - a) < 0
⇒ 7| α |2 =1
⇒ 1 − 4 + 4a < 0
1
⇒ 4a < 3 ⇒ | α |=
7
3
⇒a <
4 Sol 24: (C,=
D) P {=
w :n n
1,2,3,..... }
3 1
Sol 23: (C) ( x − x0 ) + ( y − y 0 ) =
2 2
r w= + ⇒ | w |= 1
2 2
⇒ | z − z 0 |=
r ⇒ All the complex number belong to set P lie on circle
of unit radius, centre at origin.
α lies on it, then
| α − z 0 |=r
⇒ ( α − z 0 ) ( α − z0 ) =r 2
| |2 − z0 α − z 0 α + | z 0 |2 =
=α r2 ….(i)
Similarly,
( x − x0 ) + ( y − y 0 )
2 2
2r
=
⇒ | z − z 0 |=
2r
1
lies on it, then
α
1
− z0 =
2r
α
M a them a ti cs | 6.87
( ) (
x + 3 y + i 3x + y − 4
⇒ Im >0 )
4
3x + y
⇒ > 0 ⇒ 3x+ y > 0
4
{
S3 z ∈ C : Re ( z ) > 0 }
⇒ z lies in either first or fourth quadrant.
Now, points of intersection of circle x2 + y 2 =
16 and
=
(
ω4 ω2n − 1 ) 3 x+y =0
2π π π π
i (k + j) 12 ik i 12 i
⇒ zk .z=
j e 10 = 1 ∑ e 7 e 7 −1 ∑ e 7 −1
k =1 k =1
2π 2π = =
⇒ cos
10
( k + j) + i sin
10
(k + j ) =
1 3 i
( 4k −2) i
π 3 i
π
∑e 7 e 7 −1 ∑e 7 −1
If k + j = 10 m (multiple of 10), then above equation is k =1 k =1
True.
(q) z1 .z = zk 12
π π
2 πk
∑ 2 sin 14 12 × 2 sin
i (k −1) = k =1
= 4 =4
zk e 10 i 2π 3 π
⇒ z= = = e 10 π
z1 i
2π ∑ 2 sin 14 3 × 2 sin
4
e 10 k =1
z10 − 1 = ( z − 1) ( z − z1 )( z − z2 )( z − z3 ) .... ( z − z9 )
10
−z 2r + z 4s
P =
2
( ) ( ) ( −z ) z 2s + zr z 2s
r
Where zk represents the roots of equation z = 10
( −z ) z + z z
r 2s r 2s 2r 4s
z +z
=1 + z + z 2 + .... + z 9
z 2s ( −z ) + zr
2r 4s r
⇒
Z10 − 1
( z − z1 ) ( z − z2 ) ...... ( z − z9 )
= z +z
z −1 =
z 2s ( −z )r + zr z 2r + z 4s
= 1 + z + z 2 + z3 + .... + z 9
z10 − 1
( z − z1 ) ( z − z2 ) .... ( z − z9 )
= 1 0 2r 4s
z −1 Given that P2 − I = ⇒ z +z = −1
0 1
= 1 + 1 + 1 …… 10
= 10 2s r
And z ( −z ) + z =
r
0
10
(s) We know that sum of roots of z − 1 =0 is zero
−1 + i 3
2k π9 9
2k π Now, we have z = =ω
⇒ 1 + ∑ cos 0 ⇒ ∑ cos
= −1
= 2
k =1 10 k =1 10
⇒ ω2r + ω4s = −1 and ω2s ( −ω) + ωr = 0
r
9
2k π
⇒ 1 − ∑ cos =2
k =1 10 Only (r, s ) ≡ (1, 1 ) satisfies both the equation.
kπ Only one pair exists.
kπ kπ i
Sol 29: Given
= αk cos + i sin =e 7
7 7
1
Sol 31: (A, C, D) S =z ∈ C : z = , t ∈R, t ≠ 0
12 i
(k +1) π i
kπ a + ib t
12
∑ | αk +1 − αk | ∑e 7 −e 7
1 a−i b t
k =1
k =1 =z ×
= a + ib t a − ib t
3
3 i ( 4k −1 )
π ( 4k −2)
∑ | α 4 k −1 − α 4k −2 | ∑e 7
i
−e 7
a−i b t
k =1 ⇒z= = x + i y (Let)
k =1
a2 + b2 t2
a bt
=⇒x = and y
22 2
a +b t a + b2 t 2
2
M a them a ti cs | 6.89
x a ay
⇒ = ⇒t=
y bt bx
a
Substituting ‘t’ in x =
a + b2 t 2
2
a
x=
a2 y 2
a2 + b2
b2 x 2
ax2
⇒ x= ⇒ a2 x2 + a2 y 2 − ax =
0
a2 x2 + a2 y 2
x
⇒ x2 + y 2 − 0
=
a
2 2
1 2 1
x − + y =
2a 2a
1
Centre ≡ , 0
2a
1
Radius = , when a > 0 , b ≠ 0
2a
If=
b 0, a ≠ 0
y = 0 ⇒ x − axis
If=a 0, b ≠ 0
x = 0 ⇒ y − axis
2017-18 100 &
op kers
Class 11 T
By E ran culty
-JE Fa r
IIT enior emie .
S fP r es
o titut
Ins
MATHEMATICS
FOR JEE MAIN & ADVANCED
SECOND
EDITION
Exhaustive Theory
(Now Revised)
Formula Sheet
9000+ Problems
based on latest JEE pattern
2 2
(c) cosec A − co t A =
1 X’ X
(d) sinA
= cosec A tanA
= cot A cos
= A sec A 1 III quadrant IV quadrant
PLANCESS CONCEPTS
A crude way to remember the sign is “Add Sugar to Coffee”. This implies the 1st letter of each word gives
you the trigonometric functions with a +ve sign.
Eg. Add-1st word ⇒ 1st quadrant 1st letter=A ⇒ All are positive to-3rd word ⇒ 3rd quadrant 1st letter-t
⇒ tan θ (cot θ ) are positive.
Ravi Vooda (JEE 2009, AIR 71)
cos 1
6+ 2 10 + 2 5 3 5 +1
4 4 2 4
tan 0
2− 3 25 − 10 5 1
5−2 5
5 3
37º 45º 53º 60º 90º
sin ≈ 3/5 1 ≈ 4/5 1
3
2
2
2 (JEE MAIN)
1 + tan2 θ
1 − tan θ
(ii) =
1 + cot θ 1 − cot θ
2
Sol: (i) Simply by using Pythagorean and product identities, we can solve these problems.
1
(i) L.H.S. = (1 + tan2 A) + 1 + = sec A + (1 + cot A)
2 2
tan2 A
1 1 sin2 A + cos2 A
=sec2 A + cosec2=
A= + = sin2 θ + cos2 θ 1
cos2 A sin2 A sin2 A.cos2 A
M a them a tics | 7.3
1 1
= = = R.H.S. cos2 θ = 1 − sin2 θ
2
(
sin A 1 − sin A 2
) sin A − sin4 A
2
Hence proved.
1 + tan2 θ sec2 θ sin2 θ
(ii) L.H.S. = = = = tan2 θ … (i)
1 + cot θ cosec θ cos2 θ
2 2
2 2
2
2
1 − tan θ 1 − tan θ 1 − tan θ 1 − tan θ
Now, R.H.S. = = = = . tan θ = tan2 θ … (ii)
( )
1 − cot θ 1 − 1 tan θ − 1 − 1 − tan θ
tan θ tan θ
From (i) and (ii), clearly, L.H.S. = R.H.S. Proved.
(sin A ) + (cos A )
2 2
2 2
sin2 A
cos2 A sin4 A + cos4 A + 2sin2 A cos2 A − 2sin2 A.cos2 A
(i) += =
cos2 A sin2 A sin2 A cos2 A sin2 A cos2 A
(sin )
2
2
A + cos2 A − 2sin2 A cos2 A 1 − 2sin2 A cos2 A
= =
sin2 A cos2 A sin2 A cos2 A
1 2sin2 A cos2 A
= − = sec2 A cosec2 A – 2 = R.H.S. Proved.
2 2 2 2
sin A cos A sin A cos A
sin4 A
(ii) L.H.S. = sec4 A (1 – sin4 A) – 2 tan2 A = sec 4 A − − 2 tan2 A = sec 4 A − tan4 A − 2 tan2 A
4
cos A
( )
2
= 1 + tan2 A − tan4 A − 2 tan2 A = 1 + 2 tan2 A + tan4 A – tan4 A – 2 tan2 A = 1 = R.H.S. Proved.
1 + cos α 1 + sin α
(i) = cosecα + cot α (ii) = secα + tan α (JEE MAIN)
1 − cos α 1 − sin α
(1 + cos α )
2
1 + cos α 1 + cos α 1 + cos α
(i) L.H.S. = = × =
1 − cos α 1 − cos α 1 + cos α 1 − cos2 α
(1 + cos α=
)
2
1 + cos α 1 cos α
= = + = cosecα + cot
= α R.H.S. Proved.
sin α 2 sin α sin α sin α
1 + sin α 1 sin α
= = + = sec α + tan=
α R.H.S. Proved.
cos α cos α cos α
=
( ) ( )
sin2 A 1 − sin2 B − 1 − sin2 A sin2 B
=
sin2 A − sin2 A sin2 B − sin2 B + sin2 A sin2 B
=
sin2 A − sin2 B
= R.H.S. Proved.
cos2 A cos2 B cos2 A cos2 B cos2 A cos2 B
1 1 1 1
Illustration 5: Prove the following identities: − = − (JEE ADVANCED)
cosecθ − cot θ sin θ sin θ cosecθ + cot θ
1 1 2
Sol: By rearranging terms we will get + = , and then using Pythagorean identity
cosecθ − cot θ cosecθ + cot θ sin θ
we can solve this problem.
1 1 1 1
We have, − = −
cosecθ − cot θ sin θ sin θ cosecθ + cot θ
1 1 1 1 1 1 2
⇒ + = + ⇒ + =
cosecθ − cot θ cosecθ + cot θ sin θ sin θ cosecθ − cot θ cosecθ + cot θ sin θ
2cosecθ 2 1
= = = R.H.S. cosecθ = Proved.
1 sin θ sin θ
Alternative Method
R.H.S
=
1
−
1
= cosec θ −
( cosec θ − cot θ )
sin θ cosec θ + cot θ cosec2 θ − cot2 θ
= cosec θ − cosec θ + cot θ
= cot θ Proved.
M a them a tics | 7.5
(i)
(1 + cot A + tanA )( sinA − cos A ) = sin2 A.cos2 A
sec3 A − cosec3 A
sinA cos A
(ii) + 1
= (JEE ADVANCED)
sec A + tanA − 1 cosecA + cot A − 1
(i) L.H.S. =
(1 + cot A + tanA )( sinA − cos A )
sec3 A − cosec3 A
cos A sinA
1 + + ( sinA − cos A )
sinA cos A
=
( sec A − cosecA ) ( sec
( )
a3 − b3 = ( a − b ) a2 + ab + b2
2
A + sec A cosec A + cosec A 2
)
=
( sinA cos A + 1)( sec A − cosecA ) × sin2 A cos2 A sin2 θ + cos2 θ =1 = sin2 A cos2 A = R.H.S. Proved.
( sec A − cosecA )(1 + sinA cos A )
sinA cos A
(ii) L.H.S. = +
sec A + tanA − 1 cosec A + cot A − 1
sinA cosecA + sinA cot A − sinA + cos A sec A + cos A tanA − cos A
=
( sec A + tanA − 1)( cosecA + cot A − 1)
1 + cos A − sinA + 1 + sinA − cos A 2
= =
1 sinA 1 cos A 1 + sinA − cos A (1 + cos A − sinA )
+ − 1 + − 1
cos A cos A sinA sinA cos A sinA
2sinA cos A ( a + b )( a − b ) = a2 − b2
=
( 2 2
1 − sin A + cos A − 2sinA cos A )
1 1 2
L.H.S. = + sin θ.cos2 θ
sec2 θ − cos2 θ cosec2 − θ sec2 θ
1 1 cos2 θ sin2 θ 2
= + sin2 θ cos
= 2
θ + 2
sin θ cos θ
1 1 4 4
1 − cos θ 1 − sin θ
− cos2 θ − sin2 θ
cos2 θ sin2 θ
cos2 θ sin2 θ
= + ( )( )
sin2 θ cos2 θ a2 − b2 = a − b a + b
( )(
1 + cos2 θ 1 − cos2 θ
) (
1 − sin2 θ 1 + sin2 θ )( )
cos2 θ sin2 θ
= + sin2 θ cos2 θ
( )
1 + cos2 θ sin2 θ cos2 θ 1 + sin2 θ
( )
=
cos4 θ
+
=
sin4 θ (
cos4 θ 1 + sin2 θ + sin4 θ 1 + cos2 θ
) ( )
2 2
1 + cos θ 1 + sin θ 2
1 + cos θ 1 + sin θ2
( )( )
=
cos4 θ + sin2 θ cos4 θ + sin4 θ + sin4 θ cos2 θ
=
(
sin4 θ + cos4 θ + sin2 θ cos2 θ cos2 θ + sin2 θ )
(1 + cos θ)(1 + sin θ)
2 2
(1 + cos θ)(1 + sin θ)
2 2
=
(1 + cos θ)(1 + sin θ)
2 2
=
(sin θ + cos θ) =
2 2
− sin θ cos θ 2 2
1 − sin θ cos θ
=
2 2
1 − sin2 θ cos2 θ
= R.H.S. Proved.
1 + sin2 θ + cos2 θ + sin2 θ cos2 θ 1 + 1 + sin2 θ cos2 θ 2 + sin2 cos2 θ
3. TRANSFORMATIONS
Circular functions of the algebraic sum of two angles can be expressed as circular functions of separate angles.
sin (A ± B) = sin A cos B ± cos A sin B; cos (A ± B) = cos A cos B sin A sin B
Circular functions of half of an angle can be expressed as circular functions of the complete angle.
sin ( −θ ) = − sin θ π
cos + θ = − sin θ
2
cos ( −θ
= ) cos θ π
tan + θ = − cot θ
2
tan ( −θ ) = − tan θ sin ( π − θ=
) sin θ
π
sin − θ=
cos θ cos ( π − θ ) = − cos θ
2
π
cos − θ= sin θ tan ( π − θ ) = − tan θ
2
π
tan − θ= cot θ sin ( π + θ ) = − sin θ
2
π
sin + θ=
cos θ cos ( π + θ ) = − cos θ
2
tan ( π + θ=
) tan θ
π π
C + −D C − +D
π 2 2
Note: sinC + cosD = sinC + sin − D = 2sin .cos
2 2 2
1 1
sinA.sinB
=
2
{ }
cos ( A − B ) − cos ( A + B ) ; cos A.cosB
=
2
{
cos ( A − B ) + cos ( A + B ) }
sin ( A + B ) .sin ( A − B=
) sin2 A − sin2 B ; cos ( A + B ) .cos ( A − =
B ) cos2 A − sin2 B
1 1
sin2=
A
2
(1 − cos2A ) cos2=
A
2
(1 + cos2A )
1 − cos2A 3sinA − sin3A 3cos A + cos3A
tan2 A = ; sin3 A = ; cos3 A =
1 + cos2A 4 4
7 . 8 | Trigonometric Ratios, Identities and Equations
PLANCESS CONCEPTS
2 3
sin2n A
• cos A.cos2A.cos2 .A cos2= A......cos2n−1 A if A ≠ nπ
2n sinA
1 if A= 2nπ
−1 if A = (2n+ 1)π
• sin
= ( A1 + A2 + .... + An ) cos A1 cos A2 ....cos An (S1 − S3 + S5 − S7 + .... )
• cos
= ( A1 + A2 + .... + An ) cos A1 cos A2 ....cos An (1 − S2 + S4 − S6 .... )
S − S3 + S5 − S7 + ....
• tan ( A1 + A2 + .... + An ) =1
1 − S2 + S 4 − S6 + ....
Where,
S1 = tan A1 + tan A2 + …. + tan An = The sum of the tangents of the separate angles.
S2 = tan A1 tan A2 + tan A1 tan A3 + …. = The sum of the tangents taken two at a time.
S3 = tan A1 tan A2 tan A3 + tan A2 tan A3 tan A4 + ….. = Sum of tangents three at a time, and so on.
If A1 = A2 = ….. = An = A, then S1 = n tan A, S2 = nC2 tan2 A. S3 = nC3 tan3 A, …..
4. TRIGONOMETRIC IDENTITY
A trigonometric equation is said to be an identity if it is true for all values of the angle or angles involved. A given
identity may be established by (i) Reducing either side to the other one, or (ii) Reducing each side to the same
expression, or (iii) Any convenient, modification of the methods given in (i) and (ii).
A +B C C A +B
3. tan ( C + A ) = − cot (B + C ) 4.=
− tanB,cot A = cos sin
= ,cos sin
2 2 2 2
1
(i) sin ( 40º +θ ) cos (10º +θ ) − cos ( 40º +θ ) sin (10º +θ ) =
2
π π π π
sin ( θ + φ )
(ii) cos − θ cos − φ − sin − θ sin − φ= (JEE MAIN)
4 4 4 4
Sol: Use sum and difference formulae of sine and cosine functions.
(i) L.H.S. = sin ( 40º +θ ) cos (10º +θ ) − cos ( 40º +θ ) sin (10º +θ )
1
{ }
= sin ( 40º +θ ) − (10º +θ ) sin ( A
= − B ) sinA cosB − cos A sinB = sin30º
= = R.H.S.
2
Proved.
π π π π
(ii) L.H.S. = cos − θ cos − φ − sin − θ sin − φ
4 4 4 4
π π π
= cos − θ + − φ cos ( A= + B ) cos A cosB − sinA =
sinB cos − ( θ + φ )
4 4 2
π
= sin ( θ + φ ) =R.H.S. cos − θ= sin θ Proved
2
1 3π 5 π
Illustration 9: Find the value of tan ( α + β ) , given that cot α= , α ∈ π, and sec β = − , β ∈ , π .
2 2 3 2
(JEE MAIN)
tan α + tan β
Sol: As we know, tan ( α + β ) = , therefore by using product and Pythagorean identities we can obtain
1 − tan α. tan β
the values of tanα and tanβ .
1
Given, cot α = ⇒ tan α =2
2
5 25 4
Also, sec β = − . Then tan β = sec2 β − 1 =± − 1 =±
3 9 3
π 4
But β ∈ , π ⇒ tan β = − tan β is − ve in II quadrant
2 3
4 2
2 + − +
4 3 = 3 = + 2
Substituting tan α =2 and tan β = − in (1), we get tan ( α + β ) =
3 4 11 11
1 − (2) −
3 3
Illustration 10: Prove that: tan3A tan2A tanA = tan3A − tan2A − tanA (JEE MAIN)
tan α + tan β
Sol: Here we can write tan3A as tan ( 2A + A ) , and then by using tan ( α + β ) = we can solve this
problem. 1 − tan α tan β
tan2A + tanA
We have: 3A = 2A + A ⇒ tan3A = tan ( 2A + A ) ⇒ tan3A =
1 − tan2A tanA
7 . 1 0 | Trigonometric Ratios, Identities and Equations
⇒ tan3A (1 − tan2A tanA ) =tan2A + tanA ⇒ tan3A − tan3A tan2A tanA = tan2A + tanA
= 2(1 + cos2
= θ) 2.2cos
= 2
θ 4 cos2 θ = 2cos θ = R.H.S. Proved.
m 1 π
Illustration 12: If tan A = and tan B = , prove that A – B = (JEE ADVANCED)
m−1 2m − 1 4
tanA − tanB
Sol: Simply using tan (A – B) = , we can prove above equation.
1 + tanA. tanB
m 1
We have, tan A = and tan B =
m−1 2m − 1
tanA − tanB
Now, tan (A – B) = ... (i)
1 + tanA. tanB
π π π
⇒ tan ( A − B ) =
tan tan = 1 ⇒ A − B = Proved.
4 4 4
nsin α cos α
Illustration 13: If tan β = ; prove that tan ( α − β =
) (1 − n) tan α (JEE ADVANCED)
1 − nsin2 α
tan α − tan β
Sol: Same as above problem tan ( α − β ) = , therefore by substituting
1 + tan α tan β
nsin α cos α
tan β = , we can prove given equation.
1 − nsin2 α
tan α − tan β
L.H.S. = tan ( α − β ) = ... (i)
1 + tan α tan β
sin α nsin α cos α
−
nsin α cos α cos α 1 − nsin2 α sin α
Substituting tan β = in (i), we get L.H.S. = tan α =
2
1 − nsin α sin α nsin α cos α cos α
1+ .
cos α 1 − nsin2 α
M a them a ti cs | 7.11
=
( )
sin α 1 − nsin2 α − nsin α cos2 α
=
3
sin α − nsin α − nsin α cos α 2
=
cos α cos α
k sin α sin α
Illustration 14: If θ + φ = α and sin=
θ k sin φ , prove
= that tan θ = ,tan φ (JEE ADVANCED)
1 + k cos α k + cos α
α + β α −β α + β + 2γ
= 2cos cos + cos
2 2 2
α − β α + β + 2γ α + β + 2γ α − β
+ −
α + β
2 2
2 2
= 2cos 2cos cos
2 2 2
7 . 1 2 | Trigonometric Ratios, Identities and Equations
2π 4π
Illustration 16: If x cos
= θ y cos θ += z cos θ + , then show that xy + yz + zx = 0. (JEE ADVANCED)
3 3
2π 4π
Sol: Consider x cos
= θ y cos θ += z cos θ + = k , obtain the value of x , y and z in terms of k, and solve
L.H.S. of given equation. 3 3
2π 4π
Let x cos
= θ y cos θ += z cos θ + = k ... (i)
3 3
2π 4π
cos θ + cos θ +
1 cos θ 1 3 1 3
=
⇒ = , = ,
x k y k z k
xyz xyz xyz 1 1 1
Now, L.H.S. = xy + yz + zx = + + = xyz + +
z x y z x y
4π 2π
cos θ + cos θ +
3 + cos θ 3 xyz 4π 2π
= xyz + [Using
= (i)] cos θ + + cos θ + + cos θ
k k k k 3 3
xyz xyz
= − cos θ + cos θ = 0 = 0 ⇒ xy + zy + zx =
0 Proved.
k k
Sol: Multiply and divide L.H.S. by 2sinθ and apply sin ( 2θ=
) 2sin θ cos θ .
Here, we observe that each angle in L.H.S. is double of the preceding angle.
L.H.S. = cos θ cos2θ cos 4θ......cos2n−1 θ
=
1
2sin θ
(
( 2sin θ.cos θ ) cos2θ.cos 4θ.....cos2n−1 θ = 2 1 ( 2sin2θ.cos2θ ) cos 4θ......cos2n−1 θ
2 sin θ
)
1
= ( 2sin 4θ.cos 4θ ) cos8θ cos16θ......cos2n−1 θ sin=
2n θ 2 sin n θ cos n θ
23 sin θ
=
1
( 2sin8 θ.cos8 θ) cos16 θ......cos2n−1
θ =
1
2sin2n−1
θ cos2
=n−1
θ
( )
sin 2n θ
= R.H.S. Proved.
24 sin θ 2n sin θ 2n sin θ
acos φ + b θ a−b φ
Illustration 18: If cos θ = , prove that tan = tan (JEE ADVANCED)
a + bcos φ 2 a+b 2
θ φ
1 − tan2 1 − tan2
2 2 acos φ + b
Sol: Substitute cos θ = and cos φ = in given equation i.e. cos θ = .
2 θ 2 φ a + bcos φ
1 + tan 1 + tan
2 2
M a them a ti cs | 7.13
2 φ
1 − tan
a. 2 +b
1 − tan2
θ 1 + tan2 φ
acos φ + b
Now, cos θ = ⇒ 2 = 2 [Using (i)]
a + bcos φ θ 2 φ
1 + tan2 1 − tan
2 a + b 2
1 + tan φ
2
2
θ a 1 − tan2 φ + b 1 + tan2 φ φ φ
1 − tan2 2 2 a − atan2 + b + b tan2
⇒ 2 = = 2 2
2 θ 2 φ 2 φ 2 φ 2 φ
1 + tan a 1 + tan + b 1 − tan a + atan + b − b tan
2 2 2 2 2
θ φ φ
2 tan2
2
2atan2 − 2b tan2
2 2
( a − b ) tan2 2φ θ a−b φ
= ⇒ tan = tan Proved
2 2a + 2b a+b 2 a+b 2
X’ /6 /6 X
B
Y’
Figure 7.2
7 . 1 4 | Trigonometric Ratios, Identities and Equations
By Graphical approach:
The graph clearly shows that sin θ =0 at
-1
PLANCESS CONCEPTS
A trigonometric identity is satisfied by any value of an unknown angle while a trigonometric equation is
satisfied by certain values of the unknown.
Vaibhav Krishnan (JEE 2009, AIR 22)
1
Illustration 20: Principal value of cos θ = is: (JEE MAIN)
2
Sol: Here cos θ is ( + ) ve hence θ will lie in 1st or 4th quadrant.
cos θ is ( + ) ve ∴ θ will lie in the 1st or the 4th quadrant.
Y
B
For the 1st quadrant, we will select the anticlockwise direction and for the 4th quadrant,
we will select the clockwise direction.
3
π −π
As a result, in the first circle, two values and are found. X’
O
X
3 3 -
π π 3
Both and − have the same numerical value.
3 3
A
π
In such a case, will be selected as the principal value, as it has a positive sign. Y’
3
Figure 7.5
Illustration 21: Find the general solutions of the following equations:
3
(i) sin2θ =0 (ii) cos θ =0 (iii) tan2 2θ =0 (JEE MAIN)
2
Sol: By using above mentioned method of finding general solution we can solve these equation.
nπ
(i) We have, sin2θ = 0 ⇒ 2θ = nπ ⇒ θ = where, n = 0, ± 1, ± 2, ± 3......
2
nπ
Hence, the general solution of sin2θ =0 =
is θ ,n ∈ Z
2
6. PERIODIC FUNCTION
A function f (x) is said to be periodic if there exists T > 0 such that f (x + T) = f (x) for all x in the domain of definition
of f (x). If T is the smallest positive real number such that f (x + T) = f (x), then it is called the period of f (x).
( 2nπ + x ) sinx,cos=
We know that, sin= ( 2nπ + x ) cos x , =
tan (nπ + x ) tanx for all n ∈ Z
Therefore, sinx, cosx and tanx are periodic functions. The period of sinx and cosx is 2π and the period of tanx is π .
Function Period
tan ( ax + b ) , cot ( ax + b ) π / 2a
(a) Trigonometric equations can be solved by different methods. The form of solutions obtained in different
methods may be different. From these different forms of solutions, it is wrong to assume that the answer
obtained by one method is wrong and those obtained by another method are correct. The solutions obtained
by different methods may be shown to be equivalent by some supplementary transformations.
To test the equivalence of two solutions obtained from two different methods, the simplest way is to put
values of n = ……….. – 2, – 1, 0, 1, 2, 3, ……. etc. and then to find the angles in [0, 2π]. If all the angles in both
the solutions are same, the solutions are equivalent.
(b) While manipulating the trigonometric equation, avoid the danger of losing roots. Generally, some roots are
lost by cancelling a common factor from the two sides of an equation. For example, suppose we have the
equation tan x = 2 sin x. Here by dividing both sides by sin x, we get cos x = 1/2.
(c) While equating one of the factors to zero, we must take care to see that the other factor does not become
infinite. For example, if we have the equation sin x = 0, which can be written as cos x tan x = 0. Here we cannot
put cos x = 0, since for cos x = 0, tan x = sin x / cos x is infinite.
(d) Avoid squaring: When we square both sides of an equation, some extraneous roots appear. Hence it is
necessary to check all the solutions found by substituting them in the given equation and omit the solutions
that do not satisfy the given equation.
For example: Consider the equation, sin θ + cos θ =1 …. (i)
Squaring, we get 1 + sin2θ =1 or sin2θ =0 …. (ii)
This gives θ= 0, π / 2, π,3π / 2 ………
Verification shows that π and 3π / 2 do not satisfy the equation as sin π + cos π = −1, ≠ 1 and
sin3π / 2 + cos3π / 2 =−1, ≠ 1 .
The reason for this is simple.
The equation (ii) is not equivalent to (i) and (ii) contains two equations: sin θ + cos θ =1 and sin θ + cos θ = −1 .
Therefore, we get extra solutions.
Thus if squaring is a must, verify each of the solutions.
Some Necessary Restriction: If the equation involves tan x, sec x, take cos x ≠ 0. If cot x or cosec x appear, take
sin x ≠ 0. If log appears in the equation, then number > 0 and base of log > 0, ≠ 1.
Also note that f ( θ ) is always positive. For example, sin2 θ = sinθ , not ± sin θ .
Verification: Students are advised to check whether all the roots obtained by them satisfy the equation and lie in
the domain of the variable of the given equation.
M a them a ti cs | 7.17
If sin θ =0 θ = nπ
If tan θ =0 θ = nπ
If sin θ =1 θ= 2nπ + π / 2= ( 4n + 1) π / 2
If cos θ =1 θ= 2nπ
θ = nπ + ( −1 ) α where α ∈ −π / 2, π / 2
n
If sin θ
= sin α
If cos=
θ cos α θ= 2nπ ± α where α ∈ 0, π
If tan=
θ tan α θ = nπ + α where α ∈ −π / 2, π / 2
If sin2=
θ sin2 α θ = nπ ± α
2
If cos= θ cos2 α θ = nπ ± α
2
If tan= θ tan2 α θ = nπ ± α
If tan=
θ tan α θ= 2nπ + α
cos=
θ cos α
m−n m−n π
And cos θ =0 ⇒ cos 2 θ =cos 2
2
m−n 2p + 1
⇒ =θ ( 2p + 1) 2π , p ∈ Z=
⇒θ π, … (ii)
2 m−n
2rπ 2p + 1
From (i) and (ii), we have θ = or
= θ π where, m,n ∈ Z
m+n m−1
Illustration 23: Solve: 4 sinx cos x + 2sinx + 2cos x + 1 =0 (JEE ADVANCED)
Sol: Simply using algebra and method of finding general equation, we can solve above equation.
We have, 4 sinx cos x + 2sinx + 2cos x + 1 =0
⇒ 2sinx ( 2cos x + 1 ) + 1 ( 2cos x + 1 ) =
0 ⇒ ( 2sinx + 1 )( 2cos x + 1 ) =
0
1 1
⇒ 2sinx + 1 =0 or 2cos x + 1 =0 ⇒ sinx = − or cos x = −
2 2
1 π π
sinx = − = sin −
⇒ sinx ⇒ x= − The general solution of this is
2 6 6
−1 )
(
n+1
n π n+1 π
x = nπ + ( −1 ) − = nπ + ( −1 ) ⇒ x =
π n+ … (i)
6 6
6
1 π 2π 2π
and cos x = − ⇒ cos
= x cos π − = cos ⇒ x=
2 3 3 3
2π 1
The general solution of this is x= 2nπ ± i.e. x = 2π n ± ... (ii)
3 3
−1 )
(
n+1
From (1) and (2), we have π n + and 2π n ± 1 are the required solutions
6 3
8.1 Factorization
Trigonometric equations can be solved by use of factorization.
sin2 x
Illustration 24: Solve: ( 2sinx − cos x )(1 + cos x ) = (JEE MAIN)
Illustration 25: If sin 5x + sin 3x + sin x = 0 and 0 ≤ x ≤ π / 2 , then x is equal to. (JEE MAIN)
α+β α −β
Sol: By using sum to product formula i.e. sin α + sin β = 2sin cos .
2 2
0 ⇒ sin3x ( 2cos2x + 1 ) =
sin 5x + sinx = – sin 3x ⇒ 2sin3x cos2x + sin3x = 0
⇒ sin3x = −1 / 2 ⇒ x = nπ, x = nπ ± ( π / 3)
0,cos2x =
0
Illustration 26: Solve cos3x + sin2x − sin 4x = (JEE MAIN)
( 2n + 1) 6π ,n ∈ I or x = nπ + ( −1 )
π
n
⇒ x= ,n ∈ I
6
π n π
∴ Solution of given equation is ( 2n + 1 ) ,n ∈ I or nπ + ( −1 ) ,n ∈ I
6 6
Illustration 27: The number of solutions of the equation sin5x cos3x = sin6x cos2x, in the interval 0, π , is:
(JEE MAIN)
Sol: Simply by using product to sum method.
1
The given equation can be written as
2
( sin8x + sin2x )= 12 ( sin8x + sin 4x )
⇒ sin 2x – sin 4x = 0 ⇒ – 2 sin x cos 3x = 0
π
Hence sin x = 0 or cos 3x = 0. That is, x = nπ (n ∈ I ) , or 3x = kπ +
2
(k ∈ I ) .
π π 5π
Therefore, since x ∈ 0, π , the given equation is satisfied if x = 0, π, , or .
6 2 6
Hence, no. of solutions is 5.
7 . 2 0 | Trigonometric Ratios, Identities and Equations
c
If c ≤ a2 + b2 , then put = cos α , so that cos ( θ − φ=
) cos α
a2 + b2
⇒ ( θ − φ )= 2nπ ± α ⇒ θ= 2nπ ± α + φ
2
Illustration 28: Solve: sinx + 3 cos x = (JEE MAIN)
3 1 2 1 π π
⇒ cos x + sinx = = ⇒ cos x − =
cos
2 2 2 2 6 4
π π π π 5π π
⇒ x − = 2nπ ± ⇒ x= 2nπ ± + ⇒ x= 2nπ + , 2nπ − where n ∈ I
6 4 4 6 12 12
Note: Trigonometric equations of the form a sin x + b cos x = c can also be solved by changing sin x and cos x into
their corresponding tangent of half the angle. i.e t=tan x/2. The following example gives you insight.
x x
1 − tan2 2 tan
Sol: As we know, cos x = 2 and sinx = 2 . Therefore by substituting these values and solving we
2 x 2 x
1 + tan 1 + tan
will be get the result. 2 2
x 1 x 1 x −1 1
⇒ tan =⇒ tan = where, α tan ,n ∈ I
tan α , where tan α = ⇒ = nπ + α ⇒ x= 2nπ + 2α =
2 2 2 2 2 2
1 1 1 π π π π
⇒ sinx + cos x. = ⇒ cos x − = cos ⇒ x − = 2nπ ±
2 2 2 4 4 4 4
π
If we take the positive sign, we get x = 2nπ + , n ∈ I
2
If we take the negative sign, we get x = 2nπ, n ∈ I
PLANCESS CONCEPTS
(i) The answer should not contain such values of angles which make any of the terms undefined or
infinite.
(ii) Never cancel terms containing unknown terms on the two sides, which are in product. It may cause
loss of the general solution.
Suppose the equation is sin x = (tan x)/2. Now, cancelling sinx on both the sides, we get only
1
cos x = , sin x = 0 is not counted.
2
(iii) Check that the denominator is not zero at any stage while solving equations.
(iv) While solving a trigonometric equation, squaring the equation at any step must be avoided if
possible. If squaring is necessary, check the solution for extraneous values.
Suppose the equation is sin x = – sin x. We know that the only solution of this is sin x = 0 but on squaring,
we get (sin x)2=(sin x)2 which is always true.
(v) Domain should not change, it if changes, necessary corrections must be made.
Shivam Agarwal (JEE 2009, AIR 27)
7 . 2 2 | Trigonometric Ratios, Identities and Equations
∴ Solution set is, x | x = nπ + ( −1)n − π ∩ x | x = nπ ± π
3 6 2
Note: Here, unlike all other problems, the solution set consists of the intersection of two solution sets and not the
union of the solution sets.
x x
Illustration 32: sinx cos − 2sinx + 1 + sin − 2cos x ( cos x ) =
0 . Find the general solution. (JEE ADVANCED)
4 4
Sol: Open all brackets of given equation and then by using sum to product formula and method of finding general
solution we will get the result.
x x
sinx cos − 2sin2 x + cos x + sin cos x − 2cos2 x =
0
4 4
x 5x 5x
sin x + + cos x =
2 ⇒ sin + cos x =2 ⇒ sin = 1 and cos x = 1
4 4 4
5x 5x π π
sin = 1 ⇒ = 2nπ + =
⇒ x 2 ( 4n + 1 ) ; cos x = 1 ⇒ x = 2mπ
4 4 2 5
⇒ x = 2π,10π,18π ……..AP ⇒ x = 2π + m − 1 8π ( )
( )
⇒ x = 2π 4m − 3 m ∈ I
π
Illustration 33: Find the general solution of 2sin 3x + = 1 + 8 sin2x cos2 2x (JEE ADVANCED)
4
Sol: First square on both side and then using sum and difference formula we can solve this illustration.
2 2
π sin3x cos3x
= 1 + 8 sin2x cos 2x
2
4 sin2 3x + = 1 + 8 sin2x cos2 2x ⇒ 4 +
4 2 2
4 sin2 3x 4 cos2 3x
⇒ + 1 + 8 sin2x cos2 2x
+ 4 sin3x cos3x =
2 2
⇒ 2sin2 3x + 2cos2 3x + 2sin6x =
1 + 8 sin2x cos2 2x
⇒ 1 + 2 sin 6x = 8 sin 2x cos2 2x ⇒ 1 + 2 sin 6x = 4 sin 4x cos 2x
⇒ 1 + 2 sin 6x = 2 (sin 6x + sin 2x) ⇒ 1 = 2 sin 2x ⇒ sin 2x = ½
π sin x cos4 x 1
4
⇒ x= + 2nπ x= + = n ∈ I
12 2 3 5
9. SIMULTANEOUS EQUATIONS
Two equations are given and we have to find the value of variable θ which may satisfy both the given equations,
M a them a ti cs | 7.23
like cos=
θ cos α and sin θ= sin α
So, the common solution is θ= 2nπ + α , n ∈ I
Similarly, sin θ= sin α and tan=
θ tan α
So, the common solution is θ= 2nπ + α , n ∈ I
1
Illustration 34: The most general value of θ satisfying the equations cos θ = and tan θ = −1 is: (JEE MAIN)
2
Sol: As above mentioned method we can find out the general value of θ .
1 π
cos=
θ = cos
2 4
π 9π 7π
⇒ θ= 2nπ ± ; n ∈ I Put n = 1 θ = ,
4 4 4
−π 3π 7π
tan θ = −1 = tan ⇒ θ = nπ − π / 4 , n ∈ I Put n = 1, θ = ; Put n= 2, θ=
4 4 4
7π
The common value which satisfies both these equation is .
4
7π
Hence, the general value is 2nπ + .
4
1
Illustration 35: The most general value of θ satisfying equations sin θ = − and tan θ =1 / 3 are: (JEE MAIN)
2
Sol: Similar to above illustration.
We shall first consider values of θ between 0 and 2π
1 π π
sin θ = − = − = sin π + or sin ( 2π − π / 6 )
2 6 6
∴ θ = 7π / 6,11π / 6 ; tan
= θ 1/ =3 tan ( π /= 6 ) tan ( π + π / 6 )
∴ θ = π / 6,7π / 6
Thus, the value of θ between 0 and 2π which satisfies both the equations is 7π / 6 .
Hence, the general value of θ is 2nπ + 7π / 6 where n ∈ I
(d) If α is the least positive value of θ which satisfies two given trigonometric equations, then the general value
of θ will be 2nπ + α . For example, sin θ= sin α and cos= θ cos α , then, θ= 2nπ + α ,n ∈ I
sin (nπ + θ ) = ( −1) sin θ, n ∈ I
n
(i)
cos (nπ + θ ) = ( −1 ) cos θ, n ∈ I
n
(ii)
sin (nπ − θ ) = ( −1 ) sin θ, n ∈ I
n−1
(iii)
FORMULAE SHEET
Pythagorean Identities 2
+ 1 sec2 θ , 1 + cot=
sin2 θ + cos2 θ =1 , tan θ = 2
θ csc2 θ
Periodic Formulas
sin ( 2n π + θ=
) sin θ , cos ( 2n π + θ=
) cos θ , tan(n π + =
θ) tan θ ,
(If n is an integer)
θ) cot θ , sec ( 2n π + θ=
cot(n π + = ) sec θ , cosec ( 2n π =
+ θ ) cosec θ
sin ( 2θ=
) 2sin θ cos θ , sin3
=θ 3sin θ − 4 sin3 θ
Double and Triple Angle
Formulas
cos ( 2=
θ ) cos2 θ − sin2 θ cos3
= θ 4 cos3 θ − 3cos θ
π π π
Complementary angles sin ± θ=
cos θ , cos ±=θ sin θ , tan ±=θ cot θ ,
2 2 2
π π π
cot − θ=
tan θ , sec = − θ cosec θ , cosec − θ=
sec θ
2 2 2
1 1 1 − cos ( 2θ )
sin2=
θ 1 − cos ( 2θ ) , cos2=
θ 1 + cos ( 2θ ) , tan2 θ =
Half Angle 2 2 1 + cos ( 2θ )
sin ( α ± β=
) sin α cos β ± cos α sinβ ,
Sum and Difference
cos ( α ± β=
) cos α cos β sin α sinβ ,
tan α ± tan β
tan ( α ± β ) = ,
1 tan α tan β
M a them a ti cs | 7.25
1
sin α=
sin β cos ( α − β ) − cos ( α + β ) ,
Product to Sum 2
1
sin α =
cos β sin ( α + β ) + sin ( α − β ) ,
2
1
cos α=
cos β cos ( α − β ) + cos ( α + β ) ,
2
1
cos α=
sin β sin ( α + β ) − sin ( α − β ) ,
2
α+β α −β
Sum to Product sin α + sin β = 2sin cos ,
2 2
α +β α −β
sin α − sin β = 2cos sin
2 2
α+β α −β
cos α + cos β = 2cos cos
2 2
α +β α −β
cos α − cos β = −2sin sin
2 2
Solved Examples
JEE Main/Boards
Example 1: Solve: 2cos2θ + 2sin θ = 2 1 − cos x cos x + 1
⇒ 8=
: cos x ≠ 0, −1
1 1 + cos x cos x
Sol: Solve this example by using sin2=
θ 1 − cos ( 2θ ) .
2
or (8 – 8 cosx)cosx = (cosx + 1)2
θ 2 (1 − cos2θ=
2sin= ) 4 sin2 θ or 8 cosx – 8 cos2x = cos2x + 2cosx + 1
4
∴ 2=
sin θ 16 sin θ : sin θ ≥ 4
or 9cos2x – 6 cosx + 1 = 0
1
sin θ =0 or sin3 θ = ∴ sin θ =0 or or (3cosx –1)2 = 0
8
1 n π
, θ= mπ : m ∈ I or θ = nπ + ( −1 ) : n∈ I or cosx =
1 1
= cos β, ( say ) , β =cos−1
2 6 3 3
x ∴ x= 2nπ ± β : n ∈ I
Example 2: Solve: 8 tan2 = 1 + sec x
2
Sol: As we know that Example 3: Solve: sinx + cos x − 2 2 sinx cos x =
0
x 1 − cos x , substitute this to solve above
tan2 = Sol: We can write given equation a sinx + cosx =
2 1 + cos x
example. sinx + cos x = 2 sin2x , multiplying and dividing L.H.S. by 2 , we will
x get the result.
8 tan2 = 1 + sec x ... (i)
2
7 . 2 6 | Trigonometric Ratios, Identities and Equations
or
1
2 sinx +
1
cos x =
2 sin2x (
a2 − 2a + 1 + tan2 π ( a + x ) =0 )
2 2 2
or ( a − 1 ) + tan π ( a + x ) =
2
0
π
or sin x + =sin2x
4 ⇒ a−1 =0 and tan π ( a + x ) =
0
n π
⇒ 2x = nπ + ( −1 ) x + : n ∈ I ⇒ tan (1 + x ) π =0 ⇒ (1 + x ) π = nπ : n ∈ I
4
∴x = n−1 : n∈ I : a = 1
Example 4: Find the general value of θ which satisfies
3 1 Example 7: Solve the equation cos7 x + sin4 x =
1
both the equations cos θ = − and sin θ = .
2 2
Sol: Use the method for simultaneous equations. Sol: Here c os7 x ≤ cos2 x and sin4 x ≤ sin2 x , hence by
solving this we will get the result.
3 5π 7π
cos θ = − ⇒θ= , ….
2 6 6 cos7 x ≤ cos2 x and sin4 x ≤ sin2 x
1 π 5π ∴
= 1 cos7 x + sin4 x ≤ cos2 x + sin2 x =
1
sin θ = ⇒ θ = , .....
2 6 6 ∴ c os7 x =
cos2 x and sin4 x = sin2 x
Hence, the general solution is given by
5π
(
cos7 x = cos2 x ⇒ cos2 x cos5 x − 1 =
0 )
θ 2nπ+
= , n∈ I ∴ cos
= x 0 or cos
= x 1
6
1
x2 + ≥ 2∀x with equality for Example 8: Solve for x and y:
x2
x 12sinx + 5cos x = 2y 2 − 8y + 21
x2 = 1 alone. Since 2cos2 sin2 x ≤ 2 ,
2
Sol: Multiply and divide L.H.S. by 13 and solve to get
x 1
∴ 2cos2 sin2 x =
x2 + holds only the result.
2 x2
x 12sinx + 5cos x = 2y 2 − 8y + 21
If x2 =1 ∴ x = 1 and cos sinx = ±1
2
1C
12 5
(
∴ 122 + 52 sinx + cos x = 2 y 2 − 4y + 4 + 13
13 13
)
i.e. cos sin1C = ±1 , which is not true.
2
or 13cos ( x − α ) 2 ( y − 2) + 13
2
=
Hence, the given equation has no solution.
5 12
: cos α
= and
= sin α
13 13
Example 6: Determine ‘a’ for which the equation
Thus, cos ( x − α ) =1 and y = 2 or
( )
a2 − 2a + sec2 π ( a + x ) =
0 has solutions and find the
solutions. x − α= 2nπ : n ∈ I and y = 2
Sol: By using algebra and tangent of angle we can get ∴ x= 2nπ + α : n ∈ I and y = 2
the result.
M a them a ti cs | 7.27
JEE Advanced/Boards ⇒ 2x = nπ + ( −1 )
n π
: n ∈ I General solution is
6
Example 1: Solve for x, y: x cos3 y + 3x cos y sin2 y =
14 nπ n π
3 2 x= + ( −1 ) ,n ∈ I
x sin y + 3x cos y sin y =
13 2 12
Sol: Divide equation 1 by equation 2 and then by Example 3: Solve: 3tan ( θ − 15º
= ) tan ( θ + 15º )
applying componendo and dividendo we can solve this
problem. tan ( θ + 15º ) 3
Sol: We can write given equation as = ,
We note that, “x = 0; sin y = 0 or cos y = 0” do not yield tan ( θ − 15º ) 1
a solution to given system. hence by applying componendo and dividendo we will
3 2 get the result.
cos y + 3cos y sin y 14
=
3 2
sin y + 3cos y sin y 13 Given, 3tan ( θ − 15º
= ) tan ( θ + 15º )
By componendo and dividendo, tan ( θ + 15º ) 3
or =
3 2 2
cos y + 3cos y sin y + 3cos y sin y + sin y 3
tan ( θ − 15º ) 1
3 2 2 3
cos y + 3cos y sin y − 3cos y sin y − sin y
tan ( θ + 15º ) + tan ( θ − 15º ) 4
3 or =
=
14 + 13 cos y + sin y
or (3)
3 tan ( θ + 15º ) − tan ( θ − 15º ) 2
= 27
=
14 − 13 cos y − sin y
sin ( θ + 15º +θ − 15º ) π
cos y + sin y 1 + tan y 3 or = 2 or sin2θ = 1 = sin
or = 3 or = sin ( θ + 15º −θ + 15º ) 2
cos y − sin y 1 − tan y 1
1 π nπ n π
⇒ 2θ = nπ + ( −1 ) + ( −1 )
n
tan y = = tan α ; y = nπ + α : n ∈ I : n ∈ I ∴=
θ : n∈ I
2 2 2 4
Since sin y and cos y have signs, we have the following
cases: Example 4: Find value of θ for sin2
= θ cos3θ , where
0 ≤ θ ≤ 2π ; Use the above equation to find the value
1 2 of sin 18º.
(i) sin y = and cos y = ; then
5 5
π
8 2 1 Sol: Here as we know= sin θ cos − θ , hence we can
x +3 . = 14 ⇒ x = 5 5 2
5 5
5 5 write given equation as cos3
=θ cos ( π / 2 ) − 2θ . { }
1 2
(ii) sin y = − and cos y = − ; then Therefore by comparing their angle we will get the
5 5 result.
−8 −2 1
x + 3 = 14 ⇒ x =−5 5 The given equation is sin2
=θ cos3θ or, cos3
=θ sin2θ
5 5 5 5
or, cos3
= {
θ cos ( π / 2 ) − 2θ }
7
Example 2: Solve: sin4 x + cos4 x =
2
sinx cos x or, 3θ= 2nπ ± {( π / 2) − 2θ} where n ∈ I
Taking + sign, 3θ= 2nπ + {π / 2 − 2θ}
Sol: By substituting 2sinx cos x = t and solving we will
be get the result. or 5=
θ ( 4n + 1)( π / 2)
7
sin4 x + cos4 x = sin2x ; ∴ sin2x > 0 θ ( 4n + 1 )( π / 10 ) , where n ∈ I
or, = … (i)
4
Again putting n = 0, ±1, ±2,......, in (ii) the value of θ in only for those (integral) values of k, n, m for which the
corresponding right members of the relations (i) are
the interval 0 ≤ θ ≤ 2 π is 3π / 2 i.e. 270º only.
positive.
Hence the required values of θ in 0 ≤ θ ≤ 2π are 18º, It is easy to see that of the first equation of (i) that
90º, 162º, 234º, 270º, 306º. is positive for integer k > 0, the right side of second
equation of (i) is positive for integral n > 0; and the
Example 5: Solve the equation: right side of the third equation of (i) is positive for
m ≥ 0 . Thus, we have to solve (i) only for the indicated
( ) ( ) ( )
cos π3x − 2cos2 π3x + 2cos 4 π3x − cos 7π3x ( ) values of k, m, n. The resulting values of x are then the
roots of the original equation:
= sin ( π3 ) + 2sin ( π3 ) − 2sin ( 4 π3 )
x 2 x x
1 2k
x log3 − + =
= ;k 1,2,......
+2sin ( π3 ) − sin ( 7π3 )
x +1 x
6 6
n 1 1
= x log
= 3 ,n 1,2,...... = log3 + m= 0,1,2
Sol: Substitute π 3x =
y , and then by using sum to 2 8 m
product formula we can solve this equation.
Example 6: Solve the equation:
Denote π 3x by y to get
Transposing all terms to the left side, Sol: Solve it like algebra by using product and
Pythagorean identity.
we have, ( cos y − cos7y ) + ( sin7y − sin y )
The given equation is
+2 ( cos 4y + sin 4y ) − 2 ( sin3y + 1 ) =
0
1
17 sec2 x + 16 tanx sec x − 1
or, 2sin 4y sin3y + 2cos 4y sin3y 2
+2 ( cos 4y + sin 4y ) − 2 ( sin3y + 1 ) =
0 = 2 tanx (1 + 4 sinx ) … (i)
[Use C & D formulae]
⇒ 17 sec2 x + 8 tanx sec x − 16 × 1
or, 2sin3y ( sin 4y + cos 4y ) + 2 = 2 tanx (1 + 4 sinx )
+2 ( cos 4y + sin 4y ) − 2 ( sin3y + 1 ) =
0
⇒ 17
⇒ 17 sec
2
sec2 xx +
+88 tanx
tanx sec
sec xx −− 16 (
16 ×sec
⇒ 17 sec2 x + 8 tanx sec x − 16 × 1
2 2
1 x − tan x )
or, ( sin3y + 1 )( sin 4y + cos 4y − 1 ) =
0 tanx(1 sinx))
= 2
=
=
tanx
22tanx (1+++444sinx
1 ( sinx )
This enables us to write down three groups of solutions:
π 2kπ nπ π mπ
⇒ 16 sec222 xx ++ 88tanx
17tan tanxsec ((
16 2sec
secxx +− sec x 2
2
x − tan2 x
⇒ 17 sec x + 8 tanx sec x − 16 sec x − tan2 x ))
y1 =− + , y 2= ,y= + = 2 tanx (1 + 4 sinx )
6 3 2 3 8 2 = 2 tanx (1 + 4 sinx )
where k, n and m are arbitrary integers. Recalling ⇒ 16 tan22 x + 8 tanx sec x + sec22 x
⇒ 16 tan x + 8 tanx sec x + sec x
that y = π3x , we obtain an infinity of equations for
determining the roots of the original equations:
= 2 tanx (1 + 4 sinx )
1 2k n
3x =− + , k = 0, ± 1, ± 2 ,…… 3x= ,n= 0, ± 2,
( 4 tanx ) + 2 × 4 tanx.sec x + sec2 x
2
6 3 2
1 m
… = + ,m = 0, ± 1, ± 2, .... = 2 tanx (1 + 4 sinx )
8 2
4 tanx + sec= x 2 tanx (1 + 4 sinx )
The equation 3x = a has a (unique) root only
for positive a and it is given by the formula
x = log3 a . Therefore, the equation (i) has a solution
M a them a ti cs | 7.29
1 ⇒ cos2 2nπ =1
cos2 x
1 1
2 y2 − y + + ≤ 1 ∴ x= 2nπ
4 4
1 2 2
cos2 x
1 1
2 y − + ≤ 1
2 2
JEE Main/Boards
Q.10 Solve the equation: Q.18 Solve the equation tanx + cot x =
2
4 sinx cos x + 2sinx + 2cos x + 1 =0
M a them a ti cs | 7.31
Q.24 Assuming that a person of normal sight can read = 3sinA ⋅ sinB ⋅ sinC then
print at such a distance that the letters subtend an
(A) ∆ABC may be a scalene triangle
angle of 5’ at his eye, find the height of the letters that
he can read at a distance of 12 meters. (B) ∆ABC is a right triangle
(C) ∆ABC is an obtuse angled triangle
Q.25 Solve the equation 4 cos2 x sinx − 2sin2 x =
3sinx (D) ∆ABC is an equilateral triangle
2cos β − 1 α β
Q.30 Does the equation sin4 θ − 2sin2 θ − 1 = 0 has a Q.7 If cos α = then tan cot has the value
2 − cos β 2 2
solution?
equal to, where ( 0 < α < π and 0 < β < π )
Q.3 The number of solution of the equation Q.9 Given A = sin2 θ + cos4θ, then for all real values
( )
sin ex= 5x + 5− x is (1991)
of θ
3
(1980)
(A) 1 ≤ A ≤ 2 (B) ≤ A ≤ 1
(A) 0 (B) 1 (C) 2 (D) Infinitely many 4
13 3 13
(C) ≤ A ≤1 (D) ≤A≤
Q.4 The number of integral values of k for which the 16 4 16
equation 7 cos x + 5sinx = 2k + 1 has a solution, is
(2002) Q.10 The expression (1986)
(A) 4 (B) 8 (C) 10 (D) 12
3π
3 sin4 − α + sin4 ( 3π + α )
2
Q.5 The set of values of θ satisfying the in equation
2sin2 θ − 5sin θ + 2 > 0 , where 0 < θ < 2π , is (2006) π
−2 sin6 + α + sin6 (5π − α ) is equal to
π 5π π 5π 2
(A) 0, ∪ ,2π (B) 0, ∪ ,2π
6 6 6 6 (A) 0 (B) 1
α β β γ γ α
(B) tan tan + tan tan + tan tan =1 Q.14 If 0 ≤ x < 2 π , then the number of real values of
2 2 2 2 2 2
x, which satisfy the equation cosx + cos2x + cos3x +
(C) tan
α β γ α β γ cos4x = 0 is: (2016)
+ tan + tan =− tan tan tan
2 2 2 2 2 2
(A) 5 (B) 7 (C) 9 (D) 3
(D) None of these
7 . 3 4 | Trigonometric Ratios, Identities and Equations
JEE Advanced/Boards
Exercise 1 π
Q.13 2sin 3x + = 1 + 8sin2x.cos2 2x
4
Q.1 Solve the equation: sin5x = 16 sin5 x
Q.14 Find the number of principal solutions of the
equation.
Q.2 Find all the solutions, of 4cos2xsinx ﹘ 2sin2x = 3sinx
sinx − sin3x + sin5x = cos x − cos3x + cos5x
3x x
equation cos3x + cos2x = sin + sin .
2 2
Q.18 Find the value of θ , which satisfy
2 2
Q.7 Solve: tan 2x + cot 2x + 2 tan2x + 2cot 2x =
6 3 ﹘ 2cos θ ﹘ 4sinθ ﹘ cos2θ + sin2θ = 0.
Column I Column II
(A) sinα + sinβ
(p) 2b Exercise 2
a+c
nπ π
(C) 2 tanx − 1 + 2cot x − 1 =2 (r) + ,n ∈ I Q.3 The number of solutions of the equation,
4 8 5
(A) A rational number (B) Irrational number (A) 2cos36° (B) 2cos144°
(
(C) 2 3 + 2 3 ) (
(D) 2 3 − 3 ) (C) 2sin18° (D) None of these
Q.5 If A = 580º then which one of the following is true? Multiple Correct Choice Type
Q.8 If 𐐭 is eliminated from the equations x=a cos(𐐭-∝) Previous Years’ Questions
x2 y 2 2xy
and y=b cos(𐐭- β ) then + − is
cos(α − β)isequalto
a2 b2 ab Q.1 Show that the equation esin x − e− sin x − 4 =
0 has no
equal to real solution. (1982)
(A) cos2 (α − β) (B) sin2 (α − β)
Q.2 Find the values of x ( −π, π ) which satisfy the
(C) sec2 (α − β) (D) cosec2 (α − β)
1 + cos x + cos2 x +.....
equation 2 =4 (1984)
Q.9 The general solution of the trigonometric equation
tanx + tan2x + tan3x =tanx.tan2x.tan3x is Q.3 If exp {(sin x + sin
2 4
) }
x + sin6 x + ..... + ∞ loge 2 ,
π 2
(A) x = nπ (B) nπ ± satisfies the equation x − 9x + 8 =0 , find the value of
3
cox π
nπ ,0 < x < . (1991)
(C) x= 2nπ (D) x = Where n ϵ I cos x + sinx 2
3
Q.10 Number of principal solutions of the equation Q.4 Determine the smallest positive value of x (in
tan3x-tan2x-tanx=0 , is degree) for which tan (x + 100º) = tan (x + 50º) tan (x)
tan (x – 50º) (1993)
(A) 3 (B) 5 (C) 7 (D) More than 7
M a them a ti cs | 7.37
(C) x = y (D) x ≠ 0, y ≠ 0
Q.17 Let P = {θ : sin θ − cos θ = 2 cos θ }
and Q = {θ : sin θ + cos θ = }
2 sin θ be two sets. Then
sin4 x cos4 x 1 (2011)
Q.10 If + = , then (2009)
2 3 5 (A) P ⊂ Q and Q − P ≠ ∅ (B) Q ⊄ P
2 2 sin8 x cos8 x 1 (C) P ⊄ Q (D) P = Q
(A) tan x = (B) + =
3 8 27 125
Q.18 The positive integer value of n > 3 satisfying the
1 2 sin8 x cos8 x 2 equation
(C) tan x = (D) + =
3 8 27 125
1 1 1
= + is (2011)
π π 2π 3π
Q.11 For 0 < θ < , the solution(s) of sin sin sin
2 n n n
6 (m − 1) π cosec θ + mπ = 4
∑ cosec θ + 4
4
2 is/are
m=1
(2009)
π π π 5π
(A) (B) (C) (D)
4 6 12 12
7 . 3 8 | Trigonometric Ratios, Identities and Equations
Q.19 Let θ, φ ∈ 0, 2 π be such that Q.21 The number of distinct solution of the equation
θ 5
2 cos θ (1 − sin=
θ
φ ) sin2 θ tan + cot cos φ − 1 cos2 2x + cos4 x + sin4 x + cos6 x + sin6 x =
2
2 2 4
,
In the interval 0,2 π is (2015)
3
tan ( 2 π − θ ) > 0 and −1 < sin θ < −
2
π
Then φ cannot satisfy (2012) S x ∈ ( −π, π ) : x ≠ 0 , ± .
Q.22 Let =
2
π π 4π
(A) 0 < φ < (B) < φ < The sum of all distinct solutions of the equation
2 2 3
3 sec x + cosec x + 2 ( tan x − cot x ) =
0 in the set S is
4π 3π 3π equal to (2016)
(C) <φ< (D) < φ < 2π
3 2 2
7π 2π 5π
(A) − (B) − (C) 0 (D)
9 9 9
Q.20 For x ∈ ( 0, π ) , the equation sin x + 2 sin 2x – sin
3x = 3 has (2014)
(A) Infinitely many solutions
(B) Three solutions
(C) One solution
(D) No solution
PlancEssential Questions
JEE Main/Boards JEE Advanced/Boards
Exercise 1 Exercise 1
Q.11 Q.17 Q.24 Q.7 Q.12 Q.17
Q.29 Q.21 Q.24 Q.26
Exercise 2 Exercise 2
Q.6 Q.9 Q.14 Q.4 Q.7 Q.8
Q.17 Q.19 Q.11
Q.13 Q.14
Answer Key
JEE Main/Boards
5π π
Q.15 2nπ + ,2nπ − ,n ∈ I
Exercise 1 12 12
nπ π 2nπ 3π
nπ n π 2nπ 2π Q.16 (i) − ,n ∈ I (ii) ± ,n ∈ I
Q.1 (i) + ( −1 ) ,,nn∈∈II (ii) ± ,,nn∈∈II 3 12 5 20
2 12 5 15
n π nπ
Q.17 (i) n π + ( −1 ) ± (ii)
= x ,n ∈ I
π 2π 6 3
Q.2 2nπ ± ,2nπ ± , n∈I
3 3 nπ
(iii) nπ,nπ ± ,n ∈ I
nπ π 3
Q.3 + ,n ∈ I
3 9 π
Q.18 nπ + ,n ∈ I
4
nπ n π
Q.4 + ( −1 ) ,n ∈ I π
2 4 Q.19 nπ,nπ ± ,n ∈ I
3
1 n π π π π
Q.5=
x nπ + ( −1 ) + 2mπ ± ,n,m ∈ I, Q.20(i) 2nπ ± + ,n ∈ I i.e.
2 3 3 4 4
1 π n π
π π π
2nπ or 2nπ + ,n ∈ I (ii) 2nπ ± + ,n ∈ I
=y 2mπ ± − nπ − ( −1 ) ,n,m ∈ I 2 3 6
2 3 3
7π 2π
Q.6 x = nπ,n ∈ I (iii) 2nπ + ,n ∈ I (iv) 2nπ,2nπ + ,n ∈ I
12 3
1 Q.21 (i) 30º (ii) 144º (iii) 68º 43’ 37.8”
Q.7 tan α =
2 π π π
Q.22 30º, 60º, 90º and , ,
Q.8 x =
=
2aπ
or x
( 2b + 1) π ,a,b ∈ I
6 3 2
m+n m−n Q.23 3466.36 km
nπ mπ π Q.24 17.4 cm
Q.9 =
x , − ,n ∈ I
2 2 8 3π n π
Q.25 x = nπ,nπ + ( −1 ) ,nπ + ( −1 )
n+1
π 2π 10 10
Q.10 x = nπ − ( −1 )
n
,2nπ ± ,n ∈ I
6 3 where n = 0, ±1, ±2,.......
π π π −π 3
Q.11 x = nπ + ( −1 )
n
or (2n+ 1) ,n ∈ I Q.26
= θ , , π − cos−1
6 2 3 3 5
2
3 Q.27 x = nπ, x = nπ ± α where sin α =
Q.12 x = nπ + tan ( − ) or x = n π + tan−1 (2)
−1
5
4
π
nπ π Q.28 θ = nπ ±
Q.13 nπ or ± , where n ∈ I 6
3 9
π Q.30 NO real Solution
Q.14 nπ,nπ − , where n ∈ I
4
7 . 4 0 | Trigonometric Ratios, Identities and Equations
Exercise 2
Q.19 D Q.20 A
Q.13 A Q.14 B
Exercise 1 Q.13 x = n π +
π
;n ∈ I
12
π
Q.1 x = nπ or x = nπ ± Q.14 10 solutions
6
π n 3π π
Q.2 nπ;nπ + ( −1 ) or nπ + ( −1 )
n
Q.15 x= 2nπ +
10 10 12
Q.3 2 Q.16 30º, 45º, 90º, 135º, 150º
±π −π
Q.4 , , ±π Q.17 No Solution
3 2
π
π n n π Q.18 θ= 2nπ or 2nπ + ;n ∈ I
Q.5 x = 2nπ or x = nπ + ( −1 ) − or x = nπ + ( −1 ) 2
2 6
Q.19 4950 π
π 5π 9π 13π
Q.6 , ,π ,
7 7 7 7 Q.20 72º
Q.21 13
nπ π nπ π nπ 5π
Q.7 x = + , − , − π
2 8 2 24 2 24 Q.23 2nπ + or ( 2n + 1 ) π − tan−1 2;n ∈ I
4
π
Q.8 x= 2nπ −
2 3 1 nπ π
2 2
Q.24 (a) − ; (b) k ∈ −1, − ; (c) =
x ±
π 4mπ π 2 2 2 6
x 4nπ − or=
Q.9 = x +
2 3 2
Q.25 A → r; B → s; C → p; D → q
where m,n ∈ W
Q.26 A → s; B → p; C → q; D → r
Q.10
= x 0 or a < −1
Q.27 A → q; B → s; C → p; D → r
2
Q.11 α − 2π; α − π, α + π, where tan α =
3
M a them a ti cs | 7.41
Exercise 2
π 2π 3 −1 π π
Q.2 ± , ± Q.3 Q.4 30º Q.5 Q.6 θ = ± Q.7 A C
3 3 2 2 2 3
Solutions
JEE Main/Boards 2π
x = 2nπ ± ,n∈I
3
Exercise 1 2π
∴ 5θ = 2nπ ± ,n∈I
3
1
Sol 1: (i) sin 2 θ = 2n π 2 π
2 ⇒θ= ± ,n∈I
5 15
1
General solution of sinx = is
2 Sol 2: 7cos2θ + 3sin2θ = 4
π
x = nπ + (–1)n 6 , n ∈ I Since sin2θ + cos2θ = 1
π ∴ 4cos2θ + 3(cos2θ + sin2θ) = 4
∴ 2θ = nπ + (–1)n 6
⇒ 4cos2θ + 3 = 4
nπ π 1 1 1
⇒θ= + ( −1)n ,n∈I ∴ cos2θ = or cosθ = , −
2 12 4 2 2
1
(ii) cos 5 θ = – π 2π
2 ∴ θ = 2nπ ± , 2nπ ± ,n∈I
3 3
1
General solution of cosx = − is
2
7 . 4 2 | Trigonometric Ratios, Identities and Equations
∴x=
2a
π or
(2b+ 1)
π a,b ∈ I ∵ cos2x + sin2x = 1
m+n m−n
∴ 2sin2x–5sinxcosx–8cos2x+2cos2x+2sin2x = 0
⇒ tan2x(1 + tan2x) = 0 3
⇒ sinx = − cosx or sinx = 2cosx
4
tan2x = 0 or tan2x = –1
3
π or tanx = − or tanx = 2
2x = nπ or 2x = mπ – n, m ∈ I 4
4 3
nπ mπ π x = nπ + tan–1 − or x = nπ + tan–1(2)
∴x= or − n, m ∈ I 4
2 2 8
Sol 13: 4sinx sin2x sin4x = sin3x
Sol 10: 4sinx cosx + 2sinx + 2cosx + 1 = 0
2sinA sinB = cos(A – B) – cos(A + B)
⇒ (2sinx + 1) × (2cosx + 1) = 0
2sinA cosB = sin(A + B) + sin(A – B)
1 1
⇒ sin x = − or cosx = − ∴ 2sinx[2sin2x sin4x] = 2sinx[cos2x – cos6x]
2 2
π 1 ⇒ 2sinx cos2x – 2sinx cos6x = sin3x
∴ x = nπ– (–1)n (when sin x = − )
6 2 ⇒ sin3x + sin(–x) – [sin7x + sin(–5x)] = sin3x
2π 1 ⇒–sinx = sin7x – sin5x
or x = 2nπ ± (when cos x = − )
3 2
–sinx = 2cos6x sinx
Sol 11: tanx + secx = 2cosx C +D C −D
sinC + sinD = 2sin cos
sinx 1 2 2
tanx = , sec x =
cos x cos x ⇒ sinx(2cos6x + 1) = 0
1 + sin x 1
sinx = 0 or cos6x = −
∴ = 2 cos x 2
cos x
2π nπ π
⇒ 1 + sin x = 2cos2x x = nπ or 6x = 2nπ ± ⇒ x = nπ, ±
3 3 9
[∵ sin2x + cos2x = 1]
7 . 4 4 | Trigonometric Ratios, Identities and Equations
cos2 θ + sin2 θ + 2sin θ cos θ (ii) tanx + tan2x + tan3x = tanx tan2x tan3x
∴ (1 + sin2θ) =
cos2 θ − sin2 θ ⇒ tanx + tan2x + tan3x (1 – tanx tan2x) = 0
1 + sin2θ tanx + tan2x
(1 + sin2θ) = ⇒ = –tan3x
cos2θ 1 − tanx tan2x
⇒ cos2θ (1 + sin2θ) – (1 + sin2θ) = 0 tanA + tanB
∵ = tan(A + B)
(cos2θ – 1) (sin2θ + 1) = 0 1 − tanA tanB
sin2θ = –1 or cos2θ = 1 ∴ tan(x + 2x) = –tan3x
π 2tan3x = 0
∴ 2θ = 2nπ – or 2θ = 2nπ, n ∈ I
2
i.e. tan3x = 0
π
∴ θ = nπ – or nπ n ∈ I nπ
4 ∵ 3x = nπ, n ∈ I or x = ,n∈I
3
tan2x – 2tanx + 1 = 0 π π
⇒ x = 2nπ ± +
3 4
⇒ (tanx – 1)2 = 0 ⇒ tanx = 1
7x π
π 2nπ + or 2nπ –
∴ x = nπ + ,n∈I 12 12
4
(iv) cosx + 3 sinx = 1
Sol 19: tan3x – 3tanx = 0,
tanx (tan2x – 3) = 0 Divide the equation by 2
tanx = 0 or tanx = ± 3 1 3 1
cosx + sinx =
π 2 2 2
x = nπ or x = nπ ± ,n∈I
3 π π
⇒ cos x − = cos
3 3
Sol 20: (i) cosx + sinx = 1 π π
∴ x = 2nπ ± +
1 3 3
Multiply whole equation by
2 2π
i.e. x = 2nπ or 2nπ + ,n∈I
1 1 1 3
⇒ cosx + sinx =
2 2 2 π
C
Sol 21: (i)
π π π 1 6
cos cosx + sin sinx = cos x − =
4 4 4 2 180° = π radian
π π 0
∴ x – = 2nπ ± 180
4 4 ∴ 1 radian →
π
π π π π 180 π
∴ x = 2nπ ± + = 2nπ or 2nπ + ,n∈I → × = 30°
4 4 2 6 π 6
(ii) secx – tanx = 3 4π
C
(ii)
1 tanx 1 − sinx 5
⇒ sec x − 1 ⇒
= =1
3 3 3 cos x 4π 180 4 π
radian = × = 144°
5 π 5
⇒ 3 cosx + sinx = 1
(iii) (1.2)c
Divide the equation by 2
180
1.2 radian = × 1.2 = 68°43’37.8”
3 1 1 π
⇒ cos x + sinx =
2 2 2 Note: 1° = 60’, 1’ = 60”
π π
cos x − = cos
6 3 Sol 22: a + b + c = 180°
π π B
∴ x = 2nπ ± + ,n∈I
3 6
b
π π
x = 2nπ – or 2nπ + , n ∈ I
6 2
1
(iii) sinx + cosx = a c
2 A C
1
Multiply the equation by Given angles are in A. P.
2
Let common difference = d
1 1 1
⇒ sinx + cosx = ∴ b = a + d, c = a + 2d
2 2 2
π π ⇒ a + (a + d) + (a + 2d) = 3(a + d) = 180°
⇒ cos x − = cos
4 3
7 . 4 6 | Trigonometric Ratios, Identities and Equations
⇒ 3a = a + 2d ⇒ a = d … (ii) −2 ± (2)
2
− 4 ( 4 )( −1 ) −2 ± 20
sinx= =
From (i) and (ii) 2× 4 8
a = d = 30° −1 ± 5
⇒ sinx =
∴ a = 30°, b = 60°, c = 90° Ans. 4
∴ Possible solution is
Sol 23: 5 −1 π π
sinx = = sin ⇒ x = nπ + (–1)n 10
4 10
A
−1 − 5 3π 3π
O and sinx = = sin − = sin π +
d 31 man 4 10 10
C
3π 3π
B ∴ x = nπ + (–1)n − = nπ + (–1)n+1 10
10
moon 384400 km
Earth
π 3π
Line OC divides AB into two equal parts ∴ x = nπ, nπ + (–1)n 10 , nπ + (–1)n+1 10
In DOBC
θ
31' OB Sol 26: 5cos2θ + 2cos2 2 + 1 = 0 − π < θ < n
tan = ∴ OB = 384400 × tan(15.5°)
2 OC cos2θ = 2cos2θ – 1
0
15.5
= 384400 × tan = 173318 Km θ
60 ∴ 2cos2 = cos θ + 1
2
AB = 2(OB) = 346636 Km and cos2θ = 2cos2θ + 1
∴ Diameter of moon = 346636 Km. Putting both these in given equation
∴ 5(2cos2θ – 1) + cosθ + 1 + 1 = 0
Sol 24:
A 10cos2θ + cosθ – 3 = 0
10cos2θ – 5cosθ + 6cosθ – 3 = 0
O
Letter B 5’ man
(5cosθ + 3) (2cosθ – 1) = 0
3 1
C ⇒ cosθ = − or cosθ =
5 2
12 m
3 π 3
Assuming letter to be symmetrically placed ∴ θ = cos–1 − 5 or θ = ± = π – cos–1 5
3
BC
tanθ = 3
OB π π
∴θ= , – , π – cos–1 5 (As – π < θ < π)
BC 3 3
tan(2.5’) =
12
Sol 27: 4sin4x + cos4x = 1
∴ BC = 12tan (2.5’) = 0.0873 m
4sin4x = 1 – cos4x = (1 – cos2x) (1 + cos2x)
∴ Total length of letter = 2BC = 0. 174 m = 17.4 m.
4sin4x = sin2x(1 + cos2x)
Sol 25: 4cos2x sinx – 2sin2x = 3sinx One of obvious solution is sinx = 0 i.e. x = np
One of the obvious solution is sinx = 0 If sinx ≠ 0
M a them a ti cs | 7.47
2
sinx = ± A B 3
5 Sol 1: (A) b cos2 + a cos2 = c
2 2 2
2
∴ x = nπ ± sin–1 5
b
( cos A + 1) + a ( cosB+ 1) =
3
c
2 2 2
2
∴ x = nπ, nπ ± α,sinα = ,n∈I bcosA + b + acosB + a = 3c
5
⇒ a + b + (acosB + bcosA) = 3c
Sol 28: tan2θ tanθ = 1 ⇒ acosB + bcosA = c
2 tan θ
tan2θ = , ∴ a + b = 2c ⇒ a,b,c are in A.P.
1 − tan2 θ
Substitute this in given equation, we get π
Sol 2: (B) a2 + 2a + cosec2 2 ( a + x ) =
0
2 tan2 θ [cosec2θ = 1 + cot2q]
∴ = 1 ⇒ 3 tan2 θ = 1
1 − tan2 θ π
⇒ a2 + 2a +1 + cot2 2 ( a + x ) =
0
1 π
tanθ = ± ∴ θ = nπ ±
3 6 π
⇒ (a + 1)2 + cot2 2 ( a + x ) =
0
Sol 29: esinx – e–sinx – 4 = 0
1 For the above equation to be valid
esinx – =4
esin x π
a + 1 = 0 and cot ( a + x ) =
0
Max. Value of L. H. S. can be attained only when esinx is 2
π π
max and
1
is min. ⇒ a = –1 and ( a + x ) = ( 2n + 1 )
sin x 2 2
e
⇒ a + x = 2n + 1 ⇒ x = 2n + 2
As max. Value of sinx is 1
x
1 1 ∴ a = –1 and ∈I
∴ esinx ≤ e1 and ≥ 2
esin x e
1 C C
Sol 3: (D) ( a + b ) sin2+ ( a − b ) cos2
2 2
∴Max. Value of LHS = e – ≈ 2 35
e 2 2
So there is no real value of x for which LHS = 4 C C
= (a2 + b2 + 2ab) sin2 + (a2 + b2 – 2ab) cos2
2 2
Sol 30: sin4θ – 2sin2θ – 1 = 0 π c c
= (a2 + b2) + 2ab sin2 − cos2
Let sin2θ = t ⇒ t2 – 2t – 1 = 0 3 2 2
= a2 + b2 + 2ab(–cos C)
2
2 ± ( −2) + 4 × 1 2± 8 a2 + b2 − c2
∴t= = =1± 2 2 = a2 + b2 – 2ab = c2
2 2 2ab
∴ sin2θ = 1 + 2 or 1 – 2
Sol 4: (D) sin3A + sin3B + sin3C = 3sinA sinB sinC
Since –1 < sinθ < 1 and 0 < sin2θ < 1
a b c
∴ No real solution. ∵ In triangle = = = k
sin A sinB sinC
a3 + b3 + c3 – 3abc = (a+b+c)(a2+b2+c2–ab–bc–ca)=0 β β
2 1 − tan2 − 1 − tan2
∴ a2 + b2 + c2 – ab – bc – ca = 0 2 2
⇒ =
⇒ (a – b)2 + (b – c)2 + (c – a)2 = 0 ⇒ a = b = c β β
2(1 + tan2 ) − 1 + tan2
2 2
∴ Triangle should be equilateral
β β
2 − 2 tan2− 1 − tan2
Sol 5: (D) sin3θ = 4sinθ sin2θ sin4θ θ ∈ [0, p] ⇒ = 2 2
2 β β
sin3θ = 2sinθ [cos2θ – cos6q] 2 + 2 tan − 1 + 2 tan2
2 2
sin3θ = sin3θ + sin(–θ) – [sin7θ + sin(–5θ)] α
1 − tan2
⇒ sin7θ – sin5θ = –sinq 1 − 3tan2 β / 2 2
⇒ =
2
1 + 3 tan β / 2 1 + tan2 α
∴ –sinθ = 2cos6θ sinq
2
−1
sinθ [2cos6θ + 1] = 0 ⇒ sinθ = 0 or cos6θ =
2 β β
2π nπ π ⇒ 1 − 3 tan2 1 + tan2
θ = nπ or 6θ = 2nπ ± ⇒θ= ± 2 2
3 3 9
2 β 2 β
π 4π 7π 2π 5π 8π = 1 − tan 1 + 3 tan
∴ θ=0, π, , , , , , 2 2
9 9 9 9 9 9
β α α β
Sol 6: (A) acos(B–C)+bcos(C – A) + ccos(A – B) ⇒ 1 − 3tan2 + tan2 − 3tan2 tan2
2 2 2 2
= a(cosB cosC + sinB sinC) + b(cosC cosA + sinC sinA) β α α β
1 + 3 tan2
= − tan2 =− 3tan2 tan2
+ c(cosA cosB + sinA sinB) 2 2 2 2
α β
= cosC[acosB + bcosA] + c cosA cosB ⇒ 2 tan2 6 tan2
=
2 2
+ a sinB sinC + b sinC sinA + c sinA sinB
α β
⇒ tan2 3 tan2
=
a = 2RsinA, b = 2RsinB, c = 2RsinC 2 2
and acosB + bcosA = c α β
⇒ tan2 3 tan2
=
abc abc abc 2 2
= c cosC + c cosA cosB + + +
( 2R ) ( 2R ) ( 2R )
2 2 2
α β
⇒ tan2 ⋅ cot =3
3abc 2 2
= c[cos(π – (A + B)) + cosAcosB] +
4R 2
3abc 2π 4π
= c[–cosAcosB + sinAsinB + cosAcosB] + Sol 8: (B) x sinθ = y sin θ + =z sin θ +
4R 2 3 3
3abc abc 3abc abc
= c sinAsinB + = + = 1 3
4R 2
4R 2
4R 2
R2 sin θ − + cos θ +
x 2 2
⇒ = = − 1 + 3 cot θ
2 cos β − 1 y sin θ 2 2
Sol 7: (D) cos α =
2 − cos β 1 3
sin θ − + cos θ −
2 β x 2 2 1 3
1 − tan = = − − cot θ
2
2 −1 z sin θ 2 2
β
1 + tan2 x x
⇒ = 2 ∴ + =−1 ⇒ xz + xy + yz = 0
2 β y z
1 − tan
2
2−
β Sol 9: (C) m = acos3α + 3acosα sin2a
1 + tan2
2
n = asin3α + 3acos2α sina
M a them a ti cs | 7.49
(m+n)=a(sin3α+cos3α+3cosα sinα(cosα+sinα)) B C s
∴ cot cot =
2 2 s − a
= a(sinα + cosα)3
∴ (m + n)2/3 + (m – n)2/3 2 2
B C
= a2/3(sinα + cosα)2 + a2/3(cosα – sinα)2 = 2a2/3 ∴ cot cot = 2
2 2
2 ⇒ cos θ ( 2 + 1 ) = sin θ
3π 3π 3π
We have, f ( π ) = 0 −π < 0 and f = tan − >0
2 2 2 ⇒ tan=
θ 2 +1 …… (i)
3π
∴ f (x) =
0 has at least one root between π and
2
Q= {θ : sin θ + cos θ = } ⇒ sin θ (
2 sin θ )
2 − 1 = cos θ
( )
Sol 3: (A) Given equation is sin ex= 5x + 5− x is tan=
θ
1
2 −1
×
2 +1
=
2 +1
( 2 + 1) ….. (ii)
LHS = sin e ( )
x x
≤ 1 , for all x ∈ R and RHS = 5 + 5 −x
≥2 ∴P =Q
( )
∴ sin ex= 5x + 5− x has no solution.
7 . 5 2 | Trigonometric Ratios, Identities and Equations
tan ( α + β + γ ) = 0 =1
π tan α + tan β
tan ( α + β ) + tan γ Sol 11: (C) α + β= ⇒ tan ( α + β )= = 0
⇒ 0
= 2 1 − tan α tan β
1 − tan ( α + β ) tan γ
γ=α−β
= 1 + sin4 θ − sin2 θ
π
2 Sol 12: (B) θ ← 0,
1 1 4
= sin2 θ − + 1 −
2 4
⇒ tan θ∈ (1, 0 )
2
1 3
= sin2 θ − + 1
2 4 ⇒ cos θ∈ , 1
2
1
For minimum, sin θ =
2 ⇒ cot θ∈ (1, 0 )
3
A= π
4 As θ∈ 0,
4
For maximum, sin θ =0
cot θ > tan θ
A=1
⇒ ( cot θ ) > ( tan θ )
cot θ cot θ
3
⇒ ≤ A ≤1
4
⇒ t 4 > t2
3π
Sol 10: (B) 3 sin4 − α + sin4 ( 3 π + α )
2 tan A cot A
Sol 13: (A) +
6π 1 − cot A 1 − tan A
−2 sin + 2 + sin6 (5 π − 2 )
2 tan A 1 / tan A
= +
1 1 − tan A
= 3 cos2 α + sin4 α − 2 cos6 α + sin6 α 1−
tan A
− tan3 A + 1
= sin4 α + cos4 α + 2cos4 α sin2 α + 2sin4 α cos2 α =
(1 − tan A ) tan A
(sin α + cos α ) − 2sin α cos α
2
2 2 2 2
=
(1 − tan A ) (1 + tan2 A + tan A )
+2cos α sin α ( cos α + sin α )
2 2 2 2 =
tanA (1 − tan A )
M a them a ti cs | 7.53
sec2 A + tanA 2π π
= ⇒ sin4x = ⇒ sinx = ±
tan A 3 3
π
sec2 A ∴ x = nπ ± , np
= +1 6
tan A
= sec A cosec A + 1 Sol 2: 4cos2x sinx – 2sin2x = 3sinx
One above solution is sinx = 0 i.e. x = np
Sol 14: (B) cos x + cos 2x + cos 3x + cos 4x = 0
If sinx ≠ 0
⇒ cos x + cos3x + cos2x + cos 4x =
0
4cos2x = 3 + 2sinx
⇒ 2cos 2x cos x + 2cos 3x cos x =
0 4(1 – sin2x) = 3 + 2sinx
⇒ 2cos x ( cos 2x + cos 3x ) =
0 4sin2x + 2sinx – 1 = 0
5x x −2 ± 4 + 4 × 4 1 ± 5
⇒ 2cos x 2 cos cos =
0 =sinx = =
2 2 2×4 4
x 5x 3π π
⇒ cos= 0 or cos
= x 0 or cos= 0 x = nπ + (–1)n+1 or nπ + (–1)n
2 2 10 10
x π π 5x π
⇒
2
( 2p + 1 ) or x =
2
( 2q + 1 ) or
2 2
=( 2r + 1 )
2 Sol 3: 1 + cosx+cos2x+sinx+sin2x+sin3x = 0
π π
Given condition < 3x − ≤ π
⇒ x= ( 2p + 1) π or x= ( 2q + 1) 2π or x= ( 2r + 1) 5π 2 2
1+cosx + 2cos2x – 1 +(sinx + sin3x)+ sin2x = 0
π 3π 7 π 9 π π 3π
=⇒x , , , , , π,
5 5 5 5 2 2 cosx(2cosx + 1) + 2sin2x cosx + sin2x = 0
x π π π
⇒ cos − = cos − x ⇒ cos − 2x = cos6x
2 4 2 2
π
⇒ − 2x= 2nπ ± 6x
2
x π π
∴ − = 2n π ± − x nπ π nπ π
2 4 2 ⇒x= + , −
4 16 2 8
4n π π π π
⇒ x= + , - 4nπ + ∴ Smallest positive value =
3 2 2 16
2 2
4nπ π π π
∴x= + , 4nπ − n ∈ I Sol 13: 2sin 3x + =
3 2 2 1 + 8 sin2x cos2 2x
4
1 1
Sol 10: As the roots are same both equations should ⇒ 2 sin3x + cos3x
be same. 2 2
Let us solve the second equation
= 1 + 4(2sin2x cos2x)cos2x
1
sinx cos2x = sin2x cos3x – sin5x
2 ⇒ [ 2 (sin3x + cos3x)]2 = 1 + 4sin4x cos2x
⇒ 2sinx cos2x=2sin2x cos3x – sin5x ⇒ 2(sin23x + cos23x + 2sin3x cos3x)
⇒ sin3x – sinx = sin5x – sinx – sin5x = 1 + 4sin4x cos2x
⇒ sin3x = 0 ∴ x = np ⇒ 2(1 + 2sin3x cos3x) = 1 + 4sin4x cos2x
Put x = nπ is given equation ⇒ 1 = 2(2sin4x cos2x) – 2(2sin3x cos3x)
a cos2x + |a| cos4x + cos6x = 1 = 2(sin6x + sin2x) – 2sin6x
a cos2nπ + |a| cos4nπ + cos6nπ = 1 1
⇒ sin2x =
⇒ a + |a| + 1 = 1 2
π π
∴a≤0 ∴ 2x − = 2nπ ±
2 3
5π π
Sol 11: 13 − 18 tan x = 6tanx – 3, –2π , x < 2p x = nπ + , nπ + ,n∈I
12 12
⇒ 13 – 18tanx = (6 tanx – 3)2 5π π
If x = ⇒ sin 3x + = –1
13 – 18tanx = 36tan2x + 9 – 36tanx 12 4
18 tan2x – 9tanx – 2 = 0 Which is not possible
⇒ (6tanx + 1) (3tanx – 2) = 0 π
∴ x = nπ +
12
1
Also 6tanx – 3 > 0 ⇒ tan x >
2
2 Sol 14: sinx – sin3x + sin5x = cosx – cos3x + cos5x
∴ tanx =
3 1 1
2 (sinx – cosx) – (sin3x – cos3x)
x = a – 2π, a – π, a, a + π, where a = tan–1 3 2 2
1
+ (sin5x – cos5x) = 0
Sol 12: ( 1 + sin 2x − 2 cos 3x =
0 ) 2
= (1 + k) –
( 2k + 3) sin22x Let tan
θ
=t
4 2
For f(x) = constant ∴ acosθ + bsinθ = c
2k + 3 3 a(1 – t2) + b(2t) = c(1 + t2)
=0 ⇒k= −
4 2 = (a + c)t2 – 2bt + c – a = 0
(b) (1 + k) –
( 2k + 3 ) sin22x = 0
α β 2b
4 ∴ tan + tan = (P) (sum of roots)
2 2 a+c
(1 + k)4
⇒ sin22x = α β c−a
(2k + 3) tan tan =
2 2 c+a
4 (1 + k ) −1 (Q) (Product of roots)
∴0≤ ≤ 1 ⇒ k ∈ −1,
2k + 3 2
π
π sin2 x − 1 1 1 1
tan x − 4 Sol 4: (B) cot 7 ° + tan67 ° − cot 67 ° − tan7 °
Sol 2: (A) 2 4 − 2 ( 0.25 ) cos 2x +1 =0 2 2 2 2
π A B B A
π = cot + tan – cot – tan
sin2 x − 1 − cos2 x − 2 2 2 2
4 1 4
=
cos 2x 2 cos 2x A = 15°, B = 135°
1 (1 − sin 2x ) 1 π 1 − tan2
A B
tan2 − 1
= = tan − x 2 + 2
2 cos 2x 2 4 =
A B
tan tan
π 2 2
∴ Take tan x − = t
4
= 2cotA – 2cotB = 2(cot15° – cot135°)
Then expression would be
= 2(2 + 3 + 1) = 2(3 + 3)
t/2
1
2t – 2 +1=0 Which is an irrational number.
4
1
t 2π
⇒ 2t – 2 + 1 = 0 Sol 5: (C) A = 580° = 3π +
9
2
A A A A
(2t)2 + (2)t – 2 = 0 1 + sinA= sin2 + cos2 + 2sin cos
2 2 2 2
(2t + 2) (2t – 1) = 0
2
⇒ 2t = – 2 (Not possible) or 2t = 1 A A A A
= sin + cos = sin + cos
⇒t=0 2 2 2 2
π π 2π
∴ tan x − = 0 ; x = nπ + ∵ for A = 3π +
4 4 9
1 A A
But in equation does not exist at 1 + sinA =
− sin + cos
cos2x 2 2
π
x = nπ + , therefore no value of x exists. A A A A
4 1 − sinA= sin2 + cos2 − 2sin cos
2 2 2 2
5
Sol 3: (A) ∑ cos r x = 0 A A
r =1 = sin − cos
2 2
cos x + cos 2x + cos 3x + cos 4x + cos 5x = 0
2π
For A = 3π +
⇒ cos x + cos 5x + cos 2 x + cos 4x + cos 3x =
0 9
x2 − x 1
Sol 6: (A) tanα = and tanβ = xy x2 y2
2
x − x +1
2
2x − 2x + 1 ⇒ cos ( α − β=
) + 1− . 1−
ab a2 b2
1
⇒2(x2 – x) + 1 =
tanβ xy x2 y 2 x2 y 2
⇒ cos ( α − β=
) + 1− − +
ab a2 b2 a2 b2
1 − tan β 1
x2 – x =
tan β 2 2 2 2 2 2
2
xy 1− x − y + x y
⇒ cos ( α − β ) − =
1 1 − tan β 1 1 − tan β ab a2 b2 a2 b2
2 tan β 2 tan β
∴ tanα = =
1 1 − tan β 1 1 + tan β x2 y 2 xy
+1 ⇒ cos2 ( α − β ) + − 2 cos ( α − β ) ×
2 tan β 2 tan β ab2 2 ab
1 − tan β x2 y 2 x2 y 2
⇒ tanα = 1 +
=− +
1 + tan β a2 b2 a2 b2
⇒ tanα + tanα tanβ = 1 – tanb
x2 y2 2xy
⇒ + − cos ( α − β ) = 1 − cos2 ( α − β )
tan α + tan β a2
b 2 ab
⇒ = tan(α +=
β) 1
1 – tan α tan β
= sin2 ( α − β )
⇒ x2 + x – 1 = 0 2 12 7
= cot α +
−1 ± 5 35 10
x= = 2sin18°
2
[24 cot α + 7]
= 3
15
Multiple Correct Choice Type
1
4 3 Sol 14: (B, C) sin t + cost =
Sol 12: (B, D) 1 + 4sinθ + 3cosθ = 1 + 5 sin θ + cos θ 5
5 5 t t
4 2 tan1 − tan2
= 1 + 5sin(θ + α)cosα = 2 + 1
2 =
5 ⇒
t t 5
∴ Maximum value is 1 + 5 = 6 1 + tan2 1 + tan2
2 2
When sin(θ + α) = 1 t
Let tan =a
Minimum value is 1 – 5 = – 4 2
2 3 3 3 4 3 4 π 2π
= − + cot α × + 2cot α × + 2 × + , ±
3 2 5 2 5 5 5 3 3
M a them a ti cs | 7.63
sin2 x sin2 x
⇒ sin ( 4x + 100º ) + 2sin150º cos50º =
0
loge 2 loge 2
= e 1 −sin2 x = e cos2 x 1
⇒ sin ( 4x + 100º ) + 2. sin ( 90º −50º ) =
0
2
2x
⇒ 2tan satisfys x2 − 9x + 8 =0 ⇒ sin ( 4x + 100º ) + sin 40º =
0
⇒ ( 4n + 1 ) π / 2 ≤ 2p Applying
For p to be least, n should be least. R3 → R3 − R1 and R 2 → R 2 − R1 we get
⇒n=0
1 + sin2 θ cos2 θ 4 sin 4θ
⇒ 2p ≥ π / 2 −1 1 0 0
=
π −1 0 1
⇒ p≥
2 2
π Applying C1 → C1 + C2
Therefore, least value of p =
2 2
2 cos2 θ 4 sin 4θ
Sol 6: Given, ⇒ 0 1 0 0
=
2θ −1 0 1
(1 − tan θ )(1 + tan θ ) sec2 θ + 2tan =0
( )(
⇒ 1 − tan2 θ ⋅ 1 + tan2 θ + 2 ) tan2 θ
=0 ⇒ 2 + 4 sin 4θ =0
2θ
⇒ 1 − tan4 θ + 2tan =0 −1
⇒ sin 4θ =
2
Substitute tan2 θ =x
n π
∴ 1 − x 2 + 2x =
0 ⇒ 4θ = nπ + ( −1 ) −
6
⇒ x2 − 1 =2x nπ n+1 π
⇒ 4=
θ + ( −1 )
x 2
Note: 2 and x − 1 are incompatible functions, 4 24
therefore, we have to consider range of both functions. 7π 11π
Clearly, θ = , are two values of θ lying between
Curvesy = x – 1 2
24 24
π
0º and
Andy = 2x 2
It is clear from the graph that two curves interest at one
point at x = 3, y = 8. Sol 8: (B, C) For 0 < φ < / π / 2 we have
∞
y x= ∑ cos2n φ = 1 + cos2 φ
n= 0
1 1 4
Now, x +=
y + 4 ± 16 − 4 × 5 ×
2 2
sin φ cos φ ⇒ sin2 x = 5
10
cos2 φ + sin2 φ 1
=
2 2
cos φ sin φ cos φ sin2 φ
2
4 ± 16 − 16
=
1 1 10
Again, = 1 − sin2 φ cos2 φ = 1 −
x xy 2
⇒ sin2 x =
5
1 xy − 1
⇒ = ⇒ xy = xyz − z 2
x xy ⇒ sin x =
±
5
⇒ xy + z =xyz
2
sin2 x =
Therefore, (b) is the answer from eq. (i) (putting the 5
value of xy)
3
⇒ cos2 x =
⇒ xyz = x + y + z 5
5
⇒ sec2 x =
Sol 9: (A, B) We know that, sec2 θ ≥ 1 3
4xy
⇒ ≥1 ⇒ tan2 x = sec2 x − 1
(x + y)
5 2
⇒ 4xy ≥ ( x + y )
2
= −1 =
3 3
⇒ ( x + y ) − 4xy ≤ 0
2
sin8 x cos8 x
Now, +
⇒ (x − y) ≤ 0 8 27
2
4 4
⇒ x−y =0 2 3
5 5 2 3 1
⇒ x=y = + = + =
8 27 5 4
5 4 125
Therefore , x + y = 2x (add x both sides)
But x + y ≠ 0 since it lies in the denominator, Sol 11: (D)
⇒ 2x ≠ 0
6
⇒ x≠0 mπ π mπ
∑ cosec θ + 4
− cosec θ +
4
=4 2
4
Hence, x = y, x ≠ 0 is the answer. m=1
m m π π
sin4 x cos4 x 1 sin θ + − θ + −
Sol 10: (A, B) + =
6
π 4 4
2 3 5 ∑ 2×
mπ mπ π
4 2
=
m=1
sin θ + sin θ + −
3 sin4 x + 2 cos4 x 1 4 4 4
⇒ =
6 5
m π mπ π m π m π π
( 6
)
4 4 4
sin θ + cos θ + − − cos θ + sin θ + −
⇒ sin x + 2 sin x + cos x = 6 4 4 4 4 4 4
5 ∑
m π
m π
π
m=1
sin θ + sin θ + −
( 6
⇒ sin4 x + 2 1 − 2 sin2 x cos2 x =
5 ) 6
4 4 4
mπ π m π
∑ 2 x cot θ + − − cot θ + = 4 2
4 2
(
⇒ sin x + 2 − 4 sin x 1 − sin x =
6
5
2
) m=1 4 4 4
6 3 π
⇒ sin4 x + 2 − 4 sin2 x + 4 sin4 x = 2 cot θ − cot θ + = 4 2
5 2
4 3π
⇒ 5 sin4 x − 4 sin2 x + 0
= ⇒ cot θ − cot + θ = 4
5
2
7 . 6 6 | Trigonometric Ratios, Identities and Equations
⇒ cot θ − tan θ = 4
Sol 13: Given, n > 3 ∈ Integer
1 1 1
⇒ tan2 θ − 4 tan θ + 1 = 0 and = +
π 2π 3π
4 ± 16 − 4 sin sin sin
⇒ tan θ = n
n
n
2
1 1 1
4 ± 12 ⇒ − =
= π 3π 2π
2 sin sin sin
n n n
4± 2 3
=
2 3π π
sin − sin
⇒ n n = 1
⇒ tan θ = ± 2 3
π 3π 2π
sin .sin sin
5π n n n
⇒θ=
12 π 3π
2π sin .sin
π n n
⇒ 2cos .sin =
Sol 12: Given, tan
= θ cot5θ n
n 2 π
sin
n
π
⇒=tan θ tan − 5θ
2 4π 3π
⇒ = π− ⇒ 7π = π ; ⇒ n = 7
π n n n
⇒ − 5θ = nπ + θ
2
π
⇒ 6θ= − nπ Sol 14: ( y + z ) =
cos 3 θ xyz sin 3 θ …..(i)
2
π nπ sin3 θ 2z cos 3 θ + 2y sin 3 θ
xyz= …..(ii)
⇒=θ −
12 6
π
xyz sin3 θ= ( y + 2z ) cos 3 θ + y sin 3 θ ….(iii)
Also, cos 4=
θ sin2=
θ cos − 2θ
2 From (i), (ii) and (iii), we get
π
⇒ 4θ= 2nπ ± − 2θ ( y + z ) cos
= 3 θ 2z cos 3 θ + 2y sin 3 θ
2
= ( y + 2z ) cos 3 θ + y sin 3 θ
Taking positive ∆ sign
⇒ y ( cos 3 θ − 2sin
= 3 θ ) z cos 3 θ and
π
⇒ 6θ= 2nπ + y ( cos 3 θ − sin 3 θ ) =0
2
nπ π
⇒=
θ + ⇒ cos =
3 θ sin 3 θ
3 12
π
Taking negative sign ⇒ tan 3 θ = 1 = tan
4
2π 2π 3π
⇒ 2sin .cos = sin π
n n n ⇒ 3 θ= n π +
4
4π 3π
⇒ sin = sin nπ π
n n ⇒=
θ +
3 4
π
⇒ 2θ= 2nπ − π 7 π 11 π
2 ⇒ , ,
4 12 12
π
⇒ θ = nπ − Total three solution in ( 0, π )
4
Above values of θ suggest that there are only 3
common solutions. Sol 15: tan
= θ cot5 θ
π
tan θ tan − 5 θ
=
2
M a them a ti cs | 7.67
π 1
⇒ θ= n π + − 5θ 1
2 is =2
2
sin θ + 3sin θ cos θ + 5cos θ 2 1/2
π
⇒6 θ = nπ +
2
nπ π π ±π ± 5 π
Sol 17: (D) P = {θ : sin θ − cos θ = 2 cos θ }
⇒θ = + ⇒θ= ± , , ..... …. (i)
6 12 12 4 12 ⇒ sin=
θ ( 2 + 1) cos θ
sin=
2 θ cos 4 θ
⇒ tan =
θ 2 +1
π
⇒ cos
= 4 θ cos − 2 θ
2 Q= {θ : sin θ + cos θ = 2 sin θ}
π ⇒ cos=θ ( 2 − 1 ) sin θ
⇒ 4=θ 2 m π ± − 2θ
2 1
mπ π π ⇒ tan =
θ = 2 +1
⇒=θ + , m π− 2 −1
3 12 4
∴P =Q
π π 5π
⇒=
θ ,− , ….(ii)
12 4 12 1 1 1
Sol 18: = +
π 2π 3π
In (i) and (ii) only (iii) solutions are common sin sin sin
n n n
π π 5π
− , ,
4 12 12 3π π
sin− sin
n n 1
⇒ =
π 3π π
Sol 16: Given sin sin sin 2
n n n
1
sin θ + 3sin θ cos θ + 5cos2 θ
2 3π π 3π π
+ −
n n n n
Let 2 cos sin
2 2 1
⇒ =
A sin2 θ + 3sin θ cos θ + 5cos2 θ
= π 3π 2π
sin sin sin
n n n
= 1 + 4 cos2 θ + 3sin θ cos θ
3 2π π
= 1 + 2 (1 + cos 2 θ ) + sin2 θ 2 cos
sin
1
2 ⇒ n n =
π 3π π
3 sin sin sin 2
= 3 + 2cos 2 θ + sin 2 θ n n n
2
a cos θ + b sin θ ≥ − a2 + b2 2π 2π 3π
= 2=
sin cos sin
n n n
3
2 4π 3π
3 − 22 +
Amin = ⇒ sin sin
=
n n
2
4π 3π
9 ⇒ sin − sin 0
=
=3 − 4 + n n
4
7π π
25 5 1 ⇒ 2 cos sin 0
=
=3 − =3 − = 2n 2n
4 2 2
7π π
The maximum value of given expression ⇒ cos = 0 or sin = 0
2n 2n
7 . 6 8 | Trigonometric Ratios, Identities and Equations
7 1
⇒ n= ⇒ − 4 sin x cos2 x − − cos x =3
2m + 1 2
For integral values of n > 3 3
2
1
⇒ − 4 sin x cos x − − =3
2 4
Sol 19: (A, C, D) θ ∈ 0, 2 π
2
1 3 3
θ θ ⇒ cos x − − =−
2 cos θ (1 − sin =
φ ) sin2 θ tan + cot cos φ − 1 2 4 4 sin x
2 2
2
θ 1 3 3
sin θ tan2 + 1
2
⇒ cos x − = −
2 cos φ − 1 2 4 4 sin x
⇒ 2cos θ (1 − sin φ )
=
θ
tan 2
2 1 3 3
⇒ cos x − = 1 −
θ 2 4 sin x
sin2 θ.sec2
⇒ 2cos θ (1 − sin φ ) 2 cos φ − 1
= L.H.S. > 0 and R.H.S. < 0
θ
tan
2 ∴ No solution
⇒ 2cos θ (1 − sin=
φ ) 2sin θ ( cos φ ) − 1
5
Sol 21: cos2 2x + cos4 x + sin4 x + cos6 x + sin6 x =
2
⇒ 1 + 2 cos
= θ 2 sin ( θ + φ ) ….(i) 4
3 5
Now, given tan ( 2 π − θ ) > 0, − 1 < sin θ < − ⇒ cos2 2x + cos4 x + sin4 x + cos4 x
2 4
3π 5π 1 − sin2 x + sin4 x 1 − cos2 x =
2
⇒ tan θ < 0 and <θ<
2 3
⇒
3π
2
<θ<
5π
3
⇒
5
4
( )
cos2 2x + 2 cos4 x + sin4 x − sin2 x cos2 x
3 π 5π
From (i), as θ ∈ ,
(sin x + cos x ) =
2 2
2
2 3
⇒ cos 2x + 2 ( sin x + cos x )
5 2 2 2
2
− 2sin2 x cos2 x
⇒ 1 < 2 cos θ + 1 < 2 4
⇒ 1 < 2 sin ( θ + φ ) < 2 − sin2 x cos2 x =
2
1 π 5π π
⇒ < sin ( θ + φ ) < 1 ⇒ θ + φ ∈ , ,θ + φ ≠ ⇒
5
cos2 2x − 5sin2 x cos2 x =
0
2 6 6 2 4
Or
5 5
⇒ cos2 2x = sin2 2x
13 π 17 π 5π π 5π 4 4
θ+φ∈ , ,θ + φ ≠ ⇒ −θ < φ < −θ
6 6 2 6 6
⇒ tan2 2x =
1
13 π 17 π
or −θ < φ < −θ π
6 6 ⇒ tan 2x = ± = tan ±
3π 2π 2π 7π 4
As, θ ∈ − ,− or , π
2 3 3 6 ⇒ 2x= n π ±
4
Sol 20: (D) sin x + 2 sin 2x – sin 3x = 3 nπ π
⇒ x= ±
2 8
⇒ sin x − sin 3x + 2sin 2x =
3
π 3 π 5 π 7 π 11 π 13 π 15 π
=⇒x , , , , , , →
⇒ 2 cos 2x sin ( −x ) + 2sin 2x =
3 8 8 8 8 8 8 8
Total 8 solution.
⇒ − 2 sin x 2 cos2 x − 1 + 2 × 2 sin x cos x =3
M a them a ti cs | 7.69
3 1 sin x cos x
⇒ + + 2 − =0
cos x sin x cos x sinx
⇒ 3 sin x + cos x + 2 sin2 − cos2 x =
0
⇒ 3 sin x + cos x =
2 cos 2x
3 1
⇒ sin x + cos x =
cos 2x
2 2
π π
⇒ sin sinx + cos cos x =
cos2x
3 3
π
⇒ cos x − = cos 2x
3
π
⇒ x − = 2n π ± 2x
3
π 2n π π
⇒ x = − 2n π + or x = +
3 3 9
π π 5π 7 π
⇒x=− , ,− ,
3 9 9 9
Sum = 0
2017-18 100 &
op kers
Class 11 T
By E ran culty
-JE Fa r
IIT enior emie .
S fP r es
o titut
Ins
MATHEMATICS
FOR JEE MAIN & ADVANCED
SECOND
EDITION
Exhaustive Theory
(Now Revised)
Formula Sheet
9000+ Problems
based on latest JEE pattern
PlancEssential
Questions recommended for revision
8. STRAIGHT LINE
1. INTRODUCTION
Co-ordinate geometry is the branch of mathematics which includes the study of different curves and figures by
ordered pairs of real numbers called Cartesian co-ordinates, representing lines & curves by algebraic equation. This
mathematical model is used in solving real world problems.
2. CO-ORDINATE SYSTEM
Co-ordinate system is nothing but a reference system designed to locate position of any point or geometric
element in a plane of space.
Co-ordinate axis: x-axis and y-axis together are called axis of co-ordinates or axis of Figure 8.1
reference.
Origin: The point ‘O’ is called the origin of co-ordinates or just the origin.
Oblique axis: When xox’ and yoy’ are not at right angle, i.e. if the both axes are not perpendicular, to each other,
then axis of co-ordinates are called oblique axis.
Illustration 1: If the Cartesian co-ordinates of any point are ( 3, 1) , find the polar co-ordinates. (JEE MAIN)
y
Sol: Polar co-ordinates of any point are (r, θ), where r = x2 + y 2 and θ = tan–1 .
x
x= 3;y=1
Let their polar co-ordinates be (r, θ) ⇒ x = r cos θ; y = r sinθ
So r ⇒ x2 + y 2 r = 3+1
y 1 π
θ ⇒ tan−1 = 2 θ ⇒ tan−1 =
x 3 6
π
∴(r, θ) = 2 , .
6
y
3. DISTANCE FORMULA
Q(x2,y2)
B(r2,2)
d
The distance between two points P(x1, y1) and Q(x2, y2) is ,y 1)
P(x 1
PQ = (x1 – x2 )2 + (y1 – y 2 )2 = (x2 − x1 )2 + (y 2 − y1 )2 (r1,1) A
x2 - x1
Distance of a point P(x1, y1) from the origin O(0, 0) is
O x2
OP = 2
X +y 2
1 1
Figure 8.3
Distance between two polar co-ordinates A(r1, q1) and B(r2, q2) is
given by
AB = r12 + r22 − 2r1r2 cos(θ1 − θ2 )
AB = (r2 cos θ2 )2 − 2r1r2 cos θ1 cos θ2 + (r1 cos θ1 )2 + (r2 sin θ2 )2 − 2r1r2 sin θ1 sin θ2 + (r1 sin θ1 )2
PLANCESS CONCEPTS
Distance between two polar co-ordinates A(r1, q1) and B(r2, q2) is given by
AB = r12 + r22 − 2r1r2 cos(θ1 − θ2 )
Vaibhav Krishnan (JEE 2009, AIR 22)
M a them a tics | 8.3
π π
Illustration 2: Find the distance between P 2, − and Q 3, . (JEE MAIN)
6 6
Sol: The distance between two points = r12 + r22 − 2r1r2 cos(θ1 − θ2 ) Therefore,
.
π π π 1
PQ = r12 + r22 − 2r1r2 cos(θ1 − θ2 ) = 4 + 9 − 2.2.3cos − − = = 4 + 9 − 12cos = 13 − 12. = 7
6 6 3 2
Illustration 3: The point whose abscissa is equal to its ordinate and which is equidistant from the points A(1, 0),
B(0, 3) is (JEE MAIN)
Sol: Given, abscissa = ordinate. Therefore distance can be found by considering the co-ordinates of required point
be P(k, k).
Now given PA = PB ⇒ (k − 1)2 + k 2 = k 2 + (k − 3)2
2k2 – 2k + 1 = 2k2 – 6k + 9 ⇒ 4k = 8 ⇒ k = 2
4. SECTION FORMULA
Let R divide the two points P(x1, y1) and Q(x2, y2) internally in the ratio m:n.
Let (x, y) be the co-ordinates of R.
y
Draw PM, QN, RK perpendicular to the x-axis. Q
R
F
Also, draw PE and RF perpendicular to RK and QN. P E
PR m
Here, = .
RQ n
x’ O M K N x
Triangles PRE and RFQ are similar.
PR PE PE m y’
∴ = ⇒ =
RQ RF RF n
Figure 8.4
But PE = x – x1 and RF = x2 – x.
x − x1 m mx2 + nx1
∴ = ⇒ x=
x2 − x n m+n
ER m
In the same way, =
FQ n
y − y1 m my 2 + ny1 mx2 + nx1 my 2 + ny1
i.e., = ⇒y= The co-ordinates of R are ,
y2 − y n m+n m+n m + n
PR ' m
If R’ divides PQ externally, so that = , triangles PER’ and QR’F are similar. y
QR ' n
PR ' PE F R’
∴ =
R 'Q R 'F Q
P E
But PE = x – x1 and R’F = x – x2.
x
x − x1
m mx2 − nx1 x’ O M N K
∴
= i.e., x =
x − x2 n m−n
my 2 − ny1 y’
Similarly, y = .
m−n
Figure 8.5
mx − nx1 my 2 − ny1
The co-ordinates of R’ are 2 ,
m−n m − n
8 . 4 | Straight Line
PR ' m m
Alternate Method: =− = By changing n into –n in the co-ordinates of R, we can obtain the co-ordinates
R 'Q n –n
of R’:
x + λx2 y1 + λy 2
Cor. From the above cor., the co-ordinates of a point dividing PQ in the ratio λ:1 are 1 , . Considering
1+ λ 1+ λ
λ as a variable parameter, i.e. of all values positive or negative, the co-ordinates of any point on the line joining the
points (x1, y1) and (x2, y2) can be expressed in the above forms.
Let the vertices of the triangle ABC be (x1, y1), (x2, y2) and (x3, y3), respectively. F 1 D
x + x3 y 2 + y 3 2 G
The mid-point D of BC is 2 , G, the centroid, divides AD internally
2 2
in the ratio 2:1. (x1, y1)A E B(x2, y2)
2. ( (x2 + x3 ) / 2 ) + 1.x1 x1 + x2 + x3
then x = = and
2+1 3
2. ( (y 2 + y 3 ) / 2 ) + 1.y1 y1 + y 2 + y 3 x + x 2 + x3 y1 + y 2 + y 3
y= = ∴ G is 1 , .
2+1 3 3 3
5.2 Incentre
Let A (x1, y1), B (x2, y2), C (x3, y3) be the vertices of the triangle.
Let AD bisect angle BAC and cut BC at D.
A
BD AB c
We know that = =
DC AC b
cx3 + bx2 cy 3 + by 2 I
Hence the co-ordinates of D are , C
c+b c+b
D
Let (x, y) be the incentre of the triangle
B
CD b BC b + c ca AI AB c b+c
= ∴ = ∴ BD = = = = Figure 8.7
BD c DB c b + c ID BD ( ca / (b + c) ) a
(b + c) ( (cx3 + bx2 ) / (c + b) ) + ax1 ax1 + bx2 + cx3
= x = ,
b+c+a a+b+c
(b + c) ( (cx3 + bx2 ) / (c + b) ) + ax1 ax1 + bx2 + cx3 (b + c) ( (cy 3 + by 2 ) / (c+ b) ) + ay1 ay1 + by 2 + cy 3
∴ x = = , y =
b+c+a a+b+c b+c+a a+b+c
(b + c) ( (cy 3 + by 2 ) / (c+ b) ) + ay1 ay1 + by 2 + cy 3
y =
b+c+a a+b+c
M a them a tics | 8.5
5.3 Ex-centres
A
The centre of the circle which touches the side BC and the extended portions of sides I3 I2
AB and AC is called the ex-centre of ∆ABC with respect to the vertex A. It is denoted by
I1 and its co-ordinates are as follows: B C
Similarly ex-centres of ∆ABC with respect to vertices B and C are denoted by I2 and I3,
respectively, and Figure 8.8
A C
Figure 8.10
A(x1, y1)
5.5 Orthocentre
The point of intersection of altitudes of a triangle that can be obtained by solving the D F
equation of any two altitudes is called Orthocentre. It is denoted by H
H
AH AG HG
∴ = 2= = ⇒ G divides line joining H and O in 2:1.
OD GD GO
(c) In an isosceles triangle centroid, orthocentre, incentre, circumcentre lie on the same line.
Figure 8.13
PLANCESS CONCEPTS
Illustration 4: If G be the centroid of the triangle ABC, prove that AB2 + BC2 + CA2 = 3(GA2 + GB2 + GC2).
(JEE MAIN)
Sol: Distance formula of two points can be used to prove AB2 + BC2 + CA2 = 3(GA2 + GB2 + GC2).
In triangle ABC, let B be the origin and BC the x-axis. Let A be (h, k) and
a+h k A(h, k)
C be (a, 0). Then centroid G is , .
3 3
LHS
= AB2 + BC2 + CA2 = (h – 0)2 + (k – 0)2 + a2 + (h – a)2 + (k – 0)2
C (a, 0)
= 2h2 + 2k2 + 2a2 – 2ah B
Figure 8.14
M a them a tics | 8.7
∴ RHS
a + h 2
k
2
a+h
2
k
2
a+h
2
k
= 3 − h + − k + − 0 + − 0 + − a + − 0
3 3 3 3 3 3
= 1/3 [(a – 2h)2 + 4k2 + (a + h)2 + k2 + (h – 2a)2 + k2] = 2h2 + 2k2 + 2a2 – 2ah
Hence, it is equal on both sides.
C
y
5.7 Area of a Triangle
Let A, B, C be the vertices of the triangle having (x1, y1), (x2, y2) and (x3, y3) as their B
1 1 1
= (LA + NC) LN + (NC + MB) NM – (LA + MB) LM
2 2 2 x
1 1 1 O L N M
= (y1 + y3) (x3 – x1) + (y3 + y2) (x2 – x3) – (y1 + y2) (x2 – x1)
2 2 2
Figure 8.15
1
= {x1(y2 – y3) + x2(y3 – y1) + x3(y1 – y2)}
2
x1 y1 1
1
= x2 y2 1
2
x3 y3 1
If the area of the triangle formed by the points (x1, y1), (x2, y2) and (x3, y3) is zero, the points lie on a straight line.
Using this, we can determine whether three point are in a straight line. i.e. the condition for (x1, y1), (x2, y2) and (x3,
x1 y1 1
y3) to be collinear is that x2 y 2 1 = 0.
x3 y3 1
1
For example, the area of the triangle formed by the points (1, 4), (3, –2) and (–3, 16) is {1(–2 –16) + 3} {(16 – 4) – 3
(4 + 2)} = 0. The three points lie on a straight line. 2
Illustration 5: The vertices of a triangle ABC are A(p2, –p), B(q2, q) and C(r2, –r). Find the area of the triangle.
(JEE MAIN)
x1 y1 1
1
Sol: Area of the triangle formed by the points (x1, y1), (x2, y2) and (x3, y3) is x2 y 2 1 . Substituting the given
co-ordinates, we can obtain area of given triangle. 2
x y 1
3 3
p2 −p 1 p2 − q2 −(p + q) 0 p − q −1 0
1 2 1 2 2 1
D= q q 1 = q −r q+r 0 = (p + q) (q + r) q − r 1 0
2 2 2 2
r −r 1 r2 −r 1 r2 −r 1
1 1
= (p + q) (q + r) [(p – q) + (q – r)] = (p + q) (q + r) (p – r)
2 2
8 . 8 | Straight Line
Note:
(a) If area of the triangle is zero, then the three points are collinear.
(b) The area of a polygon with vertices A1(xi, yi), i = 1, …, n (vertices taken in anti-clockwise order)
1
[(x y – x2y1) + (x2y3 – x3y2) +…+(xny1 – x1yn)]
2 1 2
6. LOCUS
Locus is a set of points which satisfies a given geometrical data. Thus, for example, locus of a point moving at a
constant distance from a given point is a circle. Locus of a point which is equidistance from two fixed points is a
perpendicular bisector of the line joining the two points.
All the points in a locus can be represented by an equation. For example,
(a) If the distance of the point (x, y) from (2, 3) is 4, then
(x – 2)2 + (y – 3)2 = 42.
i.e. x2 + y2 – 4x – 6y – 3 = 0.
This equation will represent a circle with its centre at (2, 3) and radius 4.
(b) If (x, y) be the point equidistant from the points (3, 4) and (2, 1), then
(x – 3)2 + (y – 4)2 = (x – 2)2 + (y – 1)2
i.e. x + 3y = 10.
From the geometrical constraint, which governs the motion, we can find a relation (locus) between the co-
ordinates of the moving point in any of its positions. Equation of locus is therefore merely on equation
relating the x and y co-ordinates of every point on the locus.
Note:
•• Locus should not contain any other variables except x and y.
•• The algebraic relation between x and y satisfied by the co-ordinates at every point
on the curve and not off the curve is called the equation of curve.
y
Illustration 6: Find the equation of locus of a point which moves so that its distance P(x, y)
(0, 1)
from the point (0, 1) is twice the distance from x-axis. (JEE MAIN)
N
Sol: Here we can obtain the equation of locus of given point by using given condition
and distance formula of two points.
x
Let the co-ordinates of such a point be (x, y). Draw PM ⊥ to x-axis. x’ 0 M
y’
Hence, PM = y
Figure 8.16
PN = 2PM (given)
M a them a tics | 8.9
Illustration 7: Locus of the centroid of the triangle whose vertices are (a cos t, a sin t), (b sin t, – b cos t) and (1, 0),
where t is a parameter is (JEE MAIN)
(A) (3x – 1)2 + (3y)2 = a2 + b2 (B) (3x + 1)2 + (3y)2 = a2 + b2
Sol: The centroid (h, k) of a triangle formed by points (x1, y1), (x2, y2) and (x3, y3) will be
x1 + x2 + x3 y1 + y 2 + y 3
h = and k .
3 3
(A) If (h, k) is the centroid, then
7. STRAIGHT LINE
Definition: It is defined as the locus of a point such that any two points of this locus have a Y
constant inclination (gradient).
Inclination: If a straight line intersects the x-axis, the inclination of the line is defined
as the measure of the smallest non-negative angle which the line makes with the X’
positive direction of the x-axis 180-
π
Slope (or gradient): If the inclination of a line (i.e. non-vertical line) is θ and θ ≠ ,
then the slope of a line is defined to be tan θ and is denoted by m. 2
Y’
∴ m = tanq Figure 8.17
(a) Slope of x-axis is zero.
(b) Slope of y-axis is not defined.
7.1 Slope
(x2, y2)Q l
Let P(x1, y1) and Q(x2, y2) be two points on a line, then slope will be
y2 - y1
(x1, y1)P
MQ M
m
= tan=
θ x2 - x 1
MP
y 2 − y1
m
= tan=
θ
x2 − x1 O
x y 1
Figure 8.18
(a) Line passing through two points (x1, y1) and (x2, y2) is x1 y1 1 = 0
x2 y2 1
Note: Above-mentioned matrix form is a condition for three points to be collinear. (x1, y1) (x2, y2)
(b) Equation of the median through A(x1, y1) is
Figure 8.19
8 . 1 0 | Straight Line
A(x1, y1)
P(x, y)
B C
M (x3, y3)
(x2, y2)
( x2 +x3 y2 +y3
2
,
2 )
Figure 8.20
x y 1 x y 1 x y 1
x1 y1 1 = 0 or x1 y1 1 + x1 y1 1 = 0
x 2 + x3 y 2 + y3 x2 y2 1 x3 y3 1
1
2 2
A(x1, y1)
x y 1 x y 1
b x1 y1 1 ± c x1 y1 1 =
0
P(x, y)
x2 y2 1 x3 y3 1
P(x, y)
(x2, y2)B b E
D C(x3, y3)
y
L2
L1
1 -2
A x
O B
Figure 8.22
M a them a ti cs | 8.11
π 1
Illustration 8: If the angle between two lines is and slope of one of the lines is . Find the slope of the other.
4 2 (JEE MAIN)
m2 – m1
Sol: We know that, tan θ = , where m1 and m2 are the slope of lines and θ is the angle between them.
1+m1m2
1 π π m – (1 / 2) m – (1 / 2)
Let m1 = , m2 = m and θ = So, tan = ⇒ 1 =± ⇒ m = 3 or –(1/3)
2 4 4 1+(1 / 2)m 1+(1 / 2)m
Illustration 9: Line through the point (–2, 6) and (4, 8) is perpendicular to the line through the point (8, 12) and
(x, 24). Find the value of x. (JEE MAIN)
Sol: Given two lines are perpendicular to each other. Therefore, product of their slope will be -1.
8–6 2 1
Slope of the line through the points (–2, 6) and (4, 8) is m1= = =
4 – (–2) 6 3
24 – 12 12
Slope of the line through the points (8, 12) and (x, 24) is m2 = =
x–8 x–8
Since two lines are perpendicular m1m2 = – 1
1 12
⇒ × = –1 ⇒x=4
3 x–8
7.3 Collinearity
C
B
Figure 8.23
PLANCESS CONCEPTS
A line of gradient m is equally inclined with the two lines of gradient m1 and m2.
m1 – m m2 – m
Then =- .
1+m1m 1+m2m
Aman Gour (JEE 2012, AIR 230)
(c) Slope intercept from: Case-I: If slope of line is m and makes y-intercept c, then equation is
(y – c) = m (x – 0) ⇒ y = mx + c
Case-II: If slope of line is m and makes x-intercept d, then equation is
y = m(x – d)
These equations are called slope intercept form. (0, c)
(d, 0)
Figure 8.26
M a them a ti cs | 8.13
(d) Intercept form: Suppose a line L makes intercept a on x-axis and intercept b on L
y-axis, i.e. the line meets x-axis at (a, 0) and y-axis (0, b). (0, b)
b–0
So, y – 0 = (x – a) b
0–a
x y
i.e. + = 1 . This is called intercept form of the line.
a b
a (a, 0)
Figure 8.27
(e) Normal form: If P is perpendicular distance from origin to the line AB and makes angle α with x-axis, then
equation of the line is x cos α + y sin α = P
OM y
Proof: cos α = OM = OL cos α = x cos α
OL
B
In DPNL,
PN
sin α =
PL
Q
PN = PL sin α = y sin α M P(x,y)
90-
MQ = PN = y sin α
N
x
O x A
Now P = OQ = OM + MQ = x cos α + y sin α L
Figure 8.28
So x cos α + y sin α = P
(f) Parametric form or distance form: The equation of the line passing through (x1, y1) and making an angle θ
with the positive x-axis is
x – x1 y – y1
= = r , where ‘r’ is the signed value.
cos θ sin θ
Hence, the co-ordinate of any point at a distance r on this line is
x = x1 + r cos θ y = y1 + r sin θ
PLANCESS CONCEPTS
Point of intersection of two lines a1x + b1y + c1 = 0 and a2x + b2y + c2 = 0 is given by
b1 b2 c1 c2
b1c2 – b2c1 c1a2 – c2a1 c1 c2 a1 a2
( x', y')= a b – a b , a b – a b = a1 a2
,
a1 a2
1 2 2 1 1 1 2 1
b1 b2 b1 b2
Illustration 10: A straight line is drawn through the point P(2, 3) and is inclined at an angle of 30° with positive
x-axis. Find the co-ordinate of two points on it at a distance 4 from P on either side of P. (JEE MAIN)
x – x1 y – y1
Sol: By using formula = = r , we can obtain co-ordinates of point.
cos θ sinθ
The equation of line
x – x1 y – y1 x–2 y –3
= = ±r ⇒ = = ± 4 ⇒ x = 2 ± 2 3, y = 3 ± 2
cos θ sinθ cos 30° sin 30°
Sol: (D) The line passing through the third vertex and orthocentre must be perpendicular to line through (–2, 3)
and (5, –1). Therefore, product of their slope will be -1.
Given the two vertices B(–2, 3) and C(5, – 1); let H(0, 0) be the orthocentre; A(h, k) the third vertex.
Then, the slope of the line through A and H is k/h, while the line through B and C has the slope
(–1 –3)/(5 + 2) = – 4/7. By the property of the orthocentre, these two lines must be perpendicular,
k 4 k 7
So we have − = – 1 ⇒ = … (i)
h
7 h 4
5 − 2 + h −1 + 3 + k
Also + = 7 ⇒ h + k = 16 … (ii)
3 3
Which is not satisfied by the points given in (A), (B) or (C).
Illustration 12: In what direction should a line be drawn passing through point (1, 2) so that its intersection point
6
with line x + y = 4 is at a distance of units. (JEE ADVANCED)
3
Sol: By using x = x1 + r cos θ and y = y1 + r sin θ, we can obtain the required angle.
For co-ordinates of B
6 6 6
Substitute r= ∴ x = 1+ cos θ & =
y 2+ sin θ B
3 3 3
A(1,2)
Substituting in x + y =4 6
3
6 6 3
⇒ 1+ cos θ + 2+ sin θ = 4 ∴ (cos θ + sin θ) =
3 3 6
1 1 3 1
∴ cos θ + sin θ = (Multiple by )
2 2 2 2 Figure 8.29
Illustration 13: If sum of the distances of the points from two perpendicular lines in a plane is 1, then find its locus.
(JEE ADVANCED)
Sol: If (h, k) be any point on the locus, then |h| + |k| = 1
Y
Let the two perpendicular lines be taken as the co-ordinate axes.
⇒ locus of (h, k) is |x| + |y| = 1 (0, 1)
x
y=
+
y
x-
shown in figure.
=1
X
(-1, 0) O (1, 0)
x
=1
+
y
y
x-
=1
(0, -1)
Figure 8.30
Illustration 14: If the circumcentre of a triangle lies at the origin and the centroid is the mid-point of the line
joining the points (a2 + 1, a2 + 1) and (2a, –2a), then the orthocentre lies on the line. (JEE ADVANCED)
(A) y = (a2 + 1)x (B) y = 2ax (C) x + y = 0 (D) (a – 1)2 x – (a + 1)2 y = 0
Sol: (D) We know from geometry that the circumcentre, centroid and orthocentre of a triangle lie on a line. So the
(a + 1)2 (a − 1)2
orthocentre of the triangle lies on the line joining the circumcentre (0, 0) and the centroid ,
2 2
2 2
(a + 1) (a − 1)
y= x or (a – 1)2 x – (a + 1)2 y = 0.
2 2
PLANCESS CONCEPTS
• A line passing through (x1, y1) and if the intercept between the axes is divided in the ratio m:n at this
nx my
point then the equation is + =m + n .
x1 y1
• The equation of a straight line which makes a triangle with the co-ordinates axes whose centroid is
x y
(x1, y1) is + =1.
3x1 3y1
B Rajiv Reddy (JEE 2012, AIR 11)
8 . 1 6 | Straight Line
b2 x – aby – ac a2 y – abx – bc
1 1
, 1 1 .
a2
+b 2
a2
+b 2 B(h, k)
Figure 8.31
The image of a point with respect to the line mirror: The image of A(x1, y1) with respect to the line mirror ax +
h – x1 k – y1 –2(ax1 +bx1 + c)
by + c = 0, B (h, k) is given by = =
a b a2 +b2 .
Special Cases
(a) Image of the point P(x1, y1) with respect to x-axis is (x1, – y1).
(b) Image of the point P(x1, y1) with respect to y-axis is (– x1, y1).
(c) Image of the point P(x1, y1) with respect to the line mirror y = x is Q(y1, x1).
(d) Image of the point P(x, y) with respect to the origin is the point (-x, -y).
Illustration 15: Find the equation of the line which is at a distance 3 from the origin and the perpendicular from
the line makes an angle of 30° with the positive direction of the x-axis. (JEE MAIN)
Sol: By using xcosα + ysinα = P, we can solve this problem. Here α = 30° and P = 3.
3 y
So equation is x cos 30° + y sin 30° = 3 x + =3 ⇒ 3x + y = 6
2 2
Position of a point w.r.t. a line
Let the equation of the given line be ax + by + c = 0 and let the co-ordinates of the two given points be P(x1, y1)
and Q(x2, y2). Let R1 be a point on the line.
m –ax1 – by1 – c
The co-ordinates of R1 which divides the line joining P and Q in the ratio m:n are =
n ax2 +by 2 + c
.
Thus, the two points (x1, y1) and (x2, y2) are on the same (or opposite) sides of the straight line ax + by + c = 0
m
whether Point R1 divides internally or externally or sign of .
n
Note:
⇒ A point (x1, y1) will lie on the side of the origin relative to a line ax + by + c = 0, if ax1 +by1 + c and c have the
same sign.
⇒ A point (x1, y1) will lie on the opposite side of the origin relative to the line ax + by + c = 0, if
ax1 + by1 + c and c have the opposite sign.
Illustration 16: For what values of the parameter α does the point M (α, α + 1) lies within the triangle ABC of
vertices A(0, 3), B(– 2, 0) and C(6, 1). (JEE ADVANCED)
Sol:Here, the point M will be inside the triangle if and only if |Area ∆MBC| + |Area ∆MCA| + |Area ∆MAB| = |Area
∆ABC|. And each individual area must be non-zero.
M a them a ti cs | 8.17
α α +1 1
1 1
Area MBC = –2 0 1 = 7α + 6
2 2
6 1 1
α α +1 1
1 1
Area MCA = 6 1 1 = –8α +12
2 2
0 3 1
α α +1 1
1 1
Area MAB = 0 3 1 = α+4
2 2
–2 0 1
Figure 8.32
0 3 1
1 1
Area ABC = –2 0 1 = .22
2 2
6 1 1
6 3
The above equation has critical points – 4, – and .
7 2
For α ≤ –4, the equation is
–7α – 6 – 8α + 12 – α – 4 =22
5 5
⇒ α=– which is not a solution, since – > –4
4 4
6 6
For a∈ –4, – , then equation is – 7α – 6 – 8α + 12 + α + 4 = 22 ⇒ α = –
7 7
6
which is solution of equation but area MBC = 0 ⇒ M lies on BC ⇒ α = – is not the desired value.
7
6 3
For a∈ – , , the equation is 7α + 68α + 12 + α + 4 = 22.
7 2
6 3
⇒ All α in the interval – , satisfy the equation.
7 2
3 3
Finally over , ∞ , we get α = implies area MCA become zero.
2 2
6 3
⇒ The desired values of α lie in the interval – , .
7 2
x+ by + c1 = 0 and ax + by + c2 = 0 is c1 – c2
a2 +b2 y=mx+c
(
(b) Lines making angle α with given line: The equations of the two straight lines (
passing through P(x’, y’) and making an angle α with the line y = mx + c P(x’,y’)
y – y’ = tan (θ + α) (x – x’)
π
Note: If θ + α or θ – α is an odd multiple of , the corresponding line has equation x = x’.
2
(c) Concurrency of lines: Lines aix + biy + c1 = 0, where i = 1, 2, 3 are concurrent if they meet at a point. The
a1 b1 c1
condition for concurrency is
a2 b2 c2 = 0
a3 b2 c3
Illustration 17: The equation of the two tangents to the circle are 3x – 4y + 10 = 0 and 6x – 8y + 30 = 0. Find
diameter of the circle. (JEE MAIN)
c1 – c2
Sol: By using formula of distance between two parallel line, i.e. , we can find the
a2 +b2
diameter of given circle. d
6x – 8y + 30 = 0 .....(ii)
15 − 10
Dividing second equation by 2 gives 3x – 4y + 15 = 0; ∴ d = =1
32 + 42
PLANCESS CONCEPTS
b b
A a B
Figure 8.36
(ii) Whose length of perpendicular from one vertices to the opposite sides are p1 and p2 and angle
PP
between sides is θ is given by Area = 1 2
Sin θ
p2
p1
θ
Figure 8.37
8. FAMILY OF LINES
Consider two intersecting lines L1: a1x + b1y + c1 = 0 and L2: a2x + b2y + c2 = 0, then
Type-1: The equation of the family of lines passing through the intersection of the lines
L1 + λL2 = 0
⇒ (a1x +b1y + c1) + λ(a2x +b2y + c2) = 0 where λ is a parameter.
Type-2: Converse, L1 + λL2 = 0 is a line which passes through a fixed point, where L1 = 0 and L2=0 are fixed lines
and the fixed point is the intersection of L1 and L2.
u1a1x+b1y+c1=0
B C
u2a2x+b2y+d2=0 u4a2x+b2y+d1=0
A D
u3a1x+b1y+c2=0
Figure 8.38
Note that second degree terms cancel and the equation u2u3 – u1u4 = 0 is satisfied by the co-ordinate points B
and D.
Illustration 18: If a, b, c are in A.P., then prove that the variable line ax + by + c = 0 passes through a fixed point.
(JEE MAIN)
Sol: By using given condition we can reduce ax + by + c = 0 to as L1 + λL2 = 0. Hence we can obtain co-ordinate
of fixed point by taking L1 = 0 and L2=0.
2b =a + c ⇒ c = 2b – a ⇒ ax + by + 2b – a = 0
∴ a (x – 1) + b(y + 2) = 0 This is of the form L1 + λL2 = 0, where b/a = l
∴ Co-ordinates of fixed point is (1, – 2).
9. ANGULAR BISECTOR
(c) Acute angle bisector and obtuse angle bisector can be differentiated from the following methods:
Let two lines a1x + b1y + c1 = 0 and a2x + b2y + c2 = 0 intersect such that constant terms are positive.
8 . 2 0 | Straight Line
If a1a2 + b1b2 < 0, then the angle between the lines that contain the origin is acute and the equation for
a x +b1 y + c a x +b2 y + c2 ax1 +by1 + c a x +b2 y + c2
the acute angle bisector is 1 =+ 2 . Therefore =– 2 is the
2 2 2 2 2 2
a +b a2 +b2 a +b a22 +b22
equation of other bisector. If, however, a1a2 + b1b2 > 0, then the angle between the lines containing the origin
a1 x +b1 y + c1 a2 x +b2 y + c2
is obtuse and the equation of the bisector of the obtuse angle is =+ ; therefore
a1 x +b1 y + c1 a2 x +b2 y + c2 a12 +b12 a22 +b22
=– is acute angle bisector.
a12 +b12 a22 +b22
(d) Few more methods of identifying an acute and obtuse angle bisectors are as follows:
Let L1 = 0 and L2 = 0 are the given lines and u1 = 0 and u2 = 0 are the bisectors L1=0
between L1 = 0 and L2 = 0. Take a point P on any one of the lines L1 = 0 or L2 = 0 and
draw a perpendicular on u1 = 0 and u2 = 0 as shown. If P p
q
|p| < |q| ⇒ u1 is the acute angle bisector. L2=0
Note: The straight lines passing through P(x1, y1) and equally inclined with the lines u2=0
u1=0
a1x + b1y + c1 = 0 and a2x + b2y + c2 = 0 are those which are parallel to the bisectors
between lines and passing through the point P. Figure 8.39
PLANCESS CONCEPTS
(a) Algorithm to find the bisector of the angle containing the origin: Let the equations of the
two lines be a1x + b1y + c1 = 0 and a2x + b2y + c2 = 0. The following methods are used to find the
bisector of the angle containing the origin:
Step I: In the equations of two lines, check if the constant terms c1 and c2 are positive. If the terms are
negative, then make them positive by multiply both the sides of the equation by –1.
Step II: Obtain the bisector corresponding to the positive sign, i.e. a1 x +b1 y + c1 a2 x +b2 y + c2
=
a12 +b12 a22 +b22
L1
Acute bisector
L2
Obtuse bisector
Figure: 8.40
This is the required bisector of the angle containing the origin, i.e. the bisectors of the angle between the
lines which contain the origin within it.
(b) Method to find acute angle bisector and obtuse angle bisector
(i) Make the constant term positive by multiplying the equation by –1.
(ii) Now determine the sign of the expression a1a2 + b1b2.
M a them a ti cs | 8.21
PLANCESS CONCEPTS
(iii) If a1a2 + b1b2 > 0, then the bisector corresponding to ‘+ve’ and ‘–ve’ signs give the obtuse and acute
angle bisectors, respectively, between the lines.
(iv) If a1a2 +b1b2 < 0, then the bisector corresponding to ‘+ve’ and ‘–ve’ signs give the acute and obtuse
angle bisectors, respectively.
Both the bisectors are perpendicular to each other. If a1a2 + b1b2 > 0, then the origin lies in the
obtuse angle and if a1a2 + b1b2 < 0, then the origin lies in the acute angle.
PLANCESS CONCEPTS
Incentre divides the angle bisectors in the ratios (b + c):a, (c + a):b and (a + b):c . Angle bisector divides
the opposite sides in the ratio of remaining sides.
Figure: 8.41
BD AB c
= =
DC AC b
Aishwarya Karnawat (JEE 2012, AIR 839)
Illustration 19: The line x + y =a meets the x- and y-axes at A and B, respectively. A triangle AMN is inscribed in
the triangle OAB, O being the origin, with right angle at N. M and N lie respectively on OB and AB. If the area of
the triangle AMN is 3/8 of the area of the triangle OAB, then AN/BN is equal to. (JEE ADVANCED)
(A) 3 (B) 1/3 (C) 2 (D) 1/2
1 –a 1 – λ 2 λa2
= a
+ a =
2 λ + 1 (1 + λ )
2
(1 + λ )2
2h a
m1 + m2 =
– and m1m2 =
b b
Combined equation of angle bisectors passing through origin: The combined equation of the bisectors of
the angles between the lines ax2 + 2hxy + by 2 =
0 (a pair of straight lines passing through origin) is given by
x2 – y 2 xy
= .
a–b h
(b) The slope of the two lines represented by a general equation is the same as that between the two lines
represented by only its homogeneous part.
10.3 Homogenisation
The equation of the two lines joining the origin to the points of intersection of the line lx + my +n = 0 and the curve
ax2 + 2hxy + by 2 + 2gx + 2fy + c =0 is obtained by homogenising the equation of the curve using the equation of
the line.
The combined equation of pair of straight lines joining origin to the points y
of intersection of the line given by lx + my + n = 0 …. (i)
The second degree curve:
lx+my+n=0
ax2 + 2hxy + by 2 + 2gx + 2fy + c =
0 …. (ii)
Using equation (i) and (ii) x
O
2
lx + my lx + my lx + my
ax2 + 2hxy + by 2 + 2gx + 2fy + c 0
= …. (iii) Figure 8.44
–n –n –n
lx + my
Obtained by homogenizing (ii) with the help of (i), by writing (i) in the form: = 1.
–n
PLANCESS CONCEPTS
Through a point A on the x-axis, a straight line is drawn parallel to y-axis so as to meet the pair of straight
lines.
ax2 + 2hxy + by 2 =
0 in B and C. If AB = BC, then 8h2 = 9ab.
Krishan Mittal (Jee 2012, Air 199)
8 . 2 4 | Straight Line
Illustration 20: The orthocentre of the triangle formed by the lines xy = 0 and x + y = 1 is (JEE MAIN)
(A) (1/2, 1/2) (B) (1/3, 1/3) (C) (0, 0) (D) (1/4, 1/4)
Illustration 21: If θ is an angle between the lines given by the equation 6x2 + 5xy – 4y 2 + 7x + 13y – 3 = 0 then
equation of the line passing through the point of intersection of these lines and making an angle θ with the positive
x-axis is (JEE ADVANCED)
(A) 2x + 11y + 13 = 0 (B) 11x – 2y + 13 = 0
(C) 2x – 11y + 2 = 0 (D) 11x + 2y – 11 = 0
Sol: (B) By taking the term y constant and using the formula of roots of quadratic equation, we can get the
2 h2 – ab
equation of two lines represented by the given equation and then by using tan θ = , we will get the
required result. a+b
⇒ 2x – y + 3 = 0 and 3x + 4y – 1 = 0 are the two lines represented by the given equation and the point of
intersection is (– 1, 1), obtained by solving these equations.
Illustration 22: If the equation of the pair of straight lines passing through the point (1, 1), and making an angle
θ with the positive direction of x-axis and the other making the same angle with the positive direction of y-axis is
x2 – (a + 2)xy + y 2 + a(x + y – 1) =
0 , a ≠ – 2, then the value of sin 2θ is (JEE ADVANCED)
2 2
(A) a – 2 (B) a + 2 (C) (D)
(a + 2) a
Sol: (C) As both line passes through (1, 1) and one line makes angle θ with x-axis and other line with y–axis, slopes
of line are tan θ and cot θ
Equations of the given lines are y – 1 = tan θ (x – 1) and y – 1 = cot θ (x – 1)
So, their combined equation is [(y – 1) – tan θ (x – 1)] [(y – 1) – cot θ (x – 1) ] = 0
⇒ (y – 1)2 – (tan θ + cot θ) (x – 1) (y – 1) + (x – 1)2 = 0
⇒ x2 – (tan θ + cot θ) xy + y2 + (tan θ + cot θ – 2) (x + y – 1) = 0
Comparing with the given equation we get tan θ + cot θ = a + 2
1 2
⇒ = a + 2 ⇒ sin2θ =
sin θ cos θ a+2
M a them a ti cs | 8.25
Sol: (D) As the product of the slopes of the four lines represented by the given equation is 1 and a pair of line
represents the bisectors of the angles between the other two, the product of the slopes of each pair is –1.
So let the equation of one pair be ax2 + 2hxy – ay2 = 0.
x2 – y 2 xy
The equation of its bisectors is = .
2a h
By hypothesis x 4 + x3 y + cx2 y 2 – xy 3 + y 4
≡ (ax2 + 2hxy – ay2) (hx2 – 2axy – hy2) = ah(x4 + y4) + 2(h2 – a2) (x3y – xy3) – 6ahx2y2
Comparing the respective coefficients, we get ah =1 and c = –6ah = –6
PLANCESS CONCEPTS
If origin is shifted to point (α, β) , then new equation of curve can be obtained by putting x + α in place
of x and y + β in placed of y.
Vaibhav Krishnan (JEE 2009, AIR 22)
Illustration 24: The line L has intercepts a and b on the co-ordinate axes. The co-ordinate axes are rotated through
a fixed angle, keeping the origin fixed. If p and q are the intercepts of the line L on the new axes, then
1 1 1 1
– + – is equal to (JEE MAIN)
2 2 2
a p b q2
(A) –1 (B) 0 (C) 1 (D) None of these
8 . 2 6 | Straight Line
Sol: (B) By using intercept form of equation of line, we will get equation of line before and after rotation. As their
perpendicular length from the origin does not change, by using distance formula the result can be obtained.
x y X Y
Equation of the line L in the two co-ordinate system is + = 1, + = 1 Where (x, y) are the new co-ordinates
a b p q
of a point (x, y) when the axes are rotated through a fixed angle, keeping the origin fixed. As the length of the
perpendicular from the origin has not changed:
1 1 1 1 1 1 1 1 1 1
= ⇒ + = + ⇒ – + – =0
2 2 2 2 2 2 2
(1 / a2 ) + (1 / b2 ) (1 / p2 ) + (1 / q2 ) a b p q a p b q2
Illustration 25: Let 0 < α < π/2 be a fixed angle. If P = (cos θ, sin θ) and Q = (cos (α – θ), (sin (α – θ)). Then Q is
obtained from P by (JEE ADVANCED)
(A) Clockwise rotation around origin through an angle α.
(B) Anti-clockwise rotation around origin through an angle α.
(C) Reflection in the line through the origin with slope tan α.
(D) Reflection in the line through the origin with slope tan α/2.
Sol: As we know angle decreases during clockwise rotation and increases during anticlockwise rotation.
D Clockwise rotation of P through an angle α takes it to the point (cos (θ – α), sin(θ – α)) and anticlockwise takes it
to (cos (α + θ), sin (α + θ))
sin θ – sin(α – θ) 2cos(α / 2)sin(θ – α / 2)
Now slope of PQ = = − cot ( α / 2 )
=
cos θ – cos(α – θ) –2sin(α / 2) – sin(θ – α / 2)
⇒ PQ is perpendicular to the line with slope tan(α/2). Hence, Q is the reflection of P in the line through the origin
α
with slope tan .
2
PLANCESS CONCEPTS
a1 b1 c1
• Parallel, if = ≠
a2 b2 c2
a1 b1
• Intersecting, if ≠
a2 b2
• Perpendicular, if a1a2 + b1b2 = 0
The three lines a1x + b1y + c1 = 0, a2x + b2y + c2 = 0 and a3x + b3y + c3 = 0 are concurrent, if
a1 b1 c1
a2 b2 c2 = 0
a3 b3 c3
Aman Gour (JEE 2012, AIR 230)
M a them a ti cs | 8.27
PROBLEM-SOLVING TACTICS
(a) In most of the questions involving figures like triangle or any parallelogram, taking origin as (0,0) helps a lot
in arriving at desired solution. One must ensure that conditions given are not violated.
(b) One must remember that in an isosceles triangle, centroid, orthocentre, incentre and circumcentre lie on the
same line.
(c) The centroid, incentre, orthocentre and circumcentre coincide in an equilateral triangle.
(d) If area of the triangle is zero, then the three points are collinear.
(e) Find the equation of the line passing through two given points, if the third point satisfies the equation of the
line, then three points are collinear
(f) Whenever origin is shifted to a new point (α, β), then new equation can be obtained by putting x + α in place
of x and y + β in placed of y.
FORMULAE SHEET
(a) Distance Formula: The distance between two points P(x1, y1) and Q(x2, y2) is
And between two polar co-ordinate A(r1, q1) and B(r2, q2) is AB = r12 + r22 − 2r1r2 cos(θ1 − θ2 )
(b) Section Formula: If P(x1, y1), Q(x2, y2) and the point R(x, y) divide the line PQ internally in the ratio m:n then
the co-ordinates of R will be
mx2 + nx1 my 2 − ny1 mx + nx1 my 2 + ny1
x= and y = i.e. R 2 ,
m+n m−n , m+n m + n
x + x 2 y1 + y 2
And if R is a mid-point of line PQ, then the co-ordinates of R will be 1 ,
2 2
(c) Centroid of Triangle: If A(x1, y1), B(x2, y2) and C(x3, y3) be the vertices of the triangle ABC and G is
x + x 2 + x3 y1 + y 2 + y 3
Centroid, then co-ordinate of G will be 1 , .
3 3 A
I3 I2
ax1 + bx2 + cx3 ay1 + by 2 + cy 3
(d) Co-ordinates
= of Incentre: x = , y B C
a+b+c a+b+c
(f) Co-ordinates of Circumcentre: If O is the circumcentre of any ∆ABC, then its co-ordinates will be
(g) Co-ordinates of Orthocentre: If H is the orthocentre of any ∆ABC, then its co-ordinates will be
(k) Length of Perpendicular: The perpendicular distance ‘p’ of a point P(x1, y1) from the line ax + by + c = 0 is
| ax1 +by1 + c |
p =
a2 +b2
c1 – c2
(i) Distance between parallel lines:
a2 +b2
(ii) Lines making angle α with given line: y – y’ = tan (θ + α) (x – x’) and y – y’ = tan (θ – α) (x – x’)
a1 b1 c1
(iii) Concurrency of lines: The lines are concurrent if a2 b2 c2 = 0
a3 b2 c3
ax +by + c a1 x +b1 y + c1
(l) Equation of bisector of the angle between two lines: =± (ab1 ≠ a1b)
a2 +b2 a12 +b12
(m) Pair of straight line:
(i) ax2 + 2hxy + by2 = 0 always represents a pair of straight lines passing through the origin and if
•• h2 > ab ⇒ lines are real and distinct.
•• h2 = ab ⇒ lines are coincident.
•• h2 < ab ⇒ lines are imaginary with real point of intersection, i.e. (0, 0)
2h a
(ii) m1 + m2 =
− and m1m2 =
b b
2 h2 – ab
(iii) tan θ =
a+b
(n) General equation for pair of straight lines: ax2 + 2hxy + by 2 + 2gx + 2fy + c =0 represents a pair of straight
a h g
lines, if h b f = 0
g f c
M a them a ti cs | 8.29
Solved Examples
JEE Main/Boards 20 2 – 10 2 + 30 2 5
= =
5 2 +5 2 + 6 2 2
Example 1: Find the ratio in which y – x + 2 = 0 divides
the line joining A (3, – 1) and B (8, 9). ay + by 2 + cy 3
y= 1
a+b+c
Sol: By considering the required ratio be λ:1, and using
section formula, we can solve above problem. –10 2 + 20 2 + 30 2 5
= =
The point of division P is internal as A and B lie on 5 2 +5 2 + 6 2 2
opposite sides of given line.
Let required ratio be λ:1. Example 3: A rectangle PQRS has its side PQ parallel to
the line y = mx and vertices P, Q, S lie on lines y = a, x = b
8λ + 3 9λ – 1 and x = –b, respectively. Find the locus of the vertex R.
Since, P , lies on y – x + 2 = 0,
λ +1 λ +1
Sol: Here sides PQ and QR must be perpendicular to
9λ – 1 8λ + 3 2
∴ – +2 = 0 or λ = each other. Therefore product of their slopes will be –1.
λ +1 λ +1 3
Let R(h, k) be any point on the locus and let S and Q
y-x+2=0
have co-ordinates (–b, β) and (b, α), respectively, as T is
mid-point of SQ and PR.
1 α–a
A B Thus P has co-ordinates (–h, a) =m
P b+h
⇒ α = a + m (b + h)
1 α –k
– = slope of QR =
Hence, required ratio is 2/3:1 or 2:3 m b–h
1
⇒α=k– (b – h)
Example 2: Find the incentre I of ∆ABC, if A is (4, – 2) B m
is (– 2, 4) and C is (5, 5). 1
a + m(b + h) = k – (b – h)
ax1 + bx2 + cx3 ay1 + by 2 + cy 3 m
Sol: Using x
= = , y , ∴ locus of R is
a+b+c a+b+c
we can obtain the incentre. x(m2 – 1) 2 my + b + am + bm2 = 0.
a = BC = (5 + 2)2 + (5 – 4)2 =
5 2 Example 4 Two equal sides AB and AC of an isosceles
b = CA = 2 2
(5 – 4) + (5 + 2) = 5 2 triangle ABC have equation 7x – y + 3 = 0 and x + y – 3
= 0, respectively. The third side BC of the triangle passes
c = AB = (–2 – 4)2 + (4 + 2)2 = 6 2 through point P(1, – 10). Find the equation of BC.
⇒ (x + m1y) (x + m2y) = 0
–1 –1
⇒ y
= = x;y x
m1 m2
B D C P
1 Example 7: Find the angle φ between the straight lines
As slope of AD is –3, slope of BC is
3 π
Equation of BC through P(1, –10) is (x2 + y2) sin2α = (x cos θ – y sin θ)2, where 0 < 2α < .
1 2
y + 10 = (x – 1) or x – 3y = 31.
3 2 h2 – ab
Sol: We know tan φ = . Solving it, angle φ
can be obtained. (a + b)
Example 5: Find the equation of the line passing
through the intersection of lines x – 3y + 1 = 0, 2x +
x2 (cos2 θ – sin2 α ) – 2xy cos θ sin θ + y 2 (sin2 θ – sin2 α ) =
0
5y – 9= 0 and whose distance from the origin is 5.
a = cos2θ – sin2α, 2h = – 2 cos θ sin θ,
Sol: Equation of any line passing through the
b = sin2θ – sin2α
intersection of two other lines will be L1 + λL2 = 0.
Therefore, by using perpendicular distance formula of 2 h2 – ab
point to line, i.e = tan φ =
(a + b)
| ax1 +by1 + c |
p=
a2 +b2 2 cos2 θ sin2 θ – (cos2 θ – sin2 α )(sin2 θ – sin2 α )
=
we can obtain required equation of line. | (cos2 θ – sin2 α ) + sin2 θ – sin2 α |
Any line through the point of intersection of given lines is
2sin α cos α sin2α
= = = tan2α ∴ φ = 2a
x – 3y + 1 + λ (2x + 5y – 9) = 0 cos2α cos2α
or (1 + 2λ) x + (– 3 + 5λ) y + 1 – 9λ = 0
Example 8: The point A divides the line joining P ≡ (–5, 1)
| 0 + 0 + 1 – 9λ | and Q = (3, 5) in the ratio k:1. Find the two values of k for
5=
(1 + 2λ )2 + (–3 + 5λ )2 which the area of ∆ABC where B ≡ (1, 5), C ≡ (7, 2) is equal
to two square units.
7
Squaring and simplifying, we get λ = .
8 Sol: By using section formula, we can obtain the co-
ordinates of point A and then values of k by using the
Hence, required line has the equation 2x + y – 5 = 0.
triangle formula.
Co-ordinates of A, dividing the line joining points
Example 6: Show that bx2 – 2hxy +ay2 = 0 represent a
P ≡ (– 5, 1) and Q ≡ (3, 5) in the ratio k:1, are given by
pair of straight lines which are at right angles to the pair
of lines given by ax2 + 2hxy + by2 = 0. (3k – 5 / k + 1, 5k + 1 / k + 1) . Also, area of the ∆ABC
1
Sol: Here if the product of slopes of a pair of straight is given by ∆ = ∑ x1 (y 2 – y 3 )
2
lines represented by the given equations is –1, then
1
they are right angle to each other. = | [x (y – y 3 ) + x2 (y 3 – y1 ) + x3 (y1 – y 2 )] |
2 1 2
Let ax + 2hxy + by
2 2
1 3k – 5 5k + 1 5k + 1
= (y – m1x) ( y – m2x) (7) + –2 – + 7 – 5 = 2
2 k + 1 k + 1 k + 1
M a them a ti cs | 8.31
3k – 5 5k + 1 5k + 1 JEE Advanced/Boards
(7) + –2 – + 7 – 5 =
±4
k +1 k +1 k +1
Example 1: If A(–1, 5), B(3,1) and C(5, 7) are vertices of
⇒ 14k – 66 =
4k + 4,10k =
70,k =
7 a ∆ABC and D, E, F are the mid-points of BC, CA and AB,
respectively, then show that area ∆ABC = 4 times area
or 14k – 66 = – 4k – 4, 18k = 62,
(∆DEF).
k = (31/9).
Sol: Co-ordinates of D, E and F are first obtained by
Therefore, value of the k = 7, 31/9
using mid-point formula, and prove the given equation
Example 9: Prove that the sum of the reciprocals of the by using formula of area of triangle.
intercepts made on the co-ordinate axes by any line Co-ordinates of D, E, F are (4, 4), (2, 6) and (1, 3),
not passing through the origin and through the point respectively.
of intersection of the lines 2x + 3y = 6 and 3x + 2y = 6 –1 5 1
is constant. 1
∴ Area of ∆ABC = 3 1 1 = 16
2
5 7 1
Sol: Equation of any line through the points of
intersection of the given lines is L1 + λL2=0.
4 4 1
2x + 3y – 6 + k (3x + 2y – 6) =
0 1
Area of ∆DEF = 2 6 1 =4
2
(2 + 3k) x + (3 + 2k)y – 6(k + 1) =
0 1 3 1
x y A
⇒ + 1
=
((6(k + 1)) / (2 + 3k)) ((6(k + 1)) / (3 + 2k))
F E
Where k ≠ – 1
and in this case, sum of the reciprocals of the intercepts
made by this line on the co-ordinate axis is equal to
B C
D
2 + 3k + 3 + 2k 5(k + 1) 5 Hence, area of ∆ABC = 4 area (∆DEF)
= = .
6(k + 1) 6(k + 1) 6
Example 2: Point P(a2, a + 1) is a point of the angle (which
However, for k = – 1, the line become
contains the origin) between the lines 3x – y + 1 = 0,
x = y, which passes through the origin. x + 2y – 5 = 0. Find interval for values of ‘a’.
Sol: As the geometrical fact to be established does or y (cos α – sin α) = x (cos α + sin α) ….(i)
not depend on position of ABC, we may assume that y
‘’D is the origin; BC and AD are along x and y axes B
respectively (as shown)”. Therefore by using intercept
form of equation of line, we can obtain required result.
C
Let BD = DC = a, and A and E have co-ordinates (0, b)
and (h, k), respectively. A
/4
x y
Line AC has the equation + = 1
a b O
x
y
A(0,b) From the figure, point A is (a cos α, a sin α).
As diagonal AC is perpendicular to diagonal OB,
equation of AC is
cos α – sin α
E Y – a sin α = (x – acos α )
F cos α + sin α
or x (cos α – sin α) + y (cos α + sin α) = a
x
B(-a,0) D C(a,0)
Example 6: Two sides of a rhombus lying in the first
h k quadrant are given by 3x – 4y = 0 and 12x – 5y = 0. If
∴ + =1 ….(i)
a b the length of the longer diagonal is 12 units, find the
equations of the other two sides of the rhombus.
Also, (k/h) (–b/a) = – 1
( AC ⊥ DE) ….(ii) Sol: Using formula of equation of bisector of the angle,
we can obtain equation of AC. Given the length AC, we
M a them a ti cs | 8.33
Since AC is perpendicular to y – 9x = 0,
y –β
B We have 2 (9) = –1 Rewrite (i) and (ii) as 9x2 – y2
x2 – α
3x - 4y = β – 9a
x
A x2 + 9y2 = α + 9β
Similarly, we obtain
Example 8: If the vertices of a triangle have integral
co-ordinates, prove that the triangle cannot be equilateral. CQ lx + my 3 + n
=– 3 …. (ii)
QA lx1 + my1 + n
Sol: Obtaining the area of triangle using D = (1/2) bc
sin A and using the co-ordinate form, we can conclude AR lx + my1 + n
and =– 1 …. (iii)
that the triangle cannot be equilateral if vertices have RB lx2 + my 2 + n
integral co-ordinates.
Multiplying (i), (ii) and (iii), we get the required
Consider a triangle ABC with vertices A ≡ (x1, y1),
result.
B ≡ (x2, y2), C ≡ (x3, y3). Let x1, x2, x3, y1, y2, y3 be the integers.
BC2 = (x2 – x3)2 + (y2 – y3)2 a positive integer. Example 10: The circumcentre of a triangle having
vertices A = (a, a tan α), B = (b, b tan β), C = (c, c tan
If the triangle is equilateral, then AB = BC = CA = a
γ) is the origin, where α + β + γ = π. Show that the
and ∠A =∠B =∠C =60° .
orthocentre lies on the line.
Area of the triangle = (1/2) bc sin A = (1/2) a2 sin 60°
α β γ α β γ
= (a2 / 2).( 3 / 2) = ( 3 / 4)a2 which is irrational. 4 cos cos cos x – 4 sin sin sin y – y = 0
2 2 2 2 2 2
a2 is a positive integer.
Sol: Consider the circumcentre ‘O’ to be the origin and
Now, the area of the triangle in terms of the co-ordinates
the equation of the circumcircle be x2 + y2 = r2. As
= (1/2) [(x1(y2 – y3) + x2 (y3 – y1) + x3 (y1 – y2 )] vertices of triangle lies on this circle, we can obtain the
which is rational number. co-ordinates of centroid by using the respective
formula.
This contradicts that the area is an irrational number, if
the triangle is equilateral. Since vertex A(a, a tan α) is r distance from the
circumcenter.
Example 9: A line L intersects the three sides BC, CA Therefore a2(1 + tan2α) = r2 ⇒ a = r cos α
and AB of a triangle ABC at P, Q and R, respectively.
A = (r cos α, r sin α)
BP CQ AR Similarly B = (r cos β, r sin β)
Show that = –1 .
PC QA RB
C = (r cos γ, r sin γ)
Sol: Using equation of line lx + my + n = 0 and section A
formula, we can prove the given equation.
O
Consider a triangle ABC with vertices A(x1, y1), B(x2, y2) G
and C(x3, y3), and let lx + my + n = 0 be equation of H
λx + x 2 λy 3 + y 2
ordinates of P are 3 ,
λ +1 λ + 1 Centroid G
Also, as P lies on L, we have
r(cos α + cos β + cos γ ) r(sin α + sin β + sin γ )
λx + x 2 λy 3 + y 2 ,
l 3 + m 3 3
+ n =0
λ +1 λ +1
Circumcentre O’ (0, 0) and let orthocentre H (h, k).
A We know that O, G, H are collinear. Therefore slope of
R OG = slope of OH
Q sin α + sin β + sin γ k
i.e. = '
cos α + cos β + cos γ h
1B P
C Point (h, k) will be on
M a them a ti cs | 8.35
Respectively, where t is a parameter. Find the Solving (i) and (ii), we get x2 =10, y2 = 5,
locus of the centroid of the ∆ABC. i.e. the co-ordinates of C are (10, 5) and thus
the equation of AC is 6x – 7y = 25
Sol: We can obtain co-ordinates of centroid G(α, β)
x + x 2 + x3 y1 + y 2 + y 3 A(3, -1)
using the formula 1 , and
3 3 Angle bisector
we will get required equation of locus of centroid by Median
solving them simultaneously.
Let G(α, β) be the centroid in any position .Then G(α, β) 6x + 80y - 59 = 0
x - 4y + 10= 0
1 + cos t + sint 2 + sint – cos t B(x1, y1) C(x2, y2)
= – or
3 3
Let the slope of BC be m1. Since BC and AC are equally
1 + cos t + sint 2 – sint – cos t
=∴ α = ,β inclined to the angle bisector
3 3
x – 4y + 10 = 0,
∴ or 3α – 1 = cos t + sin t …. (i)
(1 / 4) – m (6 / 7) – (1 / 4) 1 – 4m 17
3β =2 – sin t – cos t …. (ii) = ⇒ =
1 + (1 / 4)m 1 + (6 / 7) × (1 / 4) 4 + m 34
Squaring and adding equations (i) and (ii), we get
2
(3α – 1)2 + (3β – 2)2 ⇒m= – Equation of BC is
9
= (cos t + sin t)2 + (sin t – cos t)2 2
y – 5 = – (x – 10) and 6x1 + 10y1 = 59
= 2 (cos t + sin t) = 2
2 2
9
∴ the equation of the locus of the centroid is (3x – 1)2+ Solving these equations, we get X1 = – 7/2, y1 = 8
(3y – 2)2 = 2
8 +1
Hence, equation of AB is y + 1 = (x – 3)
9(x2 + y2) –6 x – 12y + 3 = 0 –7 / 2 – 3
∴ 3(x2 + y2) – 2x – 4y + 1 = 0
Example 13: A triangle has the lines y = m1x and y
Example 12: Find equations of the sides of the = m2x for two of its sides, with m1 and m2 being roots
triangle having (3, –1) as a vertex, x – 4y + 10 = 0 and of the equation bx2 + 2hx + a = 0. If H(a, b) is the
6x + 10y – 59 = 0 being the equations of an angle orthocentre of the triangle, show that the equation of
bisector and a median, respectively, drawn from the third side is (a + b) (ax + by) =ab(a + b – 2h).
different vertices.
Sol: Line OD passes from orthocentre. Therefore it
Sol: Consider the vertices of the triangle to be A(3, – 1), must be perpendicular to the side AB. By considering
B(x1, y1) and C(x2, y2). Here the mid-point of AC lies on equation of AB as y = mx +c, we will get co-ordinates
the median through B. of A and B. Using slope point form of equation of line,
we can solve the problem.
Equation of the median through B be 6x + 10y – 59 = 0
and the equation of the angle bisector from C be The given lines y = m1x and y = m2x intersect at the
origin O (0, 0). Thus one vertex of the triangle is at the
x – 4y + 10 = 0; x2 – 4y2 + 10 = 0 ….(i)
origin O. Therefore, let OAB be the triangle and OA and
8 . 3 6 | Straight Line
Let the equation of the third side AB be or (ax + by)(a + b)= ab(a + b – 2h)
y = mx +c …..(iii)
Given that H(a, b) is the orthocentre of the OAB, Example 14: Find the co-ordinates of the centroid,
circumcentre and orthocentres of the triangle
∴ OH ⊥ AB formed by the lines 3x – 2y = 6, 3x + 4y + 12 = 0 and
⇒ (b/a) × m = – 1 ⇒ m = –a/b ….(iv) 3x – 8y + 12 = 0.
Solving (iii) with (i) and (ii), the co-ordinates of Sol: Solving the given equations, we can obtain the
c cm1 co-ordinates of vertices of triangle. Using appropriate
A = , and formula for finding the co-ordinates of centroid,
m
1 – m m1 –m
circumcentre and orthocentre, the problem can be
c cm2 solved.
B = ,
m2 – m m2 – m Let sides AB, BC and CA have the
perpendicular to OB is 3x – 8y + 12 = 0 ….(ii)
3x + 4y + 12 = 0 ….(iii)
cm1 1 c
y– =– x – or Solving (ii), (iii) we get y = 0, x = – 4,
m1 – m m2 m1 – m
A
x c(m1m2 + 1)
y= – + ….(v)
m2 m2 (m1 – m)
perpendicular to OA is
x c (m1 m2 + 1)
y= − + ….(vi)
m1 m1 (m2 − m) B C
3x - 8y+12=0
C = (– 4, 0)
The point of intersection of (v) and (vi) is the orthocentre
H (a, b). Solving (i), (ii) we get y = 3, x = 4
∴ Subtracting (vi) from (v), we get
B = (4, 3)
–cm(m1m2 + 1)
x=a=
(m1 – m)(m2 – m) Solving (i,) (iii) we get y = – 3, x = 0;
–[m1m2 – m(m1 + m2 ) + m2 ]a A = (0 – 3)
or c = …..(vii)
m(m1m2 + 1)
M a them a ti cs | 8.37
2 α + 3 β – 4 =0 …. (i) C
8 α – 6 β + 7 =0 …. (ii)
1 23
Solving (i) and (ii), we get =
α ,=
β
12 18 (0,b)D
E
45 B
1 23 45 (a, 0)
Circumcentre = ,
12 18
O
A(0, -3)
A
E
a–b (a + b) a–b
m= or – AB is y – 0 = (x – a)
M a+b a–b a+b
a+b
AD is y– b = – (x – 0)
B(4, 3) D C(-4, 0) a–b
By solving these equation we get
MG 1 a–b b – a
Use = = The point A is
GH 2 , . C is obtained by using the
2 2
fact that mid-point of AC and BD is same.
Let H(α, β) be the orthocentre
1 a+b a+b
α+2 C= ,
12 1 2 2
=0 = ⇒α – …. (i)
3 6
The opposite vertices B, D move on two perpendicular
23 lines x-axis and y-axis. Now the point
β+2 –1 –23
18 23
=0 = ⇒β – Then H , . a–b b – a a+b a+b
3 9 6 9 A , lies on y = –x and point C ,
2 2 2 2
lies on y = x.
8 . 3 8 | Straight Line
Q(x2,y2)
JEE Main/Boards
Q.2 Show that the line joining (2, – 3) and (– 5, 1) Q.8 Find the equation of the straight line which passes
is (i) Parallel to the line joining (7, – 1) and (0, 3), through the origin and trisects the intercept of line
(ii) Perpendicular to the line joining (4, 5) and (0, – 2). 3x + 4y = 12 between the axes.
Q.3 A quadrilateral has the vertices at the points Q.9 A straight line passes through the point (3, – 2).
(– 4, 2), (2, 6), (8, 5) and (9, – 7). Show that the mid- Find the locus of the middle point of the portion of the
points of the sides of this quadrilateral are vertices of a line intercepted between the axes.
parallelogram.
Q.10 Find the equation of the straight line which passes
Q.4 Find the values of x and y for which A(2, 0), through the point (3, 2) and whose gradient is 3/4. Find
B(0, 2), C(0, 7) and D(x, y) are the vertices of an isosceles the coordinates of the point on the line that are 5 units
trapezium in which AB ||DC. away from the point (3, 2).
Q.5 Find the equations of the diagonals of the rectangle, Q.11 Find the distance of the point (2, 5) the lines 3x
whose sides are x = a, x = a’, y = b and y= b’. +y + 4 = 0 measured parallel to line having slope 3/4.
Q.6 In what ratio is the line joining the points (2, 3) and Q.12 The extremities of a diagonal of a square are
(4, –5) divided by the line joining the points (6, 8) and (1, 1), (– 2, – 1). Obtain the other two vertices and the
(– 3, – 2) ? equation of the other diagonal.
M a them a ti cs | 8.39
Q.13 In the given figure, PQR is an equilateral Q.23 A ray of light is sent along the line x – 2y – 3 = 0.
triangle and OSPT is a square. If OT =2 2 units, find Upon reaching the line 3x – 2y – 5 = 0, the ray is reflected
the equation of the lines OT, OS, SP, OR, PR and PQ. from it. Find the equation of the line containing the
y reflected ray.
Q.4 The line x + 3y – 2 = 0 bisects the angle between a Q.11 The straight line, ax + by = 1 , makes with the
pair of straight lines of which one has equation x – 7y + curve px2 + 2axy + qy2 = r a chord which subtends a
5 = 0. The equation of the other line is right angle at the origin. Then:
(A) 3x + 3y – 1 = 0 (B) x – 3y + 2 = 0 (A) r(a2 + b2) = p + q (B) r(a2 + p2) = q + b
(C) 5x + 5y – 3 = 0 (D) None (C) r(b2 + q2) = p + a (D) None
Q.9 Let PS be the median of the triangle with vertices Q.16 Let PS be the median of the triangle with vertices
P (2, 2), Q(6, – 1) and R(7, 3). The equation of the line P(2, 2), Q(6, -1) and R(7, 3). The equation of the line
passing through (1, – 1) and parallel to PS is (2000) passing through (1, -1) and parallel to PS is (2014)
(A) 2x – 9y – 7 =0 (B) 2x – 9y – 11 = 0 (A) 4x - 7y - 11 = 0 (B) 2x+ 9y+ 7 = 0
(C) 2x + 9y – 11 = 0 (D) 2x + 9y + 7 = 0 (C) 4x+ 7y+ 3 = 0 (D) 2x - 9y - 11 = 0
Q.10 The incentre of the triangle with vertices Q.17 Let a, b, c and d be non-zero numbers. If the point
(1, 3),(0, 0) and (2, 0) is (2000) of intersection of the lines 4ax + 2ay + c = 0 and 5bx +
2by + d= 0 lies in the fourth quadrant and is equidistant
3 2 1 from the two axes then (2014)
(A) 1, (B) ,
2 3 3 (A) 2bc - 3ad= 0 (B) 2bc+ 3ad= 0
29 11
(A) (B) 5 (C) 6 (D)
5 5
(A) 2 − 2 (B) 1 + 2
(C) 1 − 2 (D) 2 + 2
M a them a ti cs | 8.43
JEE Advanced/Boards
Q.13 The sides of a triangle have the combined equation Q.20 The triangle ABC, right angled at C, has median
x2 – 3y2 – 2xy + 8y – 4 = 0. The third side, which is variable AD, BE and CF. AD lies along the line y = x + 3, BE lies
always passes through the point (– 5, – 1). If the range of along the line y = 2x + 4. If the length of the hypotenuse
values of the slope of the third line so that the origin is is 60, find the area of the triangle ABC.
an interior point of the triangle, lies in the interval (a, b),
1 Q.21 A triangle has side lengths 18, 24 and 30. Find
then find a + .
b2 the area of the triangle whose vertices are the incentre,
circumcentre and centroid of the triangle.
Q.14 Consider a line pair 2x2 + 3xy – 2y2 – 10x + 15y
– 28 = 0 and another line L passing through origin
with gradient 3. The line pair and line L form a triangle Q.22 The points (1, 3) & (5, 1) are two opposite vertices
whose vertices are A, B and C. of a rectangle. The other two vertices lie on the lines
y = 2x + c. Find c & the remaining vertices.
(i) Find the sum of the cotangents of the inter ior angles
of the triangle ABC.
Q.23 A straight line L is perpendicular to the line
(ii) Find the area of triangle ABC. 5x – y = 1. The area of the triangle formed by the line L
(iii) Find the radius of the circle touching all the 3 sides & the coordinate axes is 5. Find the equation of the line.
of the triangle.
Q.24 Two equal sides of an isosceles triangle are given
Q.15 Show that all the chords of the curve by the equations 7x – y + 3 = 0 and x + y –3 = 0 & its
3x2 – y2 – 2x + 4y = 0 which subtend a right angle at the third side passes through the point (1, – 10). Determine
origin are concurrent. Does this result also hold for the the equation of the third side.
curve, 3x2 + 3y2 + 2x + 4y = 0 ? If yes, what is the point
of concurrency & if not, give reasons. Q.25 The equations of the perpendicular bisectors of
the sides AB & AC of a triangle ABC are x – y +5 = 0 &
Q.16 A straight line is drawn from the point (1, 0) to the x + 2y = 0, respectively. If the point A is (1, –2). Find the
curve x2 + y2 + 6x – 10y + 1 = 0, such that the intercept equation of the line BC.
made on it by the curve subtends a right angle at the
origin. Find the equations of the line. Q.26 Let P be the point (3, 2). Let Q be the reflection
of P about the x-axis. Let R be the reflection of Q about
Q.17 The two line pairs y2 – 4y + 3 = 0 andx2 + 4xy + the lines y = – x and Let S be the reflection of R through
4y2 – 5x –10y + 4 = 0 enclose a 4 sided convex polygon the origin. PQRS in a convex quadrilateral. Find the area
find. of PQRS.
(i) Area of the polygon.
Q.27 Two parallel lines 1 and 2 having non-zero
(ii) Length of the diagonals.
slope, are passing through the points (0, 1) and (– 1,0)
respectively. Two other lines 1 and 2 are drawn
Q.18 Find the equations of the two straight lines which through (0, 0) and (1, 0) which are perpendicular to 1
together with those given by the equation 6x2 – xy – y2 and 2 respectively. The two sets of lines intersect in
+ x + 12y – 35 = 0 will make a parallelogram whose four points which are vertices of a square. If the area of
diagonals intersect in the origin. p
this square can be expressed is the form q where p ∈
N, then the least value of (p + q)?
Q.19 A straight line passing through O(0, 0) cuts the
lines x = α, y = β and x + y = 8 at A, B and C respectively
such that OA . OB . OC = 482 and f(α, β) = 0 where Q.28 In an acute triangle ABC, the base BC has the
y 3 equation 4x – 3y + 3 = 0. If the coordinates of the
f(x,=
y) – + (3π – 2y)6 + ex + 2y – 2e – 6 orthocentre (H) and circumcentre (P) of the triangle are
x 2
(1, 2) and (2, 3) respectively, then the radius of the circle
(i) Find the point of intersection of lines x = α and y = β. m
circumscribing the triangle is , where m and n are
(ii) Find the value of (OA + OB + OC) a
relatively prime. Find the value of (m+ n).
(iii) Find the equation of line OA.
M a them a ti cs | 8.45
(You may use the fact that the distance between Q.4 The lines 3x + 4y = 9 & 4x – 3y + 12 = 0 intersect
orthocentre and circumcentre of the triangle is given at P. The first line intersects x-axis at A and the second
R 1 – 8 cos A cosB cosC ) line intersects y-axis at B. Then the circum radius of the
triangle PAB is
Q.29 The points (– 6, 1), (6, 10), (9, 6) and (– 3, – 3) are (A) 3/2 (B) 5/2 (C) 10 (D) None
the vertices of a rectangle. If the area of the portion of
this rectangle that lies above the x axis is a/b, find the Q.5 If the lines ax + y + 1 = 0, x + by + 1 = 0 & x + y +
value of (a +b), given a and b are coprime. c = 0, where a, b & c are distinct real numbers different
from 1 are concurrent, then the value of
Q.30 Consider the triangle ABC with sides AB and AC 1 1 1
+ + =
having the equation L1 = 0 and L2 = 0. Let the centroid. 1–a 1–b 1–c
Orthocentre and circumcentre of the ∆ABC and G, H and
S respectively. L = 0 denotes the equation of sides BC. (A) 4 (B) 3 (C) 2 (D) 1
(i) If L1: 2x – y = 0 and L2: x + y = 3 and G(2, 3) then find Q.6 The points A(a, 0), B(0, b), C(c, 0) & D(0, d) are such
the slope of the line L= 0. that ac = bd & a, b, c, d are all non zero. The points
thus:
(ii) If L1: 2x + y = 0 and L2: x – y + 2 = 0 and H(2, 3) then
find the y-intercept of L = 0. (A) Form a parallelogram (B) Do not lie on a circle
(iii) If L1: x + y – 1 = 0 and L2: 2x – y + 4 = 0 and S(2, 1) (C) Form a trapezium (D) Are concyclic
then find the x-intercept of the line L= 0.
Q.7 The angles between the straight lines joining the
origin to the points common to 7x2 + 8y2 – 4xy + 2x –
Exercise 2 4y – 8 = 0 and 3x – y = 2 is
π π π
(A) tan–1 2 (B) (C) (D)
Single Correct Choice Type 3 4 2
Q.1 Given the family of lines, a (3x + 4y + 6) + b (x + y Q.8 Distance between two lines represented by the line
+ 2) = 0. The line of the family situated at the greatest pair, x2 – 4xy + 4y2 + x – 2y – 6 = 0 is
distance from the point P(2, 3) has equation: 1
(A) (B) 5 (C) 2 5 (D) None
(A) 4x + 3y + 8 = 0 (B) 5x + 3y + 10 = 0 5
(C) 15x + 8y + 30 = 0 (D) None
Q.9 If the straight lines joining the origin and the points
of intersection of the curve
Q.2 On the portion of the straight line, x + 2y = 4 5x2 + 12xy – 6y2 + 4x – 2y + 3 = 0.
intercepted between the axes, a square is constructed
on the side of the line away from the origin. Then the And x + ky – 1= 0 are equally inclined to the co-ordinate
point of intersection of its diagonals has co-ordinates: axes then the value of k:
(A) (2, 3) (B) (3, 2) (C) (3, 3) (D) None (A) Is equal to 1
(B) Is equal to – 1
Q.3 The base BC of a triangle ABC is bisected at the (C) Is equal to 2
point (p, q) and the equation to the side AB & AC are
(D) Does not exist in the set of real numbers
px + qy = 1 and qx + py = 1. The equation of the
median through A is:
Q.10 If the vertices P and Q of a triangle PQR are given
(A) (p – 2q) x + (q – 2p)y + 1 = 0 by (2,5) and (4,–11) respectively, and the point R moves
(B) (p + q) (x + y) – 2 = 0 along the line N: 9x + 7y + 4 = 0, then the locus of the
centroid of the triangle PQR is a straight line parallel to
(C) (2pq – 1)(px+qy –1) =(p2 + q2 – 1)(qx+py – 1)
(A) PQ (B) QR (C) RP (D) N
(D) None
8 . 4 6 | Straight Line
(D) (a – b)x2 – hx – (a – b) = 0 x y
Q.19 Line + = 1 cuts the co-ordinate axes at
a b
x y
Q.13 A Triangle is formed by the lines 2x – 3y – 6 = 0; A(a, 0) & B(0, b) & the line + = –1 at A’ (–a’, 0) &
3x – y + 3 = 0 and 3x + 4y – 12 = 0. If the points P(α, 0) a' b'
and Q(0, β) always lie on or inside the ∆ABC, then B’(0, – b’ ). If the points A, B, A’, B’ are concyclic then the
orthocentre of the triangle ABA’ is
(A) α ∈ [– 1, 2] & β ∈ [– 2, 3]
(B) α ∈ [– 1, 3] & β ∈ [– 2, 4] aa' bb'
(A) (0, 0) (B) (0, b’) (C) 0, (D) 0,
(C) α ∈ [– 2, 4] & β ∈ [– 3, 4] b a
(D) α ∈ [– 1, 3] & β ∈ [– 2, 3]
Q.20 If one vertex of an equilateral triangle of side
‘a’ lies at the origin and the other lies on the line
Q.14 In a triangle ABC, side AB has the equation 2x + 3y
= 29 and the side AC has the equation, x + 2y = 16. If the x – 3y = 0 then the co-ordinates of the third vertex
mid-point of BC is (5, 6), then the equation of BC is are:
3a a 3a a
(A) x – y = – 1 (B) 5x – 2y = 13 (A) (0,a) (B) , – (C) (0, – a) (D) – ,
2 2 2 2
(C) x + y = 11 (D) 3x – 4y = – 9
Q.21 Three vertices of a triangle are A(4, 3); B(1, –1) and
Q.15 The vertex of a right angle o triangle lies on the C(7, k). Value(s) of k for which centroid, orthocentre,
straight line 2x + y – 10 = 0 and the two other vertices, incentre and circumcentre of the ABC lie on the same
at point (2, – 3) and (4, 1) then the area of triangle in straight line is/are:
sq. units is - 19
33 (A) 7 (B) – 1 (C) – (D) None
(A) 10 (B) 3 (C) (D) 11 8
5
Q.22 Equation of a line through (7, 4) and touching the
Multiple Correct Choice Type circle x2 + y2 – 6x + 4y – 3 = 0 is
(A) 5x – 12y + 13 = 0 (B) 12x – 5y – 64 = 0
Q.16 The area of triangle ABC is 20 cm2. The co-
ordinates of vertex A are (– 5, 0) and B are (3, 0). The (C) x – 7 = 0 (D) y = 4
vertex C lies upon the line, x – y = 2. The co-ordinates
of C are Q.23 The circumcentre of the triangle formed by the
lines, xy + 2x + 2y + 4 = 0 and x + y +2 = 0 is
(A) (5, 3) (B) (– 3, – 5) (C) (– 5, – 7) (D) (7, 5)
(A) (– 2, – 2) (B) (–1 , – 1)
(C) (0, 0) (D) (– 1, – 2)
M a them a ti cs | 8.47
Q.24 The sides of a triangle are x + y = 1, 7y = x and Match the conditions expressions in column I with
– 3y + x = 0. Then the following is an interior point of statement in column II
the triangle.
(A) Circumcentre (B) Centroid Q.2 Consider the lines given by (2008)
Q.1 Lines L1: y – x = 0 and L2: 2x + y =0 intersect the line L3: Q.7 The straight line 2x – 3y = 1 divides the circular
y + 2 = 0 at P and Q, respectively. The bisector of the acute region x2 + y2 ≤ 6 into two parts. If
angle between L1 and L2 intersects L3 at R. (2007)
3 5 3 1 1 1 1
Statement-I: The ratio PR: RQ equals 2 2 : 5 . S = 2, , , , , – , , , (2011)
4 2 4 4 4 8 4
Because
Then the number of point(s) lying inside the smaller
Statement-II: In any triangle, bisector of an angle
part is……………
divides the triangle into two similar triangles.
8 . 4 8 | Straight Line
Q.8 For points P = (x1, y1) and Q = (x2, y2) of the Q.11 Using coordinate geometry, prove that the three
coordinate plane, a new distance d(P,Q) is defined by altitudes of any triangle are concurrent. (1998)
d(P, Q) = | x1 – x2 | + | y1 – y2 | . Let Q = (0, 0) and A =
(3, 2). Prove that the set of points in the first quadrant Q.12 A Straight line L through the point (3, -2) is
which are equidistant (with respect to the new distance) inclined at an angle 60° to the line 3x + y =. 1 If L
from O and A consists of the union of a line segment also intersects the x-axis, then the equation of L is
of finite length and an infinite ray. Sketch this set in a (2011)
labelled diagram. (2000)
Q.13 For a > b > c > 0, the distance between (1, 1) and
Q.9 A straight line L through the origin meets the line x the point of intersection of the lines ax + by + c = 0 and
+ y = 1 and x + y = 3 at P and Q respectively. Through bx + ay + c = 0 is less than 2 2 . Then (2013)
P and Q two straight lines L1 and L2 are drawn, parallel
to 2x – y = 5 and 3x + y = 5 respectively. Lines L1 and L2 (A) a + b – c > 0 (B) a – b + c < 0
intersect at R, show that the locus of R as L varies, is a (C) a – b + c > 0 (D) a + b – c < 0
straight line. (2002)
PlancEssential Questions
JEE Main/Boards JEE Advanced/Boards
Exercise 1 Exercise 1
Q.10 Q.13 Q.23 Q.2 Q.6 Q.10
Q.24 Q.26 Q.27 Q.15 Q.19 Q.24
Q.27
Exercise 2
Q.11 Q.12 Q.16 Exercise 2
Q.18 Q.2 Q.6 Q.8
Q.12 Q.18 Q.21
Previous Years’ Questions Q.23 Q.26
Q.2 Q.7 Q.10
Q.11 Q.13 Previous Years’ Questions
Q.2 Q.5 Q.7
Q.9 Q.10
M a them a ti cs | 8.49
Answer Key
JEE Main/Boards
Q.14 x = 2, x + 9y – 14 = 0, 7x – 9y – 2 = 0
Exercise 1
Q.15 (– 4, – 3)
1
Q.1 – 36 45
6 Q.16 – ,–
7 7
Q.4 (7, 0) or (2, 5) Q.17 5
Exercise 2
Exercise 2
Single Correct Choice Type
Solutions
JEE Main/Boards 5
−1 +
2 3 1
Slope of GH = = =
Exercise 1 17 5
− 12 4
2 2
Sol 1: Slope of line joining [4, –6] & [–2, –5] 5
4+
−5 + 6 1 Slope of EH = 2 = −13
= = 5
−2 − 4 −6 7
−1 −
2
Sol 2: Line 1 joining (2, –3) and (–5, 1) EF || GH and FG || EH, hence midpoints E, F, G, H form a
parallelogram.
4
⇒ Slope = −
7
Sol 4: AB || DC
2 → line joining (7, –1) and (0, 3)
2−0
−4 Slope of AB = = –1
⇒ Slope = 0−2
7
7−y y −7
3 → line joining (4, 5) and (0, –2) Slope of DC = =
0−x x
7 y – 7 = –x ⇒ x + y = 7 – x …(i)
⇒ Slope =
4
1 || 2 and m2 ⋅ m3 =
−1 A B
9 − 4 −7 + 2 5 −5 ⇒ x(9 – x) = 14 ⇒ x2 – 9x + 14 = 0
AD = H , = ,
2 2 2 2 ⇒x = 7, 2
11 If x = 7, y = 0 and if x = 2, y = 5
2 − 4 3 1
Slope of EF = = = Ans is (7, 0) and (2, 5)
5 + 1 12 4
Sol 5:
11 D C y = b’
+1 −13
13
Slope of FG = 2 = = y=b
17 −7 7 A B
5−
2
x=a x = a’
8 . 5 2 | Straight Line
Coordinates are A(a, b), B(a′, b), C(a′, b′) and D(a, b′) It is a square hence b – a = c
b′ − b H lies of x = 2y ⇒ a = 2c ⇒ b = 3c
Slope of line AC =
a′ − a G lies on x + y = 3 ⇒ b + c = 3
b′ − b 4c = 3
Slope of line BD =
a − a′
3 9 6
Equation of line AC ⇒c= ; b = ; a=
4 4 4
b′ − b Coordinates of square are
⇒y–b= (x – a)
a′ − a
3 9 9 3 3 3
⇒ (b′ – b)x + (a – a′)y – ab′ + a⇒b = 0 E ,0 , F ,0 , G , , H ,
2 4 4 4 2 4
Equation of line BD
b′ − b
⇒y–b= (x – a′) Sol 8:
a − a′
⇒ (b′ – b)x – (a – a′)y – a'b' + ab = 0 (0,3) A
B
2
3 + m −2 − 3m
E = (a, 0), F = (b, 0), G = (b, c) Mid point of AB ,
2 2
H = (a, c)
M a them a ti cs | 8.53
−3m − 2 3m + 2 −3
y= ,x= Slope of AC =
2 2m 2
2y + 2 BD = 13
m = –
3 Length of side =
13
−2y − 2 −2y − 2 2
⇒ 2x = 3 +2
3 3 Let slope of AB be m
⇒ –4xy – 4x = –6y – 6 + 6 2
m−
tan 45º = 3 =1
⇒ 2xy + 2x – 3y = 0 2m
1+
3
y −2 3
Sol 10: = and y = 3x – 1 m 5 5m −1 −1
x −3 4 = ⇒ m = 5 or = ⇒m=
3 3 3 3 5
Points which are 5 units away from (3, 2) are
13 13
A = 1 + cos θ, 1 + sin θ
(3 ± 5 cos θ, 2 ± 5 sin θ )
2 2
3 −1 −5
tan θ = ⇒(3 ± 4, 2 ± 3)⇒ [7, 5] [–1, –1] Now cos θ = , sin θ =
4
26 26
x-3y=2 x+y-6=0 O
G E
F (0, 0)
C B B C
5x-3y+2=0
(-2, 3) (a, b)
Solving
Orthocentre is (0, 0)
x + y – 6 = 0, x – 3y – 2 = 0 gives A[5,1]
1
x + y – 6 = 0, 5x – 3y + 2 = 0 gives B[2,4] (slope)OA = −
3
x – 3y – 2 = 0, 5x – 3y + 2 = 0 gives C[–1,–1] b−3
(slope)BC = 3 = …. (i)
7 5 1 3 a+2
Midpoints are E , , F , , G[2, 0]
2 2 2 2 −3
(slope)OB =
2
Equation of median
2 b +1
1 (slope)AC = = …. (ii)
y −1 3 a−3
AF ⇒ = 2 ⇒ x + 9y = 14
x −5 9 Solving (i) and (ii), we get
−
2 b = 3a + 9 and 3b = 2a – 9
y−4 1
BG ⇒ = ⇒x=2 9a + 27 = 2a – 9
x−2 0
36 −108 + 63 45
y +1 7 a= − ; b= = −
CE ⇒ = ⇒ 7x – 9y = 2 7 7 7
x +1 9
36 45
⇒ C − , −
Sol 15: 7 7
C(-1,3)
Sol 17: Lines are concurrent intersection of 1 and M 3
gives x = 1 y = –1
It lies on 2 ⇒ p – 2 – 3 = 0 ⇒ p = 5
A(0,0)
B(2,-1)
x−6
−4 Sol 18: 1 ⇒ y = 3x + 5 2 ⇒ y =
Slope of line BC = 3
3 1
m1 = 3 m2 =
3 3
Slope of line perpendicular to BC =
4 1
3−
⊥ bisector through A 3 1
tan θ = =
3x 3 3
y= …. (i) 1+
4 3
Slope of line AC = –3
⇒ θ = 30º, 150º
1
Slope of line perpendicular to AC =
3 Sol 19:
⊥ bisector through B
3
A(2, –1) mAB = −
3y = x – 5 …. (ii) 2
Solving (i) & (ii), we get 1
B(0, 2) mBC =
x = –4; y = –3 3
−3
C(3, 3) mCD =
Ans is (–4, –3) 2
M a them a ti cs | 8.55
1
D(5, 0) mAD = A (2, -7)
3
BC || AD and AB || CD B
4x+y=1
⇒ ABCD is a parallelogram
C
⇒ Diagonals are AC & BD 3x-4y+1=0
−1 − 3
Slope of AC → =4 Solving equations we get B
2−3
3x + 1
2 − 0 −2 = 1 – 4x ⇒ 19x = 3
Slope of BD → = 4
0 −5 5 3 7
−2 ⇒x= ,y=
m1 = 4; m2 = 19 19
5
2 3 7
4+ ∴ B ,
5 22 −22 19 19
tan θ = = =
8 −3 3
1− 2 2
5 3 7
AB = 2 − + + 7
19 19
−22
φ = tan–1
3 (35)2 + (140)2 35
= = 17
19 19
Sol 20: A(–2, 0) ; B(0, 5)
C = (2 + r cosθ, –7 + r sinθ)
5
mAB = rAB = 29 3(2 + r cosθ) – 4(–7 + r sinθ) + 1 = 0
2
35 + 3r cosθ – 4r sinθ = 0
C = 0 + 29 cos θ,5 + 29 sin θ
3 17 4 17
1+ cosθ – sinθ = 0
D = −2 + 29 cos θ,0 + 29 sin θ 19 19
19
–3 cosθ + 4 sinθ =
−2 2 −5 17
tan θ = ; sin θ = ; cos θ =
5 29 29 −52
tan θ = –4 or
C = [–5, 7] D = [–7, 2] 89
y+7 −52
Equation of AC ⇒ = –4 or =
x−2 89
y −2
Sol 21: =m 52x + 89y + 519 = 0 or 4x + y = 1
x −3
1 Sol 23:
m−
2 m=m
tan 45 = m=1/2 m=–2/3
m
1+
2 x–2y–3=0
2m − 1
= ±1
m+2
1 m=3/2 3x–2y–5=0
m = +3, m = −
3
Points of intersection is (1, –1) and both the lines
⇒y – 2 = 3x – 9 ⇒ y = 3x – 7 x – 2y – 3 = 0 and reflected Line are equally inclined to
normal on 3x – 2y 5 = 0
1
& y – 2 = − (x – 3) ⇒ x + 3y = 9 2 −2 1
3 m+ −
3 = 3 2
2m 1
Sol 22: 1− 1−
3 3
8 . 5 6 | Straight Line
3m + 2 −7 13
⇒ = ⇒ Slope of BC =
3 − 2m 4 61
33
⇒ 2(6m + 4) = –21 + 14m y −b 13 y+
⇒ = ⇒ 2 = 13
⇒ 2m = +29 x−a 61 x + 13 61
29 61 × 33
⇒ m=+ ⇒ 61y = 13x + 13×13 –
2 2
29 ⇒ 26x – 122y – 1675 = 0
⇒ (y + 1) = (x – 1)
2
⇒ 2y + 31 = 29x Sol 26: The equation of line passes through point of
intersection of x+3y+y=0 and 3x+y+4=0 is
Sol 24:
3x+y+4 + λ (x+3y+4) = 0
(-8, 12)
(λ+3)x + (1+3 λ)y + 4 + 4 λ =0
The obtained line is equally inclined to axes, then
4x+7y+13
Slope of line = ± 1
λ+3 λ+3
(a, b) − 0
= and − −1
=
1 + 3λ 1 + 3λ
a+8 b − 12 65 ⇒ λ + 3 = - 1 -3 λ or λ + 3 = 1 +3 λ
= = –2 .
4 7 65 ⇒ λ = -1 or ⇒ λ = 1
a+8 b − 12 Eqn of line is (-1 + 3)x + (1 - 3)y + 4 - 4=0
⇒ = = –2
4 7
⇒ 2x − 2y= 0 ⇒ x= y and
⇒ a = –16 & b = –2
(1+3)x + (1 + 3)y + 4 + 4=0
Image ≡ (–16, –2)
⇒ 4x + 4y + 8 =0
⇒ x+y+2 = 0
Sol 25: A
2 1 1 1
1 2 m+ − −
sin 60º = ⇒a= = 3
2a 3 3 = 3 7
m 1
1− 1−
3 21
Exercise 2
3m + 1 −10 3m + 1 −1
⇒ = ⇒ =
Single Correct Choice Type 3−m 20 3−m 2
⇒ 6m + 2 = m – 3 ⇒ m = –1
Sol 1: (C) 3x – 8y – 7 = 0
−1 7
3(0) − 8( −1) − 7 =
1 It passes through ,
(A) different sides 10 10
3(0) − 8(0) − 7 =−7 7
y−
10 = –1⇒ x + y = 7 – 1 ⇒ 5x + 5y = 3
3(0) − 8(1) − 7 =−15
(B) different sides 1 10 10
3(3) − 8(0) − 7 =2 x+
10
3( −1) − 8(1) − 7 =−2
(C) same sides Sol 5:(C) X = x – h = 3 – 4 = –1
3(3) − 8(7) − 7 =−54
Y = y – k = –5 + 3 = –2
Sol 2: (A) 3x – 4y + 7 = 0; 12x + 5y – 2 = 0
x y
p1p2 + q1q2 = 36 – 20 = 16 > 0 Sol 6:(A) + =1
a b
3x − 4y + 7 −(12x + 5y − 2) 3 4
= + = 1 are concurrent at a fixed point.
5 13 5a 5b
39x – 52y + 91 = –60x – 25y + 10 3 4
Point is x = and y =
5 5
99x – 27y + 81 = 0 ⇒ 11x – 3y + 9 = 0
3 4
⇒ ,
Sol 3: (A) 5 5
(5,3)
(1,2) C
A Sol 7: (D) P (1, 0) ; Q (–1, 0) ; R (2, 0)
2SP2 = SR2 + SQ2
B
(x+1)2+y2+(x–2)2+y2 = 2(x–1)2+2y2
⇒ x2+1+2x+x2+4–4x=2(x2+1–2x)
C’
(5,-3)
⇒ 5 – 2x = 2 – 4x
C’ is the reflection of C w.r.t. x-axis. 3
2x = –3 ⇒ x = −
∴ Eq. of AB = Eq. of AC’ 2
y
2a 1 + m2
2r = =2a ⇒r=a
1 + m2
x
Sol 13: (A) 12x2 – 16xy + 9xy – 12y2 = 0
4x(3x – 4y) + 3y(3x – 4y)
(4x + 3y)(3x – 4y) = 0
|x|+|y|=1
2y=x
It forms a square.
3x-4y=0
1 2 9 3 3 1
3 −5 5 = 0 m− −
4
= 4 2
a b 1 3m 3 1
1+ 1+ ×
4 4 2
–5 – 5b – 2(3 – 5a) + 9(3b + 5a) = 0
–5 – 5b – 6 + 10a + 27b + 45a = 0 4m − 3 2
= ⇒ 44m – 33 = 8 + 6m
55a + 22b – 11 = 0 4 + 3m 11
41x
5a + 2b = 1 38m = 41 ⇒ y =
38
The straight line 5x + 2y = 1 passes through (a, b)
Sol 14: (C) A = (4, –1)
Sol 11: (A)
B is symmetric to A w. r. t. y = x
B is [–1, 4]
AB = 25 + 25 = 5 2
49 + 4 + 70 – 28 – 151 < 0 x = 5y
Point is inside 7y − x 2y −1
Slope of line AB = = =
y−x −4y 2
A(–7, +2) and Centre (5, 7)
r= 25 + 49 + 151 = 15
Sol 3: (C) Let B, C, D be the position of the point A(4, 1) Sol 9: (D) Since, S is the mid point of Q and R.
after the three operations I, II and III respectively. Then,
7 + 6 3 − 1 13
B(1, 4), C(1+2, 4) ie, (3, 4). The point D is obtained from ∴ S≡ , = ,1
2 2 2
C by rotating the coordinate axes through an angle π/4
in anticlockwise direction. 2 −1 2
Now, slope of PS = m = = −
Therefore, the coordinates of D are given by 2 − 13 / 2 9
1 7
∴ Coordinates of D are − , Sol 10: (D) Let the vertices of triangle be A (1, 3) , B(0,
2 2 0 ) and C (2, 0).
Here AB = BC = CA = 2.
Sol 4: (A) The point of intersection of three lines are
A(1, 1), B(2, -2), C(-2, 2). Therefore, it is an equilateral triangle. So the incentre
coincides with centroid.
Now, | AB |= 1+9 = 10 ,| BC |= 16 + 16 = 4 2,
and | CA |= 9 +1 = 10 0 +1+ 2 0 + 0 + 3 1
∴ I= , = 1,
3 3 3
∴ Triangle is an isosceles
Sol 11: (A) On solving equations 3x + 4y = 9 and y =
Sol 5: (A) By the given condition, we can take two mx + 1, we get
perpendicular lines as x and y axes. If (h, k) is any point 5
on the locus, then |h|+|k|=Sol 1 Therefore, the locus is x=
3 + 4m
|x|+|y|=1 This consist of a square of side 1.
Hence, the required locus is a square. Now, for x to be an integer
3 + 4m = ± 5 or ± 1
Sol 6: (C) Orthocentre of right angled triangle is at
the vertex of right angle. Therefore, orthocentre of the The integral values of m satisfying these conditions are – 2
triangle is at (0, 0). and – 1
1 1 1 ⇒k=
6
= 2 × × OA × DB = 2 × ×
2 2 m–n
1 1 Sol 15: (A) Sides OA = OB = 2 and AB = 2 2
= =
m–n |m–n| X-coordinates of incentre of ∆ OAB
R(3, 3 3)
Q(0, 0)
P(-1, 0) 2 2 ×0 + 2 × 0 + 2 × 2
≡
2 2 +2+2
Equation of the line passing through points Q(0, 0) and
R(3, 3 ) is; 4 2
= = = 2− 2
4 + 2 2 2+ 2 2
y −0 3 3 y 3 3
= ⇒=
x−0 3−0 x 3
6 + 7 −1 + 3 13
⇒ y =3x … (ii) Sol 16: (B) S ≡ , ; ≡ , 1
2 2 2
3x − y 3x + y
⇒ y = or y=
−
2 2
⇒ 3x
= 3x + y 0 .
− 3y 0 or =
2
Sol 14: (C) Point T is given by Equation of line passes through having slope −
9
3 × 2 + 2 × 1 3 × 4 + 2 × 1 8 14 2
T ≡
3+2
,
3+2
≡ ,
5 5
y + 1 =−
9
( x − 1) ⇒ 9y + 9 =−2x + 2
⇒ 2x + 9y − 7 =0
From figure
Similarly, AB || DC
PT = P’T
⇒ Equation of DC x – y + λ2 = 0, passes through
(P' T )
2
⇒ PT2 = (-3, -6)
⇒ (h − 1 ) + (k − 2 ) =( 2 − 1 ) + ( 3 − 1 )
2 2 2 2
⇒ x − y −3 =0
⇒ (h − 1 ) + (k − 2 ) = 1 + 1 = 2
2 2
1 8 7 4
⇒ ( x − 1) + ( y − 2) =
2
2
2 ⇒ C , − and A − , −
3 3 3 3
Locus is a circle.
A 1 2
S G H
(7/2, 5/2) (2, 4)
2+7 4 +5
G= , = (3, 3)
3 3
A = (3, 3) = (a, a)
On line x = 2, there are 38 points inside ∆ DAB 3a a
B = ,a −
Total points 2 2
= 39 + 38 + ….. + 2 + 1
a a a a
39 ( 40 ) C = a + + ,a − +
= = 780 2 4 2 4
2 a a a a a a
⇒ a + + + + ...,a − + − ....x
2 4 8 2 4 8
Sol 20: (B) Let two sides AB and BC be x – y + = 0 and
7x – y – 5 = 0 respectively.
M a them a ti cs | 8.63
A = (–2, 3)
a a 2a
N= , = 2a, ⇒ Eq. of AB is x + y = 1 … (i)
1 − 1 1 + 1 3
Eq. of AC is x + 2y = 4 … (ii)
2 2
8a 3 5
α+β= =8 S= ,
3 2 2
⇒ Eq. of perpendicular bisector of AB and AC are
Sol 2: y = x + 1 … (iii)
A(-3,1)
1
y = 2x – … (iv) respectively.
2
(,) Eq. (i) and (iii) ⇒ E = (0, 1) [Midpoint of A, B]
9/5 3 C1 - C2
=
9 –5C1 = 5 ⇒ C1 = 1
2
a +b
2
5
m ⇒ Diagonals are
4x+3y=3
4y + 2x – 10 = 0 ⇒ 2y + x – 5 = 0
(-2, -7)
6y – 12x + 0 = 0 ⇒ y = 2x
(–2+r cosθ, –7+r sinθ) lies on 4x + 3y = 3 If A vertex is at y-axis ⇒ x = 0
–2+(r+3)cosθ, –7+(r+3)sinθ lies on 4x+3y = 12 y = 2(0) = 0 ⇒ (0, 0)
4(rcosθ – 2) + 3(rsinθ – 7) = 3 …. (i) 2y + 0 – 5 = 0 ⇒y = 5/2 ⇒(0, 5/2)
4((r+3)cosθ – 2) + 3((r+3)sinθ–7)=12 ….(ii)
On solving (i) and (ii), we get Sol 7: O(0, 0) ; A(6, 0) ; B(3, 3)
If m > 0 then 2x + y = 4 (If x > 1 and y > 2) (x12 – d2)y2 + (y12 – d2)x2 – 2x1y1xy = 0
1 + β β −5 1
Mid point of AC = , Slope of L1 [(–5, –1) to (0, 0)] =
5
2 2
M a them a ti cs | 8.67
x y −2
Slope of + =1⇒ = –1
2 2 2 1 3 1
1
Range (–1, 1/5) Area = 4 12 1
2
1 18
(∴ Third line is go through (0, 0) and for triangle parallel − 1
5 5
x y
to + = 1 meet at infinity)
2 2
1 = 1 1 12 − 18 − 3 4 + 1 + 1 18.4 + 1.12
−1, = (a, b) 2 5 5 5 5
5
1 42 3.21 72 + 12
1 = − +
⇒ a + 2 = –1 + 52 = –1 + 15 = 24 2 5 5 5
b
1 84 + 42 − 63 63
= = 10
Sol 14: 2x + 3xy – 2y – 10x + 15y – 28 = 0
2 2 2 5
y = 3x ⇒ a1 = 9 + 81 = 3 10
(-1/5,18/5)
36 9 3
C B ⇒ a2 = + = 45 = 45
(4,12) 25 25 5
A
(1,3) 2 2
21 18 441 + (42)2 21
⇒ a3 = + 12 − = = 5
5 5 25 5
2x2+9x2–2(9x2)–10x+15(3x)–28 = 0
Incentre will be
–7x2 + 35x – 28 = 0
x2 – 5x + 4 = 0 ⇒ x = 1, 4 1 × 21 5 4 × 3 5 1
+ − × 3 10
5 5 5 ,
∴ y = 3, 12
15 10 3 5 21 5
+ +
∂p 5 5 5
= 4x + 3y – 10 = 0
dx
∂p 3 × 21 5 12 × 3 5 18
= –4y + 3x + 15 = 0 + + 10
dy 5 5 8
18 24 5 + 15 10
25y = 90 ⇒ y = 5
5
54 11 5 − 10 33 5 + 18 10
10 − = ,
x= 5 = −1
8 5 + 5 10 8 5 + 5 10
4 5
3 Radius = distance of incentre from any of the sides.
−1
⇒ m1 = 3, m2 = 5 = , 4 − 3x 18 10 + 3 10 21
−6 2 = =
5 10 10(8 5 + 5 10 ) 8 5 + 5 10
18
12 −
5 42 21(8 5 − 5 10 ) 3
⇒ m3 = = =2 = = (8 5 − 5 10 )
1 21 70 10
4+
5
3−2 1 Sol 15: y = mx + c
⇒ tanq1 = =
1+6 7 y − mx y − mx
3x2–y2– 2x +4y =0
1 c c
3+
⇒ tanq2 = 2 = –7
2xy 2mx2 4y 2 4mxy
3 3x2–y2 – + + − =0
1− c c c c
2
1 50
⇒ cotq1 + cotq2 + cotq3 = 17 + +0=
7 7
8 . 6 8 | Straight Line
3m 24 −4m 2 2
x2 3 − + y − 1 + xy + =0
-1
c c c c
6xy 10y 2 1
⇒ x2 + y2 + 6x2 – – 10xy + + x2 = [–6 – (–18)] = 6 units
m m 2
Diagonals are = 49 + 4 , −1 + 4 = 53 , 5
y2 2xy
+ – =0
m2 m
Sol 18: 6x2 – xy – y2 + x + 12y – 35 = 0
10 1 6 2
⇒ 8x2 + y2 1 + + + xy − − 10 − = 0 (y – m1x – c1)(y – m2x – c2) = 0
m 2 m m
m
y2 – m2xy – c2y – m1xy + m1m2x2
a 8
⇒ = = –1 + m1xc2 – c1y + c1m2x + c1c2
b 10 1
1+ +
m m2 m1m2x2 + y2 + xy(–m2 – m1) + x(m2c1 + m1c2)
β2 1600
y 2 + m2 y 2 β2 + α2 + α2m2 = 48 ⇒ AM.BM =
− 10
6
m2
1
8
αβ
(1 + m2 )3/2
= 48
=
Area of ∆AMB
2
( AM)(BM) Sin θ
(1 + m) (m)
1 1600 1 400
= × 10 × =
⇒ (1 + m2)3/2 = 2 m(1 + m) 2 6 10 3
⇒ (1 + m2)3 = 2m2(1+m2+2m) Area of ABC 3 × Area of ∆AMB
⇒ m + 1 + 3m + 3m = (m + m + 2m )
6 2 4 4 2 3 2
400
3×
= 400 Sq. units
=
m + m + m + 1 = 4m
6 4 2 3
3
⇒m=1
β 3
f(α, β) = − + (3x − 2y)2 + e(x − 2) + 2(y − 3) = 0
α 2
8 . 7 0 | Straight Line
Sol 26:
⇒ 6m2 + 16m − 6 =0
⇒ 3m2 + 8m − 3 =0
P
⇒ 3m2 + 8m − 3 =0
⇒ 3m2 + 9m − m − 3 =0
⇒ (3m− 1)(m+ 3) =
0 Q
⇒m= −3, 1/ 3 R
y=-x
Equations
P(3, 2), Q(3, –2), R(2, –3), S = (–2, 3)
3x + y=
+ 7 0 or x − 3y −
= 31 0
3 2
Sol 25:
3 −2
1
Area of PQRS will be, A = 2 −3
A(1,-2) 2
−2 3
3 2
m=-1 1 1
D E = [–6–9+6–4–(6–4+6+9)] = [–13 – 17] = 15 units
m=2 2 2
(,-1-)B C ( -10 5
,
3 3 ) Sol 27:
(,2-4)
x+2y=0 x-y+5=0
l3
l4
y+2 (0,1)
Eq. of AB = = –1
x −1 A
(1,0)
y+2=1–x (-1,0)
(0,0)
B
C
x+y+1=0 D l1
y+2 l2
Eq. of AC = =2
x −1
y – 1 = mx → 1
y + 2 = 2x – 2 ⇒ y + 4 = 2x
y = m(x + 1) → 2
α + 1 −α − 3 β + 1 2β − 6
D→ , ; E → , −1x
α α 2 2 y= → 3
m
α +1 α +3 −1(x − 1)
+ +5 = 0 y= → 4
2 2 m
−x
α + 7 = 0 ⇒ α = –7 1 intersection 3 → mx + 1=
m
−m 1
β +1 2β − 6 x= ,y=
= −2 1+m 2
1 + m2
2 2
11 −(x − 1)
β + 1 = –4β + 12; β= 1 intersection 4 → mx +1 =
5 m
11 2 −x 1
(–7, 6) , mx + 1 = +
m m
5 5
y −6 28 −14 m2 + 1 1−m
= = x =
x+7 −46 23 m m
23y + 14x = 138 – 98 1−m
x=
14x + 23y = 40 1 + m2
8 . 7 2 | Straight Line
m − m2 m+1 5
⇒ x1 + x2 + x3 = … (iii)
y= +1 =
1 + m2 m2 + 1 8
⇒ y1 + y 2 + y 3 = … (iv)
1−m m+1
B 2 , From (i), (ii), (iii) and (iv), we get
m + 1 m2 + 1
−x ⇒ 4 ( x 2 + x3 ) − 3 ( y 2 + y 3 ) + 6 =0
2 intersect 3 → m(x + 1) =
m ⇒ 4 (5 − x1 ) − 3 ( 8 − y1 ) + 6 =0
x
mx + = –m
m ⇒ 20 − 4x1 − 24 + 3y1 + 6 =0
2
−m
x= ⇒ −4x1 + 3y1 + 2 =0 … (v)
1 + m2
−m2 Now AH ⊥ BC
m m
y= C ,
1 + m2 2 2 y1 − 2 4
1 + m 1 + m × =−1
x1 − 1 3
−1
2 intercept 4 ⇒ m(x + 1) = (x − 1)
m ⇒ 4y1 − 8y =
−3x1 + 3
x 1
⇒ mx + = –m ⇒ 3x1 − 4y1 =
11
m m … (vi)
(1 − m2 ) From (v) and (vi), we get
x=
2
1+m 41 38
(x
( x1,1, yy11)) = ,
2 25 25
y = m
2
1 +m 2 2
41 38
(1 − m2 ) 2m Radius = AO= 2 − + 3 −
25 25
⇒ D ,
2 2
1 + m m + 1
2 58 m
m−1 = =
Area of square = =
(m − 1)2 ( 25) n
1 + m2 m2 + 1
AB = BD
⇒ m 58, =
= n 25 ⇒ m + n =83
m2 + m4 = 2m(1 + m2) Sol 29: (–6, 1) (6, 10), (9, 6), (–3, –3)
m2 = 2m
(6, 10)
m=2 B
1 p
Area = =
5 q A
C
(9, 6)
(-6, 1)
p+q=6 E
(1,0)
2 1
Sol 28: P(2,3)
D
H(1,2) C (-3,-3)
α – 2 = 2α + 3 ⇒ α = –5
1 1 25
[x1 − x2 ] + 3 = Area of ∆ = × 3 = 75
2 2 4 8 and –8β + 8α = 5β + 10α + 10
75 × 7 525 a −13β − 10
Area = = = α= ,β=0
8 8 b 2
β(–5, +10)C, (0, 2)
a + b = 533
−8
m(BC) =
5
Sol 30:
y − 2 −8
A(1,2) =
x 5
y intercept = 2
L1=0 L2=0
(iii) L1 = x + y – 1 = 0
(,2)B C(,3-) L2 = 2x – y + 4 = 0
L=0
A
(i) L1 = 2x – y = 0
L2 = x + y = 3 E F
S
G = (2, 3), A(1, 2) (2,1)
⇒α+β+1=6 B C
(,1 - ) D
(,2+4)
⇒ 2 + 2α + 3 – β = 9
⇒ 2α – β = 4 ⇒ 3α = 9 S(2, 1)
α = 3, β = 2 A(–1, 2)
B(3, 6)C(2, 1) α + β 2β − α + 5
D ,
y −1 2 2
Eq. ⇒ =5 α −1 3 − α β −1
x−2 E= , ; F = ,β + 3
2 2 2
y + 5 = 5x
m=5 Now, m(SE) = 1
(ii) If H = (2, 3) α −1
−2
L1 = 2x + y = 0 / L2 = x – y + 2 = 0 ⇒ 2 =1
3−α
−1
−2 4 −2 4 2
x= y= A ,
3 3 3 3
⇒ α – 5 = 1 – α; α = 3
B(α, –2α), C (β, β + 2) −1
m(SF) =
4 2
3−
3 5
Slope of AH = = β −1
2 8 −2
β −5
2+
3 ⇒ 2 = = –2
β + 3 −1 2(β + 2)
−8 B + 2α + 2
Slope of BC = = –4(β + 2) = β – 5
5 β−α
3 + 2α −3
Slope of BH = 5β = –3β =
2−α 5
2 −3 14
β+ β = (3, –2), C ,
α−2 3 =1
Slope of AC = = 5 5
3 + 2α 2
β+ y+2 −24 y+2 −4
3 = ⇒ =
x −3 18 x −3 3
8 . 7 4 | Straight Line
3 3 2 1
⇒ x intercept 3 – = sin θ = , cos θ =
2 2 5 5
C = (6, 4)
Exercise 2 D = 0 + 2 5 cosθ, 2 + 2 5 sinθ = (2, 6)
Eq. of AC
Single Correct Choice Type
y −2 1
= ⇒ x = 3y – 6 ….(i)
Sol 1: (A) a(3x + 4y + 6) + b(x + y + 2) = 0 x−0 3
(2, 3) is situated at greater distance y −0
Eq. of BD = = –3
x−4
a(6 + 12 + 6) + b(2 + 3 + 2) y
D= x=4– ….(ii)
2 2
(3a + b) + (4a + b) 3
Solving eq. (i) & (ii) we get the required Point is (3, 3)
24a + 7b 24T + 7
D= =
(3a + b)2 + (4a + b)2 (3T + 1)2 + (4T + 1)2 Sol 3: (C)
A
a
Where T =
b
px+qy=1 qx+py=1
dD
=0
dT
B C
⇒ (3T + 1)2 + (4T + 1)2 × 24 (p,q)
= (2pq – 1)(px + qy – 1) 9 7
⇒ 4β – 8 = 3α – ⇒ 3α – 4β + =0
= (p2 + q2 – 1) (qx + py – 1) 2 2
27 36 25β − 36 4
Sol 4: (B) 3x + 4y = 9 × 4 Mid point of AC= , = =
25 25 25α − 27 3
⇒ 12x + 16y = 36 (Slope of ⊥ to AC)
4x – 3y + 12 = 0 × 3 4α
⇒ 100α – 108 = 75β – 108 ⇒β =
⇒12x – 9y = –36 3
3
25y = 72 α= β=2
2
72
y= 9 5
25 Circumradius = +4 =
4 2
4 × 72
9−
25 4 × 24 21
⇒x = = 3− = − a 1 1
3 25 25
Sol 5: (D) 1 b 1 = 0
A(3,0) 1 1 c
3x − y 3x − y
2 12x2 + y2(3k2 – 2k – 6) + xy(6k + 4k + 10) = 0
– 4 y – 8 =0
2 2 −2h −(10k + 10)
m 1 + m2 = = =0 ⇒ k = –1
7x2 + 8y2 – 4xy + 3x2 – xy – 6xy b 3k 2 − 2k − 6
1 2 1 14 + 1 10 + 2
=4⇒ − = –4 ⇒ m = + C= , ⇒ (5,4)
m 2 3 3
m2
2 2m − 1
(C1 + C2) + =1 = ±1
a m+2
1 2m – 1 = –2 – m
(C1 – C2) =
2 1 1
m= − or m = 3(m = − is not possible)
6 3 3 3
C 1C 2 = − = −
4 2 ⇒ y = 3x – 11 ⇒ 3x – y – 11 = 0
C1 − C2
Distance between lines =
1 + m2 Sol 12: (A) ax2 + 2hxy + by2 = 0
1
C 1 + C2 =
2 m2
−6
C 1C 2 =
4 m
1
+6
C1 − C2 4 5
= = = 5
1 + m2 1 5 m1
1+ 2
4 4
m − m1 m2 − m
=
1 + mm1 1 + m2m
Sol 9: (B) 5x2 + 12xy – 6y2 + 4x(x + ky) [Homogenising]
⇒ m + m 2m 2 – m 1 – m 1m 2m
– ay(x + ky) + 3(x + ky)2 = 0
= m2 – m + mm1m2 – m2m1
9x2 – 6y2 – 2ky2 + 10xy + 4kxy + 3(x2 + k2y2 + 2kxy) = 0
⇒ 2m – (m1 + m2) + (m1 + m2)m2 = 2m1m2m
M a them a ti cs | 8.77
2h 2h 2 2a mAB. mBC = –1
⇒ 2m + – m = m
b b b 9 − 2x
mAB =
⇒ 2mb – 2am = –2h + 2hm 2 x−4
hm2 + m(a – b) – h = 0 13 − 2x
mBC =
x−2
Sol 13: (D) (9 – 2x)(13 – x) = –1(x2 – 6x + 8)
3x-y=-3 117 – 18x – 26x + 4x2 = –x2 + 6x – 8
5x2 – 50x + 125 = 0
(0,3)
2x-3y = 6 x2 – 10x + 25 = 0 ⇒ x = 5; y = 0
1 1
Area = × BC × AB = 9+9 1+1 = 3
(-1,0)
2 2
(3,0)
(0,-2)
3x+4y = 12 Multiple Correct Choice Type
2x+3y=29 x+2y=16 ⇒ h = 7 or –3
∴ Co-ordinate are (7, 5) or (–3, –5)
(,)B C(10-,12-)
(5, 6)
Sol 17: (B, C) y + 3x = 2
2α + 3β = 29
y– 3x = 2
10 – α + 24 – 2β = 16
x = 0, y = 2
α + 2β = 18 …(i)
2α + 4β = 36 …(ii) -1/3 A
m
4/3
On solving, we get 1/3 60
β = 7, α = 4 2/3
So we have, B(4, 7) A
y −7
Eq. of BC = = –1 ⇒ x + y = 11 2/3
x−4
B
C(2,-3) ⇒ 3 – m –3m+m2 3 =m2/ 3 +m+3m+ 3
(x,10-2x)
2x+y=0
8 . 7 8 | Straight Line
⇒m=0
B(0, b)
Bisector line is y = 2
(, )
P 2
⇒ cos 60º = ; P= A’(-a’, 0) A(0, a)
4/ 3 3
2 B’(0, b’)
⇒ Given point is ,2
3 −b
m(AB) =
a
Or the other possibility is m = ∞
b
⇒ Foot of perpendicular = (0, 0) m(BA′) =
a′
Let orthocentre be at (α, β)
Sol 18: (A, B, D)
A(1,3) b−β
=∞⇒α=0
0−α
β−0 a
P(x,y) =
α + a′ b
(5,6)B C(-1,2)
aa′
β= = b′ [From (i)]
y −6 3 b
AB - =
x −5 4
Sol 20: (A, B, C, D)
4y – 24 = 3x – 15 ⇒ 3x – 4y + 9 = 0
y −6 2 C(0,a)
BC - =
x −5 3 B(x,x/3)
60
3y – 18 = 2x – 10 ⇒ 2x – 3y + 8 = 0 30
y −3 (0,0) -30
1
AC - = x2 + x2/3 = a2
x −1 2
2y – 6 = x – 1 ⇒ x – 2y + 5 = 0 (0,-a) x=3a/2
Sol 21: (B, C) Vertices are (4, 3)(1, –1) & (7, k) Sol 23: (B, C) (x + 2) (y + 2) = 0; x + y + 2 = 0
x=-2 y
(1,-1)B C(7,k)
2+k A
C = 4, x
3
That occurs only in isosceles ∆ B C y=-2
(1 – 4)2 + (3 + 1)2 = (k – 3)2 + 9
y=-x-2
AC = AB
Sol 24: (B, C) x + y = 1; x = 7y; x = 3y
16 = (k – 3)2
Centroid and In centre always Lie inside of the triangle.
⇒ k – 3 = ±4
⇒ k = 7, –1 or (7/8, 1/8)
25 = (k + 1) + 36
2
(0,0)
(BC = AC)
Sol 25: (B, C)
18 – 6k = 37 + 2k
y+2x=5
−19
⇒ 8k = –19 ⇒ k = m
8
For k = 7AB = 5
AC = 5, BC = 10
[∆ is not possible] m − ( −2)
tan θ =
1 + m( −2)
−19
So k = –1 or m+2 1
8 =±
1 − 2m 2
Sol 22: (A, C) (7, 4) 2m + 4 = ±(1 – 2m)
y – 4 = m(x – 7) ⇒ 2m + 4 = 2m – 1 ⇒(m = ∞)
Centre ≡ (3, –2) ⇒ x = 2 (B)
(3 − 7)m + 4 + 2 2m + 4 = 1 – 2m
⇒ =4
1 + m2 3
4m = –3 ⇒ m = −
⇒ (–4m + 6) = 16(1 + m )
2 2
4
⇒ 9 + 4m2 – 12m = 4 + 4m2 y −3 −3
=
x−2 4
5
⇒ m= or m = ∞
12 3x + 4y = 18 (C)
5
⇒ y–4= (x – 7)
12
⇒5x – 12y + 13 = 0 and x = 7
8 . 8 0 | Straight Line
/2
y – a = m(x – 1) L3 P R Q
y=-2
(-2,-2) (1,-2)
y = m(x – 1)
y’
− sin75º
m = – tan 75º =
cos75º
Sol 2: (A) Solving equations L1 and L2.
−( 3 + 1)
= = – (2 + 3) x y 1
( 3 − 1) ⇒ = =
–36 + 10 –25 + 12 2 – 15
y = – (2 + 3) (x – 1) ∴ x = 2, y = 1
(2 + 3) x + y = (2 + 3) (C) L1, L2, L3 are concurrent if point (2, 1) lies on L2
x + (2 − 3) y = 1 ∴ 6 – k – 1= 0 ⇒ k = 5
(A) → (S)
Sol 27: (B, C) y – y1 = m(x – x1) (B) Either L1 is parallel to L2 or L3 is parallel to L2, then
y = y1 + m(x – x1) 1 3 3 –k
= or =
(B) Set of parallel lines 3 –k 5 2
–6
(C) All these lines pass through x = x1 ⇒ k = – 9 or k =
5
(B) → (p, q)
(C) L1, L2, L3 form a triangle, if they are not concurrent,
or not parallel.
6 5
∴ k ≠ 5, – 9, – ⇒k=
5 6
(c) → (r)
(D) L1, L2, L3 do not form a triangle. If
6
Previous Years’ Questions k = 5, – 9, –
5
(D) → (p, q, s)
Sol 1: (C) It is not necessary that the bisector of an
angle will divide the triangle into two similar triangle, Sol 3: (A, C) Given lines px + qy + r = 0, qx + ry + p = 0
therefore, statement-II is false.
Now we verify statement-I and rx + py + q = 0 are concurrent.
x + y= 3 – x + 2 – y P
x
O
5
⇒ 2x + 2y = 5 ⇒x+y=
2 x+y=1 x+y=3
Infinite segment
Sol 10: Let the square S is to be bounded by the lines x
y 1 1
x=1/2 =± and y = ±
2 2
(1/2,2)
Finite segment
y=2
2 2 2
x+y=5/2,
1 1
We have,
= a2 x1 – + – y1
2 2
1
y
x A(x1,1/2) 1/2
O 1/2 (5/2, 0) 3
D(-1/2,y2) d
a
1/2
Now, Eq.(i) becomes x + y = x – 3 + 2 – y x’ O
x
-1/2 B(1/2, y1)
c
1
⇒ 2y = – 1 or y= – b
2 C(x2-1/2) -1/2
Hence, no solution. y’
Case IV : When x ≥ 3, y ≥ 2
1
= x12 – y12 – x1 – y1 +
In this case (i) changes to x + y = x – 3 + y – 2 2
⇒0=–5 1
Similarly,
= b2 x22 – y12 – x2 + y1 +
2
Which is not possible.
1
Hence, this solution set is {(x, y)} | x = 12, y ≥ 2} ∪ {(x, y)} | c2 = x22 – y 22 + x2 + y 2 +
2
x + y = 5/2, 0 < x < 3, 0 < y > 2 } 1
d2 = x12 – y 22 + x1 + y 2 +
The graph is given in adjoining figure. 2
0 ≤ x12 + x22 + y12 + y 22 ≤ 1 Sol 11: Let the vertices of a triangle be, O(0, 0) A(a, 0)
and B(b, c) equation of altitude BD is x =b.
⇒ 0 ≤ 2(x12 + x22 + y12 + y 22 ) ≤ 2
c
Slope of OB is .
But 2 ≤ 2(x12 + x22 + y12 + y 22 ) + 2 ≤4 b
b
Alternate Solution Slope of AF is – .
c
2
c= x22 + y 22 ….(i) Now, the equation of altitude AF is
y
y
B(b,c)
A(x1,1)
a
d B(1,y1) F 90
90 E
P
(0,y2)D
b x’ x
c O D(b,0) A(a,0)
x’ x y’
O C(x2,0)
b
y – 0 = – (x – a)
y’ c
=a2 (1 – y1 )2 + (1 – x1 )2 …..(iii) a – b
Coordinates of P are b,b
c
2
d= x12 + (1 – y 2 ) 2
.....(iv)
On adding Eqs. (i), (ii), (iii) and (iv), we get a–b
Let m1 be the slope of OP =
2 2
a +b +c +d = 2 2
{x12 + (1 – x1 ) } 2
+ {y12 2
+ (1 – y1 ) } c
+ {x22 + (1 – x2 )2 } + {y 22 + (1 – y 2 )2 } c
and m2 be the slope of AB =
b–a
Where x1, y1, x2, y2 all vary in the interval [0, 1].
a – b c
Now, consider the function y = x2 + (1 – x)2, 0 ≤ x ≤ 1 Now, m1m2 =
= –1
c b – a
dy
Differentiating ⇒ ⇒ 2x – 2(1 – x) . For maximum
= We get that the line through O and P is perpendicular
dx
dy to AB.
or minimum =0.
dx
⇒ 2x – 2(1 – x ) = 0 ⇒ 2x – 2 + 2x = 0 Sol 12: Since, line L make 60° with line 3 x + y =
1, then
1
⇒ 4x = 2 ⇒x=
2
d2 y
Again, =2+2 = 4
dx2
⇒ Which is positive.
1
Hence, y is minimum at x = and its minimum value
1 2
is . − 3 + tan 60°
= m1 = 0
4
Clearly, value is maximum when x = 1.
1 − − 3 ( tan 60° ) ( )
− 3 − tan 60° −2 3 −2 3
1 1 1 1 m2 = = = = 3
∴ Minimum value of a2 + b2 + c2 + d2 = + + + =2
2 2 2 2 ( )
1 + − 3 tan 60° 1−3 −2
y +=
2 3 ( x − 3) α −β α+β
⇒2≤ + ≤4
⇒ y + 2= 3x − 3 3 2 2
⇒ y − 3x + 2 + 3 3 =0
Case 2: α<β
Sol 14: Let P be ( α , β ) , then
−α + β α+β
α −β 2≤ + ≤4⇒ 2 ≤β≤2 2
d1 (P ) = 2 2
2
( ) − ( 2) = 8 – 2
2 2
α+β Area = 2 2
d2 (P ) =
2 = 6 sq. unit
⇒ 2 ≤ d1 (P ) + d2 (P ) ≤ 4
2017-18 100 &
op kers
Class 11 T
By E ran culty
-JE Fa r
IIT enior emie .
S fP r es
o titut
Ins
MATHEMATICS
FOR JEE MAIN & ADVANCED
SECOND
EDITION
Exhaustive Theory
(Now Revised)
Formula Sheet
9000+ Problems
based on latest JEE pattern
PlancEssential
Questions recommended for revision
9. CIRCLE
1. INTRODUCTION
Definition: The locus of a point which moves in a plane such that its distance
from a fixed point in that plane always remains the same (i.e., constant) is P moving point
known as a circle.
The fixed point is called the centre of the circle and the distance between the .
fixed point
fixed point and moving point is called the radius of the circle.
PLANCESS CONCEPTS
• The general equation ax2 + 2hxy + by 2 + 2gx + 2fy + c =0 can be written in matrix form as
a h g x
a h x
x y + 2gx + 2fy + c =0 and x y 1 h b f y = 0
h b y g f c 1
• Degeneracy condition depends on determinant of 3x3 matrix and the type of conic depends on
determinant of 2x2 matrix.
• Also the equation can be taken as intersection of z =ax2 + 2hxy + by 2 and the plane z =
− 2gx + 2fy + c ( )
Vaibhav Gupta (JEE 2009, AIR 54)
P(x,y)
X’ X
O (0,0)
2 2 2
X +Y =a
Y’
C(h,k)
2 2 2
(x-h) +(y-k) =r
X’ X
O
Y’
Figure 9.3: Central form
M a them a tics | 9.3
x2 + y 2 x y 1
x12 + y12 x1 y1 1
=0
x22 + y 22 x2 y2 1
x32 + y 32 x3 y3 1
PLANCESS CONCEPTS
Whenever the problem seems to be very complicated using formulas and geometrical approach, then
try to apply trigonometric approach as well. Like the given circle may be in circle or ex-circle of some
triangle. May be using properties of triangle we can solve it.
Vaibhav Gupta (JEE 2009, AIR 54)
9 . 4 | Circle
Illustration 1: Find the equation of the circle which passes through the point of intersection of the lines x – 4y – 1 = 0
and 4x + y – 21 = 0 and whose centre is (2, –3). (JEE MAIN)
Sol: By solving given equation of lines simultaneously we will get point of intersection of lines i.e. P. Therefore
using distance formula we will get radius of circle and using centre and radius we will get required equation.
Let P be the point of intersection of the lines
x – 4y – 1 = 0 …. (i)
and 4x + y – 21 = 0 …. (ii)
( ) ( )
From (i) and (ii), we get x = 5 , y = 1 . So, coordinates of P are 5,1 .Let C 2, −3 be the centre of the circle.
Since the circle passes though P, therefore
(5 − 2 ) + (1 + 3 )
2 2
CP= radius ⇒ = radius ⇒ radius = 5
Illustration 2: Find the equation of a circle of radius 10 whose centre lies on x-axis and passes through the point
(4, 6). (JEE MAIN)
(a − 4 ) + (0 − 6)
2 2
⇒ CA =10 ⇒ = 10
⇒ (a – 4)2 + 36 = 100 ⇒ (a – 4)2 = 64
⇒ a – 4 = ±8 ⇒ a = 12 or a = – 4.
Thus, the coordinates of the centre are (12, 0) or − 4,0 . ( )
Hence, the equations of the required circles are Y’
Figure 9.5
( x − 12) + ( y − 0 ) 102 and ( x + 4 ) + ( y − 0 ) =
2 2 2 2
= 102
Illustration 3: Find the equation of the circle concentric with the circle x2 + y 2 + 4x + 6y + 11 =
0 and passing
( )
through the point 5, 4 . (JEE MAIN)
Sol: Since both circle are concentric therefore their centre should be same. (5,4)
Hence equation of a required circle can be written as x2 + y 2 + 4x + 6y +
(constant term) = 0.
Let the equation of the concentric circle be
x2 + y2 + 4x + 6y + k = 0
(-2,-3)
Since the point (5,4) lies on this circle,
∴ (5)2 + (4)2 + 4(5) + 6(4) + k = 0
⇒ 25 + 16 + 20 + 24 + k = 0 ⇒ k = –85
Therefore, the equation of the required circle is
x2 + y2 + 4x + 6y – 85 = 0 Figure 9.6
M a them a tics | 9.5
Illustration 4: Find the equation of a circle passing through the origin and making intercepts 4 and 3 on the y and
x axis respectively. (JEE MAIN)
Sol: By observing the problem we conclude that given intercepts are end Y
(0,4)
points of diameter of this circle. Therefore by using diametric form we can
P
obtain the equation of circle.
(
Illustration 5: Find the equation of the circle which is circumscribed about the triangle whose vertices −2,3 , 5,2 )( )
(
and 6, −1 . ) (JEE ADVANCED)
Sol: Consider (a, b) as the centre of circle and r as the radius. As circle passes from given vertices, therefore their
distance from the centre are same. Therefore by using distance formula, we will get the value of a, b and r.
( )( )
Since the circle passes through the points −2,3 , 5,2 and 6, −1 . ( )
∴ (–2 – a) + (3 – b) = r ⇒ a + 4a + 4 + b – 6b + 9 = r
2 2 2 2 2 2
…. (i)
(5 – a)2 + (2 – b)2 = r2 ⇒ a2 – 10a + 25 + b2 – 4b + 4 = r2 …. (ii)
(6 – a)2 + (–1 – b)2 = r2 ⇒ a2 – 12a + 36 + b2 + 2b + 1 = r2 …. (iii)
Subtracting (ii) from (i), we have
14a – 21 – 2b + 5 = 0 i.e., 14a – 2b = 16 …. (iv)
Subtracting (iii) from (ii), we get
2a – 11 – 6b + 3 = 0 ⇒ 2a – 6b = 8 …. (v)
Solving (iv) and (v), we get
a = 1 and b = –1
Putting the values of a = 1 and b = −1 in (i), we get
1 + 4 + 4 + 1 + 6 + 9 = r2 ⇒ 25 = r2 ⇒r=5
Thus, the required equation of the circle is
(x – 1)2 + (y + 1)2 = 25 ⇒ x2 + 2x + 1 + y2 + 1 + 2y = 25 ⇒ x2 + y2 – 2x + 2y – 23 = 0
9 . 6 | Circle
X’ a X
O
Y’
Figure 9.8
(b) ( x – h )2 + ( y ± a )2 = a2 ( h , ±a ) ; a Touches x axis only, y Y
coordinate of centre = ±a
C1 (h,a)
X’ O X
G
C2 (h,-a)
Y’
Figure 9.9
(c) ( x ± a )2 + ( y – k )2 = a2 ( ±a, k ) ; a Touches y–axis only, x Y
coordinate of centre = ±a
(-a,k) (a,k)
C1 C2
X’ X
O
Y’
Figure 9.10
M a them a tics | 9.7
C2 C1
(-a,a) (a,a)
X’ X
O
(-a,-a) (a,-a)
C3 C4
Y’
Figure 9.11
(e) x2 + y2 – 2ax = 0 C (a, 0) ; a When the circle passes Y
through the origin and
centre lies on x axis
X’ X
O (a,0)
Figure 9.12
(f) x2 + y2 – 2ay = 0 C (0, a); a When the circle passes Y
through the origin and
centre lies on y axis.
(0,a)
X’ X
O
Y’
Figure 9.13
9 . 8 | Circle
1
α2 + β2 ,
2 2 2
X’ X
O
Y’
Figure 9.14
(h, h); h When circle touches both
( ) + ( y − h)
2 2
(h) x − h h2 or
=
the axes
x2 + y 2 − 2hx − 2hy + h2 =
0
(h,h)
Figure 9.15
Y
Illustration 6: Find the equation of the circle which passes through two points on
the x–axis which are at distances 12 from the origin and whose radius is 13.
C
Sol: There are two circles which passes through two points A and A’ on x–axis which
are at a distance 12 from the origin and whose radius is 13. The centre of these circles X’
A’ O A
X
lie on y–axis (perpendicular bisector of chord AA’)
In ∆AOC, AC2 = OA2 + OC2
⇒ 132 = 122 + OC2 ⇒ OC = 5
So the coordinates of the centre of the required circles are (0, 5) and C’ (0, –5). Hence Y’
the equations of the required circles are Figure 9.16
(x – 0 ) + (y ± 5 ) = 13 ⇒ x + y ± 10y – 144 = 0.
2 2 2 2 2
The lengths of intercepts made by the circle x2 + y2 +2gx + 2fy + c = 0 on X and Y axes
are 2 g2 − c and 2 f 2 − c respectively.
Therefore,
(a) The circle x2 + y2 +2gx + 2fy + c = 0 cuts the x–axis in real and distinct points, X’
O
X
touches or does not meet in real points according as g2 >c- Distinct points
Y’
g2 = c – Touches
Figure 9.17: Intercept made
g2 < c – Does not meet by circle on x-axis
(b) Similarly, the circle x2 + y2 +2gx + 2fy + c = 0 cuts the y–axis in real and distinct points, touches or does not
meet in real points according as f2 >, = or < c .
M a them a tics | 9.9
(-g,-f)
X’ X
O
Y’
Illustration 7: Find the equation to the circle which touches the positive axis of y at a distance 4 from the origin
and cuts off an intercept of 6 from the axis of x. (JEE MAIN)
Sol: As circle touches Y axis therefore Y coordinate of centre of circle is 4 so by Y
using formula of intercept we will get the value of X coordinate of centre of circle
and c.
Consider a circle x2 + y2 + 2gx + 2fy + c = 0. (5,4)
This meets the axis of y in points given by y2 + 2fy + c = 0 5
4
3
The roots of this equation must be each equal to 4, so that it must be equivalent X’
O
X
to (y – 4)2 = 0 ⇒ 2f = –8 & c = 16
∵ Intercept made on the x-axis = 6
Y’
Figure 9.19
⇒ 6 = 2 g2 − 16 ⇒ g = ±5.
Hence, the required equation is x2 + y2 ± 10x – 8y + 16 = 0.
r
C
(i)
Figure 9.20 (i)
P
r
C
(ii)
Figure 9.20 (ii)
9 .10 | Circle
(c) CP > radius, then the point P lies outside the circle.
P
r
C
(x1 + g)2 + (y1 + f)2 > = < g2 + f 2 − c ⇒ (x1 + g2) + (y1 + f )2 >
= < (g2 + f2 – c )
⇒ x12 + y12 + 2gx1 + 2fy1 + c > = < 0
Or, S1 > = < 0 where S1 = x12 + y12 + 2gx1 + 2fy1 + c
Therefore, a point (x1, y1) lies outside, on or inside a circle
S ≡ x2 + y2 + 2gx + 2fy + c = 0 according as S1 ≡ x12 + y12 + 2gx1 + 2fy1+ c is positive, zero or negative.
P (x1,y1)
C
D
Figure 9.21
The power of P(x1, y1) w.r.t. S = x2 + y2 + 2gx + 2fy + c = 0 is equal to PA.PB which is S1, where S1 = x12 + y12 + 2gx1 + 2fy1 + c .
Power remains constant for the circle i.e. independent of A and B
PA×PB = PC×PD = PT2 = square of the length of a tangent
Illustration 8: The coordinates of the point on the circle x2 + y2 – 2x – 4y – 11 = 0 farthest from the origin are
(JEE MAIN)
8 4 4 8 8 4
(A) 2 + ,1+ (B) 1 + , 2+ (C) 1 + , 2+ (D) None of these
5 5 5 5 5 5
Sol: (B) The required point lies on the normal to circle through the origin, i.e. on the line 2x = y. Therefore by
substituting y = 2x in above equation of circle we will get coordinates of required point.
4 4
x2 + 4x2 – 2x – 8x – 11 = 0 ⇒ 5x2 – 10x – 11 = 0 ⇒ x = 1 ± and y = 2 1 ±
5 5
4 8
and the required point farthest from the origin is 1 + , 2+ .
5 5
Illustration 9: The point (1, 3) is inside the circle S whose equation is of the form x2 + y2 – 6x – 10y + k = 0, k
being an arbitrary constant. Find the possible values of k if the circle S neither touches the axes nor cuts them.
(JEE ADVANCED)
Sol: As (1, 3) lies inside the circle S therefore S1 < 0 and it does not touches x and y axes. On the basis of this we
can solve the problem and will get range of k.
12 + 32 – 6×1 –10×3 + k < 0; ∴ k < 26 …(i)
Solving y = 0 and x + y – 6x – 10y + k = 0, we get x – 6x + k = 0
2 2 2
i.e., the line will intersects the circle at two real and different points.
(b) If a2 (1 + m2 ) = c2, | c | a 1 + m2
or, =
i.e., the line will touch the circle at only one point i.e. the line will be a tangent. (2,4)
i.e., the line will meet the circle at two imaginary points.
Figure 9.23
Illustration 10: Show that the line 3x – 4y – c = 0 will meet the circle having centre at (2, 4) and the radius 5 in real
and distinct points if – 40 < c < 20. (JEE MAIN)
Sol: Since the line cuts the circle so length of perpendicular from centre of circle upon line is less than the radius
of circle.
9 .12 | Circle
3× 2 − 4 × 4 − c
<6 ⇒ 10 + c < 30
9 + 16
⇒ –30 < 10 +c < 30 ⇒ –40 < c < 20
Illustration 11: If 4l2 – 5m2 + 6l +1 = 0 then show that the line lx + my +1 = 0 touches a fixed circle. Find the radius
and centre of the circle. (JEE ADVANCED)
Sol: If line touches the circle then perpendicular distance from centre of circle to the line is equal to the radius of
circle so by using distance formula of point to line we will get one equation and other is given 4l2 – 5m2 + 6l +1 =
0 . hence by solving these two equation we will get required answer.
Let the circle be (x – α)2 + (y – β)2 = a2
The line lx + my + 1 = 0 touches the circle if
l α + mβ + 1
a= or a2(l 2 + m2) = (lα + mβ +1)2
2 2
l +m
or (a2 – α2) l2 + (a2 – β2) m2 –2 lβα m – 2 l α – 2mβ – 1 = 0 ... (i)
But 4l – 5 m + 6l + 1 = 0
2 2
... (ii)
It is possible to find α,β, a if (i) and (ii) are identical.
The condition is
a2 − α2 a2 − β2 −2α 2β −1
= = = = ... (iii)
4 −5 6 0 1
a2 − α2
∴ β = 0, α =3 and = –1 which implies a2 – 32 = –4, i.e., a = 5 .Also α = 3, β = 0, a = 5 satisfies equation
4
( )
2
(iii) and hence the line touches the fixed circle (x – 3 )2 +(y – 0)2 = 5
Illustration 12: Find equation of a line with slope gradient 1 and such that x2 + y2 = 4 and x2 + y2 – 10x – 14y + 65 = 0
intercept equal length on it ? (JEE ADVANCED)
Sol: As given slope of line is 1, therefore its equation will be y = x + c Hence by using perpendicular distance
formula we will get distance of line from centre of respective circle and then
by using Pythagoras we can obtain length of intercepts made by line to these
A’
circles and which are equal. Therefore we can obtain value of c and required
equation of circle.
C(5,7)
Let 2 be the length of the intercept made by the two circle.
For x2 + y2 = 4, centre ≡ (0,0) and radius = 2, and A
For x2 + y2 – 10x – 14y + 65 = 0, centre ≡ (5,7) and
O(0,0)
radius = 3.
c 5−7+c c−2
∴ OA = and CA’ = ⇒CA’ = Figure: 9.24
2 2 2
4 – OA2 = 2 …(i)
M a them a ti cs | 9.13
Illustration 13: Find the values of α for which the point (2α, α +1) is an interior point of the larger segment of the
circle x2 + y2 – 2x –2y – 8 = 0 made by the chord whose equation is x – y + 1 = 0. (JEE ADVANCED)
Sol: As point (2α, α +1) lies inside the circle S, therefore S1 < 0. Hence by substituting
the point in the equation, we will get the range of α and as it lies in larger segment =0
+1
made by line x – y + 1 = 0. The centre of circle i.e. (1, 1) and (2α, α +1) will have the x-y
same sign.
∴ (2α)2 + (α +1)2 – 2×2α – 2(α + 1) – 8 < 0
Centre
⇒ 5a2 – 4α – 9 < 0 or (5α – 9)(α + 1) < 0
(2,+1)
⇒ –1 < α <9/5 ... (i)
Also, as the point lies in the larger segment, the centre (1, 1) and the point (2α, α + 1) Figure 9.25
must be on the same side of the line x – y + 1 = 0.
Clearly, 1 – 1 + 1 > 0; So, 2α – (α + 1) + 1 > 0;
∴ α > 0 … (ii)
9
∴ The set of values of α satisfying (i) and (ii) is 0, .
5
7. TANGENTS
2 2 2
x +y=a
P(x1,y1)
Figure 9.26
(b) The equation of tangent at (x1, y1) to circle x2 + y2 + 2gx + 2fy + c = 0 is xx1 + yy1 + g(x + x1) + f(y + y1) + c = 0.
Illustration 14: For what value of c will the line y = 2x + c be a tangent to the circle x2 + y2 = 5. (JEE MAIN)
∴c=± 5 × 1 + 22 ⇒ c = ± 5
The length of the tangent drawn from a point P(x1, y1 ) to the circle S =x2 + y2 + 2gx +2fy + c = 0 is PT1 = PT2 =
Note:
(i) P
T2 is called the power of the point with respect to a given circle, where PT is the tangent from a point P to a
given circle.
(ii) Area of quadrilateral PT1C T2 = 2 × (Area of triangle PT1C), and
r
(iii) The angle between tangents PT1 and PT2 is equal to 2tan–1
P(x1,y1)
S .
1
Illustration 15: If OA and OB are tangents from the origin O, to the circle x2 + y2 +2gx +2fy
+ c= 0, c > 0 and C is the centre of the circle, then area of the quadrilateral OACB is
(JEE MAIN)
(A)
1
2
( )
c g2 + f 2 − c (B) ( )
c g2 + f 2 − c
g2 + f 2 − c
(C) c g2 + f 2 − c (D)
c
T1
Sol: (B) As we know quadrilateral OACB is formed by two right angle triangle OAC and r
C
T3
triangle OBC. Line OA and OB are tangent to the circle from common point O. Therefore
OA = OB and (AC = CB) radius of circle, hence both triangle are equal. Therefore Area of
the quadrilateral OACB = 2 Area of the triangle OAC. 2 2
x +y -2gx+2fc c=0
OA = OB = S1 = c (Length of the tangent from the origin) Figure 9.27
M a them a ti cs | 9.15
Illustration 16: The locus of a point which moves such that the tangents from it to the
two circles x2 + y2 – 5x – 3 = 0 and 3x2 + 3y2 + 2x + 4y – 6 =0 are equal, is (JEE MAIN)
(A) 7x + 4y – 3 = 0 (B) 17x + 4y + 3 = 0 (C) 3x – 4y + 9 = 0 (D) 13x – 4y + 15 =0
Sol: (B) Use the formula for length of tangent. Let P(h, k) be any point on the locus.
2 4 6
Similarly, the length of the tangent to the other circle is h2 + k 2 + h + k − . …(ii) O (0,0)
3 3 3
On equating (i) and (ii), we get 17h + 4k + 3 = 0, Figure 9.28
Pair of tangents from point (0, 0) to the circle are at right angles if g2 + Figure 9.29
f2 = 2c.
Illustration 17: Find the equation of the pair of the tangents drawn to the circle x2 + y2 – 2x + 4y = 0 from the
point (0, 1). (JEE MAIN)
Sol: Here (x1, y1) = (0, 1). So by using formula SS1 = T2 we can get required equation, where S = x2 + y 2 − 2x + 4y =
0,
S1 ≡ x12 + y12 +2gx1 +2fy1 + c and T = xx1 + yy1 – (x+x1) +2(y+x1).
0
Given circle is S ≡ x2 + y 2 − 2x + 4y = …(i)
Let P be the point (0, 1).
∴ S1 ≡ x12 + y12 +2gx1 +2fy1 + c ⇒ S1 ≡ 02 + 12 – 2.0 + 4.1 = 5
And, T ≡ xx1 + yy1 – (x+x1) +2(y+x1) ⇒ T ≡ x(0) + y(1) – (x + 0) + 2(y + 1)
i.e., T ≡ – x + 3 y + 2.
Hence, the equation of pair of tangents from P (0, 1) to the given circle is SS1 = T2
( )
i.e. 5 x2 + y 2 − 2x + 4y = ( −x + 3y + 2 )
2
Illustration 18: From a point on the line 4x – 3y = 6 tangents are drawn to the circle x2 + y2 – 6x –4y + 4 = 0 which
24
make an angle of tan–1 between them. Find the coordinates of all such points and the equations of tangents.
7 (JEE ADVANCED)
Illustration 19: Obtain the locus of the point of intersection of the tangents to the circle x2 + y2 = a2 which include
an angle α. (JEE ADVANCED)
Sol: Consider (x1, y1) as the point of intersection of tangents to the given circle (x1,y1)
α a
and then use tan = to get the desired result.
2 S 1
O
Let (x1, y1) be the point of intersection of a pair of tangents to the given circle.
If the pair of straight lines includes an angle α, then
a
2 2
x +y =a
2 2
α a S1
⇒ tan = ⇒ tan α = Figure 9.31
2 S1 a2
1−
S1
2a x12 + y12 − a2
⇒ tan α =
y12 + x12 − 2a2
(
⇒ x12 + y12 − 2a2 )
2
( 2 2
tan2 α = 4a2 x1 + y1 − a
2
)
Hence, the required locus is (x2 + y2 – 2a2)2 tan2 α =4a2 (x2 + y2 – a2).
Illustration 20: Find the equation of the director circle of the circle (x – 2)2 + (y + 1)2 = 2. (JEE MAIN)
Sol: As we know, for the circle x2 + y2 = a2, the equation of the director circle is x2 + y2 = 2a2 .
For the given circle, Centre ≡(2, –1) & Radius = 2.
∴ The centre of the director circle ≡ (2, – 1), and the radius of the director circle = 2 × 2 = 2.
∴ The required equation is (x – 2)2 + (y + 1)2 = 4.
8. NORMALS
The normal of a circle at any point is a straight line, perpendicular to
the tangent and passing through the centre of the circle.
(a) Equation of normal: The equation of normal to the general l
form of the circle x2 + y2 + 2gx + 2fy + c = 0 at any point rma C
No
(x1, y1) on the circle is P
y1 + f x − x1 y − y1
(y – y1) = (x – x1) or, =
x1 + g x1 + g y1 + f
Tangent
The equation of normal to the circle x2 + y2 = a2 at any point (x1 ,y1) is
x y Figure 9.33
xy1 – xy1 = 0 or, = .
x1 y1
x y
(b) Parametric Form: Equation of normal at (a cos θ, a sin θ) to the circle x2 + y2 = a2 is =
acos θ asinθ
x y
or, = or, y = x tan θ or, y = mx (where m = tan θ).
cos θ sin θ
Illustration 21: Find the equation of the circle having the pair of lines x2 + 2xy + 3x + 6y = 0 as its normal and
having the size just sufficient to contain the circle x(x – 4) + y(y – 3) = 0. (JEE ADVANCED)
Sol: By solving equation x2 + 2xy + 3x + 6y = 0 we will get point of intersection of normals i.e. centre of required
circle. As given circle x(x – 4) + y(y – 3) = 0 lies inside the required circle hence distance between centres will be
equal to the difference between their radius, therefore we can find out radius of required circle by using distance
formula.
Given the equation of pair of normal is x(x + 3) + 2y(x + 3) = 0
⇒ (x + 3)(x + 2y) = 0
∴ Either (x+3)=0 …(i) or (x+2y)=0 …(ii)
3
On solving (i) and (ii), we get x = −3 and y =
2 (-3, 3/2) (2, 3/2)
3
∴ The centre ≡ −3, (The point of intersection of the normals).
2
For the circle x2 + y2 – 4x – 3y = 0 …(iii)
2
3 −3 5
( −2)
2 Figure 9.34
centre = 2, and radius, r = + − 0 = .
2 2 2
If the circle x2 + y2 – 4x – 3y = 0 lies inside another circle of radius ‘a’, then
3 3
a – r = distance between the centres −3, and 2,
2 2
2
5 5 15
( −3 − 2) + 32 − 32
2
⇒a– = ⇒a=5+ ∴a= .
2 2 2
M a them a ti cs | 9.19
2 2
3 15
Hence, the equation of the circle is (x + 3) + y − = or, x2 + y2 + 6x – 3y = 45.
2
2 2
Consider a point P(x1, y1) lying outside the circle. Tangents are drawn to touch the
given circle at Q and R respectively (as shown in the diagram). The chord joining P
Chord of
the points of contact of the two tangents to a circle (or any conic) from the point (x1, y1)
contact
P, outside it, is known as the chord of contact.
(x”, y”)R
9.1 Equation of Chord of contact Figure 9.35
The equation of the chord of contact of tangents drawn from a point (x1 , y1) to
the circle x2 + y2 = a2 is xx1 + yy1 = a2. Equation of chord of contact at (x1, y1) to the circle x2 + y2 + 2gx + 2fy + c = 0
is xx1 + yy1 + g(x + x1) + f(y + y1) + c =0 .
Clearly, the equation of the chord of contact coincides with the equation of the tangent.
PLANCESS CONCEPTS
( )
3/2
1 a x12 + y12 − a2 RL3
• Area of ∆PQR = × PM × QR = =
2 x12 + y12 R 2 + L2
where, the length of the tangent, L = x12 + y12 − a2 and,
radius of circle, R = a.
Illustration 22: Find the equation of the chord of contact of the tangents drawn from (1, 2) to the circle x2 + y2 – 2x
+ 4y + 7 = 0. (JEE MAIN)
Illustration 23: The locus of the point of intersection of the tangents at the extremities of a chord of the circle x2
+ y2 = a2 which touches the circle x2 + y2 – 2ax = 0 passes through the point (JEE ADVANCED)
a a
(A) , 0 (B) 0, (C) (0, a) (D) (a, 0)
2 2
Sol: (A) and (C) Apply the condition of tangency to the equation of chord of contact.
Let P (h, k) be the point of intersection of the tangents at the extremities of the chord AB of the circle x2 + y2 = a2.
∴ The equation of the chord of contact AB w.r.t. the point P is hx + ky = a2.
h(a) + k(0) − a2
The line hx + ky = a2 touches the circle x2 + y2 – 2ax = 0 if =a
h2 + k 2
⇒ (h – a)2 = h2 + k2
Therefore, the locus of (h, k) is (x – a)2 = x2 + y2 or, y2 = a(a – 2x).
Clearly, points (A) and (C) satisfy the above equation.
PLANCESS CONCEPTS
The smallest chord of a circle passing through a point 'M' at a maximum distance from the centre is the
one whose middle point is M.
Shrikant Nagori (JEE 2009, AIR 30)
Illustration 24: Find the equation of the chord of the circle x2 + y2 + 6x + 8y – 11 = 0, whose middle point is (1, –1).
(JEE MAIN)
Sol: Use T = S1
Given, S ≡ x2 + y2 + 6x + 8y – 11 = 0
For point L (1, 1), S1 = 12 + (–1)2 + 6.1 + 8(–1) – 11 = –11 and
T = x.1 + y.(–1) + 3(x + 1) + 4(y – 1) – 11 i.e. T = 4x + 3y – 12
Now equation of the chord having middle point, L(1, –1) is
∴ 4x + 3y – 12 = –11 ⇒ 4x + 3y – 1 = 0
Second method:
Let C be the centre of the given circle, C ≡ (–3, –4)
−4 + 1 3
∴ Slope of CL = =
−3 − 1 4
∴ Equation of chord whose middle point is L, is
M a them a ti cs | 9.21
4
∴ y + 1 = – (x – 1) [ chord is perpendicular to CL]
3
Or, 4x + 3y – 1 = 0
Illustration 25: Find the locus of the middle points of chords of given circle x2 + y2 = a2 which subtends a right
angle at the fixed point (p, q). (JEE ADVANCED)
Sol: As M(h, k)be the midpoint of the chord AB which subtends an angle of 900 at the point N (p , q) therefore a
circle can be drawn with AB as the diameter and passing through the point N, hence AM = MN.
∴ AM = MN ⇒ AM2 = MN2 ⇒a2 – (h2 + k2) = (h - p)2 + (k - q)2 A
⇒ a2 – h2 – k 2 =
h2 + p2 – 2hp + k 2 + q2 − 2kq
⇒ 2h2 + 2k 2 – 2ph − 2qk + p2 + q2 − a2 =
0 N(p,q) M(h,k)
1 2
⇒ h2 + k 2 – ph − qk +
2
(
p + q2 − a2 = )0
1
( )
B
Hence, the required locus is x2 + y 2 – px − qy + p2 + q2 − a2 =
0. Figure 9.38
2
Equation of common chord: The equation of the common chord of two circles
S1 ≡ x2 + y2 + 2g1x + 2f1y + c1 = 0 …. (i)
and S2 ≡ x2 + y2 + 2g2x + 2f2y + c2 = 0 …. (ii)
is given by S1 – S2 = 0 i.e., 2x(g1 – g2) + 2y (f1 – f2) + c1 – c2 = 0.
C1 M C2(2,-4)
=
Note: If the two circles touch each other, then the length of common chord is zero and the common chord is the
common tangent to the two circles at the point of contact.
9 .22 | Circle
PLANCESS CONCEPTS
The length of the common chord AB is maximum when it is the diameter of the smallest circle.
Nitish Jhawar (JEE 2009, AIR 7)
Illustration 26: Find the equation and the length of the common chord of two circles.
2x2 + 2y2 + 7x – 5y + 2 = 0 and x2 + y2 – 4x + 8y – 18 = 0 (JEE MAIN)
Sol: Use the formula for equation of common chord and length of common chord. Equation of common chord of
circle is S1 – S2 = 0 i.e., 2x(g1 – g2) + 2y (f1 – f2) + c1 – c2 = 0 and length of common chord is 2 C2 A2 − C2M2
.
7 5
Given S1 = x + y + x – y + 1 =0
2 2
…. (i)
2 2
S2 = x2 + y2 – 4x + 8y – 18 = 0 …. (ii)
Therefore, the equation of the common chord AB is S1 – S2 = 0
15 21
i.e. x– y + 19 = 0 ⇒ 15x – 21y + 38 = 0 .… (iii)
2 2
30 + 84 + 38 152
The length of the perpendicular from the centre C2(2, –4) to the common chord AB is C2M = =
Radius of the circle S2 = 0 is, C2A = 38 152 + 212 666
Illustration 27: Tangents are drawn to the circle x2 + y2 = 12 at the points where it is met by the circle x2 + y2 – 5x
+ 3y – 2 = 0; find the point of intersection of these tangents. (JEE ADVANCED)
Sol: As we know that, if (x1, y1) is a point of intersection of tangents of circle x2 + y2 = a2 then equation of chord of
contact is xx1 + yy1=a2 and the equation of common chord of two circles are S1 – S2 = 0 i.e., 2x(g1 – g2) + 2y (f1 – f2)
+ c1 – c2 = 0. By using these two formulae we can solve the problem.
Given circles are S1 ≡ x2 + y2 – 12 = 0 … (i)
and S2 ≡ x2 + y2 – 5x + 3y – 2 = 0 … (ii)
The equation of common chord is S1 – S2 = 0 i.e. 5x – 3y – 10 = 0 … (iii)
Let this line meet circle (i) at A and B, and P(α, β) be the point of intersection of the tangents at A and B. Therefore,
the equation of the chord of contact AB is xα + yβ – 12 = 0 … (iv)
As (iii) and (iv) represent the same line, therefore on comparison, we get
α β 6 18 18
= = ∴ α = 6 and β = – . Hence, P ≡ 6, − .
5 −3 5 5 5
O
11. DIAMETER OF CIRCLE P(h,k)
The locus of the middle points of a system of parallel chords of a circle Diameter
x+my=0
is known as the diameter of the circle.
Let the equation of parallel chords be y=mx+c
Figure 9.40
M a them a ti cs | 9.23
y = mx + c (where, c is a parameter).
∴The equation of the diameter bisecting parallel chords
of the circle x2 + y2 = a2 is given by x + my = 0.
Polar
A’ Polar P(x1,y1)
B
Pole B
P(x1,y1) A
Q’ B’
Q(h,k1)
Figure 9.41(a): Polar of a point P outside the circle Figure 9.41(b): Polar of a point P inside the circle
Equation of polar of the circle x2 + y2 + 2gx + 2fy + c = 0 w.r.t. point P (x1, y1) is xx1 + yy1 + g(x + x1) + f(y + y1) + c = 0
i.e. T = 0.
If the circle is x2 + y2 = a2, then its polar w.r.t. (x1, y1) is xx1 + yy1 – a2 = 0 i.e. T = 0.
Pole of a line w.r.t. the circle x2 + y2 = a2
Consider a line lx + my + n = 0 and let (x1, y1) be the pole of the line w.r.t. the circle x2 + y2 = a2.
For the point (x1, y1),
The equation of polar w.r.t. the circle x2 + y2 = a2 is xx1 + yy1 – a2 = 0.
Since lx + my + n = 0 and xx1 + yy1 – a2 = 0 represent the same line.
x1 y1 −a2 a2l a2m
∴ = = ⇒ x1 = – and y1 = – .
l m n n n
a2l a2m
Hence, the pole of the line lx + my +n = 0 is − ,−
n n
Pole of a line w.r.t. the circle x2 + y2 + 2gx + 2fy + c = 0
Consider a line lx + my + n = 0.
If (x1, y1) is the pole, then the equation of polar is xx1 + yy1 + g(x + x1) + f(y + y1) + c = 0.
Now, since lx + my + n = 0 and xx1 + yy1 + g(x + x1) + f(y + y1) + c = 0 represent the same line,
x1 + g y1 + f gx1 + fy1 + c
∴ = =
l m n
x1 + g y1 + f g2 + f 2 − c
On simplification, we get = =
l m l g+ fm − n
x1 + g y1 + f r2
⇒ = = , where r is radius of the circle.
l m l g+ mf − n
9 .24 | Circle
PLANCESS CONCEPTS
Points P (x1, y1) and Q (x2, y2) are conjugate points w.r.t. the circle x2 + y2 + 2gx + 2fy + c = 0 if x1x2 + y1y2
+ g (x1 + x2) + f(y1 + y2) + c = 0.
If P and Q are conjugate points w.r.t. a circle with centre at O and radius ‘a’ then PQ2 = OP2 + OQ2 – 2a2.
Shivam Agarwal (JEE 2009, AIR 27)
Illustration 28: Find the pole of the line 3x + 5y + 17 = 0 with respect to the circle x2 + y2 + 4x + 6y + 9 = 0.
(JEE MAIN)
Sol: If P(α, β) be the pole of line with respect to the given circle. Then the equation of polar of point P(α, β) w.r.t.
the circle is xα + yβ + 2(x + α) + 3(y + β) + 9 = 0. And this equation represent same line which is represented by
equation 3x + 5y + 17 = 0. By solving these two equation simultaneously we will get required pole.
Given circle is x2 + y2 + 4x + 6y + 9 = 0 ... (i)
and, given line is 3x + 5y + 17 = 0 … (ii
⇒ (α + 2) x + (β + 3) y + 2α + 3β + 9 = 0 ... (iii)
Since equation (ii) and (iii) represent the same line,
α+2 β + 3 2α + 3β + 9
∴ = = ⇒ 5α + 10 = 3β + 9
3 5 17
⇒ 5α – 3β = –1 ... (iv)
and, 17α + 34 = 6α + 9β + 27 ⇒ 11α – 9β = –7 ... (v)
From (iv) and (v), we get α = 1, β = 2
Hence, the pole of the line 3x + 5y + 17 = 0 w.r.t. the circle x2 + y2 + 4x + 6y + 9 = 0 is (1, 2).
Illustration 29: A variable circle is drawn to touch the axis of x at origin. Find locus of pole of straight line lx + my
+ n = 0 w.r.t. circle. (JEE ADVANCED)
Sol: As circle touches x-axis at origin therefore let (0, λ) be its centre then equation of circle will be x2 + (y – λ)2 =
λ2. Hence by considering P(x1, y1) be the pole and using polar equation we will get required result.
Let the centre of the circle be (0, λ).
Then the equation of the circle is x2 + (y – λ)2 = λ2
M a them a ti cs | 9.25
⇒ x2 + y2 – 2λy = 0.
Let P(x1, y1) be the pole of the line lx + my + n = 0 w.r.t. the circle,
then, the equation of the polar is xx1 + yy1 – λ(y + y1) = 0 (0,)
xx1 + y (–λ + y1) – λy1 = 0
x −λ + y1 −λy1
∴ On comparison, we =
get 1 = .
l m n
Figure 9.42
Hence, the locus of the pole is ly2 = mxy – xn.
Illustration 30: Prove that if two lines at right angles are conjugate w.r.t. circle then one of them passes through
centre. (JEE ADVANCED)
Sol: Let two perpendicular lines which are conjugate to each other be
ax + by + c = 0 ... (i)
bx – ay + λ = 0 ... (ii)
∴ The equation of the polar of a point (x1, y1) is xx1 + yy1 – r2=0 … (iii)
x1 y1 −r 2
On comparing (i) and (iii), we get = = .
a b c
From the definition of conjugate lines, we know that the point (x1, y1) should satisfy the equation bx – ay + λ = 0,
−br 2a ar 2b
hence + + λ = 0 ⇒ λ = 0.
c c
Therefore, bx - ay + λ = 0 passes through (0, 0).
Direct common
tangent
r1
r2
P C1
C2 T
Transverse common
tangents
Figure 9.43: Common tangents for non-intersecting and non-overlapping circles
In this case four common tangents can be drawn, in which two are direct common tangents and the other two are
transverse common tangents.
9 .26 | Circle
The points P and T , the point of intersection of direct common tangents and transverse common tangents
respectively, always lie on the line joining the centres of the two circles. The point P and T divide the join of C1 and
C2 externally and internally respectively in the ratio r1 : r2.
C1P r1 C1 T r1
i.e. = (externally) and = (internally)
C2P r2 C2 T r2
r x −r x r y −r y r1 x2 + r2 x1 r1 y 2 + r2 y1
∴ P ≡ 1 2 2 1 , 1 2 2 1 and T ≡ , .
r1 − r2 r1 − r2 r1 + r2 r1 + r2
Note: Length of an external (or direct) common tangent, Lext = d2 − (r1 − r2 )2 , and Length of an internal (or
transverse) common tangent, Lint = d2 − (r1 + r2 )2 . where, d is the distance between the centres of the two circles,
and r1, r2 are the radii of the two circles. Therefore, the length of internal common tangent is always less than the
length of the external common tangent.
Case-II: When the distance between the centres is equal to the sum of radii (Circles touching externally)
C1 C2= r1 + r2
Direct common
tangent
r1
r2
P C1
C2 T
Transverse common
tangents
In this case three common tangents can be drawn, two direct common tangents and one transverse common
tangent.
Case III: When the distance between the centres is less than the sum of radii. (Intersecting circles)
r1 − r2 < C1 C2 < r1 + r2
Direct common
Tangent
P C1
C2
In this case two direct common tangents can be drawn as shown in the diagram.
Case IV: When the distance between the centres is equal to the difference of the radii. (Circles touching each other
internally), i.e.C1C2 = r1 –r2.
Tangent at
the point of
contact
r2
P C1
C2
r1
Figure 9.46: Common tangents for circles touching each other internally
r2
C1
C2
r1
Figure 9.47
Illustration 31: Examine if the two circles x2 + y2 – 2x – 4y = 0 and x2 + y2 – 8y – 4 = 0 touch each other externally
or internally. (JEE MAIN)
Sol: When distance between centre of circle is equal to the sum of their radius then they touches eachother
externally and when it is equal to the difference of their radius then circle touches eachother internally.
Let C1 and C2 be the centres and r1 and r2 the radii of S1 ≡x2 + y2 – 2x – 4y = 0 and S1 ≡x2 + y2 – 8y – 4 = 0 respectively.
∴ C1 ≡ (1, 2), C2 ≡ (0, 4), r1 = 5 , r2 = 2 5
(1 − 0 ) + ( 2 − 4 )
2 2
Now, C 1C 2 = = 5 and
Illustration 32: Prove that the two circles x2 + y2 + 2ax + c = 0 and x2 + y2 +2by + c = 0 touch each other,
1 1 1
if + = (JEE ADVANCED)
2
a b 2 c
Sol: Two circles touch each other if distance between centres of these two circles are equal to the sum or difference
of their radius.
Let centres of given circles be C1 and C2 and their radii be r1 and r2 respectively.
9 .28 | Circle
1 1 1
⇒ a2c+ b2c = a2b2 ∴ + =.
a 2 2
b c
Sol: (C) Calculate the distance between the centres and use the different cases of two circles.
Let S1 ≡x2 + y2 + 14x – 4y + 28 = 0 …(i) A2
A1
⇒ C1 = (-7 , 2) and r1 = 5
9
5
and, S2 ≡x + y - 14x + 4y - 28 = 0
2 2
…(ii) P
⇒ C2 = (7 , -2) and r2 = 9
( 7 + 7 ) + ( −2 − 2)
2 2
∴ C1C=
2 > r1 + r2.
Figure 9.48
Hence, four common tangents are possible.
For x – 7 = 0, … (iii)
Clearly, C2 lies on the (iii).
For y + 7 = 0 … (iv)
Length of perpendicular from C1 = 9 > r1.
For 28x + 45y + 371 = 0 … (v)
o
180 -
28( −7) + 45(2) + 371 265
Length of perpendicular from=
C1 = = r1 . r1 r2
282 + 452 53 C1
d C2
Condition of Orthogonality: Two circles are said to be orthogonal to each other if the angle of intersection of the
two circles is 90°.
⇒ 2(g1g2 + f1f2) = c1 + c2 .
PLANCESS CONCEPTS
If two circles are orthogonal, then the polar of a point ‘P’ on first circle w.r.t. the second circle passes
though the point Q which is the other end of the diameter through P. Hence locus of a point which
moves such that its polar w.r.t. the circles S1 = 0, S2 =0 & S3 = 0 are concurrent in a circle which is
orthogonal all the three circles.
Ravi Vooda (JEE 2009, AIR 71)
Illustration 34: If a circle passes through the point (3, 4) and cuts the circle x2 + y2 = a2 orthogonally, the equation
of the locus of its centre is (JEE MAIN)
(A) 3x + 4y – a2 = 0 (B) 6x + 8y = a2 + 25
(C) 6x + 8y + a2 + 25 = 0 (D) 3x + 4y =a2 + 25
Sol : (B)
As we know Two circle are said to be orthogonal if 2(g1g2 + f1f2) = c1+c2. So by considering required equation of
circle as x2 + y2+ 2gx + 2fy + c = 0 and As point (3, 4) satisfies this equation so by solving these two equation we
will get required equation of the locus of its centre.
Let the equation of the circle be x2 + y2+ 2gx + 2fy + c = 0 ... (i)
As the point (3, 4) lies on (i), we have 9 + 16 + 6g +8f + c = 0
⇒ 6g + 8f + c = – 25 ... (ii)
⇒ 2g×0 + 2f×0 = c – a2 ⇒ c = a2 .
∴ From equation (ii), we have 6g + 8f + a2 + 25 = 0.
Hence locus of the centre (–g, –f) is 6x + 8y – (a2 + 25) = 0.
Illustration 35: Obtain the equation of the circle orthogonal to both the circles x2 + y2 +3x – 5y + 56 = 0 and
4x2 + 4y2 – 28x + 29 = 0 and whose centre lies on the line 3x + 4x + 1 = 0. (JEE ADVANCED)
Sol: By considering the required circle to be S ≡ x2 + y2 + 2gx + 2fy + c = 0 and using orthogonality formula
2(g1g2 + f1f2) = c1+c2 we will get a relation between g and f. Also as the centre lies on the line 3x + 4x + 1 = 0 , by
solving these equation we will get required result.
Let the required circle be S ≡ x2 + y2 + 2gx + 2fy + c = 0 … (i)
Given S1 ≡ x + y + 3x – 5y + 56 = 0
2 2
… (ii)
29
and, S2 ≡ x2 + y2 – 7x + = 0. ... (iii)
4
Since (i) is orthogonal to (ii) and (iii)
3 5
∴ 2g + 2f − =c + 6 + 2f ⇒ 3g – 5f = c + 6 ... (iv)
2
2
9 .30 | Circle
7 29 29
and 2g − + 2f.0 ⇒ c+ ⇒ –7g = c + ...(v)
2 4 4
From (iv) and (v), we get 40g –20f = –5 ... (vi)
Given line is 3x + 4y = –1 ... (vii)
(–g, –f) also lies on the line (vii). ⇒ –3g – 4f = –1 ... (viii)
1 29
∴ g =0, f = and c =– [From (vi) and (viii)]
4 4
1 29
∴ The equation of the circle is x2 + y2 + y– = 0 or, 4(x2 + y2) + 2y –29 = 0
2 4
S’=0
S=0
Figure 9.51
(b) The equation of the family of circles passing through the point of intersection of circle S = 0 and a line L = 0
is given by S + λL = 0, (where λ is a parameter)
S+L=0 S=0
L=0
Figure 9.52
(c) The equation of the family of circles touching the circle S =0 and the line L = 0 at their point of contact P is
S + λL = 0, (where λ is a parameter)
S+L=0 S=0
L=0
Figure 9.53
M a them a ti cs | 9.31
(d) The equation of a family of circles passing through two given points P(x1, x1) and Q(x2, x2) can be written in the
form
x y 1
(x –x1) (x – x2) + (y –y1)(y – y2) + λ x1 y1 1 = 0, (where λ is a parameter)
x2 y 2 1
Figure 9.54
In this equation, (x –x1) (x – x2) + (y –y1)(y – y2) = 0 is the equation of the circle with P and Q as the end points
x y 1
of the diameter and x1 y1 1 =0 is the equation of the line through P and Q.
x2 y2 1
(e) The equation of the family of circles touching the circle S ≡ x2 + y2 + 2gx +2fy +c = 0 at point P(x1, y1) is x2 +
y2 + 2gx + 2fy + c + λ {xx1 + yy1 + g(x +x1) + f(y + y1) +c} = 0 or, S + λL = 0, where, L = 0 is the equation of
the tangent to the circle at P(x1, y1) and λ ∈ R.
p(x1,y1)
Figure 9.55
(f) The equation of family of circle, which touch y – y1 = m(x –x1) at (x1, y1) for any finite m is
(x – x1)2 + (y – y1)2 + λ {(y – y1) –m(x – x1)} = 0. And if m is infinite, the family of circle is
(x – x1)2 + (y – y1)2 + λ(x – x1) = 0, (where λ is a parameter)
p(x1,y1)
Figure 9.56
9 .32 | Circle
Note that (x – x1)2 + (y – y1)2 = 0 represents the equation of a point circle with centre at (x1, y1)
(g) Equation of the circles given in diagram is
(x – x1)(x – x2) + (y – y1) (y – y2) ±cot θ {(x – x1)(y – y2) – (x – x2) (y – y1)} = 0
(h) Family of circles circumscribing a triangle whose side are given by L1 = 0; L2 = 0 and L3 = 0 is given by L1 L2 + λL2
L3 + µL3 L1 = 0 provided coefficient of xy = 0 and co-coefficient of x2 = co-efficient of y2.
(x1,y1)
(x2,y2)
Figure 9.57
(i) Equation of circle circumscribing a quadrilateral whose sides in order are represented by the lines L1 = 0, L2 =
0, L3 = 0 & L4 = 0 are L1 L3 + λL2 L4 = 0 where value of λ can be found out by using condition that co-efficient
of x2 = y2 and co-efficient of xy = 0.
Illustration 36: Find the equation of circle through the points A(1, 1) & B(2, 2) and whose radius is 1.(JEE MAIN)
Sol: As we know that, equation of family of circle passing through (x1, y1) and
(x2, y2) is given by (x − x1 )(x − x2 ) + (y − y1 )(y − y 2 ) + λ(x − y) = 0
Equation of AB is x – y = 0
∴ Equation of the family of circle passing through A and B is
(x – 1)(x – 2) + (y – 1) (y – 2) + λ (x – y) = 0 or x2 + y2 + (λ – 3)x – (λ + 3) y + 4 = 0
( λ − 3) ( λ + 3)
2 2
∴ Radius = + −4 .
4 4
( λ − 3) ( λ + 3)
2 2
Illustration 37: Find the equations of circles which touches 2x – y + 3 = 0 and pass through the points of intersection
of the line x + 2y – 1 = 0 and the circle x2 + y2 –2x + 1 = 0. (JEE MAIN)
Sol: Here in this problem the equation of family of circle will be S + λL = 0 by solving this equation we will get centre
and radius of required circle in the form of λ and as this circle touches the line 2x – y + 3 = 0 hence perpendicular
distance from centre of circle to the line is equal to the radius of circle.
Let the equation of the family of circles be S + λL = 0.
∴ x2+ y2 – 2x + 1 + λ (x + 2y – 1) = 0 or, x2 + y2 – x(2 – λ) + 2λy + (1 – λ) = 0 ... (i)
⇒ Centre (–g, –f) is ({2 – λ}/2, – λ)
(2 − λ )
2
1 λ
⇒r= g2 + f 2 −=
c + λ 2 − (1 − =
λ) λ2
5= 5.
4 2 2
M a them a ti cs | 9.33
2. ( 2 − λ ) / 2 − ( −λ ) + 3 λ λ
∴ = 5 or, 5 = ± 5 ⇒ λ = ±2 … (ii)
± 5 2 2
Illustration 38: If P and Q are the points of intersection of the circles x2 + y2 + 3x +7y +2p – 5 = 0
and x2 + y2 +2x + 2y + p2 = 0, then there is a circle passing through P, Q and (1, 1) for (JEE MAIN)
(A) All except two values of p (B) Exactly one value of p
(C) All values of p (D) All except one value of p.
Sol: (D) Here in this problem the equation of family of circle will be S + λL = 0 . and as the circle passes through
(1, 1), we can find the values of P such that λ is any real no. except – 1.
Equation of a circle passing through P and Q is
x2 + y2 + 3x +7y +2p – 5 + λ (x2 + y2 + 2x + 2y – p2) = 0 … (i)
Since (i) also passes through (1 , 1), we get (7 + 2p) – λ (p2 – 6) = 0
7 + 2p
⇒λ= ≠ –1 ⇒ p ≠ –1.
p2 − 6
Illustration 39: C1 and C2 are circles of unit radius with centres at (0, 0) and (1, 0) respectively. C3 is a circle of unit
radius, passes through the centres of the circles C1 and C2 and have its centre above x-axis. Equation of the common
tangent to C1 and C3 which does not pass through C2 is (JEE ADVANCED)
(A) x – 3 y + 2 = 0 (B) 3x–y+2=0
(C) 3 x – y – 2= 0 (D) x + 3 y + 2 = 0
Sol: (B) Equation of any circle passing through any two point (x1, y1)
Y
and (x2, y2) is given by
x y 1
( x – x1 ) ( x – x2 ) + ( y – y1 ) ( y – y 2 ) + λ 0 0 1 = 0
1 0 1
(x – 0) (x – 1) + (y – 0) (y – 0) + λ 0 0 1 = 0
1 0 1
⇒ x + y – x + λy = 0.
2 2
… (i) X’
C1 C2
X
(0,0) (1,0)
If (i) represents C3, its radius = 1
⇒ 1 = (1/4) + (λ2/4) ⇒ λ = - 3 (as λ cannot be +ve)
Y’
Hence, the equation of C3 is x2 + y2 – x – 3y = 0.
Figure 9.58
Since the radius of C1 and C3 are equal, their common tangents will
be parallel to the line joining their centres (0, 0)
1 3
and ,
2 2
9 .34 | Circle
Since the tangent does not pass through C2, the equation of the required common tangent is 3 x – y + 2 = 0.
Illustration 40: Find the equation of circle circumscribing the triangle whose sides are 3x – y – 9 = 0, 5x – 3y– 23 = 0
& x + y – 3 = 0. (JEE ADVANCED)
Sol: Given L1 ≡ 3x – y – 9 = 0 L2 ≡ 5x – 3y – 23 = 0 L3 ≡ x + y – 3 = 0
Family of circles circumscribing a triangle whose side are L1 = 0; L2 = 0 and L3 = 0 is
L1L2 + λL2 L3 + µL3 L1 = 0 provided coefficient of xy = 0 & co-coefficient of x2 = co-efficient of y2 .
∴ L1L2 + λL2L3 + µL1L3 = 0
⇒ (3x – y – 9)(5x – 3y – 23) + λ(5x – 3y – 23)(x + y – 3) + µ(3x – y – 9) (x + y – 3) = 0
⇒ (15x2 + 3y2 – 14xy – 114x + 50y + 207) + λ(5x2 – 3y2 + 2xy – 38x – 14y + 69)
+ µ(3x2 – y2 + 2xy –18x – 6y + 27) = 0
⇒ (5λ + 3µ + 15) x2 + (3 – 3λ – µ) y2 + xy(2λ + 2µ – 14) – x(114 + 38λ + 18µ)
+ y(50 – 14λ – 6µ) + (207 + 69λ +27µ) = 0 ... (i)
The equation (i) represents a circle if
coefficient of x2 = coefficient of y2
⇒ 5λ + 3µ + 15 = 3 – 3λ – µ ⇒ 8λ + 4µ + 12 = 0 ; 2λ + µ + 3 = 0 ... (ii)
and, coefficient of xy = 0
⇒ 2λ + 2µ – 14 = 0 ⇒λ + µ – 7 = 0 ... (iii)
From equation (ii) and (iii), we have λ = –10, µ = 17
Putting these values of λ & µ in equation (i), we get 2x2 + 2y2 – 5x + 11y – 3 = 0
⇒ S1 = S2 ⇒ S1 − S2 = 0
Figure 9.59
Consider two circles given by S1 = 0 and S2 = 0. Then the equation of
the radical axis of the two circle is S1 – S2 = 0
i.e. 2x (g1 – g2) + 2y(f1 – f2) + c1 – c2 = 0,
which is a straight line.
S1
Properties of the radical axis S1-S2=o
S2
(a) For two intersecting circles the radical axis and common chord are identical. Also, the
radical axis and the common tangent are same for two circles touching each other. Figure 9.60
M a them a ti cs | 9.35
(b) The radical axis is perpendicular to the line joining the centres of the two circles.
(c) If two circles cut a third circle orthogonally, the radical axis of the two circles will pass through the centre of
the third circle.
(d) Radical axis does not exist if circles are concentric.
(e) Radical axis does not always pass through the mid-point of the line joining the centre of the two circles.
(f) The radical axis of two circles bisects all common tangents of the two circles.
L
16.2 Radical Centre
s1=0
The point of intersection of the radical axis of three circles, taken in pairs,
is known as their radical centre. s3=0
Let the three circles be
S1 = 0 …..(i), S2 = 0 ...(ii), S3 = 0 ...(iii)
O
Refer to the diagrams shown alongside.
N
Let the straight line OL be the radical axis of the circles S1 = 0 & S3 = 0
and the straight line OM be the radical axis of the circles S1 = 0 & S2 = 0.
The equation of any straight line passing through O is given by (S1 – S2) M
s2=0
+ λ(S3 – S1) = 0 , where λ is any constant.
Figure 9.61
For λ= 1 , this equation become S2 – S3 = 0 , which is, equation of ON.
Clearly, the third radical axis also passes through the point where the
straight lines OL and OM meet. Hence, the point of intersection of the three radical axis, O is the radical centre.
PLANCESS CONCEPTS
Alternate approach to find the equation of the tangent of a circle passing through a point lying on a
given circle.
Consider a point (x1 , y1) on the given circle S1 = 0. Then the equation of a point circle with (x1 , y1) as the
centre is S2 ≡ (x – x1)2 + (y – y1)2= 0. Now we have two circles - one given circle and another point circle.
We now have to find the radical axis of those two circles, which is S1 – S2 = 0.
E.g.: Given a circle x2 + y2 = 8 and the point on circle is (2, 2), we need to find equation of a tangent to
the circle at point (2, 2).
Point circle: (x – 2)2 + (y – 2)2= 0 ⇒ x2 + y2 – 4x – 4y +8= 0
Hence, the radical axis is S1 – S2 = 0. ⇒ x + y = 4, which is also the tangent to the given circle at the
point (2, 2).
Akshat Kharaya (JEE 2009, AIR 235)
9 .36 | Circle
Illustration 41: Find the co-ordinates of the point from which the lengths of the tangents to the following three
circles be equal. 3x2 + 3y2 + 4x – 6y – 1= 0, 2x2 + 2y2 – 3x – 2y – 4 = 0, 2x2 + 2y2 – x + y – 1 = 0. (JEE MAIN)
Sol: Here by using formula S1 – S2 = 0, S2 – S3 = 0, and S3 – S1 = 0 we will get equations of radical axis and solving
these equations we will get required co-ordinate.
Reducing the equation of the circles to the standard form,
4 1
S1 ≡x2 + y2 + x – 2y – = 0
3 3
3
S2 ≡x2 + y2 - x–y–2=0
2
1 1 1
S3 ≡x2 + y2 - x + y – = 0
2 2 2
Hence, the equations of the three radical axis is given by
17 5
L1 ≡ x – y + = 0 … (i)
6 3
3 3
L2 ≡–x – y – = 0, … (ii)
2 2
11 5 1
and, L3 ≡– x+ y+ = 0. … (iii)
6 2 6
16 31
Solving (i) and (ii), we get the point − , , which also satisfies the equation (iii).
21 63
This point is called the radical centre and by definition the length of the tangents from it to the three circles are
equal.
Illustration 42: Find the equation of the circle orthogonal to the three circles x2 + y2 – 2x + 3y – 7 = 0, x2 + y2 +
5x – 5y + 9 = 0 and x2 + y2 + 7x – 9y + 29 = 0 (JEE ADVANCED)
Sol: By using formula of radical axis we will get co- ordinate of radical centre which is also equal to the centre of
required circle.
The given circles are
S1 ≡x2 + y2 – 2x + 3y – 7 = 0 … (i)
S2 ≡x2 + y2 + 5x – 5y + 9 = 0 … (ii)
and S3 ≡x + y + 7x – 9y + 29 = 0
2 2
... (iii)
The radical axis of S1 = 0 and S2 = 0 is 7x – 8y + 16 = 0 ... (iv)
The radical axis of S2 = 0 and S3 = 0 is x – 2y + 10 = 0 ...(v)
∴ The radical centre is (8, 9).
Therefore, the length of the tangent from (8, 9) to each of the given circles is 149 .
∴ The required equation is (x – 8)2 + (y – 9)2 = 149 or x2 + y2 – 16x – 18y – 4 = 0.
Illustration 43: If two circles intersect a third circle orthogonally. Prove that their radical axis passes through the
centre of the third circle. (JEE ADVANCED)
Illustration 44: Prove that the square of the length of tangent that can be drawn from any point on one circle to
another circle is equal to twice the product of the perpendicular distance of the point from the radical axis of the
two circles, and the distance between their centres. (JEE ADVANCED)
Sol: Consider two circle as S1 ≡ x2 + y2 = a2 and S2 ≡ (x – h)2 + y2 = b2 and then by using radical axis formula and
perpendicular distance formula we will prove given problem.
We have to prove that PQ2 = 2 ×PN×C1 C2
Let the equation of the two circles be
S1 ≡ x2 + y2 = a2, and … (i)
S2 ≡ (x – h)2 + y2 = b2 … (ii)
Let P ≡ (a cos θ, a sin θ) be a point on the circle S1 =0 ∴ PQ = 2 2 2
a − b + h − 2ahcos θ
and, Radical axis is {x2 + y2 - a2} – {(x – h)2 + y2 - b2 } = 0
h2 + a2 − b2
⇒ –2 hx + h2 + a2 - b2 = 0 or x=
2h
h2 + a2 − b2 h2 + a2 − b2 − 2ahcos θ
⇒ PN = − acos θ =
2h 2h
h2 + a2 − b2 − 2ahcos θ PQ2
⇒ PN×C1C2 = ×h ⇒ PN × C1C2 =
2h 2
∴ PQ2 = 2 PN×C1C2
S+P=0 S+P=0
S+P=0
S=0
P=0
Figure 9.62
(2) The equation of a co-axial system of circles, when the equation of any two circles of the system are S1 ≡ x2 + y2
+ 2g1x + 2f1y + c1 = 0 and S2 ≡ x2 + y2 + 2g2x + 2f2y + c2 = 0 respectively, is S1 + λ(S1 – S2) = 0
9 .38 | Circle
S2 + (S1 - S2) = 0
S2 + (S1 - S2) = 0
S1 + (S1 - S2) = 0
S1+S1= 0 S2 = 0
S1 = 0
S1 = 0 S2 = 0 S1 - S2 = 0
Figure 9.63
or S2 + λ1(S1 – S2) = 0
Other form
S1 + λS2 = 0, (λ ≠ –1)
PLANCESS CONCEPTS
The equation of a system of co-axial circles in the simplest form is x2 + y2 + 2gx + c = 0 , where g is a
variable and c is a constant. This is the system with center on x-axis and y-axis as common radical axis.
Anvit Tawar (JEE 2009, AIR 9)
Illustration 45: Find the equation of the system of coaxial circles that are tangent at ( )
2, 4 to the locus of point
of intersection of mutually perpendicular tangents to the circle x2 + y2 = 9. (JEE ADVANCED)
Sol: The locus of point of intersection of mutually perpendicular tangents is known as the Director circle. Hence by
using formula of director circle and co-axial system of circle we will get required result.
∴ The equation of the locus of point of intersection of perpendicular tangents is
x2 + y2 = 18 ... (i)
Since, ( )
2, 4 satisfies the equation x2 + y2= 18,
∴ The tangent at ( )
2, 4 to the circle x2 + y2 = 18 is x⋅ 2 + y⋅4 = 18 ... (ii)
Also any two circles of (iii) have the same radical axis 2x + 4y – 18 = 0
Illustration 46: Find equation of circle co-axial with S1=x2+y2+4x+2y+1=0 and S2 = 2x2 + 2y2 – 2x – 4y – 3 = 0 and
centre of circle lies on radical axis of these 2 circles. (JEE MAIN)
Sol: By using S1 – S2 = 0 and S1 + λL = 0 we will get equation of radical axis and equation of co-axial system of
circle respectively.
5
S1 – S2 = 0 ⇒ 5x + + 4y = 0 ⇒ 10x +8y + 5 = 0
2
∴ The equation of the radical axis is 10x + 8y + 5 = 0
The equation of the coaxial system of circles is x2 + y2 + 4x + 2y + 1 +λ(10x + 5 + 8y) = 0
23
⇒ Centre ≡ [ −(2 + 5λ ), − (1 + 4λ )] which lies on radical axis, after substituting we get ⇒ λ = −
82
Illustration 47: For what values of l and m the circles 5(x2 + y2) + ly – m = 0 belongs to the coaxial system
determined by the circles x2 + y2 + 2x + 4y – 6 = 0 and 2(x2 + y2) – x = 0 ? (JEE ADVANCED)
Sol: By using radical axis formula i.e. S1 – S2 = 0 we will get equations of radical axis and by solving them
simultaneously we will get required value of l and m.
Let the circles be S1 ≡ x2 + y2 + 2x + 4y – 6 = 0;
1
S2 ≡ x2 + y2 – x = 0;
2
l m
S3 ≡ x2 + y2 + y– = 0.
5 5
The equation of the radical axis of circles S1 = 0 and S2 = 0 is S1 – S2 = 0,
1
i.e., x2 + y2 + 2x + 4y – 6 – x2 + y 2 − x = 0 or, 5x + 8y – 12 = 0 ... (i)
2
The equation of the radical axis of circles S2 = 0 and S3 = 0 is S2 – S3 = 0,
1 l m
i.e., x2 + y2 – x – x2 + y 2 + y − = 0 or, 5x + 2ly – 2m = 0 ... (ii)
2 5 5
5 8 −12 4 6
On comparing (i) and (ii), = = ⇒ 1= = ∴ l = 4, m = 6.
5 2l −2m l m
PLANCESS CONCEPTS
If origin is a limiting point of the coaxial system containing the circles x2 + y2 + 2gx + 2fy + c = 0
−gc −fc
then the other limiting point is , .
g2 + f 2 g2 + f 2
π
A common tangent drawn to any two circles of a coaxial system subtends an angle of at the limiting
points. 2
Illustration 48: Equation of a circles through the origin and belonging to the co-axial system, of which the limiting
points are (1, 2), (4, 3) is (JEE ADVANCED)
(A) x2 + y2 – 2x + 4y = 0 (B) x2 + y2 – 8x – 6y = 0
(C) 2x2 + 2y2 – x – 7y = 0 (D) x2 + y2 – 6x – 10y = 0
Sol: (C) As we know, if (a, b) and (α , β) be two limiting points of a coaxial system of circles. Then, the equation of
the corresponding point circles are S1 ≡ (x – a)2 + (y – b)2 = 0 and S2 ≡ (x – α)2 + (y – β)2 = 0 so by using the formula
of co-axial system i.e. S1 + λS2 = 0 we will get required result.
Equations of the point circles having (1, 2) and (4,3) as centres is
S1 ≡ (x – 1)2 + (y – 2)2 = 0 ⇒ x2 + y2 – 2x – 4y + 5 = 0
and, S2 ≡ (x – 4)2 + (y – 3)2 = 0 ⇒ x2 + y2 – 8x – 6y + 25 = 0
∴ The co-axial system of circles is S1 + λ S2 = 0.
i.e. x2 + y2 – 2x –4y + 5 + λ(x2 + y2 – 8x – 6y + 25) = 0 … (i)
If (0, 0) lies on the circle given by equation (i), then
02 + 02 – 2(0) – 4(0) + 5 + λ(02 + 02 – 8(0) – 6(0) + 25) = 0
1
⇒ 5 + 25λ = 0 or, λ = – .
5
∴ The equation of the required circle is 5(x2 + y2 – 2x – 4y + 5) – (x2 + y2 – 8x – 6y + 25) = 0
⇒ 4x2 + 4y2 – 2x – 14y = 0 ⇒ 2x2 + 2y2 – x – 7y = 0.
From (i) and (ii), we get the centre of the image circle and the radius is
hence the equation of the image.
(g
2
)
+ f 2 − c (same as the given circle), and
C1 Given
circle
r
Given line
PROBLEM-SOLVING TACTICS
(a) Let S = 0, S’ = 0 be two circles with centers C1, C2 and radii R1, R2 respectively.
(i) If C1C2 > r1 + r2 then each circle lies completely outside the other circle.
(ii) If C1C2 = r1 + r2 then the two circles touch each other externally. (Trick) the point of contact divides C1C2
in the ratio r1 : r2 internally.
(iii) If |r1 – r2| < C1C2 < r1 + r2 then the two circles intersect at two points P and Q.
(iv) If C1C2 = |r1 – r2| then the two circles touch each other internally. (Trick) The point of contact divides C1C2
in the ratio r1 : r2 externally.
(v) If C1C2 < |r1 – r2| then one circle lies completely inside the other circle.
(b) Two intersecting circles are said to cut each other orthogonally if the angle between the circles is a right angle.
Let the circles be S = x2 + y2 + 2gx + 2fy + c = 0, S’ = x2 + y2 + 2g’x + 2f’y + c’ = 0.
And let d be the distance between the centers of two intersecting circles with radii r1, r2. The two cir-
cles will intersect orthogonally if and only if
(i) D2 = and
(ii) 2g g’ + 2f f’ = c + c’.
9 .42 | Circle
FORMULAE SHEET
(ii) r = g2 + f 2 − c
(i) a = b ≠ 0 (ii) h = 0 (iii) ∆ = abc + 2hgf – af2 – bg2 – ch2 ≠ 0 (iv) g2 + f2– c ≥ 0
3. if centre of circle is (h, k) and radius ‘r’ then equation of circle is: (x – h)2 + (y – k)2 = r2
4. The equation of the circle drawn on the straight line joining two given points (x1 , y1) and (x2 , y2) as diameter
is : (x – x1) (x – x2) + (y – y1) (y – y2) = 0
2 2
x + x 2 y1 + y 2 x2 − x1 y 2 − y1
Centre : 1 .r= +
2 , 2
2 2
x=–g+ (g 2
)
+ f 2 − c cos θ and y = –f + (g 2
)
+ f 2 − c sin θ , (0 ≤ θ < 2π)
8. Intercepts length made by the circle On X and Y axes are 2 g2 − c and 2 f 2 − c respectively.
9. Position of point (x1, y1) lies outside, on or inside a circle S ≡ x2 + y2 + 2gx + 2fy + c = 0.
When S1 ≡ x12 + y12 + 2gx + 2fy + c > = < 0 respectively.
10. The power of P(x1, y1) w.r.t. S = x2 + y2 + 2gx + 2fy + c = 0 is equal to PA. PB which is S1 = x12 + y12 + 2gx1 + 2fy2 + c.
PA. PB = PC.PD = PT2 = square of the length of a tangent
M a them a ti cs | 9.43
11. Intercept length cut off from the line y =mx + c by the circle x + y = a is 2 2 2 2
( )
a2 1 + m2 − c2
2
1+m
12. The equation of tangent at (x1, y1) to circle x2 + y2 + 2gx + 2fy + c = 0 is xx1 + yy1 + g(x +x1) + f(y + y1) + c = 0.
ma ±a
and the point of contact of tangent y = mx ± a 1 + m2 is ,
2
1+m 1 + m2
15. The length of the tangent from a point P(x1,y1) to the circle S = x2 + y2 +2gx + 2fy + c = 0 is equal to
16. Pair of tangent from point (0, 0) to the circle are at right angles if g2 + f2 = 2c.
21. The equation of the chord of contact of tangents drawn from a point (x1, y1)
to the circle x2 + y2 = a2 is xx1 + yy2 = a2. And
to the circle x2+ y2 + 2gx + 2fy + c = 0 is xx1 + yy1 + g(x + x1) + f(y + y1) + c = 0.
3
A R
23. The equation of the chord of the circle x2 + y2 + 2gx + 2fy + c = 0. Bisected at the point (x1, y1) is given T = S1.
i.e., xx1 + yy1 + g(x + x1) + f(y + y1) + c = x12 + y12 + 2gx1 + 2fy1 + c.
24. The equation of the common chord of two circles x2 + y2 + 2g1x + 2f1y + c1 = 0 and x2 + y2 + 2g2x + 2f2y + c2 = 0
is equal to 2x(g1 – g2) + 2y(f1 – f2) + c1 – c2 = 0 i.e., S1 – S2 = 0.
2 2
25. Length of the common chord : PQ = 2(PM) = 2 C1P − C1M . Where,
C1P = radius of the circle S1 = 0
C1M =perpendicular length from the centre C1 to the common chord PQ.
9 .44 | Circle
a2l a2m
27. The pole of the line lx + my + n = 0 with respect to the circle x2 + y2 = a2: − ,−
n n
28. P (x1, y1) and Q(x2, y2) are conjugate points of the circle x2 + y 2 + 2gx + 2fy + c =0
When x1 x2 + y1 y 2 + g(x1 + x2 ) + f(y1 + y 2 ) + c =.
0
If P and Q are conjugate points w.r.t. a circle with centre at O and radius r then PQ2 = OP2 + OQ – 2r2.
29. The points P and T are a intersection point of direct common tangents and transverse. Common tangents
respectively, and it divide line joining the centres of the circles externally and internally respectively in the
ratio of their radii.
C1P r1
= (externally)
C2P r2
C1 T r1
= (internally)
C2 T r2
Hence, the ordinates of P and T are.
r x −r x r y −r y r1 x2 + r2 x1 r1 y 2 + r2 y1
P ≡ 1 2 2 1 , 1 2 2 1 and T ≡ ,
r1 − r2 r1 − r2 r1 + r2 r1 + r2
30. If two circles S ≡ x2+ y2 + 2g1x + 2f1y + c1 = 0 and S’ ≡ x2+ y2 + 2g2x + 2f2y + c2 = 0 of r1 , r2 and d be the distance
between their centres then the angle of intersection θ between them is given by
32. S1 – S2 = 0 the equation of the radical axis of the two circle. i.e. 2x (g1 – g2) + 2y (f1 – f2) + c1 – c2 = 0 which is
a straight line.
33. The two limiting points of the given co-axial system are ( c , 0) and ( − c , 0).
34. If two limiting points of a coaxial system of circles is (a, b) and (α, β).
then S1 + λS2 = 0, λ ≠ –1. or, {(x – a)2 + (y – b)2} + λ{(x – α)2 + (y – β)2} = 0, λ ≠ –1 is the
Coaxial system of circle.
35. If origin is a limiting point of the coaxial system containing the circle x2 + y2 +2gx + 2fy + c = 0 then the other
−gc −fc
limiting point is , .
g2 + f 2 g2 + f 2
M a them a ti cs | 9.45
Solved Examples
or x2 + y2 – 2hx – 2ky + k2 = 0 ⇒ 10 h = 50
or 6h – 4k + 3 = k2 …..(ii) 5h + 12k − 10
=3
13
Solve (i) and (ii) : h = 3, k = 3
9 .46 | Circle
∴ Required locus is Sol: Using the concept of family of circle and the
condition for two circles to be orthogonal, we can find
x ( x − x1 ) + y ( y − y1 ) =
0 the equation of the required circle.
As the circle is touching the line x + y = 5. It (-2, 7).
M a them a ti cs | 9.47
Consider the equation of circle as Sol: As polar of point (x1, y1) with respect to the circle x2
x y
( x + 2) + ( y + 7 ) + λ ( x + y − 5) =0
2 2
+ y2 = c2 is same as line + = 1.
a b
⇒ x2 + y 2 + x ( 4 + λ ) + y ( λ − 14 ) + 53 − 5λ = 0 ….(i) On comparing the two equations, we can prove the
given statement.
∴ As the circle given equation (i) is orthogonal to
Let the pole be (x1, y1). Then the polar of (x1,y1) with
x2 + y 2 + 4x − 6y + 9 =0, respect to the circle x2 + y2 = c2 is
We have xx1 + yy1 = c2 … (i)
( 4 + λ ) .2 + ( λ − 14 )( −3=) 53 − 5λ + 9 x y
Now, the line (i) and + = 1 must be the same line.
⇒ 8 + 2λ − 3λ + 42= 62 − 5λ a b
⇒ 4λ =12 x1 y1 c2
∴ comparing coefficients, = =
1/a 1/b 1
⇒λ =3
or ax
= 1 by
= 1 c2 ,
∴ Equation of the circle is x + y + 7x − 11y + 38 =
2 2
0.
∴ ax1 = by1
Example 7: Find the equation of the circle described on ∴ (x1 , y1) always lies on the line ax = by which is a fixed
the common chord of the circles x2 + y2 - 4x – 5 = 0 and line.
x2 + y2 + 8y + 7 = 0 as diameter.
Example 9: Inside the circle x2 + y2 = a2 is inscribed
Sol: Use Geometry to find the centre and the radius of an equilateral triangle with the vertex at (a , 0). The
the required circle. equation of the side opposite to this vertex is
and Radius
= AC2 − AM2
= 9 −5
=2
1
X
M O P(a,o)
Equation of circle is ( x − 1 ) + ( y − 2 ) =
2 2
4.
and hence the equation of BC is As they are drawn from A(–2, 0), conditions are
−2m ± 1 + m2
0=
a
x = – or 2x + a = 0. 1
2 ⇒m=±
3
Example 10 : Find the radical centre of the three circles Equations of tangents become
x2 + y2 = a2, (x – c)2 + y2 = a2 and x2 + (y – b)2 = a2.
T1 : 3y= x + 2
Sol: Here by using the formula
T2 : 3y =−x − 2
S 1 – S2 = 0 , S 2 − S 3 =
0 and S3 − S1 =
0
Circles touching C and having T1 and T2 as tangents
we will get equation of radical axis and by solving them must have their center on x-axis (the angle bisector of
we can obtain requird radical centre. T1 and T2).
Radical axis of first & second circle is given by Let C1 and C2 be the 2 circles and M (h1, 0) & L(h2, 0) be
(x + y ) – (x + y – 2cx + c ) = 0
2 2 2 2 2 their respective centers where
⇒ a2 > 8b2 h2 + 2
Similarly for circle C2, –h2 – 1 =
2
Example 2: Let T1, T2 be two tangents drawn from (–2 , 0) ⇒ –2h2 – 2 = h2 + 2
to the circle C : x2 + y2 = 1. Determine circles touching C
and having T1, T2 as their pair of tangents. Further find (∴ h2 > –2; see figure)
the equation of all possible common tangents to these ⇒ –3h2 = 4
circles, when taken two at time.
4 1
or h2 = – and radius = .
3 3
M a them a ti cs | 9.49
A
r
B
X’ X 4x+3y
O - 42=0
4x − 3y − 24 4x + 3y − 42
=±
5 5
33
B i.e., y = 3 & x=
4
Y’ Since O lies on one of these bisectors and x-coordinate
of O is less then or equal to 8,
r r
r cos θ + − ∴ O lies on y = 3.
2 2,
∴ x1 = Let O be (a,3). Then, OA = CP
3
2
r r 4a − 33
=( a − 2 ) + 25
2
r sin θ − − or
y1 = 2 2 5
3 or 16a2 – 264a + (33)2 = 25{a2 – 4a + 29}
3x1 = r cos θ ; 3y1 = r(sin θ – 2) or 9a2 + 164a – 364 = 0
Eliminating θ, we get or (a – 2)(9a + 182) = 0
2
3x1 3y1
2 182
∴ a =2 or a = −
∴ r + r + 2 =
1 9
and radius = OP.
9 .50 | Circle
Example 5: Coordinates of a diagonal of a rectangle Sol: The condition for one circle to be within the other is
are (0, 0) and (4, 3). Find the equations of the tangents
C1C2 < r1 − r2
to the circumcircle of the rectangle which are parallel to
this diagonal. Without the loss of generality,
λ2 µ2 λµ
⇒2 − c − c < 2. − 2c
4 4 4
λ 2 µ2 λ 2 µ2 2 λ 2 µ2
− c + +c < + c2 −
16 4 4 16
P(4,3)
λµ
2×c×
4
λ 2 µ2 λ µ
c + − 2× × > 0
4 4 2 2
O(0,0)
2
λ µ
c − > 0
2 2
⇒C>0
Also ∴ λ > µ
3 −µ
A line parallel to OP is y = x+c , 0 will be inside
4 2
It is a tangent to the circumscribed circle.
x 2 + y 2 + λx + c = 0
Therefore length of perpendicular from
µ2 µλ
3 ⇒ +0− +c < 0
3 5 4
( 2) − 23 + C 5 4 2
M 2, to it = ⇒ = µ2
+c>0
2 2 9 2 4
1+
16 λ µ µ2
5 5 25 ∴ > +c
or C =± ⋅ =± 2 4
2 4 8
λµ
3 25 ∴ >0
Hence tangents are y = x± 2
4 8
⇒ λµ>0
25
or 3x – 4y ± = 0. Hence, proved.
2
Example 6: The equations two circles are Example 7: A circle touches the line y = x at a point P
x + y + λx + c = 0 and x + y + µx + c = 0. Prove that
2 2 2 2 such that OP = 4 2 where O is the origin. The circle
one of the circles will be within the other if λµ > 0 and contains the point (–10, 2) in its interior and the length
c > 0. of its chord on the line x + y = 0 is 6 2 . Find the
equation of the circle.
M a them a ti cs | 9.51
Sol: In this question, the concept of rotation of axes Example 8: Derive the equation of the circle passing
would be useful. through the centres of the three given circles x2 + y2 –
4y – 5 = 0,
Let the new co-ordinate axis be rotated by an angle of
45° in the clockwise direction. Then x2 + y2 + 12x + 4y +31 = 0 and
=X x cos ( θ) + y sin ( θ ) x2 + y2 + 8x + 10y + 32 = 0.
Y =−x sin ( θ ) + y cos ( θ )
Sol: Find the relation between the centres of the circle
Where θ= 45° and there use the appropriate form of circle.
x−y Let P, Q and R denote the centres of the given circle
∴ X =
2 P ≡ ( 0, 2 ) , Q =( −6, − 2 ) and
x+y
Y =
2
R≡ ( −4, − 5)
The image after rotation would be −2 − 2 −4 2
∴ mPQ = = =
−6 − 0 −6 3
−5 + 2 −3
mQR
= =
−4 + 6 2
2 −3
∴ mPQ . mQR =× −1
=
C1 (0, 4 2) 3 2
⇒ PQ is perpendicular to QR
A
B
∴ Using diameter form, we get
3 2 (x - 0) (x + 4) + (y - 2) (y + 5) =0
C2 (0, -4 2)
0 − 2r Similarly, equation of BQ is
⇒ (=
y − 2r ) ( x − a)
2a − a asin β − 0
(y − v)
=
a cos β − a
( x − a)
−2r
⇒ ( y − 2r
= ) ( x − a)
a β β
2 a.sin
.cos
⇒ ay − 2ar =
−2rx + 2ar 2 2 x−a
=⇒y
β
( )
⇒ 2rx + ay − 4ar =
0 −a × 2 sin
2
∴ QR is a tangent to the circle β
⇒y=− cot ( x − a) …(ii)
2r 2 + ar − 4ar 2
∴ r
=
Now, we eliminate α , β using (i) and (ii)
4r 2 + a2
α − β =2r
r ( 2r − 3a)
⇒ r
= α β
2 2 ⇒ − = r
4r + a 2 2
α β
⇒ ( 2r − 3a) = 4r 2 + a2
2
⇒ tan − =tan r
2 2
⇒ 4r 2 + 9 a2 − 12 ar = 4r 2 + a2
α β
2
tan − tan
⇒8a =
12 ar ⇒ 2 2 = tan r
α β
⇒ 2a2 =
3 ar 1 + tan . tan
2 2
y a−x
⇒ 2a2 =×
3 6 −
x+a y
⇒a =3 ⇒ tan γ
=
y a−x
1+ ×
∴ r 2 (=
= ar 6 ) a+ x y
y 2 − a2 + x2
Example 10: A circle having centre at (0, 0) and radius ⇒ tan γ
=
ay + xy + ay − xy
equal to 'a' meets the x - axis at P and Q. A(α) and B(β)
are points on this circle such that α – β = 2γ, where γ ⇒ x2 + y 2 − 2ay tan γ − a2 =0
is a constant. Then locus of the point of intersection of
PA and QB is a a-r
(A) x – y – 2ay tan γ = a
2 2 2
∴ P ≡ ( −a, 0 ) and Q = ( a, 0 )
∴ Equation of PA is
a sin α − 0
( y − 0)
=
acos α + a
( x + a)
a sin α
=⇒y
a ( cos α + 1 )
( x + a)
α α
.cos a.2 sin
2 2 x+a
=⇒y ( )
2 α
a.2 cos
2
α
⇒ y tan
=
2
x+a ( ) …(i)
M a them a ti cs | 9.53
JEE Main/Boards
Q.3 Find the centre, the radius and the equation of the Q.17 A circle of radius 2 lies in the first quadrant
circle drawn on the line joining A(–1, 2) and B(3, –4) as and touches both the axes of co-ordinates, Find the
diameter. equation of the circle with centre at (6, 5) and touching
the above circle externally.
Q.4 Find the equation of the tangent and the normal to
the circle x2 + y2 = 25 at the point P(–3, –4). 1
Q.18 If mi , ; i = 1, 2, 3, 4 are four distinct point on
mi
Q.5 Show that the tangent to x2 + y2 = 5 at (1, –2) also
a circle, show that m1m2m3m4 = 1.
touches the circle x2 + y2 – 8x + 6y + 20 = 0
Q.6 Find the equation of the tangents to the circle x2 + Q.19 Show that the circle on the chord
y2 – 2x + 8y = 23 drawn from an external point (8, –3). xcosα + ysinα – p = 0 of the circle x2 + y2 = a2 as
diameter is x2 + y2 – a2 – 2p (xcosα + y sin α – p) = 0.
Q.7 Find the equation of the circle whose centre is
(–4, 2) and having the line x – y = 3 as a tangent Q.20 Find the length of the chord of the circle x2 + y2 = 16
which bisects the line joining the points (2, 3) and (1, 2)
Q.8 Find the equation of the circle through the points perpendicularly.
of intersections of two given circles
Q.21 Find the angle that the chord of circle x2 + y2 – 4y=0
x2 + y2 – 8x – 2y + 7 = 0 and along the line x + y = 1 subtends at the circumference
x2 + y2 – 4x + 10y + 8 = 0 and passing through (3, –3). of the larger segment.
Q.9 Find the equation of chord of the circle x2 + y2 – 4x Q.22 Prove that the equation x2 + y2 – 2x – 2λ y – 8=0,
= 0 which is bisected at the point (1, 1). where λ is a parameter, represents a family of circles
passing through two fixed points A and B on the x-axis.
Q.10 Find the equation of chord of contact of the circle Also find the equation of that circle of the family,
x2 + y2 – 4x = 0 with respect to the point (6, 0). the tangents to which at A and B meet on the line
x + 2y + 5 = 0.
Q.11 Find the length of the tangent drawn from the
point (3, 2) to the circle 4x2 + 4y2 + 4x + 16y + 13 = 0. Q.23 Find the area of the quadrilateral formed by
a pair of tangents from the point (4, 5) to the circle
Q.12 Obtain the equations of common tangents of the x2 + y2 – 4x – 2y – 11 = 0 and a pair of its radii.
circles x2 + y2 = 9 and x2 + y2 – 12x + 27 = 0.
Q.24 If the lines a1x + b1y + c1=0 and a2x + b2y + c2=0
Q.13 The centres of the circle passing through the cut the co-ordinate axes in concyclic points, prove that
points (0, 0), (1, 0) and touching the circle x2 + y2 = 9 a1a2 = b1b2.
1
are , ± 2 . Q.25 Show that the length of the tangent from any
2
point on the circle
Q.14 The abscissae of two points A and B are the roots
of the equation x2+ 2ax – b2 = 0 and their ordinates x2 + y2 + 2gx + 2fy + c = 0 to the circle
are the roots of the equation x2 + 2px – q2 = 0. Find x2 + y2 + 2gx + 2fy + c1 = 0 is c1 − c .
the equation and the radius of the circle with AB as
diameter.
9 .54 | Circle
Q.26 Find the point from which the tangents to the (A) Are the vertices of a right triangle
three circles x2 + y2 – 4x + 7 = 0,
(B) The vertices of an isosceles triangle which is not
2x + 2y – 3x + 5y + 9 = 0
2 2
regular
and x2 + y2 + y = 0 are equal in length. Find also this (C) Vertices of a regular triangle
length.
(D) Are collinear
Q.27 The chord of contact of tangents from a point on
the circle x2 + y2 = a2 to the circle x2 + y2 = b2 touches Q.2 2x2 + 2y2 + 2λ x + λ2 = 0 represents a circle for :
the circle x2 + y2 = c2. Show that a, b, c are in G.P. (A) Each real value of λ
Q.28 Obtain the equation of the circle orthogonal to (B) No real value of λ
both the circles (C) Positive λ
x2 + y2 + 3x – 5y + 6 = 0 and (D) Negative λ
4x + 4y – 28x + 29 = 0 and whose centre lies on the
2 2
Exercise 2
Q.7 The equation of the image of the circle x2 + y2 +16x
Single Correct Choice Type – 24y + 183 = 0 by the line mirror 4x + 7y + 13 = 0 is;
(A) x2 + y2 + 32x – 4y + 235 = 0
Q.1 Centres of the three circles
(B) x2 + y2 + 32x + 4y – 235 = 0
x2 + y2 – 4x – 6y – 14 = 0
(C) x2 + y2 + 32x – 4y – 235 = 0
x2 + y2 + 2x + 4y – 5 = 0
(D) x2 + y2 + 32x + 4y + 235 = 0
and x2 + y2 – 10x – 16y + 7 = 0
M a them a ti cs | 9.55
Q.8 The circle described on the line joining the points y= 4 and the x-axis is
(0, 1), (a, b) as diameter cuts the x-axis in points whose y
abscissae are roots of the equation : y=4
(A) x + ax + b = 0
2
(B) x – ax + b = 0
2
x/2
y=
(C) x2 + ax – b = 0 (C) x2 – ax – b = 0 x
O
(C) Form a trapezium (D) Are concyclic (A) (–1, –1) (B) (3, 3) (C) (2, 2) (D) (–2, –2)
Q.21 Locus of all point P (x, y) satisfying x3 + y3 + 3xy = Q.7 The triangle PQR is inscribed in the circle x2 + y2
1 consists of union of = 25. If Q and R have coordinates (3, 4) and (–4, 3)
respectively, then ∠ QPR is equal to (2002)
(A) A line and an isolated point
π π π π
(B) A line pair and an isolated point (A) (B) (C) (D)
2 3 4 6
(C) A line and a circle
Q.8 The number of common tangents to the circles
(D) A circle and an isolated point.
x2 + y2 = 4 and x2 + y2 – 6x – 8y = 24 is (1998)
(A) C1 and C2 touch each other only at one point Statement-I: The tangents are mutually perpendicular.
(B) C1 and C2 touch each other exactly at two points
Statement-II: The locus of the points from which a
(C) C1 and C2 intersect (but do not touch) at exactly two mutually perpendicular tangents can be drawn to the
points given circle is x2 + y2 = 338.
(D) C1 and C2 neither intersect nor touch each other
Q.10 Find the equation of circle touching the line
Q.3 If one of the diameters of the circle x + y – 2x – 6y
2 2 2x + 3y + 1 = 0 at the point (1, –1) and is orthogonal to
+ 6 = 0 is a chord to the circle with centre (2, 1), then the circle which has the line segment having end points
the radius of the circle is (2004) (0, –1) and (–2, 3) as the diameter. (2004)
Q.15 The circle through (1, ﹘2) and touching the axis of
x at (3,0) also passes through the point (2013)
3 3 1 1
(A) (B) (C) (D)
(A) ( 2, −5 ) (B) (5, −2 ) (C) ( −2,5 ) (D) ( −5,2 ) 2 2 2 4
2 2 2 2
(A) x + y − 6y + 7 =0 (B) x + y − 6y − 5 =0 Q.19 The centres of those circles which touch the
2 2
(C) x + y − 6y + 5 =0 2 2
(D) x + y − 6y − 7 =0 circle, x2 + y 2 − 8x − 8y − 4 =
0 , externally and also
touch the x-axis, lie on: (2016)
Q.17 let C be the circle with centre at (1,1) and radius (A) An ellipse which is not a circle
=1. If T is the circle centred at ( 0, y ) , passing through
origin and touching the circle C externally, then radius (B) A hyperbola
of T is equal to (2014)
(C) A parabola
(D) A circle
JEE Advanced/Boards
Q.10 Find the equation of the circle passing through the column-I, the ratio of b/a is
points of intersection of circles x2 + y2 – 4x – 6y – 12 = 0 and
x2 + y2 + 6x + 4y – 12 = 0 and cutting the circle x2 + y2 Column I Column II
– 2x – 4 = 0 orthogonally.
(A) C1 and C2 touch each other
(p) 2 + 2
Q.11 The centre of the circles S = 0 lie on line 2x – 2y
(B) C1 and C2 are orthogonal (q) 3
+ 9 = 0 & S = 0 cuts orthogonally the circle x2 + y2 = 4.
Show that circle S = 0 passes through two fixed points (C) C1 and C2 intersect so that
(r) 2 + 3
& find their coordinates. the common chord is longest
(D) C2 passes through the
Q.12 Find the equation of a circle passing through the (s) 3 + 2 2
centre of C1
origin if the line pair, xy – 3x + 2y – 6 = 0 is orthogonal
to it. If this circle is orthogonal to the circle x2 + y2 – kx (t) 3 – 2 2
+ 2ky – 8 = 0 then find the value of k.
Q.20 A circle with centre in the first quadrant is tangent
Q.13 Find the equation of the circle which cuts the
to y = x + 10, y = x – 6, and the y-axis. Let (h, k) be the
circle x2 + y2 – 14x – 8y + 64 = 0 and the coordinate
axes orthogonally. centre of the circle. If the value of (h + k) = a + b a
where a is a surd, find the value of a + b.
Q.14 Show that the locus of the centres of a circle
which cuts two given circles orthogonally is a straight Q.21 Circles C1 and C2 are externally tangent and they
line & hence deduce the locus of the centres of the are both internally tangent to the circle C3. The radii
circles which cut the circles x2 + y2 + 4x – 6y + 9 = 0 & of C1 and C2 are 4 and 10, respectively and the centres
x2 + y2 – 5x + 4y + 2 = 0 orthogonally. Intercept the locus. of the three circles are collinear. A chord of C3 is also
a common internal tangent of C1 and C2. Given that
Q.15 Find the equation of a circle which touches the m n
line x + y = 5 at the point (–2, 7) and cuts the circle the length of the chord is where m, n and p are
p
x2 + y2 + 4x – 6y + 9 = 0 orthogonally. positive integers, m and p are relatively prime and n is
not divisible by the square of any prime, find the value
Q.16 Find the equation of the circle passing through
of (m + n + p).
the point (–6, 0) if the power of the point (1, 1) w.r.t. the
circle is 5 and it cuts the circle x2 + y2 – 4x – 6y – 3 = 0 Q.22 Find the equation of the circle passing through
orthogonally. the three points (4, 7), (5, 6) and (1, 8). Also find the
coordinates of the point of intersection of the tangents
Q.17 As shown in the figure, the five circles are tangent
to the circle at the points where it is cut by the straight
to one another consecutively and to the lines L1 and
line 5x + y + 17 = 0.
L2. If the radius of the largest circle is 18 and that of
the smaller one is 8, then find the radius of the middle Q.23 The line 2x – 3y + 1 = 0 is tangent to a circle
circle. S = 0 at (1, 1). If the radius of the circle is 13 . Find the
equation of the circle S.
L1
Q.24 Find the equation of the circle which passes
through the point (1, 1) & which touches the circle
x2 + y2 + 4x – 6y – 3 = 0 at the point (2, 3) on it.
L2
Q.7 A straight line with slope 2 and y - intercept 5 Q.14 The locus of the midpoint of a line segment that
touches the circle, x2 + y2 + 16x +12y + c =0 at a point is drawn from a given external point P to a given circle
Q. Then the coordinates of Q are with centre O (where O is origin) and radius r, is
(A) (–6, 11) (B) (–9, –13) (A) A straight line perpendicular to PO
(C) (–10, –15) (D) (–6, –7) (B) A circle with centre P and radius r
9 .60 | Circle
(C) A circle with centre P and radius 2r Q.19 The circles x2 + y2 + 2x + 4y – 20 = 0 and x2 + y2
r + 6x – 8y + 10 = 0
(D) A circle with centre at the midpoint PO and radius
2 (A) Are such that the number of common tangents on
Multiple Correct Choice Type them is 2
(B) Are not orthogonal
Q.15 Locus of the intersection of the two straight lines
passing through (1, 0) and (–1, 0) respectively and (C) Are such that the length of their common tangent
1
including an angle of 45° can be a circle with 12 4
is 5
(A) Centre (1, 0) and radius 2 . 5
(B) Centre (1, 0) and radius 2. (D) Are such that the length of their common chord is
(C) Centre (0, 1) and radius 2. 3
5 .
2
(D) Centre (0, –1) and radius 2.
Q.20 Three distinct lines are drawn in a plane. Suppose
Q.16 Consider the circles there exist exactly n circles in the plane tangent to all
the three lines, then the possible values of n is/are
S1 : x2 + y2 + 2x + 4y + 1 = 0
(A) 0 (B) 1 (C) 2 (D) 4
S2 : x2 + y2 – 4x + 3 = 0
S3 : x2 + y2 + 6y + 5 = 0 Q.21 The equation of a circle C1 is x2+y2+14x–4y + 28=0.
Which of this following statement are correct? The locus of the point of intersection of orthogonal
tangents to C1 is the curve C2 and the locus of the point
(A) Radical centre of S1, S2 and S3 lies in 1st quadrant.
of intersection of perpendicular tangents to C2 is the
(B) Radical centre of S1, S2 and S3 lies in 4st quadrant. curve C3 then the statement (s) which hold good?
(C) Radical centre of S1, S2 and S3 orthogonally is 1. (A) C3 is a circle
(D) Circle orthogonal to S1, S2 and S3 has its x and y (B) Area enclosed by C3 is 100π sq. unit
intercept equal to zero.
(C) Area of C2 is 2 times the area of C1.
(D) C2 and C3 are concentric circles.
Q.17 Consider the circles
C1 : x2 + y2 – 4x + 6y + 8 = 0
Q.22 The circles x2 + y2 – 2x – 4y + 1 = 0 and x2 + y2 +
C2 : x + y – 10x – 6y + 14 = 0
2 2 4x + 4y – 1 = 0
Which of the following statement (s) hold good in (A) Touch internally
respect of C1 and C2?
(B) Touch externally
(A) C1 and C2 are orthogonal.
(C) Have 3x + 4y – 1 = 0 as the common tangent at the
(B) C1 and C2 touch each other. point of contact.
(C) Radical axis between C1 and C2 is also one of their (D) have 3x + 4y + 1 = 0 as the common tangent at the
common tangent. point of contact.
(D) Middle point of the line joining the centres of C1
and C2 lies on their radical axis. Q.23 Which of the following is/are True? The circles x2
+ y2 – 6x – 6y + 9 = 0 and x2 + y2 + 6x + 6y + 9 = 0 are
such that
Q.18 A circle passes through the points (–1 , 1), (0, 6)
and (5, 5). The point (s) on this circle, the tangent (s) (A) They do not intersect.
at which is/are parallel to the straight line joining the
(B) They touch each other.
origin to its centre is/are:
(C) Their direct common tangents are parallel.
(A) (1, –5) (B) (5, 1) (C) (–5, –1) (D) (–1, 5)
(D) Their trannsverse common tangents are
perpendicular.
M a them a ti cs | 9.61
Q.24 Two circles x2 + y2 + px + py – 7 = 0 and x2 + y2 Q.30 Let A(x1, y1), B(x2, y2) and C(x3, x3) are the vertices
– 10x + 2py + 1 = 0 intersect each other orthogonally of a triangle ABC.
then the value of p is
Statement-I : If angel C is obtuse then the quantity
(A) 1 (B) 2 (C) 3 (D) 5 (x3 – x1)(x3 – x2) + (y3 – y1) (y3 –y2) is negative.
Statement-II: Diameter of a circle subtends obtuse
Q.25 Which of the following statements is/are incorrect? angle at any point lying inside the semicircle.
(A) Two circles always have a unique common normal.
(B) Radical axis is always perpendicular bisector to the Q.31 Let C be a circle with centre ‘O’ and HK is the
line joining the centres of two circles. chord of contact of pair of the tangents from point
A. OA intersects the circle C at P and Q and B is the
(C) Radical axis is nearer to the centre of circle of smaller midpoint of HK, then
radius.
Statement-I: AB is the harmonic mean of AP and AQ.
(D) Two circles always have a radical axis.
Statement-II: AK is the Geometric mean of AB and AO
and OA is the arithmetic mean of AP and AQ.
Assertion Reasoning Type
(A) Statement-I is true, statement-II is true and Comprehension Type
statement-II is correct explanation for statement-I.
(B) Statement-I is true, statement-II is true and Paragraph for questions 32 to 34
statement-II is NOT the correct explanation for statement-I. Let A, B, C be three sets of real numbers (x, y) defined as
(C) Statement-I is true, statement-II is false. A : {(x, y): y ≥ 1}
(D) Statement-I is false, statement-II is true. B : {(x, y): x2 + y2 – 4x – 2y – 4 = 0}
Q.26 Consider the lines L : (k + 7)x – (k – 1)y – 4(k – 5)=0 C : {(x, y): x + y = 2}
where k is a parameter and the circle
C : x2 + y2 + 4x + 12y – 60 = 0 Q.32 Number of elements in the A ∩ B∩ C is
Statement-I: Every member of L intersects the circle ‘C’ (A) 0 (B) 1 (C) 2 (D) infinite
at an angle of 90°
Q.33 (x + 1)2 + (y – 1)2 + (x – 5)2 + (y – 1)2 has the value
Statement-II: Every member of L tangent to the circle C.
equal to
Q.27 Statement-I: Angle between the tangents drawn (A) 16 (B) 25 (C) 36 (D) 49
from the point P(13, 6) to the circle S : x2 + y2 – 6x +
8y – 75 = 0 is 90° . Q.34 If the locus of the point of intersection of the pair
of perpendicular tangents to the cirlc B is the curve S
Statement-II: Point P lies on the director circle of S.
then the area enclosed between B and S is
Q.28 Statement-I: From the point (1, 5) as its centre, (A) 6π (B) 8π (C) 9π (D) 18π
only one circle can be drawn touching the circle
x2 + y2 – 2x = 7. Paragraph for questions 35 to 36
Statement-II: Point (1, 5) lies outside the circle Consider a circle x2 + y2 = 4 and a point P(4, 2). θ denotes
x2 + y2 – 2x = 7. the angle enclosed by the tangents from P on the circle
and A, B are the points of contact of the tangents from
Q.29 Statement-I: Let C1 (0, 0) and C2(2, 2) be centres P on the circle.
of two circle and L : x + y –2 = 0 is their common chord.
Q.35 The value of θ lies in the interval
If length of common chord is equal to 2 , then both
circles intersect orthogonally. (A) (0, 15º) (B) (15º, 30º)
Statement-II: Two circles will be orthogonal if their (C) (30º, 45º) (D) (45º, 60º)
centres are mirror images of each other in their common
chord and distance between centres is equal to length
of common chord.
9 .62 | Circle
Q.36 The intercept made by a tangent on the x-axis is Q.4 Let PQ and RS be tangents at the extremities of the
diameter PR of a circle of radius r If PS and RQ intersect
9 10 11 12
(A) (B) (C) (D) at a point X on the circumference of the circle, then 2r
4 4 4 4 equals (2001)
PQ + RS
(A) PQ ⋅ RS (B)
Paragraph for questions 37 to 39 2
Q.37 Length of the chord AB equal Q.5 Let AB be a chord of the circle x2 + y2 = r2 subtending
a right angle at the centre. Then the locus of centroid of
(A) 2 5 (B) 5 (C) 5 2 (C) 10 the triangle PAB as P moves on the circle is (2001)
(A) A parabola (B) A circle
Q.38 The angle subtended by the chord AB in the
minor arc of S is (C) An ellipse (D) A pair of straight lines
3π 5π 2π π
(A) (B) (C) (D) Q.6 If two distinct chords, drawn from the point
4 6 3 4
(p, q) on the circle x2 + y2 = px + qy (where pq ≠ 0) are
bisected by the x-axis, then (1999)
Q.39 Acute angel between the line L and the circle S is
(A) p2 = q2 (B) p2 = 8q2
π π π π
(A) (B) (C) (D)
2 3 4 6 (C) p2 <8 q2 (D) p2 > 8q2
Q.7 Consider
Previous Years’ Questions L1 : 2x + 3y + p – 3 = 0
L2 : 2x + 3y + p + 3 = 0
Q.1 Tangents drawn from the point P (1, 8) to the circle
where p is a real number and
x2 + y2 – 6x – 4y – 11 = 0 touch the circle at the point A
and B. The equation of the circumcircle of the triangle C : x2 + y2 – 6x + 10y + 30 = 0 (2008)
PAB is (2009)
Statement-I: If line L1 is a chord of circle C, then line L2
(A) x2 + y2 + 4x – 6y + 19 = 0 is not always a diameter of circle C. and
(B) x2 + y2 – 4x – 10y + 19 = 0
Statement-II: If line L1 is a diameter of circle C, then line
(C) x2 + y2 – 2x + 6y – 29 = 0 L2 is not a chord of circle C.
(D) x2 + y2 – 6x – 4y + 19 = 0
Paragraph 1: Let ABCD be a square of side length 2
unit. C2 is the circle through vertices A, B, C, D and C1 is
Q.2 Let ABCD be a quadrilateral with area 18, with side
the circle touching all the sides of square ABCD. L is the
AB parallel to the side CD and AB = 2CD. Let AD be
line through A. (2006)
perpendicular to AB and CD. If a circle is drawn inside
the quadrilateral ABCD touching all the sides, its radius
is (2007) Q.8 If P is a point of C1 and Q is a point on C2, then
3 PA2 + PB2 + PC2 + PD2
(A) 3 (B) 2 (C) (D) 1 is equal to
2 QA2 + QB2 + QC2 + QD2
Q.3 The locus of the centre of circle which touches (A) 0.75 (B) 1.25 (C) 1 (D) 0.5
(y – 1)2 + x2 = 1 externally and also touches x axis, is
(2005)
Q.9 A circle touches the line L and the circle C1 externally
(A) {x2 = 4y , y ≥ 0}∪{(0, y), y < 0}
such that both the circle are on the same side of the
(B) x2 = y line, then the locus of centre of the circle is
(C) y = 4x2 (A) Ellipse (B) Hyperbola
(D) y2 = 4x ∪ (0, y), y∈R (C) Parabola (D) Parts of straight line
M a them a ti cs | 9.63
Q.10 A line M through A is drawn parallel to BD. Point S Q.15 Let T1, T2 and be two tangents drawn from
moves such that its distances from the line BD and the (–2, 0) onto the circle C : x2 + y2 = 1. Determine the
vertex A are equal. If locus of S cuts M at T2 and T3 and circles touching C and having T1, T2 as their pair of
AC at T1. then area of ∆T1T2T3 is tangents. Further, find the equations of all possible
1 common tangents to these circles when taken two at a
2
(A) sq unit (B) sq unit time. (1999)
2 3
(C) 1 sq unit (D) 2 sq unit
Q.16 Two parallel chords of a circle of radius 2 are at
a distance 3 + 1 apart. If the chords subtend at the
Paragraph 2: A circle C of radius 1 is inscribed in an
π 2π
equilateral triangle PQR. The points of contact of C with center, angles of and , where k > 0 , then the
the sides PQ, QR, RP are D, E, F respectively. The line k k
value of k is
PQ is given by the equation 3 x + y – 6 = 0 and the [Note : k denotes the largest integer less than or
3 3 3 equal to k ] (2010)
point D is , . Further, it is given that the origin
2 2
and the centre of C are on the same side of the line PQ. Q.17 The circle passing through the point ( −1,0 ) and
(2008) touching the y − axis at ( 0,2 ) also passes through the
point (2011)
Q.11 The equation of circle C is
3 5
(A) (x – 2 3 )2 + (y – 1)2 = 1 (A) − ,0 (B) − ,2
2 2 2
1
(B) (x – 2 3 ) + y + = 1
2
2 3 5
(C) − , (D) ( −4,0 )
(C) (x – 3 ) + (y + 1)2 = 1
2 2 2
(D) (x – 3 )2 + (y – 1)2 = 1
Paragraph 3: A tangent PT is drawn to the circle
Q.12 Point E and F are given by x2 + y 2 =
4 at the point P ( )
3,1 . A straight line
2 2
(A) y = x + 1, y = – x –1 Q.19 A possible equation of L is (2012)
3 3
1 (A) x − 3y =
1 (B) x + 3y =
1
(B) y = x, y = 0
3 (C) x − 3y =
−1 (D) x + 3y =
5
3 3
(C) y = x + 1, y = – x–1 2
2 2 Q.20 Let S be the focus of the parabola y = 8x
and let PQ be the common chord of the circle
(D) y = 3 x, y = 0
x2 + y 2 − 2x − 4y =
0 and the given parabola. The area
of the triangle PQS is (2012)
Q.14 Let 2x2 + y2 – 3xy = 0 be the equation of a pair of
tangents drawn from the origin O to a circle of radius 3 Q.21 The locus of the mid-point of the chord of contact
with centre in the first quadrant. If A is one of the points of tangents drawn from points lying on the straight line
of contact, find the length of OA. (2001) 2 2
4x – 5y = 20 to the circle x + y =9 is (2012)
9 .64 | Circle
PlancEssential Questions
JEE Main/Boards JEE Advanced/Boards
Exercise 1 Exercise 1
Q.12 Q.18 Q.21 Q.5 Q.9 Q.14
Q.23 Q.29 Q.17 Q.19 Q.21 Q.24
Exercise 2 Exercise 2
Q.3 Q.7 Q.14 Q.2 Q.4 Q.9
Q.15 Q.20 Q.13 Q.16 Q.21
Q.22 Q.25 Q.27
Previous Years’ Questions
Q.22 Q.25 Q.26
Q.1 Q.3 Q.5 Q.29 Q.32
Q.8 Q.11 Q.13
Previous Years’ Questions
Q.1 Q.3 Q.6
Q.7 Q.9 Q.13
Answer Key
Q.15 x2 + y2 + 18x – 2y + 32 = 0
Q.1 x2 + y2 + 3x + 12y + 2 = 0
Q.16 32 sq. units
Q.3 (1, –1), 13 , x2 + y2 – 2x + 2y – 11 = 0
Q.17 x2 + y2 – 12x – 10y + 52 = 0
Q.4 3x - 4y = 7, 4x + 3y = 0
Q.20 4 2
Q.6 13x + 9y = 77, 3x – y – 27 = 0
1
Q.21 cos–1
Q.7 2x2 + 2y2 + 16x – 8y – 41 = 0 2 2
Q.8 23x2 + 23y2 – 156x + 38y + 168 = 0 Q.22 x2 + y2 – 2x – 6y – 8 =0
Exercise 2
Single Correct Choice Type
Q.7 C Q.8 B Q.10 2x2 + 2y2 – 10x – 5y + 1 = 0 Q.11 (a,b) and (0,0)
Q.18 B Q.19 C
JEE Advanced/Boards
Exercise 1
Q.1 (i) x2 + y2 – 4x – 3y = 0, (ii) x2 + y2 = 24, (iii) 4x + 3y = 24 Q.2 32
23
Q.3 625 Q.4 4 Q.6 2,
3
1 1 1
Q.7 x2 + y2 – 3x – 3y + 4 = 0 Q.8 (1, 0)& , ; r = Q.9 4x2 + 4y2 + 6x + 10y – 1 = 0
2 2 2 2
1 1
Q.10 x2 + y2 + 16x + 14y – 12 = 0 Q.11 (–4, 4); − , Q.12 x2 + y2 + 4x – 6y = 0; k= 1;
2 2
Exercise 2
Single Correct Choice Type
Comprehension Type
⇒ 3y + 12 = 4x + 12
Solutions
⇒ 4x − 3y =
0
⇒ – 6h + 9 – 4h + 1 = 4h + 4 – 12h + 9 – 2h =3 x – 2y = 5
3 −3 1 5
∴h=– ∴
= C , −6 ∴y = x–
2 2 2
2
∴ Equation of the circle is Equation of C2 is (x – 4)2 + (x + 3)2 = ( 5 )2
⇒ x2 + 3x +
9
+ y2 + 12y + 36 =
81
+16 4 2
4 4 ∴The given line is tangent to C2
x2 + y2 + 3x + 12y + 2 = 0
Sol 6: Equation of circle is
Sol 2: We can see that (0, 0), (1, 1) & (6, –4) form a right C ≡ (x – 1)2 + (y + 4)2 = (2 10 )2
angled triangle with (0, 0) & (6, –4) as diameter
Shifting origin to (1, –4)
Equation of circle is (x – 0) (x – 6) + y(y + 4) = 0
∴ C’ ≡ X2 + Y2 = (2 10 )2 & P = (7, 1)
⇒ C = x(x – 6) + y (y + 4) = 0
∴ Y – 1 = m(X – 7)
We can see that (5, –5) satisfies this equation
∴Y = mX + (1 – 7m)
∴ 4 points are concyclic
∴c2 = a2(1 + m2)
Sol3: A ≡ (–1, 2), and B ≡ (3, –4) (1 − 7m)
2
(
= 40 1 + m2 )
Equation of the circle is (x + 1) (x – 3) + (y – 2) (y + 4) ⇒ 9m – 14m – 39 = 0
2
⇒ x + y – 2x + 2y – 11 = 0
2 2
⇒ 9m2 – 27m + 13m – 39 = 0
∴ C =( −g, − f ) =(1, − 1 ) −13
m = 3 or m =
9
Radius = g2 + f 2 − c = 1 + 1 + 11 = 13
Since slope remains same in both system
∴Equation of lines in old co-ordinates are
Sol 4: Given equation of circle is x2 + y2 = 25 P ≡ ( −3 , − 4 )
−13
0+4 4 (y + 3) = 3(x – 8) &(y + 3) = (x – 8)
∴ Slope of normal=
OP = 9
0+3 3
Or 3x − y − 27 =0 and 13x + 9y =
77
4
∴ Equation of normal is ( y + 4 )= ( x + 3)
3
M a them a ti cs | 9.69
(x 2
) ( )
+ y 2 − 8x − 2y + 7 + λ x2 + y 2 − 4x + 10 y + 8 =
0
-3
3
6
T3
As (3, -3) lies on it
∴ ( 9 + 9 − 24 + 6 + 7 ) + λ ( 9 + 9 − 12 − 30 + 8 ) =0 ∴ The equation of tangents are
⇒ 7 − 16 λ =0 y = 3, x = 3 & y = –3 (from figure itself)
7
⇒λ=
16 Sol 13: Family of circles passing through two points is
∴ Equation of the circle is (x – x1) (x – x2) + (y – y1) (y –y2) + λL = 0
(x2 + y 2 − 8x − 2y + 7 + )
7 2
16
(
x + y 2 − 4x + 10y + 8 = 0 ) ∴x(x – 1) + y2 + ly = 0
∴x2 + y2 – x + ly = 0
= 023 x2 + 23y 2 − 156 x + 38y + 168 =
or 0
1 −λ
Centre = ,
2 2
Sol 9: C ≡ x2 + y2 – 4x = 0 Now since the circle touches internally [∵ (0, 0), & (1, 0)
Centre = (2, 0) lie inside the circle]
1−0 ∴r1 – r2 = distance between their centres
Slope of line perpendicular to chord = = −1
1−2
1 λ2 1 λ2
∴ Slope of chord = 1 ∴3 – + = +
4 4 4 4
⇒ y – 1 = 1 (x – 1)
1 + λ2
∴ y = x is the equation of chord ∴9 = 4
4
Alternative 1
∴λ = ±2 2 ∴Centre = , ± 2
Equation of a chord bisected at a given point is T = S1 2
∴x x1 + y y1 − 2 ( x + x1 ) = x12 + y12 − 4x1 Sol 14: Let the coordinates of diameter be (h1, k1) &
Or, x + y − 2x − 2 = 1 + 1 − 4 (h2, k2)
x2 + ( 2 − x ) =
2
2 As circle with center (6,5) touches it externally
⇒ x2 + 4 − 4x + x2 =2 Y
⇒ 2x2 − 4x + 2 =0 (6,5)
⇒ 2 ( x − 1) =
0
2 ⇒ r2 + 4r + 4 = 16 + 9
⇒x =1 ⇒ r2 + 4r – 21 = 0
∴ AC = BC x2 + y2 + 2gx + 2fy + c = 0
( )
⇒ x2 + y 2 − 2x − 8 − 2λ ( y ) =0 ….(i)
Let S ≡ x2 + y 2 − 2x − 8 =0 and …(iii)
0
L≡ y = …(iii)
∴ The equation is represents a family of circles passing
3 5
And mid-point of AB, M ≡ , through the intersection of S = 0 & L = 0.
2 2
∴ On solving (ii) and (iii), we get
∴ Equation of line L1 is
x2 − 2x − 8 =0
5 3
y − =−1 x − 2 ± 4 + 32
2 2 x
⇒= = 4 or − 2
2
Or, (2y - 5) = - (2x - 3) ∴ The fixed point are A (4, 0) and B (-2, 0) from the
Or 2x + 2y – 8 = 0 diagram, the perpendicular bisector of AB is con.
Current with the tangents at P
Or, x + y – 4 = 0
∴ Length of perpendicular from (0, 0) on L1 is
0+0−4
=2 2
2
( )
2
( a)
2
∴ Length of the chord
= 2 − 2 2
( )
2
= 2 16 − 2 2
=4 2
x2 + y 2 − 2x − 6y − 8 =0 ⇒5x + 5y – 5 = 0 ⇒ x + y – 1 = 0
and S1 – S3 = 0 ⇒ – 4x – y + 7 = 0
Sol 23: Length of tangent = S11 4x + y – 7 = 0
∴ Area of PQRS = 4 × 2 = 8 ⇒ b2 =
ac
∴ a, b and c are in G.P.
Sol 24: The equation of any curve passing through the
intersection of Sol 28: Let the required circle be
L1 ≡ a1x + b1y + c1 = 0 x2 + y2 + 2gx + 2fy + c = 0 … (i)
L2 ≡ a2x + b2y + c2 = 0 The given circles are
L3 ≡ y = 0 & L4 ≡ x = 0 is L1 L2 + λ L3 L 4 x2 + y2 + 3x – 5y + 6 = 0 … (ii)
⇒ (a1x + b1y + c1) (a2x + b2y + c2) + lxy = 0 29
and x2 + y2 – 7x + =0 … (iii)
where λ is a parameter 4
Now 1, 2 & 1, 3 are orthogonal
This curve represents a circle if coeff. of x2 = coeff. of y2
3 −5
∴ a1a2 = b1b2 ∴2g + 2f =c+6
2 2
−7 29
Sol 25: Let any point on c2 be (h, k) 3g – 5f = c + 6 & 2g × + 2f × 0 = c +
2 4
Length of tangent from any point to circle 29
⇒–7g = c +
= S1 4
−5
∴l = h2 + k 2 + 2gh + 2fk + c1 ∴10g – 5f =
4
Now since (h, k) satisfies circle 1 ∴8g – 4f = – 1
∴h + k + 2gh + 2fk = – c
2 2
Equation of circle is
∴l = c1 − c (8g + 1)
x2 + y2 + 2gx + y+c=0
2
Sol 26: The tangents to the these circle are equal in The centre lies on the line
length 3x + 4y + 1 = 0
∴The point is radical centre (8g + 1)
⇒ 3(– g) – 4 +1=0
4
M a them a ti cs | 9.73
⇒ –11g = 0 ∠ OPR
= 90° − ∠ QPR= 45°
1 29
⇒ g = 0, f = and c = – OM
4 4 ∴ In ∆OMP, sin 45° =
∴Equation of circle is OP
4x2 + 4y2 + 2y – 29 = 0 1 a
⇒ =
2 b
Sol 29: Given equation of circle is x 2 + 4x + ( y − 3) =
2
0 ⇒ b =2 a .
0+h 3+k
Let M ≡ (h, k ) ∴B ≡ ,
2 2 Sol 32: According to condition
h 3+k
∴B ≡ , S1 S2
2 2
Y 1
(h, k)
1
2
R 3
(0,3) θ1 + θ2 = 90º
∴tanθ1tanθ2 = 1
B r1 1
X tanθ1 = =
M Length of tangent h2 + k 2 − 1
3
tanθ2 =
As point B lies on the circle
h2 + k 2 − 3
h2 h 3+k
2
According to condition –
∴ + 4× + − 3 =0
4 2 2 ∴ 3 = (h2 + k2 –1) (h2 + k2 – 3)
Sol 30: Let (h, k) be middle points Sol 33: Let the other end of diameter be (h, k)
xh – (x + h) + yk = h2 – 2h + k2 … (i) (x – a) (x – h) + (y – b) (y – k ) = 0
As the chord given by equation (i) passes through (0, 0) a+h b+k
∴ Center ≡ ,
2 2
∴ On substituting, x = 0 and y = 0, we get
Since the circle touches the x-axis
– h = h2 – 2h + k2
∴ | y-coordinate| = radius
∴Locus of midpoint is x2 – x + y2 = 0
2 2
b+k a+h b +k
⇒ = + − (ah + bk)
Sol 31: Given, OM = a and OP = b 2 2 2
From the diagram, 2
a+h
∠ PRQ =
90°
∴ = (ah + bk)
2
And PR = QR
∠ QPR =
∠ PQR =
45°
9 .74 | Circle
∴Locus of point is λ2
Sol 2: (B) S ≡ x2 + y2 + lx + =0
x2 + 2ax + a2 = 4ax + 4by 2
(x – a)2 = 4by λ
2
λ2 λ2
Radius = g2 + f 2 − c = − = −
2 2 4
Sol 34: Let G be perpendicular from C on AB
∴Radius is not defined for any real value of l
And M be midpoint of CD
Let radius = R Sol 3: (A) For an equilateral B
E triangle inscribed in circle of
radius r, in ∆ OAB using cosine a r
rule, we get
o
120
r O
r 2 + r 2 − a2
cos120º = A
C M D F 2r 2 a/2
⇒ – r2 = 2r2 – a2
A
G O
B ⇒a= 3r
Area of equilateral triangle
3 2 3 3 3 2
∴MO2 + MD2 = OD2 (O is centre) = a = × ( 3 r)2 = r
4 4 4
R2
MO2 = R2 –
4 Radius of given circle = g2 + f 2 − c = 1
3R
MO = 3 3
4 ∴A =
4
3R
⇒ CG =
4 Sol 4: (B) Let the centre of circle be (– h, 0)
R R
AG = AO – GO = AO – CM = R – = where h > 0
2 2
3R 2 R2 Radius = 5
AC2= AG2 + GC2 = +
4 4 ∴ Equation of circle is (x + h)2 + y2 = 25
∴ AC = R It passes through the point (2, 3)
AE AB ∴(h + 2)2 = (4)2 ⇒ h = 2 or h = –6
∴ = (As ∆ AEB ∆ ACG )
AC AG
But h > 0 ⇒ h = 2 ⇒ ( 2 + h) + 9 =
2
h 2 or − 6
25 ⇒=
AE AB ∴ Equation of the circle is x2 + y2 + 2x – 21 = 0
⇒ = ⇒ AE = 2AB
R R
∴ Intercept made on y-axis = 2 f 2 − c = 2 21
2
Sol 5: (A) S1 : x2 + y2 = 1
Exercise 2
S2 : x2 + y2 – 2x – 6y = 6
Single Correct Choice Type S3 : x2 + y2 – 4x – 12y = 9
Sol 1: (D) The centers are A = (2, 3) ; B = (–1, –2) ; C = r1 = 1; r2 = 12 + 32 + 6 =4; r3= 22 + 62 + 9 =7
(5, 8)
∴r1, r2, r3 are in A.P.
3 − ( −2) 5
∴ Slope of AB = =
2 − ( −1) 3 Sol 6: (D) S1: x2 + y2 + 2lx + 4 = 0
8−3 5 S2: x2 + y2 – 4lx + 8 = 0
and slope AC = =
5−2 3 Since both represent real circles
∴ The three points are collinear
M a them a ti cs | 9.75
∴r1 ≥ 0 & r2 ≥ 0
∴l2 – 4 ≥ 0 ∴ λ ≤ –2 or λ ≥ 2 ... (i) 1
Let x, be a point on the circle.
∴4l2 – 8 ≥ 0 ∴λ ≤ – 2 or λ ≥ 2 ... (ii) x
=
{
−2 4 × ( −8 ) + 7 × 12 + 13 } ∴R=
12 × 6
=
8 15
2
4 +7 2 3 15 5
⇒ x2 + 8x – (y – 3)2 = 0
Sol 18: (D) Let circle be S ≡ x2+y2+2gx+2fy + c = 0
S1 ≡ x2 + y2 = 1
S2 ≡ x2 + y2 + 2x – 3 = 0
M a them a ti cs | 9.77
A C
∴ Equation of circle passing through (0, 2) and touching
x=0
∴ ∆ ADC ∆ ABC ⇒ (x − 0)2 + (y − 2)2 + λx = 0 …..(i)
AC AD
⇒ = …(i) Also, it passes through (-1, 0)
BC AB
⇒ 1 + 4 − λ= 0 λ ⇒ 5
Also, BC2 = AB2 + AC2 …(ii)
∴ Eq. (i) becomes,
AD2
AC2 = (AB2 + AC2) x2 + y 2 − 4y + 4 + 5x =0
AB2
⇒ x2 + y 2 + 5x − 4y + 4 =
0,
9 .78 | Circle
For x-intercept put y = 0 Sol 4: (A) Given, circle is inscribed in square formed by
2
⇒ x + 5x + 4 = 0 the lines
(x + 1)(x + 4) =
0 x2 − 8x=
+ 12 0 and y 2 − 14y =
+ 45 0
∴ x =−1, −4 ⇒ x 6 and
= = x =2, y 5 and= y 9
Sol 2: (B) For the point of intersection of the two given Which could be plotted as
curves y
C1 : y 2 = 4x
D (2,9) C (6,9)
y=9
and C 2 : x2 + y 2 − 6x + 1 =0
y
(1, 2)
y=5
A (2,5) B (6,5)
x’ x x’ x
O
(0, 0) (3, 0)
y’
Thus, the given curves touch each other at exactly two Sol 5: (C) The line 5x-2y+6=0 meets
point (1, 2) and (1, -2).
The y-axis at the point (0, 3) and therefore the tangent
has to pass through the point (0, 3) and required length
Sol 3: (C) Here radius of smaller circle,
= x12 + y12 + 6x1 + 6y1 − 2
AC= 12 + 32 − 6= 2 Clearly, from the figure the radius
AC– = 02 + 32 + 6(0) + 6(3) − 2
of bigger circle
= 02 + 32 + 6(0) + 6(3) −=
2 25 5
=
2 2 2 2
r = 2 + [(2 − 1) + (1 − 3) ] = 25 5
=
2
r =9 Sol 6: (A) Since, the given circles intersect orthogonally.
⇒r= 3
2 ( g1 g2 + f1 f2 ) =
G + C2
C C1(2,1)
(1,3) Sol 7: (C) Let O is the point at centre and P is the point
2 at circumference. Therefore, angle QOR is double the
angle QPR. So it is sufficient to find the angle QOR.
B
M a them a ti cs | 9.79
4,3)
externally have a radius r and centre at O
x’ R (- O (0,0)
x
C2 C1
O
r 2
2
P r
y’ O1 O
c
Now, slope of OQ, m1 = 4/3, slope of OR, m2 = ‒3/4 r1
Here, m1m2 = −1
Threfore, ∠QOR =
π/2
Which implies that ∠QPR =
π/4
Now, OO2 =
r + r2 and OO1 =
r1 − r
⇒ OO1 + OO2 =+
r1 r2
Sol 8: (B) Given, x2 + y 2 =
4
Centre ≡ C1 ≡ (0,0) and R1 =
2 Which is greater than
Again, x2 + y 2 − 6x − 8y
= − 24 0, then C2 ≡ (3, 4) O1O2 as O1O2 < r1 + r2 (C2 lies inside C1 )
x2 + y 2 =
169 is a director circle having equation (h − a)2 + (k − b)2 =r + r2 and h2 + k 2 =r1 − r
x2 + y 2 =
338
⇒ (h − a)2 + (k − b)2 + h2 + k 2 = r1 + r2
2(2λ − 2) 2(3λ + 2) x2 y 2
∴ ⋅1 + ( −1) = λ + 2 − 3 Or + =1at(x1 , y1 )
2 2 25 4
⇒ 2λ − 2 − 3λ − 2 = λ − 1 xx yy
Then, Eq. (i) and 1 + 1 = 1 are identical
⇒ 2λ = −3 ⇒ λ = −3 / 2 25 4
y1
∴ From Eq. (i) equation of circle, x1 / 25 4 = 1
∴ =
2 2 cos θ sin θ r
2x + 2y − 10x − 5y + 1 =0 25cos θ 4 sin θ
⇒ x1
= ,=
y1
r r
9 .80 | Circle
The line (i) meet the coordinates axes in A(r sec θ,0) π π π π
and β(0,r cosecθ) . Let (h, k) be mid point of AB. θ− +θ+ θ− −θ−
1 3 3 ⋅ cos 3 3 + 2r sin θ
= r 2sin
3 2 2
r sec θ r cosec θ
Then,
= h = and k
2 2
1
r r = [r{2cos θ cos π / 3} + 2r cos θ]
Therefore,
= 2h = and 2k 3
cos θ sin θ
1
25 4 = [r ⋅ cos θ + 2r cos θ=] r cos θ
∴ x=
1 and y=1 3
2h 2k 1 π π
x2 y 2 and q
= [r sin θ − + r sin θ + + 2r sin θ]
As (x1 , y1 ) lies on the ellipse + 1
= 3 3 3
25 4
1 π π
1 625 1 4 = [r{sin θ − + sin θ + } + 2r sin θ]
We get + = 1 3 3 3
25 4h2 4 k 2
1
= [r(2sin θ cos π / 3) + 2r sin θ]
25 1 3
⇒ + =1
4h2 k 2 =
1
[r(sin θ) + 2r sin θ]
or 2
25k + 4h = 2 2 2
4h k 3
= r sin θ
Therefore, required locus is 4x2 + 25y 2 = 4x2 y 2
Now, (p,q) = (r cos θ,r sin θ) lies on x2 + y 2
Sol 13: Let the coordinate of point P be (2r cos θ, 2r sin θ) = r 2 , which is C1
We have, OA = r, OP = 2r
Sol 14: (A) Eq. of circle touching x − a × y at (1,0 ) u
Since, ∆OAP is a right angled triangle. 2 2
( x − 1) + ( y − k ) k2
=
B ⇒ 6k =
10
10
⇒ 2k =
3
cos φ =1 / 2
Sol 15: (B) The eq. of circle touching the
⇒ φ = π/3
∴ Coordinates of A and B are
( )
a − a × u at 3,0 is
2 2
{r cos(θ − π / 3),r sin(θ − π / 3)] and (1 − 3) + ( −2, −k ) =k 2
⇒ 4 + 4 + 4k + k 2 =
k2
π π
r cos θ + , r sin θ + ⇒ 4k =
−8
3 3
⇒ k =−2
If p,q is the centroid of ∆PAB , then 2 2
Circle: ( x − 3) ( −2 − k ) =k 2
1
=p [r cos(θ − π / 3) + r cos(θ + π / 3) + 2r cos θ] Point (5, −2 )
3
2 2
=
1
[r{cos(θ − π / 3) + cos(θ + π / 3)} + 2r cos θ] (5, −3) + ( −2 + 2 ) = 2+2 = 4
3
Only (5, −2 ) lies on circle.
π π π π
θ− +θ+ θ− −θ−
1 3 3 ⋅ cos 3 3 + 2r cos θ
r 2cos
3 2 2
M a them a ti cs | 9.81
x2 y 2
Sol 16: (D) + 1
=
10 9
to is = (± 7 ,0 )
Circle having cente as ( 0,3)
2
x2 + ( y − 3) =γ 2 passes through focus , then
2 2
Sol 19: (C) x + y − 8x − 8y − 4 =
0
(± 7 )
2 2
+ ( 0 − 3) =γ2
2 2
2
(x − 4) + (y − 4) 36
=
7 + 9 =γ
Circles touch each other exotically
⇒ γ 2 =16
2 2
⇒ x2 + ( y − 3) =
16
2 k + 6= (n − 4 ) + (k − 4 )
⇒ x2 + y 2 − 6y − 7 =0 k 2 + 36 + 12k = h2 + 16 − 3h + k 2 − 3k + 16
⇒ h2 − 3h − 9 =20k
Sol 17: (D) If circles C and T touch each other externally
⇒ x2 − 3x − 20y − 4 =
0
then
2 2 If y < 0
1 + y= (1 − 0 ) + (1 − y )
2
⇒ (1 + y ) =1 + 1 − y 2 ( )
⇒ 12 + y 2 + 2y =1 + 1 + y 2 − 2y (4,4) (h,k)
1 K
⇒y=
4
(h,-k)
2 2 2
(1,1) ( −k + 6 ) = (n − 4 ) + (k − 4 )
⇒ h2 − 8h + 4k − 4 =
0
T ⇒ x2 − 8x + 4y − 4 =
0
Locus is Parabola
Exercise 1
2 2
( x + 3) + ( y + 9 ) 64
=
2 2
C1C2 = ( 2 + 3) + (3 + 9 ) Sol 1: The equation of line through origin is y = mx
Let point on circle be (h1, mh1) and (h2, mh2)
= 25 + 144
S = x2 + y2 – 8x – 6y + 24 = 0
= 169 13
=
O = origin
r1 + r2 = 3 + 5 = 13
(i) The equation of chord of S whose mid-point is
⇒ r1 + r2 =
c1 c2 (h, k) is
hx+ ky – 4(x + h) – 3(y + k) + 24
Circles touch each other externally therefrom, three
tangents are possible = h2 + k2 – 8h – 6k + 24
9 .82 | Circle
OT = S(0,0) = 24 y−4
Z=
x−7
∴The locus of P is the circle of radius 24 and centre 4
= origin In this the slope from the point (7, 4); tanq2 =
7
⇒ x2 + y2 = 24 is the locus of P
(7, 4)
2OA.OB OA. OB m
(iii) OP = =
OA + OB OM
∴OP × OM = OA × OB
∴ A and M are harmonic conjugates of P & B
2 1
A P M B
AM AB AM
∴ = ⇒ =2
PM MB PM
∴ P is mid-point of A & M
m − tan θ2
tanθ = tan (θ1 – θ2) =
∴Locus of P: 1 + mtan θ2
x + y – 8x – 6y + 24 – (x + y – 4x – 3y) = 0
2 2 2 2
r m− 4 /7
∴4x + 3y = 24 is locus of P =
tangent 4
1 + m×
7
Sol 2: Radius of given circle= 4 + 2 − C = 6 − C
1 m− 4 /7 7m − 4
⇒ ± = =
r = 2 r1 and r1 = 2 r2, r2 = 2 r3 8 4m 7 + 4m
1+
Sum of radii of all circles 7
3 5
r r r r ∴M = and m =
=r+ + +...............= ⇒ =2 4 12
2 2 1 1
1− 1− 2×3 6 ×5
2 2 ∴2M + 6m = + =4
∴r = 2 – 2 4 12
⇒ 6−C = 2 – 2 ⇒6–C=4+2–4 2
∴C = 4 2 = 32 ⇒ n = 32
M a them a ti cs | 9.83
and radius = 1 x2 + y2 – 8 – 3x – 3y + 12 = 0
∴x = 2 & y =
23
∴ P=
23
2, ∴5 ×
(1 − 2λ ) + λ+3 5
+ =0
3 3 λ+2 λ+2 2
⇒ 10 – 20λ + 2λ + 6 + 5λ + 10 = 0
Sol 7: The locus of point of intersection of mutually
perpendicular tangent is the director circle ⇒ 13λ = 26∴λ = 2
x2 + y2 – 8 + λ (x × 2 + y × 2 – 8) =0 s2 ≡ x2 + y2 + 6x + 4y – 12 = 0
xy – 3x + 2y – 6 = 0 (x + 2)2 + (y –7)2 = 0
b 4 ± 16 − 8 4±2 2
∴ = = =2± 2
a 2 2
C1 But b > a
C2
∴b = 2 + 2
a
C2 ≡ x2 + y2 – 2bx – 2by + b2 = 0 h + k = 2h + 2 = a + b a
4ab = a2 + b2 ∴h = 4 2
For x = +1; | y | = 2 ∴ y ± 2 3 − 2m
Now y = mx is tangent to the circle ∴ = 13
∴ Three possible solutions are possible 1 + m2
4m2 – 12m + 9 = 13m2 + 13 ⇒ 9m2 + 12m + 4 = 0
Alternate method
9m2 + 6m + 6m + 4 = 0
Plotting the graph of |y| = x + 1 and (x – 1)2 + y2 = 4
(3m + 2)2 = 0
2
m=–
3
∴Equation of line is 3y + 2x = 0
-1 1
With (x2, y1) & (x1, y2) as ends of diameter ∠ABC = 90º
Sol 8: (A) Centre of C1 = (2, 0) R1 = 4 & R2 = 4
and ∠ADC = 90º
Centre of C2 = (– 2, 0)
∴ ABCD are concyclic
(5 − 7 + C)2
Sol 9: (A) From (3, 4) chords are drawn to l2 = 2 (3)2 −
2
x2 + y2 – 4x = 0 l1 = l2
Let mid points of chord be (b, h) C2 (C − 2)2
∴4 – =9–
∴h2 + k2 – 4h = xh + yk –2(h + x) 2 2
Now (3, 4) pass through these chords ∴C2 – 4C + 4 – C2 = 10
∴h2 + k2 – 4h = 3h + 4k –2(h + 3) 3
C=–
2
∴Locus of mid-point is x2 + y2 – 5x – 4y + 6 = 0
3
Line is y = x –
2
Sol 10: (B) Let p = (x, y)
2x – 2y – 3 = 0
20 cos θ + 15 20 sin θ + 15
(x, y) = ,
5 5 Sol 14: (D) Equation of circle is x2 + y2 = r2
x −1 y −1 Let P ≡ (a, b)
cos θ = & sinθ =
4 4
Let the midpoint of a point (h, k) on circle &
(x – 1)2 + (y –1)2 = 16 P(a, b) be M(x, y)
This is a circle. a+h b+k
(x, y) = ,
2 2
Sol 11: (B) (3, 4) & (–1, –2) are ends of diameter h = 2x – a ; k = 2y – b
(x – 3)(x + 1) + (y – 4)(y + 2) = 0
(2x –a)2 + (2y –b)2 = r2 is locus of M
x + y – 2x – 2y – 11 = 0
2 2
2 2 2
a b r
x − +y − =
Sol 12: (A) Shortest distance from line to circle 2 2 2
= ⊥ distance – radius
Multiple Correct Choice Type
Centre of circle ≡ (3, –4) & radius = 5
Sol 15: (C, D) Let h, k be the point of intersection
9 − 16 − 25 32
∴ ⊥ distance = =
25 5 k k
∴ Slope of lines is and
h−1 h+1
32 7
∴shortest distance = –5= For point (1, 0) and (–1, 0)
5 5
tan θ − tan θ1
And tan (θ – θ1) =
Sol 13: (A) Slope of the line is 1 1 + tan θ tan θ1
∴ The angle between lines is either 45º or 135º
∴y = x + c
θ – θ1 = 45º or 135º
The two circle are
s1 ≡ x2 + y2 = 4 k k
−
c1 = (0, 0) & R = 2 ± 1 = h−1 h+1
k2
1+ 2
s2 ≡ x2 + y2 – 10x – 14y + 65 = 0 h −1
c2 = (5, 7) & R = 3 2k
±1=
h + k2 − 1
2
9 .90 | Circle
6x + 4y – 2 = 0 −2 −2
L:y–3= (x –2) tanθ =
3 3
3x + 2y - 1 = 0
2x + 3y – 13 = 0
Radical axes of s3 and s2 is
Let points be (h, k)
6y + 4x + 2 = 0
When θ is in 2nd quadrant
3y + 2x + 1 = 0
sinθ > 0 & cosθ < 0
5x + 3y = 0
h = a + r cosθ ; k = a + r sinq
x = 1 y = –1
−3
(1, -1) is the radical centre ∴h = 2 + 13 ×
13
It is a known property that circle which is orthogonal to 2
3 circle has its center equal to radical center & radius k=3+ 13 ×
13
= length of tangent from radical center to any circles.
∴P1 = (–1, 5)
Radices = 1+1+ 2− 4 +1 = 1
When q lies in 4th quadrant
Equation of orthogonal circle is (x –1) + (y +1) = 1
2 2
sinθ < 0 & cosθ > 0
This circle touches both x & y axis.
3
h=2+ × 13
Its x & y-intercept are 1 13
−2
k=3+ × 13
Sol 17: (B, C) c1 ≡ x2 + y2 – 4x + 6y + 8 = 0
13
c2 ≡ x2 + y2 – 10x – 6y + 14 = 0 ∴P2 = (5, 1)
Centre of c1 ≡ (2, –3)
Sol 19: (A, C, D) s1 : x2 + y2 + 2x + 4y – 20 = 0
Centre of c2 ≡ (5, 3)
s2 ≡ x2 + y2 + 6x – 8y + 10 = 0
r1 = 4+9−8 = 5
c1 = (–1, –2) & c2 = (–3, 4)
r2 = 25 + 9 − 14 = 2 5
r1 = 12 + 22 + 20 = 5
c1c2 = r1 + r2
r2 = 32 + 42 − 10 = 15
c1c2 = (5 − 3)2 + (6)2 = 3 5
c1c2 = 22 + 62 = 40 = 2 10
∴ c1 & c2 touch each other
c1c2 = r1+r2
∴ Radical axis is the common tangent and the mid-
point of c1c2 doesn’t lie on radical axis as their radius and c1c2 > | r1 – r2 |
are not the same. ∴ The two circles intersect each other at 2 points
2gg1 + 2ff1 = 2 × 3 + 4x – 4 = – 10 = c + c1
Sol 18: (B, D) A = (–1, 1); B = (0, 6); C = (5, 5)
AB ⊥ BC
∴The circle passing through ABC will have AC as a
diameter
M a them a ti cs | 9.91
C1 r2 = 32 + 32 − 9 = 3
C 1C 2 = 62 + 62 = 6 2
C3 C4
r1 + r2 = 6
C2
They do not intersect with each other
Since their radius are same
When the 3 line is parallel to one of the line then
rd
∴ External direct common tangents are parallel
r
C1
M
(D) Consider two circles having same centre these ABCD is a square since diagonals are equal & ⊥ to
circles donot have a radical axis each other B
r r
Assertion Reasoning Type
L L
A C
1 2 (0, 0) 2 2 (2, 2)
Sol 26: (C) L : k(x − y − 4) + 7x + y + 20 = 0 r r
L are the lines passing through intersection of L1 & L2
D
Point of intersection is (– 2, – 6)
Which is center of circle c
M a them a ti cs | 9.93
When their centres are mirror image of each other AH2 – HB2 = AP × AQ – [AB × AP
1
then the common chord bisects C1C2 and × length + AB × AQ – AB2 – APAQ]
2
1 AB2 = AP × AQ – [AB(AP + AQ) – AB2 – APAQ]
of common chord = c1c2
2 2AP × AQ
∴ AB =
AP + AQ
∴ Statement-I is true
C 1 C2
2 Statement-II: AK2 = AB × AO & AK2 = AP × AQ
C1 C2
C1 C2 (AP + AQ)
2 ∴AB × = AP × AQ
2
tanθ = 1 2AP × AQ
∴AB =
(AP + AQ)
θ = 45º
The circles are orthogonal
Comprehension Type
When the centres are mirror image & length of chord
= distance between centres then the two circles are
Paragraph 1: (32-34)
orthogonal. The inverse is not true
∴ Statement-II is wrong Sol 32: (B) A : {(x, y) : y ≥ 1 }
B : {(x, y) : x2 + y2 – 4x – 2y – 4 = 0}
Sol 30: (A) Let AB = diameter
C : {(x, y) : x + y = 2
A
B
C
P
A B 2
Paragraph 2: (35-36) 1 10
= × =1
2 5/2
Sol 35: (D) Let m be slope of tangents
∴ (y – 2) = m( x – 4) are equation of tangent θ = 45º
π 3π
s = x2 + y2 = 4 Angle at minor arc = π – =
4 4
For tangents c2 = a2 (1 + m2)
∴(2 – 4m)2 = 4(1 + m2) Sol 39: (C)
A
12 m2 – 16m = 0 1
/4
4 m(3m – 4) = 0 90
O
4 4
m = 0 or m = ⇒ tanθ =
3 3
θ ∈ (45º, 60º) Ans.(D) B
π π π
Sol 36: (B) the tangents are q1 = – =
2 4 4
y = 2 & 4x – 3y – 10 = 0 π
10 5 & θ + q1 =
∴ Intercepts made on x axis by 2nd tangent = = 2
4 2 π
θ=
4
Paragraph 3: (37-39)
6 −1 5
= = ∴ (x − 1)(x − 3) + (y − 8)(y − 2) =
0
10 2
⇒ x2 + y 2 − 4x − 10y + 19 =
0
5 5
AB = 2 5− = 2 = 10
2 2
1
Sol 2: (B) 18
= (3α )(2r) ⇒ α=
r 6
2
Sol 38: (A)
y
2
D C (, 2r)
(0, 2r)
B
(r, r)
(0, 1) (h, k)
r lkl
2r O A
O 1
90 l kl
- x’ x
r O B
y’
C 2 x-r D
2x
Distance between O and A is always 1+|k|,
x
r x r ie, (h − 0)2 + (k − 1)2 =1+ | k |,
B A
⇒ h2 + k 2 − 2k + 1
=1 + k 2 + 2 | k |
r
2r ⇒ h2 =+
2 | k | 2k
O
90 ⇒ x2 =
2 | y | +2 y
-
r y, y ≥ 0
where | y |= y, y ≥ 0
where | y |= − y, y < 0
− y, y < 0
C 2 x-r D
2
∴ x2 =2y + 2y, y ≥ 0
2x ∴ x =2y + 2y, y ≥ 0
and x22 = 2y + 2y, y < 0
and x = 2y + 2y, y < 0
Alternate solution 2
⇒ = x2 4y when y ≥ 0
⇒ = x 4y when y ≥ 0
1 and x22 0 when y < 0
=
(x + 2x) × 2r =
18 and x
= 0 when y < 0
2 ∴ {(x,
= y) : x22 4y, when y ≥ 0} ∪ {(0, y) : y < 0}
xr = 6 ...(i) ∴ {(x,
= y) : x 4y, when y ≥ 0} ∪ {(0, y) : y < 0}
x −r
In ∆AOB, tan θ =
r Sol 4: (A) From figure it is clear that ∆PRQ and ∆RSP
and in ∆DOC are similar.
2x − r
tan(90o − θ) =
r
x −r r P
r r
R
∴ = 90-
r 2x − r
⇒ x(2x − 3r) =
0
3r
⇒ x= ....(ii) X
2
-
90
Sol 5: (B) Choosing OA as x-axis, A=(r, 0), B=(0, r) and ⇒ 9p2 − 8p2 − 8q 2 > 0
any point P on the circle is (r cos θ,r sin θ) . If (x, y) is the ⇒ p2 − 8q2 > 0
centroid of ∆PAB , then
y
⇒ p2 > 8q2
(x − 3)2 + (y + 5)2 =9 + 25 − 30
A (p,q)
ie, (x − 3)2 + (y + 5)2 =
22
C (h,0)
x’ x
O Centre = (3, -5)
B
y’
If L1 is diameter, then 2(3) + 3( −5) + p − 3 = 0 ⇒ p = 12
3x r cos θ + r + 0
= ∴ L1 is 2x + 3y + 9 =0
and 3y
= r sin θ + 0 + r L2 is 2x + 3y + 15 =
0
∴ (3x − r)2 + (3y − r)2 =
r2 Distance of centre of circle from L2 equals
A (p,q)
and C2 : (x − 1)2 + (y − 1)2 =
( 2)2
C (h,0)
x’ x C2
O Q
B
D (0,2) C (2,2)
y’ C1 P
and Q(1 + 2 cos θ,1 + 2 sin θ) (Rejecting ‘2’ because origin and centre of C are on the
same side of PQ).
∴ PA2 + PB2 + PC2 + PD2
The point ( 3,1) satisfies Eq. (i).
= {(1 + cos θ)2 + 1 + sin θ)2 } + {(cos θ − 1)2 + (1 + sin θ)2 }
+ {(cos θ − 1)2 + (sin θ − 1)2 } ∴ Equation of circle C is (x − 3)2 + (y − 1)2 =
1.
+ {(1 + cos θ)2 + (sin θ − 1)2 }
Sol 12: (A) Slope of line joining centre of circle to point
= 12 D is
Similarly, QA2 + QB2 + QC2 + QD2 =
16 3
−1
2 1
∑ PA2
12 tan θ
= =
∴ = 0.75
= 3 2 3
∑ QA 16 2 − 3
2
It makes an angle 30o with x-axis.
Sol 9: (C) Let C be the centre of the required circle.
Now, draw a line parallel to L at a distance of r1
(radius of C1 ) from it.
Now, CC1 = AC
⇒ C lies on a parabola.
E D Again, in ∆OCA
3 3
30 tan θ= ,OA
=
OA tan θ
(3,1) 3
=
( −3 + 10 )
3(3 + 10 )
F
∴ =
( −3 + 10 )(3 + 10 )
3(3 + 10 )
tan 45o + tan2θ = = 3(3 + 10 )
⇒ 2
= (10 − 9)
1 − tan 45o × tan2θ
Sol 15:
y y
y = 2x
y=x T1
3 B1
C
3
A 30 A1 L
x’ x
S c1 M O N C2
A2
45 B2
x’
O T2
y’ y’
1 + tan2θ
⇒ = 2 l
1 − tan2θ
C1 C2
(1 + tan2θ) − (1 − tan2θ) 2 −1 1
⇒ = =
(1 + tan2θ) + (1 − tan2θ) (2 + 1) 3
2 tan2θ 1
⇒ =
2 3
1 From figure it is clear that, triangle OLS is a right triangle
⇒ tan2θ = with right angle at L.
3
2 tan θ 1 Also, OL = 1 and OS = 2
⇒ =
1 − tan θ 3
2
1
∴ sin(∠LSO)= ⇒ ∠LSO= 30o
2
⇒ (2 tan θ) ⋅ 3 = 1 − tan2 θ
Since, SA
= SA2 , ∆SA1 A2 is an equilateral triangle.
⇒ tan2 θ + 6 tan θ − 1 = 0 1
⇒ −
8m 1
= 1 + m2 ⇒ (1 )( 2h + 1 ) =−4
15 3
h = −5 / 2
⇒ 64m2 =
25(1 + m2 )
Circle is
⇒ 39m2 =
25
2 2
5 5 2 5
⇒ m= ± x + + ( y − 2) =
39 2 2
Therefore, these tangents are Only ( −4,0 ) satisfies the eq. of circle.
5 4 D is the Answer.
y=
± x +
39 5
2 2
Sol 18: (D) Any tangent to circle x + y =4 and
2 2
Sol 16: Let equation of Circle be x + y =
4 and parallel 2
3x1 + 0 × y1 − 4
=1
x12 + y12
3x1 − 4
=1
y
⇒ 3x1 − 4 =
2
⇒ x1 =
2,2 / 3
2 4 2
P ≡ (1,13) , Q ≡ (1, −13) ⇒ ( x1 , y1 ) ≡ ( 2,0 ) & ,
3 3
( ) (
R ≡ − 3,1 ,S ≡ − 3, −1 )
9 .100 | Circle
45k
Tangents x − 3y =
1 4t − 20 =
h + k2
2
⇒ t 4 + 4t2 − t2 − 4t =
0 ⇒ 36h − 20 n2 + k 2 = (
45k )
(
⇒ t ( t − 1) t + t + 4 =
2
0 ) ( )
⇒ 20 h2 + k 2 − 36h + 45k =
0
⇒t=0,1
( )
⇒ 20 x2 + y 2 − 36x + 45y =
0
(0 − α ) = ( y − k ) = k2
2 2
⇒ ( y − k ) = k 2 − α2
2
P ≡ ( 2, 4 ) Q ≡ ( 0,0 ) S ≡ ( 2,0 )
⇒ y = k ± k 2 − α2
1
∆= × 2 × 4 = 4 sq units
2 = 2 k 2 −=
Intercept α2 2 7
4t − 20 ⇒ k 2 = 7 + α2
Sol 21: (A) Let point P be t,
5 From (i) α =3
⇒ k 2 = 7 + 9 = 16
4t − 20
Eq. of chord of contact xt + y =9 ⇒ k =±4
5
(5t ) x + y ( 4t − 20 ) =
45 … (i) Circle: ( x − 3) + ( y − 4 ) =
2
16
2
( x − 3) + ( y + 4 )
2 2
16
=
M a them a ti cs | 9.101
Sol 23: (D) Let tangent to parabola y2=8x Passes through (2, 8), then
Be ty=x+2t2 8=2m-2m-m3 ⇒ m=-2
( 4 − 2) + ( 4 − 8 )
2 2
(
⇒ 4t 4 =2 1 + t2 ) SP = = 4 + 16 = 2 5 units
⇒ 2t 4 − t2 − 1 =0 Let SQ : QP = 1 : λ
( )(
⇒ 2t2 + 1 t2 − 1 =
0 ) 1
.
λ
S ( 2,8 ) Q (h,k ) P ( 4, 4 )
⇒ t =±1
⇒ S ≡ ( 2, 4 ) & R ≡ ( 2, −4 ) 4 + 2λ 8λ + y
Q (h,k ) ≡ , lies on
1+ λ 1+ λ
⇒ P ≡ ( −1,1) & Q ( −1,1)
Circle, then
1
Area=
2
( 2 + B ) × 3= 15 sq units 4 + 2λ
2
8λ + 4
2
− 2 + − 8 =
4
1+ λ 1+ λ
2 2
Sol 24: (B, C) Let circle be x + y + 2yx + 2y + C =
0 2 2
2 −4
Applying condition for orthogenality ⇒ + 4
=
1+ λ 1+ λ
2gx − 1 + 2f × 0 = C + ( −15)
20
⇒ 4
=
15 and 2g × 0 + 2f × 0 = C − 1
⇒ 2g + c = (1 + λ )
2
⇒C=
1
⇒g=
7 ⇒ 1+ λ = 5
⇒=
λ 5 −1
Also,
1 + 2f + C =
0
⇒ f =−1
SQ
=
QP
( 5 −1 )
Centre ≡ ( −g, −f ) ≡ ( −7,1)
x – intercept of normal at P is 6 slope of tangent at Q
Radius
= 2
g +f −C
= 2
49 + 1 −=
1 7 1
is
2
Hence, B and C are the correct options
3
2x + y = (i) 1 − cos θ
⇒ θ ≡ 1,
sin θ
Eqs. of circle C2 and C3
y cos θ
C2 ≡ x2+(y-y2)2 =12 ⇒ E ≡ 1, , y ≡ (h,k ) (let )
sin θ
C2 ≡ x2+(y-y3)2 =12
1 − Cosθ cos θ
⇒ h=
.
If line (i) touches circle, then Sinθ sin θ
2 ×0 + y −3 1 − Cosθ
=2 3 k=
2 +1 Sinθ
⇒ y −3 =
6
⇒ y −3 =±6 h
1−
⇒y=−3,9
K= h + K2
2
K
⇒ y2 =
−3 and y 3 =
9 1−
h2 + K 2
⇒ Centres Q 2 ≡ ( 0, −3)
Q 3 ≡ ( 0,9 )
⇒ Q 2Q 3 =
12
( )
R 2 ≡ 2 2, −1 and R 3 ≡ −2 2,7 ( )
(4 2)
2
+ (8) =
2
R 2R3= 32 + 64= 96= 16 × 6= 4 6
0 0 1
1
Area of ∆ OR2R3= 2 2 −1 1
2
−2 2 7 0
=
1
2
( 1
7×2 2 −2 2 = ×6×2 2
2
)
= 6 2 sq. units
Now
Area of ∆ PQ2Q3
2 1 1
1
= 0 −3 1
2
0 9 1
1
= 2 ( −3 −=
9 ) 6 2 sq units
2
2017-18 100 &
op kers
Class 11 T
By E ran culty
-JE Fa r
IIT enior emie .
S fP r es
o titut
Ins
MATHEMATICS
FOR JEE MAIN & ADVANCED
SECOND
EDITION
Exhaustive Theory
(Now Revised)
Formula Sheet
9000+ Problems
based on latest JEE pattern
PlancEssential
Questions recommended for revision
10. PA R A B O L A
3D View:
Circle
Figure 10.1
(b) If e = 1, it is a parabola.
(c) If e < 1, it is an ellipse. S
(d) If e = 0, it is a circle. M
(e) If the focus is (α, β) and the directrix is ax + by + c = 0 then the equation of the conic Figure 10.2
(ax + by + c)2
section whose eccentricity =e, is (x – α)2 + (y – β)2 = e2.
a2 + b2
1.3 General Equation of Conic
The general equation of a conic with focus (p, q) and directrix lx + my + n = 0 is:
(l2 + m2) [(x – p)2 + (y – q)2] = e2(lx + my + n)2
This equation, when simplified, can be written in the form ax2 + 2gx + 2hxy + by2 + 2fy + c = 0.
This general equation represents a pair of straight lines if it is degenerate, i.e. ∆ = 0,
a h g
where ∆ = h b f . Further, this equation represents
g f c
(a) A pair of parallel straight lines, if ∆ = 0 and h2 = ab
(b) A pair of perpendicular straight lines, if ∆ = 0 and a + b = 0
(c) A point, if ∆ = 0 and h2 < ab
The general equation given above, represents a conic section if it is non-degenerate, i.e. ∆ ≠ 0 , also this equation
represents
(a) A circle, if ∆ ≠ 0, a = b and h = 0
(b) A parabola, if ∆ ≠ 0 and h2 = ab
(c) An ellipse, if ∆ ≠ 0 and h2 < ab
(d) A hyperbola, if ∆ ≠ 0 and h2 > ab
PLANCESS CONCEPTS
The standard form of a parabola is taken with the origin as its vertex and the focus Focus
lying on the x-axis.
Let F be the focus and ZM be the directrix of the parabola.
Figure 10.3
Let P be a point on the parabola.
Join FP and from P drawn PM ⊥ to ZM. ∴ FP = PM
M a them a ti cs | 10.3
To find the co-ordinates of focus and equation of directrix: From F draw FZ ⊥ on ZM. Bisect FZ in A, i.e. FA =
AZ. Then A lies on the parabola. Let FZ = 2a then AF = AZ = a.
(a,l)
Take A as the origin, AF as the x-axis and AY as the y-axis. L
x+a=0
Then the co-ordinates of F are (a, 0) and the equation of the directrix is x = –a, of x + a = 0. A
(0,0) F(a,0)
Let the co-ordinates of P be (x, y). We know that for a parabola FP = PM.
L’
(x − a)2 + (y − 0)2 = x + a
Note: Focus F is (a, 0); Directrix is x + a = 0; Vertex A is (0, 0); Axis AF of the parabola is y = 0.
Illustration 1: Find the equation of the parabola whose focus is (1, 1) and the tangent at the vertex is x + y = 1.
Also find its latus rectum. (JEE MAIN)
Sol: In order to get the equation of a parabola, we need to find the equation of a directrix. Using the equation of
the tangent at the vertex and the focus we can find the directrix and hence the equation of the parabola.
The directrix is parallel to the tangent at the vertex V.
∴ The directrix will be of the form x + y = λ …(i)
Now, V is the foot of the perpendicular from S(1, 1) to the line x + y = 1.
Let V = (α, β). Then α + β = 1 …(ii)
β −1
and .(–1) = –1, i.e., α = β …(iii)
α −1
1 1 1
Solving (ii), (iii) we get α = β = . So V = ,
2 2 2
Let M = (x1, y1). As MV = VS, V is the middle point of MS. M V S(1,1)
x1 + 1 1 y1 + 1 1
∴ = , =
2 2 2 2 directrix
x+y=1
∴ x1 = 0, y1 = 0. So M = (0, 0)
Figure 10.5
As M is on the directrix, (0, 0) satisfies (i). Hence, λ = 0
∴ the equation of the directrix is x + y = 0. 2
x+y
Using focus-directrix property, the equation of the parabola is (x – 1) + (y – 1) =
2 2
2 2
1 +1
⇒ 2[(x – 1) + (y – 1) ] = (x + y)
2 2 2
⇒ 2(x2 + y2 – 2x – 2y + 2) = x2 + y2 + 2xy
⇒ x2 + y2 – 2xy – 4x – 4y + 4 = 0
⇒ (x – y)2 = 4(x + y – 1)
2 2
1 1 1
Length of latus rectum = 4 × VS = 4 − 1 + − 1 =4. = 2 2
2 2 2
1 0 . 4 | Parabola
Illustration 2: Find the equation of the parabola whose focus is (–1, –2) and equation of the directrix is
x – 2y + 3 = 0. (JEE MAIN)
Illustration 3: Find the equation of the parabola whose focus is the point (4, 0) and whose directrix is x = –4. Also,
find the length of the latus rectum. (JEE MAIN)
x = -4
M P
FP (x,y)
= 1; FP = PM ⇒ FP = (PM)
2 2
PM Z X
2 (0,0)
x+4 F(4,0)
(x – 4)2 + (y – 0)2 =
1 Z’
⇒ x2 – 8x + 16 + y2 = x2 + 8x + 16
Figure 10.6
⇒ y2 = 16x
Length of latus rectum = coefficient of x = 16.
Y’
2
y = 4ax
Figure 10.7
Left handed parabola: The equation of this type of parabola is of the Y M
form y2 = –4ax, a > 0. See Fig. 10.8
A
X’ X
F Z
Y’
2
y = -4ax
Figure 10.8
M a them a ti cs | 10.5
Y’
2
x = 4ay
Figure 10.9
A
X’ X
Y’
2
x = -4ay
Figure 10.10
PLANCESS CONCEPTS
Illustration 4: Find the vertex, the axis, the focus, the directrix, and latus rectum of the parabola, 4y2 + 12x – 20y + 67 = 0.
(JEE MAIN)
Sol: Represent the given equation in the standard form and then compare it with the standard forms to get vertex,
axis, focus, directrix and latus rectum to get the answers.
1 0 . 6 | Parabola
Vertex: The coordinates of the vertex of Y2 = –3X are (X = 0, Y = 0) So, the coordinates of the vertex before
7 5
transformation are − , .
2 2
Axis: The equation of the axis of the parabola Y2 = –3X is Y = 0. So, the equation of the axis before transformation
is y = 5/2.
17 5
Focus: The coordinates of the focus are (X = –3/4, Y = 0). So, the coordinates of the focus are − , .
4 2
Directrix: The equation of the directrix is X = +a = +3/4. So, the equation of the directrix before transformation is
x = –11/4
Latus rectum: The length of the latus rectum of the given parabola is 4a = 3.
Illustration 5: Prove that 9x2 – 24xy + 16y2 – 20x – 15y – 60 = 0 represents a parabola. Also find its focus and
directrix. (JEE MAIN)
Sol: A general equation of a conic represents a parabola if ∆ ≠ 0 and h2 = ab. In order to get the focus and the
directrix, convert the given equation into the standard form and compare with the standard form.
9 −12 −10
−15
Here h2 – ab = (–12)2 – 9(16) = 144 – 144 = 0. Also, ∆ = −12 16 ≠0
2
−15
−10 −60
2
∴ The given equation represents a parabola. Now, the equation is (3x – 4y)2 = 5(4x + 3y + 12).
Clearly, the lines 3x – 4y = 0 and 4x + 3y + 12 = 0 are perpendicular to each other. So, let
3x − 4y 4x + 3y + 12
= Y, = X … (i)
32 + ( −4)2 42 + 32
1
The equation of the parabola becomes Y2 = X = 4. X
4
1
∴ Here a = in the standard equation.
4
1
∴ The focus = (a, 0)X, Y = , 0
4 X, Y
1
If, X = , Y = 0, then from the equations of transformation in (i), we get
4
3x − 4y 4x + 3y + 12 1 −43
= 0, = ⇒ 3x – 4y = 0, 4x + 3y =
5 5 4 4
M a them a ti cs | 10.7
3x 1 43 3x 1 43
⇒ y= , y = − − 4x ∴ = − − 4x
4 3 4 4 3 4
−43 3 3 −43 −129
⇒ 9x = –43 – 16x ; ∴ x = and y = x = . =
25 4 4 25 100
−43 −129
∴ Focus = ,
25 100
1
The equation of the directrix is X + a = 0, i.e., X + =0
4
4x + 3y + 12 1 5 53
or + = 0 or 4x + 3y = – – 12 = –
5 4 4 4
53
∴ The equation of the directrix is 4x + 3y + = 0.
4
Illustration 6: Find the equation of the parabola whose latus rectum is 4 units, the axis is the line
3x + 4y – 4 = 0 and the tangent at the vertex is the line 4x – 3y + 7 = 0. (JEE ADVANCED)
Sol: The square of the distance of a point from the directrix is equal to the product of latus rectum and the distance
of the point from the axis.
Let P(x, y) be any point on the parabola and let PM and PN be perpendiculars from P to the axis and to the tangent
at the vertex respectively. Then
2
3x + 4y − 4 4x − 3y + 7
PM = (Latus rectum) (PN)
2
⇒ = 4 ⇒ (3x + 4y – 4)2 = 20(4x – 3y + 7)
2 2 2 2
3 +4 4 + ( −3)
This is the equation of the required parabola.
Note: In the above examples, we have learnt how to find the vertex, the focus, the axis, the directrix etc. of
parabolas reducible to one of the various forms given. If the equation of a parabola is quadratic in both x and y,
then to find its vertex, focus, axis, etc., we follow the following algorithm.
Step I: Obtain the equation of the parabola and express it in the form (ax + by + c)2 = (Constant) (bx – ay + c′)
It should be noted here that ax + by + c = 0 and bx – ay + c′ = 0 represent perpendicular lines.
2
2 2
ax + by + c bx − ay + c′
Step II: Divide both sides by a + b to obtain = (Constant)
2 2 2 2
a +b a +b
ax + by + c bx − ay + c′
Step III: Now substitute = Y and = X in step II to obtain Y2 = (Constant) X.
2 2
a +b a2 + b2
Step IV: Compare the equation obtained in step III with Y2 = 4ax to obtain various elements like vertex, focus, axis,
etc., and use the transformation in step III to obtain the corresponding elements of the given parabola.
Illustration 7: Find the equation of the parabola whose axis is parallel to the y-axis and which passes through the
points (0, 4), (1, 9) and (–2, 6). Also find its latus rectum. (JEE ADVANCED)
Sol: Consider a standard equation of a parabola with the vertex at (α , β) such that the axis is parallel to the Y-axis.
Substitute the points given and find the. (see Fig. 10.11) equation
As the axis is parallel to the y-axis, it will be of the form x – α = 0 and the tangent to the vertex (which is
perpendicular to the axis) will be of the form y – β = 0 for some β.
Hence, the equation of the parabola will be of the form
(x – α)2 = 4a(y – β) … (i)
where α, β, a are the unknown constants and 4a being the latus rectum.
1 0 . 8 | Parabola
Illustration 8: Find the set of values of α in the interval [π/2, 3π/2] for which the point (sin α, cosα) does not lie
outside the parabola 2y2 + x – 2 = 0. (JEE MAIN)
Sol. Use the concept of the Position of a point w.r.t. a parabola. If the point (sin α, cos α) lies inside or on the
parabola 2y2 + x – 2 = 0.
2cos2α + sin α – 2 ≤ 0 ⇒ 2 – 2 sin2α + sin α – 2 ≤ 0
⇒ sin α(2 sinα – 1) ≥ 0 ⇒ sin α ≤ 0 or, 2 sin α – 1 ≥ 0
⇒ α ∈ [π, 3π/2] or, α ∈ [π/2, 5π/6] ⇒ α ∈ [π/2, 5π/6] ∪ [π, 3π/2]
M a them a ti cs | 10.9
6. CHORD
Note: (a) If the chord joining points t1 and t2 on the parabola y2 = 4ax passes through the focus then t1 t2 = –1.
(b) If one end of a focal chord of the parabola y2 = 4ax is P(at2, 2at), then the coordinates of the other end is
a −2a
Q , .
t2 t
(a) Let P(at2, 2at) be one end of a focal chord PQ of the parabola y2 = 4ax. Then, the length of the focal chord
with ends as P and Q is a(t + 1/t)2
2
1 1
(b) We know that, t + ≥ 2 for all t ≠ 0. ∴ a t + ≥ 4a ⇒ PQ ≥ 4a.
t t
Thus, the length of the smallest focal chord of the parabola is 4a which is the length of its latus rectum.For
this reason, the latus rectum of a parabola is the smallest focal chord.
(c) he semi-latus rectum of the parabola y2 = 4ax is the harmonic mean between the segments of any focal
T
chord of the parabola.
(d) The circle described on any focal chord of a parabola as a diameter of that circle, also touches the directrix.
(e) he line y = mx + c meets the parabola y2 = 4ax at two points that can be real, coincident or imaginary
T
according to a >,=,< cm ⇒ condition of tangency is, c = a/m.
4
(f) Length of the chord intercepted by the parabola on the line y = mx + c is: a(1 + m2 )(a − mc)
2
m
PLANCESS CONCEPTS
The length of the chord joining two points t1 and t2 on the parabola y2 = 4ax is
Illustration 9: A quadrilateral ABCD is inscribed in y2 = 4ax and 3 of its sides AB, BC, CD pass through fixed points
(α, 0), (β, 0) and (γ, 0), then show that the 4th side also passes through a fixed point. Also find this fixed point.
Sol: Consider four parametric coordinates and form equations according to the given conditions.
Let t1, t2, t3 and t4 be the parametric coordinates of A, B, C and D respectively. C (t3)
Equation of AB is y(t1 + t2) = 2x + 2at1t2. Given that this line passes through (α, 0). (t4)
α D
⇒ t1t2 = −
a α γ
t1 t2 t3 t 4 − . −αγ
β γ
Similarly, t2t3 = − , t3t4 = − Accordingly, t1t4 = = a a = A
a a t t
2 3 β βa (t1)
a B (t2)
αγ
From the above result we can say that AD always passes through a point , 0 Figure 10.12
β
1 0 . 1 0 | Parabola
Illustration 10: Find the relation between the line y = x + 1 and the parabola y2 = 4x. (JEE MAIN)
Sol: Solve the two given equations and based on the intersection we can find the relation between the two.
Solving the line and parabola, we get (x + 1)2 = 4x ⇒ (x – 1)2 = 0. Therefore y = x + 1 is a tangent to the parabola.
Illustration 11: Through the vertex O of a parabola y2 = 4x, the chords OP and OQ are drawn at right angles to one
another. Show that for all positions of P, PQ intersects the axis of the parabola at a fixed point. Also find the locus
of the middle point of PQ. (JEE MAIN)
Sol: Use parametric coordinates for P (t1) and Q (t2) to find the relation between them and obtain the equation of
chord PQ. Let the coordinates of P and Q be (t12, 2t1) and (t22, 2t2) respectively. Then,
2 2
m1 = Slope of OP = ; m2 = Slope of OQ =
t1 t2
Since OP is perpendicular to OQ. Therefore, m1m2 = –1
2 2
⇒ × = –1 ⇒ t1t2 = –4
t1 t2
The equation of chord PQ is y(t1 + t2) = 2x + 2t1t2
⇒ y(t1 + t2) = 2x – 8
Clearly, it passes through (4, 0) for all values of t1 and t2. Thus, PQ cuts x-axis at a fixed point (4, 0) for all position
of point P.
Let R(h, k) be the mid-point of PQ. Then, 2h = t12 + t22 and k = t1 + t2 ... (ii)
∴ (t1 + t2)2 = t12 + t22 + 2t1t2 ⇒ k2 = 2h – 8 [Using (i) and (ii)]
Hence, the locus is y = 2x – 8.
2
(This is also a parabola)
Illustration 12: Find the locus of the centre of the circle described on any focal chord of a parabola y2 = 4ax as
diameter. (JEE ADVANCED)
Illustration 13: A triangle ABC of area ∆ is inscribed in the parabola y2 = 4ax such that the vertex A lies at the vertex
of the parabola and side BC is a focal chord. Prove that the difference of the distances of B and C from the axis of
the parabola is 2∆/a. (JEE ADVANCED)
Sol: Use the parametric form for the points B and C and proceed according to the conditions given in the question.
Let the coordinates of B and C be (at12, 2at1) and (at22, 2at2) respectively.
Since BC is a focal chord of the parabola y2 = 4ax. Therefore,
Since, ∆ = Area of ∆ABC
M a them a ti cs | 10.11
0 0 1
1 2
⇒ ∆ = at1 2at1 1 ⇒ ∆ = |a2t1t2(t1 – t2)| ⇒ ∆ =|–a2(t1 – t2)| [ t1t2 = –1]
2 2
at2 2at2 1
⇒ ∆ = a2|t2 – t1|
We have, BL = 2at1 and CM = 2at2 ∴ |BL – CM| = |2at1 – 2at2|
2a|t1 – t2| = 2a × ∆/a2 = 2∆/a
Illustration 14: Let PQ be a variable focal chord of the parabola y2 = 4ax whose vertex is A. Prove that the locus of
the centroid of ∆APQ is a parabola whose latus rectum is 4a/3. (JEE ADVANCED)
Sol: Take two general points on the focal chord of the parabola and use the formula for a centroid to obtain a
relation between the ordinate and the abscissae of the centroid.
Let the coordinates of P and Q be (at12, 2at1) and (at22, 2at2) respectively. Since PQ is a focal chord.
Therefore t1t2 = –1 … (i)
Let (h, k) be the coordinates of the centroid of ∆APQ. Then,
at12 + at22 2at1 + 2at2 3h
h= and k = ⇒ = t12 + t22 and 3k/2a = t1 + t2 … (ii)
3 3 a
2
3k 3h 9k 2 3h − 2a
Now (t1 + t2)2 = t12 + t22 + 2t1t2 ⇒ = – 2 ⇒ =
2a
a 4a 2 a
4a 4a 2a
⇒ k2 = (3h – 2a) ⇒ k2 = h −
9 3 3
4a 2a
Hence, the locus of (h, k) is y2 = x −
3 3
4a
Clearly, it represents a parabola whose latus rectum is
3
Illustration 15: A variable chord through the focus of the parabola y2 = 4ax intersects the curve at P and Q. The
straight line joining P to the vertex cuts the line joining Q to the point (–a, 0) at R. Show that the locus of R is y2 +
8x2 + 4ax = 0. (JEE ADVANCED)
Sol: Consider two points on the parabola and obtain the equation of the straight line passing through P and Q.
Then obtain the locus of the intersection point.
Let the coordinates of P and Q be (at12, 2at1) and (at22, 2at2) respectively. It is given that the chord PQ passes through
the focus S(–a, 0). Therefore t1t2 = –1.
2
Equation of OP is y = x … (i)
t1
2at2 − 0 2t2
Equation of OQ is y – 0 = (x + a) ⇒ y = (x + a) … (ii)
2 2
at2 + a t2 + 1
2 2t2
Let R(h, k) be the point of intersection of OP and OQ. Then, k = h and k = (h + a)
t1 2
t2 + 1
2t2
⇒ k = –2ht2 and k = (h + a) [ t1t2 = –1]
2
t2 + 1
−k / h
⇒ k= (h + a) ⇒ k2 + 4h2 = –4h2 – 4ah ⇒ k2 + 8h2 + 4ah = 0
2
k
1+ 2
4h
Hence, the locus of (h, k) is obtained by replacing (h, k) with (x, y) in the above equation.
1 0 . 1 2 | Parabola
Illustration 16: AP is any chord of the parabola y2 = 4ax passing through the vertex A. PQ is a chord perpendicular
to AP. Find the locus of the mid-point of PQ. (JEE ADVANCED)
Sol: Use the parametric form to find the equation of the line PQ and then the locus of the mid-point of PQ.
Let the coordinates of P and Q be (at12, 2at1) and (at22, 2at2) respectively. Then
2 2
m1 = Slope of AP = m2 = Slope of PQ =
t1 t1 + t2
Since AT ⊥ PQ. Therefore, m1m2 = –1
4
⇒ = –1 ⇒ t1(t1 + t2) = –4 … (i)
t1 (t1 + t2 )
16a2 4a
⇒ t1t2 = – – 4 t1 = −
k2 k
4
⇒ t1t2 = − (4a2 + k 2 ) … (iv)
k2
k2 2h 8
Now (t1 + t2)2 = (t12 + t22) + 2t1t2 ⇒ = − (4a2 + k2) [Using (2), (3) and (4)]
a 2 a k2
⇒ k4 = 2k2ha – 8a2(4a2 + k2) Hence, the locus of (h, k) is
y4 = 2y2xa – 8a2(4a2 + y2) or, y4 + 8a2y2 – 2axy2 + 32a4 = 0
Illustration 17: Find a point K on axis of y2 = 4ax which has the property that if chord PQ of the parabola is drawn
1 1
through it then 2
+ is same for all positions of the chord.
PK KQ2
Sol: Consider a point of the axis of the parabola and use the distance form of a straight line to find the relation
1 1
between the parameters. The next step is to prove that 2
+ remains unchanged.
PK KQ2
1 1 x −b y −0
+ = constant = = r (r sinθ)2 = 4a( r cosθ + b)
PK 2
KQ 2 cos θ sin θ
P
⇒ r2sin2θ – 4a r cosθ – 4ab = 0
K
r12 + r22 (r1 + r2 )2 − 2r1r2
⇒ =
r12 r22 r12 r22
Q
( )
( )
2
2 2 Figure 10.13
(4acos θ) / (sin θ) − 2 ( −4ab) / (sin θ) 16a2 cos2 θ + 8ab sin2 θ
⇒ =
2 2 4
16a b / sin θ 16a2b2
8a(2acos2 θ + b sin2 θ)
f(θ) = ; f ′(θ) = sin2θ(b – 2a) = 0 b = 2a
16a2b2
Equation of the chord bisected at a given point: The equation of the chord of the parabola y2 = 4ax which is
bisected at (x1, y1) is yy1 – 2a(x + x1) = y12 – 4ax1 or T = S1
where S1 = y12 – 4ax1 and T = yy1 – 2a(x + x1)
M a them a ti cs | 10.13
Y’ Q(x2,y2)
Figure 10.14
PLANCESS CONCEPTS
1
1. The area of a triangle formed inside the parabola y2 = 4ax is (y1 – y2)(y2 – y3)(y3 – y1) where y1, y2, y3
8a
are the ordinates of vertices of the triangle.
2. If the vertex and the focus of a parabola are on the x-axis and at distance a and a′ from the origin
respectively, then the equation of the parabola is y2 = 4(a′ – a)(x – a)
Illustration 18: A ray of light is coming along the line y = b, (b > 0) from the positive direction of the x-axis and
strikes a concave mirror whose intersection with the x-y plane is the parabola y2 = 4ax, (a > 0). Find the equation
of the reflected ray and show that it passes through the focus of the parabola. (JEE ADVANCED)
Sol: In this question, we need to use the concept of angle between two lines. Use this concept to find the equation
of the reflected ray and to show that the focus lies on the reflected ray.
b2
Let P be the point of incidence. Then P is the intersection of the line y = b and the parabola y2 = 4ax. ∴P = , b
4a
b
2
b 2
∴ The equation of the tangent PT at P is y.b = 2a x + or by = 2ax + … (i)
2 2
2a 2a
‘m’ of (i) is . So, tan θ =
b b
m − (2a / b) Y T
Let the slope of the reflected ray PQ be m. ∴ tan θ =
1 + m.(2a/ b) P
2a m − (2a/ b) 2a y=b
m − (2a / b)
or = ∴ =±
b 1 + m.(2a/ b) 1 + m.(2a/ b) b
X
2a 2a 4a2 2a 2a 4a2 2
or m – = ± + m. ∴ m – = + m. Q y =4ax
b b b2 b b b2
Figure 10.15
2a 2a 4a2 4a2 4a
and m – =– – m. ∴ m1 − =
b b b2 2
b b
4a2 4a2 4ab
and m 1 + = 0. But m ≠ 0 ∴ m1 − = 4a ⇒ m =
b2 2
b b b − 4a2
2
4ab b2
∴ The equation of the reflected ray PQ is y – b = x−
b2 − 4a2 a2
or (b2 – 4a2)y – b(b2 – 4a2) = 4abx – b3 or (b2 – 4a2)y – 4abx + 4a2b = 0
This will pass through the focus (a, 0) if (b2 – 4a2)0 – 4ab . a + 4a2b = 0, which is true.
∴ The reflected ray passes through the focus.
1 0 . 1 4 | Parabola
7. TANGENT
The equation of tangent at (x1, y1) to any conic section can be obtained by replacing x2 by xx1, y2 by yy1, x
x + x1 y + y1 xy1 + x1 y
by , y by and xy by and without changing the constant (if any) in the equation of the curve.
2 2 2
a a a 2a
y2 = 4ax y = mx + c= 2,
m m m m
a a a −2a
y2 = –4ax y = mx – c=– − 2 ,
m m m m
a a 2a a
x2 = 4ay x = my + c= , 2
m m m m
a a −2a −a
x2 = –4ay x = my – c=– ,
m m 2
m m
M a them a ti cs | 10.15
Algorithm
Step I: Observe the equations of the two conics.
Step II: Identify the conic whose equation is either in standard form or it is reducible to standard form.
Step III: Write the equation of the tangent in slope form to the conic obtained in step II.
Step IV: Apply the condition that the tangent obtained in step III also touches the second conic and find the value
of m (m is the slope).
Step V: Substitute the value(s) of m obtained in step IV in the equation written in step III. The equation obtained
is the required tangent(s).
(f) The circumcircle of the triangle formed by any three points on a parabola passes through the focus of the
parabola.
(g) The tangent at any point on a parabola is equally inclined to the focal distance of the point and axis of the
parabola.
(h) If SZ is perpendicular to the tangent at a point P of a parabola, then Z lies on the tangent at the vertex and
SZ2 = AS.SP, where A is the vertex of the parabola.
(i) The image of focus w.r.t. any tangent to a parabola lies on its directrix.
( j) The length of the subtangent at any point on a parabola is equal to twice the abscissa of that point.
(k) If the tangents to the parabola y2 = 4ax at the points P and Q intersect at T, then TP and TQ subtend equal
angles at the focus.
PLANCESS CONCEPTS
• The locus of the point of the intersection of the tangent at P and the perpendicular from the focus to
this tangent is the tangent at the vertex of the parabola.
• y = mx – am2 is a tangent to the parabola x2 = 4ay for all values of m and its point of contact is (2am, am2).
The formula given in the previous table for the parabola x2 = 4ay is different as in that case, the slope
of the tangent is 1/m. Don’t confuse between the two formulae
• Angle between the tangents at two points P(at12, 2at1) and Q(at22, 2at1) on the parabola
t2 − t1
y2 = 4ax is θ = tan–1
1 + t1 t2
B Rajiv Reddy (JEE 2012, AIR 11)
Illustration 19: Prove that the straight line y = mx + c touches the parabola y2 = 4a(x + a) if c = ma + a/m.
(JEE MAIN)
Illustration 20: A tangent to the parabola y2 = 8x makes an angle of 45º with the straight line y = 3x + 5. Find its
equation and its point of contact. (JEE MAIN)
Sol: Find the slopes of the lines making an angle of 45o. Then use the standard equation of the tangent to get the
answer.
3±1
Slope values of the required tangents are m = ⇒ m1 = – 2, m2 = 1/2
1 3
Equation of the tangent of slope m to the parabola y2 = 4ax is y = mx + a/m
1
∴ Tangents are y = –2x – 1 at , – 2 and 2y = x + 8 at ( 8,8 ) .
2
Illustration 21: Find the equation of the common tangents of the parabola y2 = 4ax & x2 = 4by. (JEE MAIN)
Sol: Use the standard slope form of the equation of the tangent to find the slope and hence, the equation of the
common tangent.
M a them a ti cs | 10.17
Illustration 22: A chord of the parabola y2 = 4ax subtends a right angle at the vertex. Find the locus of the point
of intersection of tangents at its extremities. (JEE MAIN)
Sol: Find the relation between the parametric coordinates of the point subtending right angle at the vertex. Then,
use this relation to find the locus of the intersection of the tangents.
Let P(at12, 2at1) and Q(at22, 2at2) be two points on the parabola y2 = 4ax such that the chord PQ subtends a right
angle at the vertex O(0, 0). Then, Slope of OP × Slope of OQ = –1
2 2
⇒ × = –1 ⇒ t1t2 = –4 ….(i)
t1 t2
Let R(h, k) be the point of intersection of tangents at P and Q. Then,
h = at1t2 and k = a(t1 + t2) ⇒ h = –4a
Hence, the locus of R(h, k) is x = –4a.
Illustration 23: The inclinations θ and φ of two tangents to the parabola y2 = 4ax with the axis are given by tan θ
= 1/m and tan φ = m/2. Show that, as m varies, the point of intersection of the tangents traces a line parallel to the
directrix of the parabola. (JEE ADVANCED)
Sol: Use the parametric form to find the relation between the two points and hence, find the locus of the intersection
point.
Let P(at12, 2at1) and Q(at22, 2at2) be two points on the parabola y2 = 4ax such that the tangents at P and Q are
inclined at angles θ and φ with the axis of the parabola. Then
tan θ = Slope of the tangent at P
and, tan φ = Slope of the tangent at Q
⇒ tan θ = 1/t1 and tan φ = 1/t2
⇒ 1/m = 1/t1 and m/2 = 1/t2
1
⇒ 1/m × m/2 = ⇒ t1t2 = 2 … (i)
t1 t2
Let R(h, k) be the point of intersection of tangents at P and Q. Then
h = at1t2 and k = a(t1 + t2) ⇒ h = 2a [Using (1)]
Hence, the locus of R(h, k) is x = 2a, which is a line parallel to the directrix of the parabola.
Illustration 24: Tangents PQ and PR are drawn to a parabola y2 = 4ax. If p1, p2, p3 be the perpendiculars from P, Q
and R to any tangent of the parabola, prove that p1 is the geometric mean of p2 and p3. (JEE ADVANCED)
Sol: Consider two points on the parabola and find the point of intersection of the tangents at these points. Then,
consider a tangent in the general form and find p1, p2 and p3.
Let the coordinates of Q and R be (at12, 2at1) and (at22, 2at2) respectively. Then, the equations of the tangents at Q
and R are t1y = x + at12 , t2y = x + at22 respectively.
1 0 . 1 8 | Parabola
Illustration 25: Two straight lines are perpendicular to each other. One of them touches the parabola y2 = 4a( x + a)
and the other touches the parabola y2 = 4b(x + b). Prove that the point of intersection of the lines is x + a + b = 0.
(JEE ADVANCED)
Sol: Consider the two equations of the tangent in the slope form. Find the intersection of the two tangents and
prove the above result.
The equation of the tangent of slope m1 to y2 = 4a(x + a) is
y = m1(x + a) + a/m1 … (i)
The equation of the tangent to the slope m2 to the parabola y2 = 4b(x + b) is
y = m2(x + b) + b/m2 … (ii)
It is given that (1) and (2) are perpendicular to each other. Therefore m2 = –1/m1
Putting m2 = –1/m1 in (1), we get y = –1/m1(x + b) – bm1 … (iii)
The x-coordinate of the point of intersection of (1) and (3) is obtained by subtracting (3) from (1),
1 1 1
we get 0 = m1 + x + a m1 + + b m1 +
m1 m1 m1
1
⇒ 0 = (x + a + b)m1 m1 + ⇒ x + a + b = 0 ⇒ x = –(a + b).
m1
Clearly, the point (–(a + b), y) lies on the line x + a + b = 0 for all values of y. Thus, the point of intersection of (i)
and (ii) lies on the line x + a + b = 0.
Illustration 26: Prove that the circle circumscribing the triangle formed by any three tangents to a parabola passes
through the focus. (JEE ADVANCED)
Sol: Consider three points on a parabola and find the intersection of the tangents at these
points. Then, find the equation of the circle passing through these three points and prove P(t1)
B
that it passes through the focus. C
R(t3)
Let P(at12, 2at1), Q(at22, 2at2) and R(at32, 2at3) be the three points on the parabola y2 = 4ax.
A
Q(t2)
The equations of tangents at P and Q are
yt1 = x + at12 ; yt2 = x + at22 Figure 10.17
M a them a ti cs | 10.19
PLANCESS CONCEPTS
• The chord of contact and the polar of any point on the directrix always pass through the focus.
• The pole of a focal chord lies on the directrix and the locus of the poles of the focal is a directrix.
• The polars of all points on the directrix always pass through a fixed point and this fixed point is the
focus.
• The polar of the focus is the directrix and the pole of the directrix is the focus.
25
Illustration 27: The general equation to a system of parallel chords of the parabola y2 = x is 4x – y + k = 0. What
7
is the equation of the corresponding diameter? (JEE MAIN)
Sol: Solve the equation of the line and the parabola. Then use the definition of the diameter to find the answer.
Let PQ be a chord of the system whose equation is 4x – y + k = 0 … (i)
Where k is a parameter.
P
Let M(α, β) be the middle point of PQ. The locus of M is the required diameter.
M
25
The equation of the parabola is y2 = x
… (ii)
7 Q y =
2 25
x
2 7
7y
Solving (i) and (ii), 4 × – y + k = 0 or 28y2 – 25y + 25k = 0. 4x-y+k = 0
25
y + y2 25 25 Figure 10.17
Let its roots be y1, y2. Then β = 1 = = ∴ The equation of the locus of
2 2 × 28 56
25
M(α, β) is y =
56
Illustration 28: Find the locus of the middle points of the normal chords of the parabola y2 = 4ax. (JEE ADVANCED)
Sol: The locus of the middle points of the normal chords is nothing but the diameter
corresponding to the normal chords. Using this, we can easily find the answer. 2
P(at1 , 2at1)
Let PQ be a normal chord to the parabola y2 = 4ax, which is normal at P(at12, 2at1) M(,)
Let the chord extend to intersect the parabola again at Q(at2 , 2at2)
2
2
Q(at2 , 2at2)
The equation of the normal at P(at12, 2at1) is
Figure 10.18
y + t1x = 2at1 + at13 … (i)
Q(at22, 2at2) satisfies it. So
2at2 + t1 . at22 = 2at1 + at13
or 2a(t2 – t1) + at1(t22 – t12) = 0. As t1 ≠ t2, we get
2 + t1(t2 + t1) = 0 … (ii)
Let M(α, β) be the middle point of the normal chord. Then
at12 + at22 a 2 2
α= = (t + t ) … (iii)
2 2 1 2
2at1 + 2at2
β= = a(t1 + t2) … (iv)
2
β β −2a
(4) ⇒ t1 + t2 = (2) ⇒ 2 + t1. = 0 ∴ t1 =
a a β
−2a 2a β 2a β 2a
∴ (2) ⇒ 2 + t2 − = 0 or − t2 − = 0 ∴ t2 = +
β β a β a β
a −2a β 2a
2 2
a 4a2 β2 4a2
α = + + = 2 + 2 + 2 + 4
2 β a β 2 β a β
a 8a2 y 2 4a3 y2
∴ The equation of the required locus is x = 2 + 2 + 4 or x= + + 2a
2 y a y2 2a
M a them a ti cs | 10.21
Illustration 29: Show that the locus of the poles of the tangents to the parabola y2 = 4ax with respect to the
parabola y2 = 4bx is the parabola ay2 = 4b2x. (JEE ADVANCED)
Sol: Consider the general equation of the tangent to parabola. Taking a point as the pole find the polar w.r.t.
y2 = 4bx . Compare the two equations and prove the above result.
Any tangent to the parabola y2 = 4ax is ty = x + at2 ….(i)
Let (α, β) be the pole of (1) with respect to the parabola y = 4bx.
2
Then (1) is the polar of (α, β) with respect to y2 = 4bx ∴ (1) and yβ = 2b(x + α) are identical.
So, comparing these,
t 1 at2 β 2 α
= = ; ∴ t= , t =
β 2b 2bα 2b a
2
β α
∴ = or ab = 4b α
2 2
2b
a
9. NORMAL
y2 = –4ax y1
y – y1 = (x – x1)
2a
x2 =4ay x1
x – x1 = – (y – y1)
2a
x2 = –4ay x1
x–x = (y – y1)
1
2a
SLOPE FORM: The equations of normals to various standard form of the parabola in terms of the slope of the
normal are as given below.
Parametric Form: The equation of the normal to the parabola y2=4ax at point (at2, 2at) is given by y+tx=2at+at3.
The equation of normals to all standard forms of parabola in terms of parameter ‘t’ are listed below for ready
reference.
1 0 . 2 2 | Parabola
The point of intersection of normals at any two points P(at12, 2at1) and Q(at22, 2at2) on the parabola y2 = 4ax is R{2a
+ a(t12 + t22 + t1t2), – at1t2(t1 + t2)]
(at12, 2at1)
P
Y
R
X’ A X
Y’ Q
2
(at2 , 2at2)
Figure 10.19
Illustration 30: If the two parabolas y2 = 4ax and y2 = 4c(x – b) have a common normal other
b
than x-axis then prove that > 2 . (JEE MAIN)
a−c
PLANCESS CONCEPTS
If a circle intersects a parabola at four points, then the sum of their ordinates is zero.
Anand K (JEE 2009, AIR 47)
Illustration 31: If a chord which is normal to y2 = 4ax at one end subtends a right angle at the vertex then find the
angle at which it is inclined to the axis. (JEE MAIN)
1 0 . 2 4 | Parabola
Sol: Let m1 and m2 be the slopes of the line joining the vertex and the two ends of the chord. Using the relation
between m1 and m2 find the slope of the chord and hence the angle it makes with the positive direction of X-axis.
2at1 2
m1 = = The point at which the normal intersects the parabola is (at12, 2at1)
at12 t1
2 2 2t1
t2 = –t1 – ⇒ m2 = = m1
t1 2 −t12 − 2 m3
t1 −
t1
4
m1m2 = -1 ⇒ = 1 ⇒ t12 + 2 = 4 m2
t12 + 2
2
t1 = ± 2 ⇒ m3 = = –t1
2
(at2 , 2at2)
t1 + t2
Figure 10.21
⇒ tan θ = ± 2 ⇒ θ = tan−1 ( ± 2)
Illustration 32: Find the equation of the normal to the parabola y2 = 4x, which is (i) parallel to the line y = 2x – 5,
(ii) perpendicular to the line 2x + 6y + 5 = 0. (JEE MAIN)
Sol: Use the slope form of the normal to get the two equations accordingly.
The equation of the normal to the parabola,
y2 = 4ax at (am2, –2am) is y = mx – 2am – am3
Where m is the slope of the normal. Here, a = 1. So, the equation of the normal at (m2, –2m) is
y = mx – 2m – m3 …(i)
(a) If the normal is parallel to the line y = 2x – 5. Then, m = Slope of the line y = 2x – 5 is 2
Putting the value of m in (i), we obtain y = 2x – 12
as the equation of the required normal at (4, –4).
(b) If the normal in (i) is perpendicular to the line 2x + 6y + 5 = 0. Then, m = 3
hence equation of the normal is y = 3x – 33
Illustration 33: Show that the distance between a tangent to the parabola y2 = 4ax and the parallel normal is a
sec2θ cosec θ, where θ is the inclination of the either of them with the axis of the parabola. (JEE MAIN)
Sol: In the equation of the normal, use the concept of distance between two parallel lines to prove that the distance
between the tangent and the normal is a sec2θ cosec θ.
Let m be the slope of the tangent or parallel normal to the parabola y2 = 4ax. Then, m = tanθ.
The equations of the tangent and normal of slope m to the parabola y2 = 4ax are
y = mx + a/m and y = mx – 2am – am3
(a / m) + 2am + am3
The distance between these two parallel lines is given by d=
1 + m2
Illustration 34: Find the values of θ for which the line y = xcosθ + 4cos3θ – 14cosθ – 1 is a normal to the parabola
y2 = 16x. (JEE ADVANCED)
M a them a ti cs | 10.25
Sol: Compare the given equation with the standard equation of the normal and obtain the value of θ.
The slope of the given line is m = cosθ.
We know that the line y = mx + c is a normal to the parabola y2 = 4ax, if c = –2am – am3. Therefore, the given line
will be a normal to the parabola y2 = 16x, if
4cos3θ – 14cosθ – 1 = –18cosθ – 4cos3θ ; ⇒ 8cos3θ – 6cosθ = 1
⇒ 2(4cos3θ – 3 cosθ) = 1 ⇒ 2 cos 3θ = 1
1
⇒ cos3θ = ⇒ cos 3θ = cos π/3 = 1
2
2nπ π
⇒ 3θ = 2nπ ± π/3, n ∈ Z ⇒ θ = ± ,n∈Z
3 9
1
Illustration 35: Prove that three normals can be drawn from the point (c, 0) to the parabola y2 = x if c > and
2
then one of the normals is always the axis of the parabola. Also find c for which the other two normals will be
perpendicular to each other. (JEE ADVANCED)
Sol: The standard equation of the normal of a parabola is a cubic equation in ‘m’ (slope). Find the condition for
the cubic equation to have three real roots. Once we have the slope of the three normals, we can easily find the
condition for the two normals, other than the axis of the parabola, to be perpendicular to each other.
Let (t2, 1) be a foot of one of the normals to the parabola y2 = x from the point (c, 0).
Now, the equation of the normal to y2 = x at (t2, t) is
−1 −1 2 dy dy 1
y–t= .(x − t2 ) ⇒ y–t= .(x − t2 ) y =x ⇒ 2y =1 ; ∴ =
dy (1 / 2t) dx dx 2y
dx t2 , t
⇒ y – t = –2t(x – t2) ⇒ y + 2tx = t + 2t3 ….(i)
It passes through (c, 0) if 0 + 2tc = t + 2t3
2c − 1
⇒ 2t3 + t(1 – 2c) = 0 ⇒ t[2t2 – (2c – 1)] = 0 ∴ t = 0, ±
2
Three normals can be drawn if t has three real distinct values.
1
So, 2c – 1 > 0, i.e., c > .
2
The foot of one of the normals is (t2, t) where t = 0, i.e., the foot is (0, 0).
From (i), the corresponding normal is y = 0, i.e., the x-axis which is the axis of the parabola.
2c − 1
For the other two normals t = ± .
2
2c − 1 2c − 1
From (i), ‘m’ of a normal = –2t ∴ ‘m’ of the other two normals are –2 . ,2. .
2 2
2c − 1 2c − 1
They are perpendicular if –2 . ×2. = –1
2 2
⇒ –2(2c – 1) = –1 ⇒ c = 3/4
(b) Length of the normal at (at2, 2at) = 2at sec Ψ = 2at (1 + tan2 Ψ ) = 2a t2 + t2 tan2 Ψ = 2a (t2 + 1)
(c) Length of subtangent at (at2, 2at) = 2at cotΨ = 2at2
(d) Length of subnormal at (at2, 2at) = 2at tanΨ = 2a.
PLANCESS CONCEPTS
Two parabolas are said to be equal when their latus rectums are equal.
The sub tangent at any point on the parabola is twice the abscissa or proportional to square of the
ordinate of the point.
GV Abhinav (JEE 2012, AIR 329)
PROBLEM-SOLVING TACTICS
A Working Rule to find the equation of a parabola when focus & directrix are given:
Step 1: Find the distance between focus and general point P(x, y) by the distance formula.
Step 2: Find the perpendicular distance from the point P(x, y) to the given directrix.
ax1 + by1 + c
(The perpendicular distance from a point P(x1, y1) to the line ax + by + c = 0 is )
a2 + b2
Step 3: Equate the distances calculated in step 1 and step 2. On simplification we get the required equation of the
parabola.
A Working Rule to find the equation of a parabola when the vertex and the focus are given:
y 2 − y1
Step 1: Find the slope of the axis formed by joining the focus and the vertex by the formula
x2 − x1
Step 2: Find the slope of the directrix by the formula m1.m2 = –1; where m1 is the slope of the axis of the parabola
and m2 is the slope of the directrix.
Step 3: Find a point on the directrix as the vertex, which is the middle point between the focus and the point on
the directrix, by means of the mid-point formula.
Step 4: Write the equation of the directrix, using the slope point formula.
Step 5: The focus and the directrix are now known so we can find the equation of the parabola by the method
given above.
M a them a ti cs | 10.27
FORMULAE SHEET
1. Definition: A parabola is the locus of a point which moves so that its distance from a fixed point is equal to
its distance from a fixed straight line.
For e.g. if the focus is (α, β) and the directrix is ax + by + c = 0 then the equation of the parabola is
(ax + by + c)2
(x – α)2 + (y – β)2 =
a2 + b2
2. The general equation of the second degree ax2 + 2hxy + by2 + 2gx + 2fy + c = 0 represents a parabola if
∆ ≠ 0 and h2 = ab.
3.
Equation of the parabola
y2 = 4ax y2 = –4ax x2 = 4ay x2 = –4ay
Properties
Vertex (Co-ordinates) (0, 0) (0, 0) (0, 0) (0, 0)
Focus (Co-ordinates) (a, 0) (–a, 0) (0, a) (0, –a)
Latus rectum (length) 4a 4a 4a 4a
Axis (Equation) y =0 y =0 x=0 x=0
Directrix (Equation) x = –a x=a y = –a y=a
Symmetry (about) x-axis x-axis y-axis y-axis
4. The equation of the chord joining points P(at12, 2at1) and Q(at22, 2at2) on the parabola y2 = 4ax is y(t1 + t2) =
2x + 2at1t2.
5. If the equation of the chord joining points t1 and t2 on the parabola y2 = 4ax passes through the focus then
t1t2 = –1.
In other words, if one end of a focal chord of the parabola y2 = 4ax is P(at2, 2at) then the co-ordinates of the
a −2a
other end is Q , .
t2 t
a −2a
6. The length of the focal chord passing through P(at2, 2at) and Q , is a(t + 1/t) .
2
t 2 t
4 2
7. The length of the chord intercepted by the parabola on the line y = mx + c is a(1 + m )(a − mc) .
m2
8. The length of the chord joining two points ‘t1’ and ‘t2’ on the parabola y2 = 4ax is a(t1 – t2) (t1 + t2 )2 + 4 .
9. The equation of the tangent to the parabola y2 = 4ax at a point (x1, y1) is given by yy1 =2a(x +x1)
10. Parametric Form
Equation of the parabola Equation of the tangent Condition of tangency Point of contact
a a a 2a
y2 = 4ax y = mx + c= 2,
m m m m
a a −a −2a
y2 = –4ax y = mx – c=– 2,
m m m m
a a 2a a
x2 = 4ay x = mx + c= , 2
m m m m
a a −2a −a
x2 = –4ay x = mx – c=– ,
2
m m m m
12. The point of intersection of tangents at the points (at12, 2at1) and (at22, 2at2) is given by (at1t2, a(t1 + t2))
13. If SZ be perpendicular to the tangent at a point P of a parabola, then Z lies on the tangent at the vertex and
SZ2 = AS × SP, where A is the vertex of the parabola.
14. Angle between tangents at two points P(at12, 2at1) and Q(at22, 2at2) on the parabola y2 = 4ax is θ = tan–1
t2 − t1
1 + t1 t2
y1
y2 = 4ax y – y1 = – (x – x1)
2a
y1
y2 = –4ax y – y1 = (x – x1)
2a
x1
x2 = 4ay x – x1 = – (y – y1)
2a
x1
x2 = –4ay x – x1 = – (y – y1)
2a
16. The point of intersection of normals at any two points P(at12, 2at1) and Q(at22, 2at2) on the parabola y2 = 4ax is
given by R[2a + a(t12 + t22 + t1t2), –at1t2(t1 + t2)]
17. (i) The algebraic sum of the slopes of the normals at the co-normal point is zero.
(ii) The centroid of a triangle formed by the co-normal points on a parabola lies on its axis.
2
18. If the normal at the point P (at12, 2at1) meets the parabola y2 = 4ax again at (at22, 2at2). Then t2 = –t1 – .
t1
19. If the normal drawn at the point P(at12, 2at1) and Q(at22, 2at2) to the parabola y2 = 4ax intersect at a third point
on the parabola then t1.t2 = 2.
20. If the normal chord at a point P(at2, 2at) to the parabola y2 = 4ax subtends a right angle at the vertex of the
parabola, then t2 = 2.
21. The chord of contact of tangents drawn from a point P(x1, y1) to the parabola y2 = 4ax is yy1 = 2a(x + x1).
22. The combined equation of the pair of tangents drawn from an external point (x1, y1) to the parabola y2 = 4ax is
SS1 = T2 where, S = y2 – 4ax, S1 = y12 – 4ax1 and T = yy1 – 2a(x + x1).
23. The equation of the chord of the parabola y2 = 4ax which is bisected at (x1, y1) is
yy1 – 2a(x + x1) = y12 – 4ax1 or, T = S1.
24. The polar of a point (x1, y1) with respect to the parabola y2 = 4ax is yy1 = 2a(x + x1).
25. The equation of the diameter of the parabola y2 = 4ax bisecting chords of slope m is y = 2a/m.
26. A circle on any focal radii of a point P(at2, 2at) as diameter touches the tangent at the vertex and intercepts a
28. Tangents and normals at the extremities of the latus rectum of a parabola y2 = 4ax constitute a square, their
points of intersection being (–a, 0) and (3a, 0).
29. The semi latus rectum of the parabola y2 = 4ax, is the harmonic mean between segments of any focal chord
2bc 1 1 1
of the parabola, i.e. 2a = or, + = .
b+c b c a
30. The orthocentre of any triangle formed by tangents at any three points P(t1), Q(t2) and R(t3) on a parabola
y2 = 4ax lies on the directrix and has the coordinates (–a, a(t1 + t2 + t3 + t1t2t3)).
31. If a normal drawn to a parabola passes through a point P(h, k), then k = mh – 2am – am3, i.e,
am3 + m(2a – h) + k = 0,
2a − h k
⇒ m1 + m2 + m3 = 0; m 1m 2 + m 2m 3 + m 3 m 1 = ; and m1m2m3 = − .
a a
32. The equation of a circle circumscribing the triangle formed by three co-normal points and which passes
through the vertex of the parabola is 2(x2 + y2) – 2(h + 2a)x – ky = 0.
33. The area of a triangle formed inside the parabola y2 = 4ax is 1 (y1 – y2)(y2 – y3)(y3 – y1) where y1, y2, y3 are the
ordinates of the vertices of the triangle. 8a
34. If the vertex and the focus of a parabola are on the x-axis and at a distance a and b from the origin respectively
then the equation of the parabola is y2 = 4(b – a)(x – a).
1 0 . 3 0 | Parabola
Solved Examples
Sol: Refer to Fig. 10.22. If we consider the origin to be Example 3: If the focus of a parabola divides a focal
the vertex of the parabola. Then we know that the point chord of the parabola in segments of length 3 and 2,
(5 , 10) will lie on the parabola. Using this we can solve the length of the latus rectum of the parabola is-
the question easily. (A) 3/2 (B) 6/5 (C) 12/5 (D) 24/5
M
Sol: Let y2 = 4ax be the equation of the parabola, then
10 cm the focus is S(a, 0). Let P(at12, 2at1) and Q(at22, 2at2) be
5 cm
A
B
x vertices of a focal chord of the parabola, then t1t2 = –1.
Let SP = 3, SQ = 2
10 cm
Let P(at2, 2at) be a point on the parabola y2 = 4ax, then Sol: Convert the given equation to the standard form.
at2 = 2at ⇒ t = 2 and thus the coordinates of P are The equation of the parabola can be written as
2
(4a, 4a). ax
y 3 3
= + − –2
Equation of the normal at P is y = –tx + 2at + at3 a
4 16
3
⇒ y = –2x + 4a + 8a 2
3 a2 35
⇒ 2x + y = 12a …(i)
or x + = y + a
4a 3a 16
Which meets the parabola y = 4ax at points given by
2
Vertex is x = –3/4a, y = –35a/16
y2 = 2a(12a – y) ⇒ y2 + 2ay – 24a2 = 0
Locus of the vertex is xy = 105/64.
⇒ y = 4a or y = –6a
y = 4a corresponds to the point P Example 8: Find the locus of the foot of the
perpendicular drawn from a fixed point to any tangent
and y = – 6a ⇒ x = 9a from (i) to a parabola.
So that the coordinates of Q are (9a, –6a). Since the
coordinates of the focus S are (a,0), slope of SP = 4/3 Sol: Take a fixed point and use it to find the foot of the
and slope of SQ = –6/8. Product of the slopes = –1. perpendicular on a general equation of a tangent.
Let the parabola be y2 = 4ax and the fixed point be
Example 6: The common tangents to the circle (h, k)
x2 + y2 = a2/2 and the parabola y2 = 4ax intersect at the
(h,k)
focus of the parabola.
P(at2, 2at)
(A) x2 = 4ay (B) x2 = –4ay
M
(C) y2 = –4ax (D) y2 = 4a(x + a) (,)
β −k
2 The equation of the normal of a parabola
β −k
β − = α + a.
α − h α −h y2 = 4ax is y = –tx + 2at + at3 …(i)
or –β(β – k)(α – h) = α(α – h)2 + a(β – k)2 The joint equation of the lines joining the vertex (origin)
to the points of intersection of the parabola and the
∴ The equation of the locus of the foot M is
line (i) is
x(x – h)2 + y(x – h)(y – k) + a(y – k)2 = 0. y + tx
y2 = 4ax
3
2at + at
Example 9: Tangents to the parabola at the extremities
of a common chord AB of the circle x2 + y2 = 5 and the ⇒ (2t + t3)y2 = 4x(y + tx)
parabola y2 = 4x intersect at the point T. A square ABCD
⇒ 4t x2 – (2t + t3)y2 + 4xy = 0
is constructed on this chord lying inside the parabola,
then [(TC)2 + (TD)2]2 is equal to ? Since these lines are at right angles co-efficient of x2 +
coefficient of y2 = 0
Sol: Find the point of intersection of the circle and the
⇒ 4t – 2t – t3 = 0 ⇒ t2 = 2
parabola. Then get the equation of the chord and the
point of intersection of the tangents at the end of the For t = 0, the normal line is y = 0, i.e. the axis of the
chord. In the last step use simple geometry to find parabola which passes through the vertex (0, 0).
[(TC)2 + (TD)2]2.
The points of intersection of the circle and the parabola Example 2: A parabola is drawn touching the x-axis at
are A(1, 2), B(1, –2) the origin and having its vertex at a given distance k
from the x-axis. Prove that the axis of the parabola is a
The equation of the common chord is x = 1, which is
tangent to the parabola x2 + 8k(y – 2k) = 0.
the latus rectum of the parabola.
x=-1 Sol: Use the relation between the tangent at the vertex
A(1,2)
D(5,2) and the axis of the parabola to prove it.
Sol: Represent the parabola in the standard form. The normals at P & Q intersect at
Compare the equation of the directrix with the given
equation and form a quadratic in k. Solve the quadratic ( ( )
M(x,y)= 2a t12 + t22 + t1 t2 − at1 t2 ( t1 + t2 ) )
for two real roots to get the desired value of k.
From (1) t1 t2 =-1
The equation of the parabola can be written as
∴ y=a ( t1 + t2 ) x=2a( t12 + t22 -1)
y = k(x – 8/k) which is of the form Y = 4AX
2 2 ;
1 0 . 3 4 | Parabola
2
y= (
a2 t12 + t22 − 2 ) (ii) – (i) ⇒ –4aα =
4abm 1 + m2
1 + m2
y2 x
⇒ + 1 = ⇒ 2y 2 + 2a2 =
xa
a2 2a b2m2 m2 α2
or a2 = ∴ = ... (iv)
Here, a=1/4 1 + m2 1 + m2 b2
2
∴ Locus is 16y= 2x − 1 Putting in (iii) from (iv)
a2 α2
Example 5: A parabola is drawn to pass through A and a2 + a2 + b2 = + b2
B, the ends of a diameter of a given circle of radius b2
a, and to have as directrix a tangent to a concentric a2
∴ 1 − 2 a2 + b2 = b2 – a2
circle of radius b; the axes of reference being AB and b
a perpendicular diameter, prove that the locus of the
b2 − a2 α2 β2
x2 y2 or a2 + b2 = b2 – a2 ∴ + =1
focus of the parabola is + = 1. b2 b2 b2 − a2
b2 b2 − a2
Sol: Consider a circle with its centre at the origin. Let ∴ The equation of the locus of the focus (α, β) is
the two points A and B lie on the X-axis. Write the x2 y2
equation of the tangent in standard form and apply the + = 1.
focus-directrix property to prove the given statement. b2 b2 − a2
Let A = (–a, 0) and B = (a, 0) Example 6: Let (xr, yr) ; r = 1,2,3,4 be the points of the
The centre of the circle = (0, 0) intersection of the parabola y2 = 4ax and the circle
x2 + y2 = b2 Prove that y1 + y2 + y3 + y4 = 0.
Any tangent to x2 + y2 = b2 is Sol: Solve the equation of the circle and the parabola.
y = mx + b 1 + m 2 Then use the theory of equations to prove y1 + y2 + y3
+ y4 = 0.
Which is the directrix of the parabola.
Let x2 + y2 + 2gx + 2fy + c = 0 … (i)
Let (α, β) be the focus.
y2 = 4ax … (ii)
Then by focus-directrix property, the equation of the
parabola will be Solving (i) and (ii), we get the coordinates of points of
2
y − mx − b 1 + m2 intersection
(x – α)2 + (y – β)2 =
y2
1+m 2 From (ii), x = putting in (i),
4a
2
It passes through A (–a, 0), B (a, 0); so y2 y2
+ y 2 + 2g. + 2fy + c = 0
2 4a 4a
ma − b 1 + m2
(a + α) + b =
2 2 1 g
1 + m2 or y 4 + 1 + y 2 + 2fy + c = 0
(4a)2
2a
m2a2 + b2 (1 + m2 ) − 2abm 1 + m2 It has four roots.
= ... (i)
1 + m2
2
Its roots are y1, y2, y3 and y4.
−ma − b 1 + m2
(a – α)2 + b2 = coefficientof y 3
Now, sum of roots = −
1 + m2 coefficientof y 4
m a + b (1 + m ) + 2abm 1 + m2
2 2 2 2 0
= ... (ii) ∴ y1 + y2 + y3 + y4 = − = 0.
1 + m2 1 / (4a)2
m2
or a2 + b2 = a2 + b2 ... (iii) Example 7: From the point, where any normal to the
1 + m2 parabola y2 = 4ax meets the axis, a line perpendicular to
M a them a ti cs | 10.35
the normal is drawn. Prove that this line always touches the line PQ whose equation is
the parabola y2 + 4a(x – 2a) = 0. 1
y t − = 2(x – a) is given by
Sol: Get the equation of the line perpendicular to the t
normal, passing through the intersection of the normal 2a 2a
p= =
and the axis. Use the theory of equation 2
(t − 1 / t) + 22 (t + 1 / t)
∴ The equation of the line through this point drawn (A) 2(a3 – 3)x – ay + a3 – 3a = 0
perpendicular to the normals is (B) 2(a3 – 3)x – ay + a3 – 3a = 0
1 (C) (a3 – 3)x – 2ay – 2a3 + 6a = 0
y–0= (x – 2a + at2 )
t
(D) None of these
{ ‘m’ of normal = –t}
or ty = x – 2a – at2 … (i) Sol: In this case we need to calculate the point of
intersection of C and D and then find the equation of
We have to prove that (1) touches the parabola the tangent to the parabola y = kx2.
y2 + 4a(x – 2a) = 0 … (ii) C and D intersect at the points for which x2 – 3 = kx2.
Solving (i) and (ii), y2 + 4a(ty + at2) = 0 But x = a(given)
or y2 + 4aty + (2at)2 = 0 or (y + 2at)2 = 0 a2 − 3
⇒ k= .
∴ y = –2at, –2at a2
∴ (i) cuts (ii) at coincident points, i.e., (i) touches (ii). So the coordinates of A are (a, a2 – 3)
The equation of the tangent L at A to D: y = kx2 is
Example 8: Consider a parabola y2 = 4ax, the length 1 a2 − 3
of focal chord is and the length of the perpendicular (y + a2 – 3) = xa
2 a2
from the vertex to the chord is p then-
⇒ 2(a2 – 3)x – ay – a3 + 3a = 0 (L)
(A) .p is constant (B) .p2 is constant
(C) 2.p is constant (D) None of these Example 10: If the line L meets the parabola C at a
point B on the line L2, other than A then a is equal to-
Sol: A quantity is constant if it does not depend on the
(A) –3 (B) –2 (C) 2 (D) 3
parameter. Represent and p in terms of the parameter
and look for the quantity in which the parameter gets
eliminated. Sol: Proceed further from the previous solution.
Let P(at2, 2at) and Q(a/t2, –2a/t) be a focal chord of the The line L meets the parabola
parabola (as t1t2 = –1) C: y = x2 – 3 at the points for which
2 2 2 2
The length of PQ = = (at − a / t ) + (2at + 2a / t) 2(a2 − 3)
x2 – 3 = x − a2 + 3 ⇒ (x – a)(ax + 6 – a2) = 0.
= a (t2 − 1 / t2 )2 + 4(t + 1 / t)2 a
But x = 1 & x ≠ a.
= a(t + 1/t) (t − 1 / t)2 + 4 = a(t + 1/t)2
a2 − 6
The length of the perpendicular from the vertex (0,0) on ⇒x= = 1 ⇒ a2 – a – 6 = 0
a
1 0 . 3 6 | Parabola
∴ The angle between OA and OB is The equation of the normal at (x1, y1) is
3 −1 y1
tan–1 = tan–1(1/2). y – y1 = – (x – x1)
1+3 2a
⇒ xy1 = 2a(y1 – y) + x1y1
Example 12: Let y2 = 4ax be the equation of a parabola,
Next, the equation of the focal chord through (x1, y1)
then
and (a, 0) is
(A) yy1 = 2a(x + x1) (p) Equation of thenormal at (x1,y1) y1 − 0
y= (x – a)
(B) xy1=2a(y1–y)+x1y1 (q) Equation of the focal chord x1 − a
through (x1,y1) ⇒ xy1 = y(x1 – a) + ay1
(C) xy1=y(x1–a)+ay1 (r) Equation of the through (x1,y1) Lastly, the equation of the line joining (–a, 0), the point
and the point of intersection of axis of intersection of the axis y = 0 and the directrix x + a
with the direct = 0 with (x1, y1) is
(D) (x+a)y1=(x1+a)y (s) Equation of the tangent at (x1,y1) (x + a)y1 = (x1 + a)y
JEE Main/Boards
Q.10 Find the equation of the common tangent to the Q.22 Show that the portion of the tangent to a parabola
parabola y2 = 32x and x2 = 108y. cut off between the directrix and the curve subtends a
right angle at the focus.
Q.11 Find the point where normal to the parabola
1 1 Q.23 If the tangent to the parabola y2 = 4ax meets
y2 = x at , cuts it again. the axis in T and the tangent at the vertex A in Y and
4 2
rectangle TAYG is completed, show that the locus of G
Q.12 Find shortest distance between y2 = 4x and is y2 + ax = 0.
x2 + y2 – 24y + 128 = 0.
Q.24 Two equal parabolas have the same vertex and
Q.13 AB is a chord of the parabola y = 4ax with the
2 their axes are at right angles. Prove that they cut again
end A at the vertex of the given parabola. BC is drawn 3
at an angle tan–1 .
perpendiculars to AB meeting the axis of the parabola 4
at C. Find the projection of BC on this axis.
Q.25 Find the locus of the point of intersection of the
Q.14 M is the foot of the perpendicular from a point P tangents to the parabola y2 = 4ax which include an
on the parabola y2 = (x – 3) to its directrix and S is the angle α.
focus of the parabola, if SPM is an equilateral triangle,
find the length of each side of the triangle. Q.26 Find the set of points on the axis of the parabola
y2 – 4x – 2y + 5 = 0 from which all the three normals
Q.15 PQ is a double ordinate of a parabola y2 = 4ax. If drawn to the parabola are real and distinct.
the locus of its points of trisection is another parabola
length of whose latus rectum is k times the length of Q.27 Show that the locus of points such that two of
the latus rectum of the given parabola, then find the the three normals to the parabola y2 = 4ax from them
value of k. coincide is 27ay2 = 4(x – 2a)3.
Q.16 Find the equation of the parabola, the extremities Q.28 If a circle passes through the feet of normals
of whose latus rectum are (1, 2) and (1, –4). drawn from a point to the parabola y2 = 4ax, Prove that
the circle also passes through origin.
Q.17 Prove that the normal chord to a parabola at the
point whose ordinate is equal to the abscissa subtends Q.29 The middle point of a variable chord of the parabola
a right angle at the focus.
y2 = 4ax lies on the line y = mx + c. Show that it always
2
Q.18 If from the vertex of the parabola y2 = 4ax a pair 2a c
touches the parabola y + = 8a x + .
of chords be drawn at right angles to one another and m m
with these chords as adjacent sides a rectangle be
drawn, prove that the locus of the vertex of the farther
angle of the rectangle is the parabola y2 = 4a(x – 8a).
Exercise 2
Q.19 Prove that the locus of the middle points of
Single Correct Choice Type
all chords of the parabola y2 = 4ax which are drawn
through the vertex is the parabola y2 = 2ax. Q.1 The length of the chord intercepted by the parabola
y2 = 4x on the straight line x + y = 1 is-
Q.20 Show that the locus of the middle point of all
chords of the parabola y2 = 4ax passing through a fixed (A) 4 (B) 4 2 (C) 8 (D) 8 2
point (h, k) is y2 – ky = 2a(x – h).
Q.2 A parabola is drawn with its focus at (3, 4) and
Q.21 Prove that the area of the triangle formed by the vertex at the focus of the parabola y2 – 12x – 4y + 4 =
tangents at points t1 and t2 on the parabola y2 = 4ax 0. The equation of the parabola is-
a2 (A) x2 – 6x – 8y + 25 = 0 (B) x2 – 8x – 6y + 25 = 0
with the chord joining these two points is |t – t |3.
2 1 2
(C) x2 – 6x + 8y – 25 = 0 (D) x2 + 6x – 8y – 25 = 0
1 0 . 3 8 | Parabola
(B) Equals the semi latus rectum of the parabola (A) 3 (B) 4 (C) 6 (D) None
(C) Equals latus rectum of the parabola
Q.13 The line 4x – 7y + 10 = 0 intersects the parabola, y2
(D) Equals double the latus rectum of the parabola = 4x at the points A and B. The coordinates of the point
of intersection of the tangents drawn at the points A
Q.7 A point P moves such that the difference between and B are-
its distances from the origin and from the axis of ‘x’ is
always a constant c. The locus- 7 5 5 7 5 7 7 5
(A) , (B) − , (C) , (D) − ,
(A) A straight line having equal intercepts C on the axis 2 2 2 2 2 2 2 2
c
(B) A circle having its centre at 0, − & passing Q.14 A line passing through the point (21,30) and
2
c normal to the curve y = 2 x can have the slope-
through c 2,
2
(A) 2 (B) 3 (C) –2 (D) –5
c
(C) A parabola with its vertex at 0, − & passing
2
c Q.15 If the chord of contact of tangents from a point
through c 2,
2 P to the parabola y2=4ax touches the parabola x2=4by,
(D) None of these the locus of P is-
(A) Circle (B) Parabola
Q.8 Tangents are drawn from the point (–1, 2) on
(C) Ellipse (D) Hyperbola
the parabola y2 = 4x. The length, these tangents will
intercept on the line x = 2, is-
Q.16 If M is the foot of the perpendicular from a point
(A) 6 (B) 6 2 (C) 2 6 (D) None P of a parabola y2=4ax its directrix and SPM is an
equilateral triangle, where S is the focus, the SP is equal
Q.9 Which one of the following equations represented to-
parametric-cally represents equation to a parabolic
(A) a (B) 2a (C) 3a (D) 4a
profile ?
M a them a ti cs | 10.39
Q.17 The latus rectum of a parabola whose focal chord Q.3 The equation of the directrix of the parabola y2 +
is PSQ such that SP = 3 and SQ = 2 is given by- 4y + 4x + 2 = 0 is- (2000)
(A) 24/5 (B) 12/5 (C) 6/5 (D) None (A) x = –1 (B) x = 1 (C) x = –3/2 (D) x = 3/2
Q.18 The normal chord of to a parabola y2 = 4ax at Q.4 The locus of the midpoint of the line segment
the point whose ordinate is equal to the abscissa, then joining the focus to a moving point on the parabola
angle subtended by normal chord at the focus is- y2=4ax is another parabola with directrix- (2000)
π π a a
(A) (B) tan–1 2 (C) tan-1 2 (D) (A) x = –a (B) x = − (C) x = 0 (D) x =
4 2 2 2
Q.19 P is any point on the parabola, y2=4ax whose Q.5 The equation of the common tangent to the curves
vertex is A. PA is produced to meet the directrix in D y2 = 8x and xy = –1 is- (2000)
and M is the foot of the perpendicular from P on the
directrix. The angle subtended by MD at the focus is- (A) 3y = 9x + 2 (B) y = 2x + 1
(A) π/4 (B) π/3 (C) 5π/12 (D) π/2 (C) 2y = x + 8 (D) y = x + 2
Q.20 A parabola y = ax2 + bx + c crosses the x-axis at Q.6 Let (x, y) be any point on the parabola y2 = 4x. Let
(α, 0), (β, 0) both to right of the origin. A circle also P be the point that divides the line segment from (0, 0)
passes through these two points. The length of a to (x, y) in the ratio 1: 3. Then, the locus of P is- (2011)
tangent from the origin to the circle is- (A) x2 = y (B) y2 = 2x (C) y2 = x (D) x2 = 2y
bc b c
(A) (B) ac2 (C) (D) Q.7 Let P(x1, y1) and Q(x2, y2), y1 < 0, y2 < 0, be the end
a a a
points of the latus rectum of the ellipse x2 + 4y2 = 4.
Q.21 TP and TQ are tangents to the parabola, y2 = 4ax The equations of parabolas with latus rectum PQ are-
at P and Q. If the chord PQ passes through the fixed (2008)
point (–a, b) then the locus of T is-
(A) x2 + 2 3y=3+ 3 (B) x2 – 2 3y=3+ 3
(A) ay = 2b(x – b) (B) bx = 2a(y – a)
(C) x2 + 2 3y=3– 3 (D) x2 – 2 3y=3– 3
(C) by = 2a(x – a) (D) ax = 2b(y – b)
Q.8 Let L be a normal to the parabola y2 = 4x. If L passes
Q.22 The triangle PQR of area ‘A’ is inscribed in the through the point (9, 6), then L is given by- (2011)
parabola y2 = 4ax such that the vertex P lies at the
vertex of the parabola and the base QR is a focal chord. (A) y – x + 3 = 0 (B) y + 3x – 33 = 0
The modulus of the difference of the ordinates of the (C) y + x – 15 = 0 (D) y – 2x + 12 = 0
point Q and R is-
A A 2A 4A Q.9 The point of intersection of the tangents at the ends
(A) (B) (C) (D) of the latus rectum of parabola y2 = 4x is…….. (1994)
2a a a a
Q.13 From a point ‘A’ common tangents are drawn Q.15 At any point P on the parabola y2 – 2y – 4x + 5 = 0
a2 a tangent is drawn which meets the directrix at Q. Find
to the circle x2 + y2 = and parabola y2 = 4ax. Find the locus of point R, which divides QP externally in the
2
1
the area of the quadrilateral formed by the common ratio :1. (2004)
tangents, the chord of contact of the circle and the 2
chord of contact of the parabola. (1996)
Q.16 The slope of the line touching both the parabolas
y 2 = 4x and x2 = −32 y is (2014)
Q.14 The angle between a pair of tangents drawn from
1 3 1 2
a point P to the parabola y2 = 4ax is 45º. Show that the (A) (B) (C) (D)
locus of the point P is a hyperbola. (1998) 2 2 8 3
JEE Advanced/Boards
Q.17 Three normals to y2 = 4x pass through the point Q.28 A variable tangent to the parabola y2 = 4ax meets
(15, 12). Show that one of the normals is given by the circle x2 + y2 = r2 at P and Q. Prove that the locus of
y = x – 3 and find the equations of the others. the mid point of PQ is x(x2 + y2) + ay2 = 0.
Q.18 A circle is described whose centre is the vertex and Q.29 A variable chord PQ of a parabola y2=4ax, subtends
whose diameter is three-quarters of the latus rectum of a right angle at the vertex. Show that it always passes
a parabola y2 = 4ax. Prove that the common chord of through a fixed point. Also show that the locus of the
the circle and parabola bisects the distance between point of intersection of the tangents at P and Q is a
the vertex and the focus. straight line. Find the locus of the mid point of PQ.
Q.20 Find the equation of the circle which passes Q.1 The tangent at P to a parabola y2=4ax meets the
through the focus of the parabola x2 = 4y and touches directrix at U and the latus rectum at V then SUV (where
it at the point (6, 9). S is the focus).
(A) Must be a right triangle
Q.21 P and Q are the point of contact of the tangents
(B) Must be an equilateral triangle
drawn from the point T to the parabola y2 = 4ax. If PQ
be the normal to the parabola at P, prove that TP is (C) Must be an isosceles triangle
bisected by the directrix.
(D) Must be a right isosceles triangle
Q.22 Prove that the locus of the middle points of the
Q.2 If the distances of two points P and Q from the focus
normal chords of the parabola y2 = 4ax is
of a parabola y2=4ax are 4 and 9, then the distance of
y 2 4a3 the point of intersection of tangents at P and Q from
+ = x – 2a.
2a y 2 the focus is-
(A) 8 (B) 6 (C) 5 (D) 13
Q.23 Two tangents to the parabola y2=8x meet the
tangent at its vertex in the point P and Q. If PQ = 4 units, Q.3 The chord of contact of the pair of tangents drawn
prove that the locus of the point of the intersection of from each point on the line 2x + y = 4 to the parabola
the two tangents is y2=8(x + 2). y2=–4x passes through a fixed point.
Q.24 Two perpendicular straight lines through the (A) (–2, 1) (B) (–2, –1)
focus of the parabola y2=4x meet its tangents to the (C) (1/2, 1/4) (D) (–1/2, –1/4)
parabola parallel to the perpendicular lines intersect in
the mid point of TT′.
Q.4 The locus of the foot of the perpendiculars drawn
from the vertex on a variable tangent to the parabola
Q.25 A variable chord PQ of the parabola y2 = 4x is
y2 = ax is-
drawn parallel to the line y=x. If the parameters of the
points P and Q on the parabola are p and q respectively, (A) x(x2 + y2) + ay2 = 0 (B) y(x2 + y2) + ax2 = 0
show, that p + q = 2. Also show that the locus of the (C) x(x2 – y2) + ay2 = 0 (D) None of these
point of intersection of the normals at P and Q is
2x – y = 12.
Q.5 Locus of the point of intersection of the perpendicular
tangents of the curve y2 + 4y – 6x – 2 = 0 is-
Q.26 Show that the circle through three points the
normals at which to the parabola y2 = 4ax are concurrent (A) 2x – 1 = 0 (B) 2x + 3 = 0
at the point (h, k) is 2(x2 + y2) – 2(h + 2a)x – ky = 0. (C) 2y + 3 = 0 (D) 2x + 5 = 0
Q.27 Find the condition such that the chord t1t2 of the
Q.6 If the tangent at the point P(x1, y1) to the parabola
parabola y2 = 4ax passes through the point (a, 3a). Find
y2 = 4ax meets the parabola y2 = 4a(x + b) at Q and R,
the locus of intersection of t tangents at t1 and t2 under
then the mid point of QR is-
this condition.
1 0 . 4 2 | Parabola
(A) (x1 + b, y1 + b) (B) (x1 – b, y1 – b) Q.15 A variable circle is drawn to touch the line
3x – 4y = 10 and also the circle x2 + y2 = 4 externally
(C) (x1, y1) (D) (x1 + b, y1)
then the locus of its centre is-
Q.7 The point (s) on the parabola y2 = 4x which are (A) Straight line
closest to the circle, x2 + y2 – 24y + 128 = 0 is/are- (B) Circle
(A) (0, 0) (B) (2, 2 2) (C) (4, 4) (D) None (C) Pair of real, distinct straight lines
(D) Parabola
Q.8 Length of the focal chord of the parabola y2 = 4ax
at a distance p from the vertex is- Q.16 The tangent and normal at P(t), for all real positive
2a 2
a 3
4a 3
p 2 t, to the parabola y2=4ax meet the axis of the parabola
(A) (B) (C) (D) in T and G respectively, then the angle at which the
p p 2
p 2
a2
tangent at P to the parabola is inclined to the tangent
Q.9 If two normals to a parabola y2=4ax intersect at at P to the circle through the points P, T and G is-
right angles then the chord joining their feet passes (A) cot–1t (B) cot–1t2 (C) tan–1t (D) tan–1t2
through a fixed point whose co-ordinates are-
(A) (–2a, 0) (B) (a, 0) (C) (2a, 0) (D) None Q.17 P is a point on the parabola y2=4x where abscissa
and ordinate are equal. Equation of a circle passing
Q.10 Locus of a point p if the three normals drawn through the focus and touching the parabola at P is-
from it to the parabola y2 = 4ax are such that two of (A) x2 + y2 – 13x + 2y + 12 = 0
them make complementary angles with the axis of the
(B) x2 + y2 – 13x – 18y + 12 = 0
parabola is-
(C) x2 + y2 + 13x – 2y – 14 = 0
(A) y2 = a(x + a) (B) y2 = 2a(x – a)
(D) None of these
(C) y2 = a(x – 2a) (D) y2 = a(x – a)
latus rectum and the axis at right angles to the of the Q.2 The equation of the common tangent touching the
given curve is- circle (x – 3)2 + y2 = 9 and the parabola y2 = 4x above
the x-axis is- (2001)
(A) x2 + 4x + 8y – 4 = 0 (B) x2 + 4x – 8y + 12 = 0
(A) 3 y = 3x + 1 (B) 3 y = –(x + 3)
(C) x2 + 4x + 8y + 12 = 0 (D) x2 + 8x – 4y + 8 = 0
(C) 3 y = x + 3 (D) 3 y = –(3x + 1)
Q.21 Two parabolas have the same focus. If their
directrices are the x-axis and the y-axis respectively,
then the slope of their common chord is- Q.3 The focal chord to y2 = 16x is tangent to
(x – 6)2 + y2 = 2, then the possible values of the slope of
(A) 1 (B) –1 (C) 4/3 (D) 3/4
this chord, are- (2003)
Q.22 Equation of common tangent to the circle, (A) {–1, 1} (B) {–2, 2} (C) {–2, 1/2} (D) {2, –1/2}
x2 + y2 = 50 and the parabola, y2 = 40 x can be-
(A) x + y – 10 = 0 (B) x – y + 10 = 0 Q.4 Axis of a parabola is y = x and vertex and focus are
at a distance 2 and 2 2 respectively from the origin.
(C) 1 + y + 10 = 0 (D) x – y – 10 = 0
Then equation of the parabola is- (2006)
(A) (x – y)2 = 8(x + y – 2) (B) (x + y)2 = 2(x + y – 2)
Q.23 The equation y2 + 3 = 2(2x + y) represents a
parabola with the vertex at- (C) (x – y)2 = 4(x + y – 2) (D) (x + y)2 = 2(x – y + 2)
1
(A) ,1 and axis parallel to x-axis Q.5 Normals at P, Q, R are drawn to y2=4x which
2
intersect at (3, 0). Then
1
(B) 1, and axis parallel to x-axis Column I Column II
2
(A) Area of DPQR (p) 2
1 3
(C) ,1 and focus at ,1
2 2 (B) Radius of circumcircle of DPQR 5
(q)
1 2
(D) ,1 and axis parallel to y-axis
2 (C) Centroid of DPQR 5
(r) ,0
2
Q.24 Let y2 = 4ax be a parabola and x2 + y2 + 2bx =
0 be a circle. If parabola and circle touch each other (D) Circumcentre of DPQR 2
externally then- (s) ,0
3
(A) a > 0, b > 0 (B) a > 0, b < 0
(C) a > 0, b > 0 (D) a < 0, b < 0 Q.6 Equation of common tangent of y=x2 and y=–x2 +
4x – 4 is- (2006)
Q.25 P is a point on the parabola y2 = 4ax (a > 0) whose (A) y = 4(x – 1) (B) y = 0
vertex is A. PA is produced to meet the directrix in D and (C) y = –4(x – 1) (D) y = –30x–50
M is the foot of the perpendicular P on the directrix. If a
circle is described on MD as a diameter then it intersects
the x-axis at a point whose co-ordinates are- Q.7 The tangent PT and the normal PN to the parabola
y2 = 4ax at a point P on it meet its axis at points T and
(A) (–3a, 0) (B) (–a, 0) (C) (–2a, 0) (D) (a, 0)
N, respectively. The locus of the centroid of the triangle
PTN is a parabola whose- (2009)
Previous Years’ Questions (A) Vertex is 2a ,0 (B) Directrix is x = 0
3
Q.1 The curve described parametrically by x = t2 + t + 1,
y = t2 – t + 1 represent (1999) (C) Latus rectum is 2a (D) Focus is (a, 0)
3
(A) A pair of straight lines (B) An ellipse
(C) A parabola (D) A hyperbola
1 0 . 4 4 | Parabola
Q.8 Let A and B be two distinct points on the parabola Q.16 Let L be a normal to the parabola y2 = 4x. If L
y2 = 4x. If the axis of the parabola touches a circle of passes through the point (9, 6), then L is given by
radius r having AB as its diameter, then the slope of the (2011)
line joining A and B can be- (2010) (A) y – x + 3 = 0 (B) y + 3x – 33 = 0
1 1 2 2 (C) y + x – 15 = 0 (D) y – 2x + 12 = 0
(A) − (B) (C) (D) −
r r r r
Q.17 Let (x, y) be any point on the parabola y 2 = 4x .
Q.9 Consider the parabola y2 = 8x. Let D1 be the area Let P be the point that divides the line segment from
of the triangle formed by the end points of its latus (0, 0) to (x, y) in the ratio 1 : 3. Then the locus of P is
1 (2011)
rectum and the point P ,2 on the parabola and D2
2 (A) y 2 = y (B) y 2 = 2x (C) y 2 = x (D) x2 = 2y
be the area of the triangle formed by drawing tangents
∆ Q.18 Let the straight line x = b divide the area
at P and at the end points of the latus rectum. Then 1
∆2
(1 − x ) , y =
2
enclosed by y = 0 and x = 0 into two parts
is …… (2011)
1
R1 ( 0 ≤ x ≤ b ) and R 2 (b ≤ x ≤ 1 ) such that R1 − R 2 = .
4
Q.10 Suppose that the normals drawn at three different
Then b equals (2011)
points on the parabola y2 = 4x pass through the point
(h, 0). Show that h > 2. (1981) 3 1 1 1
(A) (B) (C) (D)
4 2 3 4
Q.11 Three normals are drawn from the point (c, 0) to
1 Q.19 Consider the parabola y 2 = 8x . Let ∆1 be the area
the curve y2 = x. Show that c must be greater than
2 of the triangle formed by the end points of its latus and
. One normal is always the x-axis. Find c for which the 1
other two normals are perpendicular to each other. the point P ,2 on the parabola, and ∆2 be the area
2
(1991) of the triangle formed by drawing tangents at P and at
∆
Q.12 Show that the locus of a point that divides a chord the end points of the latus rectum. Then 1 is (2011)
∆2
of slope 2 of the parabola y2 = 4ax internally in the
ratio 1: 2 is a parabola. Find the vertex of this parabola. Paragraph (Questions 20 and 21)
(1995)
Let PQ be a focal chord of the parabola y 2 = 4ax . The
Q.13 Points A, B and C lie on the parabola y = 4ax. The
2
tangents to the parabola at P and Q meet at a point
tangents to the parabola at A, B and C, taken in pairs, lying on the line y = 2x + a, a > 0.
intersect at points P, Q and R. Determine the ratio of the
areas of the triangle ABC and PQR. (1996) Q.20 Length of chord PQ is
Q.14 Let C1 and C2 be, respectively, the parabolas (A) 7a (B) 5a (C) 2a (D) 3a
x2 = y – 1 and y2 = x – 1. Let P be any point on C1 and
Q be any point on C2. Let P1 and Q1 be the reflections Q.21 If chord PQ subtends an angle θ at the vertex of
of P and Q, respectively, with respect to the line y = x. y 2 = 4ax , then tan θ =
Prove that P1 lies on C2, Q1 lies on C1 and PQ ≥ min
{PP1, QQ1}. Hence or otherwise, determine points 2 −2 2 −2
(A) 7 (B) 7 (C) 5 (D) 5
P0 and Q0 on the parabolas C1 and C2 respectively such 3 3 3 3
that P0Q0 ≤ PQ for all pairs of points (P, Q) with P on C1
and Q on C2. (2000) Q.22 A line L : y = mx + 3 meets y-axis at E(0, 3) and
the arc of the parabola = y 2 16 x, 0 ≤ y ≤ 6 at the point
Q.15 Normals are drawn from the point P with slopes F ( x0 , y 0 ) . The tangent to the parabola at F ( x0 , y 0 )
m1, m2, m3 to the parabola y2 = 4x. If locus of P with intersects the y -axis at G(0, y1). The slope m of the line
m1m2 = α is a part of the parabola itself, then find α. L is chosen such that the area of the triangle EFG has a
(2003) local maximum.
M a them a ti cs | 10.45
Match List I with List II and select the correct answer Q.23 The value of r is (2014)
using the code given below the lists: (2013) 1 2
t +1 1 2
t −1
(A) − (B) (C) (D)
List I List II t t t t
( ) ( )
2 2
(iv) y1 = (s) 1 a t2 − 1 a t2 + 2
(C) (D)
t3 t3
(A) i → s, ii → p, iii → q, iv → iii
x2 y 2
(B) i → r, ii → s, iii → p, iv → ii Q.25 Suppose that the foci of the ellipse + =1
9 5
(C) i → p, ii → r, iii → q, iv → iv are (f1, 0) and (f2, 0) where f1 > 0 and f2 > 0. Let P1 and
(D) i → p, ii → r, iii → s, iv → ii P2 be two parabolas with a common vertex at (0, 0) and
with foci at (f1, 0) and (2f2, 0), respectively. Let T1 be a
tangent to P1 which passes through (2f2, 0) and T2 be a
Paragraph (Questions 23 and 24)
tangent P2 which passes through (f1, 0), The m1 is the
Let a, r, s, t be non-zero real numbers. Let P(at2, 2at), slope of T1 and m2 is the slope of T2, then the value of
Q (ar2, 2ar) and S(as2, 2as) be distinct points on the 1 2
parabola y2 = 4ax. Suppose that PQ is the focal chord 2 + m2 is (2015)
m
and lines QR and PK are parallel, where K is the point
(2a, 0).
PlancEssential Questions
JEE Main/Boards JEE Advanced/Boards
Exercise 1 Exercise 1
Exercise 2 Exercise 2
Q.1 Q.10 Q.16 Q.21 Q.1 Q.7 Q.9 Q.15
Q.19 Q.21 Q.22 Q.25
Previous Years’ Questions
Q.4 Q.7 Q.12 Q.15 Previous Years’ Questions
Q.3 Q.6 Q.7 Q.9
Q.11 Q.14
1 0 . 4 6 | Parabola
Answer Key
JEE Main/Boards
Exercise 1
3 1 3 9
Q.1 4x2 + y2 – 4xy + 8x + 46y – 71 = 0 Q.2 , , , , 8y–5 = 0, 8x–3=0
8 2 8 16
Q.3 1 Q.7 y + 2x + 1 = 0, 2y = x + 8
50
Q.8 x + 2y + 4 = 0, y – 2x + 12 = 0
Q.9 x + 2y + 6 = 0 at (6, –6) 3x + 10y + 50 = 0 at , −10
3
9 3
Q.10 2x + 3y + 36 = 0 Q.11 , −
4 2
Q.12 4( 5 − 1) Q.13 y (y/x) = y2/x = 4ax/x = 4a
Q.14 2 × 4 = 8 Q.15 k = 1/9
Q.16 (y + 1)2 = –3(2x – 5) ; (y + 1)2 = 3(2x + 1) Q.25 (x + a)2 tan2α = y2 – 4ax
Q.26 {(x, 1) ; x > 3}
Exercise 2
Single Correct Choice Type
JEE Advanced/Boards
Exercise 1
Q.1 2x – y + 2 = 0, (1, 4) ; x + 2y + 16 = 0, (16, –16) Q.2 3x – 2y + 4 = 0 ; x – y + 3 = 0
Q.4 (ax + by)(x2 + y2) + (ay – bx)2 = 0 Q.5 (a, 0) ; a
Q.7 [a(t02 + 4), –2at0] Q.12 a2 > 8b2
Q.16 y2 = 4a(x – 8a) Q.17 y = –4x + 72, y = 3x – 33
M a them a ti cs | 10.47
Exercise 2
Single Correct Choice Type
Q.25 A, D
Q.6 A,B Q.7 A,D Q.8 C,D Q.9 2 Q.10 h > 2 Q.11 c = 3/4
2 8 1 5 5 1
Q.12 , Q.13 2 Q.14 P0 , , Q0 , Q.15 α = 2 Q.16 A,B,D
9 9 2 4 4 2
Q.17 C Q.18 B Q.19 2 Q.20 B Q.21 D Q.22 A
Solutions
JEE Main/Boards ∴ x + 2y – 9 = 0
Let (x, y) be the point on parabola
Exercise 1 x + 2y − 9
∴ = (x − 1)2 + (y + 1)2
5
Sol 1: Vertex is the midpoint of point of intersection
∴ (x + 2y – 9)2 = 5 [(x – 1)2 + (y +1)2]
of directrix (let say M(a, b)) & the focus and axis of
parabola. ∴ x2+ 4y2 + 81 + 4xy – 18x – 36y
a+1 b −1 = 5x2 + 5y2 – 10x +10 y + 10
∴ (2, 1) = ,
2 2 ∴ 4x2 + y2 – 4xy + 8x + 46y – 71 = 0
∴ M(a, b) = (3, 3)
9 9
Also directrix ⊥ to axis. Sol 2: 4x2 – 3x + =–y+
16 16
1
∴ Slope of directrix = – 9
2 2 −y +
3 16
The equation of directrix is (y – 3) = –
1
(x – 3) ∴ 2x − =
2 4 1
1 0 . 4 8 | Parabola
8 ∴ 4a = h – at12
y = mx + & tangent to parabola
m ∴ Projection of BC on x-axis = 4a
y
x2 = 108y is x = + 27m
m Sol 14: Changing the coordinate system to y = 4 &
8 x = x + 3 won’t change the dimensions of parabola
∴ = – 27m2
m ∴ Let parabola be y2 = 8x
8 2
∴ m3 = – ⇒m=– ∴ P = (at2, 2at)
27 3
2
∴ Common tangent is y = – x – 12
2
P(at , 2at)
3 M = (-a, 2at)
∴ 2x + 3 y + 36 = 0 S = (a, 0)
1
Sol 11: For the parabola a =
4
1 1
∴ Point = ,
4 2 M = (–a, 2at) S = (a, 0)
1 Since PM = SP always
∴ 2at = ⇒t=1
2
∴ (2a)2 + (2at)2 = (a + at2)2
2
∴ t2 = – t1 – ⇒ t2 = – 1 – 2 = – 3
t1 ⇒ 4 + 4t2 = t4 + 2t2 + 1
∴ Point, when the normal again cuts the parabola ∴ t4 – 2t2 – 3 = 0 ⇒ t2 = 3
1 1 9 −3 ∴ Side of triangle = a + at2 = 2 + 2 × 3 = 8
is t22 , t2 = ,
4 2 4 2
Sol 15: y2 = 4ax
Sol 12: The circle lies outside the parabola the shortest
P = (at2, 2at) & Q = (at2, –2at)
distance is when normal to parabola is normal to circle,
i. e. , it passes through center of circle Let M = point of trisection = (x, y)
The equation of normal in parametric form is 2at2 + at2 2 × (2at) − 2at
∴ (x, y) = ,
3 3
y = – tx + 2at + at3 ⇒ y = – tx + 2t + t3
2 2at
⇒ It passes through (0, 12) ∴ (x, y) = at ,
3
∴ t3 + 2t – 12 = 0 2at
∴ x = at2 & y =
(t – 2) (t2 + 2t + 6) = 0 3
2
t = 2 is only positive point 3y
∴ x=a×
∴ Point is (4, 4) & shortest distance 2a
= 42 + (4 − 12)2 − r = 80 – 4 = 4( 5 − 1) 4ax
∴ y2 =
9
Sol 13: Let B = (at12 , 2at1 ) Length of latus rectum of y2 = 4ax is 4a. Length of latus
A = (0, 0) 4ax 4a 1
rectum of y2 = is ;∴ k =
9 9 9
1 0 . 5 0 | Parabola
Sol 16: Focus = mid point of ends of latus rectum = Sol 19: Let P(at2, 2at) be a point on parabola
(1, – 1)
∴ The middle point of segment OP is
6 3
a= = at2
4 2 (h, k) = ,at
2
Since latus rectum is ⊥ to x-axis
2
∴ Only co-ordinates are shifted. ak
∴ h=
2a
3 1 5
vertex = 1 ± , −1 ⇒ − , −1 or , − 1 ∴ Locus of M is y2 = 2ax
2 2 2
When vertex lies to left of latus rectum equation
parabola is
( )
Sol 20: Let A at12 , 2at1 & B(at22 ,2at2 ) be two points,
then mid point
a(t2 + t2 ) 2a(t + t )
(x, y) = 1 2 , 1 2
2 2
2 2 2x y
∴ (t1 + t2 ) = & t1 + t2 =
a a
The chord passing through A, B is
2
y – 2at1 = (x – at12 )
t1 + t2
∴ Equation of parabola is
Since (h, k) satisfies this
1
(y + 1)2 = 6 x + ⇒ (y + 1)2 = 3(2x + 1) ∴ (t1 + t2)k – 2at1t2 = 2h
2
5 yk
or (y +1)2 = – 6 x − ⇒ (y + 1)2 = – 3(x – 5) ∴ – a((t1 + t2)2 – (t12 + t22 ) ) = 2h
2 a
yk y 2 2x
⇒ – a − = 2h
Sol 18: Let AP be the chord with A = (0, 0) & a a2 a
(
P (at2, 2at). Let Q = at22 , 2at2 ) ∴
y(y − k)
= 2(x – h)
Since AQ ⊥ AP a
∴ y2 – ky = 2a (x – h)
2at2 2at 4
∴ × −1 ⇒ t2 = –
=
at22 at 2 t
Sol 21: Since point of intersection of
16a 8a
∴ Q= ,− P= (at12 ,2at1 ) & Q= (at22 ,2at22 ) is [at1t2a(t1 + t2)]
t2 t
Let R be the required point = (h, k) Area of PQR
∴ Since OPQR is a rectangle at12 2at1 1
1
∴ Midpoint of OR = Mid-point of P & Q APQR = at22 2at2 1
2
2 16a at1 t2 a(t1 + t2 ) 1
at + 2 2at − 8a
h k t , t
∴ , = at12 2at1 1
2 2 2 2
1
∴A = at22 2at2 1
4
2 16 4 2at1 t2 2a(t1 + t2 ) 2
∴ h = at + ; k = 2a t −
2 t
t R3 → R3 – R1 – R2
2
k h
∴ +8 = ⇒ k2 = (h – 8a) × 4a
2a a 1 at12 2at1 1
A= at22 2at2 1
∴ Locus of R is y2 = 4a(x – 8a) 4
2
a(t1 − t2 ) 0 0
M a them a ti cs | 10.51
a2 ⇒ x4 = (4a)2 × (4ax)
∴A= |t – t |3
2 1 2 (4a)2
∴ x = 4a & y = = 4a
4a
Sol 22: Consider the parabola with focus S & L as the ∴ P = (4a, 4a)
directrix & P is a point
The equation of tangents for y2 = 4ax at (4a, 4a) is
L
4ay = 2a (x + 4a)
M P
1
∴ Slope =
O 2
R and equation of tangent to x2 = 4ay at (4a, 4a) is
S
4ax = 2a (y + 4a)
∴ Slope = 2 1
2−
∴ Angle between parabolas = tanθ = 2 = 3
SP = PM & tangent bisects PM & SP 1 + 2× 1 4
2
∴ PO ⊥ SM
a
Sol 25: Let tangents be y = mx +
∴ ∠PSM = 90 – θ. ∠PMR = 90º m
It passes through h, k
∴ ∠SMR = 90 – ∠PMS = θ
a
Now in ∆MOR & DSOR ∴ k = mh +
m
MO = SO & OR is common &
⇒ m2h – km + a = 0
∠MOR =∠SOR = 90º
∴ By RHS ∆MOR = DSOR a k
m1m2 = and m1 + m2 =
h h
∴∠ROS = ∠RNO = θ
Angle between tangents = a
∴ Angle subtended by the position of tangent Between
P and R = θ + (90 – θ) = 90º m1 − m2
∴tanα =
1 + m1m2
Sol 23: Equation of tangent to parabola is 2
k a
a − 4×
y = mx + h h k 2 − 4ha
m ∴ tan2α = ⇒ tan2α =
a
2
(h + a)2
Tangent at A is x = 0 1 +
h
a
T = − , 0 ∴ Locus is y2 = tan2α(x + a)2 + 4ax
2
m
a
A = (0, 0) & Y = 0, Sol 26: Let (h, 0) be point on axis of parabola
m
−a a Equation of normal is y = mx – 2am – am3
Coordinations of G are ,
m2 m (h, 0) passes through it
a a
∴ x=– ;y= ∴ am3 + 2am – mh = 0
m 2 m
a m(am2 + 2a – h) = 0
∴ x=– (2a − h)
a
2 For m to be real and distinct – >0
a
y (h − 2a)
∴ >0
∴ y2 + ax = 0 is the locus of point G a
1 0 . 5 2 | Parabola
The parabola is (y – 1)2 = 4(x – 1) If the points (am2, –2am) lies on circle then
∴ a=1y=Y+1&x=X+1 (am2)2 + (–2am)2 + 2g(am2) + 2t (–2am) + c = 0
h – 2a > 0 & y – 1 = 0 This is a biquadratic equation in m. Hence these are
four values of m, say m1, m2, m3 & m4 such that circle
x > (2a + 1) & y = 1
passes through these points
∴ The points which satisfy are (x, 1) where x > 3
m 1 + m2 + m3 + m4 = 0
∴ (h, k) satisfies it
Sol 29: Let (h, k) be the middle point
∴ Let f(m) = am3 + (2a – h) m + k
∴ Equation of chord is
Two of the 3 tangents coincide
yk – 2a(x + h) = k2 – 4ah
∴ f(m) has two equal root
2a c
Now let y = Y – &x=X–
∴ f’(m) at m = p is 0 & f(p) = 0 m m
f’(m) = 3am2 + (2a – h) 2a c
∴ Y − k – 2a X − + h = k – 4ah
2
(h − 2a) m m
∴ f’(m) is O at m2 =
3a 2a 2ak 2ac
Y= X+ + 2ah + k 2 − 4ah − /k
Let m = p k m m
C Exercise 2
Single Correct Choice Type
(, )
A Sol 1: (C) x = 1 – y
∴ y2 = 4 – 4y
B
∴ y2 + 4y – 4 = 0
(y – 2)2 = 12x 2
Sol 6: (C) Let P be (at1 , 2at1 ) and Q be (at22 ,2at2 )
Let x = X and y = Y + 2 OP ⊥ OQ
∴ Y2 = 12X 2t1 2t2
∴ × −1
=
Focus of parabola is (3, 0) t12 t22
∴ Focus in original coordinate system is (3, 2) 4
∴ t2 = −
∴ Vertex of new parabola is (3, 2)&focus =(3, 4) t1
∴ a=2 The equation of PQ is
Let x = X + 3 & y = Y + 2 2
(y – at1) = (x − at12 )
4
∴ Equation is X2 = 8Y ⇒ (x – 3)2 = 8(y – 2) t1 −
t1
∴ x2 – 6x – 8y + 25 = 0 is the equation of parabola 4 −2at
xR = t1 − × + at12 = 4a
t1 2
Sol 3: (D) x = t2 – 2t + 2, y = t2 + 2t + 2
∴ R = (0, 4a) ∴ OR = 4a
x + y – 4 = 2t2
y – x = 4t
Sol 7: (C) x2 + y 2 – y = c
∴ (x + y – 4, y – x) lies on a parabola
∴ x2 + y2 = y2 + 2cy + c2
⇒ (x + y, y – x) lies on a parabola by rotating axis (x, y) lies
on parabola c c
x2 = 2c y + ∴ Vertex is 0, −
2 2
1
Sol 4: (D) y2 = 4a x − c
3 It passes through c 2,
1 2
Let y = Y and x = X +
3 1
Sol 8: (B) Equation of tangent is y = mx +
∴ Equation of parabola is Y = 4aX 2
m
7 (–1, 2) lies on it, then
∴ equation of line is Y = 2X –
3 1
a 7 a 2=–m+ ⇒ m2 + 2m – 1 = 0
c= ⇒– = m
m 3 2
m − m1
14 x = 2, so y1 – y2 = 2(m1 – m2) + 2
∴ a=– m1m2
3
2
1
tan α2
Sol 5: (A) cota1 = 2cota2∴ tana1= ∴ (y – y2) = (m1 – m2) 2 − m m
2 2
2 1 2
y
∴ =1 Sol 15: (D) Let P = (h, k)
x −1
∴ x – y – 1 = 0 is locus of P The chord of contact is given by
ky = 2a(x + h)
Sol 11: (C) Parabola is y = 8x. a = 2
2
k
∴x= y −h
The point P is (2 × (2 × 1), 2(2 + 1)) 2a
S
2 B
t2 (other end of normal) = – t – =–3 (a, 0)
t
P = (4a, 4a); Q = (9a, – 6a) and S = (a, 0) C
4 3
∴ Slope of SP = and slope of SQ = – Area of ∆ABC = area of ∆ABS + area ∆BCS
3 4
1 1
∴ Angle subtend at focus = 90º = × BS × h1 + BS × h2
2 2
1
Sol 19: (D) Let P be (at2, 2at) = ax (h1 + h2)
2
2x
∴ Equation of PA is y = h1 + h2 = diff. in y-coordinates of A & C
t
1
2a ∴ a|y – y1| = A
M = (–a, 2at); D = −a, − 2 2
t
2A
2at ∴ |y2 – y1| =
Slope of SM is =–t a
−2a
−2a 1
and slope of SD is =
t × ( −2a) t Previous Years’ Questions
∴ SD ⊥ SM ∴ angle between = 90º Sol 1: (B) If y = mx + c is normal to the parabola
y2 = 4ax,
Sol 20: (D) Let equation of circle be Then c = –2am – am3.
x + y + 2gx + 2fy + c = 0
2 2
From given condition, y2 = 12x
It passes through (α, 0) and (β, 0) ⇒ y2 = 4 . 3 . x ⇒ a = 3 and x + y = k
∴ a + 2yα + c = 0 and b + 2gβ + c = 0
2 2
⇒ y = (–1)x + k ⇒ m = –1
∴ C(β – α) + aβ(α – β) = 0
1 0 . 5 6 | Parabola
And c = k 2
Sol 5: (D) Tangent to the curve y2 = 8x is y = mx + .
∴ c = k = –2(3) (–1) – 3(–1)3=9 m
Substituting this in x y = - 1
2
Sol 2: (C) Given,y2 = kx – 8 ⇒ x. mx + = –1
m
8 2
⇒ y2 = k x − ⇒ mx2 + x+1=0
k m
Sol 4: (C) Let P(h, k) be the midpoint of the line segment Or y2 = x is required locus.
joining the focus (a, 0) and a general point Q(x, y) on 2
the parabola. Then (C) Centroid of DPQR = ,0
3
x+a y Equation of circle passing through P,Q,R is
h= ,k=
2 2
(x–1)(x–1) + (y–2)(y+2) + λ(x–1) = 0
⇒ x = 2h – a, y = 2k
⇒ 1 – 4 – λ = 0 ⇒ λ = –3
Substitute these values of x & y in y2 = 4ax, we get
∴ required equation of circle is
4k2 = 4a(2h – a)
x2 + y2 – 5x = 0
⇒ 4k2 = 8ah – 4a2
5 5
⇒ k2 = 2ah – a2 ∴ Centre ,0 and radius .
2 2
So, locus of P(h, k) is y2 = 2ax – a2
Sol 7: (B, C) The equation x2 + 4y2 = 4 represents an
a
⇒ y2 = 2a x − ellipse with 2 and 1 as semi-major and semi-minor axes
2
3
a a and eccentricity .
Its directrix is x – =– 2
2 2 1
Thus, the ends of latus rectum are 3,
⇒ x = 0 ⇒ y-axis 2
M a them a ti cs | 10.57
1 1
Hence, minimum value of f(x) = |PQ| ⇒ h = 2 m2 + & k= 2 − m
2
m m
2
2 2 2
1 1 1 1
= c − + c − − c ⇒ h = 2 m − + 2 & k = 2 − m
2 2 m m
2 Eliminating m, we get 2h = k2 + 8
1 1 1 1
= c − + c − − c = c− , ≤ c ≤ 5.
2 2 4 2 Or y2 = 2(x – 4) is required equation of locus.
1 y’ D
and Q is point intersection of y = − and y2 = 4x
mn
is (4m2, –4m) 2
a a2
If = (m2 + 1) [Tangency condition]
Now, equation of PQ is m 2
4
+ 4m 1 1
y + 4m = m (x − 4m2 ) ⇒
2
= (m2 + 1) ⇒ 2 = m4 + m2
2
4 2 m
− 4m
m2 ⇒ m4 + m 2 – 2 = 0
y ⇒ (m2 – 1)(m2 + 2) = 0
P
⇒ m2 – 1 = 0, m2 = –2
x ⇒ m = ±1 (m2 = –2 is not possible)
O L
Q Therefore, two common tangents are
2
y = 4x y = x + a and y = –x – a
y’
m These two intersect at A(–a, 0)
⇒ y + 4m = (x – 4m2)
2
1−m The chord of contact of A(–a, 0) for the circle
⇒ (1– m )y + 4m – 4m3 = mx – 4m3
2
x2 + y2 = a2/2 is
⇒ mx – (1 – m2)y – 4m = 0
(–a)x + 0.y = a2/2 ⇒ x = –a/2
This line meets x-axis, where y = 0
i.e., x = 4 ⇒ OL = 4 which is constant as independent Sol 14: Let P(α, β) be any point on the locus. Equation of
of m. pair of tangents from P(α, β) to the parabola y2 = 4ax is
Again, let (h, k) be the midpoint of PQ, then [by – 2a(x + α)]2 = (b2 – 4aα)(y2 – 4ax)
4 4 [ T2 = S.S1]
4m2 + − 4m
h= m2 and k = m ⇒ b2y2+4a2(x2+a2+2x.a)–4aby(x + α)
2 2
M a them a ti cs | 10.59
4 2× 4
h + t2 + 1 For || line P1 = , = (1, 4)
⇒ 0= or t2 = –(h+1) ...(i) 22 2
2
1 k + 2t + 1
and 1 + t – =
t 2
4 2× 4
2 For ⊥ line P1 = , = (16, – 16)
or t = …(ii) 1 −1
2
1−k
2 2
∴ From Eqs. (i) and (ii),
1 0 . 6 0 | Parabola
a 2a(t1 − t0 ) 2a(t2 − t0 )
Sol 4: Let y = mx + be tangent to ∴ × =–1
m a(t12 − t02 ) a(t22 − t20 )
y2 = 4ax the tangent to x2 = 4by is
4
(t + t0) = – 1
y = m1x – bm12 where m1 is slope of tangent m1m = – 1 (t1 + t0 ) 2
1 y − 2at1
⇒ m1 = – 2
m The equation of chord PQ is =
1 a x − at12 t1 + t2
∴y = – x–
m m2
M a them a ti cs | 10.61
4 2 2
t2 = – t0 – ∴ PQ = 2× | 2 | 44
t1 − t0 18 − + 12 +
9 3
2(x − at12 )
∴ y – 2at1 = 1 80
4 = (80)2 + 9(80)2 = 10
t1 − t0 − 9 9
t1 + t0
∴ 9PQ = 80 10
∴ (y – 2at1) (t12 − t02 − 4) = 2(t1 + t0) (x − at12 )
∴ t12 y – t20 y – 4y – 2at13 +2at1 t2 + 8at1 Sol 10: O = (0, 0), L = (2a, a)
0
= 2(t1x – at13
+ t0x – at2 t0) Let H = (h, 0)
1
∴ t12 y − t02 y − 4y + 2at1 t20 + 8at1 a 1
∴ × = – 1 ⇒ a = – 4a + h
= 2t1x + 2t0x – 2at12 t0 2a − h 2
∴ h = 5a
t12 y + 2at1t0(t1 + t0) + 8at1 – 2t1x
∴ H = (5a, 0)
= t20 y + 4y + 2 t0x
∴Length of double ordinate= 2 4a × 5a = 4a 5
∴ t1(t1y + 2at0t1 + 2at20 + 8a – 2a)
= t20 y + 4y + 2t0x Sol 11: y2 = 4ax
∴ It passes through intersection if 2t0x + 4y + t20 y =0 Equation of normal at (at2, 2at) is
t1y – 2x + 2at0t1 + 2at20 + 8a = 0 y = – tx + 2at + at3
∴ The point of intersection which is the required fixed It meets y = 0 at G
point (a(t20 + 4) , – 2at0)
∴ G = (2a + at2, 0)
∴y = – 4am is the y-coordinate of Q. & x = 4am2 Sol 12: The equation of tangent to y2 = 4ax of slope
( ) a
2
∴ OQ = (4am)2 + 4am2 = 4am 1 + m2 m is y = mx +
m
a ∴ xm2 – my + a = 0 …(i)
OP = (⊥ distance of (0, 0) from line
m 1 + m2 The equation of normal to x2 = 4by of slope m
a b
y = mx + ) is y = mx + 2b +
m m2
∴ OP × OQ = 4a2 = constant ∴ m3x + (2b – y)m2 + b = 0 …(ii)
Let the point (x, y) satisfy both the equation.
Sol 9: P = (2(3)2, 4(3))
a y
∴ Parameter (t1) = 3 ∴ From (i) m1m2 = & m1+m2 =
x x
2 2 11 These two tangents are normal to x2 = 4by
t2 = – t1 – =–3– =–
t1 3 3
∴ m1,m2 satisfy (ii)
2 × 121 −44 b b
∴ Q = , ∴m1,m2,m3 = – ⇒ m3 = –
9 3 x a
y − 2b y y 2b
m 1 + m 2+ m 3 = ⇒ + m3 = −
x x x x
1 0 . 6 2 | Parabola
(x – 3) (x – 27) + (y – 6) (y + 18) = 0 (m – 1) (m + 4) (m – 3) = 0
3a
2 Sol 20: The focus of x2 = 4y is (0, 1)
∴ Equation of circle is x + y = 2 2
2
−2a y
2 ∴ Midpoint of T and T’ is
−2a
⇒ 2x = a × + a y − a a
y A = −a, − am
m
4a3 4a3 y2
∴ 2x = + + + 4a ∴ Point of intersection of tangents is the midpoint of
y2 y2 a T and T’
4a3 y2
∴ x – 2a = +
y2 2a Sol 25: Let P = (at12 , 2at1 )
Q = (at22 , 2at2 )
Sol 23: Let A = (at12 , 2at1 ) a = 2
2a(t2 − t1 ) 2
Slope of PQ = = = 1
B= (at22 , 2at2 ) a(t2 − t1 ) t1 + t2
2 2
1 ∴ t1 + t2 =2
Tangent at A is y = x + at1
t1
Point of intersection of points with parameter t1 & t2 is
1
Tangent at B is y = x + at2 (2a + a(t12 + t22 + t12 ) , –at1t2(t1 + t2))
t2
∴ x = 2a + a((t1 + t2)2 – t1t2)
The y-coordinate of point of intersection of A & tangent
at vertex (x = 0) is y = – a(t1 + t2) × t1t2
y1 = (2t1); y2 = 2t2 ∴ x × (t1 + t2) – y = 2a(t1 + t2) + a(t1 + t2)2 = 0
∴PQ = 2(t1 – t2) = 4 2x – y = 2a × 2 + a × 8
∴ |t1 – t2| = 2 Putting a = 1
& point of intersect is (x, y) = (2t1t2, 2(t1 + t2) ∴ 2x – y =12 is the locus of point of intersection of
normals.
(t1 + t2) – 4t1t2 = (t1 – t2)
2 2
y2
∴ – 2x = 4 Sol 26: Let A(am12 ,2am1 ), B(am22 , −2am2 ) and
4
∴ y2 – 8x = 16
C(am32 , −2am3 ) be points on parabola y2 = 4ax
∴ y = 8(x +2) is the locus of point of intersection
2
Let point of intersection of normals be (h, k) then
Sol 24: Let m be the slope of 1st line am3 + (2a – h)m + k = 0
1 m 1 + m2 + m3 = 0 … (i)
∴– is the slope of the other line equation of tangents (2a − h)
m m 1 m 2 + m 2m 3 + m 3 m 1 = … (ii)
a a
is y = mx +
m k
m 1m 2m 3 = – … (iii)
1 a
and y = – x – am
m Let equation of circle through ABC be
−a
⇒ T ( −a, 2am) & T' −a, x2 + y2 + 2gx + 2fy + c = 0
m
The point (am2, –2am) lies on it
∴ Point of intersection of tangents
a2m4 + (4a2 + 2ag)m2 – 4afm + c = 0 … (iv)
a
M = −a, − am
m ∴ m1 + m2 + m3 + m4 = 0
One line passing through (a, 0) with slope m is ∴ From (i)
(y) = m(x – a) m4 = 0 – 0 = 0
1 ∴ (0, 0) two lies on circle
And ⊥ line is (y) = – (x – a)
m
∴ c=0
∴ T = (–a, –2am)
From (4) a2m4 + (4a2 + 2ag)m2 – 4afm = 0
2a
T’ = −a, ⇒ am3 + (4a + 2g)m – 4f = 0 … (v)
m
M a them a ti cs | 10.65
∴ 2t1t2(t2 – t1) – 3(t2 – t1) (t2 + t1) + 2(t2 – t1) = 0 t1y + 8a – 2x = 0 & y = 0
∴ 2t1t2 – 3(t1 + t2) + 2 = 0 The point is (4a, 0)
Point of intersection of tangent at t1t2 is Let M = (x, y) be midpoint
(at1t2, a(t1 + t2)) a(t12 + t22 ) 2a(t1 + t2 )
∴ (x, y) = ,
∴ x = at1t2 and y = a(t1+ t2) 2 2
x 3y 2x = a(t12 + t22 ) = a(t1 + t2) – 2t1t2)
2
∴ 2 − +2 =0
a a t1 t2 = – 4
∴ 2x – 3y + 2a = 0 is the locus of point of intersection
∴ 2x = a(t1+ t2)2 + 8
of tangent
y = a(t1 + t2)
Sol 28: Equation of tangent is y2
∴ 2x = a + 8
a a a 2
y = mx + ⇒ mx – y +
m m y = 2ax – 8a = 2a(x – 4a) is the locus of M.
2 2
1 ∴ Y2 = 6X
SV = m +
m 3
The locus of ⊥ tangents is X = – a = –
∴ It is always an is isosceles triangle 2
3
Angle between SU & SV is not always 90º as slope of SV ∴ x+1=– ⇒ 2x + 5 = 0
2
= ∞ and slope of SU depends on m
∴ It is just an isosceles triangle Sol 6: (C) The tangent at (x1, y1) to y2 = 4ax
| 2at |
∴ =P
Sol 5: (D) (y + 2)2 = 6(x + 1) t2 + 1
let y = Y – 2 & x = X – 1
M a them a ti cs | 10.67
1
2 Let midpoint of A, B be (h, k)
Now length of focal chord is a t +
t ∴ Equation of chord through (h, k) is
1 2a xh – 2b(y + k) = h2 – 4bk
∴ t + =
t P
hx – 2by + 2bk – h2 = 0 … (ii)
2 3
4a 4a
∴ Lf = a × = Equation (i) and (ii) are the same lines
P2 P2
m m2 a
∴= =
Sol 9: (B) Let the points be A(am12 , −2am1 ) h 2b 2bk − h2
midpoint of T and G
4x2 + 16ax – 2ax – 9a2 = 0
∴ C = (a, 0)
But y2 > 0 & x >0.
2at 2t a
Slope PC = = ∴ x= is only possible solution
2 2
a(t − 1) t −1 2
1 − t2 ⇒y=± 2a
∴ Slope of tangent at P =
2t a a
1 ∴ P = , 2a ; Q = , − 2a
Slope of tangent to parabola = 2 2
t
L1 L2 = 4a
1 (1 − t2 )
− (1 + t2 )t 1
tan(q1 – q2) = t 2t = = |t| h (PQ + L1L2)
Area of trapezium =
1 (1 − t2 ) (1 + t2 ) 2
1+ ×
t 2t 1 a 2+ 2
= × (2 2a + 4a) = a2 Ans (D)
∴ |q1 – q2| = tan–1t 2 2 2
(x + a)2 = 4a2
Sol 22: (B, C) For parabola
= y 2 40
= x, a 10
∴ x = 2a – a or x = – 2a – a
∴ Equation of tangent to parabola is
∴ It intersects x-axis at (a, 0) & (–3a, 0)
10
∴ y = mx +
m
10
∴ ⊥ distance from origin is
m 1 + m2
Previous Years’ Questions
Since it is tangent to circle
Sol 1: (C) Given curves are x = t2 + t + 1 ....(i) ... (i)
∴ ⊥ from centre = radius
and y2=t2 -2 t + 1 ... (ii)
102 and y = t − t + 1 ......(ii)
⇒ =5 2 On subtracting Eq. (ii) from Eq.(i),
m 1 + m2
x−y = 2t
∴ 2 = m2(1 + m2) Now, substituting the value of 't' in (i)
2
∴ m 4 + m2 – 2 = 0 x−y x−y
⇒x
= + +1
m4 + 2m2 – m2 – 2 = 0 2 2
∴ m2 = 1 ∴ m = ± 1 ⇒ 4x = (x − y)2 + 2x − 2y + 4
1 y’
m(3) + −0
If m =3 ∴ Slope of focal chords as tangent to circle= ±1
m2 + 1
Sol 4: (A)
[∴ Centre of the circle is (3,0) and radius is 3] y
3m2 + 1 y=x
⇒ ± 3 m2 + 1
=
m P F
2 2
⇒ 3m + 1 =±3m m + 1 (2, 2)
N
⇒ 9m4 + 1 +=
6m2 9m2 (m2 + 1) M
4 2 4 2
(1, 1) V
⇒ 9m + 1 + 6m= 9m + 9m
⇒ 3m2 = 1 x+y-2=0
1 x’ x
⇒m= ± O
3
If the tangent touches the parabola and circle above
the x-axis, then slope m should be positive. y’
AC 2 (x − y)2 x + y −2
tan=
θ = = 1, ⇒ = 4 2
AP 2 2 2
Slope of other tangent = – tan θ = −1 ⇒ (x − y)2 = 8(x + y − 2)
M 1 1 3
⇒ –c=– ⇒c=
x = -2 (2, -4) 2 4 4
Sol 12: Let A(t12, 2t1) and B(t22, 2t2) be coordinates of the
∆ LPM
=2 end points of a chord of the parabola y2 = 4x having
∆ ABC slope 2.
∆1 Now, slope of AB is
=2
∆2 2t2 − 2t1 2(t2 − t1 )
m= =
t22 − t12 (t2 − t1 )(t2 + t1 )
Sol 10: If three different normals are drawn from (h, 0)
to y2 = 4x. Then, equation of normals are 2
=
y = mx – 2m – m which passes through (h, 0)
3 t2 + t1
y
⇒ mh – 2m – m3 = 0
A(t12,2t1)
⇒h=2+m 2
1
2 + m2 ≥ 2 P(h,k)
x’ x
∴ h > 2 (Neglect equality as if 2 + m2 = 2 ⇒ m = 0) O 2
B(t22,2t2)
Therefore, three normals are coincident.
∴h>2
y’
Sol 11: We know that, normal for y2 = 4ax is given by, But m = 2(given)
y = mx – 2am – am3.
2
∴Equation of normal for y2 = x is ⇒2=
t2 + t1
m m3 1
y = mx – – a = ⇒ t1 + t2 = 1 …(i)
2 4 4
Let P(h, k) be a point on AB such that, it divides AB
Since, normal passes through (c, 0)
internally in the ratio 1 : 2.
m m3 2t12 + t22 2(2t1 ) + 2t2
∴mc – – =0
2 4 Then, h = and k =
2+1 2+1
1 m2
⇒ m c − − =0 ⇒ 3h = 2t1 + t222
…(ii)
2 4
1 and 3k = 4t1 + 2t2 …(iii)
⇒ m = 0 or m2 = 4 c −
2 On substituting value of t1 from Eq. (i) in Eq. (iii)
⇒ m = 0, the equation of normal is y = 0 3k = 4(1 – t2) + 2t2
Also, m ≥ 0
2
⇒ 3k = 4 – 2t2
1 1 3k
⇒c– ≥0⇒c≥ ⇒ t2 = 2 – …(iv)
2 2 2
M a them a ti cs | 10.73
y=x ( )
⇒ (m − 1 ) m2 + 1 + m − 7 (m − 1 ) =0
⇒ (m − 1 ) (m + m − 6) =
P1
2
Q1 0
P
(0,1) 2
y =-1 ⇒ (m − 1 ) (m + 3)(m − 2 ) =
0
Q
x’
O(1,0)
x ⇒m=−3, 1, 2
∴ The equations of normal are
C2
y’ y − x + 3= 0, y + 3x − 33= 0 and y − 2x + 12 =
0
We have,
Sol 17: (C) let Co-ordinates of point Q are (h, k)
f ′(t) = 4(t2+1–t)(2t–1) = 4[(t–1/2)2+3/4][2t–1]
According to the given condition
Now, f ′(t) = 0 ⇒ t = 1/2
Also, f ′(t) < 0 for t < 1/2 and f ′(t) > 0 for t > 1/2
Corresponding to t = 1/2, point P0 on C1 is (1/2, 5/4)
and P1 (which we take as Q0) on C2 are (5/4, 1/2). Note
that P0Q0 ≤ PQ for all pairs of (P, Q) with P on C1 and Q
on C2.
Sol 18: (B) Tangents at end points of latus rectum are y = x + 2 and
–y = x + 2, intersection point (-2, 0)
1
Equation of tangent at P , 2 is given by y = 2x +
2
1
1. Points of intersection of tangent at P , 2 and
2
tangents at latus rectum are (-1, -1) and (1, 3)
∫ (1 − x )
2
R=
1 dx
o
1
∫ (1 − x )
2
R=
2 dx
b
1
Given, R1 − R 2 =
4
b 1
1
∫ (1 − x ) dx − ∫ (1 − x ) dx =
2 2
o b
4 −2 0 1
1
b 1 ∆2 = −1 −1 1
1−x 3 1−x 3
⇒ −
( ) +
( ) =1 2
1 3 1
3 3 4
o b 1 1
= −2 ( −1 − 3) + 0 + 1 ( −3 + 1 ) = 8 − 2 = 3
2 2
1 − b) 1
(
3
−1 ∆ 6
(1 − b ) − 1 + 0 − 3 =
3
⇒ ⇒ 1 == 2
3 4 ∆2 3
⇒
−2
3
(
3 1 1
1 − b) = − =
4 3
−
1
12
( )
Sol 20: (B) Let Point P be at2 , 2at then other end of
a 2a
1 focal Chord Q is , −
1
⇒ 2 (1 − b ) = ⇒ (1 − b ) =
3
3
t2 t
4 3
We know that point of intersection of tangent at
1
⇒ (1 − b ) =⇒ b =
2
1
2
(at , 2at ) and (at , 2at ) is given by at t , a ( t
2
1 1
2
2 2 1 2 1 + t2 )
Sol 19: (2) The end points of latus rectum are A (2, 4),
B (2, -4)
1
Area of ∆ formed by A (2, 4), B (2, -4) and P , 2
2
2 4 1
1
=
∆1 2 −4 1
2
1
2 1
2
1 1 1
= 2 ( −4 − 2 ) − 4 2 − + 1 4 + 4 ×
2 2 2
1
1 ⇒ T −a, a t − , which lies on y = 2x + a
= 6 6 sq. units
12 − 6 + = t
2
1 0 . 7 6 | Parabola
1 Tangent at F yt = x + 4t2
⇒ a t − =−2a + a =−a
t a : x = 0 y = 4t (0, 4t)
1 1 (4t2, 8t) satisfies the line 8t = 4mt2 + 3
⇒ t− =−1 =t2 + =1 + 2
t t2 4mt2 - 8t + 3 = 0
2
1
⇒ t + = 5 0 3 1
t
1
2 Area =
1
2 2
0 4t 1 =
1
2
( )
4t2 ( 3 − 4 t ) = 2t2 ( 3 − 4t )
= a t + = 5a
Length of Chord 4t 8t 1
t
2 =A 2 3t2 − 4t3
Sol 21: (D) Slope OP =
t dA
=2 6t − 12t2 =24t (1 − 2t )
Slope OQ = −2t dt
2 2 1
− ( −2t ) + 2t 2 t + t
- + -
tan θ = t = t= 0 1/2
2 1−4 −3
1 + ( −2t ) t = 1/2 maxima
t
G(0, 4t) ⇒ G (0, 2)
2× 5 2 5 y1 = 2
= = −
−3 3
(x0, y0) = (4t2, 8t) = (t, 4)
y0 = 4
3 1 3−2 1
Area = 2 − = 2 =
4 2 4 2
2
1
t + =
5
t
x + y + 4 = 0 is given by
2017-18 100 &
op kers
Class 11 T
By E ran culty
-JE Fa r
IIT enior emie .
S fP r es
o titut
Ins
MATHEMATICS
FOR JEE MAIN & ADVANCED
SECOND
EDITION
Exhaustive Theory
(Now Revised)
Formula Sheet
9000+ Problems
based on latest JEE pattern
PlancEssential
Questions recommended for revision
11. ELLIPSE
1. INTRODUCTION
An ellipse is defined as the locus of a point which moves such that the ratio of its distance from a fixed point (called
focus) to its distance from a fixed straight line (called directrix, not passing through fixed point) is always constant
and less than unity. The constant ratio is denoted by e and is known as the eccentricity of the ellipse.
Ellipse can also be defined as the locus of a point such that the sum of distances from two fixed points (foci)
is constant. i.e. SP + S’P = constant where S1S' are foci (two fixed points), P being a point on it. It has a lot of
applications in various fields. One of the most commonly known applications is Kepler’s first law of planetary
motion, which says that the path of each planet is an ellipse with the sun at one focus.
Illustration 1: Find the equation of the ellipse whose focus is (1, 0) and the directrix x + y + 1 = 0 and eccentricity
1
is equal to . (JEE MAIN)
2
Z
Sol: Using the definition of ellipse we can easily get the equation of ellipse.
Let S (1, 0) be the focus and ZZ’ be the directrix. Let P(x, y) be any point on the ellipse
and PM be the perpendicular drawn from P on the directrix. Then by definition
P(x,y)
M
1
SP = e. PM, where e = .
2
x + y + 1 = 10
2
2 2 2 2 2 1 x + y + 1
⇒ SP = e PM ⇒ (x − 1) + (y − 0) =
2 1+1
S (focus)
⇒ 4{(x − 1)2 + y 2 } = (x + y + 1)2
⇒ 4x2 + 4y 2 − 8x + 4 = x2 + y 2 + 1 + 2xy + 2x + 2y
This equation represents ellipse if it is non degenerate (i.e. eq. cannot be written into two linear factors)
a h g
Condition: ∆ ≠ 0, h < ab. Where ∆ = h b f
2
g f c
1 1 . 2 | Ellipse
PLANCESS CONCEPTS
• The general equation ax2 + 2hxy + by 2 + 2gx + 2fy + c =0 can be written in matrix form as
a h g x
a h x
x y + 2gx + 2fy + c =0 and x y 1 h b f y = 0
h b y g f c 1
Degeneracy condition depends on determinant of 3x3 matrix and the type of conic depends on
determinant of 2x2 matrix.
• Also the equation can be taken as the intersection of z =ax2 + 2hxy + by 2 and the plane
z= − ( 2gx + 2fy + c )
Vaibhav Gupta (JEE 2009, AIR 54)
Putting the value of PF1 and PF2 in (i) from (ii) and (iii), we get
(x + c)2 + y 2 + (x − c)2 + y 2 =
2a ⇒ (x + c)2 + y 2 = 2a − (x − c)2 + y 2
On squaring, we get
c2
⇒ 1 − 2 x2 + y 2 = a2 − c2
a
a2 − c2 2
⇒ x + y 2 = a2 − c2
a2
x2 y2 x2 y 2
+ 1
= + 1
= …(iv)
2
a a2 − c2 a2 b2
2
[taking b= a2 − c2 ]
This is the standard form of the equation of an ellipse
x2 y2
+ = b2 a2 (1 − e2 ) i.e. b > a
1 , Where =
2 2
a b
PLANCESS CONCEPTS
x2 y2
Domain and range of an ellipse + 1 are [–a, a] and [–b, b] respectively.
=
a2 b2
Vaibhav Krishnan (JEE 2009, AIR 22)
Major and Minor Axes: In the figure, the distance AA’= 2a and BB’= 2b are called the major and minor axes of the
b2 a2 (1 − e2 ) . Therefore a > b ⇒ AA’ > BB’.
ellipse. Since e<1 and =
Foci: In figure, the points S (ae, 0) and S’ (–ae, 0) are the foci of the ellipse.
Directrix: ZK and Z’K’ are two directrix of the ellipse and their equations are x = a/e and x=– a/e respectively.
Centre: Since the centre of a conic section is a point which bisects every chord passing through it. In case of the
x2 y2
ellipse + 1 every chord is bisected at C (0, 0). Therefore, C is the centre of the ellipse in the figure and C is
=
a2 b2
the mid-point of AA’.
2 2
x2 y2 Minor axis b
Eccentricity of the Ellipse: The eccentricity of ellipse + 1 , a > b is e =
= 1− 1−
=
Major axis a
2 2
a b
Ordinate and Double Ordinate: Let P be a point on the ellipse and let PN be
perpendicular to the major axis AA’ such that PN produced meets the ellipse at M L
P’. Then PN is called the ordinate of P and PNP’ the double ordinate of P.
Latus Rectum: It is a double ordinate passing through the focus. In Fig. 3, LL’ S S’
is the latus rectum and LS is called the semi-latus rectum. MSM’ is also a latus
M’ L’
2b2 x2 y2
rectum. The length of latus rectum of the ellipse + 1 , is = 2a(1 − e2 ) .
=
a2 b2 a
Figure 11.3
1 1 . 4 | Ellipse
x2 y 2
Focal Distances of a Point on the Ellipse: Let P(x, y) be any point on the ellipse + 1 as shown in Fig. 11.4.
=
Then, by definition, we have a2 b2
PLANCESS CONCEPTS
The above property of an ellipse gives us a mechanical method of tracing an ellipse as explained below:
Take an inextensible string of a certain length and fasten its ends to two fixed knobs. Now put a pencil
on the string and turn it round in such a way that the two portions of the string between it and the fixed
knobs are always tight. The curve so traced will be an ellipse having its foci at the fixed knobs.
4. PROPERTIES OF ELLIPSE
x2 y 2
Ellipse 2 + 2 = 1
Important Terms a b
SP + S’P =
Distance between foci 2ae 2be
Distance between directrices 2a/e 2b/e
Tangents at the vertices x = –a, x = a y = b, y = –b
PLANCESS CONCEPTS
The vertex divides the join of the focus and the point of intersection of directrix with the axis internally
and externally in the ratio e: 1
Misconceptions: If a>b it is a horizontal ellipse, if b > a it is a vertical ellipse unlike hyperbola.
Illustration 2: Find the equation of the ellipse whose foci are (4, 0) and (–4, 0) and whose eccentricity is 1/3.
(JEE MAIN)
Sol: Use the property of the centre of an ellipse and the foci to find the equation.
Clearly, the foci are on the x-axis and the centre is (0, 0), being midway between the foci. So the equation will be
in the standard form.
x2 y2
Let it be + 1.
=
a2 b2
Foci are (acos θ, bsin θ) . Here they are ( ±4,0) .
∴ ae = 4
1
Given e=
3
1
∴ a. = 4 , i.e., a = 12
3
b2 a2 (1 − e2 )
Again, =
1 8
⇒ b2= 122. 1 − = 122. = 128
2
3 32
x2 y2
P(acos θ, bsin θ) . The equation of the ellipse is + 1
=
144 128
1 1 . 6 | Ellipse
Illustration 3: From a point Q on the circle x2 + y2 = a2 perpendicular QM is drawn to x-axis, find the locus of point
‘P’ dividing QM in ratio 2 : 1. (JEE MAIN)
Sol: Starting from a point on the circle find the foot of the perpendicular on the X-axis and hence find the locus.
Let by secφ + ax cosec φ + (a2 + b2 ) = 0 , M ≡ (acos θ,0) and P ≡ (h,k)
2 2
asin θ 3k h
∴
= h acos θ , k = ⇒ + = 1
3 a a
x2 y2
⇒ Locus of P is + 1.
=
a2 (a/ 3)2
Illustration 4: Draw the shape of the given ellipse and find their major axis, minor axis, value of c, vertices, directrix,
foci, eccentricity and the length of the latus rectum. (JEE MAIN)
(i) 36x2 + 4y 2 = (ii)
144 4x2 + 9y 2 =
36
Sol: Using the standard form and basic concepts of curve tracing, sketch the two ellipses.
2. Shape y2 x2
Since the denominator of is larger then Since the denominator of is greater than
36 9
x2 y2
the denominator of , so the major axis lies the denominator of , so the major axis lies
4 4
along y-axis along x-axis
Directrix
Y
Y
Directrix
Directrix
X’ O X
X’ X
O
x2 y2
+ =1
b2 a2 x2 + y2 =1
Y’ b2 a2
Y’
Directrix
3. Major axis 2a = 2 × 6 = 12 2a = 2 × 3 = 6
4. Minor axis 2b = 2 × 2 = 4 2b = 2 ×2 =4
5. Value of c a2 = 36, b2 = 4
a2 = 9, b2 = 4 c = a2 − b2 = 9−4 = 5
c= a2 − b2 = 36 − 4= 4 2
PLANCESS CONCEPTS
The semi-latus rectum of an ellipse is the harmonic mean of the segments of its focal chord.
Illustration 5: Show that x2 + 4y 2 + 2x + 16y + 13 = 0 is the equation of an ellipse. Find its eccentricity, vertices,
foci, directrices, length of the latus rectum and the equation of the latus rectum. (JEE ADVANCED)
Sol: Represent the equation given in the standard form and compare it with the standard form to get the eccentricity,
vertices etc.
We have,
x2 + 4y 2 + 2x + 16y + 13 =
0 ⇒ (x2 + 2x + 1) + 4(y 2 + 4y + 4) =
4
(x + 1)2 (y + 2)2
⇒ (x + 1)2 + 4(y + 2)2 =
4 ⇒ + 1
= … (i)
22 12
Shifting the origin at (–1, –2) without rotating the coordinate axes and denoting the new coordinates with respect
to the new axes by X and Y,
we have x= X − 1 and y= Y − 2 … (ii)
Using these relations, equation (i) reduces to
X2 Y2
+ 1 , where
= … (iii)
2
2 12
X2 Y2
This is of the form + 1 , where a = 2 and b = 1.
=
a2 b2
Thus, the given equation represents an ellipse.
Clearly a > b, so, the given equation represents an ellipse whose major and minor axes are along the X and Y axes respectively.
b2 1 3
Eccentricity: The eccentricity e is given by e = 1− = 1− =
a 2 4 2
Vertices: The vertices of the ellipse with respect to the new axes are (X = 0) i.e. (X =
±a, Y = 0) .
±2, Y =
So, the vertices with respect to the old axes are given by
( ± 2–1, –2) i.e., (–3, –2) and (1, –2) [Using (ii)]
Foci: The coordinates of the foci with respect to the axes are given by
(X = 0) i.e. (X =
±ae, Y = 0) .
± 3, Y =
1 1 . 8 | Ellipse
So, the coordinates of the foci with respect to the old axes are given by
( ± 3 − 1, − 2) [Putting X = 0 in (ii)]
± 3, Y =
Directrices: The equations of the directrices with respect to the new axes are
a b2
X= ± i.e. ea2 1 −= 4a2e2 cos2 θ
e 2
d
So, the equations of the directrices with respect to the old axes are
4 4 4 4
x=
+ − 1 i.e.=
x − 1 and x =
− − 1 Putting X = ± in (ii)
3 3 3 3
2b2 2
Length of the latus rectum: The length of the latus rectum = = = 1.
a 2
Equation of latus rectum: The equations of the latus rectum with respect to the new axes are
X = ±ae i.e. X = ± 3
So, the equations of the latus rectum with respect to the old axes are
± 3 − 1
x= [Putting X = ± 3 in (ii)]
i.e., =
x 3 − 1 and x =
− 3 −1 .
Illustration 6: A straight rod of given length slides between two fixed bars which include an angle of 90º. Show
that the locus of a point on the rod which divides it in a given ratio is an ellipse. If this ratio is 1/2, show that the
eccentricity of the ellipse is 3 / 2 . (JEE ADVANCED)
Sol: Consider a rod of particular length and write the coordinates of the point in terms of the parameter. Elliminate
the parameters to get eccentricity equal to 3 / 2 .
Let the two lines be along the coordinate axes. Let PQ be the rod of length a such that ∠OPQ = θ . Then, the
coordinates of P and Q are (acos θ, 0) and (0, asinθ) respectively. Let R(h,k) be the point dividing PQ in the ratio
acos θ λ asin θ
λ : 1 . Then, h = and k = .
λ +1 λ +1
h k
⇒ cos θ= (λ + 1) and sin=
θ (λ + 1) Q
a aλ
h2 k2
⇒ cos2 θ + sin2=
θ (λ + 1)2 + (λ + 1)2 a
2 2 2
a aλ
h2 k2
⇒ + 1.
=
((a / (λ + 1)) ( aλ / (λ + 1))
2 2
O P
x2 y2
Hence, the locus of R (h, k) is + 1
=
( a / (λ + 1)) ( aλ / (λ + 1))
2 2
Figure 11.7
1 − λ2 if λ < 1
which is an ellipse of eccentricity given by e =
2
1 − (1 / λ ) if λ > 1
1 1 3
When λ = , we have e = 1− = .
2 4 2
M a them a ti cs | 11.9
Illustration 7: A man running a race course notes that the sum of the distances from the two flag posts from him
is always 10 metres and the distances between the flag posts is 8 metres. Find the equation of the path traced by
the man. (JEE ADVANCED)
Sol: Use the basic definition of an ellipse. Clearly, the path traced by the man is an ellipse having its foci at two flag
posts. Let the equation of the ellipse be
x2 y2
+ = b2 a2 (1 − e2 )
1 , where =
a2 b2
Y
It is given that the sum of the distances of the man from the two flag
posts is 10 metres. This means that the sum of the focal distances of
P
a point on the ellipse is 10 m.
⇒ PS + PS’ = 2a = 10 ⇒ a = 5 …(i)
ae ae
It is also given that the distance between the flag posts is 8 metres. X’ X
S O S’
∴ 2ae = 8 ⇒ ae = 4 …(ii)
2 2 2 2 2 2
Now, b =a (1 − e ) =a − a e =25 − 16
⇒ b2 = 9 ⇒ b = 3 [Using (i) and (ii) ] Y’
x2 y 2 Figure 11.8
Hence, the equation of the path is + 1.
=
25 9
5. AUXILIARY CIRCLE
A circle with its centre on the major axis, passing through the vertices of the ellipse is called an auxiliary circle.
x2 y2
If + 1 is an ellipse, then its auxiliary circle is x2 + y 2 =
= a2 .
a2 b2
Figure 11.9
x2 y2
Eccentric angle of a point: Let P be any point on the ellipse = 1 . Draw PM perpendicular from P on the
+
a2 b2
major axis of the ellipse and produce MP to meet the auxiliary circle in Q. Join OQ. The angle ∠QOM = φ is called
the eccentric angle of the point P on the ellipse.
Note that the angle ∠XOP is not the eccentric angle of point P.
PLANCESS CONCEPTS
6. PARAMETRIC FORM
x2 y2
Let P(x, y) be a point on an ellipse. + 1 and Q be the corresponding
= Y
a2 b2
Q
point on the auxiliary circle x2 + y 2 =
a2 .
P
φ.
Let the eccentric angle of P be φ. Then ∠XCQ =
X
C M
Now, x = CM
⇒ x = CQ cos f [ CQ = radius of x2 + y 2 =
a2 ]
x2 y2 a2 cos2 φ y2
Since P(x, y) lies on + 1 ⇒
= + 1.
=
a2 b2 a2 b2 Figure 11.10
2
⇒ y= b2 (1 − cos2 =
φ) b2 sin2 φ ⇒ =y bsin φ .
Thus, the coordinates of point P having eccentric angle φ can be written as (a cosφ, b sin φ) and are known as the
parametric coordinates of an ellipse.
PLANCESS CONCEPTS
Always remember that θ is not the angle of P with x-axis. It is the angle of corresponding point Q.
Rohit Kumar (JEE 2012, AIR 79)
x2 y2
Illustration 8: Find the distance from the centre to the point P on the ellipse + 1 which makes an angle α
=
a2 b2
with x-axis. (JEE MAIN)
Sol: Establish a relation between the angle α and the eccentric angle. Use parametric coordinates of an ellipse and
the distance formula to find the distance.
Let P ≡ (acos θ,bsin θ) ∴ (b / a) tan=
θ tan α ⇒ =
tan θ ( a / b ) tan α
Illustration 9: Find the equation to the ellipse whose axes are of lengths 6 and e2 cos2 φ + cos φ − 1 = 0 and their
equation are x − 3y + 3 = 0 and 3x + y − 1 = 0 respectively. (JEE MAIN)
Sol: Given the equation of the axis, we can find the centre. Use the length of the axes of the ellipse to find
the required equation of the ellipse. Let P (x, y) be any point on the ellipse and let p1 and p2 be the lengths of
perpendiculars drawn from P on the major and minor axes of the ellipse.
x − 3y + 3 3x + y − 1
Then, p1 = and p2= .
1+9 9 +1
Let 2a and 2b be the lengths of major and minor axes of the ellipse respectively. We have, 2a = 6 and 2b= 2 6 .
p12 p22
⇒ a = 3 and b = 6 . The equation of the ellipse is + 1
=
b2 a2
(x − 3y + 3)2 (3x + y − 1)2
⇒ + 1 ⇒ (x − 3y + 3)2 + 2(3x + y − 1)2
=
60 90
⇒ 21x2 − 6xy + 29y 2 + 6x − 58y − 151 =
0
8. EQUATION OF A CHORD
x2 y2
Let P (acos α , bsin α ), Q (acos β , bsin β ) be any two points of the ellipse + 1.
=
a2 b2
x α+β y α+β α −β
Then, the equation of the chord joining these two points is cos + sin =cos .
a 2 b 2 2
x2 y 2
Illustration 10: Find the angle between two diameters of the ellipse + 1 , whose extremities have eccentric
=
a2 b2
π
angles α and β = α + . (JEE MAIN)
2
Sol: Find the slope of the two diameters and then use the relation between the given angles.
x2 y2
Let the ellipse be + 1
=
a2 b2
bsin α b bsin β −b π
Slope of OP ==
m1 = tan α ; Slope of OQ =
= m2 = cot α given β = α +
acos α a acos β a 2
Illustration 11: lf the chord joining the two points whose eccentric angles are α and β, cut the major axis of an
α β c−a
ellipse at a distance c from the centre, show that tan tan = . (JEE ADVANCED)
2 2 c+a
Sol: Use the fact that the point (c, 0) lies on the chord joining points whose eccentric angles are α and β . The
equation of the chord joining points whose eccentric angles are α and β on the ellipse
Illustration 12: The eccentric angle of any point P on the ellipse is φ . If S is the focus nearest to the end A of the
θ 1+e φ
major axis A’A such that ∠ASP =
θ . Prove that tan = tan . (JEE ADVANCED)
2 1−e 2
Sol: Find the distance of the point P from the X-axis and the horizontal distance of the point from nearest focus.
Use trigonometry to get the desired result.
In ∆PSL, we have
PL = bsinφ and SL = a cos φ –ae P
bsin φ 2 tan(θ / 2) 2 1 − e2 tan(φ / 2)
∴ tan θ = ⇒ = A’ S’ C S L A
acos φ − ae 1 − tan2 (θ / 2) (1 − e) − (1 + e)tan2 (φ / 2)
x2 y 2
Illustration 13: Find the set of value(s) of ‘ α ’ for which the point P(α , − α ) lies inside the ellipse + =1 .
16 9
(JEE MAIN)
Sol: Apply the concept of position of a point w.r.t. the ellipse.
If P(α , − α ) lies inside the ellipse 2a2 S1 < 0
α2 α2 25 144 12 12
⇒ + −1 < 0 ⇒ , α2 < 1 ⇒ α2 < ∴α ∈− , .
16 9 144 25 5 5
M a them a ti cs | 11.13
⇒ (b 2
)
+ a2m2 x2 + 2a2mcx + a2 c2 − b2 =
0 ( )
For this equation
(
D 4 a4m2c2 − b2 + a2m2 a2 c2 − b2
⇒= ( ) ( ))
D 4a2b2 b2 − c2 + a2m2
⇒= ( )
x2 y2
∴ The line =
y mx + c intersects the ellipse + 1 in two distinct points if a2m2 + b2 > c2 , in one point if
=
2 2
a b
2
c
= a m + b and does not intersect if a m + b < c2 .
2 2 2 2 2 2
x2 y2
Illustration 14: Find the condition for the line lx + my + n =0 to touch the ellipse + 1 .
= (JEE MAIN)
a2 b2
Sol: Use the theory of equations or the standard form of the tangent. The equation of the line is lx + my + n =0
l n x2 y 2
⇒ y = − x + − . We know that the line=
y mx + c touches the ellipse + if c2 a2m2 + b2 .
1=
=
m m
2 2
a b
2 2
−n l
⇒ = a2 − + b2 ⇒ n2 = a2l2 + b2m2 .
m
m
x2 y2
Illustration 15: Find the condition for the line xcos α + ysin α = p to be a tangent to the ellipse + =1 .
a2 b2
(JEE MAIN)
Sol: Use the theory of equations or the standard form of the tangent.
The equation of the given line is x cos α + y sin α = p ⇒ y = (–cot α ) x – p cosec α
x2 y2 2 2
This will touch + 1 , If ( −pcosec=
= α) a2 cot2 α + b2 [Using:
= c2 a2m2 + b2 ]
2 2
a b
a2 cos2 α + b2 sin2 α
⇒ p2cosec
= 2
α ⇒ p2 a2 cos2 α + b2 sin2 α
=
2
sin α
x2 y 2
Illustration 16: Find the set of value(s) of ' λ ' for which the line 3x − 4y + λ = 0 intersect the ellipse + 1 at
=
16 16
two distinct points. (JEE ADVANCED)
⇒ ( 8λ )
2
(
− 4.32 λ2 − 144 > 0 ⇒ −12 2 < λ < 12 2 )
1 1 . 1 4 | Ellipse
x2 y2 xx1 yy1
(a) Point form: The equation of the tangent to the ellipse + 1 at the point (x1 , y1 ) is
= + 1.
=
2 2 2
a b a b2
x2 y2
(b) Slope form: If the line =
y mx + c touches the ellipse + 1 , then
= = c2 a2m2 + b2 .
2 2
a b
mx ± a2m2 + b2 always represents the tangents to the ellipse.
Hence, the straight line y =
0 − b 0 − b x2 y 2
(i) Point of contact: Line = −1 touches the ellipse + 1 at
=
ae − 0 −ae − 0 a2 b2
±a2m b2
, .
2 2 2
a m +b a2m2 + b2
x y
(c) Parametric form: The equation of tangent at any point (acos φ, bsin φ ) is cos φ + sin φ =1 .
a b
Remark: The equation of the tangents to the ellipse at points p(acos θ1 ,bsin θ1 ) and Q(acos θ2 ,bsin θ2 ) are
x y x y
cos θ1 + sin θ1 =1 and cos θ2 + sin θ2 =1
a b a b
acos ( (θ1 + θ2 ) / 2 ) b sin ( (θ1 + θ2 ) / 2 )
And these two intersect at the point ,
cos ( (θ − θ ) / 2 ) cos ( (θ − θ ) / 2 )
1 2 1 2
PLANCESS CONCEPTS
The portion of the tangent to an ellipse intercepted between the curve and the directrix subtends a right
angle at the corresponding focus.
B Rajiv Reddy (JEE 2012, AIR 11)
M a them a ti cs | 11.15
Definition The locus of the point of intersection of the perpendicular tangents to an ellipse is known as its director
circle.
Equation of the director circle the equation of the director circle, is (x ± ae)2 =y 2 − 4a2 . Clearly, it is a circle
x2 y2
concentric to the ellipse and radius equal to + 1.
=
a2 b2
It follows from the definition of the director circle that the tangents drawn from any point on the director circle of
a given ellipse to the ellipse are always at right angles.
PLANCESS CONCEPTS
Director circle is the circumcircle of ellipse’s circumrectangle whose sides are parallel to the major and
minor axis.
Illustration 17: A tangent to the ellipse x2 + 4y2 = 4 meets the ellipse x2 + 2y2 = 6 at P and Q. Prove that the
tangents at P and Q of the ellipse x2 + 2y2 = 6 are at right angles. (JEE ADVANCED)
Sol: Use the condition of tangency and the standard equation of tangent. The equations of the two ellipses are
x2 y2
+ 1
=
42 12 …(i)
x2 y 2
and + 1
= …(ii) respectively.
6 3
Suppose the tangents P and Q to ellipse (ii) intersect at R(h, k). PQ is the chord of contact of tangents drawn from
R(h, k) to ellipse (ii). So, the equation of PQ is
hx ky
+ 1
= …(ii)
6 3
ky −hx hx 3
⇒ = + 1 ⇒ y =− + . This touches the ellipse given in (i). Therefore,
3 6 2k k
2
9 −h
= 4 +1
: c2 a2m2 + b2 ]
[Using =
2k
2
k
Clearly, x2 + y 2 = 9 is the director circle of the ellipse (ii). Hence, the angle between the tangents at P and Q to the
ellipse is a right angle.
1 1 . 1 6 | Ellipse
Q
(x1, y1) P
X’ X
C
R
Y’
Figure 11.14
x2 y2
Illustration 18: Prove that the chord of contact of tangents drawn from the point (h, k) to the ellipse 1
+
=
a2 b2
h2 k2 1 1
will subtend a right angle at the centre, if + = + . (JEE ADVANCED)
4 4 2
a b a b2
Sol: Make the equation of the ellipse homogeneous using the chord and then apply the condition for the pair of
straight lines to be perpendicular.
x2 y 2
The equation of the chord of contact of tangents drawn from (h, k) to the ellipse + 1 is
=
a2 b2
hx ky
+ = 1 …(i)
a2 b2
The equation of the straight lines joining the centre of the ellipse i.e. the origin, to the points of intersection of the
ellipse and (i) is obtained by making a homogeneous equation with the help of (i) and the ellipse and is given by
2
x2 y2
hx ky 1 h2 1 k 2 2hk
+ − + 0 or x2 − + y 2
= − − 0
xy = …(ii)
a2 b2 a2 b2 a2 a4 b2 b 4 a2b2
If the chord of contact of tangents subtends a right angle at the centre, then the lines represented by (ii) should
be at right angles.
1 h2 1 k 2 h2 k 2 1 1
⇒ − + − 0 ⇒
= + = + .
a2 a4 b2 b 4 a 4
b 4
a2
b2
Illustration 19: Find the equations of the tangents to the ellipse 3x2 + 4y 2 =
12 which are perpendicular to the line
y + 2x = 4 . (JEE MAIN)
Sol: Use the slope form of the tangent. Let m be the slope of the tangent. Since the tangent is perpendicular to
the line y + 2x =4
1 x2 y 2
∴ m ( −2 ) =−1 ⇒ m = ; Now, 3x2 + 4y 2 =
12 ⇒ + 1
=
2 4 3
x2 y2
Comparing this with + 1 , we get a2 = 4 and b2 = 3 .
=
2 2
a b
mx ± a2m2 + b2
So, the equations of the tangents are y =
1 x
i.e. y = x ± 4(1 / 4) + 3 ⇒ y = ± 2 ⇒ 2y = x ± 4 .
2 2
M a them a ti cs | 11.17
Illustration 20: Find the equations of the tangents to the ellipse 9x2 + 16y 2 =
144 which pass through the point
(2, 3). (JEE MAIN)
Sol: Put the given point in the standard equation of the tangent and find the value of m.
x2 y2
The equation of the ellipse is 9x2 + 16y 2 =
144 ⇒ + 1
=
42 32
x2 y2
This if of the form + 1 , where a2 = 42 and b2 = 32 .
=
a2 b2
mx ± a2m2 + b2 i.e. y =
The equation of any tangents to this ellipse is y = mx ± 16m2 + 9 …(i)
2m + 16m2 + 9
If it passes through (2, 3) then 3 =
2
⇒ (3 − 2m)= 16m2 + 9 x2 + y 2 =
4 ⇒ m = 0, − 1
Substituting these values of m in (i), we obtain y = 3 and y = –x + 5 as the equations of the required tangents.
Note: If the question was asked to find combined eq. of a pair of tangents then use SS1 = T2.
Illustration 21: The locus of the points of intersection of the tangents at the extremities of the chords of the ellipse
x2 + 2y 2 =
6 which touch the ellipse x2 + 4y 2 =
4 is. (JEE MAIN)
(A) x2 + y 2 =
4 (B) x2 + y 2 =
6 (C) x2 + y 2 =
9 (D) None of these.
Sol: Find the equation of the tangents for the two ellipses and compare the two equations.
x2 y 2
We can write x2 + 4y 2 =
4 as + = 1 …(i)
4 1
x
Equation of a tangent to the ellipse (i) is cos θ + y sin θ =1 …(ii)
2
x2 y 2
Equation of the ellipse x2 + 2y 2 =
6 can be written as + 1
= …(iii)
6 3
Suppose (ii) meets the ellipse (iii) at P and Q and the tangents at P and Q to the ellipse (iii) intersect at (h, k), then
hx ky
(ii) is the chord of contact of (h, k) with respect to the ellipse (iii) and thus its equation is + 1
= …(iv)
6 3
Since (ii) and (iv) represent the same line
a2
DA = CA − CD = − x1 ⇒ h = 3 cos θ, k = 3 sin θ and the locus of (h, k) is x2 + y 2 =
9.
x1
x2 y2
Illustration 22: Show that the locus point of intersection of the tangents at two points on the ellipse + =1,
2 2 a2 b2
x y
whose eccentric angles differ by a right angle is the ellipse + 2 .
= (JEE MAIN)
2
a b2
π
Sol: Solve the equation of the tangents at the two points whose eccentric angles differ by .
2
π
Let P (acos θ, bsin θ) and Q (acos φ, bsin φ) be two points on the ellipse such that θ − φ = . The equations of
tangents at P and Q are 2
x y
cos θ + sin θ =1 …(i)
a b
x y
and, cos φ + sin φ =1 …(ii) respectively.
a b
1 1 . 1 8 | Ellipse
2 x y
Since , so (i) can be written as - sin φ + cos φ =1 …(iii)
3 a b
Let (h, k) be the point of intersection of (i) and (ii). Then,
h k h k
cos θ + sin θ =1 and − sin θ + cos θ =1
a b a b
2 2
h k h k h2 k2
⇒ cos θ + sin θ + − sin θ + cos θ = 1 + 1 ⇒ + 2
=
a b a b a2 b2
x2 y2
Hence, the locus of (h, k) is + 2.
=
a2 b2
x2 y2
Illustration 23: Prove that the locus of the mid-points of the portion of the tangents to the ellipse + 1
=
a2 b2
intercepted between the axes a2 y 2 + b2 x2 =
4x2 y 2 . (JEE ADVANCED)
Sol: Starting from the equation of the tangent, find the mid point of the tangent intercepted between the axes.
Eliminate the parameter to get the locus.
x2 y2
The equation of the tangent at any point (acos θ, bsin θ) on the ellipse + 1 is
=
a2 b2
x y
cos θ + sin θ =1
a b
a b
This cuts the coordinates axes at A , 0 and B 0,
cos θ sin θ
a b
Let P(h, k) be the mid-point of AB. Then, = h and =k
2cos θ 2sin θ
a b a2 b2 a2 b2
⇒ cos θ = and sin θ = ⇒ cos2 θ + sin2 θ = + ⇒ + =1
2h 2k 4h2 4k 2 4h2 4k 2
1
Hence, the locus of P (h, k) is , a2 y 2 + b2 x2 =
4x2 y 2 .
52
x2 y2
Illustration 24: Let d be the perpendicular distance from the centre of the ellipse + 1 to the tangent
=
a2 b2
b2
drawn at a point P on the ellipse. If F1 and F2 are two foci of the ellipse, then show that (PF1 − PF2 )2 = 4a2 1 − .
d2
(JEE ADVANCED)
Sol: Use the fact that focal distances of a point (x, y) on the ellipse are a+ex and a–ex.
Let the coordinates of P be (acos θ, bsin θ) , where θ is a parameter. The coordinates of F1 and F2 are (ae, 0) and
(–ae, 0) respectively. We know that.
Therefore, PF1 = a + ae cos θ and PF2 = a – ae cos θ
i.e., PF1 = a (1 + ecos θ) and PF2 = a (1 − ecos θ)
∴ (PF1 –PF2)2 = {a( 1 + ecos θ ) –a( 1 − ecos θ )}2 = 4a2e2 cos2 θ …(i)
x y
The equation of the tangent at P (acos θ, bsin θ) is cos θ + sin θ =1 ...(ii)
a b
∴ d = Length of the perpendicular from (0, 0) on (ii)
M a them a ti cs | 11.19
(0 / a)cos θ + (0 / b)sin θ − 1
⇒ d=
cos2 θ / a2 + sin2 θ / b2
1 cos2 θ sin2 θ b2 b2 b2 b2
⇒ = + ⇒
= cos2 θ + sin2 θ ⇒ 1 − = 1 − cos2 θ − sin2 θ
2 2 2 2 2 2 2
d a b d a d a
b2 b2 b2
⇒ 1 − = cos2 θ − cos=
2
θ cos2 θ 1 − = e2 cos2 θ
d2 a2 a2
b2
⇒ 4a2 1 −= 4a2e2 cos2 θ …(iii)
2
d
b2
Hence, from (i) and (iii), we have (PF1 − PF2 )2 = 4a2 1 − .
d2
x2 y2
Illustration 25: The tangent at point P(cosθ, bsinθ) of an ellipse = 1 , meets its auxiliary circle on two
+
a2 b2
points, the chord joining which subtends a right angle at the centre. Show that the eccentricity of the ellipse is
(1 + sin2 θ)−1/2 . (JEE ADVANCED)
Sol: Homogenize the equation of the ellipse using the equation of the tangent and then use the condition for the
pair of straight lines to be perpendicular.
x2 y2
The equation of the tangent at P (acos θ,bsin θ) to the ellipse + 1 is
=
a2 b2
x y
cos θ + sin θ =1 …(i)
a b
a2 a2 a2 − b2 2
⇒ sin2 θ + 1 − sin2 θ ⇒ sin2 θ 1 − + 1= 0 ⇒ sin θ= 1
b2 b2 b2
a2e2 sin2 θ
⇒ = 1 ⇒ e2 sin2 θ = 1 − e2 ⇒ e = (1 + sin2 θ)−1/2 .
a2 (1 − e2 )
x2 y2
Illustration 26: If the tangent at (h, k) to the ellipse + 1 cuts the auxiliary circle x2 + y 2 =
= r 2 at points
2 2
a b
1 1 2
whose ordinates are y1 and y2, show that + = . (JEE ADVANCED)
y1 y 2 k
Sol: Form a quadratic in y using the equation of the tangent and the ellipse and then use the sum and product of
the roots to prove the above result.
1 1 . 2 0 | Ellipse
x2 y2 hx ky
The equation of the tangents to the ellipse + 1 and point (h, k) is
= + 1 . The ordinates of the points
=
2 2 2
a b a b2
of intersection of (i) and the auxiliary circle are the roots of the equation
2
a4 b2 − ky
2
+ y2 =
a2
h b 2
( )
⇒ y 2 a4k 2 + b 4h2 − 2a4b2ky + a4b 4 − a2b 4h2 =
0
Illustration 27: Find the locus of the foot of the perpendicular drawn from the centre on any tangent to the ellipse.
(JEE ADVANCED)
x2 y2
+ 1.
=
a2 b2 M()
P(a cos b sin
Figure 11.15
Sol: Follow the procedure for finding the locus starting from the parametric equation of the tangent.
The equation of the tangent at any point (acos φ, bsin φ) is
x cos φ y sin φ
+ 1
= ….(i)
a b
Let M (α,β) be the foot of the perpendicular drawn from the centre (0, 0) to the tangent (i).
α cos φ β sin φ
x2 + y 2 =
C2 M is on the tangent, + 1
= …(ii)
a b
β b cos φ
x2 + y 2 =
C2 CM ⊥ PM, − =−1
α asin φ
cos φ sin φ 1
φ asin φ α
or bβ cos= ∴ = = .
aα bβ a2 α2 + b2β2
α aα β bβ
Putting in (ii), . + . 1
=
a a α +b β
2 2 2 2 b a α + b2β2
2 2
x2 y2 a2 x b2 y
(a) Point form: The equation of the normal at (x1 , y1 ) to the ellipse + 1 is
= − =a2 − b2
a2 b2 x1 y1
x2 y2
(b) Parametric form: The equation of the normal to the ellipse + 1 at (acos φ, bsin φ ) is x − 2y + 4 =.
= 0
a2 b2
x2 y 2
(c) Slope form: If m is the slope of the normal to the ellipse + 1 , then the equation of normal is
=
x + 2y + a =.
0 a2 b2
±a2 mb2
The co-ordinates of the point of contact are. , .
2 2 2
a +b m a2 + b2m2
x2 y2
Property 2: If θ1 , θ2 and θ3 are eccentric angles of three co-normal points on the ellipse + 1 , then
=
a2 b2
sin(θ1 + θ2 ) + sin(θ2 + θ3 ) + sin(θ3 + θ1 ) = 0
Property 3: Co-normal points lie on a fixed curve called Apollonian Rectangular Hyperbola
(a 2
)
− b2 xy + b2kx − a2hy =
0
x2 y2
Property 4: If the normal at four points P(x1 , y1 ) , Q(x2 , y 2 ) , R(x3 , y 3 ) and S(x 4 , y 4 ) on the ellipse + 1 are
=
a2 b2
1 1 1 1
concurrent, then (x1 + x2 + x3 + x 4 ) + + + 4.
=
x1 x2 x3 x 4
1 1 . 2 2 | Ellipse
Illustration 28: If the normal at an end of the latus rectum of an ellipse passes through one extremity of the minor,
show that the eccentricity of the ellipse is given by e4 + e2 − 1 =0 . JEE MAIN)
Sol: Subtitute the point (0, ±b) in the equation of the normal and simplify it.
x2 y2
Let + 1 be the ellipse. The coordinates of an end of the latus rectum are (ae, b2 / a) .
=
a2 b2
a2 x b2 y
The equation of normal at (ae, b2 / a) is − a2 b2
=−
ae b2 / a
It passes through one extremity of the minor axis whose coordinates are (0, ±b).
∴ ± ab = a2 − b2
Illustration 29: Any ordinate MP of an ellipse meets the auxiliary circle in Q. Prove that the locus of the point of
intersection of the normal P and Q is the circle x2 + y 2 =(a + b)2 . (JEE MAIN)
Sol: Consider a point on the ellipse and find the intersection of the ordinate with the circle. Next find the intersection
of the normal at P and Q and eliminate the parameter θ .
x2 y2
Let P(acos θ, bsin θ) be any point on the ellipse = 1 , and let Q (acos θ, asin θ) be the corresponding point
+
a2 b2
on the auxiliary circle x2 + y 2 =
a2 . The equation of the normal at P (acos θ, bsin θ) to the ellipse is
ax sec θ − by cosec θ= a2 − b2 …(i)
The equation of the normal at Q (acos θ, asin θ) to the circle x2 + y 2 =
a2 is
=y x tan θ …(ii)
Let (h , k) be the point of intersection of (i) and (ii). Then,
ahsec θ − bk cosecθ= a2 − b2 …(iii)
and, P(acos θ, bsin θ) …(iv)
k2 h2
ah 1 + − bk 1 + = a2 − b2
h2 k2
Illustration 30: If the length of the major axis intercepted between the tangent and normal at a
sec θ cosec θ 1
2π 2π x2 y 2
point sec θ + cosec θ + 1 on the ellipse 2 + 2 =1 is equal to the semi-major axis, prove that the
3 3 a b
2π 2π
sec θ − cosec θ − 1
3 3
Sol: Obtain the points of intersection of the tangent and the normal and then use the distance formula.
x2 y2
The equation of the tangent and normal to the ellipse + 1 at the point P
=
a2 b2
sec θ cosec θ 1
2π 2π
sec θ + cosec θ + 1 are given by
3 3
2π 2π
sec θ − cosec θ − 1
3 3
x y
cos θ + sin θ =1 …(i)
a b
θ (a2 − b2 )
and, ax sec θ − by cosec= …(i) respectively.
Suppose (i) and (ii) meet the major axis i.e. y = 0 at Q and R respectively. Then, the coordinates of Q and R are given
by
a2 − b2
Q(asec θ,0) and R cos θ,0 ∴ QR = a [Given]
a
a2 − b2
⇒ asec θ − cos θ = a ⇒ a2 − (a2 − b2 )cos
=2
θ a2 cos θ ⇒ a2 − a2e2 cos
=2
θ a2 cos θ
a
2
⇒ 1 − e2 cos2 θ = cos θ ⇒ e2 cos2 θ = 1 − cos θ ⇒ e= e {sec θ(sec θ − 1)}1/2
sec θ(sec θ − 1) ⇒=
x2 y2
Illustration 31: If ω is one of the angles between the normals to the ellipse + 1 at the points whose
=
a2 b2
π 2cot ω e2
eccentric angles are θ and + θ , then prove that = . (JEE ADVANCED)
2 sin2θ 1 − e2
Sol: Evaluate the equation of the normal at the two points and then use the formula of the angle between two lines.
x2 y2 π
The equation of the normal to the ellipse + 1 at the points whose eccentric angles are θ and
= + θ are
a2
b 2 2
ax sec θ − by cosecθ= a − b and, −ax cosecθ − by secθ = a2b2 respectively. Since ω is the angle between these
2 2
(a / b)tan θ + (a / b)cot θ
two normals, therefore, tan ω =
1 − (a2 / b2 )
2a2 1 − e2 2cot ω e2
⇒ tan ω = ⇒ =−
a2e2 sin2θ sin2θ 1 − e2
Illustration 32: If the tangent drawn at point (t2 ,2t) on the parabola y 2 = 4x is the same as the normal drawn at
a point ( 5 cos θ, 2sin θ) on the ellipse 4x2 + 5y 2 =
20 , find the values of t and θ . (JEE ADVANCED)
Sol: Write the equation for the tangent and normal in terms of the parameter. Compare the two equations to get
the values of t and θ .
1 1 . 2 4 | Ellipse
The equation of the normal at point ( 5 cos θ, 2sin θ) on the ellipse 4x2 + 5y 2 =
20 is
⇒ ( 5 sec θ)x − ( 2cosecθ)y − 1 =
0 … (ii)
5 sec θ − 2cosec θ −1
It is given that (i) and (ii) represent the same line. Therefore,= = 2
1 −t t
2cosec θ 1 2 1
⇒ t= and t = − ⇒
= t cot θ and t =
− sin θ
5 sec θ 2cosec θ 5 2
2 1
⇒ cot θ = − sin θ ⇒ 4 cos θ = − 5 sin2 θ ⇒ 4 cos θ = − 5(1 − co s2 θ)
5 2
1
θ cos−1 −
⇒= [cosθ ≠ − 5]
5
1 1 1 1 1
Putting cosθ = − in t =
− sin θ we get t =− 1− =−
5 2 2 5 5
1 1
Hence,
= θ cos−1 − and t = − .
5 5
Illustration 33: The normal at a point P on the ellipse x2 + 4y 2 =16 meets the x-axis at Q. If M is the mid point of
the line segment PQ, then the locus of M intersects the latus rectums of the given ellipse at the points.
(JEE ADVANCED)
3 5 2 3 5 19 1 4 3
(a) ± , ± (b) ± ,± (c) ±2 3, ± (d) ±2 3, ±
7 7 2 4 7 7
Sol: Put y = 0 in the equation of the normal to get the point Q in terms of θ . Get the locus of the mid-point as
required. In the last step solve the equation of the locus and the latus rectum.
x2 y 2
Equation of the ellipse is + 1
=
16 4
Y
Equation of the normal at P(4 cos θ, 2sin θ) to the ellipse is
θ 42 − 22 ⇒ 2x sec θ − y cosec θ = 6
4x sec θ − 2y cosec = P(4cos 2sin)
It meets x-axis at Q(3cos θ,0) . If (h, k) are the coordinates of M, then M
O Q X
4 cos θ + 3cos θ 2sin θ + 0
h = ,k
2 2
2h
⇒ =
cosθ ,=
sin θ k
7
2
4h2 x2 y Figure 11.16
⇒ + k2 =
1 Locus of M is ⇒ + 1.
=
49 (7 / 2)2 1
M a them a ti cs | 11.25
12 × 4 1 1
So locus of M meets the latus rectum at points for which y 2 =1− = ⇒y=±
49 49 7
1
And hence the required points are ±2 3, ± .
7
x2 y2
Illustration 34: Find the locus of the midpoint of a focal chord of the ellipse + 1 .
= (JEE MAIN)
a2 b2
Sol: In the equation T = S1, substitute x = ae and y = 0.
x2 y2
Let (h, k) be the midpoint of a focal chord of the ellipse + 1 . Then, the equation of the chord is
=
2 2
a2 b2
hx ky h k
+ −1 = + − 1 [Using : T = S1]
2 2 2
a b a b2
hx ky h2 k2
or, + = + It passes through the focus (ae, 0) of the ellipse.
a2 b2 a2 b2
hae h2 k2 xe x2 y 2
∴ +0= + . Hence, the locus of (h, k) is = + .
a2 a2 b2 a a2 b2
x2 y2
Illustration 35: Find the locus of the mid-point of the normal chords of the ellipse + 1.
=
a2 b2
(JEE ADVANCED)
Sol: Similar to the previous question.
Let (h, k) be the mid point of a normal chord of the given ellipse. Then, its equation is
hx ky h2 k2
+ −1 = + − 1 [Using: T = S1]
a2 b2 a2 b2
hx ky h2 k2
or + = + …(i)
a2 b2 a2 b2
h2 k2
+
h k 2
b2
∴ = = a
a3 sec θ −b3 cosecθ a2 − b2
1 1 . 2 6 | Ellipse
a3 h2 k 2 −b3 h2 k 2
⇒ cos θ
= =
+ , sin θ +
h(a2 − b2 ) a2 b2 k(a2 − b2 ) a2 b2
14. DIAMETERS
Definition: A chord through the centre of an ellipse is called a diameter of the ellipse.
x2 y2
(y mx + c) of slope m of the ellipse
The equation of the diameter bisecting the chords = + 1
=
a2 b2
b2
is y = − x , which is passing through (0, 0)
a2m
Y
y=mx+c
X’ X
2
-b
Diameter y = x
a2m
Y’
Figure 11.18
Conjugate diameter: Two diameters of an ellipse are said to be conjugate diameters if each bisects all chords
parallel to the other. The coordinates of the four extremities of two conjugate diameters are
P(acos φ, bsin φ) ; P'( −acos φ, − bsin φ)
Q A
B P
o
90
X
X’
C
P’ Q’
A’ B’
Y’
Figure 11.19
M a them a ti cs | 11.27
−b2
If y = m1 x and y = m2 x are two conjugate diameters of an ellipse, then m1m2 = .
a2
(a) Properties of diameters:
(i) The tangent at the extremity of any diameter is parallel to the chords it bisects or parallel to the conjugate
diameter.
(ii) The tangents at the ends of any chord meets on the diameter which bisects the chord.
A’ A
C
P’ D’
Figure 11.20
(ii) The sum of the squares of any two conjugate semi-diameters of an ellipse is constant and equal to the
sum of the squares of the semi axes of the ellipse i.e., CP2 + CD2 =a2 + b2 .
A’ A
S C S’
P’ D’
Figure 11.21
(iii) The product of the focal distances of a point on an ellipse is equal to the square of the semi-diameter
which is conjugate to the diameter through the point i.e., SP.S'P = CD2 .
(iv) The tangents at the extremities of a pair of conjugate diameters form a parallelogram whose area is
constant and equal to the product of the axes i.e., Area of parallelogram = (2a)(2b) = Area of rectangle
contained under major and minor axes.
Y
Q
M
D P
R’ R
X’ X
C
P’ D’
Q’
Y’
Figure 11.22
(v) The polar of any point with respect to an ellipse is parallel to the diameter to the one on which the point
lies. Hence obtain the equation of the chord whose mid point is ( x1 , y1 ) , i.e., chord is T = S1 .
(vi) Major and minor axes of ellipse is also a pair of conjugate diameters.
1 1 . 2 8 | Ellipse
(c) Equi-conjugate diameters: Two conjugate diameters are called equi-conjugate, if their lengths are equal i.e.,
(CP)2 = (CD)2 .
(a2 + b2 )
∴(CP) = (CD) = for equi-conjugate diameters.
2
x2 y2
Illustration 36: If PCP’ and DCD’ form a pair of conjugate diameters of the ellipse + 1 and R is any point
=
a2 b2
c2 , then prove that PR 2 + DR 2 + P'R 2 + D'R 2 = 2(a2 + b2 + 2c2 ) .
on the circle x2 + y 2 = (JEE MAIN)
Sol: Using the definition of conjugate diameters, get the coordinates of the point P, P’, Q and Q’. Starting from the
L.H.S. prove the R.H.S.
Let R(h, k) be any point on the circle x2 + y 2 =
c2 . Then h2 + k 2 =
c2 …(i)
Since PCP’ and DCD’ form a pair of conjugate diameters, the coordinates of the extremities are:
P(acos θ, bsin θ) , P'( −acos θ, − bsin θ) D( −asin θ, bcos θ) , D'(asin θ, − bcos θ)
∴ PR 2 + DR 2 + P'R 2 + D'R 2 = (h − acos θ)2 + (k − bsin θ)2 + (h + asin θ)2 + (k − bcos θ)2
= 2(a2 + b2 + 2c2 )
x2 y2
Illustration 37: CP and CD are conjugate semi-diameters of the ellipse + 1.
=
a2 b2
x2 y21
Show that the locus of the mid-point of PD is the ellipse + = . (JEE MAIN)
a2
b22
Sol: Consider two points which lie on two conjugate diameters. Find the mid point of these two points and eliminate
the parameter θ to get the locus of the mid point.
Let P (acosθ, bsinθ), D (-asinθ, bcosθ) and (h, k) be the mid-point of PD. Then,
2h acos θ − asin θ and =
= 2k b sin θ + b cos θ
2h 2k 4h2 4k 2
⇒ = cos θ − sin θ and = sin θ + cos θ ⇒ + = (cos θ − sin θ)2 + (sin θ + cos θ)2
a b a2 b2
h2 k2 1 x2 y 2 1
⇒ + = . Hence, the locus of (h, k) is ⇒ + =.
a2 b2 2 a2 b2 2
2 1 1 2
(a) (b) (c) (d)
3 3 2 5
Sol: Use the condition of conjugacy of diameters in an ellipse to find the eccentricity.
x2 y2 2
Let the equation of the ellipse be + 1 . Slope of the given diameters are m1 = 1 , m2 = − .
=
a2
b2 3
M a them a ti cs | 11.29
2 b2
⇒ m1m2 =− =− [Using the condition of conjugacy of two diameters]
3 a2
2 1 1
3b2 = 2a2 ⇒ 3a2 (1 − e2 ) = 2a2 ⇒ 1 − e2 = ⇒ e2 = ⇒e=
3 3 3
Illustration 39: Show that the locus of the point of intersection of tangents at the end-point of the conjugate
x2 y2
diameters of the ellipse + 1 is another ellipse of the same eccentricity.
= (JEE ADVANCED)
a2 b2
Sol: Using two points at the end points of the conjugate diameters of an ellipse, write the equation of the tangent.
Solve the two equations to eliminate the parameter θ .
x2 y2
Let CP and CD be two conjugate semi-diameters of the ellipse = 1 . Then, the eccentric angles of P and D
+
a2 b2
π
are θ and + θ respectively. So, the coordinates of P and D are (acos θ, bsin θ) and ( −asin θ, bcos θ) respectively.
2
The equation of the tangents at P and D are
x y
cos θ + sin θ =1 …(i)
a b
−x y
and sin θ + cos θ =1 …(ii)
a b
h k −h k
Let (h, k) be the point of intersection (i) and (ii). Then, cos θ + sin θ =1 and sin θ + cos θ =1
a b a b
2 2
h k h k h2 k 2
⇒ cos θ + sin θ + − sin θ + cos θ = 1 + 1 ⇒ + = 2
a b a b a2 b2
h2 k2
Hence, the locus of (h, k) is + 2 which represents an ellipse of eccentricity e, given by
=
a2 b2
2b2 b2
e1 = 1 − =1−
2a2 a2
x2 y2
Clearly, it is same as the eccentricity of + 1.
=
a2 b2
Illustration 40: If α and β are the angles subtended by the major axis of an ellipse at the extremities of a pair of
conjugate diameters, prove that cot2 α + cot2 β = constant. (JEE MAIN)
Sol: Using the co-ordinates of the co-ordinates of the end points of a diameter, find the angle subtended by the
major axis. Repeat the same process for the other end of the diameter. Then find the value of cot2 α + cot2 β and
prove that it is independent of the parameter.
x2 y 2
Let CP and CD be a pair of conjugate semi-diameters of the ellipse + 1 . Then, the coordinates of P and D
=
are (acos θ, bsin θ) and ( −asin θ, bcos θ) respectively. a2 b2
bsin θ b θ
m1 = Slope of AP = = − cot
acos θ − a a 2
bsin θ b θ
m2 = Slope of A’P = = tan
acos θ + a a 2
1 1 . 3 0 | Ellipse
ab
θ θ 2ab 1
⇒ tan α
= cot + tan ⇒ tan α = 2
2
a −b 2
2 2 a − b2 sin θ
π 2ab 1
Replacing θ by + θ , we get tan β =
2 a2 − b2 cos θ
2 2
2
a2 − b2
2
a2 − b2
∴ cot α
= + cot β (sin2 θ=
+ cos2 θ) = Constant.
2ab 2ab
Illustration 41: Find the locus of the points of intersection of normals at two points on an ellipse which are
extremities of conjugate diameters. (JEE MAIN)
Sol: Solve the equation of the normal at the extremities of conjugate diameters.
x2 y2
Let PP’ and QQ’ be two conjugate diameters of the ellipse + 1.
=
a2 b2
π
Let the eccentric angle of the point P be ' φ ' . Then the eccentric angle of Q is ' φ + ' .
2
∴ = P (acos φ, bsin φ) Q’+
2
D P’’
π π
=Q acos φ + , bsin φ +
2 2
O
x − acos φ y − bsin φ
The equation of the normal =
at P (acos φ, bsin φ) is = P’ D’
2
(acos φ) / a (bsin φ) / b2
Figure 11.23
or ax sec φ − bycosec φ= a2 − b2 …(i)
Similarly, the equation of the normal at Q is
π π 2 2
ax sec φ + − bycosec φ + = a −b
2 2
The locus of the point of intersection of (i) and (ii) is obtained by eliminating φ from them. Now we have
ax sec φ − by cosec φ − (a2 − b2 ) = 0
By cross multiplication,
sec φ cosec φ a2 − b2
= =
−by + ax −by − ax a2 x2 + b2 y 2
a2 x2 + b2 y 2 1
∴ cos φ = .
ax − by a − b2
2
a2 x2 + b2 y 2 1
sin φ = .
−(ax + by) a − b2
2
M a them a ti cs | 11.31
⇒ 2(a2 x2 + b2 y 2 )3 =
(a2 − b2 )2 .(a2 x2 − b2 y 2 )2 .
(a) (b)
Figure 11.24
Note: If the pole lies outside the ellipse then the polar passes through the ellipse. If the pole lies inside the ellipse
then the polar lies completely outside the ellipse. If the pole lies on the ellipse then the polar becomes the same
as the tangent.
x2 y 2 xx1 yy1
Equation of polar: Equation of polar of the point (x1, y1) with respect to ellipse + =1 is given by 2 + 2 = 1,
a2 b2 a b
i.e., T = 0
x2 y2 −a2l −b2m
Coordinates of Pole: The pole of the line lx + my + n =0 with respect to ellipse + 1 is P
= , .
a2 b2 n n
Properties of pole and polar:
(a) If the polar of P (x1 , y1 ) passes through Q(x2 , y 2 ) , then the polar of Q(x2 , y 2 ) goes through P (x1 , y1 ) and
x1 x2 y1 y 2
such points are said to be the conjugate points. Condition for conjugate points is + 1.
=
2
a b2
(b) If the pole of line l1 x + m1 y + n1 =
0 lies on another line l2 x + m2 y + n2 =
0 , then the pole of the second line will
lie on the first and such lines are said to be conjugate lines.
(c) Pole of a given line is the same as the point of intersection of tangents at its extremities.
(d) Polar of focus is directrix.
x2 y2
Illustration 42: Obtain the locus of poles of tangents to the ellipse + 1 with respect to concentric ellipse
=
a2 b2
x2 y2
+ 1 .
= (JEE MAIN)
α2 β2
x2 y 2
Sol: Taking a point (h , k), write the equation of the polar w.r.t. the ellipse + 1 . In the next step put the
=
condition for polar to be the tangent to the other given ellipse. α2 β2
1 1 . 3 2 | Ellipse
hx ky β2h β2
The equation of the polar is + 1⇒ y=
= − x+
α2 β2 α2k k
2 2
x2 y2 β2 −β2h
This touches + 1 . Therefore,
= = a2 + b2
a2 b 2 k 2
αk
a2 x2 b2 y 2
Hence, the locus of (h, k) is ⇒ + 1.
=
α4 β4
x2 y2
Illustration 43: Find the locus of the mid-points of the chords of the ellipse = 1 whose poles are on the +
a2 b2
auxiliary circle or the tangents at the extremities of which intersect on the auxiliary circle. (JEE ADVANCED)
Sol: Compare the equation of the chord and the tangent to get the point which lies on the auxiliary circle. Substitute
the point in the equation of the circle to get the required locus.
Let (h, k) be the mid-point of a chord of the ellipse. Then, its equation is
hx ky h2 k2
+ = + …(i)
a2 b2 a2 b2
x2 y2
Let (x1, y1) be its pole with respect to the ellipse + 1 . Then, the equation of the polar is
=
a2 b2
xx1 yy1
+ 1
= …(ii)
2
a b2
2
h2 k 2
It is given that (x1, y1) lies on auxiliary circle. Therefore x12 + y12 2 2
= a ⇒ h +k = a + 2
. 2
a2 b2
2
x2 y 2
Hence the locus of (h, k) is x2 + y=
2
a2 + .
a2 b2
PLANCESS CONCEPTS
Misconception: As there is no y1 term involved in the above results, don’t think that the lengths are
x12 y12
independent of y1. Always remember that + 1
=
a2 b2
Vaibhav Krishnan (JEE 2009, AIR 22)
PROBLEM-SOLVING TACTICS
x2 y2 m2 (a2 − b2 )2
•• If the line=
y mx + c is a normal to the ellipse + 1 then c2 =
= is the condition of normality
a2 b2 a2 + b2m2
of the line to the ellipse.
•• he tangent and normal at any point of an ellipse bisect the external and angles between the focal
T
radii to the point. It follows from the above property that if an incident light ray passing through the
focus (S) strikes the concave side of the ellipse, then the reflected ray will pass through the other
focus (S’).
•• If SM and S’M’ are perpendicular from the foci upon the tangent at any point of the ellipse, then SM. S’M’ = b2
and M, M’ lie on the auxiliary circle.
x2 y 2
•• If the tangent at any point P on the ellipse + 1 meets the major axis in T and minor axis in T’, then CN.
=
2 2
CT = a , CN’. CT’ = b
2 2 a b
Where N and N’ are the feet of the perpendicular from P on the respective axis.
•• If SM and S’ M’ are perpendicular from the foci S and S’ respectively upon a tangent to the ellipse, then CM
and CM’ are parallel to S’P and SP respectively.
FORMULAE SHEET
1. The general equation of second order ax2 + 2hxy + by 2 + 2gx + 2fy + c =0 represents an ellipse
a h g
if ∆ ≠ 0, h2 < ab. where ∆ = h b f
g f c
2. The sum of the focal distance of any point on an ellipse is a constant and is equal to the length of the major
axis of the ellipse i.e. SP + S’P = 2a.
x2 y2
3. Standard equation of an ellipse is + 1
=
a2 b2
Where a = length of semi-major axis,
b = length of semi-minor axis
1 1 . 3 4 | Ellipse
4.
x2 y 2
Ellipse 2 + 2 = 1
Imp. Terms a b
(0 ≤ φ < 2π)
Focal radii SP = a – ex1 SP = b –ey1
S’P = a + ex1 S’P = b + ey1
Sum of focal radii (SP + S’P =) 2a 2b
Distance between foci 2ae 2be
Distance between directrices 2a/e 2b/e
Tangents at the vertices x = –a, x = a y = b, y = –b
x2 y2
+ 1 , where φ is the parameter.
=
a2 b2
6. (i) If the centre of the ellipse is at (h, k) and the axes are parallel to the coordinate axes, then its equation is
(x − h)2 (y − k)2
+ 1.
=
a2 b2
(lx + my + n)2 (mx − ly + p)2
(ii) If the equation of the ellipse is + 1 , where lx + my + n =
= 0 and mx − ly + p =0
a2 b2
lx + my + n mx − ly + p
are perpendicular lines. Substitute = X and = Y , to put the equation in the standard form.
2 2
l +m l2 + m2
x2 y2
7. If P(acos α, bsin α ) and Q(acos β, bsin β) are any two points on the ellipse + 1 , then the equation of a
=
a2 b2
x α+β y α+β α −β
chord joining these two points is cos + sin =cos .
a 2 b 2 2
M a them a ti cs | 11.35
x2 y 2 x2 y2
8. The point P(x1 , y1 ) lies outside, on, or inside the ellipse + 1 according to 1 + 1 − 1 > 0, =
= 0 or < 0
2 2 2 2
respectively. a b a b
x2 y2
9. The line =
y mx + c intersects the ellipse + 1 on two distinct points if a2m2 + b2 > c2 , on one point
=
2 2
a b
2 x2 y 2
if=c a m + b and does not intersect if a m2 + b2 < c2 . For an ellipse
2 2 2
+ 2
1 , the auxiliary circle is
=
x2 + y 2 =a2 . a2 b2
x2 y2 xx1 yy1
10. The equation of the tangent to the ellipse + 1 at the point (x1 , y1 ) is
= + 1 . The equation
=
2 2 2
a b a b2
11. Equation of pair of tangents drawn from an outside point P(x1 , y1 ) is SS1 = T2 .
x2 y2
12. For an ellipse + 1 , the equation of director circle is x2 + y 2 = a2 + b2 .
=
2 2
a b
x2 y2 a2 x b2 y
13. The equation of normal to the ellipse + 1 at the point (x1 , y1 ) is
= − =a2 − b2 . The equation of
a2 b 2 x1 y 1
2 2
x y
normal to the ellipse + ) a2 − b2 .
1 at any point (acos φ, bsin φ) is (ax sec φ − bycosecφ=
=
2 2
a b
x2 y2
14. If m is the slope of the normal to the ellipse + 1 , then the equation of the normal is
=
a2 b2
m(a2 − b2 ) ±a2 ±mb2
y mx ±
= . The co-ordinates of the point of contact are , .
2
a2 + b2m2 a +b m
2 2
a2 + b2m2
x2 y 2
(ii) Property 2: If θ1 , θ2 and θ3 are eccentric angles of three co-normal points on the ellipse + 1,
=
then sin(θ1 + θ2 ) + sin(θ2 + θ3 ) + sin(θ3 + θ1 ) = 0 . a2 b2
(iii) Property 3: Co-normal points lie on a fixed curve called an Apollonian Rectangular
(
Hyperbola a2 − b2 xy + b2kx − a2hy =
0 )
(iv) Property 4: If the normal at four points P(x1y1), Q(x2y2), R(x3y3) and S(x4y4) on the
x2 y2 1 1 1 1
ellipse + 1 are concurrent, then (x1 + x2 + x3 + x 4 ) +
=
x
+ + 4.
=
a2 b2 1 x 2 x3 x 4
1 1 . 3 6 | Ellipse
16. If SM and S’M’ are perpendiculars from the foci upon the tangent at any point of the ellipse, then
SM × S'M' = b2 and M, M’ lie on the auxiliary circle.
x2 y2
17. If the tangent at any point P on the ellipse + 1 meets the major axis at T and minor axis at T’, then
=
a2 b2
CN × CT = a2, CN’ × CT’ = b2. Where N and N’ are the feet of the perpendiculars from P on the respectively axis.
x2 y2
18. The equation of the chord of the ellipse + 1 , whose mid point is (x1 , y1 ) , is T = S1 .
=
a2 b2
x2 y2 xx1 yy1
19. The chord of contact from a point P(x1 , y1 ) to an ellipse + 1 is T = 0 is
= + 1.
=
2 2 2
a b a b2
x2 y2
20. The equation of the diameter bisecting the chords =
(y mx + c) of slope m of the ellipse + 1
=
a2 b2
b2
is y = − x.
a2m
x2 y2 −b2
21. If m1 and m2 are the slopes of two conjugate diameters of an ellipse + 1 , then m1m2 =
= .
a2 b2 a2
22. The eccentric angle of the ends of a pair of conjugate diameters of an ellipse differ by a right angle,
π
i.e., φ − φ ' = .
2
23. The sum of the squares of any two conjugate semi-diameters of an ellipse is constant and is equal to the sum
of the squares of the semi axes of the ellipse i.e., CP2 + CD2 = a2 + b2.
24. The product of the focal distances of a point on an ellipse is equal to the square of the semi-diameter which
is conjugate to the diameter through the point i.e., SP × S'P = CD2.
25. The tangents at the extremities of a pair of conjugate diameters form a parallelogram whose area is constant
and equal to the product of the axes.
i.e. Area of the parallelogram = (2a)(2b) = Area of the rectangle contained under major and minor axes.
26. Two conjugate diameters are called equi-conjugate, if their lengths are equal i.e., (CP)2 = (CD)2
(a2 + b2 )
∴(CP) = (CD) = for equi-conjugate diameters.
2
x2 y2 −a2l −b2m
27. Equation of the polar of the point (x1, y1) w.r.t. an ellipse + 1 is P
= , .
a2 b2 n n
x2 y2 −a2l −b2m
28. The pole of the line lx + my + n =0 with respect to the ellipse + 1 is P
= , .
a2 b2 n n
x1 x2 y1 y 2
29. Condition for a conjugate point is + 1.
=
a2 b2
x2 y2 a2
30. The length of a sub tangent at P(x1 , y1 ) to the ellipse + 1 is
= − x1 .
a2 b2 x1
x2 y2 b2
31. The length of a sub normal at P(x1 , y1 ) to the ellipse + 1 is
= x1= (1 − e2 )x1 .
a2 b2 a2
M a them a ti cs | 11.37
Solved Examples
(x + 1)2 (y − 2)2
+ 1
=
9 5 Example 2: If the chord through point θ1 and θ2 on an
Shift the origin to O' ≡ ( −1,2) x2 y2
ellipse + 1 intersects the major axis at (d, 0)
=
∴ X= x + 1 ; Y= y − 2 a2 b2
θ1 θ2 d−a
X2 Y2 prove that tan tan = .
∴ + 1
= …(i) 2 2 d+a
9 5
Sol: Substitute the point (d , 0) in the equation of the
This is in standard form
chord to prove the given result.
∴ =a 3,=
b 5
Equation of the chord joining the points θ1 and θ2 is
2 2
a −b 4 2 x θ + θ2 y θ1 + θ2 θ − θ2
e2
∴= = ⇒e= cos 1 cos 1
a2 9 3 + sin =
a 2 b 2 2
2 a 9 Since (d, 0) lies on it
Also ae = 3 . = 2 and = .
3 e 2
P(1)
Now for an ellipse in the standard form we have Centre
a R(d,)
≡ (0, 0); foci ≡ (±ae, 0); directrices x = ± ; axes x = 0,
e O
y = 0, length of major axis = 2a, length of minor axis Q(1)
= 2b.
Now for (i) the centre is given by X = 0, Y = 0
d θ + θ2 θ1 − θ2
⇒ x + 1= 0, y − 2= 0 ∴ cos 1 = cos
a 2 2
ax by
i.e. Centre + c
= cos ( (θ1 − θ2 ) / 2 )
3 4 d
=
Foci are given by X =
±ae, Y =
0 cos ( (θ1 + θ2 ) / 2 ) a
i.e. = y 2 and x =
x 1,= −3, y =
2 d − a cos ( (θ1 − θ2 ) / 2 ) − cos ( (θ1 + θ2 ) / 2 )
=
∴ Foci ≡ (1,2); ( −3, 2) d + a cos ( (θ1 − θ2 ) / 2 ) + cos ( (θ1 + θ2 ) / 2 )
1 1 . 3 8 | Ellipse
2sin(θ1 / 2)sin(θ2 / 2)
=
2cos(θ1 / 2)cos(θ2 / 2)
θ θ
tan 1 tan 2
2 2
Equation of normal at Q ( )
5 cos φ,2sin φ is
0,
b
B
⇒ ( )
5 1 − cos2 φ + 4 cos φ =0
sin 1
P 1
3
R(d,) A ⇒ cos φ = −
x 5
cos ,0
a 1
∴ φ= 2nπ ± cos−1 − where 0 ≤ φ ≤ 2π .
5
∴ Corresponding values of t are given by
a b cos φ 1
∴A≡ ,0 and B ≡ 0, t2 =
− = .
cos θ sin θ 5
5
Now P divides segment AB in the ratio 3 : 1 1
∴ t= ±
a 3b 5
∴ P≡ , …(ii)
4 cos θ 4 sin θ
Example 5: Show that the sum of the squares of the
By (i) and (ii), we have x2 y2
perpendiculars on any tangent to += 1 from 2
1 3 a2 b2
cos
= θ ; sin
= θ
∴ 2 2 points on the minor axis, each of which is at a distance
∴ Equation of tangent at P is bx + a 3y =
2ab . a2 − b2 from the centre, is 2a2 .
Example 4: If the tangent drawn at a point (t2 ,2t); t ≠ 0 Sol: Use the standard equation of a tangent in terms of
m and then proceed accordingly,
on the parabola y 2 = 4x is the same as the normal
drawn at a point ( 5 cos φ,2sin φ ) on the ellipse The general equation of a tangent to the ellipse is
4x2 + 5y 2 =
20 , find the value of t and φ . mx ± a2m2 + b2
y= …(i)
Sol: Write the equation of the tangent and the normal Let the points on the minor axis be P(0,ae) and
using ‘t’ and ‘ φ ’ and compare.
b2 a2 (1 − e2 )
Q(0, − ae) as =
2 2
Equation of the tangent at P(t ,2t) to y = 4x is
Length of the perpendicular from P on (i) is
yt= x + t2 …(i)
M a them a ti cs | 11.39
⇒ (x2 + y 2 )2 = a2 x2 + b2 y 2
⇒ 3 ( x − 2 ) + 4 ( y + 3) =
2 2
12
which does not represent a circle, an ellipse or a
⇒ 3x2 + 4y 2 − 12x + 24y + 36 =
0 hyperbola.
Example 7: Show that the angle between pair of Example 9: The ellipse x2 + 4y 2 = 4 is inscribed in a
tangents drawn to the ellipse 3x2 + 2y 2 =
5 from the rectangle aligned with the coordinate axes, which in
turn is inscribed in another ellipse that passes through
12
point (1, 2) is tan−1 − . the point (4, 0). The equation of the ellipse is
5
(A) 4x2 + 48y 2 =
48 (B) 4x2 + 6y 2 =
48
Sol: Starting from the standard equation of a tangent
in terms of m, satisfy the point (1 ,2) and get the values (C) x2 + 16y 2 =
16 (D) x2 + 12y 2 =
16
of m. Using the value of m, find the angle between the
two tangents. Let the equation of the tangents be Sol: Consider the standard equation of the ellipse. Use
the two points given in the question to find the value
of ‘a’ and ‘b’.
1 1 . 4 0 | Ellipse
h + 3k − 3 sin(α − β) 7
(h − 3)2 + (k − 4)2 = ⇒ =
1+9 sin α − sin β 4
M a them a ti cs | 11.41
cos[(α − β) / 2)] 7 PQ = 8 + 2 = 10
⇒ =
cos[(α + β) / 2] 4 ∴ Area of quadrilateral ABCD
α
⇒ tan tan
=
β
7 − 4 8 7 − 23
= .
1
2
1
2
(
= (AB + CD)PQ = 8 2 + 3 2 10 =55 2 sq. units. )
2 2 7+4 9
Example 2: Show that the angle between the tangents
x2 y2
JEE Advanced/Boards to the ellipse =1 (where a > b ), and the circle
+
a2 b2
Example 1: Common tangents are drawn to the x2 + y 2 =
ab at their points of intersection in the first
parabola y2 = 4x and the ellipse 3x2 + 8y2 = 48 touching (a − b)
quadrant is tan−1 .
the parabola A and B and the ellipse at C and D. Find
ab
the area of the quadrilateral ABCD.
Sol: We find the point of intersection of the ellipse and
Sol: Write the standard equation of the parabola in the the circle. Then we find the slope of the tangents to the
slope. Use the condition for the line to be a tangent and circle and the ellipse and hence the angle.
obtain the value of m. We then find the points of contact
At the points of intersection of ellipse and circle,
with the ellipse and parabola and then find the area.
1 ab − y 2 y2
Let =
y mx + be a tangent to the parabola y 2 = 4x . + 1
=
m a2 b2
x2 y2 1 1 1 b ab2
It will touch the ellipse + 1 , if= 16m2 + 6
= ⇒ y2 − = 1− ⇒ y2 =
42 ( 6)2
m2 b2 a2 a a+b
c2 a2m2 + b2 ]
[Using:=
a b
∴ y = ±b and x = ±a
⇒ 16m4 + 6m2 − 1 =0 a+b a+b
1 a b
⇒ (8m2 − 1)(2m2 + 1) =
0 ⇒ m= ± b a
8 P ,
a+b a + b
We know that a tangent of slope m touches the
a 2a lies in first quadrant
parabola y 2 = 4ax at , . So, the coordinates of
m2 m Equation of tangent at P to the circle is
the points of contact of the common tangents of slope
xa b yb a
1 + ab
=
m= ± to the parabola y 2 = 4x are A(8, 4 2 ) and a+b a+b
2 2
a
B(8, –4 2 ). Its slope is: m1 = −
b
We also know that a tangent of slope m touches the Equation of the tangent at P to the ellipse is
x 2
y 2 a2m b2
ellipse + 1 at
= ,± . xa b yb a
a2 b2 + 1
=
a2m2 + b2 a2m2 + b2 a2
a+b b 2
a+b
Therefore, the coordinates of the points of contact of b3/2
1 Its slope in m2 = −
common tangents of slope m ± to the ellipse are a3/2
2 2
If α is the angle between these tangents, then
3 3
C −2, and D −2, − .
2 2 m2 − m1 −(b3/2 / a3/2 ) + (a1/2 / b1/2 )
tan α
= =
1 + m1m2 1 + (b3/2 / a3/2 )(a1/2 / b1/2 )
Clearly AB || CD. So, the quadrilateral ABCD is a
trapezium. a2 − b2 a−b
= .
We have, AB = 8 2 , CD = 3 2 and the distance a 1/2 1/2
b (a + b) ab
between AB and CD is
1 1 . 4 2 | Ellipse
Example 3: Any tangent to an ellipse is cut by the Then P, Q, and R have coordinates given by:
tangents at the extremities of the major axis at T and
T’. Prove that the circle on TT’ as the diameter passes 2π 2π
P(acos θ, bsin θ) Q acos θ + , bsin θ + and
through the foci. 3 3
with the axis and then use these points to find the P
equation of the circle.
x2 y 2
Let the equation of the ellipse be + = 1. O
a2 b2 Q R
The extremities A and A’ of the major axis are A (a, 0), B C
A’ (–a, 0). Equations of tangents A and A’ are x = a and
x y 4π 4π
x = –a. Any tangent to the ellipse is cos θ + sin θ =1 . R acos θ + , bsin θ + respectively.
a b 3 3
The points of intersection are
Normals at P, Q, R to ellipse are concurrent, if the
b(1 − cos θ) b(1 + cos θ) determinants of the coefficients is zero. i.e., if
T a, , T' −a,
sin θ sin θ sin(θ1 + θ2 ) + sin(θ2 + θ3 ) + sin(θ3 + θ1 ) = 0
The equation of the circle on TT’ as diameter is
2π 6π 4π
∴ sin 2θ + + sin 2θ + + sin 2θ +
b bcos θ
2 2
3 3 3
(x 2
− a2 )
+ y −
− =
sin θ sin θ
0
= sin ( 2θ ) + sin 2θ +
2π 4π
+ sin 2θ +
3 3
b b2 (1 − cos2 θ)
⇒ x2 − a2 + y 2 − 2y + =0 = 0 for all values of θ
sin θ sin2 θ
∴ The normals are concurrent.
2by
⇒ x2 + y 2 − + b2 − a2 =
0
sin θ
Example 5: Prove that the sum of the eccentric angles
2by of the extremities of a chord which is drawn in a given
⇒ x2 + y 2 − a2e2
=
sin θ direction is constant and equal to twice the eccentric
angle of the point, at which the tangent is parallel to
Foci S (ae, 0) and S’ (–ae, 0) lie on this circle.
the given direction.
Example 4: Let ABC be an equilateral triangle inscribed Sol: Consider two points on the ellipse and evaluate
in the circle x2 + y 2 =
a2 . Suppose perpendiculars from the slope of the chord. If the slope is constant prove
that the sum of the angles is constant.
x2 y2
A, B, C to the major axis of the ellipse = 1,+
a2 b2 Slope of chord AB = m
(a > b), meets the ellipse at P, Q, R respectively so that b(sin α − sin β)
= −
P, Q, R lie on the same side of the major axis as are the a(cos α − cos β)
corresponding points A, B, C. Prove that the normals to
the ellipse drawn at the points P, Q, R are concurrent. 2b cos((α + β) / 2). sin((α − β) / 2) b α+β
= = − cot .
2asin((α + β) / 2).sin((β − α ) / 2) a 2
Sol: Find the points of intersection of the perpendicular
and the ellipse. Then apply the condition for the α+β
∴ = constant if m is constant
normals at these three points to be concurrent. 2
x y
Let A, B, C have coordinates (acos θ, asin θ) , Eq. of a tangent is cos θ + sin θ =1
a b
2π 2π b
acos θ + , asin θ + , Slope of this tangent is − cot θ .
3 3 a
b α+β
4π 4π − cot θ , then θ =
Now if m =
acos θ + , asin θ + respectively. a 2
3 3
M a them a ti cs | 11.43
So, the slopes are equal. They are parallel to each other. 1 1 1 1
Hence proved. get x2 − + y 2 − = 0
a2 p2 b2 q2
Example 6: P and Q are two points of the ellipse S P
2 2
x y
+ 1 such that sum of their ordinates is 3. Prove
= C (0,0)
25 9
Q R
that the locus of the intersection of the tangents at P
and Q is 9x2 + 25y 2 =.
150y Above equation will represent a pair of conjugate
diameters of the first ellipse if
Sol: Find the relation between the ordinate and use it
to find the locus. b2
m1m2 = −
a2
If (h, k) is the point of intersection of tangents at θ and
φ , then
∴
((1 / a ) − (1 / p )) = − b
2 2
2
h cos ( (θ + φ) / 2 ) k sin ( (θ + φ) / 2 )
= ;
((1 / b ) − (1 / q )) a
2 2 2
a cos ( (θ − φ) / 2 ) b cos ( (θ − φ) / 2 ) 1 1 1 1
⇒ a2 − + b2 − = 0
a2 p2 b2 q2
h2 k2 1
∴ + = …(i)
a 2
b2
cos ( (θ − φ) / 2 )
2
⇒
a2
+
b2
2
=
p2 q2
We are given that sum of ordinates is 3.
x2 y 2 4 b2
C : x2 + y 2 =
9, E: + 1 , L : y = 2x
= 2m = − mm' = − 2
9 4 9 a
Example 10: If L represents the line joining the point C (1, 2) = 1 + 22 − 9 < 0
P and C to its centre O and intersects E at M, then the 1
equation of the tangent at M to the ellipse E is ⇒ R lies inside C; E (1, 2) = +1 −1 > 0
9
(A) x + 3y =
3 5 (B) 4x + 3y =5 ⇒ R lies outside E.
(C) x + 3y + 3 5 =
0 (D) 4x + 3 + 5 =0 Example 13: If CF is perpendicular from the centre C
x2 y 2
Sol: Find the point of intersection of the line L and E. of the ellipse + =1 on the tangent at any point
a2 b2
Write the equation of the tangent at M. P, and G is the point where the normal at P meets the
Line L: y = 2x meets the circle C : x2 + y 2 =
9 at points minor axis, then (CF × PG)2 is equal to
3
for which x2 + 4x2 =
9 ⇒ x =± . Sol: Consider a parametric point on the ellipse and
5
proceed to find CF and PG.
3 6
Coordinates of P are ( ± ,± ) Equation of the tangent at P(7 cos θ, 5sin θ) on the
5 5
x y
3 4 ellipse is cos θ + sin θ =1 , then
⇒ Coordinates of M are ( ± ,± ) 7 5
5 5
72 × 52 25 × 49
Equation of the tangent at M to the ellipse E is (CF)2 = =
2 2 2
5 cos θ + 7 sin θ 2
25cos2 θ + 49 sin2 θ
x( ±3) y( ±4)
+ 1 ; x + 3y =
= ±3 5 .
9 5 4 5
M a them a ti cs | 11.45
JEE Main/Boards
Exercise 1
x2 y 2
Q.10 The common tangent of + =1 and α lies
Q.1 Find the equation of the ellipse whose vertices are 25 4
(5, 0) and (–5, 0) and foci are (4, 0) and (–4, 0). in 1st quadrant. Find the slope of the common tangent
and length of the tangent intercepted between the axis.
Q.2 Find the eccentricity of the ellipse 9x2 + 4y2 ‒ 30y = 0.
Q.11 Find a point on the curve x2 + 2y 2 = 6 whose
7 is minimum.
distance from the line x + y =,
Q.3 Find the equations of the tangents drawn from the
point (2, 3) to the ellipse 9x2 + 16y 2 =
144 .
Q.12 Find the equations to the normals at the ends of
the latus recta and prove that each passes through an
Q.4 Find the eccentric angle of a point on the ellipse 4 2
end of the minor axis if e + e = 1.
x2 y 2
+ = 2 at a distance 3 from the centre.
5 4
Q.13 Find the co-ordinates of those points on the
x2 y 2
Q.5 Obtain equation of chord of the ellipse ellipse + = 1 , tangent at which make equal
4x2 + 6y2 = 24 which has (0, 0) as its midpoint. a2 b2
angles with the axes. Also prove that the length of the
perpendicular from the centre on either of these is
Q.6 Find the foci of the ellipse
1 2
25(x + 1)2 + 9(y + 2)2 =
225 . (a + b2 ) .
2
Q.16 A circle of radius r is concentric with the ellipse Q.25 The co-ordinates of the mid-point of the variable
x2 y 2 1
+ = 1 . Prove that the common tangent is chord=y (x + c) of the ellipse 4x2 + 9y 2 =
36 are
a2 b2 2
r 2 − b2
inclined to the major axis at an angle tan−1 . Q.26 A triangle ABC right angled at ‘A’ moves so that it
a2 − r 2 x2 y 2
walsys circumscribes the ellipse + =1 . The locus
Q.17 Show that the locus of the middle points of a2 b2
of the point ‘A’ is
x2 y 2
those chords of the ellipse + = 1 which are
a2 b2
drawn through the positive end of the minor axis is
Exercise 2
x2 y 2 y
+ = .
a2 b2 b Single Correct Choice Type
x2 y2
Q.18 Tangents are drawn from a point P to the circle Q.1 The equation + +1 =0 represents an
ellipse, if 2 −r r −5
x2 + y 2 =r 2 so that the chords of contact are tangent
to the ellipse a2 x2 + b2 y 2 =
r 2 . Find the locus of P. (A) r > 2 (B) 2 < r < 5 (C) r > 5 (D) r ∈ {2,5}
Q.21 Prove that the circle on any focal distance as (C) End of the major axis (D) End of the minor axis
diameter touches the auxiliary circle.
Q.4 If the line =y 2x + c be a tangent to the ellipse
2 2
x y x 2
y 2
Q.22 Let P be a point on the ellipse = 1 , 0<b<a.
+ + 1 , then c is equal to
=
2
a b2 8 4
Let the line parallel to y-axis passing thorugh P meet
(A) ±4 (B) ±6 (C) ±1 (D) ±8
the circle x2 +y2 = a2 at the point Q such that P and Q
are on the same side of the x-axis. For two positive real
numbers r and s, find the locus of the point R on PQ Q.5 If the line 3x + 4y = − 7 touches the ellipse
such that PR : RQ = r : s as P varies over the ellipse. 3x2 + 4y 2 =
1 then, the point of contact is
0
Q.23 Consider the family of circles x2 +y2 = r2 , 2<r<5. 1 1 1 −1
(A) , (B) ,
In the first quadrant, the common tangent to a circle of 7 7 3 3
this family and the ellipse 4x2 + 25y2 = 100 meets the
coordinate axes at A and B, then find the equation of 1 −1 −1 −1
(C) , (D) ,
the locus of the mid-point of AB. 7 7 7 7
x2 y2
Q.24 A tangent to the ellipse =1 , meets the
+ Q.6 The point of intersection of the tangents at the
2 2 a2 b2 x2 y 2
x y point P on the ellipse + = 1 and its corresponding
ellipse + =a + b in the points P and Q. Prove that
a 2
b2 a2 b2
the tangents at P and Q are at right angles. point Q on the auxiliary circle meet on the line:
M a them a ti cs | 11.47
a
(A) x = (B) x = 0 (C) y = 0 (D) None of these (C) ay =bx − a4 + a2b2 + b 4
e
(D) by
by==ax++ a4 + a2b2 + b 4
ax
Q.7 The equation of the normal to the ellipse
x2 y 2
+ =1 at the positive and of latus rectum Q.13 In the ellipse the distance between its foci is 6 and
a2 b2
its minor axis is 8. Then its eccentricity is
(A) x + ey + e2a =
0 (B) x − ey − e3a =
0 4 1 3
2 (A) (B) (C) (D) None of these
(C) x − ey − e a =
0 (D) None of these 5 52 5
x2 y 2
Q.14 Equation of a tangent to the ellipse + 1
=
Q.8 The normal at an end of a latus rectum of the ellipse 25 16
x2 y 2 which cuts off equal intercepts on the axes is-
+ = 1 passes through an end of the minor axis, if:
a2 b2
(A) x + y − 41 =
0 (B) x − y + 9 =
0
4 2 3 2
1
(A) e + e = (B) e + e =
1 (C) x + y − 9 =
0 (D) None of these
2
(C) e + e =
1 (D) e3 + e =
1
Q.15 An ellipse has OB as a semi minor axis. FBF’ are
Q.9 If CF is perpendicular from the centre of the ellipse its foci, and the angle FPF’ is a right angle. Then the
x2 y 2 eccentricity of the ellipse, is
+ = 1 to the tangent at P and G is the point
a2 b2 (A)
1
(B)
1
where the normal at P meets the major axis, then the 3 2
product CF.PG is: 1
(C) (D) None of these
2
(A) a2 (B) 2b2 (C) b2 (D) a2 − b2
(A) b = 3 ; x + 2y + a =0 (B) b = 3; x + 2y + 4 =
0 Q.17 If the distance between a focus and corresponding
1
(C) b = 3 ; x + 2y − 4 =
0 (D) b = 3 ; x − 2y − 4 =
0 directrix of an ellipse be 8 and the eccentricity be ,
then length of the minor axis is 2
16
Q.11 An ellipse is such that the length of the latus (A) 3 (B) 4 2 (C) 6 (D)
3
rectum is equal to the sum of the lengths of its semi
principal axes. Then: x2 y 2
Q.18 Let ‘E’ be the ellipse + =1 & ‘C’ be the
9 4
(A) Ellipse bulges to a circle
circle x2 + y 2 =9 . Let P and Q be the points (1, 2) and
(B) Ellipse becomes a line segment between the two foci (2, 1) respectively. Then:
(C) Ellipse becomes a parabola (A) Q lies inside C but outside E
(D) None of these (B) Q lies outside both C and E
(C) P lies inside both C and E
Q.12 Which of the following is the common tangent to
(D) P lies inside C but outside E
x2 y2 x2 y2
the ellipses, + 1 and
= + 1?
=
a2 + b2 b2 a2 a2 + b2
Q.19 The line, x + my + n =0 will cut the ellipse Q.25 The equation 2x2 + 3y2 ‒ 8x ‒ 18y + 35 = K
x2 y2 represents:
+ 1 in poins whose eccentric angle differe by
=
a2 b2 (A) A point if K = 0
π
if: (B) An ellipse if K < 0
2
(C) A hyperbola if K < 0
(A) x2 2 + b2n2 =
2m2 (B) a2m2 + b2 2 =
2n2 (D) A hyperbola if K > 0
(C) a2 2 + b2m2 =
2n2 (D) a2n2 + b2m2 =
22
Previous Years’ Questions
Q.20 The locus of point of intersection of tangents to
x2 y 2 Q.1 If P = (x, y), F1 = (3, 0), F2 = (‒3, 0) and 16x2 + 25y2 = 400,
an ellipse + =1 at two points the sum of whose
a2 b2 then PF1 + PF2 equals (1998)
eccentric angles is constant is: (A) 8 (B) 6 (C) 10 (D) 12
(A) A hyperbola (B) An ellipse
(C) A circle (D) A straight line Q.2 The number of values of c such that the straight
x2
line y = 4x + c touches the curve + y2 =
1 is (1998)
Q.21 Q is a point on the auxiliary circle of an ellipse. P 4
is the corresponding point on ellipse. N is the foot of (A) 0 (B) 2 (C) 1 (D) ∞
perpendicular from focus S, to the tangent of auxiliary
circle at Q. Then Q.3 The line passing through the extremity A of the
major axis and extremity B of the minor axis of the
(A) SP = SN (B) SP = PQ
ellipse x2 + 9y2 = 9 meets its auxiliary circle at the point
(C) PN = SP (D) NQ = SP M. Then the area of the triangle with vertices at A, M
and the origin O is (2009)
Q.22 A tangent to the ellipse 4x2 + 9y 2 =
36 is cut by 31 29 21 27
the tangent at the extremities of the major axis at T (A) (B) (C) (D)
10 10 10 10
and T’. The circle on TT’ as diameter passes thorugh
the point y2
x2
Q.4 Let P be a variable point on the ellipse = 1 +
(A) (0, 0) (B) ( ±5,0) (C) ( ± 5,0) (D) ( ±3,0) a2 b2
with foci F1 and F2. If A is the area of the triangle PF1F2,
Q.23 Q is a point on the auxiliary circle corresponding then the maximum value of A is……………. (1994)
x2 y 2
to the point P of the ellipse + =1 . If T is the foot
a2 b2 Q.5 An ellipse has OB as a semi minor axis. F and F’ are
of the perpendicular dropped from the focus S onto the its foci and the angle FBF’ is a right angle. Then, the
tangent to the auxiliary circle at Q then the ∆ SPT is: eccentricity of the ellipse is…………. (1997)
Q.8 A focus of an ellipse is at the origin. The directrix Q.12 The equation of the circle passing through the
is the line x = 4 and the eccentricity is 1/2. Then the x2 y 2
length of the semi−major axis is (2008) foci of the ellipse + =1 , and having centre at
16 9
8 2 4 5 (0, 3) is (2013)
(A) (B) (C) (D)
3 3 3 3
(A) x2 + y 2 − 6y − 7 =0 (B) x2 + y 2 − 6y + 7 =0
( )
2
(C) 4x2 + 48y 2 =
48 (D) 4x2 + 64y 2 =
48 (A) x2 + y 2 =6x2 + 2y 2
(B) ( x + y )
2
Q.10 Equation of the ellipse whose axes are the axes of 2 2
=6x2 − 2y 2
coordinates and which passes through the point (-3, 1)
and has eccentricity is
(C) ( x − y )
(2011) 2 2
2
=6x2 + 2y 2
(A) 5x2 + 3y 2 − 48 =
0 (B) 3x2 + 5y 2 − 15 =
0
(D) ( x − y )
2
2 2
=6x2 − 2y 2
(C) 5x2 + 3y 2 − 32 =
0 (D) 3x2 + 5y 2 − 32 =
0
Q.11 An ellipse is drawn by taking a diameter of the circle Q.14 The area (in sq.units) of the quadrilateral formed
(x ‒ 1)2 + y2 = 1 as its semiminor axis and a diameter of the by the tangents at the end points of the latera recta to
the Ellipse (2015)
circle x2 + (y ‒ 2)2 = 4as its semi-major axis. If the centre
of the ellipse is the origin and its axes are the coordinate 27 27
(A) (B) 18 (C) (D) 27
axes, then the equation of the ellipse is (2012) 4 2
(A) 4x2 + y 2 =
4 (B) x2 + 4y 2 =
8
(C) 4x2 + y 2 =
8 (D) x2 + 4y 2 =
16
JEE Advanced/Boards
Q.7 Prove that the equation to the circle having double Q.16 Prove that the length of the focal chord of the
x2 y 2 x2 y 2
contact with the ellipse + = 1 at the ends of a ellipse + = 1 which is inclined to the major axis
a2 b2 a2 b2
2ab2
latus rectum, is x2 + y 2 − 2ae3 x = a2 (1 − e2 − e4 ) . at angle θ is .
a2 + sin2 θ + b2 cos2 θ
Q.25 An ellipse is drawn with major and minor axes Q.5 An ellipse is described by using an endless string
of lengths 10 and 8 respectively. Using one focus as which passes over two pins. If the axes are 6 cm and 4
centre, a circle is drawn that is tangent to the ellipse, cm, the necessary length of the string and the distance
with no part of the circle being outside the ellipse. The between the pins respectively in cm, are
radius of the circle is_________.
(A) 6, 2 5 (B) 6, 5
Q.26 Point ‘O’ is the centre of the ellipse with major (C) 4,2 5 (D) None of these
axis AB and minor axis CD. Point F is one focus of the
ellipse. 1 f OF = 6 & the diameter of the inscribed
Q.6 If F1 & F2 are the feet of the perpendiculars from the
circle of triangle OCF is 2, then the product (AB) (CD) =
_____________________________. x2 y 2
foci S1 and S2 of an ellipse + = 1 on the tangent at
5 3
any point P on the ellipse, then (S1F1). (S2F2) is equal to:
Exercise 2 (A) 2 (B) 3 (C) 4 (D) 5
Q.3 The equation of the ellipse with its centre at (1, 2), x2 y2
+ = 1 of eccentricity e meets the axes of the
focus at (6, 2) and passing through the point (4, 6) is a2 b2
ellipse in Q and R then the locus of the mid-point of QR
(x − 1)2 (y − 2)2 (x − 1)2 (y − 2)2 is a conic with an eccentricity e’ such that:
(A) + 1 (B)
= + 1
=
45 20 20 45
(A) e’ is independent of e
2
(x − 1) 2
(y − 2) (x − 1) 2
(y − 2) 2 (B) e ‘ = 1
(C) + 1 (D)
= + 1
=
25 16 16 25 (C) e ‘ = e
(D) e ‘ = 1/e
Q.4 A line of fixed length (a+b) moves so that its ends
are always on two fixed perpendicular straight lines. The Q.10 A circle has the same centre as an ellipse & passes
locus of the point which divided this line into portions through the foci F1 and F2 of the ellipse, such that the
of lengths a and b is: two curves intersect in 4 points. Let ‘P’ be any one of
(A) An ellipse (B) An hyperbola their point of intersection. If the major axis of the ellipse
is 17 and the area of the triangle PF1F2 is 30, then the
(C) A circle (D) None of these distance between the foci is:
(A) 11 (B) 12
(C) 13 (D) None of these.
1 1 . 5 2 | Ellipse
(x − 5)2 y 2 (x − 5)2 y 2 Q.18 The tangents at any point F on the standard ellipse
(C) + 1 (D)
= + 1
=
25 9 9 25 with foci as S and S’ meets the tangents at the vartices
A and A’ in the points V and V’, then:
Q.3 The normal at a point P on the ellipse x2 + 4y2 = 16 Q.8 Let d be the perpendicular distance from the centre
meets the x-axis at Q. if M is the mid point of the line x2 y2
segment PQ, then the locus of M intersects the latusrectum of the ellipse + = 1 to the tangent drawn at a
a2 b2
of the given ellipse at the points. (2009)
point P on the ellipse. If F1 and F2 are the two foci of the
3 5 2 3 5 19 ellipse, then show that (1995)
(A) ± , ± (B) ± ,±
2 7 2 4 b2
(PF1 − PF2 )2 = 4a2 1 −
d2
1 4 3
(C) ±2 3, ± (D) ±2 3, ±
7 7
Q.9 A tangent to the ellipse x2 + 4y2 = 4 meets the
1 ellipses x2 + 2y2 = 6 at P and Q. Prove that tangents at P
Q.4 An ellipse has eccentricity and one focus at the and Q of ellipse x2 + 2y2 = 6 are at right angles. (1997)
2
1
point P , 1 . Its one directrix is the common tangent,
2
Q.10 Find the coordinates of all the points P on the ellipse
nearer to the point P, to the circle x2 + y 2 =
1 and the
hyperbola x2 − y 2 = 1 . The equation of the ellipse, in x2 y 2
+ = 1 , for which the area fo the triangle PON is
the standard form is……………. (1996) a2 b2
maximum, where O denotes the origin and N be the foot
of the perpendicular from O to the tangent at P. (1999)
Paragraph Based Questions 5 to 7
Tangents are drawn from the point P(3, 4) to the ellipse Q.11 Let ABC be an equilateral triangle inscribed in the
x2 y 2
+ = 1 touching the ellipse at points A and B. circle x2 + y 2 =
a2 . Suppose perpendiculars from A, B,
9 4
(2010) x2 y 2
C to the major axis of the ellipse + 1 , (a > b)
=
a2 b2
Q.5 The coordinates of A and B are
meets the ellipse respectively at P, Q, R so that P, Q, R lie
on the same side of the major axis as A, B, C respectively.
(A) (3, 0) and (0, 2)
Prove that, the normals to the ellipse drawn at the
8 2 161 points P, Q and R are concurrent. (2000)
9 8
(B) − , and − ,
5 15 5 5
Q.12 Prove that, in an ellipse, the perpendicular from
8 2 161 a focus upon any tangent and the line joining the
(C) − , and (0, 2) centre of the ellipse of the point of contact meet on the
5 15
corresponding directrix. (2002)
9 8
(D) (3,0) and − ,
5 5 Q.13 Let P(x1, y1) and Q(x2, y2), y1<0,y2<0, be the end
points of the latus rectum of the ellipse x2 + 4y2 = 4. The
Q.6 The orthocenter of the triangle PAB is equations of parabolas with latus rectum PQ are (2008)
8 7 25 11 8 8 7 (A) x2 + 2 3 y =
3+ 3 (B) x2 − 2 3 y =
3+ 3
(A) 5, (B) , (C) , (D) ,
7 5 8 5 5 25 5 (C) x2 + 2 3 y =
3− 3 (D) x2 − 2 3 y =
3− 3
(C) Equation of ellipse is x2 + 2y2 = 4 Q.19 A vertical line passing through the point (h, 0)
(
(D) The foci of ellipse are ± 2,0 ) intersects the ellipse
x2 y 2
4
+
3
= 1 at the points P and
(A) 2x − 5 y − 20 =
0 (B) 2x − 5 y + 4 =
0 dm 1
(D) = for y1 > 0
dx1 3
(C) 3x − 4y + 8 =0 (D) 4x – 3y + 4 = 0
Q.22 If the tangents to the ellipse at M and N meet at
x2 y 2 R and the normal to the parabola at M meets the x-axis
Q.18 The ellipse E1 : + = 1 is inscribed in a
9 4 at Q, then the ratio of area of the triangle MQR to area
rectangle R whose sides are parallel to the coordinate of the quadrilateral MF1NF2 is (2016)
axes. Another ellipse E2 passing through the point (0, 4) (A) 3 : 1 (B) 4 : 5
circumscribes therectangle R. The eccentricity of the
ellipse E2 is (2012) (C) 5 : 8 (D) 2 : 3
2 3 1 3
(A) (B) (C) (D)
2 2 2 4
M a them a ti cs | 11.55
PlancEssential Questions
JEE Main/Boards JEE Advanced/Boards
Exercise 1 Exercise 1
Q.6 Q.13 Q.16 Q.3 Q.5 Q.7
Q.19 Q.21 Q.22 Q.9 Q.12 Q.15
Q.23 Q.25 Q.27 Q.17 Q.26
Exercise 2 Exercise 2
Q.6 Q.8 Q.11 Q.1 Q.4 Q.6
Q.14 Q.15 Q.21 Q.10 Q.13
Q.23 Q.25
Previous Years’ Questions
Previous Years’ Questions Q.1 Q.4 Q.11
Q.2 Q.5 Q.6 Q.12
Answer Key
JEE Main/Boards 2 14
Q.10 slope = ± ; length =
3 3
Exercise 1
Q.11 (2, 1)
x2 y 2 (r + s)2
Q.22 + 1
= Q.23 4x=
2 2
y 25y 2 + 4x2
a2 (ra + sb)2
Exercise 2
Single Correct Choice Type
Q.25 A
JEE Advanced/Boards
Exercise 1
Q.6 (a) 4x − 3y + 3 21 =
0 ; 4x − 3y − 3 21 =
0 Q.8 x + y − 5= 0, x + y + 5= 0
π 5π
Q.9 θ = or ; 4x ± 33y − 32 =
0 Q.10 24 sq. units
3 3
1 1
Q.11 , Q.13 y −=
3 0 & x +=
y 5
2 2
x2 y21
Q.14 55 2 sq. units Q.19 + =
4 4
a b c2
±ax a4 + a2b2 + b 4
Q.20 by = Q.23 r 2 − b2
Exercise 2
Single Correct Choice Type
±a2 ±b2
Q.5 D Q.6 C Q.7 A Q.10 ,
2 2
a +b a2 + b2
Solutions
9x + 4 y − −
2 0
=
4 4
Exercise 1
2 2
15 15
or 9x2 + 4 y − =
Sol 1: 2a = 10 4 2
⇒a=5 2
15
4 4 2 y −
ae = 4 ⇒ e = = x 4
a 5 or + 1
=
2 2
5 15
\b2 = a2(1 – e2)
2 4
16
b2 = 25 1 − =9
25 15 5
∴a= ,b=
\b = ± 3 4 2
x2 y 2
Equation of ellipse is + 1 or 9x2 + 25y2 = 225
= b2 25 16 4 5
25 9 e2 = 1 − =1– × =1– =
a2 4 225 9 9
Sol 2: 9x2 + 4y2 – 30y = 0 5
∴e =
15 3
9x2 + 4 y 2 − y = 0
2
1 1 . 5 8 | Ellipse
\ y = 3 or y = – x + 5 2b2
Sol 7: Length of latus rectum =
a
i.e. y – 3 = 0 or x + y = 5
2b2
(a) If =a
a
x2 y 2 x2 y 2
Sol 4: + 2 ⇒
= + 1
= ⇒ 2b2 = a2
5 4 10 8
b2 1 1 1
let φ be eccentric angle ∴ = or e2 =1 − =
a2 2 2 2
∴ Any point on ellipse will be (a cosφ, b sinφ) 1
∴ e=
2
\P= ( 10 cos φ, 8 sin φ )
2b2
(b) if =b
∴ a
( 10 cos φ)2 + ( 8 sin φ)2 =3
⇒ 2b = a
⇒10cos2φ + 8sin2φ = 9
∴ 4b2 = a2
or 2cos2φ = 1 b2 1
or −
1 a 2 4
or cos2φ =
2 1 3
∴ e2 = 1 − =
1 π 3π 4 4
\cosφ = ± or φ = ,
2 4 4 3
∴ e=
2
x2 y 2
Sol 5: + 1
= x2 y2
4 6 Sol 8: + 1
=
a2 b2
Since midpoint of chord is (0, 0)
lx + my + n = 0
∴ Take one point as (a cosα, b sinα)
M a them a ti cs | 11.59
n
y=– x− y = mx ± 25m2 + 4
m m
x2 y 2
Equation of normal equation of tangent for + 1 is
=
16 16
(a2 − b2 )m'
⇒ y = m’x y = mx ± 4 m2 + 1
2 2 2
a + b m'
\25m2 + 4 = 16m2 + 16
2 2
-l (a − b )m' n 2
∴ =m' ; =– or 9m2 = 12 or m = ±
m 2
a + b (m')2 2 m
3
2 4
(a2 − b2 )
equation tangent is y = ± x ± 4 +1
n m 3 3
∴ =
m 2 2 4 7
a2 + b2 ×
y=± x±
m2 3 3
a2m2 + b2 2 4 7
⇒ = (a2 – b2) y intercept =
m n 3
2m2 x intercept = 2 7
∴ a m + b l = (a – b )
2 2 2 2 2 2 2
2
n
16
a 2
(a − b ) b 2 2 2 2 ∴ Length = 4×7 + ×7
⇒ + = 3
2 m2 n2
7 × 28 7×2 14
Sol 9: Normal of P(a cosθ, b sinθ) = = =
3 3 3
x2 y 2
+ 1
=
14 5 x2 y 2
Sol 11: Point on curve + 1
=
a2 x b2 y 6 3
− = a2 – b2
acos θ b sin θ is ( 6 cos θ, 3 sin θ)
∴ Equation of normal is 1 2
= (a + b2 )
2
a2 x b2 y
– = a2 – b2
−ae b2 Hence proved.
±
a
a2 x b2 y Sol 14: Let P = (acosθ, bsinθ)
or − = a2 – b2
−ae b2 b
± Slope of tangent = –
a atanθ
if they passed through (0, b) ∴ Slope of normal to tangent
⇒ ab = a2 – b2 atan θ
=
2
b
b2 b2
⇒ − 1 = ∴ Equation of line
a2 a2
a
FN = (y) = tanθ(x – ae) …(i)
⇒ (–e ) = 1 – e
2 2 b
⇒ e4 + e2 =1 And equation of CP
b
y= tan θx …(ii)
x2 y2 a
Sol 13: + 1
=
a2 b2 bx a
∴= (x − ae)
Let point be (a cosφ, b sinφ) a b
or (a2 – r2)m2 = r2 – b2 1 h2 1 k2
⇒ − + − 0
=
a2 a4 b2 b4
r 2 − b2
∴m= h2 k2 1 1
a2 − r 2 or + = +
4 4 2
a b a b2
r 2 − b2 (r 2 − b2 )
∴ tanθ = or θ = tan–1 1 2
a2 − r 2 (a2 − r 2 ) Sol 20: Equation of chord whose middle point is ,
is 2 5
2 2
1 2 1 2
Sol 17: Let (h, k) be midpoints of chords, x× y×
2 + 5 = + 5
2
∴ Equation of chord with midpoint (h. , k) is 25 16
25 16
xh 4k h2 k2
+ −1 = + −1 x y 1
a2 b2 a2 b2 ∴ + =
50 40 50
It passes through (0, b) 4
y=– (x – 1)
h2 k2 k x2 y 2 y 5
\ Equation is: + =or + = 2
a2 b2 b a2 b2 b 4 2
2 (x − 1)
x 5
∴ + 1
=
Sol 18: Let P be (h, k) 25 16
∴ Chord of contact is
⇒ x2 + x – 12 = 0
xh + yk – r2 = 0
∴ x1 = – 4 and x2 = 3
hx r 2
or y = – + ∴ Length of chord
k k
c2 = a2m2 + b2
L= (x1 − x2 )2 + (y1 − y 2 )2
r4 r2 b2 r2
or = × + 16 7
k2 a2 k2 b2 = (x1 − x2 ) 1 + m2 = 7 1 + = 41
25 5
x2 y2
or r2 = + is locus of P
a2 b2 Sol 21: Let F = (ae, 0) & p = (acosθ, bsinθ)
Radius of circle
Sol 19: We have to find the locus of P(h, k) such that the
1 2 2
chord to contact subtends 90º at centre the equation of = b sin θ + a2 (e − cos θ)2
chord of contact is 2
1 2 2
hx ky = b sin θ + a2e2 − 2aecos θ + a2 cos2 θ
+ 1
= …(i) 2
a2 b2 1 2
= a − b2 cos2 θ + a2 cos2 θ − 2aecos θ
the equation of the straight line joining the centre of 2
1 1 . 6 2 | Ellipse
1 2 2 a2m2 + b2
= a e cos2 θ − 2aecos θ + a2 ± =r
2
m2 + 1
1 1
= (a − aecos θ)2 = (a – ae cosθ)
2 2 or a2m2 + b2 = r2(m2 + 1)
Radius of auxillary circle = a m2(25 – r2) = r2 – 4
1
\r1 – r2 = (a + ae cosθ) r2 − 4
2 m=–
Centre of circle with FP as diameter 2r − r 2
since tangent lies in first quadrant m < 0
ae + cos θ b sin θ
=C= , (r 2 − 4)
2 2 \m2 =
25 − r 2
Distance between centre equation of tangent is
y = mx ± a2m2 + b2 or lx + my = n
x2 y2
or mx – y ± a2m2 + b2 = 0 It is touches + 1
=
a2 b2
it is tangent to circle a2l2 + b2m2 = n2
M a them a ti cs | 11.63
a2 × x12 b2 xy1 2 4 5
+ =1 ∴ e= 1− =
2
a (a + b) 2 2
b (a + b) 2 9 3
∴ r ∈ (2, 5) ∴ x – ey – e3a = 0
Sol 2: (D) 4x2 + 8x + 9y2 + 36y + 4 = 0 Sol 8: (A) Consider normal at positive end of
latus rectum from above equation of normal is
⇒ (2x + 2)2 + (3y + 6)2 = 36 x – ey – e3a = 0
It passes through (0, –b)
(x + 1)2 (y + 2)2
⇒ + =1 ∴ be – e3a = 0
9 4
⇒ b – e2a = 0
1 1 . 6 4 | Ellipse
b a aa2 a2
∴ = e2 Sol
⇒ y12:
=(B)
± ⇒ a2±⋅ x+±a2 a+2 b⋅ 2 + a2 + b2
x ±y =
a b bb2 b2
⇒ 1 – e2 = e4 or e2 + e4 = 1
a a4 +aa2b2 + ab44 + a2b2 + b 4
⇒ y =± x ±y =±
⇒ x±
b b b2 b2
Sol 9: (C) Tangent at P is
a 1 a 1
x cos θ y sin θ ⇒ y =± x ±y =±a4 +x a±2b2 + ab44 + a2b2 + b 4
⇒
+ –7=0 b b b b
a b
4
1 ⇒ yb =± ax
⇒ ±yb a= a2b±
±+ ax 2
+ ab44 + a2b2 + b 4
∴ CF =
cos2 θ sin2 θ
+
a2 b2 Sol 13: (C) 2ae = 6
P = (acosθ, bsinθ)
2b = 8
Equation of normal at
∴ ae = 3 & b = 4
ax by
P= − = a2 – b2 \a2 – b2 = 9 & b2 = 16
cos θ sin θ
\a2 = 25
(a2 − b2 )cos θ
∴ G= ,0
a 16 3
e= 1− =
25 5
b4 2 sin2 θ cos2 θ
\PG= b2 sin2 θ + cos2 θ = b +
Sol 14: (A) Slope of ellipse = – 1
a2 b2 a2
∴ Equation is y = – x ± 25 + 16
∴ PG. CF = b2
∴x+y=± 41
x
Sol 10: (A) y = + 2
2
Sol 15: (B) FBF’ is 90º
∴ c2 = a2m2 + b2
We know that BF = BF’ = a
1
⇒ 4 = 4 × + b2 ⇒ b = 3
4 ∴ 2a2 = 4a2e2
∴ The other common tangent has slope – m 1 1
∴ e2 = ⇒e=
1 2 2
∴c= =–2
1
− Sol 16: (C) Ellipse is
2
1
∴ Equation is y = – x –2 x2 y2
2 + 1
=
2 2
or x + 2y + 2 = 0 1 1
3 2
2b2 since a < 0
Sol 11: (A) =a+b
a 2a2 2×1 4
∴ Latus rectum is = =
or 2b2 = a2 + ab b 1 9
9×
2
a2 + ab – 2b2 = 0 a
Sol 17: (D) – ae = 8
e
b ± b2 + 8b2 1
a=– e=
2 2
b + 3b 3a 16
a=– =b ∴ = 8 ;a =
2 2 3
∴ Ellipse bulges to circle
M a them a ti cs | 11.65
1 b2 a
= 1− ae −
4 ∴ SN = cos θ = |aecosθ – a|
a2
16 16 3 1 + tan2 θ
\b2 = a2 (1 – l2) = × ×
3 3 4
8 SP = (ae − acos θ)2 + b2 sin2 θ
b=
3
16 = a2 (cos2 θ + a2 − b2 cos2 θ − 2a2ecos θ)
∴ Length of minor axis = 2b =
3
= (a cos θ)2 + a2 − 2a2ecos θ = |aecosθ – a|
Sol 18: (D) E(P) > 0 , E(Q) < 0
∴ SP = SN
C(P) < 0 and C(Q) < 0
x y
∴ P lies inside C but outside E Sol 22: (C) Equation of tangent is cos θ + sinθ = 1
3 2
T is x = 3 and T’ x = – 3
Sol 19: (C) Let P = (acosθ, bsinθ) ∴ Point of intersection of tangent & T let say
π π 2(1 − cos θ) θ
Q = acos + θ ,bsin + θ P = 3, − = 3,2 tan
2 2 sin θ 2
= (– asinθ, bcosθ)
2(1 + cos θ) θ
P’ = −3, = −3,2cot
alcosθ +mbsinθ + n = 0 sin θ 2
And ⇒ alcosθ + mbsinθ = – n → 1 – alsinθ + mbcosθ
∴ Equation of circle is
+ n = 0 & –alsinθ + mbcosθ = – n(2)
θ θ
squaring and adding 1 and 2 (x + 3) (x – 3) + y − 2 tan y − 2cot =0
2 2
we get a2l2 + m2b2 = 2n2 ∴ When y = 0
x2 – 5 = 0
Sol 20: (B) Q1 + Q2 = C
∴x=± 5
Point of intersection of tangent at (q1) and (q2) is
∴ It always passes through ( ± 5, 0)
θ + θ2 θ1 + θ2
acos 1 b sin i.e. it always passes through focus.
2 , 2
(x, y) =
cos θ1 − θ2 cos θ1 + θ2
Sol 23: (A) Q = (acosθ, asinθ)
2 2
P = (acosθ, bsinθ)
x y
∴ =
c C ∆ SPT is an isosceles triangle.
acos b sin
2
2
Sol 24: (C) Equation of normal in slope form is
\ Locus of P is a straight line
m(a2 − b2 )
y = mx
Sol 21: (A) P = (acosθ, bsinθ) a2 + b2m2
Q = (acosθ, asinθ)
m2 (a2 − b2 )2
equation of tangent at Q ∴ c2 =
a2 + b2m2
is (y – asinθ) = – 1tanθ (x – acosθ)
a Sol 25: (A) ( 2x − 2 2)2 + ( 3y − 3 3)2 = k
x + ytanθ – = 0 & (ae, 0)
cos θ
2(x − 2)2 3(y − 3)2
∴ + 1
=
k k
1 1 . 6 6 | Ellipse
⇒ c=
± 65 =65 or − 65 Sol 5: Since, angle FBF’ is right angled
y
Sol 3: (D) Equation of auxiliary circle is x2 + y 2 =
9 …(i)
B(0, b)
x y
Equation of AM is + = 1 …(ii)
3 1
y x’ x
F’ O F
(-ae,0) (ae,0)
) 125 , 95)
M
B(0,1)
y’
x’ x
N O A(3,0)
0 − b 0 − b
⇒ = −1
ae − 0 −ae − 0
M a them a ti cs | 11.67
1 ⇒ x2 + 16
=
⇒ a2 (1 − e2 ) =
a2e2 ⇒ e2 = a
2
1 a2b2 + b2 x2 − 2ab2 x
⇒ e= ⇒ x2 + 16
=
2 a2
PR r b2 2b2
Sol 6: Given, = ⇒ x 2 + b2 + x2 − .x =
16
RQ s y a2 a
b2 2b2
Q (a cos, a sin) ⇒ x2 1 + − x + b2 − 16 =
0
a2 a
R (a cos, )
P (a cos, a sin)
x’ x For unique solution
O (0,0)
4b2 b2
⇒
a2
− 4 1 + 12 − 16 =
a2
0 ( )
y’ b4 b4 16b2
⇒ = b2 − 16 + −
⇒ α − bsin θ = r a2 a2 a2
asin θ − α s
⇒ αs − b sin= b2
θ.s rasin θ − αr ⇒ b2 − 16 =
16
a2
⇒ αs +=
αr rasin θ + b sin θ.s
⇒ a2b2 − 16a2 =
16b2
⇒ α(s + r) = sin θ(ra + bs)
⇒ α=
sin θ(ra + bs) (
⇒ a2b2 = 16 a2 + b2 ) (ii) ... (ii)
r+s
x2 y 2
Similarly (i) is tamest to ellipse + 1 are will get
=
Let the coordinate of R be (h, k) 25 4
the relatiesn
⇒=h acos θ
b2 4a2 + 25b2
a2= (3) ... (iii)
(ar + bs)sin θ
and k = α =
r+s Solving (i) (ii) we get a = 2 7
h k(r + s)
⇒ cos
= θ , sin
= θ 7
a ar + bs b=4
3
On squaring and adding, we get
x y
⇒ Eq. of tangent +
h2 k 2 (r + s)2 2 7 7
sin2 θ + cos2 =
θ + 4 =1
a2
(ar + bs) 2 3
14
h2 k 2 (r + s)2 Distances = a2 + b2 =
1
⇒= + 3
a2 (ar + bs)2
Focus (S = 6, 2)
x2 y 2 (r + s)2
Hence, locus of R is + 1.
=
a2 (ar + bs)2
P(2,1)
1
v’ v
A 2 A 2 (4,0)
r
a
− ae =
4
e
9 7
1 a=4,b =3,e =1 − ⇒
a 2 − =
4 16 4
2
a= .
8 Focii is ( ± ae, 0 ) (
⇒ ± 7 ,0 )
3
( ae)
2
=r + b2
Sol 9:
x2 y 2 7+9 = 4
x2 + 4y 2 = 4 ⇒ + = 1 ⇒ a = 2,b = 1 ⇒ P = ( 2,1 )
4 1
Now equation of circle is ( x − 0 ) + ( y − 3) =
2 2
16
x y2 x2 y 2
Required Ellipse is 2 + 2 =1 ⇒ 2 + 2 =1 x2 + y 2 − 6y − 7 =0
a b 4 b
(2, 1) lies on it
Sol 13: (A)
4 1 1 1 3 4
⇒ + =1 ⇒ =1 − = ⇒ b2 = x2 y2
16 b2 b 2 4 4 3 Here ellipse is + = a2 6,b
1 , where= = 2
2
a2 b2
x2 y2 x2 3y 2 Now, equation of any variable tangent is
∴ + =1 ⇒ + =1 ⇒ x2 + 12y 2 =16
16 4 16 4
mx ± a2m2 + b2
y= ….(i)
3
where m is slope of the tangent
3 3a2
2
Sol 10: (D) b = a 1 − e 2
( 2
) 2
= a 1 − = a2 =
2
5 5 5
So, equation of perpendicular line drawn from
−x
x2 y2 9 5 centre to tangent is y = ….(ii)
+ =1 ⇒ + =1 m
a2 b2 a2 3a2 Eliminating m, we get
(x )
2
2
32 + y2 = a2 x2 + b2 y 2
a2 =
3
( )
2
2 32 ⇒ x2 + y 2 = 6x2 + 2y 2
b =
5
∴ Required equation of ellipse 3x2 + 5y 2 − 32 =
0
x2 y 2
Sol 14: (D) + 1
=
9 5
Sol 11: (D) Semi minor axis b = 2
b2
Semi major axis a = 4 =a 3,=
b 5 ae,
a
x2 y2
Equation of ellipse = + =1 b2 5 5
a2
b 2 = , 2,
a 3 3
x2 y 2
⇒ + 1
=
16 4 5 2
e= 1− =
9 3
⇒ x2 + 4y 2 =
16 .
M a them a ti cs | 11.69
JEE Advanced/Boards x1 =
a(2a2 − k 2 )
2a2 + k 2
Exercise 1
(x1)2 + 2(y1)2 = a2
Sol 1: Let X = x – 1 x 2 y 12
∴ 1 + a2
=
And Y = y – 2 a2 a 2
∴ Centre = (0, 0) 2
= (3, 4)
Sol 3: Equation of auxiliary circle is
F1P + F2P = 2a x2 + y2 = a2 …. (i)
= 80 + 20 = 2a Equation of tangent at P(α) is
3 20 = 2a x cos α y sin α
+ 1
= …(ii)
a b
∴ a=3 5
Equation of pair of lines OA, OB is obtained by making
homogenous equation of i w. r. t. (ii)
ae = 5
2
x y
\a2 – b2 = 25 ∴ x + y = a a cos α + b sin α
2 2 2
∴ b2 = 20
2xyasin α cos α
b= 2 5 ∴(1 – cos2α)x2 –
b
x2 y2 x2 y 2
∴ Equation of ellipse is + 1 or
= + 5
=
a2 b2 9 4 a2
+y 1 − sin2 α =
2 0
2
(x − 1)2 (y − 2)2 b
⇒ + =5
9 4 But ∠AOB = 90º
4x2 + 9y2 – 8x – 36y – 175 = 0 ∴ coeff of x2 + coeff of y2 = 0
a2
Sol 2: Let T = (h, k) ∴ 1 – cos2α + 1 – sin2α = 0
b2
∴ AP is a2 − b2
1= sin2 α
2
xh + yk = a 2 b
1 1 . 7 0 | Ellipse
a2e2 γ δ α β c − a −(c + a)
1= sin2α tan tan tan tan = × =1
2 2 2 2 c + a −(c − a)
a2 (1 − e2 )
1 3
⇒ e2 = or e = (1 + sin2α)–1/2 Sol 6: (a) m of line = –
(1 + sin2 α ) 4
4
∴ Slope of line ⊥ to given line =
Sol 4: (–3, 1) = (a cosa1, bsina1) 3
(a1 – α) = (a cosa2, bsina2) x2 y2
Equation of ellipse is + 0
=
(3)2 5
9 1 4 4
∴ + 1&
= + 1
= 2
a2 b2 a2 b2 4 4
∴ Equation of tangent is y = x ± 9× +5
3 3
32 4x
∴ =3 y= ± 21
a2 3
32 32 ∴ 3y = 4x ± 3 21
∴ a2 = & b2 =
3 5
(b) Equation of normal to the ellipse is axsecθ – by
x2 y 2 cosecθ = a2 – b2
∴ Equation of ellipse is + =1
32 32
ax by
3 5 or − =a2 − b2
∴ 3x2+ 5y2 = 32 cos θ sin θ
ax by
the normal given is + c
=
Sol 5: Let α and β form a chord which interests the 3 4
major axis at (c, 0)
3 4 a2 − b2
∴ Equation of chord is ∴= =
cos θ − sin θ c
α β a2 x b2 y
2coscos − = a2 – b2
⇒ 2 2 = a+c ae b2 / a
α β a−c
−2sin sin For centre y = 0
2 2
a2 b2
α β c−a ∴ x centre = 1 − 2 = ae3
⇒ tan tan = a a
2 2 c+a
Similarly let γ and δ intersect major axis at (– c, 0) ∴ Equation of circle is
2
b2
γ δ −c − a (x – ae ) + y = (ae – ae ) +
3 2 2 3 2
∴ tan tan = 9
2 2 a−c
M a them a ti cs | 11.71
b2
2 4
y=– x±8
(x – ae ) + y = (ae – ae ) +
3 2 2 3 2
3
9
3
∴A=± ×8=±6&B=±8
x – 2ae + a e + y = (ae – ae ) + a (1 – e )
2 3 2 6 2 3 2 2 2 2
4
∴ x2 – 2ae 3 + y2 = a2(1 – e2 – e4) 1 1
∴ Area of A = A ×B = × 6 × 8 = 24
2 2
2
4 4
y=– x± 18 × + 32
3 3
C B
1 1 . 7 2 | Ellipse
1
2
AB || CD ∴ Quadrilateral is trapezium
y −
x2 3 1
∴ + =1 Area = × h (AB + CD)
4a 2
4a2 2
3 9 1 6
= × 10 × 8 2 + = 55 2 sq. units
b2 1 2 2 2 2
\ e2 = 1 – =1– = ⇒e=
a2 3 3 3
Sol 15: Equation of normal at P(acosθ, bsinθ) is
Sol 13: Let equation of tangent be ax by
− = a2 – b2
cos θ mnθ
y = mx ± 16m2 + 9 is passes through (2, 3)
cos θ 2
2 G= (a − b2 ), 0
∴ (3 –2m) = ± 16m + 9 a
⇒ 4m2 – 12m + 9 = 16m2 + 9 − sin θ 2
And g = 0, (a − b2 )
b
⇒ 12m2 + 12m = 0
∴ a2CG2 + b2 (Cg)2
m = 0 or m = – 1
cos2 θ sin2 θ
Rechecking we get when m = 0 C > 0 & when m = – 1 = a2 × (a2 – b2)2 + b2 (a2 – b2)2
C>0 a2 b2
= (a2 – b2)2
∴ Equation of tangent is
b2
y = 3 and y = – x + 5 or x + y = 5. CG = acosθ 1 −
a2
1
Sol 14: Let y = mx + be tangent to parabola y2 = 4x. = acosqe2 = e2 × abscissa of P
m
x2 y2
It will touch ellipse + 1 if
= Sol 16: ∴ (ae + rcosθ, rsinθ) lies on ellipse
42 ( 6 )2
1 (ae + r cos θ)2 r 2 sin2 θ
= 16m2 + 6 ∴ + =1
m2 a2 b2
⇒ 16m4 + 6m2 – 1 = 0 cos2 θ sin2 θ 2ecos θ
∴ + r2 + r + e2 – 1 = 0
a2 b 2 a
⇒ (8m2 – 1) (2m2 + 1) = 0
b2
x2 + y2 =1 ∴ tanatanβ= –
a2 b2 a2
Y’ the point of intersection to tangents is
It touches circle x2 + y2 = c2
4a2b 4
= | −1 |
(b2 cos2 θ + a2 sin2 θ)2 ∴ =c
h2 k2
+
2ab2 a4 b4
∴ (r1 – r2) =
(b2 cos2 θ + a2 sin2 θ)
h2 k 2
∴ 1 = c 4 + 4
2
x cos θ y sin θ a b
Sol 17: The tangent at P is + 1
=
a b x2 y2 1
where p = (acosθ, bsinθ) on + = is locus of P.
4 4
a b c2
a
∴ T= ,0 & N = (acosθ, 0)
cos θ
Sol 20: Equation of tangent to ellipse
equation of circle with TN as diameter is
x2 y2
a + 1
=
x − (x – acosθ) + y = 0 a2 + b2 b2
2
cos θ
y = mx ± (a2 + b2 )m2 + b2
1
⇒ x2 – a + cos θ x + y2 + a2 = 0
cos θ It is also tangent to the ellipse,
Equation of auxiliary circle is x + y – a = 0 2 2 2
x2 y2
+ =1
a 1 a2 a2 + b2
2gg1 + 2ff1= 2 + cos θ ×0+2×0×0 = 0
2 cos θ \c2 = a2 m2 + (a2+ b2)
c1 + c2 = a2 – a2 = 0 ∴ (a2+ b2)m2 + b2 = a2m2 + (a2+ b2)
The two circle as orthogonal
1 1 . 7 4 | Ellipse
( x − 1) ( y − 2)
2 2
x=
2
& y=
3 Now, C2 = a2 − b2 ⇒ a2 − b2 = 25 (3) (iii)
cos θ 2sin θ
Solving (ii) and (iii)
4 9
∴ + 1
= Substituting in (i)
x2 4y 2
( x − 1) ( y − 2)
2 2
\16y2 + 9x2 = 4x2y2 + 1
=
45 20
1 1 . 7 6 | Ellipse
Sol 4: (A) Let the lines be x-axis & y-axis Sol 8: (B) Let P = (h, k)
Let P = (h, 0) and Q = (0, k) Foot of perpendicular from focus to any tangent of the
ellipse lies on its auxiliary circle.
h2 + k2 = (a + b)2
∴ Midpoint of P & S lies on auxiliary circle
bh ak
(x, y) = ,
(a + b) (a + b) h ± ae k
∴ M= ,
2 2 2 2
x y
+ = 1 2 2
b a h±ae k 2
⇒ + =a .
2 2
∴ Equation of P is an ellipse
2 2
x±ae y 2
Sol 5: (A) Now sum of distance of points from two
⇒ + =a .
2 2
foci = constant = 2a for an ellipse
∴ Necessary length of string = 2a = 6 Sol 9: (C) Equation of normal is
a = 3 and b = 2 ax secθ – by cosecθ = a2 – b2
Distance between pins = 2ae
a2 − b2
Q= cosθ
b2 6× 5 a
= 6 1− = = 2 5
a 2 3
a2 − b2
R=– sinθ
Sol 6: (B) It is a known property that b
SF1·SF2 = b2 = 3
a2 − b2 (a2 − b2 )
M = (x, y) = × cos θ, − sin θ
2a 2b
Sol 7: (B) Ellipse 1 is
∴ Locus of M is
x2 y2
+ 1
=
2 2 (a2 − b2 )2
3 3 (ax)2 + (by)2 =
4
a b
x2 y2
∴ + 0
=
And E2 is a2 − b2
2
a2 − b2
2
x2
+
y2
1
= 2a 2b
2 2
3 3 coeff of y is > coeff of x.
a+b a−b 2
a2 − b2
2a b2
are of ellipse = pab e’ = 1 – =1– =0
2
a2 − b2 a2
∴ pa1b1 = pa2b2
2b
3 3
⇒ =
ab a2 − b2 Sol 10: (C) Equation of ellipse is
⇒ a – b = ab
2 2
x2 y2
+ 1 and of a circle is
=
a a
2
a2 b2
⇒ − −1 =0
b b x2 + y2 = a2 – b2
∴ a 1+ 5 a2 − b2 − y 2 y2
= ∴ + 1
=
b 2 a2 b2
M a them a ti cs | 11.77
9 α −β
62 + ×4 cos
4 3 5 12 1
= 2
= = ×4 =
9 5 5 e α+β
1− cos
4 2
12 α −β α+β
\θ = tan–1 cos − cos
5 1−e 2 2
5 =
12 1+e α −β α+β
other angle = π – tan–1 = π – cot–1 12 cos + cos
5 2 2
x’ x
-1 1
Sol 2: (B) Given tangent is drawn at (3 3 cos θ, sin θ)
O
x2 y 2
to + 1.
=
27 1
y’
1 1 . 8 0 | Ellipse
2 ⇒ x + 3y = 3 …(i)
1 2 1
⇒ x − + (y − 1) = x −1 Equation of the straight line perpendicular to AB
2 2
5.
through P is 3x − y =
1
2
1 Equation of PA is x − 3 =0.
⇒ x − + (y − 1)2 = (x − 1)2
2 4 The equation of straight line perpendicular to PA
9 8 8
1 1 2 through B − , is y = .
⇒ x2 − x + + y 2 − 2y +=
1 (x − 2x + 1) 5 5 5
4 4
11 8
Hence, the orthocenter is , .
⇒ 4x2 − 4x + 1 + 4y 2 − 8y + 4 = x2 − 2x + 1 5 5
⇒ 3x2 − 2x + 4y 2 − 8y + 4 =
0 1
Sol 7: (A) Equation of AB is y − 0 =− (x − 3)
3
1 1
2
2
⇒ 3 x − − + 4(y − 1)2 =
0 x + 3y − 3 =0 x + 3y − 3 = 10[(x − 3)2 + (y − 4)2 ]
3 9
(Look at coefficient of x2 and y 2 in the answers).
2
1 1
⇒ 3 x − + 4(y − 1)2 =
3 3 Sol 8: Let the coordinates of point P be (acosθ, bsinθ).
2
Then equation fo tangent at P is
1
x − x y
3 (y − 1)2 cos θ + =1 …(i)
⇒ + 1
= a bsin θ
1/9 1 / 12
We have , d = length of perpendicular from O to the
tangent at P.
Comprehenesion Type
y
Sol 5: (D) Figure is self explanatory.
y’
0 + 0 −1
Sol 6: (C) Equation of AB is d=
cos2 θ sin2 θ
+
P(3, 4)
a2 b2
1 cos2 θ sin2 θ
) )
9 8
5‘5 B A(3, 0)
⇒
=
d a2
+
b2
1 cos2 θ sin2 θ
⇒
= +
d2 a2 b2
8 b2
y −0
= 5 (x − 3) We have, to prove (PF1 − PF2 )2 = 4a2 1 − .
d2
9
− −3 b
2
5 Now, RHS = 4a2 1 −
d2
8
= (x − 3) 4a2b2
−24 = 4a2 −
1 d2
⇒ y=
− (x − 3)
3
M a them a ti cs | 11.81
Sol 9: Given, x2 + 4y 2 =
4 k 2 1 h2 k 2
And coefficient of y 2 = − + − 1
x2 y 2
9 3 6 3
or + 1
= …(i)
4 1 k 2 h2 k 2 1 h2 1
= − − + =− +
Equation of any tangent to the ellipse on (i) can be 9 18 9 3 18 3
written as
Again, coefficient of x2 + coefficient of y 2
x
cos θ + y sin θ =1 …(ii) 1 2 1 1
2 = − (h + k 2 ) + +
18 6 3
Equation of second ellipse is
1 1
y =− (9 cos2 θ + 9 sin2 θ) +
A 18 2
9 1
= − + =0
Q 18 2
3
P
1 Which shows that two lines represent by equation (v)
x’
-2 O 2
x are at right angles to each other.
6
- 6
-1
- 3
y’
x2 + 2y 2 =
6
x2 y 2
⇒ + 1
= …(iii)
6 3
1 1 . 8 2 | Ellipse
Sol 10: Let the coordinates of P be (acos θ, bsin θ) . sec2 θ(a2 tan2 θ + b2 − 2a2 tan2 θ)
Equations of tangents at P is =
y (a2 tan2 θ + b2 )2
C
2 2 2 2 2 2
sec θ(a tan θ + b ) − tan θ(2a tan θ sec θ)
f '(θ) = y’
(a2 tan2 θ + b2 )2
[It is given that P, Q, R are on the same side of x-axis as
A, B and C].
M a them a ti cs | 11.83
Equation of the normal to the ellipse at P is The equation of line perpendicular to tangent is,
ax by x sin θ y cos θ
− =a2 − b2 or ax sin θ − by cos θ − =
λ
cos θ sin θ b a
1 Since, it passes through the focus (ae, 0), then
= (a2 − b2 )sin2θ ……(i)
2 aesin θ
− 0 =λ
Equation of normal to the ellipse at Q is b
2π 2π ⇒ λ =aesin θ
ax sin θ + − by cos θ + = b
3 3
x sin θ y cos θ aesin θ
∴ Equation is − = …(i)
1 2 4π b a b
(a − b2 )sin 2θ + …(ii)
2 3 Equation of line joining centre and point of contact
Equation of normal to the ellipse at R is P(acos θ, b sin θ) is
b
4π 4π =y (tan θ)x …(ii)
ax sin θ + − by cos θ + = a
3 3
Point of intersection Q of Eqs. (i) and (ii) has x coordinate
1 2 8π a
(a − b2 )sin 2θ + …(iii) .
2 3 e
a
4π 2π 2π Hence, Q lies on the corresponding directrix x = .
But sin θ + = sin 2π + θ − = sin θ − e
3 3 3
x2 y 2
4π 2π 2π Sol 13: (B, C) + 1
=
and cos θ + = cos 2π + θ − = cos θ − 3 4 1
3 3
8π 4π 4π
(
b2 a2 1 − e2
= )
and sin 2θ + = sin 4 π + 2θ − = sin 2θ −
3 3 3 3
⇒e=
Now, eq. (iii) can be written as 2
1 1
ax sin(θ − 2π / 3) − by cos(θ − 2π / 3) = ⇒ P 3, − and Q − 3, −
2 2
1 2
(a − b2 )sin(2θ − 4 π / 3) …(iv) (given y1 and y 2 less than 0).
2
For the liens (i), (ii) and (iv) to be concurrent, we must Co-ordinates of mid-point of PQ are
have the determinant 1
R ≡ 0, − .
2
1 2
asin θ −bcos θ (a − b2 )sin2θ
2 PQ = 2 3 = length of latus rectum.
2π 2π 1 2 4π
∆1 asin θ +
= −bcos θ + (a − b2 )sin 2θ + = 0 ⇒ two parabola are possible whose vertices are
3 3 2 3
2π 2π 1 2 4π
asin θ − −bcos θ − (a − b2 )sin 2θ − 3 1 3 1
3 3 2 3 0, − − and 0, − .
2 2
2 2
Thus, line (i), (ii) and (iv) are concurrent. Hence the equations of the parabolas are
x2 − 2 3y =
3+ 3
Sol 12: Any point on the ellipse
x2 y2 And x2 + 2 3y =
3− 3 .
+ 1 be P (acos θ, bsin θ) .
=
a2 b2
Sol 14: (A, B) Ellipse and hyperbola will be confocal
The equation of tangent at point P is given by;
1 2 sec θx − 3tan θy =6
⇒±a× ,0 ≡ ( ± 1, 0 )
2 It is also tangent to circle x2 + y 2 − 8x =
0
1
⇒ a =2 and e =
2 8 sec θ − 6
⇒ 4
=
2 2
(
⇒ b = a 1 − e ⇒b = 1 2
) 2 4 sec2 θ + 9 tan2 θ
∴ Equation of ellipse
x2 y 2
+ 1
=
( 8 sec
= θ − 6)
2
(
16 13 sec2 θ − 9 )
2 1
⇒ 12 sec2 θ + 8 sec θ − 15 = 0
Sol 15: (D) Equation of line AM is x + 3y −3 = 0 5 3 5
⇒ sec θ = and − but sec ≠
3 6 2 6
Perpendicular distance of line from origin =
10 3 5
⇒ sec θ = − and ⇒ tan θ =
9 9 2 2
Length of AM = 2 9 − =2×
10 10 ∴ Slope is positive
1 9 3 27 Equation of tangent = 2x − 5 y + 4 = 0
⇒ Area = ×2× × = sq. units
2 10 10 10
Sol 18: (C)
Sol 16: A → p; B → s, t; C → r; D → q, s y
1 h2 (0, 4)
(p) = 4 1 +
k2 k 2
(-3, 2) y=2
x=3
⇒= (
1 4 k 2 + h2 ) x = -3
x
2
1 (-3, -2) y = -2 (3, -2)
∴ h2 + k 2 =
which is a circle.
2
(q) If z − z1 − z − z 2 =where
k k < z1 − z 2 the locus is
a hyperbola.
⇒x
= 3 cos 2 α and sin2 α =y ( y + 2)( y − 2) + λ ( x + 3)( x − 3) =0
x 4
or cos 2 α = and sin 2 α =y It passes through (0, 4) ⇒ λ =
3 3
x2 2 x2 y 2
∴ y
+= sin2 2α + cos2=
2α 1 which is an ellipse. Equation of ellipse is + 1
=
3 12 16
(s) If eccentricity is 1, ∞ ) , then the conic can be a 1
e= .
parabola (if e = 1) and a hyperbola if e∈ (1, ∞ ) . 2
(t) Let z =
x + iy; x, y ∈ R
Sol 19: (9)
⇒ ( x + 1 ) − y 2 = x2 + y 2 + 1
2
x2 y 2
⇒ y2 =
x ; which is a parabola. + 1
=
4 3
xx1 4
PQ is chord of contact, so =1 ⇒ x = d 1
4 x1 = 1−
dx1 3x12
which is equation of PQ, x = h
4 4 dm 1 dm 1 dy1 x1
So =h ⇒ x1 = = = , =
x1 h dy1 3 dx1 3 dx1 3 x2 − 1
1
1
∆ (h) = area of ∆ PQR = × PQ × RT
2 Sol 22: (C)
1 2 3 3
( )
3/2
= × 4 − h2 × ( x1 − h) = 4 − h2 x×3 y 6
2 2 2h Equation of tangent at M is + 1
=
2×9 8
=∆ ' (h)
(
− 3 4 + 2h2 ) 4 − h2
Put y = 0 as intersection will be on x-axis.
∴ R ≡ ( 6, 0 )
2
2h
which is always decreasing. Equation of normal at M is
3
45 5 1 3 3
So ∆1 = maximum of ∆ (h) = at h = 3
X +=
y 2 +
8 2 2 2 2
9
∆2 = minimum of ∆ (h) =at h = 1 3 7
2 Put y = 0, x = 2 + =
2 2
8 8 45 5 9 7
So ∆1 − 8 ∆2 = × − 8. = 45 − 36= 9 ∴ Q ≡ , 0
5 5 8 2 2
1 7 5
∴ Area ( ∆ MQR ) = × 6 − × 6 = 6 sq. units.
Sol 20: (D) 2 2 4
Area of quadrilateral
The equation of P1 is y 2 − 8x =
0 and P2 is y 2 + 16 x =
0
Tangent to y 2 − 8x =
0 passes through (-4, 0) (MF1NF2 ) =
2 × Area ( ∆ F1 F2 M)
1
2 1 = 2× ×2× 6 = 2 6
⇒ 0= m1 ( −4 ) + ⇒ = 2 2
m1 m12
5/4 5
∴ Required Ratio = =
Also tangent to y 2 + 16 x =
0 passes through 2 8
(2, 0)
4
⇒ 0= m2 × 2 − ⇒ m22 = 2
m2
1
⇒ + m22 =
4
m12
1
3x1 +
x1 y
For=
centroid = ,m 1
3 3
2017-18 100 &
op kers
Class 11 T
By E ran culty
-JE Fa r
IIT enior emie .
S fP r es
o titut
Ins
MATHEMATICS
FOR JEE MAIN & ADVANCED
SECOND
EDITION
Exhaustive Theory
(Now Revised)
Formula Sheet
9000+ Problems
based on latest JEE pattern
PlancEssential
Questions recommended for revision
12. HYPERBOLA
1. INTRODUCTION
A hyperbola is the locus of a point which moves in the plane in such a way that Z
the ratio of its distance from a fixed point in the same plane to its distance
X’ P
from a fixed line is always constant which is always greater than unity.
M
The fixed point is called the focus, the fixed line is called the directrix. The
constant ratio is generally denoted by e and is known as the eccentricity of the Directrix
hyperbola. A hyperbola can also be defined as the locus of a point such that S (focus)
the absolute value of the difference of the distances from the two fixed points Z’
(foci) is constant. If S is the focus, ZZ′ is the directrix and P is any point on the
hyperbola as show in figure. Figure 12.1
SP
Then by definition, we have = e (e > 1).
PM
Note: The general equation of a conic can be taken as ax2 + 2hxy + by 2 + 2gx + 2fy + c =0
This equation represents a hyperbola if it is non-degenerate (i.e. eq. cannot be written into two linear factors)
a h g
∆ ≠ 0, h2 > ab. Where ∆ = h b f
g f c
PLANCESS CONCEPTS
1. T he general equation ax2 + 2hxy + by 2 + 2gx + 2fy + c =0 can be written in matrix form as
a h g x
a h x
x y + 2gx + 2fy + c =0 and
x y 1 h b f y = 0
h b y
g f c 1
Degeneracy condition depends on the determinant of the 3x3 matrix and the type of conic depends on
the determinant of the 2x2 matrix.
2. Also the equation can be taken as the intersection of z =ax2 + 2hxy + by 2 and the plane
z= − ( 2gx + 2fy + c )
Vaibhav Gupta (JEE 2009, AIR 54)
1 2 . 2 | Hyperbola
Squaring both the sides, we get (x + c)2 + y2 = 4a2 + 2(2a). (x − c)2 + (y)2 + (x – c)2 + y2
2c
F1 F2
Focus Focus
Figure 12.3
(b) Centre: The midpoint of the line joining the foci is called the center of the hyperbola.
Center
Figure 12.4
M a them a ti cs | 12.3
(c) Transverse-Axis: The line through the foci is called the transverse axis. Length of the transverse axis is 2a.
Transverse Axis
2a
Figure 12.5
(d) Conjugate-Axis: The line segment through the center and perpendicular to the transverse axis is called the
conjugate axis. Length of the conjugate axis is 2b.
Conjugate Axis
2b
Figure 12.6
(e) Vertices: The points at which the hyperbola intersects the transverse axis are called the vertices of the
hyperbola. The distance between the two vertices is denoted by 2a.
Vertices
Figure 12.7
c
(f) Eccentricity: Eccentricity of the hyperbola is defined as and it is denoted by e. And e is always greater than
1 since c is greater than 1. a
a2
(g) Directrix: Directrix is a line perpendicular to the transverse axis and cuts it at a distance of from the centre.
c
a2 a2
i.e. x=± or y=±
c c
a2 a2
a
2 x= x=
y= c c
c 2 O Directrix
a O
y=
c Directrix
Figure 12.8
1 2 . 4 | Hyperbola
(h) L ength of The Latus Rectum: The Latus rectum of a hyperbola is a line segment perpendicular to the
transverse axis and passing through any of the foci and whose end points lie on the hyperbola. Let the length
of LF be . Then, the coordinates of L are (c, )
L(c,)
x2 y 2
Since, L lies on hyperbola − = 1.
a2 b2
latus rectum
c2 2
Therefore, we have − =1 O
a2 b2
b2 L’
2 c2 c2 − a2 b4 b2
⇒ − −1 = ⇒ =b =
2 2
⇒ Figure 12.9
b2 a2 a2 a2 a2 a
b2 b2 2b2
Latus rectum LL′ = LF + L′F = + =
a a a
x2 y 2
(i) Focal Distance of a Point: Let P(x, y) be any point on the hyperbola − = 1 as shown in figure. Then by
definition, a2 b2
Illustration 1: Find the equation of the hyperbola, where the foci are (±3, 0) and the vertices are (±2, 0).
(JEE MAIN)
Sol: Use the relation c2 = a2 + b2, to find the value of b and hence the
equation of the hyperbola.
O
We have, foci = (±c, 0) = (±3, 0) ⇒ c = 3
F1 4 F2
and vertices (±a, 0) = (±2, 0)
a=2
But c2 = a2 + b2 ⇒ 9 = 4 + b2 ⇒ b2 = 9 – 4 = 5 ⇒ b2 = 5 Figure 12.11
Here, the foci and vertices lie on the x-axis, therefore the equation of the
hyperbola is of the form
x2 y2 x2 y 2
− = 1 ⇒ − =1
a2 b2 4 5
Illustration 2: Find the equation of the hyperbola, where the vertices are (0, ±5) and the (0,5)
foci are (0, ±8). (JEE MAIN)
y2 x2 y 2 x2
hyperbola is of the form − = 1. i.e., − =1
a2 b2 25 39
which is the required equation of the hyperbola.
Illustration 3: If circle c is a tangent circle to two fixed circles c1 and c2, then show that the locus of c is a hyperbola
with c1 and c2 as the foci. (JEE MAIN)
C1 C2
r1 r2
r r
C(h, a)
Figure 12.13
Illustration 4: Find the equation of the hyperbola whose directrix is 2x + y = 1 and focus, (1, 2) and eccentricity 3 .
(JEE MAIN)
Illustration 5: Find the equation of the hyperbola when the foci are at (± 3 5 , 0), and the latus rectum is of
length 8. (JEE ADVANCED)
Sol: Use the formula for the length of the latus rectum to get a relation between a and b. Then use the foci and the
relation between a and b to get the equation of the hyperbola.
Here foci are at (± 3 5 , 0) ⇒ c= 3 5
2b2
Length of the latus rectum = =8
a
⇒ b2 = 4a … (i)
We know that
c2 = a2 + b2
F1 F2
O
(3 5)2 = a2 + 4a
(-3 5,0) 4 (3 5,0)
45 = a2 + 4a
a2 + 4a – 45 = 0
Figure 12.14
1 2 . 6 | Hyperbola
(a + 9)(a – 5) = 0
a = –9, a = 5 (a cannot be –ve)
Putting a = 5 in (i), we get
b2 = 5 × 4 = 20 ⇒ b2 = 20
Since, foci lie on the x-axis, therefore the equation of the hyperbola is of the form
x2 y2 x2 y 2
− = 1 i.e., − =1
a2 b2 25 20
Illustration 6: Find the equation of the hyperbola when the foci are at (0, ± 10 ), and passing through (2, 3)
(JEE ADVANCED)
Sol: Start with the standard equation of a hyperbola and use the foci and the point (2 , 3) to find the equation.
Here, foci are at (0, ± 10 )
⇒ c= 10 Here the foci lie at the y-axis.
So the equation of the hyperbola is of the form
y2 x2
−= 1 … (i)
a2 b2
Point (ii, iii) lies on (i).
9 4 9 4 9 b2 + 4 9b2
So − = 1 ⇒ =1+ ⇒ = a2 = … (ii)
a2 b2 a2 b2 a2 b2 b2 + 4
We know that
c2 = a2 + b2
9b2
⇒ 10 = + b2
b2 + 4 (0, 10) F2
⇒ 10b2 + 40 = b4 + 13b2
⇒ b4 + 3b2 – 40 =0
⇒ (b2 + 8) (b2 – 5) = 0
⇒ b2 + 8 = 0, b2 – 5 = 0 F1
(0, 10)
⇒ b2 = –8 & b2 = 5 (b2 = –8 not possible)
⇒ b2 = 5 in (ii), we get Figure 12.15
9 ×5 45
a2 = = =5
5+4 9
Again putting a2 = 5 and b2 = 5 in (i), we get
y 2 x2
− = 1 ⇒ y2 – x2 = 5
5 5
Which is the required equation of the hyperbola.
M a them a ti cs | 12.7
Illustration 7: An ellipse and hyperbola are confocal i.e., having same focus and conjugate axis of hyperbola &
1 1
minor axis of ellipse. If e1 and e2 are the eccentricities of the hyperbola and ellipse then find + .
e1 e22
2
(JEE ADVANCED)
Sol: Consider the standard equation of an ellipse and hyperbola by taking the
eccentricity as e1 and e2 respectively. Find the relation between the eccentricities
by using the condition that they have the same focus.
x2 y2 x2 y2
Let − 1 and
= + 1⇒
= ae1 = Ae2 and B = b
a2 b2 A2 B2 e1 e2
a2e12 1 1
∴ (e22 − 1) = a2(1 – e12) ∴ + =2
e22 e12 e22
Figure 12.16
Illustration 8: Find the equation of a hyperbola if the distance of one of its vertices from the foci are 3 and 1. Find
all the possible equations. (JEE ADVANCED)
Sol: Consider two cases when the major axis is parallel to the X – axis and the minor axis is parallel to the Y-axis
and vice versa.
Case I: ae – a = 1
ae + a = 3 A2 A1
⇒ e = 2 S2 S1
(-ae, 0) (ae, 0)
⇒ a = 1
⇒ b2 = 3
Figure 12.17
x2 y 2
Equation of hyperbola is − =1
1 3
Case II b(e – 1) = 1
b(e + 1) = 3
⇒ e = 2, b = 1, a2 = 3
y 2 x2
Equation of hyperbola is − 1
=
3 1
4. CONJUGATE HYPERBOLA
The hyperbola whose transverse and conjugate axes are respectively the
conjugate and transverse axis of a given hyperbola is called the conjugate B1
hyperbola of the given hyperbola. The hyperbola conjugate to the (0, b) A
A2
hyperbola 1
(-a, 0) (a, 0)
x2 y2 y2 x2 (0,-b)
− = 1 is
=1 −
a2 b2 a2 b2 B2
The eccentricity of the conjugate hyperbola is given by a2 = b2(e2 – 1)
2a2 Figure 12.18
and the length of the latus rectum is
b
Condition of similarity: Two hyperbolas are said to be similar if they have the same value of eccentricity.
Equilateral hyperbola: If a = b or L(T.A.) = L(C.A) then it is an equilateral or rectangular hyperbola.
1 2 . 8 | Hyperbola
x2 y 2 x2 y2
Equation of the Hyperbola − =1 − −1
=
a2 b 2 a2 b2
F1
y
A
x’ x x x’
A’ O A O
Figure F1 F2
y’ A’
Figure 12.19 F2
y’
Figure 12.20
Transverse axis 2a 2b
Conjugate axis 2b 2a
Relation between a, b, c c2 = a2 + b2 c2 = a2 + b2
c c
Eccentricity e= e‘=
a b
PLANCESS CONCEPTS
•• If e1 and e2 are the eccentricities of a hyperbola and its conjugate hyperbola then
1 1
+ 1
=
e12 e22
b2 a2
e12 = 1 + e22 = 1 +
a2 b2
1 1
∴ + =1
e12 e22
•• The foci of a hyperbola and its conjugate hyperbola are CONCYCLIC and form vertices of square.
M a them a ti cs | 12.9
(0,be) F3
(0, b)
F2 F1
(-ae,0) (ae,0)
F4
(0,-be)
Figure 12.21
6. AUXILIARY CIRCLE
A circle described on the transverse axis as diameter is an auxiliary circle and its equation is x2 + y2 = a2
Any point of the hyperbola is P ≡ (a sec θ, b tan θ)
P, Q are called corresponding point and θ is eccentric angle of P.
2 2 2
x +y =a
Figure 12.22
PLANCESS CONCEPTS
7. PARAMETRIC COORDINATES
x2 y2
Let P(x, y) be any point on the hyperbola = 1. Draw PL perpendicular from P on OX and then a tangent
−
a2 b2
LM from L to the circle described on A′A as diameter.
1 2 . 1 0 | Hyperbola
Illustration 9: Find the eccentricity of the hyperbola whose latus rectum is half of its transverse axis. (JEE MAIN)
Sol: Establish the relation between a and b and then use the eccentricity formula.
x2 y2 2b2
Let the equation of the hyperbola be − 1 . Then transverse axis = 2a and latus rectum =
=
a2 b2 a
2
2b 1
According to the question = (2a)
a 2
⇒ 2b2 = a2 ⇒ 2a2(e2 – 1) = a2 ⇒ 2e2 – 2 = 1 ⇒ e2 = 3/2 ∴e= 3/2
Illustration 10: If the chord joining two points (a sec θ1, b tan θ1) and (a sec θ2, b tan θ2) passes through the focus
x2 y 2 θ θ 1−e
of the hyperbola − = 1, then prove that tan 1 tan 2 = . (JEE ADVANCED)
a 2
b 2 2 2 1+e
Sol: Obtain a relation between the two given eccentric angles by substituting y
the point in the equation of chord. P(x,y)
The equation of the chord joining (a sec θ1, b tan θ1) and (a sec θ2, b tan θ2) is M
x θ − θ2 y θ1 + θ2 θ1 + θ2 x’ L x
cos 1 − sin
= cos
C(0,0) A (x,0)
a 2 b 2 2
(-x,0) A
θ − θ2 θ1 + θ2
If it passes through the focus (ae, 0) then e cos 1 = cos
2 2 y’
cos ( (θ1 − θ2 ) / 2 )
⇒ =1/e
cos ( (θ1 + θ2 ) / 2 ) Figure 12.23
θ1 θ2 1−e
using componendo dividendo rule we get tan tan = .
2 2 1+e
Clearly, PL > QL
y12 y 22 y12 y 22 x12 y12 x12 y 22 x12 y12
⇒ y1 > y2 ⇒ > ⇒ − <− ⇒ − < − ⇒ − <1
b2 b2 b2 b2 a2 b2 a2 b2 a2 b2
x2 y 2
Q(x1 , y 2 ) lies on 2 − 2 = 1
x12 y12 a b
⇒ − – 1 < 0
a2 b2 2
x1 y 22
− =1
a2 b2
x2 y2 x12 y12
Thus the point (x1, y1) lies outside the hyperbola − = 1. If − –1<0
a2 b2 a2 b2
Similarly, we can prove that the point (x1, y1) will lie inside or on the hyperbola according to
x12 y12
− – 1 > 0 or, = 0.
a2 b2
x12 y12
P lies outside/on/inside − – 1 < 0/ = 0 / > 0
a2 b2
Illustration 11: Find the position of the points (7, –3) and (2, 7) relative to the hyperbola 9x2 – 4y2 = 36.
(JEE MAIN)
Sol: Use the concept of position of a point w.r.t. the hyperbola.
x2 y 2
The equation of the given hyperbola is 9x2 – 4y2 = 36 or, − = 1. Now,
4 9
72 ( −3)2 41 22 72 49 −49
− −1= > 0 and, − ⇒ 1– ⇒ 1= < 0.
4 9 4 4 9 9 9
Hence, the point (7, –3) lies inside the parabola whereas the point (2, 7) lies outside the hyperbola.
Illustration 12: Find the position of the point (5, –4) relative to the hyperbola 9x2 – y2 = 1. (JEE MAIN)
condition of tangency
1 2 . 1 2 | Hyperbola
x2 y2
⇒ y = mx + a2m2 − b2 is tangent to the hyperbola − =1.
a2 b2
PLANCESS CONCEPTS
if m1m2 = – 1 x2 + y 2 = a2 − b2
Shrikant Nagori (JEE 2009, AIR 30)
Sol: Start with the standard equation of a tangent to a parabola and apply the condition for it be a tangent to
3x2 – y2 = 3.
2 x2 y 2
Tangent to the parabola is of the form y = mx + . For this line to be tangent to − 1 c2 = a2m2 – b2
=
m 1 3
4
⇒ = m2 – 1 ⇒ m2 = 4 ∴ ± y = 2x + 1 are the common tangents.
2
m
10. TANGENT
x2 y2 xx1 yy1
Point Form: The equation of tangent to the hyperbola − = 1 at (x1, y1) is
= 1. −
2 2 2 2
a b a b
x2 y 2
Slope Form: The equation of tangents of slope m to the hyperbola − = 1 are given by
a2 b2
y = mx ± a2m2 − b2
a2m b2
The coordinates of the points of contact are ± ,±
a2m2 − b2 a2m2 − b2
x2 y2 x y
Parametric Form: The equation of a tangent to the hyperbola − = 1 at (a sec θ, b tan θ) is sec θ − tan θ =1
a 2
b 2 a b
Note:
(i) The tangents at the point P (a sec θ1, b tan θ1) and Q (a sec θ2, b tan θ2) intersect at the point R
acos((θ1 − θ2 ) / 2) bsin((θ1 + θ2 ) / 2)
, .
cos((θ1 + θ2 ) / 2) cos((θ1 + θ2 ) / 2)
(ii) If | θ1 + θ2 | = π, then the tangents at these points (θ1 & θ2) are parallel.
M a them a ti cs | 12.13
(iii) There are two parallel tangents having the same slope m. These tangents touch the hyperbola at the extremities
of a diameter.
x2 y2
(iv) Locus of the feet of the perpendicular drawn from focus of the hyperbola − = 1 upon any tangent is its
a2 b2
auxiliary circle i.e. x + y = a and the product of these perpendiculars is b2.
2 2 2
(v) The portion of the tangent between the point of contact & the directrix subtends a right angle at the
corresponding focus.
(vi) The foci of the hyperbola and the points P and Q in which any tangent meets the tangents at the vertices are
concyclic with PQ as the diameter of the circle.
x2 y 2
Illustration 14: Prove that the straight line lx + my + n = 0 touches the hyperbola − = 1 if a2l2 – b2m2 = n2.
2 2
a b (JEE MAIN)
Sol: Apply the condition of tangency and prove the above result.
The given line is lx + my + n = 0 or y = –l/m x – n/m
Comparing this line with y = Mx + c ∴M = – l/m and c = –n/m ….(i)
x2 y2
This line (i) will touch the hyperbola − = 1 if c2 = a2M2 – b2
a2 b2
n2 a2l2
⇒ = – b2 or a2l2 – b2m2 = n2. Hence proved.
m2 m2
Illustration 15: Find the equations of the tangent to the hyperbola x2 – 4y2 = 36 which is perpendicular to the line
x – y + 4 = 0. (JEE MAIN)
Sol: Get the slope of the perpendicular line and use it to get the equation of the tangent.
Let m be the slope of the tangent. Since the tangent is perpendicular to the line x – y = 0
m × 1 = –1
⇒ m = –1
x2 y 2
Since x2 – 4y2 = 36 or − =1
36 9
x2 y2
Comparing this with − =1 ∴ a2 = 36 and b2 = 9
a2 b2
So the equation of the tangents are y = (–1)x ± 36 × ( −1)2 − 9
⇒ y = –x ± 27 ⇒ x+y± 3 3 =0
x2 y2
Illustration 16: If two tangents drawn from any point on hyperbola x2 – y2 = a2 – b2 to the ellipse + = 1 make
a2 b2
angles θ1 and θ2 with the axis then tan θ1 . tan θ2 . (JEE ADVANCED)
Sol: Establish a quadratic in m, where m is the slope of the two tangents. Then use the sum and product of the
roots to find tan θ1 . tan θ2 .
Let c2 = a2 – b2
Any tangent to the ellipse y = mx ± a2m2 + b2
c2 tan2 θ − b2
= = 1.
c2 sec2 θ − a2
11. NORMAL
x2 y2 a2 x b2 y
Point Form: The equation of the normal to the hyperbola − = 1 at (x1, y1) is + = a2 + b2.
a2 b2 x1 y1
x2 y 2
Parametric Form: The equation of the normal at (a sec θ, b tan θ) to the hyperbola − = 1 is a x cosθ + b y
cotθ = a2 + b2. a2 b2
x2 y2
Slope Form: The equation of a normal of slope m to the hyperbola − = 1 is given by
a2 b2
m(a2 + b2 ) a2 b2m
y = mx at the points ± ,
a2 − b2m2 a2 − b2m2 a2 − b2m2
Note:
(i) At most four normals can be drawn from any point to a hyperbola.
(ii) Points on the hyperbola through which, normal through a given point pass are called co-normal points.
(iii) The tangent & normal at any point of a hyperbola bisect the angle between the focal radii. This illustrates the
reflection property of the hyperbola as “An incoming light ray” aimed towards one focus is reflected from
the outer surface of the hyperbola towards the other focus. It follows that if an ellipse and a hyperbola have
the same foci, they cut at right angles at any of their common points.
Light Ray
Y Tangent
Q
P
X
S’ S
Figure 12.25
x2 y2 x2 y2
(iv) The hyperbola − = 1 and the hyperbola − = 1 (a > k > b > 0) are confocal and therefore
a2 b2 a2 − k 2 k 2 − b2
orthogonal.
(v) The sum of the eccentric angles of co-normal points is an odd multiple of π.
x2 y2
(vi) If θ1, θ2 and θ3 are eccentric angles of three points on the hyperbola − = 1. The normals at which are
a2 b2
concurrent, then sin(θ1 + θ2) + sin(θ2 + θ3) + sin(θ3 + θ1) =0
x2 y2
(vii) If the normals at four points P(x1, y1), Q(x2, y2), R(x3, y3) and S(x4, y4) on the hyperbola − = 1 are concurrent,
a2 b2
1 1 1 1
then (x1 + x2 + x3 + x4) + + + = 4.
x1 x2 x3 x 4
M a them a ti cs | 12.15
x2 y2
Illustration 17: How many real tangents can be drawn from the point (4, 3) to the hyperbola − = 1. Find the
equation of these tangents and the angle between them. 16 9 (JEE MAIN)
Sol: Use the concept of Position of a Point w.r.t. the hyperbola to find the number of real tangents.
Given point P = (4, 3)
x2 y 2
Hyperbola S ≡ − =1=0
16 9
16 9
∵ S1 ≡ – – 1 = -1 < 0
16 9
⇒ Point P ≡ (4, 3) lies outside the hyperbola
∴ Two tangents can be drawn from the point P(4, 3). Equation of a pair of tangents is SS1 = T2.
2
x2 y 2 4x 3y
⇒ − − 1 (–1) ≡ − − 1
16 9 16 9
2
x 2
y x 2
y2 xy x 2y 4
⇒ − + +1= + +1– + ⇒ 3x2 – 4xy – 12x + 16y = 0 and θ = tan–1
16 9 16 9 6 2 3 3
y = mx ± a2m2 − b2
x2 y2
H2: − =1
( −b2 ) ( −a2 )
a2m2 – b2 = (–b2) m2 – (–a2)
∴ a2(m2 – 1) = b2(1 – m2)
m=±1 Figure 12.26
Illustration 19: If the normals at (xr, yr); r = 1, 2, 3, 4 on the rectangular hyperbola xy = c2 meet at the point Q(h, k),
prove that the sum of the ordinates of the four points is k. Also prove that the product of the ordinates is –c4.
(JEE ADVANCED)
Sol: Write the equation of the normal in the parametric form and then use the theory of equations.
c
Any point on the curve xy = c2 is ct,
t
c
The equation of the normal to the hyperbola at the point ct, is
t
c −1
y– = (x – ct).
t dy
c
dx ct,
t
c2 dy −c2
Here, xy = c2 ; or y = ∴ =
x′ dx x2
1 2 . 1 6 | Hyperbola
dy c2 1
∴ = 2 2 = − 2
dx ct, c c t t
t
c
∴ The equation of the normal at ct, is
t
c
y– = t2(x – ct) or ty – c = t3(x – ct) or ct4 – t3x + ty – c = 0
t
The normal passes through (h, k). So
ct4 – t3h + tk – c = 0 … (i)
c
Let the roots of (i) be t1, t2, t3, t4. Then xr = ct, yr =
tr
∴ sum of ordinates = y1 + y2 + y3 + y4
c c c c t t t +t t t +t t t +t t t
= + + + = c 2 3 4 3 4 1 4 1 2 1 2 3
t1 t2 t3 t 4 t1 t2 t3 t 4
−k / c
=c. = k, {from roots of the equation (i)} and, product of the ordinates
−c / c
c c c c c4 c4
= y1y2y3y4 = . . . = = = –c4.
t1 t2 t3 t 4 t1 t2 t3 t 4 −c / c
Hence proved.
x2 y2
Illustration 20: The perpendicular from the centre on the normal at any point of the hyperbola − = 1 meet
a2 b2
at R. Find the locus of R. (JEE ADVANCED)
Sol: Solve the equation of the normal and the equation of line perpendicular to it passing through the origin.
Let (x1, y1) be any point on the hyperbola.
x12 y12
So, − = 1 … (i)
a2 b2
x − x1 y − y1 x1 y1
The equation of the normal at (x1, y1) is = or (y − y1 ) + (x − x1 ) = 0 … (ii)
x1 y1 a 2
b2
−
a2 y1 a2 b2
‘m’ of the normal = −
b2 x1
x2 + y 2 x2 + y 2 a2 (x2 + y 2 )
∴ t= ; ∴ x1 = a2 y =
xy(a2 + b2 ) xy(a2 + b2 ) x(a2 + b2 )
M a them a ti cs | 12.17
x2 + y 2 b2 (x2 + y 2 )
and y1 = b2 x =
xy(a2 + b2 ) y(a2 + b2 )
1 a4 (x2 + y 2 )2 1 b 4 (x2 + y 2 )2
∴ from (i), . − . =1
a2 x2 (a2 + b2 ) b2 y 2 (a2 + b2 )2
a2 b2
or {x2 + y2)2. − = (a2 + b2)2.
x2 y 2
x2 y2
Illustration 21: A normal to the hyperbola = 1 meets the axes in M and N and lines MP and NP are
−
a2 b2
drawn perpendicular to the axes meeting at P. Prove that the locus of P is the hyperbola a2x2 – b2y2 = (a2 + b2)2.
(JEE ADVANCED)
Sol: Find the co-ordinates of the point M and N and then eliminate the parameter between the ordinate and
abscissae.
x2 y 2
The equation of normal at the point Q(a sec φ, b tan φ) to the hyperbola − = 1 is
a2 b2
ax cosφ + by cot φ = a2 + b2 … (i)
a2 + b2 a2 + b2
The normal (i) meets the x-axis in M sec ϕ, 0 and y-axis in N 0, tan ϕ
a b
∴ Equation of MP, the line through M and perpendicular to axis, is
a2 + b2 ax
x= sec ϕ or sec φ = 2 … (ii)
a (a + b2 )
and the equation of NP, the line through N and perpendicular to the y-axis is
a2 + b2 by
y= tan φ or tan φ = 2 … (iii)
b (a + b2 )
The locus of the point is the intersection of MP and NP and will be obtained by eliminating φ from (ii) and (iii), so
we have sec2φ – tan2φ = 1
a2 x2 b2 y 2
⇒ − = 1 or a2x2 – b2y2 = (a2 + b2)2 is the required locus of P.
(a2 + b2 )2 (a2 + b2 )2
Illustration 22: Prove that the length of the tangent at any point of hyperbola intercepted between the point of
contact and the transverse axis is the harmonic mean between the lengths of perpendiculars drawn from the foci
on the normal at the same point. (JEE ADVANCED)
P cos θ P P 1 P 2
Similarly we get = 1− ∴ + =2 ⇒ + =
P1 e P1 P2 P1 P2 P
Hence Proved.
1 2 . 1 8 | Hyperbola
13. CHORD
a−d α β
= tan tan
a+d 2 2
1−e α β
if d = ae ⇒ = tan tan
1+e 2 2
M a them a ti cs | 12.19
PLANCESS CONCEPTS
x2 y 2
Illustration 23: If tangents to the parabola y2 = 4ax intersect the hyperbola − = 1 at A and B, then find the
locus of point of intersection of tangents at A and B. a2 b2 (JEE MAIN)
Sol: The point of intersection of the tangents at A and B is nothing but the point for which AB is the chord of
contact. Use this information to find the locus.
Let P ≡ (h, k) be the point of intersection of tangent at A and B
xh yk
∴ Equation of the chord of contact AB is = 1 − ….(i)
2
a b2
Which touches the parabola. Equation of the tangent to the parabola y2 = 4ax
y = mx – a/m ⇒ mx – y = –a/m ….(ii)
equation (i) and (ii) must be same
m −1 −a / m h b2 ak
∴ = = ⇒ m= and m = −
((h / a )) ( −(k / b ))
2 2 1 k a2 b2
hb2 ak b4
∴ = − ⇒ locus of P is y2 = – x.
ka2 b2 a3
Illustration 24: A point P moves such that the chord of contact of a pair of tangents from P to y2 = 4x touches the
rectangular hyperbola x2 – y2 = 9. If locus of ‘P’ is an ellipse, find e. (JEE MAIN)
Sol: Write the equation of the chord of contact to the parabola w.r.t. a point (h , k). Then solve this equation with
the equation of the hyperbola.
A
P (h,k)
Figure 12.29
2x 2h 4h2 4
yy1 = 2a(x + x1) ; yk = 2(x + h) ⇒ y= + ; = 9. –9
k k k 2
k2
1 2 . 2 0 | Hyperbola
x2 y 2 4 5
4h = 36 – 9k
2 2 + 1
= e2 = 1 – e=
9 4 9 3
x2 y2
Illustration 25: Find the locus of the mid-point of focal chords of the hyperbola = 1. (JEE MAIN) −
a2 b2
Sol: Use the formula T = S1 to get the equation of the chord and substitute the co-ordinates of the focus.
Let P ≡ (h, k) be the mid-point
xh yk h2 k2
∴ Equation of the chord whose mid-point (h, k) is given − –1= − – 1 since it is a focal chord.
2 2 2
a b a b2
∴ It passes through the focus, either (ae, 0) or (–ae, 0)
ex x2 y 2
∴ Locus is ± = −
a a2 b2
x2 y2
Illustration 26: Find the condition on ‘a’ and ‘b’ for which two distinct chords of the hyperbola − =1
2a2 2b2
passing through (a, b) are bisected by the line x + y = b. (JEE ADVANCED)
Sol: Consider a point on the line x + y = b and then find a chord with this point as the mid-point. Then substitute
the point in the equation of the chord to get the condition between ‘a’ and ‘b’.
Let the line x + y = b bisect the chord at P(α, b – α)
∴ Equation of the chord whose mid-point is P(α, b – α) is:
xα y(b − α ) α2 (b − α )2
− = −
2a2 2b2 2a2 2b2
α (b − α ) α2 (b − α )2
Since it passes through (a, b) ∴ − = −
2a 2b 2a2 2b2
1 1 1 1
α2 − + α − = 0 ⇒ a = b
2
b a
2
a b
x2 y 2
Illustration 27: Locus of the mid points of the focal chords of the hyperbola − = 1 is another hyperbola
whose eccentricity is e. a2 b2 (JEE ADVANCED)
Sol: Use the formula T = S1 and then homogenise the equation of the hyperbola using the equation of the chord
to find the locus.
Let (h, k) be the mid-point of the chord of the hyperbola. Then its equation is
hx ky h2 k2
− = − … (i)
a2 b2 a2 b2
The equation of the lines joining the origin to the points of intersection of the hyperbola and the chord (i) is
obtained by making a homogeneous hyperbola with the help of (i)
((hx / a ) − (ky / b ))
2
2 2
x 2
y2
∴ − =
((h / a ) − (k / b ))
2
a2 b2 2 2 2 2
2 2
1 h2 k 2 2 1 h2 k 2 2 h2 2 k 2 2 2hk
⇒ − x − 2 2 − 2 y = 4 x + 4 y − 2 2 xy … (ii)
a2 a2 b2 b a b a b ab
The lines represented by (ii) will be at right angles if the coefficient of x2 + the coefficient of y2 = 0
2 2 2
1 h2 k 2 h2 1 h2 k 2 k2 h2 k 2 1 1 h2 k 2
⇒ − − − − − =0 ⇒ − − = +
a2 a2 b2 a4 b2 a2 b2 b 4 a2 b2 a2 b2 a4 b 4
2
x2 y 2 1 1 x2 y 2
hence, the locus of (h, k) is − 2 − 2 = 4 + 4
a2 b 4 a b a b
14. DIAMETER
The locus of the mid-points of a system of parallel chords of a hyperbola is called a diameter. The point where a
diameter intersects the hyperbola is known as the vertex of the diameter.
x2 y2 b2
− = 1 is y = x.
a2 b2 a2m
b2
y= x … (i)
a2m
b2 b2
Clearly, (i) and y = m2x represent the same line. Therefore, m2 = ⇒ m 1m 2 =
a2m1 a2
x2 y2 b2
Thus, y = m1x and y = m2x are conjugate diameters of the hyperbola − = 1, if m1m2 =
a2 b2 a2
1 2 . 2 2 | Hyperbola
PLANCESS CONCEPTS
•• In a pair of conjugate diameters of a hyperbola, only one meets the hyperbola on a real point.
x2 y2
•• Let P(a sec θ, b tan θ) be a point on the hyperbola = 1 such that CP and CD are conjugate
−
a2 b2
diameters of the hyperbola. Then, the coordinates of D are (a tan θ, b sec θ)
•• If a pair of conjugate diameters meet the hyperbola and its conjugate in P and D respectively then
CP2 – CD2 = a2 – b2.
Shivam Agarwal (JEE 2009, AIR 27)
x2 y 2
Equation of polar: Equation of the polar of the point (x1 , y1 ) with respect to the hyperbola − = 1 is given by
xx1 yy1 a2 b2
− = 1 , i.e., T = 0
a2 b2
x2 y 2 −a2l b2m
Coordinates of Pole: The pole of the line lx + my + n = 0 with respect to hyperbola + =1 is P , .
a2 b2 n n
Properties of pole and polar:
1. If the polar of P (x1 , y1 ) passes through Q(x2 , y 2 ) , then the polar of Q(x2 , y 2 ) goes through P (x1 , y1 ) and such
x x y y
points are said to be conjugate points. Condition for conjugate points is 1 2 − 1 2 = 1.
a2 b2
2. If the pole of line l1 x + m1 y + n1 = 0 lies on another line l2 x + m2 y + n2 =0 , then the pole of the second line will
lie on the first and such lines are said to be conjugate lines.
3. Pole of a given line is the same as the point of intersection of the tangents at its extremities.
4. Polar of focus is its directrix.
16. ASYMPTOTES
An asymptote to a curve is a straight line, such that distance between (0,b)
the line and curve approaches zero as they tend to infinity.
In other words, the asymptote to a curve touches the curves at infinity
(-a,0) (a,0)
i.e. asymptote to a curve is its tangent at infinity.
The equations of two asymptotes of the hyperbola
(0,-b)
x2 y2 b x y
− = 1 are y = ± x or ± = 0
a2
b 2 a a b Figure 12.31
M a them a ti cs | 12.23
x2 y2
Combined equation of asymptote = 0
−
a2 b2
Note: If the angle between the asymptotes of the hyperbola is θ, then its eccentricity is sec θ.
PLANCESS CONCEPTS
x2 y2 x2 y2
•• The combined equation of the asymptotes of the hyperbola − = 1 is − = 0.
a2 b2 a2 b2
•• When b = a, the asymptotes of the rectangular hyperbola x2 – y2 = a2 are y = ±x, which are at right
angles.
•• A hyperbola and its conjugate hyperbola have the same asymptotes.
•• The equation of the pair of asymptotes differ from the hyperbola and the conjugate hyperbola by the
same constant, i.e. Hyperbola – Asymptotes = Asymptotes – Conjugate hyperbola
•• The asymptotes pass through the centre of the hyperbola.
•• The bisectors of the angles between the asymptotes are the coordinates axes.
•• The asymptotes of a hyperbola are the diagonals of the rectangle formed by the lines drawn through
the extremities of each axis parallel to the other axis.
•• Asymptotes are the tangents to the hyperbola from the centre.
x2 y 2
•• The tangent at any point P on − = 1 with the centre C meets asymptotes at Q, R and cut off
2
∆CQR of constant area = ab. a b2
•• The parts of the tangent intercepted between the asymptote is bisected at the point of contact.
•• If f(x, y) = 0 is an equation of the hyperbola then the centre of the hyperbola is the point of intersection
∂f ∂f
of = 0 and = 0.
∂x ∂y
Ravi Vooda (JEE 2009, AIR 71)
Illustration 30: Find the equation of that diameter which bisects the chord 7x + y – 20 = 0 of the hyperbola
x2 y 2
− = 1. (JEE ADVANCED)
3 7
1 2 . 2 4 | Hyperbola
Sol: Consider a diameter y = mx and solve it with the equation of the hyperbola to form a quadratic in x. Find the
midpoint of the intersection of the chord and hyperbola. Use this point to find the slope of the diameter.
The centre of the hyperbola is (0, 0). Let the diameter be y = mx … (i)
The ends of the chord are found by solving
7x + y – 20 = 0 … (ii)
2 2
x y
and − = 1 … (iii)
3 7
x2 1
Solving (ii), (iii) we get − (20 − 7x)2 = 1
3 7
or 7x2 – 3(400 – 280x + 49x2) = 21 or 140x2 – 840x + 1221 = 0
Let the roots be x1, x2
840
Then x1 + x2 = = 6 ... (iv)
140
or 42 + y1 + y2 – 40 = 0, using (iv) ; ∴ y1 + y2 = –2
x + x 2 y1 + y 2 6 −2
∴ The middle point of the chord = 1 , = , = (3, –1)
2 2 2 2
1 1
This lies on (i). So –1 = 3m ; ∴ m = − ∴ the equation of the diameter is y = − x.
3 3
Illustration 31: The asymptotes of a hyperbola having centre at the point (1, 2) are parallel to the lines
2x + 3y = 0 and 3x + 2y = 0. If the hyperbola passes through the point (5, 3) show that its equation is (2x + 3y – 8)
(3x + 2y + 7) = 154. (JEE ADVANCED)
Sol: With the information given, find out the equation of the asymptotes and then use the fact that the point (5, 3)
lies on the hyperbola to find the equation of the hyperbola.
Let the asymptotes be 2x + 3y + λ = 0 and 3x + 2y + µ = 0. Since the asymptote passes through (1, 2) then λ = –8
and µ = –7
Thus the equation of the asymptotes are 2x + 3y – 8 = 0 and 3x + 2y – 7 = 0
Let the equation of the hyperbola be (2x + 3y – 8) (3x + 2y – 7) + v =0 … (i)
It passes through (5, 3), then (10 + 9 – 8) (15 + 6 – 7) + v = 0
⇒ 11 × 14 + v = 0
∴ v = – 154
putting the value of v in (i) we obtain (2x + 3y – 8) (3x + 2y – 7) – 154 =0
which is the equation of the required hyperbola.
Remarks: Since the transverse and conjugate axis of a rectangular hyperbola are equal. So, its eccentricity e is
given by
b2 b2
e= 1+ = 1+ = 2
a2 a2
2 2
a2 a2
⇒ XY = ⇒ XY = c2, where c2 =
2 2
Thus, the equation of the hyperbola referred to its asymptotes as the coordinates axes is
a2
xy = c2, where c2 =
2
Remark: The equation of a rectangular hyperbola having coordinate axes as its asymptotes is xy = c2.
If the asymptotes of a rectangular hyperbola are x = α, y = β, then its equation is
(x – α) (y – β) = c2 or xy – ay – bx + λ = 0 ; ( λ ≤ αβ )
17.2 Tangent
Point Form
x y
The equation of the tangent at (x1, y1) to the hyperbola xy =c2 is xy1 + yx1 = 2c2 or, + = 2.
x1 y1
Parametric Form
c x
The equation of the tangent at ct, to the hyperbola xy = c2 is + yt = 2c.
t t
c c 2ct1 t2 2c
Note: Tangent at P ct1 , and Q ct2 , to the rectangular hyperbola xy = c2 intersect at .
t1 t2 t1 + t2 t1 + t2
1 2 . 2 6 | Hyperbola
17.3 Normal
Point Form
The equation of the normal at (x1, y1) to the hyperbola xy = c2 is xx1 – yy1 = x12 – y12
Parametric Form
c y c
The equation of the normal at ct, to the hyperbola xy = c2 is xt − = ct2 −
t t t2
Note:
c
(i) The equation of the normal at ct, is a fourth degree equation in t. So, in general, at most four normals
t
can be drawn from a point to the hyperbola xy = c2.
(ii) The equation of the polar of any point P(x1, y1) with respect to xy = c2 is xy1 + yx1 = 2c2.
(iii) The equation of the chord of the hyperbola xy = c2 whose midpoint (x,y) is xy1 + yx1 = 2x1y1 or, T = S′. where
T and S′ have their usual meanings.
(iv) The equation of the chord of contact of tangents drawn from a point (x1, y1) to the rectangular xy = c2 is xy1
+ yx1 = 2c2.
Illustration 32: A, B, C are three points on the rectangular hyperbola xy = c2, find
(i) The area of the triangle ABC
(ii) The area of the triangle formed by the tangents A, B and C (JEE ADVANCED)
Sol: Use parametric co-ordinates and the formula for the area to get the desired result.
c c c
Let co-ordinates of A, B and C on the hyperbola xy = c2 be ct1 , . ct2 , and ct3 , respectively
t1 t2 t3
c c c
ct1 ct2 ct3
1 t1 t2 t3 c2 t1 t2 t2 t3 t3 t1
(i) Area of triangle ABC = + + = − + − + −
2 c c c 2 t2 t1 t3 t2 t1 t3
ct2 ct3 ct1
t2 t3 t1
c2 c2
= | t32 t3 − t22 t3 + t1 t22 − t32 t1 + t2 t32 − t12 t2 | = | (t − t )(t − t )(t − t ) |
2t1 t2 t3 2t1 t2 t3 1 2 2 3 3 1
(ii) Equation of tangents at A, B, C are x + yt12 – 2ct1 = 0, x + yt22 – 2ct2 = 0 and x + yt32 – 2ct3 = 0
2
1 t12 −2ct1
1
∴ Required Area = 1 t22 −2ct2 … (i)
2 | C1C2C3 |
1 t32 −2ct3
( x − h) (y − k)
2 2
(a) In general convert the given hyperbola equation into the standard form − 1 and compare it
=
a2 b2
2 2 2 2
x y x y
with − 1 . Then solve using the properties of the hyperbola
= − 1 . So, it is advised to remember
=
2 2 2
a b a b2
the standard results.
x2 y 2
(b) Most of the standard results of a hyperbola can be obtained from the results of an ellipse + 1 just by
=
changing the sign of b2. a2 b2
FORMULAE SHEET
HYPERBOLA
Hyperbola x2 y2 x2 y2
− 1
= − + 1
=
Imp. Terms a2 b2 a2 b2
x2 y2
or − −1
=
a2 b2
0 ≤ φ < 2π 0 ≤ φ < 2π
Focal radii SP = ex1 – a SP = ey1 – b
S(0,be)
Y
N B L
M′ M P(x,y) B (0, b) b
y=
Rectum
Z a
X′ X X′ X
A′ Z′ C Z A S(ae,0) C b
S′(–ae,0) y=−
Latus
(–a,0) (a,0) a
a B′(0, –b)
a x=
N′ x=– B′ e L′
S’(0,–be)
e
Y′
Y′
(b) Special form of hyperbola: If (h , k) is the centre of a hyperbola and its axes are parallel to the co-ordinate
(x − h)2 (y − k)2
axes, then the equation of the hyperbola is − =1
a2 b2
(c) Parametric equations of a hyperbola: The equation x = a sec φ and y = b tan φ are known as the parametric
equation of the standard hyperbola
x2 y2
− = 1.
a2 b2
x2 y2 x12 y12 xx1 yy1
S
If = − , then S1 = − −1 ; T = − −1
2 2 2 2 2
a b a b a b2
(d) Position of a point and a line w.r.t. a hyperbola: n The point (x1, y1) lies inside, on or outside the hyperbola
x2 y2 x12 y12
− = 1 according to–1 being >, = or < zero. −
a2 b2 a2 b2
The line y = mx + c intersects at 2 distinct points, 1 point or does not intersect with the hyperbola according as c2
>, = or < a2m2 – b2.
(e) Tangent:
x2 y2
(i) Point form: The equation of tangent to the hyperbola − = 1 at (x1, y1) is
a2 b2
xx1 yy1
− = 1.
2
a b2
x2 y 2
(ii) Parametric form: The equation of tangent to the hyperbola − = 1 at parametric coordinates (a sec φ, b
2 2
x y a b
tan φ) is sec φ − φ =11.
a b
x2 y 2
(iii) Slope form: The equation of the tangents having slope m to the hyperbola − = 1 are
a2 b2
y = mx ± a2m2 − b2 and the co-ordinates of points of contacts are
a2m b2
± ,±
a2m2 − b2 a2m2 + b2
x2 y2
(f) Equation of a pair of tangents from an external point (x1, y1) to the hyperbola − = 1 is SS1 = T2.
2 2
a b
M a them a ti cs | 12.29
(g) Normal:
x2 y2
(i) Point form: The equation of the normal to the hyperbola − = 1 at (x1, y1) is
2 2
a2 b2
a x b y
+ = a2 + b2.
x1 y1
(ii) Parametric form: The equation of the normal at parametric coordinates (a sec θ, b tanθ) to the hyperbola
x2 y2
− = 1 is ax cos θ + by cot θ = a2 + b2.
a2 b2
2 2
(iii) Slope form: The equation of the normal having slope m to the hyperbola x − y = 1 is
a2 b2
m(a2 + b2 )
y = mx
a2 − b2m2
x2 y 2
(iv) Condition for normality: y = mx + c is a normal to the hyperbola − = 1 if
2 2 2 a2 b2
m(a + b )
c2 =
(a2 − m2b2 )
a2 mb2
(v) Points of contact: Co-ordinates of the points of contact are ± , .
a2 − b2m2 a2 − b2m2
x2 y2
(h) The equation of the director circle of the hyperbola = 1 is given by x2 + y2 = a2 – b2.
−
a2 b2
(i) Equation of the chord of contact of the tangents drawn from the external point (x1, y1) to the hyperbola is
xx1 yy1
given by − = 1.
2
a b2
x2 y2
( j) The equation of the chord of the hyperbola − = 1 whose mid point is (x1, y1) is T = S1.
a2 b2
(k) Equation of a chord joining points P(a sec f1, b tan f1) and Q (a sec f2, b tan f2) is
x φ − φ2 y φ1 + φ2 φ + φ2
cos 1 − sin cos 1
=
a 2 b 2 2
(l) Equation of the polar of the point (x1, y1) w.r.t. the hyperbola is given by T = 0.
x2 y2 a2 b2m
The pole of the line lx + my + n = 0 w.r.t. − = 1 is − ,
a2 b2 n n
x2 y2 b2
(m) The equation of a diameter of the hyperbola − = 1 corresponding to the chords of slope m is y = x
a2 b2 a2m
b2
(n) The diameters y = m1x and y = m2x are conjugate if m1m2 =
a2
(o) Asymptotes:
•• Asymptote to a curve touches the curve at infinity.
x2 y2 b
•• The equation of the asymptotes of the hyperbola − = 1 are y = ± x.
a2
b 2 a
1 2 . 3 0 | Hyperbola
x2 y2 x2 y2
•• * The combined equation of the asymptotes of the hyperbola − = 1 is − =0
a2 b2 a2 b2
x2 y2 a2
•• * The angle between the asymptotes of − = 1 is 2 tan–1
or 2 sec–1 e.
a2 b2 b2
•• A hyperbola and its conjugate hyperbola have the same asymptotes.
•• The bisector of the angles between the asymptotes are the coordinate axes.
•• Equation of the hyperbola – Equation of the asymptotes = constant.
Solved Examples
Example 2: Obtain the equation of hyperbola whose [(a secθ + ae)2 + a2tan2q]
asymptotes are the straight lines x + 2y + 3 = 0 & 3x +
= a4[(sec2θ + tan2θ + e2)2 – 4e2sec2q]
4y + 5 = 0 and which passes through the point (1, –1)
= a4[(2sec2θ – 1 + 2)2 – 4.2 sec2q]
Sol: Use the following formula:
= a4[(2sec2θ + 1)2 – 8sec2q]
Equation of hyperbola – Equation of asymptotes =
= a4[(2sec2θ – 1)2]
constant.
∴ SP.S’P = a2(2sec2θ – 1)
The equation of the hyperbola, is
= a2(sec2θ + tan2θ)
(x + 2y + 3)(3x + 4y + 5) = k, k being a constant.
= CP2.
This passes through the point (1, –1)
∴ (1 + 2(–1) + 3)(3(1) + 4(–1) + 5) = k Example 5: Find the equation of the hyperbola
⇒ k=2×4=8 conjugate to the hyperbola
∴ The equation of the hyperbola is 2x2 + 3xy – 2y2 – 5x + 5y + 2 = 0
(x + 2y + 3)(3x + 4y + 5) = 8 Sol: Use the formula:
Equation Hyperbola + Conjugate Hyperbola
Example 3: If e and e′ are the eccentricities of two
hyperbolas conjugate to each other, = 2(Asympototes)
1 1 Let asymptotes be
show that + = 1.
2 2
e e′ 2x2 + 3xy – 2y2 – 5x + 5y + λ = 0
Sol: Start with the standard equation of two hyperbolas The equation above represents a pair of lines if
and eliminate ‘a’ and ‘b’.
abc + 2fgh – af2 – bg2 – ch2 = 0
x2 y2 y2 x2
Let − = 1 and − =1 ∴ λ = –5
a2 b2 b2 a2
Equation Hyperbola + Conjugate Hyperbola
be the two hyperbolas with eccentricities e and e′
respectively = 2(Asympototes)
2
1 a ∴ Conjugate Hyperbola
b2 = a2(e2 – 1) ⇒ =
e2 a2 + b2 = 2(Asymptotes) – Hyperbola
2
1 b 2x2 + 3xy – 2y2 – 5x + 5y – 8 = 0
a2 = b2(e¢2 – 1) ⇒ =
2 2 2
e′ a +b
1 1 a2 b2 Example 6: If (5, 12) and (24, 7) are the foci of a hyperbola
∴ + = + =1 passing through the origin then the eccentricity of the
e2 e′2 (a2 + b2 ) (a2 + b2 )
hyperbola is
Example 4: If any point P on the rectangular hyperbola Sol: Use the definition of the hyperbola S¢P – SP = 2a.
x2 – y2 = a2 is joined to its foci S, S′ show that SP.S¢P = Let S(5, 12) and S′(24, 7) be the two foci and P(0, 0) be
CP2, where C is the centre of the hyperbola. a point on the conic then
Any point on the rectangular hyperbola and SS′ = (24 − 5)2 + (7 − 12)2 = 192 + 52 = 386
x2 – y2 = a2 is P(a secθ, a tanθ) ; eccentricity of a
since the conic is a hyperbola, S¢P – SP = 2a, the length
rectangular hyperbola is 2 .
of transverse axis and SS′ = 2ae, e being the eccentricity.
S is (ae, 0), S′ is (–ae, 0) and C is (0, 0) SS′ 386
⇒ e= =
(SP) .(S’P) = [(a secθ – ae) + a tan q] ×
2 2 2 2 2
S′P − SP 12
1 2 . 3 2 | Hyperbola
Hence proved. a 3
X± =0 or x+1± =0
e 5/3
Example 9: The normal to the curve at P(x, y) meets the 9
i.e., x + 1 ± =0
x-axis at G. If the distance of G from the origin is twice 5
the abscissa of P, then the curve is- 14 4
∴ x= − and x =
5 5
Sol: Similar to the previous question.
M a them a ti cs | 12.33
a −b
R = , Example 5: Find the equation of the hyperbola,
(x1 / a) + (y1 / b) (x1 / a) + (y1 / b) whose eccentricity is 5/4, whose focus is
(a, 0) and whose directrix is 4x – 3y = a. Find also the
The midpoint of QR has coordinate (x1, y1) which is also
coordinates of the centre and the equation to other
the point of contact of the tangent.
directrix.
Area of ∆OQR =
Sol: Use the basic definition of a hyperbola.
a −b
(4x − 3y − a)2
1 (x1 / a) − (y1 / b) (x1 / a) + (y1 / b)
(x – a)2 + (y – 0)2 = e2
25
2 b a
− x2 – 2ax + a2 + y2 =
(x1 / a) − (y1 / b) (x1 / a) + (y1 / b)
25 1
(16x2 + 9y2 + a2 – 24xy – 8ax + 6ay)×
=ab sq. units 16 25
7y2 + 24xy – 24ax – 6ay + 15a2 = 0 … (i)
Example 4: Prove that if a rectangular hyperbola
circumscribes a triangle it also passes through the S’
Directrix
orthocentre of the triangle.
M 4x-3y=a
Sol: Take three points on the hyperbola and find C
Directrix
the coordinates of the orthocentre. Prove that the P
orthocentre satisfies the equation of the hyperbola.
S’(a,0)
Let the equation of the curve referred to its asymptotes Transverse axis
be xy = c2 ….(i)
Let the angular points of the triangle be P, Q and R and Differentiating with respect to ‘x’
let their co-ordinates be 24y – 24a = 0 … (ii)
c c Differentiating with respect to ‘y’
P ≡ ct1 , , Q ≡ ct2 , and
t1 t2
14y + 24x – 6a = 0 … (iii)
c
R ≡ ct3 , respectively. Solving (ii) and (iii)
t3
C ≡ (–a/3, a)
Equation of QR is x + t2t3 y = c (t2 + t3)
Transverse axis is
The equation of altitude through P and perpendicular
3x + 4y = 3a
to QR is
‘P’ is the point of intersection of the transverse axis and
c
y– = t2t3(x – ct1) the directrix:
t1
13a 9a
∴ P≡ , ‘C’ is mid point of MP
c 25 25
i.e. y + c t1t2t3 = t2t3 x + … (ii)
t1 t2 t3
M a them a ti cs | 12.35
JEE Main/Boards
Exercise 1 Q.3 Show that the line 21x + 5y = 116 touches the
hyperbola 7x2 – 5y2 = 232 and find the co-ordinates of
the point of contact.
Q.1 Find the centre, eccentricity and foci of the
hyperbola 9x2 – 16y2 – 18x – 64y – 199 = 0
Q.4 Find the locus of the middle points of the portion
Q.2 Find the equation to the tangent to the hyperbola x2 y 2
of the tangents to the hyperbola − = 1 included
4x2 – 3y2 = 13 at the point (2,1). a2 b2
between the axes.
M a them a ti cs | 12.37
Q.5 A point P moves such that the tangents PT1 and at the point Q(h, k), prove that
PT2 from it to the hyperbola 4x2 – 9y2 = 36 are mutually (i) x1 + x2 + x3 + x4 = h (ii) y1 + y2 + y3 + y4 = k
perpendicular. Find the equation of the locus of P.
(iii) x1x2x3x4 = y1y2y3y4 = –c4
Q.9 Find the eccentric angle of the point lying in fourth Q.19 A tangent to the parabola x2 = 4ay meets the
quadrant on the hyperbola x2 – y2 = 4 whose distance hyperbola xy = k2 in two points P and Q. Prove that the
from the centre is 12 units. middle point of PQ lies on a parabola.
Q.10 Find the acute angle between the asymptotes of Q.20 Show that the locus of the middle points of the
4x2 – y2 = 16. normal chords of the rectangular hyperbola x2 – y2 = a2
is (y2 – x2)3 = 4a2x2y2.
Q.22 Locus of the middle points of the parallel chords (A) (–2, 6 ) (B) (–5, 2 6 )
with gradient m of the rectangular hyperbola xy = c2 is-
1 1
(A) y + mx = 0 (B) y – mx = 0 (C) , (D) (4, − 6 )
2 6
(C) my – mx = 0 (D) my + x = 0
x2 y 2
Q.6 If e1 is the eccentricity of the ellipse + = 1 and
16 25
Q.23 The locus of the middle points of chords of e2 is the eccentricity of the hyperbola passing through
hyperbola 3x2 – 2y2 + 4x – 6y = 0 parallel to y = 2x is- the foci of the ellipse and e1e2 = 1, then equation of the
hyperbola is- (2006)
(A) 3x – 4y = 4 (B) 3y – 4x + 4 = 0
x2 y 2 x2 y 2
(C) 4x – 4y = 3 (D) 3x – 4y = 2 (A) − = 1 (B) − = –1
9 16 16 9
x2 y 2
(C) − = 1 (D) None of these
9 25
1 2 . 4 0 | Hyperbola
x2 y2
Q.7 A hyperbola, having the transverse axis of length Q.9 Let P(6, 3) be a point on the hyperbola −
2sinθ, is confocal with the ellipse 3x2 + 4y2 = 12. Then a2 b2
its equation is- (2007) = 1. If the normal at the point P intersect the x-axis at
(A) x2cosec2θ – y2sec2θ = 1 (B) x2sec2θ – y2cosec2θ = 1 (9, 0), then the eccentricity of the hyperbola is- (2011)
(C) x2sin2θ – y2cos2θ = 1 (D) x2cos2θ – y2sin2θ = 1 5 3
(A) (B) (C) 2 (D) 3
2 2
Q.8 Consider a branch of the hyperbola
x2 – 2y2 – 2 2 x – 4 2y–6=0 Q.10 The eccentricity of the hyperbola whose length
with vertex at the point A. Let B be one of the end points of the latus rectum is equal to 8 and the length of its
of its latus rectum. If C is the focus of the hyperbola conjugate axis is equal to half of the distance between
nearest to the point A, then the area of the triangle ABC its foci, is: (2016)
is- (2008) 4 2 4
(A) (B) (C) 3 (D)
2 3 3 3 3
(A) 1 − sq. unit (B) − 1 sq. unit
3 2
2 3
(C) 1 + sq. unit (D) + 1 sq. unit
3 2
JEE Advanced/Boards
Q.3 For the hyperbola x2/100 – y2/25 = 1, prove that the Q.9 If C is the centre of hyperbola x2/a2 – y2/b2 = 1, S, S′
(i) eccentricity = 5 /2 its foci and P a point on it. Prove that SP.S´P = CP2 – a2 + b2.
(ii) SA.S′A = 25, where S and S′ are the foci and A is the
vertex. Q.10 Tangents are drawn to the hyperbola 3x2 – 2y2 =
25 from the point (0, 5/2). Find their equations.
Q.4 Find the centre, the foci, the directrices, the length
of the latus rectum, the length and the equations of the Q.11 If the tangent at the point (h, k) to the hyperbola
axes and the asymptotes of the hyperbola x2/a2 – y2/b2 = 1 cuts the auxiliary circle in points whose
16x2 – 9y2 + 32x + 36y – 164 = 0. ordinates are y1 and y2 then prove that 1/y1 + 1/y2 = 2/k.
Q.5 If a rectangular hyperbola have the equation, Q.12 Tangents are drawn from the point (α, β) to the
xy = c2, prove that the locus of the middle point of the hyperbola 3x2 – 2y2 = 6 and are inclined at angles θ and
chords of constant length 2d is (x2 + y2)(xy – c2) = d2xy. φ to the x-axis. If tanθ . tanφ = 2, prove that β2 = 2α2 – 7.
x2 y2
Q.22 Find the equations of the tangents to the Q.5 P is a point on the hyperbola = 1 , N is the
−
hyperbola x2 – 9y2 = 9 that are drawn from (3, 2). Find a2 b2
foot of the perpendicular from P on the transverse axis.
the area of the triangle that these tangents form with
The tangent to the hyperbola at p meets the transverse
their chord of contact.
axis at T. If O is the centre to the hyperbola, the OT.ON
is equal to-
Q.23 Let ‘p’ be the perpendicular distance from the
centre C of the hyperbola x2/a2 – y2/b2 = 1 to the tangent (A) e2 (B) a2 (C) b2 (D) b2/a2
drawn at point R on the hyperbola. If S and S′ are the
Q.6 The equation to the chord joining two point (x1, y1)
two foci of the hyperbola, then show that
and (x2, y2) on the rectangular hyperbola xy = c2 is-
b2
(RS + RS′)2 = 4a2 1 + . x y x y
p2 (A) + = 1 (B) + =1
x1 + x2 y1 + y 2 x1 − x2 y1 − y 2
x y x y
(C) + = 1 (D) + =1
y1 − y 2 x1 − x2 y1 − y 2 x1 − x2
1 2 . 4 2 | Hyperbola
Q.7 The eccentricity of the hyperbola whose latus (C) x = et + e–t & y = et – e–t
rectum is 8 and conjugate axis is equal to half the t
(D) x2 – 6 = 2cot & y2 + 2 = 4cos2 2
distance between the foci, is-
4 4 2
(A) (B) (C) (D) None of these Q.14 Circles are drawn on chords of the rectangular
3 3 3 hyperbola xy = a2 parallel to the line y = x as diameters.
All such circles pass through two fixed points whose
Q.8 The equation to the chord of the hyperbola x2 – y2 = 9 co-ordinates are-
which is bisected at (5, –3) is-
(A) (c, c) (B) (c, –c) (C) (–c, c) (D) (–c, –c)
(A) 5x + 3y = 9 (B) 5x – 3y = 16
(C) 5x + 3y = 16 (D) 5x – 3y = 9 Q.15 If the normal at (xi, yi) i = 1, 2, 3, 4 to its rectangular
hyperbola xy = 2 meet at the point (3, 4), then-
dx 3y (A) x1 + x2 + x3 + x4 = 3 (B) y1 + y2 + y3 + y4 = 4
Q.9 The differential equation = represents a
dy 2x
family of hyperbolas (except when it represents a pair (C) x1x2x3x4 = –4 (D) y1y2y3y4 = –4
of lines) with eccentricity-
3 5 2 5
Q.16 If (5, 12) and (24, 7) are the foci of a conic passing
(A) (B) (C) (D) through the origin then the eccentricity of conic is-
5 3 5 2
(A) 386 / 12 (B) 386 / 13 (C) 386 / 25 (D) 386 / 38
Multiple Correct Choice Type
Q.17 The value of m for which y = mx + 6 is a tangent
Q.10 Equation of a tangent passing through (2, 8) to x2 y2
the hyperbola 5x2 – y2 = 5 is- to the hyperbola − = 1 is-
100 49
(A) 3x – y + 2 = 0 (B) 3x + y – 14 = 0
17 17 20 20
(C) 23x – 3y – 22 = 0 (D) 3x – 23y + 178 = 0 (A) (B) − (C) (D) −
20 20 17 17
Q.11 The equation 16x2 – 3y2 – 32x + 12y – 44 = 0
represent a hyperbola - x2 y2
Q.18 The equation + = 1 represents-
12 − k k − 8
(A) The length of whose transverse axis is 4 3
(A) A hyperbola if k < 8
(B) The length of whose conjugate axis is 8
(B) An ellipse if 8 < k < 12, k ≠ 10
(C) Those centre is (1, 2)
(C) A hyperbola if 8 < k < 12
19
(D) Those eccentricity is (D) Circle if k = 10
3
Q.12 A common tangent to 9x2 – 16y2 = 144 and x2 + Q.19 Equations of a common tangent to the two
y2 = 9 is- x2 y 2 y 2 x2
hyperbolas − =1& − = 1 is-
3 15 2 15 a2 b2 a2 b2
(A) y = x+ (B) y = 3 x+
7 7 7 7
(A) y = x + a2 − b2 (B) y = x – a2 − b2
3 2 15
(C) y = 2 x + 15 7 (D) y = 3 x− (C) y = – x + a2 − b2 (D) y= – x – a2 − b2
7 7 7
Q.13 Which of the following equation in parametric
Q.20 The equation of the tangent lines to the hyperbola
form can represent a hyperbola, profile, where ‘t’ is a
x2 – 2y2 = 18 which are perpendicular the line y = x are-
parameter
a 1 b 1 (A) y = –x + 7 (B) y = –x + 3
(A) x = t + & y = t −
2 t 2 t (C) y = –x – 4 (D) y = –x – 3
tx y x ty
(B) − +t=0& + –1=0
a b a b
M a them a ti cs | 12.43
Previous Years’ Questions Q.6 Equation of a common tangent with positive slope
to the circle as well as to the hyperbola is-
Q.1 If the circle x2 + y2 = a2 intersects the hyperbola
xy = c2 in four points P(x1, y1), Q(x1, y1), R(x3, y3), S(x4, y4), (A) 2x – 5 y – 20 = 0 (B) 2x – 5y+4=0
then- (1998) (C) 3x – 4y + 8 = 0 (D) 4x – 3y + 4 = 0
(A) x1 + x2 + x3 + x4 = 0 (B) y1 + y2 + y3 + y4 = 0
Q.7 Equation of the circle with AB as its diameter is-
(C) x1x2x3x4 = c4 (D) y1y2y3y4 = c4
(A) x2 + y2 – 12x + 24 = 0 (B) x2 + y2 + 12x + 24 = 0
Q.2 An ellipse intersects the hyperbola 2x2 – 2y2 = 1 (C) x2 + y2 + 24x – 12 = 0 (D) x2 + y2 – 24x – 12 = 0
orthogonally. The eccentricity of the ellipse is reciprocal
to that of the hyperbola. If the axes of the ellipse are
Q.8 The line 2x + y = 1 is tangent to the hyperbola
along the coordinate axes, then (2009)
x2 y 2
(A) Equation of ellipse is x2 + 2y2 = 2 − = 1. If this line passes through the point of
a2 b2
(B) The foci of ellipse are (± 1, 0) intersection of the nearest directrix and the x-axis, then
(C) Equation of ellipse is x2+2y2 = 4 the eccentricity of the hyperbola is …….. (2010)
x2 y2
Q.10 Match the conics in column I with the statements/ Q.12 Let P(6, 3) be a point on the hyperbola −
= 1.
expressions in column II. (2009) a2 b2
If the normal at the point P intersects the x-axis at (9, 0),
then the eccentricity of the hyperbola is (2011)
Column I Column II
(A) Circle (p) T
he locus of the point (h, k) for
5 3
(A) (B) (C) 2 (D) 3
which the line hx + ky = 1 touches 2 2
the circle x2 + y 2 = 4
x2 y2
(B) Parabola (q) P
oints z in the complex plane Q.13 Let the eccentricity of the hyperbola = 1
−
a2 b2
satisfying z + 2 − z − 2 =
±3 be reciprocal to that of the ellipse x2 + y 2 =4 . If the
(C) Ellipse (r) P
oints of the conic have parametric hyperbola passes through a focus of the ellipse, then
representation (2011)
2 2
1 − t2 x y
2t (A) The equation of the hyperbola is − =1
=x 3
= ,y 3 2
1 + t2 1 + t2
(B) A focus of the hyperbola is (2, 0)
(D) Hyperbola (s) The eccentricity of the conic lies in 5
the interval 1 ≤ x < ∞
(C) The eccentricity of the hyperbola is
3
(t) P
oints z in the complex plane (D) The equation of the hyperbola is x2 − 3 y 2 =
3
satisfying
(
Re z + 1 )
2 2
=z +1 x2 y 2
Q.14 Tangents are drawn to the hyperbola − = 1,
9 4
parallel to the straight line 2x −y = 1. The points of
Q.11 The line 2x + y = 1 is tangent to the hyperbola contact of the tangents on the hyperbola are (2012)
x2 y 2
− = 1 . If this line passes through the point of 9 1 9 1
a2 b2 (A) , (B) − ,−
2 2 2 2 2 2
intersection of the nearest directrix and the x-axis, then
the eccentricity of the hyperbola is (2010)
( )
(C) 3 3, − 2 2 (D) −3 3, 2 2 ( )
PlancEssential Questions
JEE Main/Boards JEE Advanced/Boards
Exercise 1 Exercise 1
Q.7 Q.12 Q.15 Q.21 Q.5 Q.11 Q.12 Q.15
Q.24 Q.25 Q.27 Q.18 Q.22 Q.25
Exercise 2 Exercise 2
Q.3 Q.11 Q.18 Q.19 Q.3 Q.6 Q.8 Q.11
Q.17 Q.23
Previous Years’ Questions
Q.2 Q.6 Q.8
Previous Years’ Questions
Q.2 Q.3 Q.4 Q.8
M a them a ti cs | 12.45
Answer Key
Q.26 –1
Exercise 2
Single Correct Choice Type
JEE Advanced/Boards
Exercise 1
48
Q.1 7x2 + 12xy – 2y2 – 2x + 4y – 7 = 0 ; Q.2 a2 = 25/2 ; b2 = 16
5
Q.6 (i) 24/11 (ii) 20/11 Q.7 x + y ± 3 3 = 0
x2 y 2 y 2 x2 4 3
Q.18 − = 1 and − = 1 Q.19 (–4, 3) and − , −
16 9 16 9 7 7
5 3
Q.22 y = x + ; x – 3 = 0; 8 sq. unit
12 4
1 2 . 4 6 | Hyperbola
Exercise 2
Single Correct Choice Type
Solutions
x y ⇒ c = ± 3 × 3 = ±9
Sol 4: Tangent = sec θ − tan θ = 1
a b
3 3
Now the tangent cuts the axes at (a cosθ, 0) and y = − x + 9 or y = − x – 9
4 4
(0, b cot θ)
b
acos θ bcot θ equation of asymptotes ⇒ y = ± x
mid points ⇒ = h and k = a
2 2
a b Sol 7: Equation (3x – 4y + 7)(4x + 3y + 1) + c = 0
⇒ = sec θ and = tan θ
2h 2k ⇒ 12x2 – 12y2 – 7xy + 31x + 17y + (7 + c) = 0
a2 b2 a2 b2 Now, it passes through origin
⇒ − =1 ⇒ − =4
4h2 4k 2 h2 k2 ⇒ 7 + c = 0 ⇒ c = –7
Replacing h and k, we get locus as ⇒ equation = 12x2 – 12y2 – 7xy + 31x + 17y = 0
a2 b2 5
− =4 ⇒ a2 y 2 − b2 x2 =4x2 y 2 Sol 8: xy = 25 ⇒ parametric ⇒ 5t & y =
2 2
h k t
dy dy / dt ( −5) / t2 −1
= = =
Sol 5: We have tangents dx dx / dt 5 t2
⇒ y = mx ± a2m2 - b2 ⇒ y = mx ± 9m2 − 4
(-5,2)
2
⇒ (y – mx) = ( 9m − 4 )
2 2
(5t,5/t)
y + m x – 4mxy = 9m – 4
2 2 2 2
⇒ sec2θ – tan2θ = 1
1 c x x1 .y1 x1 y12 .c2
2 2 4
= − −
2
c2 c2 (c2 / y1 )2 a2h2 b2k 2
⇒ − =1
(a2 + b2 )2 (a2 + b2 )2
2
4 4 2 2
1 x1 y1 1 x1 − y1 a2 x2 b2 y 2
= + − 2(x1 .y1 )2 = ⇒ − =1
2 c2 c2 2 c (a2 + b2 )2 (a2 + b2 )2
P (a sec, b tan)
ab2 + a3 sec θ 2
Q= ,a b tan θ
a2 + b2 O T S’
S
slope of NQ
x y S' T × −h
Which should be same as + =1 ∴ Area of ∆ PTS' = P
h k 2
S' T × h
Area of ∆PTS =
acos θ bcot θ a2 + b2 2
⇒ = =
1/h 1/k 1 Using fine rule: S O T S’
PS'× PT sin φ
a2 + b2 a2 + b2 Area of ∆ PTS' =
⇒ h= ,k= 2
acos θ bcot θ
PS'× PT sin θ
ah bk Area of ∆ PTS' =
⇒ = secθ, =tanθ 2
2 2
a +b a + b2
2
1 2 . 5 0 | Hyperbola
c −2kh −2xy
Sol 16: Let x = ct and y = ⇒ a2 – h2 = ⇒ a2 − x2 =
t λ λ
dy dy / dt −1 / t −1
then, = = =
dx dx / dt t t2 Sol 18: x = a secθ, b = tan θ
so normal = t 2
x.sec θ y
− . tan θ = 1
(y − c / t) c a b
thus, = t2 ⇒ y – = t2x – ct3
(x − ct) t
⇒ ty – c = t3x – ct4 ⇒ ct4 – t3x + ty – c = 0 R
this satisfies h, k
Q
thus,ct4 – ht3 + kt – c = 0
(acos,0)
h h
h
thus, ∑ ti = ⇒ ∑ c.ti =h
i=1 c i=1
Now coordinate of Q ⇒ x = a cosθ.
⇒ x1 + x2 + x3 + x4 = h
Coordinates of R
similarly we have t1. t2. t3. t4 = –1 and
−k (sec θ − 1)b
(t1. t2. t3) + t2. t3. t4 + t3. t4. t1 + t4. t1. t2 = ⇒ =y
e tan θ
h
dividing by ∏ ti both sides Now
i=1
2h = a + a cosθ = a(1 + cosθ). . (i)
c c c c
⇒ + + + =k ⇒ Σy i =
k (sec θ − 1)b
t1 t2 t3 t 4 and2k = 0 +
tan θ
(iii) = –1
(1 − cos θ)b
2k =
⇒ –c4 = c4. sin θ
–c4 = x1. x2. x3. x4 Now
(1 − cos θ)2 b2 + b2 sin2 θ
And = –1 4k2 + b2 =
sin2 θ
c c c c
⇒ –c4 = . . . ⇒ y1. y2. y3. y4 = –c1
t1 t2 t3 t 4 (2 − 2cos θ)2 b2
(4k2 + b2) =
sin2 θ
a.sec θ.x b. tan.y
Sol 17: tangent ⇒ – =1 2(1 − cos θ)b2 2.(b2 )
2
a b 2 = = from (i)
sin2 θ (1 + cos θ)
x y
sec θ − tan θ =1 2b2
a b ⇒ 4k2 + b2 =
(2h / a)
tangent ⇒ y = mx ± a2m2 − b2
⇒ ab2 = h(4k2 + b2)
tangent passes through h, k
Sol 19: Let x = 2at
k = mh + a2m2 − b2
y = at2
(k – mh) = a m – b
2 2 2 2
dy 2at
then = =t
dx 2a
M a them a ti cs | 12.51
k 2 − h2 −h c
cosθ = and tan θ = ⇒ y − = t2(x – ct)
k t
k 2 − h2
⇒ ty – c = t3x – ct4
a.k −ah
so points A , ⇒ ct4 – t3x + ty – c = 0
2 2
k −h k 2 − h2
Thus, passes through
this satisfies the line
h.ak ahk
⇒ + =(h2–k2)
2 2 2 2
k −h k −h
1 2 . 5 2 | Hyperbola
1 2c
be of the form y = − x+ x2 y 2
t 2 t Sol 4: (A) − =1
9 4
1 2x
y = mx + 2c −m ⇒ t = y= +C
−m 5
c x2 − 9
∴ Point is ,c −m 4x2 4Cx
4 = + C2 +
−m 9 25 5
c 16 4Cx
Sol 26: x = ct and y = 4x2 × − − 4 − C2 = 0
t 225 5
dy −1 64x2 – 180Cx – 180 – 45C2 = 0
Now =
dx t2 D=0
t1
180C2 = 4×64(–180 – 45C2)
M a them a ti cs | 12.53
a2h b2k
⇒ + = a2 + b2
x1 y1
b2
b = a 1−
a2 b
ae
b2 1
⇒ =
a2 2
c / t − c / t′
We have t2 =
ct − ct′ a2
ehyperbola = 1+ = 1+2 = 3
b2
(t′ − t)( −1) −1
⇒ t2 = ⇒ t′ =
t.t′.(t − t′) t3
Sol 17: (C) Let any tangent of (x1, y1)
x.x1 y.y1
Sol 13: (A) 9(x2 – 4x + 16) – 16(y2 – 6y + 9) then + = 1 [1st tangent]
2
a b2
–252 + 144 – 144 = 0
and x. x1 – y. y1 = c2 [2nd tangent]
⇒ 9(x – 2)2 – 16(y – 3)2 = 252
Now, m1. m2 = –1
⇒ Centre ⇒ (2, 3)
−b2 x1 x1
⇒ = –1
x2 y 2 a2 y y
Sol 14: (D) − =1 1 1
25 16 (4)
tangents ⇒ −b2 x1
2 x2
1
⇒ . = –1 ⇒ +b2 2 = + a2 … (i)
-5 5 2 2
a y1 y
y = mx + 25m2 − 16 1
(-4)
⇒ (y – mx) = 25m – 16
2 2
x12 y12
Now + = 1 & x12 – y12 = c2
⇒ the point (1, 2 2 ) satisfy this a2 b2
a2 (b2 + c2 ) 1
so b2. × = a2
2 2
(a + b ) b (a − c2 )
2 2
a2 + b2 (h,k)
⇒ a2 – b2 = 2c2 (ct2,c/t2)
y2 y
Sol 18: (B) + +1=0
b 2
ba2 Now, 2h = c(t1 + t2)
2
1 4 ×1 c c c.(t1 + t2 )
a2 – 4ac ⇒ − ≥0 and 2k = + =
2 t1 t2 t1 .t2
ba b2
1 4 1 4 c / t2 − c / t1 −1
⇒ − ≥ 0[b2 > 0] ⇒ − ≥0 m= =
b a 2 4
b 2
a 4 1 ct2 − ct1 t1 t2
1 1 1 −c.(t1 + t2 )
⇒ ≥4⇒ ≥2⇒ ≥ a2 m=
a 4 2
a 2 t1 .t2c(t1 + t2 )
−2k
hx ky h2 k2 m=
Sol 19: (C) − = − 2h
a2 b2 a2 b2 ⇒ k + mh = 0 ⇒ y + mx = 0
2 2
hα kβ h k
− = −
a 2
b 2
a 2
b2 Sol 23: (A) Let (h, k) be the midpoints of chords having
slope 2
2 2
h α k β α2 β2 2 1
⇒ − − − = −
⇒ tan θ =2 ⇒ sin θ = and co s θ =
a 2a b 2b 4a2 4b2 5 5
h α β Let the two endpoints of the chord be a distance r from
⇒ Centre: = and k = (h, k)
a 2a 2
⇒ endpoints of the chord are
x2 y2
Sol 20: (B) − =1
5 5cos2 α (h + r cos θ, k + r sin θ ) and (h − r cos θ, k − r sin θ )
r 2r r 2r
b2 2 h
=+ ,k+ h
and =− ,k−
so e1 = 1+ = 1 + cos α 5 5 5 5
a2
a2 + b2 a2 + b2
Previous Years’ Questions Thus, y = –
b
i.e., k = −
b
Sol 1: (B) Given equation is
Sol 3: (B) Let (h, k) be point whose chord of contact
x2 y2 with respect to hyperbola x2 – y2 = 9 is x = 9.
− = 1, where | r | < 1
1−r 1+r We know that, chord of contact of (h, k) with respect to
⇒ 1 – r is (+ve) and 1 + r is (+ve) hyperbola x2 – y2 = 9 is T = 0
x2 y2 ⇒ h.x + k(–y) – 9 = 0
∴ Given equation is of the form − =1
2 2
a b ∴hx – ky – 9 = 0
Hence, it represents a hyperbola when | r | < 1. But it is the equation of the line x = 9.
This is possible when h = 1, k = 0 (by comparing both
Sol 2: (D) Firstly we obtain the slope of normal to equations).
x2 y2 Again equation of pair of tangents is T2 = SS1.
− = 1 at (a secθ, b tanθ)
a2 b2 ⇒ (x – 9)2 = (x2 – y2 – 9)(t2 – 02 – 9)
On differentiating w.r.t. x, we get ⇒ x2 – 18x + 81 = (x2 – y2 – 9)(–8)
2x 2y dy dy b2 x ⇒ x2 – 18x + 81 = –8x2 + 8y2 + 72
− × =0⇒ =
a 2
b 2 dx dx a2 y
⇒ 9x2 – 8y2 – 18x + 9 = 0
Slope, for normal at the point (a sec θ, b tan θ) will be
(
∴
φ + θ = π/2) Thus, the point of contact is (4, – 6 )
sec θ − cosecθ x2 y 2
= = –1 Sol 6: (B) The eccentricity of + = 1 is
cosecθ − sec θ 16 25
M a them a ti cs | 12.57
16 3
e1 = 1− = Sol 9: (B) Equation of normal to hyperbola at (x1, y1) is
25 5
a2 x b2 y
5 + = (a2 + b2)
∴e2 = (∵e1e2 = 1) x1 y1
3 a2 x b2 y
∴At (6, 3) ⇒ + = (a2 + b2)
⇒ Foci of ellipse (0, ±3) 6 3
⇒ Equation of hyperbola is It passes through (9, 0)
x2 y 2 a2 .9
− = –1 ⇒ = a2 + b2
16 9 6
3a2 a2
⇒ – a2 = b2 ⇒ =2
Sol 7: (A) The given ellipse is 2 b2
x2 y 2 b2 1
+ = 1 ⇒ a = 2, b = 3 ∴e2 = 1 + =1+
4 3 a2 2
1
∴3 = 4(1 – e2) ⇒ e = 3
2 ⇒ e=
2
1
∴ae = 2 × =1
2 1 ae
Sol 10: (B)
= 2b . ( 2ae )=
⇒b
Hence, the eccentricity e1, of the hyperbola is given by 2 2
a2e2
1 = e1sin θ ⇒ e1 = cosec q
(
⇒ a2 e2 − 1 = ) 4
⇒ 3e2 = 4 ⇒ e =
2
3
⇒ b2 = sin2θ(cosec2θ – 1) = cos2q
x2 y2
Hence, equation of hyperbola is − =1
sin2 θ cos2 θ JEE Advanced/Boards
or x2cosec2θ – y2sec2θ = 1
Exercise 1
Sol 8: (B) Given equation can be rewritten as
(x − 2)2 (y + 2)2 SP
− =1 Y Sol 1: = 3
4 2 PM
B
For point A(x, y) (x − 1)2 + (y − 1)2
X’ X = 3
2 A (2x + y − 1)
3 C
e= 1+ = 5
4 2
Y’
Squaring
x– 2 =2⇒ x=2+ 2
5[(x – 1)2 + (y – 1)2] = 3(2x + y – 1)2
For point C(x, y)
⇒ 7x2 – 2y2 + 12xy + 4y – 2x – 7 = 0
x– 2 = ae = 6
x= 6+ 2 x2 y2
Sol 2: − =1
a2 b2
Now,
7x + 13y = 87
AC = 6+ 2 –2– 2 = 6 –2
5x – 8y = –7
b2 2
andBC = = =1 87 − 7x 5x + 7
a 2 ⇒ =
Area of ∆ABC 13 8
1 3 ⇒ 8. 87 – 7. 13 = 121 x
= × ( 6 − 2) × 1 = – 1 sq. unit
2 2 ⇒ 121x = 605
1 2 . 5 8 | Hyperbola
x = 5, y = 4 Centre (–1, 2)
2b2 32 2 16 5
= e= 1+ =
a 5 9 3
5b2 = 16 2 a
foci = (–1 + ae, 2) = (4, 2)
25 16
− =1
a2 b2 = (–1 – ae, 2) = (–6, 2)
25 16 9 4
− =1 Directrix x + 1 = ⇒ x=
a2
a 2 5 5
−9 −14
25 2 – 5a = a2 2 x+1= ⇒ x=
5 5
a2 2 + 5a – 25 2 =0 2b2 2.16 32
Latus rectum = = =
a 3 3
−5 ± 25 + 200 5 −10
a= = or Length of major axis = 2×4 = 8
2 5 2 2
Length of minor axis = 2×3 = 6
Now, 5b = 16 2
2a Equation of axis is y = 2
5
⇒ a>0⇒ a=
2 Sol 5: P1(ct1, c/t1) P2(ct2, c/t2)
x2 y2 2h
Sol 3: − =1 t1 + t2 =
100 25 c
1 5 1 1 2k 2h 2k h
e= 1+ = + = ⇒ = t1t2=
4 2 t1 t2 c ct1 t2 c k
2
5 1 1
S = (ae,0) = × 10,0 = ( 5 5 , 0) c (t1 – t2) + c − = 4d2
2 2 2
2
t1 t2
S′ = (–ae, 0) = (– 5 5 , 0) 1 1
2
(t1 + t2)2–4t1t2+ +
A = (10, 0) t1 t2
SA = (10 – 5 5 ) 4 4d2
– =
t1 t2 c2
S′A = (10 + 5 5 )
2 2
SA. S′A = 100 – 75 = 25 2h 2k 4 4d2
+ – 4t1t2 – =
c c t1 t2 c2
Sol 4:
(2h)2 + (2k)2 h k 4d2
− 4 + =
c2 k h c2
16x2 – 9y2 + 32x + 36y – 164 = 0 (h2 + k2)hk – c2(h2 + k2) = d2kh
= 164 + 16 – 36 = 144
Sol 6: y – 2 = m(x – 6)
(x + 1)2 (y − 2)2
− =1 y = mx + 2 – 6m
9 16
M a them a ti cs | 12.59
11m2 – 24m + 20 = 0
= (a2 sec2 θ + a2e2 + b2 tan2 θ)2 − (2a2esec θ)2
24
m 1 + m2 =
11 = a2sec2θ + b2tan2θ – (a2 + b2)
20
m 1m 2 = = CP2 – (a2 + b2)
11
5
Sol 10: y – = mx
Sol 7: y = –x + c 2
x2 – 4(c – x)2 = 36 5
y = mx +
2
x2 – 4(c2 + x2 –2cx) = 36
25
3x2 – 8cx + 4c2 + 36 = 0 3x2 – 25 = 2 m2 x2 + + 5mx
4
⇒ x + y = ±3 3
75
64c2 = 12(4c2 + 36) x2(3 – 2m)2 – 10mx – =0
2
16c2 = 12(4c2 + 36)
75
100m2 = 4(3 – 2m2) −
4c2 = 3c2 + 27 2
c2 = 2 ⇒ c = ±3 3 50m2 = 150m2 – 225
100m2 = 225
Sol 8: Equation of chord
9 3
x θ − θ2 y θ1 + θ2 θ1 + θ2 m2 = ; m = ±
cos 1 − sin = cos 4 2
a
2 b 2 2
2y = 3x + 5 or 2y + 3x = 5
If it pass through (ae, 0)
θ + θ2 y −k b2h
cos 1 Sol 11: =
2 x −h a2k
e=
θ − θ2
cos 1 ⇒ x2 + y2 = a2
2 2
a2k
θ − θ2 h + 2 (y − k) + y2 = a2
cos 1 bh
1 2
=
e θ1 + θ2 a4 k 2 2a2k
cos h2+ (y2 + k2 – 2ky)+ (y – k) + y2 = a2
2 4 2
b h b2
1 2 . 6 0 | Hyperbola
−h
a4 k 2 −2k 3a4 2a2k Slope of normal =
k
y2
4 2
+ 1 + y 4 2 + 2 + h2 – a2
b h b h b
[slope of OR =
k
]
h
a4 k 4 2a2k 2
+ − =0 that has equation:
b 4h2 b2
a2 x b2 y
2a2k 2k 3a4 + = a2 + b2
− x1 y1
y1 + y 2 b2 h2b 4
=
y1 y 2 a4k 4 2a2k 2 a2 y1 −a2 b − tan θ −a
h2 − a2 + 4 2 − ⇒ slope = – × = × = sin θ
b h b2 b 2 x1 b 2 asec θ b
a2k 2 − b2h2
x2 y 2 ⇒ cos θ =
Sol 12: − = 1; y – β = m(x – α) ax
2 3
x2 − 2 1 bh
= (mx – mα + β)2 and tanθ =
α 3
a k − b2h2
2 2
2
3x= − 6 2(m2 x2 + m2 α2 + β2 − 2m2 αx − 2mα + 2mxβ) Putting
x2 (3 − 2m2 ) + 2x(2m2 α − 2mβ) − 6
bh a2 x
−2m2 α2 − 2β2 + 4mαβ = 0 x1 = b − , y1 = asec θ =
a2k 2 − b2h2 a2k 2 − b2h2
(4m2 α − 4mβ=
)2 4(3 − 2m)2 (4mαβ − 2m2 α2 − 2β2 − 6)
2m2 (mα − β)2= (3 − 2m2 )(2mαβ − m2 α2 − β2 − 3) a2 x b2 y
in equation + =a2 + b2 and simplifying, we get
2m4 α2 + 2m2β2 − 4m3 αβ = −4m3 αβ + 2m4 α2 + x1 y1
6mαβ − 3m2 α2 − 3β2 + 9 + 2m2β2 + 6m2
locus as (x2 + y 2 )2 (a2 y 2 − b2 y 2 )= x2 y 2 (a2 + b2 )2
2 2 2
m (3α − 6) − 6mαβ + 3β + 9 = 0
3β2 + 9
= 2 ⇒ β2 + 3 = 2α2 − 4 Sol 15:
3α2 − 6
β2 = 2α2 − 7 (a sec, a tan)
G
Sol 13: Equation of any normal to the hyperbola is S(ae,0)
m(a2 + b2 )
y mx −
=
a2 − b2 m2
⇒ (a2 − b2 m2 ) (y − mx)=
2
m2 (a2 + b2 )2 a2 x b2 y
Normal: + = a2 + b2
x1 y1
If it passes through the point (x1, y1), then
⇒ ax. cosθ + by. cotθ = a2 + b2
(a2 − b2 m2 ) (y1 − mx1 )2 = m2 (a2 + b2 )2
Now for coordinates of G ⇒ put y = 0 in above equation
It is a 4 degree equation in m, so it gives 4 values of m.
corresponding to these 4 values, four normal can be (a2 + b2 )
⇒ x= .sec θ
drawn from the point (x1, y1). a
b2 b2 a2 + b2
Sol 14: also e2 = 1 + ⇒ e= 1+ =
a2 a2 a
(h,k)
(a sec, b tan)
Now
R
2
(a2 + b2 )sec θ
O SG =
2
− a2 + b2
a
M a them a ti cs | 12.61
x y mx + 6
Given that = ….(ii)
tan θ = ⊥ or x sec θ − y tan θ = a … (i)
a Equating (i) and (ii)
Equation of other two sides of the triangle are
⇒ 100m2 − 49 =
6
x – y = 0 ...(ii) ⇒ 100m2 49 =
36
17
⇒ 100m2 = 85 =
x + y = 0 … (iii) 100 20
⇒ m =17
Solving (ii) and (iii), (iii) and (i), (i) and (ii) in pairs, 20
the co-ordinates of the vertices of the triangle
a a
are (0, 0); , and Sol 19:
sec θ + tan θ sec θ + tan θ
a a
, 3x/4
sec θ − tan θ sec θ − tan θ
S’ 2x P 3x S
∴ Area of triangle =
1 2 . 6 2 | Hyperbola
Now, a = 4, b = 3 θ− 0
= a cos … (i)
9 25 5 2
⇒ e2 = 1 + = ⇒ e=
16 16 4
And normal to the hyperbola at P(a sec θ, a tan θ) is
so coordinates of S = (5, 0) and given by
S′ = (–5, 0) x y
+ 2a
= … (ii)
sec θ tan θ
3 × (–5) + 2 × 5 0 × 3 + 2 × 0
so P = , = (–1, 0)
3+2 3+2 Note that equation (i) and (ii) are the same lines
comparing these lines, we get
Now slope of line through P ⇒ –1
⇒ y = –x + C θ− 0 θ− 0 θ− 0
cos − sin a cos
= 2 = 2 2
⇒ 0 = 1 + c ⇒ c = –1
1 1 1
so line through P = y = – x – 1 sec θ tan θ 2a
x y Solving above and simplifying, we get
Now asymptotes ⇒ − = 0
4 3 0 tan θ (4 sec2 θ − 1)
tan =
x y
and + = 0
4 3 x2
Sol 22: − y2 = 1
9
Point of intersection ⇒
now, line: y = mx + 9m2 − 1
x (x + 1) x (x + 1)
+ =0 − =0
4 3 4 3 ⇒ 2 = 3m + 9m2 − 1
7x + 4 = 0x = –4
−4 ⇒ (2 – 3m)2 = 9m2 − 1
x= y=3
7
⇒ 9m2 – 6m × 2 + 4 = 9m2 – 1
−3
and y =
7 (one m = ∞)
A
−4 −3
, and (–4, 3) S = 12 m
7 7
5
⇒ m= (-3,0) (3,0)
12
x2 y 2 C
Sol 20: Eq. of Hyperbola; x2 − 2y 2 =
18 ⇒ − 1
=
18 9 so one tangent ⇒ x = 3 B
5 3
and one is y = x+
mx ± m2a2 − b2
Eq. of tangent y = 12 4
⇒ y = mx ± m2 .18 − 9 12y = 5x + 9
Now tangent at B
∴ this is perpendicular to y = x
x.x1
⇒ the value of m =-1 – y. y1 = 1
9
⇒ y = -1 x ± 18 − 9 ⇒ same –5x + 12y = 9
−5 12 9 −4
y =−x ± 9 ⇒ = = ⇒ y1 =
x1 / 9 − y1 1 3
y =−x ± 3
x1 = –5
Sol 21: The chord joining the points P(a sec θ, a tan θ) 1
so ∆ = × AC × height
θ− 0 θ− 0 2
and given by x cos − y sin
2 2 1
= × 2 × [(3 – (–5)] = 8 sq. unit
2
M a them a ti cs | 12.63
Sol 23:
h2 k 2
R (a sec, b tan) −
Now d = 4 9 =2
2
h k2
+
S’ (-a,0) (a,0) S a2+b2 16 81
22 P
)- a+b ,0)
Sol 2: (A) Equation of tangent,
x.sec θ y tan θ
We have − =1
a b
(S ⇒ R – SR)2 =S ⇒ R2 + SR2 – 2S ⇒ R. RS = (2a)2 h.sec θ
so for h ⇒ =1
⇒ (S⇒ R + SR)2 = (S⇒ R – SR)2 + 4S⇒ R. SR a
⇒ h = a cos θ O A
= 4a2 + 4. S⇒ R × SR …(i) (h,0)
and h′ = –b cotθ
Now, tangent B P
2 2
x.sec θ y. tan θ a −b (k,0)
=1
⇒ − (h,k)
⇒ − =1
a b h k
1 a2 b2
⇒ − 1
=
sec2 θ tan2 θ h2 k2
+
a2 b2
h−0 k −0 −( −2c t)
a2b2 Sol 3: (D) = =
⇒ P = 2
…(ii) 1 t 2
1 + t2
b2 sec2 θ + a2 tan2 θ
2ct 2ct3
2 2 2 2 2 h= ,k=
SR =2
(asec θ − a + b ) + b tan θ 1 + t4 1 + t4
P2 P
Sol 4: (B) We have
Exercise 2 2s = t2 + 1and2t = 2/s
⇒ t = 1/s
Single Correct Choice Type
1
⇒ 2s = +1
hx ky h2 k 2 s2
Sol 1: (B) − = −
4 9 4 9 ⇒ 2s3 = 1 + s3
1 2 . 6 4 | Hyperbola
⇒ 2s3 – s – 1 = 0 y(t1 t2 ) x
− =1
⇒ (s – 1) (2s + s + 1) = 0
2 c(t1 + t2 ) c(t1 + t2 )
−x x2 y 2
⇒ y= +N Sol 10: (A, C) − =1
t1 .t2 1 5
c −c ⇒ (8 – 2m)2 = m2 – 5
= +N
t1 t2
⇒ 4m2 + 64 – 32m = m2 – 5
1 1
⇒ N = c + ⇒ 3m2 – 32m + 69 = 0
t1 t2
t + t ⇒ 3m2 – 23m – 9m + 69 = 0
−x
Now, y = +c. 1 2
t1 .t2 t1 .t2 ⇒ m(3m – 23) – 3(3m – 23) = 0
M a them a ti cs | 12.65
23 t
⇒ m = 3 or m = and y2 + 2 = 2 sin2 − 1 +2
3 2
Now y = 3x + 2(A) y2 = 2cos t
23x (23)2 − 45 ⇒ x2 – y2 = 6
or3y = ±
3 3
b x
⇒ 3y = 23x ± 22 (B)t = 1 −
y a
x2 y 2 x2 y2
− ⇒ + =1
Sol 12: (B, D) =0 a2 b2
16 9
Now tangent
Sol 14: (A, D) We have equation of circle
2 2 2
1 ⇒ y = mx ± a m −b (x – x1)(x – x2) + (y0 – y1)(y – y2) = 0
Now,
y = mx ± 16m2 − 9
x1 = ct1&y1 = c/t1
tangent 2 ⇒ y = mx ± 3 m2 + 1
x2 = ct2&y2 = c/t2
so16m – 9 = 9(m + 1)
2 2
c / t2 − c / t1 −1
⇒ 7m2 = 18 so slope ⇒ =
ct2 − ct1 t1 .t2
2 Now, slope = 1
⇒ m = ±3
7
1
⇒ –t1.
2 18 t2
soy = 3 x ± 3 +1
7 7
−1
⇒ t1 =
2 16 t2
y= 3 x±
7 7
putting this above
1
(x2 + y2 – 2c2) + c[x – y] − t1 = 0 Now AO + BO = 2a
t1
52 + 122 + 242 + 72 = 13 + 25 = 38
Now when x = y & x2 + y2 = 2c2
this is satisfied for So2ae = 192 + 52
⇒ a2m2 – b2 = a2 – b2m2
Sol 16: (A, D) B
(24,7) ⇒ a2(m2 – 1) = (m2 – 1)(–b2)
A ⇒ m = ±1
(5,12)
So, y = x ± a2 − b2 or y = –x ± a2 − b2
O
M a them a ti cs | 12.67
x2 y 2
Sol 20: (B, D) − =1 a2 x b2 y
18 9 Normal: + = a2 + b2
Now m = –1 x1 y1
So y = mx ± a2m2 − b2 x1 .(a2 + b2 )
⇒ g ⇒ x1 = x =
a2
⇒ y = –x ± 18( +1) − 9
sec θ.(a2 + b2 )
x= = 4 secθ
⇒ y=–x±3 a
⇒ y = –x ± 3 tan θ(a2 − b2 )
and G ⇒ y = ⇒ 4 tanθ
b
⇒ x + y = 3 and x + y = –3
PC = 2 sec2 θ + tan2 θ
Sol 21: (C, D) 9(x2 + 2y) – 16(y2 – 2y) = 151
sec2 θ.(a2 + b2 )2 tan2 θ(a2 + b2 )
Og = +
9(x + 1) – 16(y – 1) = 151 + 9 – 16
2 2
a2 b2
= 1/ 2 ⇒ b2 = 1
2 2
x y
Let equation of ellipse be + = 1 (a > b) Equation of ellipse is x2 + 2y2 = 2
2
a b2
Coordinate of foci
1 b2
∴ = 1−
2 a2 1
(±ae, 0) = ± 2. ,0 = (± 1, 0)
2
b2 1 Hence, option (A) and (B) are correct.
⇒ = ⇒ a2 = 2b2
a 2 2
If major axis is along y-axis, then
∴x2 + 2y2 = 2b2 … (ii) 1 a2
= 1−
Let ellipse and hyperbola intersect as 2 b2
1 1
A sec θ, tan θ ⇒ b2 = 2a2
2 2
∴2x2 + y2 = 2a2
On differentiating Eq. (i),
2x
⇒ y′ = −
dy dy x y
4x – 4y =0⇒ =
dx dx y −2
⇒ y′ 1 1
=
2 2
a x b y sec θ , tan θ sin θ
∴ At (6, 3) ⇒ + = (a2+b2) 2 2
6 3
As ellipse and hyperbola are orthogonal
It passes through (9, 0)
−2
a2 .9 ∴ .cosecθ = –1
⇒ = a2 + b2 sin θ
6 π
⇒ cosec2θ = 1 ⇒ θ = ±
3a 2
a 2 4
⇒ − a2 = b2 ⇒ =2
2 b2 ∴ 2x2 + y2 = 2a2
b2 1 3 1
∴e2 = 1 + =1+ ⇒ ⇒ 2+ = 2a2
a 2 2 2 2
5
dy sec θ ⇒ a2 =
= = cosecq 4
dx at A tan θ
5
⇒ 2x2 + y2 = , corresponding foci are (0, ± 1)
and differentiating Eq. (ii) 2
dy
2x + 4y =0 x2 y 2
dx Sol 3: (B, D) Here, equation of ellipse + =1
4 1
dy x 1 b2 1 3
= − = − cosecq ⇒ e2 = 1 – =1– =
dx at A 2y 2 a 2 4 4
3
Since, ellipse and hyperbola are orthogonal ∴e = and focus (± ae, 0)
2
1
∴ − cosec2θ = –1
2 ⇒ (± 3 , 0)
⇒ cosec2θ = 2 ⇒ θ = ± π x2 y2
For hyperbola − =1
4 a2 b2
1 1 b2
∴ A 1, or 1, − e12 = 1 +
2 2 a2
∴ From Eq. (i), 1 4
where, e12 = =
e 2 3
2
1 b 4 2
1 + 2 = 2b
2
⇒ 1+ =
2 a2 3
M a them a ti cs | 12.69
t
b2 1 e
2t1
−e
−2t1
1 e2t e−2t 1
∴ = … (i) = − − − 2t
a2 3 4 2 2 2 0
and hyperbola passes through (± 3 ,0)
2t −2t1
e 1 −e 1 2t1 −2t
3 = − (e − e 1 − 4t1 ) =t1
⇒ =1 4 4
a2
⇒ a2 = 3 … (ii)
Sol 5: Let any point on the hyperbola is (3secθ, 2tanθ)
From Eqs.(i) and (ii), we get
∴ Chord of contact of the circle x2+y2 = 9 with respect
b2 = 1 … (iii) to the point (3sectθ, 2tanθ) is,
2 2
x y (3secθ)x + (2tanθ)y = 9 … (i)
∴ Equation of hyperbola is − =1
3 1 Let (x1, y1) be the mid point of the chord of contact
Focus is ( ± ae1, 0)
⇒ Equation of chord in mid point form is
2
⇒ ± 3. ,0 ⇒ (± 2, 0) xx1 + yy1 = x12 + y12 … (ii)
3
Since, Eqs. (i) and (ii) are identically equal
∴ (B) and (D) are correct answers.
3sec θ 2 tan θ 9
∴ = =
et1 + e− t1 et1 − e− t1 x1 y1 x1 + y12
2
Sol 4: Let P = ,
2 2 9x1
⇒ secθ =
3(x12 + y12 )
e− t1 + et1 e− t1 − et
and Q = , 9y1
2 2 and tanθ =
2(x12 + y12 )
We have to find the area of the region bounded by the
curve x2 – y2 = 1 & the lines joining the centre x = 0, Thus, eliminating ‘θ’ from above equation, we get
P(f1)
( sec2θ – tan2θ = 1)
∴
x2 y 2 (x2 + y 2 )2
x’
A
x ∴Required locus is − =
-1 C 1 N 9 4 81
x2 x2 − 8x
For their points of intersection + =1 Sol 10: A→p; B→s, t; C→ r; D→ q, s
9 4
⇒ 4x2 + 9x2 – 72x = 36 1 h2
(p) = 4 1 +
⇒ 13x2 – 72x – 36 = 0 k2 k 2
⇒ 13x2 – 78x + 6x – 36 = 0
⇒ 13x(x – 6) = 6(x – 6) = 0
(
1 4 k 2 + h2
⇒= )
2
13 1
⇒ x = 6, x = − ∴ h2 + k 2 =
which is a circle.
6 2
13
x= − not acceptable (q) If z − z1 − z − z 2 =where
k k < z1 − z 2 the locus is
6
a hyperbola.
Now, for x = 6, y = ± 2 3
(r) Let t = tan α
Required equation is (x – 6)2 + (y + 2 3 )(y – 2 3 )=0
⇒ x2 – 12x + y2 + 24 = 0 ⇒x
= 3 cos 2 α and sin2 α =y
x
⇒ x2 + y2 – 12x + 24 = 0 or cos 2 α = and sin 2 α =y
3
a x2
Sol 8: On substituting ,0 in y = –2x + 1, ∴ y 2 sin2 2α + cos2=
+= 2α 1 which is an ellipse.
e 3
(s) If eccentricity is 1, ∞ ) , then the conic can be a
2a
we get 0 = − +1
e parabola (if e = 1) and a hyperbola if e∈ (1, ∞ ) .
a 1 (t) Let z =
x + iy; x, y ∈ R
⇒ =
e 2
⇒ ( x + 1 ) − y 2 = x2 + y 2 + 1
2
(x − 2 ) − (y + 2 )
2 2
4 2 a2 a2
=a 2,=
b 2
1
3 e2 = 1 + 4 −
e= a2
2
Area =
1
(
a= e − 1) ×
(
b2 1 3 − 2 × 2
=
) ( 3− 2 ) e2= 5 −
e2
1
2 a 2 2 2
3 ⇒ e4 − 5e2 + 4 =
0
⇒ Area = − 1 .
2
( )(
⇒ e2 − 1 e2 − 4 =
0 )
M a them a ti cs | 12.71
i.e., 2x − y =±4 2
e2 − 1 ≠ 0 e =2
x y x y
⇒ − 1 and −
= − 1
=
Sol 12: (B) Equation of normal is 2 2 4 2 2 2 4 2
2
a2 xx1 yy1
( y − 3) = −a2 ( x − 6 ) ⇒ 2
= 1⇒e =
3
2
. Comparing it with
9
−
4
1
=
2b 2b
9 1
2 2 We get point of contact as ,
x y 2 2 2
Sol 13: (B, D) Ellipse is − 1
=
3 2 9 1
and − ,− .
2 2 2
2
1= (
22 1 − e2 ⇒=
e ) 2
3
MATHEMATICS
FOR JEE MAIN & ADVANCED
SECOND
EDITION
Exhaustive Theory
(Now Revised)
Formula Sheet
9000+ Problems
based on latest JEE pattern
PlancEssential
Questions recommended for revision
13. R E L AT I O N S A N D
FUNCTIONS
1. INTRODUCTION TO SETS
A collection of well-defined objects that are distinct are known as sets.
Well-defined object clearly defines if the object belongs to a given collection or not.
Well-defined collections
(a) Odd natural numbers less than 10, i.e., 1, 3, 5, 7, 9.
(b) Rivers of India.
(c) Vowels in the English alphabet a, e, i, o, u.
PLANCESS CONCEPTS
Symbol Meaning
⇒ Implies
∈ Belongs to
A ⊂ B A is subset of B
⇔ Implies and is implied by
∉ Does not belong to
| or : or S.t. Such that
∀ For all
∃ There exists
iff If and only if
& And
Symbol Meaning
a/b a is divisor of b
N Set of natural numbers
W Set of whole numbers
I or Z Set of integers
Q Set of rational numbers
Qc Set of irrational numbers
R Set of real numbers
C Set of complex numbers
3. REPRESENTATION OF SETS
The following two methods are used to represent sets:
(a) Roster or Tabular form
(b) Set builder form
4. SUBSET
Set A is a subset of B if B has all the elements of A, denoted by A ⊂ B (read as A is subset of B).
5. TYPE OF SETS
5.1 Null or Empty Set
Any set is called empty or null set if no elements are present in that set. It is denoted by { }.
1 3 . 4 | Relations and Functions
PLANCESS CONCEPTS
Sol: Refer to the definition of different types of sets in the above section.
(i) The elements 4, 8, 12 belong to both sets A and B. But 16 ∈ B and 18 do not belong to both A and B. So A ≠ B .
(ii) All elements present in set A is present in set B, and vice versa. Therefore A = B.
(iii) Given
A = {10, 20, 30, 40, …} and B = {10, 15, 20, 25, 30, …}
Since 15, 25, 35 are not multiples of 10, B does not belong to A.
∴ A ≠B.
Illustration 2: From the following sets, select equal sets:
A = {2, 4, 8, 12}, B = {1, 2, 3, 4}, C = {4, 8, 12, 14}, D = {3, 1, 4, 2}, E = {–1, 1}
F = {0, a}, G = {1, –1}, H = {0, 1} (JEE MAIN)
6. VENN DIAGRAMS
The diagrams drawn to represent sets and their relationships are called Venn diagrams
or Euler–Venn diagrams. Here we represents the universal U as set of all points within
U
rectangle and the subset A of the set U is represented by a circle inside the rectangle. B A
If a set A is a subset of a set B, then the circle representing A is drawn inside the circle
representing B. If A and B are not equal but they have some common elements, then we
represent A and B by two intersecting circles. Figure 13.2
U
A
A
Figure 13.3
7. SET OPERATIONS
7.1 Union of Sets
If A and B are two sets, then union (∪) of A and B is the set of all elements belonging to set A and set B. It is also
defined as A ∪ B = {x : x ∈ A or x ∈ B}. It is represented by shaded area in following figures.
U U
A B A B
U
A B
Figure 13.6
M a them a ti cs | 13.7
U
A B
A- B
Figure 13.7
AB
Figure 13.8
RELATIONS
1. ORDERED PAIR
A pair of elements written in a particular order is called an ordered pair. Let A and B be two sets. If a ∈ A, b ∈ B, then
elements (a, b) denotes an ordered pair, with first component a and second component b. Here, the order in which
the elements a and b appear is important. The ordered pair (1, 2) and (2, 1) are different, because they represent
different points in the co-ordinate plane.
Equality of ordered pairs: Ordered pair (a1, b1) is equal to (a2, b2) iff a1 = a2 and b1 = b2.
M a them a ti cs | 13.9
Illustration 3: If A = {2, 4} and B = {3, 4, 5}, then find (A ∩ B) × (A ∪ B) (JEE MAIN)
Note:
(i) Elements of A × B are called 2-tuples.
(ii) Elements of A × B × C are called 3-tuples.
(iii) Elements of A1 × A2 × … × An are also called n-tuples.
1 3 . 1 0 | Relations and Functions
Illustration 4: If n(A) = 7, n(B) = 8 and n(A ∩ B) = 4, then match the following columns: (JEE MAIN)
(i) n(A ∪B) (a) 56
(ii) n(A × B) (b) 16
(iii) n((A × B) × A) (c) 392
(iv) n((A × B) ∩ (B × A)) (d) 96
(v) n((A × B) ∪ (B × A)) (e) 11
1 a
2
3 b
Figure 13.9
Each element of set A to each element of set B is represented by lines from A to B in Fig. 13.9.
M a them a ti cs | 13.11
3. RELATION
A relation R from one set to another say from set X to set Y (R : X → Y) is a correspondence between set X to set Y
through which some or more elements of X are associated with some or more elements of Y. Therefore a relation
R, from a non-empty set X to another non-empty set Y, is a subset of X × Y, i.e. R: X → Y is a subset of A × B.
Every non-zero subset of A × B is defined as a relation from set A to set B. Therefore, if R is a relation from A → B
then R = {(a, b) | (a, b) ∈ A × B and a R b}.
If A and B are two non-empty sets and R: A → B be a relation such that R: {(a, b) | (a, b) ∈ R, and a ∈ A and b ∈ B},
then
(a) ‘b’ is an image of ‘a’ under R.
(b) ‘a’ is a pre-image of ‘b’ under R.
For example consider sets X and Y of all male and female members of a royal family of the kingdom Ayodhya. X =
{Dashrath, Ram, Bharat, Laxman, Shatrughan} and Y = {Koshaliya, Kaikai, Sumitra, Sita, Mandavi, Urmila, Shrutkirti}.
A relation RH is defined as “husband of” from set X to set Y.
RH
Dashrath Kaushlya
Sumitra
Ram Kakai
Bharat Urmila
Laxman Sita
Shatrughan Madhvi
Sharukirti
x y
Figure 13.10
RH = {(Dashrath, Koshaliya), (ram, sita), (Bharat, Mandavi), (Laxman, Urmila), (Shatrughan, Shrutkirti), (Dashrath,
Kakai), (Dashrath, Sumitra)}
(a) If a is related to b, it is symbolically written as a R b.
(b) It is not necessary for each and every element of set A to have an image in set B, and set B to have a pre-
image in set A.
(c) Elements of set A having image in B are not necessarily unique.
(d) Number of relations between A and B is the number of subsets of A × B.
Number of relations = no. of ways of selecting a non-zero subset of A × B.
= mnC1 + mnC2 + … +mnCmn = 2mn – 1.
3.1 Domain
Domain of a relation is a collection of elements of the first set participating in the correspondence, i.e. it is set of
all pre-images under the relation. For example, from the above example, domain of RH = {Dashrath, Ram, Bharat,
Laxman, Shatrughan}.
3.2 Codomain
All elements of any set constitute co-domain, irrespective of whether they are related with any element of
correspondence set or not, e.g. Y = {Koshaliya, Kakai, Sumitra, Sita, Mandavi, Urmila, Shrutkirti} is co-domain of RH.
1 3 . 1 2 | Relations and Functions
3.3 Range
Range of relation is a set of those elements of set Y participating in correspondence, i.e. set of all images. Range of
RH = {Koshaliya, Kakai, Sumitra, Sita, Mandavi, Urmila, Shrutkirti}.
Illustration 5: A = {1, 2, 3, 4, 5} and B = {2, 4, 5}. Relation between A and B is defined as a R b ⇒ a and b are
relatively prime or co-prime (i.e., HCF is 1). Find domain and range of R. (JEE MAIN)
Sol: Write the elements of the Relation and then write the domain and range.
R = {(1, 2), (1, 4), (1, 5), (2, 5), (3, 2), (3, 4), (3, 5), (4, 5), (5, 2), (5, 4)}
Domain of R {1, 2, 3, 4, 5}
Range of R {2, 4, 5}
Illustration 6: A = {Jaipur, Patna, Kanpur, Lucknow} and B = {Rajasthan, Uttar Pradesh, Bihar}
a R b ⇒ a is capital of b, a ∈ A and b ∈ B. Find R. (JEE MAIN)
4. REPRESENTATION OF A RELATION
4.1 Roster Form
In this form we represent set of all ordered pairs (a, b) such that (a, b) ∈ R where a ∈ A, b ∈ B.
1. Roster form
R = {(0-2, 1), (–2, 4), (–2, 9), (–1, 1), (–1, 4), (–1, 9), (4, 9)}
M a them a ti cs | 13.13
3. Arrow-diagram (mapping)
A B
1 a
2
3 b
Figure 13.11
5. TYPES OF RELATION
5.1 Reflexive Relation
R: X → Y is said to be reflexive iff x R x ∀ x ∈ X, i.e. every element in set X must be a related to itself. Therefore ∀
x ∈ X; (x, x) ∈ R, then relation R is called as reflexive relation.
N N
Figure 13.12
a R b ⇒ a + 2b = 10
R = {(2, 4), (4, 3), (6, 2) (8, 1)}
R–1 = {(4, 2), (3, 4), (2, 6) (1, 8)}
For example, a relation defined on the set of lines.
1. aR b ⇒ a || b
It is a symmetric relation because if line ‘a’ is || to ‘b’, the line ‘b’ is || to ‘a’.
where (a, b) ∈ L {L is a set of || lines}
2. L1RL2 L1⊥ L2 It is a symmetric relation
L1 , L2 ∈ L {L is a set of lines}
3. a R b ⇒ ‘a’ is brother of ‘b’ is not a symmetric relation as ‘b’ may be sister of ‘a’.
4. a R b ⇒ ‘a’ is cousin of ‘b’. This is a symmetric relation. If R is symmetric.
5. R = R–1.
6. Range of R = Domain of R.
FUNCTIONS
1. INTRODUCTION
The concept of function is of fundamental importance in almost all branches of Mathematics. It plays a major role
to solve real world problems in mathematics. As a matter of fact, functions are some special type of relations.
M a them a ti cs | 13.15
General definition:
Definition 1: Consider two sets A and B and let there exist a rule or manner or correspondence ‘f‘ which associates
to each element of A to a unique element in B. Then f is called a function or mapping f
from A to B. It is denoted by symbol f and represented by f: A →B (read as ‘f’ is a A B
function from A to B or ‘f maps A to B’).
If element a ∈ A is associated with an element b ∈ B, then b is called the ‘f image of a’
or ‘image of a under f’ or ‘the value of function f at a’. Also a is called the pre-image of
b or argument of b under the function f. We write it as
Figure 13. 13
b = f(a) or f: a → b or f: (a, b)
Function as a set of ordered pairs:
A function f: A → B can be expressed as a set of ordered pairs in which each ordered pair is such that its first
element belongs to A and second element is the corresponding element of B.
As such a function f: A → B can be considered as a set of ordered pairs (a, f(a)), where a ∈ A and f(a) ∈ B, which is
the f image of a. Hence, f is a subset of A × B.
Definition 2: A relation R from a set A to a set B is called a function if f
A B
(i) each element of A is associated with some element of B.
(ii) each element of A has unique image in B.
a b
Thus a function ‘f’ from set A to set B is a subset of A × B in which element a belonging
to A appears in only one ordered pair belonging to f. Hence, a function f is a relation
from A to B satisfying the following properties: Figure 13. 14
PLANCESS CONCEPTS
Every function is a relation, but every relation may not necessarily be a function.
f f f f
A B A B A B A B
x1 y1 x1 y1
X1 Y1 y2 x2 y2
X1 x2 y3 x3 y3
Y1 x3
X2 Y2 y4 x4 y4
x4 y5 x5 y5
It is a function Not a function It is a function Not a function
Figure 13.15
2. RELATION VS FUNCTION
L
y
y
y = f(x)
y2 C y = f(x)
y3
y1 y2 B
y1 A
x
O x1 x2 x3
(a) x0 x
O
(b)
Figure 13.16 Figure 13.17
These figures show the graph of two arbitrary curves. In figure 16, any line drawn parallel to y-axis would meet the
curve at only one point. That means each element of X would have only one image. Thus figure 16 (a) represents
the graph of a function.
In figure 17, certain line parallel to y-axis (e.g., line L) would meet the curve in more than one point (A, B and C).
Thus element x0 of X would have three distinct images. Thus, this curve does not represent a function.
Hence, if y = f(x) represents a function, then lines drawn parallel to y-axis through different point corresponding to
points of set X should meet the curve at one point.
x2 y2
Equation of an ellipse + 1 is a relation, which is a combination of two functions
=
a2 b2
x2 x2
=y b 1− and y = – b 1 − . Similarly, the equation of the parabola y2 = x is a combination of two functions
a 2
a2
as shown in Fig. 13.18.
y y
2
x
y=b 1- 2
a y= x
x’ x x’ x
O O
2
x y =- x
y = -b 1- 2
a
y’ y’
Figure 13.18
f B
A c
o
d d
o o
m m
a a
i i Elements of range
n n
Domain
Figure 13.19
4. COMMON FUNCTIONS
4.1 Polynomial Function
If a function f is defined by f(x) = a0xn + a1xn–1 + a2xn–2 + …. an–1x + an where n is a non-negative integer and a0, a1,
a2, …., an are real numbers and a0 ≠ 0, then f is called a polynomial function of degree n.
2
2
f(x) = x + 1 f(x) = x + 1
PLANCESS CONCEPTS
(a) A polynomial of degree one with no constant term is called an odd linear function i.e., f(x) = ax, a ≠ 0
(b) There are two polynomial functions, satisfying the relation ; f(x) f(1/x) = f(x) + f(1/x)
They are :
(i) f(x) = xn + 1 and (ii) f(x) = 1 –xn, where n is positive integer.
(c) Domain of a polynomial function is R
(d) Range for odd degree polynomial is R whereas for even degree polynomial range is a subset of R.
(i) If f(x) + f(y) = f(xy) then f(x) = k log x
(ii) If f(x) · f(y) = f(x + y) then f(x) = akx
(iii) If f(x) + f(y) = f(x+ y) then f(x) = kx
Rohit Kumar (JEE 2012, AIR 79)
PLANCESS CONCEPTS
All polynomial functions are Algebraic but not the converse. A function that is not algebraic is called
Transcendental function.
Anvit Tawar (JEE 2012, AIR 9)
PLANCESS CONCEPTS
+ +
x
f(x)=a ,a 1
(0, 1) x
45o (0, 1) f(x)=a ,0 < a < 1
(1, 0) (1, 0)
x
x
y=
g(x)=logax, a 1
Figure 13.22
y=-x y=x
x
O
Figure 13.23
x
O y=sgn x
y=-1 if x > 0
Figure 13.24
Note:
graph of y =[x] y
(a) –1 ≤ x < 0⇒ [x] = –1 3
0≤ x <1⇒ [x] = 0 2
1 ≤ x <2⇒ [x] = 1 1
2≤ x <3⇒ [x] = 2 etc. x
-3 -2 -1 1 2 3
(b) f(x) = [x], domain is R and range is I. -1
1 1
(c) f(x) = domain is R – [0, 1) and range is n ∈ I − {0} -2
[x] n
-3
Properties of greatest integer function:
(a) [x] ≤x < [x] + 1 and x – 1 < [x] ≤ x, 0 ≤ x – [x] < 1. Figure 13.25
(a) Given graph of a function y = f(x) and we have to draw the graph of y = f(x – a) [means replacing x by x–a, a
> 0] then shift the entire graph through a distance a units in positive direction of x-axis.
y
y=sin(x-2) -2+2 1 +2 2+2
2 3+2
x
-4+2 0 2
-3+2 -+2 -1 + 2 +2
2
Figure 13.27
(b) Given graph of a function y = f(x), draw a graph of y = f(x + a) [means replacing x by x + a, a > 0]. Shift the
entire graph through in negative direction of x-axis.
y 3
-2
y=sin(x+2) 1 2
-
-2
2 0 x
-2
2-2
-3
-2 --2
+2 -1 -2
2 2
Figure 13.28
(c) Given graph of a function y = f(x), draw a graph of y = af(x) [means replacing y by y/a, a >0]. Then multiply all
the values by a on y-axis.
y
y=2sinx 2
-/2 3
0 x
-2 - 2
2
-2
Figure 13.29
M a them a ti cs | 13.21
(d) Given graph of a function y = f(x), draw a graph of y = f(ax) [means replacing x by ax, a >0]. Then divide all
the values by a on x-axis.
y
y=sin(2x)
- -3
4 2
x
-3 0
2
- - -1
4 2 4 2
Figure 13.30
(e) Given graph of a function y = f(x), draw the graph of y = f(x) + a [means replacing y by y – a, a >0]. Then shift
the entire graph in positive direction of y-axis.
y
y=(sin x)+2 3
2
-4+2 2
1
-3 -2 - 0 2 2
2
Figure 13.31
(f) Given graph of a function y = f(x), draw the graph of y = f(x) –a [means replacing y by y + a, a > 0]. Then shift
the entire graph in negative direction of y-axis.
y=(sinx)-2 y
-4 -3 -2 - 2 0 2 - 2 3 4
x
1
-2
-3
Figure 13.32
(g) Given graph of a function y = f(x), draw the graph of y = f(–x) [means replacing x by –x]. Then take the
reflection of the entire curve in y-axis.
y
y=sin (-x) 1
-
3 2 2 3
-3 -2 - -
3
-1
Figure 13.33
(h) Given graph of a function y = f(x), draw the graph of y = –f(x) [means replacing y by –y]. Then take the
reflection of the entire curve in x-axis.
1 3 . 2 2 | Relations and Functions
y
y= -sin x 1
-
3 2 2 3
-4 -3 -2 - -
2
-1
Figure 13.34
(i) Given graph of a function y = f(x), draw the graph of y = f(|x|) [means replacing x by |x|]
(a) Remove the portion of the curve, on left-hand side of y-axis.
(b) Take the reflection of right-hand size on the left-hand side.
y
y=sin x 1
-2 3
- x
-4 -3 0 2 4
-1
Figure 13.35
( j) Given graph of a function y = f(x), draw the graph of y = | f(x) |.The projection of the curve lying below x-axis
will go above the axis.
y
y = sin x 1
x
-3 -2 - 0 2 3
Figure 13.36
(k) Given graph of a function y = f(x), draw the graph of |y| = f(x) [means replacing y by |y|]. Then remove a portion
of the curve below x-axis and then take the reflection of the upper part on the lower part.
y = sin x 1
-2
x
-4 -3 - 0 2 3
-1
Figure 13.37
x2 + 2x − 3 (x + 3)(x − 1)
For example, f(x) = =
2
x + 2x − 8 (x + 4)(x − 2)
D = 0 at x = –4, 2.
y=2
x
-4 -3 2
Figure 13.38
(b) If numerator and denominator has a common factor (say x = a) it would mean removable discontinuity at x = a,
(x − 2)(x − 1)
E.g. f(x) = . Such a function will always be monotonic, i.e. either increasing or decreasing and
(x + 3)(x − 2)
removable discontinuity at x = 2. y
2 y=1
x
-3 -1 -3
Figure 13.39
(c) Compute point where the curve cuts both x-axis and y–axis by putting y = 0 and x = 0, respectively, and mark
points accordingly.
y
1
f(x) = x – 1
x-
)=
f(x
x = 0, y = –1
1
y = 0, x = 1 O x
Figure 13.40
dy
(d) Compute and find the intervals where f(x) increases or decreases and also where it has horizontal
tangent. dx
y
y = x2 – 3x + 2
3
x=
2 x
1
1
y=
4
Figure 13.41
(e) In regions where curves is monotonic, compute y if x →∞ or x → –∞ to find whether y is asymptotic or not.
x −1
f(x) =
x −3 y x=3
y=1
1/3
x
0
Figure 13.42
1 3 . 2 4 | Relations and Functions
(f) If denominator vanishes, say at x = a and (x – a) is not a common factor between numerator and denominator,
then examine Lim and Lim to find whether f approaches ∞ or –∞. Plot the graphs of the following function.
x →a− x →a+
x
Illustration 8: Draw the graph of functions f(x) =
nx
Sol: Calculate the domain of the given function. Then use the derivative of the given
function to trace the given curve.
1. nx − x·(1 / x) e
Domain of f(x) is x ∈ R (0, 1) ∪ (1, ∞) , f’(x) =
( nx)2 0 1 e
f’(x) = 0 at x = e
also as x approaches zero f(x) approaches zero from negative side and x approaches ∞
f(x) approaches +∞. Figure 13.43
From the graph we can observe the range of f(x) is (–∞, 0) ∪ [e, ∞).
7.1 Domain
(a) Expression under even root (i.e. square root, fourth root, etc.) ≥ 0 and denominator ≠ 0.
(b) If domain of y = f(x) and y = g(x) are D1 and D2 respectively, then the domain of f(x) + g(x) or
f(x) · g(x) is D1 ∩ D2.
f(x)
(c) Domain of is D1∩D2 – {g(x) = 0}.
g(x)
7.2 Range
(a) For the real valued function, real values of y for x ∈ domain of f are the range of the function.
Therefore, find domain of f and then impose restriction upon y, for the values of x in domain.
(b) If f is a continuous real valued function, then the range of function = [minimum f, maximum f].
PLANCESS CONCEPTS
Whenever we substitute the variable t for g(x), care should be taken that the corresponding condition
on t should be written immediately. Further analysis of the function will be according to the condition.
Illustration 9: Find domain and range of f(x) = cot −1 log 4 (5x2 − 8x + 4) . (JEE MAIN)
5
Sol: Use the definition of Domain and Range.
Consider the quadratic expression P(x) = 5x2 –8x + 4.
For the above quadratic, D < 0 ⇒ the expression always positive. /2
Figure 13.44
\ Range of log 4 (5x2 − 8x + 4) is (–∞, 1]
5
Now, draw the graph of cot–1(t) for t (–∞, 1]
π
From the graph we can observe that range of f(x) is , π
4
x2 − x + 1
Illustration 10: Draw a graph of f(x) = and also evaluate its domain and range. (JEE MAIN)
x2 + x + 1
Sol: Find derivative of the given function and use the techniques of curve tracing.
For drawing the graph of f(x) first find out those point where f’(x) = 0
2(x2 − 1) y
f’(x) =
(x2 + x + 1)2 3
f’(x) = 0 at x = –1, 1
(0, 1)
1 x
f(1) = ; f(–1) = 3 1/3
3 -1
0 1
Also when x approach ±∞, f approaches 1.
1 Figure 13.45
From the graph, domain is x ∈ R and range , 3
3
ax + b
Note: Graph of f(x) = is always monotonic.
cx + d
1
Illustration 11: If f(x) = sin–1 x2 + ln x − [x] + cot . Then find its domain and range. ( JEE ADVANCED)
2
1 + 2x
Sol: Follow the steps discussed above.
1 3 . 2 6 | Relations and Functions
Domain of function is {–1, 1} – {0}, because x – [x] = 0 for integral value of x; hence, middle term will not be defined.
Also [{f}] = 0, whenever f is meaningful.
1
cot −1 x = tan−1
Therefore value of f(x) = sin–1x2 + tan–1 (1 + 2 x2) x
when x > 0
Function is continuous and is even.
π
Least value of the function will occur when x → 0 and is
4.
π 3π 7π
Maximum value = Lim f(x) = sin–1 1 + tan–1 (1 + 2 ) = + =
x →±1 2 8 8
π 7π
Therefore, range of f(x) is , .
4 8
A. ALGEBRAIC FUNCTION
R, if n is odd
(i) xn, (n ∈ N) R = set of real numbers
R+ ∪ {0}, if n is even
1 R – {0}, if n is odd
(ii) , (n ∈ N) R – {0}
x n R+, if n is even
R, if n is odd R, if n is odd
(iii) x1/n (n ∈ N)
R+ ∪ {0}, if n is even R+ ∪ {0}, if n is even
B. TRIGONOMETRIC FUNCTION
π
(iii) tan x R – (2k + 1) ,k∈I R
2
π
(iv) sec x R – (2k + 1) ,k∈I (–∞, –1] ∪ [1, ∞)
2
(iii) tan–1 x R π π
− ,
2 2
D. EXPONENTIAL FUNCTION
E. LOGARITHIMIC FUNCTION
1
(ii) logxa =
loga x R+ – {1} R – {0}
(a > 0) (a ≠ 1)
G. FRACTIONAL FUNCTION
1
(ii) R–I (1, ∞)
{x}
1 3 . 2 8 | Relations and Functions
H. MODULUS FUNCTION
1 R – {0} R+
(ii)
|x|
I. SIGNUM FUNCTION
|x|
sgn(x) = R {–1, 0, 1}
x
J. CONSTANT FUNCTION
( x )
2
(xiii) f(x) = x2 ; g(x) = (N.I.)
x2n − 1
(xvii) f(x) = Limit ; g(x) = sgn(| x | –1) (I)
n→∞ x2n + 1
(xxiii) f(x) = (fog)(x); G(x) = (gof)(x) where f(x) = ex; g(x) = ln x (N.I.)
A B A B
a 1 a 1
b 2 OR b 2
c 3 c 3
Figure 13.46
Note:
(i) If there is increase or decrease of continuous function in whole domain, then f(x) is one–one.
(ii) If any line parallel to x-axis cuts the graph of the function only at one point, then the function is one–one.
Figure 13.47
Note:
(i) If any continuous function has at least one local maximum or local minimum, then f(x) is many–one. In other
words, if a line parallel to x-axis cuts the graph of the function at least at two points, then f is many–one.
(ii) If a function is one–one, it cannot be many–one and vice versa.
One–one + Many–one = Total number of mapping.
Figure 13.48
Figure 13.49
Note: If a function is onto, it cannot be into and vice versa. A polynomial of degree even and odd defined from R
→ R will always be into and onto, respectively.
Thus a function can be one of these four types:
M a them a ti cs | 13.31
Figure 13.50
A B
x1 y1
x2 y2 y4
x3 y3
Figure 13.51
A B
x1
y1
x2
x3 y2
Figure 13.52
Figure 13.53
Illustration 12: Let A be a finite set. If f: A → A is an onto function, then show that f is one–one. (JEE MAIN)
Illustration 13: Let C and R denote the set of all complex and all real numbers respectively. Then show that f: C →
R given by f(z) = | z |, for all z ∈ C is neither one–one nor onto. (JEE MAIN)
Sol: Using two complex conjugate numbers, we can prove that the given function is not one-one. For the second
part, use the fact that modulus of a number cannot be negative.
Injectivity: We find that z1 = 1 – i and z2 = 1 + i are two distinct complex numbers such that | z1 | = | z2 |, i.e. z1¹z2
but f(z1) = f(z2).
It is clear that different elements may have the same image. So, f is not an injection.
Surjectivity: f is not a surjection, because negative real numbers in R do not have their pre-image in C. In other
words, for every negative real number there is no complex number z ∈ C such that f(z) = | z | = a. So, f is not a
surjection.
Illustration 14: For function f: A →A, fof = f. Prove that f is one–one if and only if f is onto. (JEE MAIN)
Sol: Starting with the relation given in the question fof = f and use the definition of one-one and onto function.
Suppose f is one–one.
Then, (fof) (x) = f(x)
f(f(x)) = f(x)
f(x) = x (f is one–one)
Thus, f(x) = x, ∀ x ∈ A.
for each x A, there is an x ∈ A such that f(x) = x
∴ f is onto.
Now suppose f is onto.
Then, for each y ∈ A, there is an x ∈ A such that f(x) = y.
Let x1, x2 ∈ A and let f(x1) = f(x2).
Then there exist y1, y2 ∈ A such that x1 = f(y1)·x2 = f(y2)
f(f(y1)) = f(f(y2))
(fof)(y1) = (fof)(y2)
f(y1) = f(y2) (fof = f)
x1 = x2
∴ f is one–one.
Illustration 15: Show that the function f: R → R given by f(x) = x3 + x is a bijection. (JEE ADVANCED)
Sol: Consider two elements x and y in the domain and prove that f(x) = f(y) implies x = y. Use the definition of onto
to prove that the function f is a bijection.
Injectivity: Let x, y ∈ R such that
f(x) = f(y)
⇒ x3 + x = y3 + y
⇒ x3 – y3 + (x – y) = 0
⇒ (x – y) (x2 + xy + y2 + 1) = 0
⇒ x – y = 0 [Qx2 + xy + y2≥ 0 for all x, y ∈R. x2 + xy + y2 + 1 ≥ 1 for all x,y ∈ R]
⇒ x=y
M a them a ti cs | 13.33
n + 1, if n is even
Illustration 16: Let f: N ∪ {0} → N ∪ {0} be defined by f(n) = . Show that f is a bijective function.
n-1, if n is odd (JEE MAIN)
27
Sol: Divide the solution in three cases. (0, 17)
Case I – When both the numbers are even, Case II – When both the numbers are odd & Case
III – When one is even and other is odd. -1 -3
Let f(n) = f(m).
-37
Case I : both n, m are even.
Then, n + 1 = m + 1. So, n = m. Figure 13.54
Sol: Find the derivative of given function and understand the nature of the curve. Also find the values of f(x) at
some particular values of x to trace the given curve.
Domain of the function is x ∈ R
Here, f’(x) = 6x2 – 12x – 18 = 6(x2 – 2x – 3) = 6(x + 1) (x – 3)
f’(x) = 0 at x = –1, 3
f(∞) = ∞
f(–∞) = −∞
f(0) = 17, f(–1) = 27, f(3) = –37
From the graph we can say that f is many one onto function.
1 3 . 3 4 | Relations and Functions
Illustration 18: Let f: R → R, g: R → R be defined by f(x) = x2 + 3x + 1, g(x) = 2x – 3. Find fog and gof.(JEE MAIN)
1
Illustration 19: If (x) = and g(x) = 0 are two real functions, show that fog is not defined. (JEE MAIN)
x2
Sol: Find the domain of fog(x).
We have,
Domain (f) = R – {0}, Range (f) = R – {0}
Domain (g) = R and Range (g) = {0}
Clearly, Range (g) ⊄ Domain (f)
∴ Domain (fog) = {x: x ∈ Domain (g) and g(x) ∈ Domain (f)}
⇒ Domain (fog) = {x: x ∈ R and g(x) ∈ Domain (f)}
⇒ Doman (fog) = f
[∵ g(x) = 0 ∉ Domain (f) for any x ∈ R]
Hence, fog is not defined.
1 1 2x + 1 1 3
Illustration 20: If f(x) = , x ≠ − , then show that, f(f(x)) = , provided that x ≠ − , − . (JEE MAIN)
2x + 1 2 2x + 3 2 2
Sol: Check the domain of f(x) and use the concept of composite functions.
1
We have, f(x) =
2x + 1
1
Clearly, domain (f) = R – −
2
1 1 1−y
Let, y = ⇒2x + 1 = ⇒x =
2x + 1 y 2y
1
Since x is a real number distinct from − , y can take any non-zero real value.
2
So, Range ( f ) = R – {0}
We observe that range
1
( f ) = R – {0} ⊄ domain ( f ) = R –
2
∴ Domain (fof) = {x: x ∈ domain ( f ) and f(x) ∈ Domain ( f )}
⇒
Domain (fof) = x : x ∈ R − −
{ }1
2 { }
1
and f(x) ∈ R − −
2
1 1
⇒ Domain (fof) = x : x ≠ − and f(x) ≠ −
2 2
1 1 1
⇒ Domain (fof) = x : x ≠ − and ≠−
2 2x + 1 2
1 3
⇒ Domain (fof) = x : x ≠ − and x ≠ −
2 2
1 3
R − − , −
2 2
1 1 2x + 1
Also, fof(x) = f(f(x)) = f = =
2x + 1 2 (1 / (2x + 1) ) + 1 2x + 3
1 3 2x + 1
Thus, fof : R – − , − → R is defined by fof(x) =
2 2 2x + 3
1 3 . 3 6 | Relations and Functions
2x + 1 1 3
Hence, f(f(x)) = for all x ∈ R. x ≠− , − .
2x + 3 2 2
Illustration 21: If f(x) = log100x 2log10 x + 2 and g(x) = {x}. If the function (fog)(x) exists then find the
range of g(x). −x (JEE ADVANCED)
Sol: Find the domain of f(x) and use the given information that fog(x) exists.
To define f(x), the following condition must hold good:
1 g f
(i) 100 x > 0 and 100x ≠ 1⇒ x ≠
100
1 1
(ii) x > 0 and log10x + 1 < 0 ⇒ 0 < x < and x ≠
10 100
1 1 1 fog
∴ Domain of f(x) is 0, ∪ ,
100 100 10 .
Figure 13.55
Here, g(x) = {x}, range of g(x) is [0, 1).
1 1 1
∴ Range of g(x) is 0, ∪ ,
100 100 10 .
Illustration 22: Consider functions f and g such that composite gof is defined and is one–one. Should f and g
necessarily be one–one? (JEE MAIN)
Note that the graphs of f and g are the mirror images of each other in the line y = x. As shown in the figure given
below a point (x’, y’) corresponding to y = x2(x ≥ 0) changes to (y’ x’) corresponding to y = x , the changed form
of x = y.
y y y
x= y x= y
2
y=x
x
y=
y=
2
y=x
0 0 0
x x x
(i) (ii) (iii)
Figure 13.56
= a [by (i)]
= (gof) (c)
–1
[by (iii)]
\ (gof)–1 = f–1og–1, which proves the theorem.
1 3 . 3 8 | Relations and Functions
PLANCESS CONCEPTS
In the line y = x, the graphs of f and g are the mirror images of each other. As shown in the following
figure, a point (x’, y’) corresponding to y = ln x(x >0) changes to (y’, x’) corresponding to y = ex, the
changed form of x = ey.
The inverse of a bijection is also a bijection.
If f and g are two bijections f: A → B, g: B → C, then inverse of gof exists and (gof)–1 = f–1og–1.
x
e
y y= =x
x
y
n
y= n x
y=
x
y=e
x x x
1 1
Figure 13.57
x, x <1
2
Illustration 23: Find inverse of the function f(x) = x , 1 ≤ x ≤ 4 (JEE MAIN)
8 x , x>4
Sol: Put f(x) = y and solve for x.
Graph of f(x)
8 x
y if y < 1 16
B
Using the above graph f (y) = x = y if 1 ≤ y ≤ 16
–1
A 2
1 x
2 x
y if y > 16
164
0 1 4
x if x < 1
or f–1(x) = x if 1 ≤ x ≤ 16 . Figure 13.58
2
x if x > 16
16
Illustration 24: Let f: N → R be a function defined as f(x) = 4x2 + 12x + 15. Show that f : N → S, where S is the range
of f, is invertible. Find the inverse of f. (JEE MAIN)
Sol: Put 4x2 + 12x + 15 = y and solve for x. Then put f-1(y) in place of x.
Let y be an arbitrary element of range f. Then y = 4x2 + 12x + 15, for some x in N, which implies that y = (2x + 3)2
( y − 6 − 3)
+ 6. This gives x = , as y ≥ 6.
2
( y − 6 − 3)
Let us define g: S → N by g(y) = .
2
Hence, gof = IN and fog = IS. This implies that f is invertible with f–1 = g.
2x − 5
Illustration 25: For the function f: R – {4} → R – {–2} : f(x) = . Find
4−x
(a) zero’s of f(x) (b) range of f(x)
(c) intervals of monotonocity (d) f–1(x)
(e) local maxima and minima if any (f) interval when f(x) is concave upward and concave downward
2
(g) asymptotes (h) ∫ f(x) dx
1
Sol:
(a) 5/2 (b) (–∞, –2) ∪ (–2, ∞)
4x + 5
(c) ↑ in its domain i.e., (–∞, 4) ∪ (4, ∞) (d) f–1(x) =
x+2
(e) no, (f) (–∞, 4) upwards and (4,∞) downwards
2
(g) (g) y = –2 (h) − 2 + 3l n
3
(i) both one-one and onto
y
0 5/2
x
( j) -1 4
-5/4
y=-2
-2
-
Figure 13.59
Sol: Check whether the given function is one-one onto. Put 3x + 2 = y and solve for x. Then put f-1(y) in place of x.
∀ x1, x2∈ A, f(x1) = f(x2) ⇒ 3x1 + 2 = 3x2 + 2 ⇒ x1 = x2
∴
f: R → R is one–one.
y −2
Now let y ∈ R. Then y = 3x + 2 ⇒ x =
3
y −2
Hence, for every y ∈ R, there is a corresponding x = ∈ R such that y = f(x). Hence, the range of f is R and so
f is onto, and the inverse of f exists. 3
1 3 . 4 0 | Relations and Functions
2x + 1
1. n x + 1 + x2 1. x 1. 2x3 – x + 1k
x
2 −1
M a them a ti cs | 13.41
1−x
2. n 2. 3
(1 − x)2 + 3 (1 + x)2 2. sin x + cos x
1+x
3. x sin2x – x3 3. constant
4. 1 + x + x2 − 1 + x + x2 4. x2 – | x |
1 + 2Kx (1 + 2x )2
5. 5.
Kx
1−2 2x
PLANCESS CONCEPTS
(a) f(x) – f(–x) = 0 ⇒ f(x) is even and f(x) + f(–x) = 0 ⇒ f(x) is odd.
(b) A function may neither be odd nor even.
(c) Inverse of an even function is not defined and an even function cannot be strictly monotonic.
(d) Every even function is symmetric about the y-axis and every odd function is symmetric about the
origin.
(e) Every function can be expressed as the sum of an even and an odd function.
E.g.,
2x + 2–x 2x – 2–x
f(x) = f(x) + f(–x) + f(x) – f(–x) 2= +
x
2 2 2 2
EVEN ODD EVEN ODD
(f) The only function which is defined on the entire number line and is even and odd at the same time
is f(x) = 0.
(g) If f and g are either both even or odd, then the function f · g will be even. But if any one of them is
odd then f · g will be odd.
f(x) g(x) f(x)+g(x) f(x) – g(x) f(x) . f(x) / (gof) / (fog) (x)
g(x) g(x) (x)
odd odd Odd odd even even odd odd
even even Even even even even even even
odd even neither odd nor even neither odd nor even odd odd even even
even odd neither odd nor even neither odd nor even odd odd even even
PLANCESS CONCEPTS
Illustration 27: Which of the following function(s) is(are) bounded on the intervals as indicated
1
x −1 1
(A) f(x) = 2 on (0, 1) (B) g(x) = x cos on (–∞,∞)
x
(C) h(x) = xe–x on(0, ∞) (D) l(x) = arc tan 2x on (–∞,∞)
Sol: Check for the continuity of the given functions. If the function is continuous then to find the value of f(x) at
the boundary points.
1 1
1 1/e
Limit f(x) Limit
(A) = = 2h−1 ; Limit
= f(x) Limit
= 2 −h 0
+
x →0 h→0 2 x →I− h→0 1
1 Figure 13.61
⇒ f(x) ∈ 0, ⇒ bounded
2
M a them a ti cs | 13.43
x
(C) Limit
= xe− x Limit
= he−h 0; Limit
= x e− x Limit
= 0
h→0 h→0 x →∞ x →∞ ex
x ex − xex 1
⇒ also y = ⇒ y’ = ex(1–x)⇒ h(x) = 0,
e
x 2x
e e
FORMULAE SHEET
Polynomial function R R
Identity function x R R
R0 R0
1
Reciprocal function
x
x3 , x x R R
R {-1,0,1}
x
Signum function
x
X+ x R R + ∪ {x}
x- x R R − ∪ {x}
x-{x} R [0,1]
(0, ∞) [0,∞]
x
ax(exponential function) R R+
sin–1x (-1,1]
−π π
,
2 2
tan–1x R
−π π
,
2 2
cot–1x R (0, π )
sec–1x R-(-1,1)
π
[0, π ]-
2
cosec–1x R-(-1,1)
−π π
, -{0}
2 2
1 1
(x)
i.e. f=
2
f ( x ) + f ( −x ) + f ( x ) − f ( −x ) =even function + odd function.
2
Increasing function: A function f(x) is an increasing function in the domain, D if the value of the function does not
decrease by increasing the value of x.
Decreasing function: A function f(x) is a decreasing function in the domain, D if the value of function does
not increase by increasing the value of x.
M a them a ti cs | 13.45
Periodic function: Function f(x) will be periodic if a +ve real number T exists such that
f ( x +=
T ) f ( x ) , ∀× ∈ Domain.
There may be infinitely many such real number T which satisfies the above equality. Such a least +ve number
T is called period of f(x).
(i) If a function f(x) has period T, then period of f(xn+a)=T/n and period of (x/n+a)=nT.
(ii) If the period of f(x) is T1 & g(x) has T2 then the period of f(x) ± g(x) will be L.C.M. of T1& T2 provided it satis-
fies definition of periodic function.
(iii) If period of f(x) & f(x) are same T, then the period of af(x)+bg(x) will also be T.
Function Period
sin x, cos x 2π
sec x, cosec x
tan x, cot x π
sin (x/3) 6π
tan 4x π /4
cos 2 π x 1
cos x π
sin4x+cos4x π /2
x−π
2 cos 6π
3
sin3 x + cos3x 2 π /3
Sin3 x +cos4x 2π
sinx
2π
sin5x
tan2 x − cot2 x π
x-[x] 1
[x] 1
1 3 . 4 6 | Relations and Functions
Solved Examples
Example 3: If A = {x: x = 2n + 1, n ∈ Z} and B = {x: x = Sol: Prove that R satisfies the conditions for reflexive,
2n, n ∈ Z}, then find A ∪ B. symmetric and transitive relation.
R is reflexive, since every triangle is congruent to itself.
Sol: Write the two sets and then take union. Further (T, T1) ∈ R2.
A ∪ B = {x: x is an odd integer} ∪ {x: x is an even integer} ⇒ T1 is congruent to T2 ⇔ T2 is congruent to T1. (T2, T1)
= {x: x is an integer} = Z. ∈ R, Hence R is symmetric.
Moreover, (T1, T2), (T2, T3) ∈ R.
Example 4: If A = {x: x = 3n, n ∈ Z} andB = {x: x = 4n, n
∈ Z}, then find A ∩ B. ⇒ T1 is congruent to T2 and T2 is congruent to T.
⇒ T1 is congruent to T3 ⇒ (T1, T3) ∈ R. Therefore, R is
Sol: Clearly, set A is a multiple of 3 and the other set is a
an equivalence relation.
multiple of 4. Hence, the intersection of the two would
be set having multiples of 12.
Example 8: Let L be the set of all lines in a plane
We have and R be the relation in L defined as R = {(L1, L2): L1
x ∈ A ∩ B ⇔ x = 3n, n ∈ Z and x = 4n, n ∈ Z is perpendicular to L2}. Show that R is symmetric but
neither reflexive nor transitive.
⇔ x is a multiple of 3 and x is a multiple of 4.
⇒ x is a multiple of 3 and 4 both. Sol: Use the definition of Symmetric relation.
⇔ x is a multiple of 12 ⇔ x = 12n, n ∈ Z. R is not reflexive as the line L1 cannot be perpendicular
to itself, i.e. (L1, L1) ∉ R. R is symmetric as(L1, L2) ∈ R.
Hence, A ∩ B = {x: x = 12n, n ∈ Z}.
⇒ L1 is perpendicular to L2.
Example 5: If A = {2, 3, 4, 5, 6, 7} andB = {3, 5, 7, 9, 11, ⇒ L2 is perpendicular to L1.
13}, then find A – B andB – A.
⇒ (L2, L1) ∈ R.
M a them a ti cs | 13.47
Example 15: Which of the following functions are gof(x) = (x + 1)2, –2 ≤ x < 1
even/odd ?
= (x + 1)2 – 2 ≤ x ≤ 1
(i) f(x) = sin x + cos x
Sol: A function is even if f(-x) = f(x) and odd if f(-x) = -f(x). Sol: Take (2 + (x – 3)3)1/3 = y and solve for x. Then put
f-1(y) in place of x.
(i) f(x) sin x + cos x
y = [(2 + (x – 3)3]1/3
f(–x) = – sin x + cos x
y3 = 2 + (x – 3)3 ; (x – 3)3 = y3 – 2
f(–x) ¹f(x) ∀ x ∈ R;
x – 3 = (y3 – 2)1/3 ; x = 3 + (y3 – 2)1/3
hence, f is not an even function.
g(y) = x = 3 + (y3 – 2)1/3 is the inverse function.
f( −x) ≠ −f(x) ∀ x ∈ R ; hence f is not an odd function.
\ f is neither even nor odd. Example 19: Let f(x) = x2 + x be defined on the interval
(ii) f(x) = 1 + x + x2 − 1 − x + x2 [0, 2]. Find the odd and even extensions of f(x) in the
interval [–2, 2].
f(–x) = 1 − x + x2 − 1 + x + x2
=–f(x) Sol: The definition is given for 0 ≤ x ≤ 2, so in order to
find the even and odd extension, define the function
∴ f is an odd function. for -2 ≤ x < 0.
2 + x : x ≥ 0 Odd extension
Example 16: Let f(x) = .
2 − x : x < 0 f(x), 0≤x≤2 x2 + x, 0 ≤ x ≤ 2
Find f of(x) f(x) = = 2
− f( − x) −2≤x<0 −x + x, −2 ≤ x < 0
Sol: Use the concept of composite functions.
Even extension
f: R → R
f(x), 0 ≤ x ≤ 2 x2 + x, 0 ≤ x ≤ 2
fof(x) = f(f(x)) f(x) = = 2
f( −x), −2 ≤ x < 0 x − x, −2 ≤ x < 0
Let x ≥ 0; fof(x) = f(2 + x) = 2 + (2 + x) as 2 + x ≥ 0
\ fof(x) = 4 + x when x ≥ 0
Example 20: The value of n ∈ I for which the function
Let x < 0; fof(x) = f(2 – x) = 2 + (2 – x) sinnx
f(x) = has 4p as its period is
(as 2 –x ≥ 0) = 4 – x sin(x / n)
(A) 2 (B) 3 (C) 5 (D) 4
4 + x if x ≥ 0
∴ fof(x) =
4 − x if x < 0 Sol: For n = 2,
sin2x
We have = 4(cos x/2) cos x.
Example 17: Let, sin(x / 2)
The period of cos x is 2p and that of cos(x/2) is 4p.
f(x) = x + 1,x ≤ 1= 2x + 1, 1 < x ≤ 2
sin2x
g(x) = x2, –1 £x < 2 = x + 2, 2 ≤ x ≤ 3 Hence the period of is 4p.
sin(x / 2)
Find fog and gof.
sin3x sin5x sin 4x
Also, the period of , and
Sol: Similar to the previous example. sin(x / 3) sin(x / 5) sin(x / 4)
cannot be 4p.
f{g(x)} = g(x) + 1, g(x) ≤ 1
= 2g(x) + 1, 1 < g(x) ≤ 2
Example 21: Find the range of the following functionf(x)
⇒ f{g(x)} = x2 + 1, –1 ≤ x ≤ 1
3
= 2x2 + 1, 1 < x ≤ 2 ; g{f(x)} = {f(x)}2, =
2 − x2
– 1 £f(x) < 2= f(x) + 2, 2 £f(x) £3
M a them a ti cs | 13.49
Sol: First find the domain of the given function and JEE Advanced/Boards
then proceed to find the values f(x) can take..
3 Example 1: Let A = {1, 2, 3}. Then show that the
Let y = = f(x) … (i)
2−x 2 number of relations containing (1, 2) and (2, 3) which
are reflexive and transitive, but not symmetric, is four.
The function y is not defined for x = ± 2
2y − 3 Sol: Try all the possibilities and prove that the number
From (i), x2 =
y of such relations can be four.
2y − 3
since for real x, x2≥ 0, We have 0 The smallest relation R1 containing (1, 2) and (2, 3)
y
≥ which is reflexive and transitive, but not symmetric, is
\ y ≥ 3/2 or y < 0 (Note that y ≠ 0)
{(1, 1), (2, 2), (3, 3), (1, 2), (2, 3), (1, 3)}. Now, if we add
Hence the range of the function is the pair (2, 1) to R1 to get R2, then the relation R2 will
[–∞, 0] ∪ [3/2, ∞) be reflexive and transitive, but not symmetric. Similarly,
we can obtain R3 and R4 by adding (3, 2) and (3, 1),
respectively, to R1 to get the desired relations. However,
Example 22: Knowing the graph of y = f(x) draw we cannot add any two pairs out of (2, 1), (3, 2) and
f(x)+ | f(x) | f(x)− | f(x) | (3, 1) to R1 at a time, as by doing so, we will be forced
y= and y =
2 2 to add the remaining third pair in order to maintain
transitivity and in the process, the relation will become
Sol: Use the basics of curve tracing. symmetric also which is not required. Thus, the total
Let graph number of desired relations is four.
y
y= f(x) Example 2: Show that the number of equivalence
y
y=f(x) relation in the set {1, 2, 3} containing (1, 2) and (2, 1)
is two.
x x
O O
Sol: Similar to the previous one.
y
y y- f(x)+ f(x) The smallest equivalence relation R1 containing (1, 2)
2 and (2, 1) is {(1, 1), (2, 2), (3, 3), (1, 2), (2, 1)}. Now we are
x
O left with only four pairs namely (2, 3), (3, 2), (1, 3) and (3,
x f(x)- f(x)
O y= 1). If we add any one, say (2, 3) to R1, then for symmetry
2
we must add (3, 2) also and now for transitivity we are
forced to add (1, 3) and (3, 1). Thus, the only equivalence
Example 23: Draw the following graphs :
relation bigger than R1 is the universal relation. This
(i) | y | = cos x (ii) | y | = sin x shows that the total number of equivalence relations
containing (1, 2) and (2, 1) is two.
Sol: Similar to the previous example.
(i) | y | = cos x Example 3: Let R be a relation on the set A of ordered
pairs of positive integers defined by (x, y) R (u, v) if and
y only if xv = yu. Show that R is an equivalence relation.
a a a h(x) = 0, if x ≤ 0= x, if x ≥ 0,
⇒ xv = yu ⇒ xv b = yu
u u v u
then find the composite function ho(fog) and determine
⇒ xb = ya and hence (x, y) R (a, b). Thus, R is transitive. whether the function fog is invertible and the function
Thus, R is an equivalence relation. h is identity function.
π
So, f(x) is a surjection. = 2y– 3 = sin π −
4
Hence, f is bijection. Consequently, it is invertible.
π
f(x) = f–1(x) ⇒ f(x) = x But –1 ≤ sin π − 1
4
≤
(x + 1)2 – 1 = x ⇒ x = 0, –1 ∴ –1 ≤ 2 – 3 ≤ 1
y
⇒ 2 ≤ 2y ≤ 4 ⇒ 2 ≤ 2y ≤ 22
Example 8: Show that the function
Hence y ∈ [1, 2]
f: R → R defined by f(x) = 3x3 + 5 for all x ∈ R is a
bijection. Hence range of f(x) is [1, 2]
Sol: Similar to the previous example. Example 10: Find the period of the following functions
Injectivity: Let x, y be any two elements of R (domain). (i) f(x) = tan 2x
Then, f(x) = f(y)
(ii) f(x) = sin4x + cos4x
⇒ 3x3 + 5 = 3y3 + 5; x3 = y3⇒ x = y
(iii) f(x) = x – [x] + | cos px |
Thus, f(x) = f(y)
+ | cos 2 px | + ….. + | cos npx |
⇒ x = y for all x, y ∈ R. so, f is an injective map. Surjectivity:
Let y be an arbitrary element of R(co-domain). Sol: Proceed according to the section 15.5.
Then, f(x) = y (i) f(x) = tan 2x has period π /2as tan x has period π.
1/3
y −5 y −5
⇒3x3 + 5 = y ⇒ x3 = ;x= (ii) f(x) = sin4x + cos4x
3 3
= (sin2x + cos2x)2 – 2 sin2x cos2x
Thus we find that for all y ∈ R (co-domain) there exists
y −5
1/3 1 1 3 1
=1– sin2 2x= 1 – (1 – cos 4x) = – cos 4x
x = ∈ R (domain) such that f(x) = 2 4 4 4
3
3 Since cos x has period 2π,
y − 5 1/3 y − 5 1/3
f
= 3 +5 2π π
3 3 ∴ cos 4x has period =
4 2.
=y–5+5=y 3 1 π
f(x) = + cos 4x has period
∴ 4 4 2.
This shows that every element in the co-domain has its
pre-image in the domain. So, f is a surjection. Hence, f (iii) x – [x] has period 1
is a bijection. | cos x | has period p
π
Example 9: Find the range of the following functions: | cos px | has period =1
π
sinx − cos x + 3 2 π 1
f(x) = log2 | cos 2px | has period =
2 2π 2
π 1
| cos npx | has period =
Sol: Put f(x) = y and simplify. Then use the range of the nπ n
trigonometric function to find the answer.
1 1 1
sinx − cos x + 3 2 l.c.m. 1, 1, , , ...., = 1
⇒ f(x) = log2 2 3 n
2
∴ f(x) has period 1.
π
= log2 sin π − + 3 = y(let)
4 Example 11: Find the domain of the following functions
π
∵ 2y = sin π − + 3 (i) f(x) = x −1 + 6 − x
4
(ii) f(x) = log1/2 (x2 + 4x + 3)
1 3 . 5 2 | Relations and Functions
1 Here, x2 – 3x + 4 = x ⇒ (x – 2)2 = 0
(iii) f(x) =
| x | −x x = 2 is the solution for the equation.
Sol: Use the fact that the quantity inside the square Example 13: Draw the graph of following function and
root should be positive. For (ii), use the condition for also find their domain and range.
logarithm to be defined.
(x + 2)(x − 1)
(i) x − 1 is defined only if x – 1 ≥ 0 (i) f(x) =
x(x + 1)
6 − x is defined only if 6 – x ≥ 0
x2 − 5x + 4
∴ f(x) is defined. (ii) f(x) =
x2 + 2x − 3
∀ x ∈ {x : x – 1 ≥ 0} ∩ {x : 6 – x ≥ 0}
Domain of f is = [1, 6] Sol: Use the method of curve tracing.
(x + 2)(x − 1)
(ii) f(x) = log1/2 (x2 + 4x + 3) (i) f(x) = , domain of f(x) is
x(x + 1)
f is defined if x2 + 4x + 3 > 0
x ∈ R – {–2, 1}
i.e. if (x + 1) (x + 3) > 0
2(2x + 1)
f’(x) =
i.e. if x > – 1 or x < –3 [x(x + 1)]2
Domain of f is (–∞, –3) ∪ (–1, ∞)
f’(x) = 0 at x = –1/2
(iii) f is defined if | x | – x > 0
Here, f(–∞) = 1– = f(+∞)
If x ≥ 0 then | x | = x
f(–1–) = –∞ =f(0–)f(–1+) = +∞ = f(0+)
If x < 0 then | x | = – x > 0 and | x | > x.
Range is (–∞, 1) ∪ (9, ∞)
∴Domain of f = {x: x < 0}
x2 − 5x + 4 (x − 4)(x − 1)
(ii) f(x) = =
Example 12: A function 2
x + 2x − 3 (x + 3)(x − 1)
We can plot the graph of f(x) for the above domain as f(–3–) = +¥ ⇒ f(–3+) = –∞
3
f(4) = 0 Range is R – − , 1
4
y
7
(0, 9)
4 3 (0, 1) y=1
x
2
-1
-2 -1/2 0 (1, 0)
For solving f(x) = f–1(x), we can solve f(x) = x or f–1(x) = x -
Any one of the above equation can be solved depending
on the fact that easiest equation is given priority.
M a them a ti cs | 13.53
JEE Main/Boards
Exercise 1 1 + x
Q.3 If f(x) = log , prove that
1 − x
Sets and Relations
2x
f = 2f(x).
1 + x2
Q.1 If R be a relation and N defined by x + 2y = 8 then
find the domain of R. x2 + 2x + 1
Q.4 Find the domain of the function f(x) =
x2 − 8x + 12
Q.2 R is a relation from {11, 12, 13} to {8, 10, 12} defined
by y = x – 3 then find R–1. Q.5 Find the domain of definition of the function
1
y= + x+2
Q.3 Let A be the set of all students of a boy’s school. log10 (1 − x)
Let relation R in set A is given by R = {(a, b) ∈ A × A is
a sister of b}. Can we say that R is an empty relation? x
Give reason. Q. 6 Find the range of the function y = .
1 + x2
Q.8 Given a relation R = {(yellow, black), (cat, dog), (red, Q.10 Let f: R → R is defined by f(x) = x and g : R → R is
green)}. White R–1. defined by g(x) = | x |. Find
(i) f + g (ii) f – g
Q.9 Let A = {1, 3, 5}, B = {9, 11} and let R = {(a, b)∈ A ×
(iii) f · g (iv) a f, a ∈ R
B: a – b is odd}. Write the relation r.
(v) f/g
Q.10 Let A = {a, b, c} and relation R in the set A be given
by R = {(a, c), (c, a)}. Is relation R symmetric? Explain. Q.11 Let f be the exponential function and g be
the logarithmic function defined by f(x) = ex and
Functions g(x) = logex. Find
2 tanx π (i) (f + g) (1) (ii) (f – g) (1) (iii) (f · g) (1)
Q.1 If f(x) = , then find f .
4
2
1 + tan x
Q.12 If f(x) = cos[p2]x + cos[–p2]x, where [x] denotes the
|x|
Q.2 If f(x) = , x ≠ 0, prove that π
x integral part of x, write the value of f .
| f(α) – f(–α)| = 2 α, a ≠ 0. 2
1 3 . 5 4 | Relations and Functions
Q.13 ({x}, represents fractional part function) (i) f(x) = x2 (ii) g(x) = | x |
(i) Domain of the function 1
(iii) h(x) = (iv) u(x) = 4 − x2
1 3 − x2
f(x) = ln (1 – {x}) + sinx + + 4 − x2 is _______.
2 Q.22 Consider the following rules:
(ii) Range of the function cos(2 sin x) is ______.
(i) f: R → R : f(x) = logex
(iii) Period of the function
(ii) g: R → R : g(x) = x
πx 1
f(x) = sin + {x} + tan2(px) is _______. (iii) h: A → R : h(x) = , where A = R – {–2, 2}
3
2
x −4
Which of them are functions? Also find their range, if
Q.14 Let A = {1, 2, 3}, B = {3, 6, 9, 10}. Which of the they are function.
following relations are functions from A to B? Also find
their range if they are function.
x2 − 4
f = {(1, 9), (2, 3), (3, 10)} Q.23 Let f: R – {2} → R be defined by f(x) =
x−2
g = {(1, 6), (2, 10), (3, 9), (1, 3)} and g: R → R be defined by g(x) = x + 2. Find whether
h = {(2, 6), (3, 9)} f = g or not.
u = {(x, y): y = 3x, x ∈ A}
Q.24 Let f = {(1, 1), (2, 3), (0, –1), (–1, –3)} be a linear
Q.15 Let A = {a, b, c, d}. Examine which of the following function from Z into Z and g(x) = x. Find f + g.
relation is a function on A?
Q.25 Find f + g, f – g, f · g, f/g and a f (a ∈ R) if
(i) f = {(a, a), (b, c), (c, d), (d, c)}
1
(ii) g = {(a, c), (b, d), (b, c)} (i) f(x) = , x ≠ –4 and g(x) = (x + 4)3
x+4
(iii) h = {(b, c), (d, a), (a, a)}
(ii) f(x) = cos x, g(x) = ex.
Q.6 Let A = {2, 3, 4, 5} and let R = {(2, 2), (3, 3), (4, 4), (5, (A) Reflexive and transitive
5), (2, 3), (3, 2), (3, 5), (5, 3)} be a relation in A. Then R is (B) Reflexive and symmetric
(A) Reflexive and transitive (C) An equivalence relation
(B) Reflexive and symmetric (D) None of these
(C) Reflexive and anti-symmetric
(D) None of these
1 3 . 5 6 | Relations and Functions
Q.14 Let L be the set of all straight lines in the xy-plane. (B)
y
Two lines l1 and l2 are said to be related by the relation
R if l1 is parallel to l2. Then the relation R is
(A) Reflexive (B) Symmetric (B) a x
O b
(C) Transitive (D) Equivalence
Q.15 Given the relation R = {(2, 3), (3, 4)} on the set
{2, 3, 4}. The number of minimum number of ordered
pair to be added to R so that R is reflexive and symmetric (C) y
(A) 4 (B) 5 (C) 7 (D) 6
x2 − y 2 x2 + y 2
(A) (B)
4 4
(C) 4xy (D) None of these 1 | sinx | sinx
Q.4 Period of the function f(x) = + is
2 cos x | cos x |
Q.2 The set of values of ‘a’ for which f: R → R f(x) = ax + (A)π /2 (B) p
cos x is bijective is
(C) 2 p (D) 4p
(A) [–1, 1] (B) R – {–1, 1}
(C) R – (–1, 1) (D) R – {0} Q.5 Let f : R → R be a function defined by
2x2 − x + 5
Q.3 The graph of function f(x) is as shown, adjacently f(x) = then f is
7x2 + 2x + 10
(A) One–one but not onto
y=f(x) (B) Onto but not one–one
(C) Onto as well as one–one
a b
(D) Neither onto nor one–one
1 1
1 Q.6 If f(x) = cos π2 x + sin π2 x, [x] denoting the
Then the graph of is 2 2
f(| x |)
greatest integer function, then
(A) y
π 1
(A) f(0) = 0 (B) f =
3 4
(A) a b π
O
x (C) f = 1 (D) f(π) = 0
2
M a them a ti cs | 13.57
Q.1 Let f(x) = | x – 1 |. Then, (1983) Q.7 If f(x) = 3x – 5, then f–1(x) (1998)
(A) f(x ) = [f(x)]
2 2
1
(A) Is given by
(B) f(x + y) = f(x) + f(y) 3x − 5
x +5
(C) f(| x |) = | f(x) | (B) Is given by
3
(D) None of the above
(C) Does not exist because f is not one–one
1 1 x (D) Does not exist because f is not onto
Q.2 If f(x) = cos(log x), then f(x).f(y) – f + f(xy)
2 2 y
has the value (1983) Q.8 If the function f : [1, ∞) → [1, ∞) is defined by
1 f(x) = 2x(x–1), then f–1(x) is (1999)
(A) –1 (B)
2
x(x −1)
1 1
(C) –2 (D) None of these (A) (B) (1 + 1 + 4log2 x )
2 2
Each of these questions also have four alternative (A) ( 0,∞ ) (B) ( −∞ ,0 )
choices, only one of which is the correct answer. You
have to select the correct choice (C) ( −∞ , ∞ ) − {0} (D) ( −∞ , ∞ )
Q.13 Let f ( x )= ( x + 1)
2
− 1, x ≥ − 1
Q.17 If f :R → R is a function defined by
Statement-I : The set
2x − 1
f ( x ) x cos
{ x : f ( x= }
) f −1 ( x )= {0, − 1}
=
2
π,
fk ( x )
Q.20 Let=
1
k
(
sink x + cosk x ) where x ∈ R and 1
Q.21 If f ( x ) + 2f =3x, x ≠ 0 , and
x
k ≥ 1 . Then f4 ( x ) − f6 ( x ) equals. (2014)
S= {
x ∈R : f ( x ) = }
f ( −x ) ; then S: (2016)
1 1 1 1 (A) Contains exactly one element
(A) (B) (C) (D)
4 12 6 3
(B) Contains exactly two elements.
(C) Contains more than two elements.
(D) Is an empty set.
JEE Advanced/Boards
Q.4 A set is defined as A = {x : x is irrational and 0.1 < x Q.14 Among 1000 families of a city, 40% read newspaper
< 0.101} then comment on whether A is null set or A is A, 20% read newspaper B, 10% read newspaper C, 5%
finite set or A is infinite set. read both A and B, 3% read both B and C, 4% read A
and C and 2% read all three newspapers. What is the
number of families which read only newspaper A?
Q.5 Which of these: f, {}, {2, 3} and {f} is a singleton Set?
Q.10 Is it true that both I = { x : x ∈ R and x2 + x + 1 = 0}, Q.20 Sets A and B have 3 and 6 elements respectively.
II = { x : x ∈ R and x2– x + 1 = 0} are empty sets? What can be the minimum number of elements in A ∪ B?
1 3 . 6 0 | Relations and Functions
x−2 1−x
(i) y = + Q.2 Find the domain and range of the following
x+2 1+x
functions. (Read the symbols [*] and {*} as greatest
1 integers and fractional part function respectively)
(ii) y = x2 − 3x + 2 +
3 + 2x − x2
(i) y = log 5 ( 2(sinx − cosx) + 3 )
1
(iii) y = x + 3 – log10 (2x – 3) 2x
x−2 (ii) y =
1 + x2
M a them a ti cs | 13.61
x2 − 3x + 2 (1 + 2x )2
(iii) f(x) = (h) f(x) =
x2 + x − 6 2x
x x x
(iv) f(x) = (i) f(x) = + +1
1+ | x | x
e −1 2
(vi) f(x) = log(cosec x – 1)(2 – [sin x] – [sin x]2) Q.8 Find the period for each of the following functions:
x +1 (a) f(x) = sin4x + cos4x
(vii) f(x) =
x−2 (b) f(x) = | sin x | + | cos x |
3 2
Q.3 Classify the following functions f(x) defined in (c) f(x) = cos x – sin x
R → R as injective, surjective, both or none. 5 7
n
Q.12 Show if f(x) = a − xn , x > 0, n ≥ 2, n ∈N, then
Q.6 (a) For what values of x is the inequality |f(x) + ϕ (x)|
(fof) (x) = x. Find also the inverse of f(x).
< |f(x)| + |φ(x)| true if, f(x) = x – 3, and ϕ(x) = 4 – x.
(b) For what values of x is the inequality | f(x) –ϕ (x)| > |
f(x) | – | φ(x) | true if, f(x) = x, and ϕ (x) = x – 2. 1 9
Q.13 f : − , ∞ → − , ∞ ,
2 4
Q.7 Find whether the following functions are even or Defined as f(x) = x + x – 2. Find f–1(x) and solve the
2
Q.10 Let R be the real line. Consider the following Q.6 The value of x in [–2π, 2p], for which the graph of
subsets of the plane R × R :
1 + sinx
the function y = – sec x and
S = {(x, y) : y = x + 1 and 0 < x < 2} 1 − sinx
T = {(x, y) : x – y is an integer} 1 − sinx
y=– + sec x, coincide are
Which one of the following is true? 1 + sinx
Q.2 In the square ABCD with side AB = 2, two points M Multiple Correct Choice Type
and N are on the adjacent sides of the square such that
MN is parallel to the diagonal BD. If x is the distance
Q.9 Let f : I → R (where I is the set of positive integers)
of MN from the vertex A and f(x) = Area (∆AMN) then
range of f(x) is be a function defined by, f(x) = x , then f is
(A) (0, 2] (B) (0, 2] (C) (0, 2 2 ] (D) (0, 2 3 ] (A) One–one (B) Many one
(C) Onto (D) Into
Q.3 If ‘f’ and ‘g’ are bijective functions and gof is defined
the, gofis : Q.10 The function f(x) = log x is defined for x
x2
(A) Injective (B) Surjective belonging to
(C) Bijective (D) Into only (A) (–∞, 0) (B) (0, 1) (C) (1, ∞) (D) (0, ∞)
Q.13 Which of the following function(s) is/are periodic? Q.3 Let g(x) be a function defined on [–1, 1]. If the area
of the equilateral triangle with two of its vertices at
(A) f(x) – x – [x]
(0, 0) and [x, g(x)] is 3 / 4 , then the function g(x) is
(B) g(x) = sin(1/x), x ≠ 0 and g(0) = 0 (1989)
(C) h(x) = x cos x (A) g(x) = ± 1 − x2 (B) g(x) = 1 − x2
(D) w(x) = (sin x)
(C) g(x) = – 1 − x2 (D) g(x) = 1 + x2
(D) If x > 5, then f(x) satisfies (s) f(x) < 1 ( f ) ' (2) is equal to
−1
(2010)
(A) T
he number of solutions of the equation (p) 1 x − x if x is odd,
f (x) =
0 in the interval 0,
xesin x − cos x =
π 1 + x − x if x is even
2
10
π
(B) Value(s) of k for which the planes (q) 2
kx + 4y + z = 0, 4x + ky + 2z = 0 and
Then the value of
10 ∫ f ( x ) cos π x dx is (2010)
−10
2x + 2y + z = 0 intersect in a straight
line ˆ
(C) Value(s) of k for which |x – 1| + |x – 2| (r) 3 Q.13 Let a =−ˆi − k, i 2 ˆj + 3kˆ be three
ˆ bˆ =− ˆi + j and c =+
+ |x + 1| + |x + 2| = 4k has integer
given vectors. If r is a vector such that r × b = c × b and
solution(s)
r. a = 0 , then the value of r.b is (2011)
(D) If y’ = y + 1 and y(0) = 1 then value(s) (s) 4
of y(ln2)
π π x∈
(t) 5 Q.14 Let f ( x ) = sin sin sinx for all R and
6 2
π
g(x) = sin x for all x∈
R. Let (f o g)(x) denote f(g(x))
Q.9 Match the statements/expressions in column I with 2
the values given in column II. (2009) and (g o f)(x) denote g(f(x)). Then which of the following
is (are) true? (2015)
Column I Column II 1 1
(A) Range of f is − ,
(A) R
oot(s) of the expression 2sin2θ +sin2 θ − 2 (p)
π 2 2
6
1 1
(B) Points of discontinuity of the function (B) Range of f o g is − ,
(q)
π 2 2
6x 3x 4 f (x) π
f ( x ) = cos , where [y]
π π (C) lim =
x →0 g(x) 6
denotes the largest integer less than or
equal to y (D) There is an x∈
R such that (g o f)(x) = 1
(C) Volume of the parallelopiped with
π
its edges represented by the vectors (r)
3
ˆi + ˆj , ˆi + 2 ˆj and ˆi + ˆj + π kˆ
(D) Angle between vectors a and b where π
(s)
a,b and c are unit vectors satisfying 2
a+ b+ 3 c =0
(t) π
1 3 . 6 6 | Relations and Functions
Q.15 Match the statements given in column I with the intervals/union of intervals given in column II (2011)
Column I Column II
2iz
(A) The set Re
2
z 1, z ≠ 1 is
: z is a complex number,= (p) ( − ∞, − 1) ∪ (1, ∞ )
1 − z
8 3 x −2
(B) The domain of the function f ( x ) = sin−1
() is (q) ( − ∞, 0 ) ∪ ( 0, ∞ )
1−3 ( )
2 x −1
1 2 3 4
Q.19 If the function e−1 f ( x ) assumes its minimum in
1 (A) r p s q
the interval [0, 1] at x = , which of the following is
4 (B) p r s q
true ? (2013) (C) r p q s
1 3 1 (D) p r q s
(A) f ' ( x ) < f ( x ) , <x< (B) f ' ( x ) > f ( x ) ,0 < x <
4 4 4
1 3
(C) f ' ( x ) < f ( x ) ,0 < x < (D) f ' ( x ) < f ( x ) , < x < 1
4 4
M a them a ti cs | 13.67
π π Q.22 Let f :f R
: → R be a continuous odd function,
Q.21 let f : − , → R be given by
2 2 1
which vanishes exactly at one point and f (1 ) = .
( )
3
f ( x ) − log ( sec x + tanx ) . Then (2014) x 2
Suppose that F ( x ) = ∫ f ( t ) dt for all x ∈ −1,2 and
(A) f (x) is an odd function −1
x
F(x) 1
(B) f (x) is a one-one function G( x) = ∫ t f ( f ( t ) ) dt for all x∈ −1,2. If lim
x →1 G (x)
=
14
,
−1
(C) f (x) is an onto function 1
then the value of f is (2015)
(D) f (x) is an even function 2
PlancEssential Questions
JEE Main/Boards JEE Advanced/Boards
Exercise 1 Exercise 1
Sets and Relations Sets and Relations
Q.4 Q.11 Q.14 Q.16 Q.6 Q.11 Q.14 Q.21
Q.28 Q.30
Functions
Q.5 Q.13 Q.19 Q.22 Functions
Q.26 Q.28 Q.31 Q.9 Q.11 Q.15 Q.19
Exercise 2 Exercise 2
Sets and Relations
Sets and Relations
Q.1 Q.5 Q.9
Q.2 Q.4 Q.8
Functions Functions
Q.3 Q.6 Q.8 Q.11 Q.15
Answer Key
JEE Main/Boards
Exercise 1
Sets and Relations
Q.1 {2, 4, 6} Q.2 {(8, 11), (10, 13)}
Q.3 Empty relation Q.4 yes
Q.5 No Q.6 yes
Q.7 No Q.8 R–1 = {(black, yellow), (dog, cat), (green, red)}
Q.9 Empty relation Q.10 Yes
Functions
Q.1 1 Q.2 2 α, a ≠ 0 Q.3 2f(x)
Q.4 f = R– {–2, –6}
1 1
Q.5 [–2, 0) ∪ (0, 1) Q.6 − ,
2 2
1
Q.7 x ∈ R – {–1, 1} ; y ∈ (–∞, 0) ∪ [1, ∞) Q.8 , 1
3
Q.9 αx2 + α
0, x ≤ 0 2x, x ≤ 0
Q.10 (i) (ii)
2x, x ≥ 0 0, x ≥ 0
−x , x ≤ 0
2
(iii) (iv) ax
2
x , x ≥ 0
−π
Q.12 ‒1 Q13 (i) x∈ , 2 (ii) [cos 2, 1] (iii) 6
6
Q.14 f, u are function g, h are not function. Range f = {3, 9, 10}, Range u = (3, 6, 9)
Q.15 Only f is a function from A to A
Q.16 (i) a = 2, b = –1 (ii) f(x) = 2x – 1
Q.17 Domain = {4, 6, 9, 10}. Range = {1, 2, 3}
Q.18 Range f = {–1, 0, 3, 8}
Q.19 (i) Dom. f = R; Range of f = R (ii) Dom. f = R; Range of f = R
(iii) Dom f = R; Range of f = [1, ∞) = {x : x ≥ –1} (iv) Dom f = R; Range of f = [2, ∞) = {x : x ≥ 2}
(v) Dom. f = [1, ∞); Range f = [0, ∞) = {x : x ≥ 0}
M a them a ti cs | 13.69
(iii) Dom h = R – { − 3, 3} , Range h = (–∞,0) ∪ [1/3, ∞), (iv) Dom u = [–2, 2], Range u = [0, 2]
Q.22 f and g are not functions as they are not defined form negative values of x. h is function. Range
1
h = −∞ , − ∪ [0, ∞).
4
1 + (x + 4)4 1 − (x + 4)4 1 α
Q.25 (i) ; ; (x + 4)2; ; x ≠ –4; (ii) cos x + ex; cos x – ex; ex cos x; ex cos x; a cos x
x+4 x+4 (x + 4) 4 x +4
Q.26 0; 4; 4; –1; 10
Q.27
x –4 –3 –2 –1 0 1 2 3 4
y=f(x)=x2 16 9 4 1 0 1 4 9 16
Domain of f = {x : x ∈ R} = R
Range of f = {x : x ≥ 0, x ∈ R} = [0, ∞)
Graph of y = f(x) i.e., y = x2 is as shown in the following figure.
Exercise 2
Functions
Q.1 A Q.2 C Q.3 C Q.4 C Q.5 D Q.6 C Q.7 A
JEE Advanced/Boards
Exercise 1
Sets and relations
Q.8 {1, 2, 3, 4} Q.11 6, 3 Q.14 330 families Q.16 n (A ∪ B) = n(A) + n(B)
Q.25 9
Functions
Q.1 (i) defined no where (ii) –1 < x ≤ 1 and 2 ≤ x < 3
3
(iii) < x < 2 and 2 < x <¥ (iv) (–∞, –1) ∪ [0, ∞)
2
1 1 1
(v) –2 ≤ x < 0 and 0 < x < 1 (vi) 0, ∪ ,
10 10 10
(vii) (–2 ≤ x <1)-{0} (viii) 1 ≤ x ≤ 4
(ix) (–3, –1]∪ {0} ∪ [1, 3) (x) {4} ∪ [5, ∞)
1
(xi) nπ, n + π , n ∈ I
2
1 π 5π π 5π
(xii) − , ∪ , 6 ∪ 2Kπ + , 2Kπ + , k ∈ I – {0}
6 3 3 3 3
1
(xiii) [–3, –2) ∪ [3, 4) (xiv) f (xv) −4, − ∪ (2, ∞)
2
5π −3π π π 3π 5π
(xvi) − , ∪ − , ∪ ,
4 4 4 4 4 4
Q.3 (a) Neither surjective nor injective (b) Neither injective nor surjective
Q.7 (a) Odd (b) Odd (c) Even (d) Even (e) Odd (f) Even (g) Odd (h) Even
(i) Neither odd nor even ( j) Even
M a them a ti cs | 13.71
Q.9 (a) y = log(10 – 10x), –∞< x < 1 (b) y = x/3 when –∞< x < 0 and y = x when 0 ≤ x < +∞
ex − e− x log2 x 1 1+x
Q.10 ±1, ±3, ±5, ±15 Q.11 (a) (b) (c) log
2 log2 x − 1 2 1−x
1
if 0 < x ≤ 1
Q.15 g(0) = x2
x2 if x >1
Exercise 2
Sets and Relations
Single Correct Choice Type
Q.11 A, B, C
Functions
Single Correct Choice Type
Q.8 A
Q.7 A → p; B → q; C → q; D → p Q.8 A→ p; B → q, s; C→ q, r, s, t; D → r
R ≡ R–1 relation R and R–1 both are same x + 2 is not defined for x + 2 < 0, x < −2
so R is symmetric.
log(1 − x) ≠ 0 ⇒ x ≠ 0
1 + x 1
Sol 3: f(x) = log Sol 7: f(x) = domain x ∈ R − { −1,1}
1 − x 1 − x2
2
1 + x2 + 2x 1 + x
= log = log x2 ≥ 0 ⇒ So the range y ∈ ( −∞ ,0) ∪ [1, ∞ )
1 + x2 − 2x
1 − x 1
Sol 8: y = ⇒ 2 − sin3x ≠ 0
2x 2 − sin3x
2x 1 +
f = log 1 + x2 = 2log 1 + x = 2f(x) sin3x ≠ 2
2 2x
1 + x 1 − x
1 − So the domain x ∈ R
1 + x2
x2 + 2x + 1 −1 ≤ sin3x ≤ 1
Sol 4: f(x) =
2
x + 8x + 12 1
Range y ∈ ,1
3
x2 + 2x + 1 (x + 1)2
f(x) =
( ) 2
x2 + 8x + 16 − 16 + 12 (x + 4) − 4 Sol 9: f + g = x3 + 1 + x + 1 = x3 + x + 2 R → R
f − g = x3 + 1 − x − 1 = x3 − x R → R
(x + 4)2 − 4 ≠ 0
f.g= (x3 + 1)= x 4 + 1 + x + x3
(x + 4) ≠ ± 2
f x3 + 1
x ≠ − 2 and x ≠ − 6 = x ≠ −1
g x +1
Domain x ∈ R − { −2, −6}
α
= f ax2 + α
1
5: y
Sol= + x+2
log(1 − x) Sol 10: f : R → R f(x) = x
Logarithm is not defined for (1 − x) ≤ 0, x ≥ 1 g : R → R g(x) = x
1 3 . 7 4 | Relations and Functions
πx
if x ≤ 0 g(x) = −x f(x) sin + {x} + tan2 πx
(iii) =
3
2x, x > 0 0, x ≥ 0 πx
f +g= ; f −g = sin has time period = 6
3
0, x ≤ 0 2x, x ≤ 0
{ x } has time period = 1
x2 x ≥ 0
f ⋅g = 2 tan2 πx has time period = 1
−x x ≤ 0
LCM (6,1,1) = 6
1 x≥0
f
= −1 x≤0 Sol 14: f
g
not defined x ≠ 0 A B
1 3
af = ax 6
2 9
3 10
Sol 11: f(x) = ex
Function
g(x) = loge x
Range {3, 9, 10}
(f + g)(1) =e(1) + loge(1) = e
A B
(f − g)(1) =e(1) − loge(1) = e 3
1
(1) 6
f.g(1)
= loge(1)
e= 0 g 2
9
3 10
22 2 2
Sol 12: π =
9.8 ⇒ [ π ] =9 and [ −π ] =−10
not a function as 3 has two values in Range.
f(x)= cos9x + cos( −10)x= cos9x + cos10x h
A B
π π π
f =
cos9 + cos10 =−1
2
2
2 1 3
2 6
1 3 9
Sol 13: (i) f(x)= n(1 − {x}) + sinx + + 4 − x2
2 10
1 − {x} > 0 (Always true)
1 not a function as 1 has no value
AND sinx + ≥0
2 u
1 A B
sinx ≥ −
2 1 3
π 7π 2 6
x ∈ 2nπ − ,2nπ +
6 6 3 9
10
And 4 − x2 ≥ 0
x2 ≤ 4 putting x =1 y = 3
x ∈[ −2,2] x = 2y = 6
−π x = 3y = 3
So domain is x ∈ ,2
6 So, it is a function and Range ∈ {3,6,9}
(ii) Range cos(2sinx)
−1 ≤ sin ≤ 1
−2 ≤ 2sinx ≤ 2
M a them a ti cs | 13.75
Sol 15: (i) Yes, it is because it cover all A and each x ∈ [1, ∞ )
corresponds to one value.
Range y ≥ 0 (Because value of under root
(ii) No, as it gives two value at x = b
(iii) No, as x=c has no image. is always non neg.) y∈ [0, ∞)
x2 + 3x + 5 x2 + 3x + 5
Sol 16: (i) f(x) = ax + b Sol
= 20: f(x) =
x2 − 5x + 4 (x − 4)(x − 1)
at x 1,=
= f(1) 1 (x − 4)(x − 1) ≠ 0
1= a + b … (i) x ≠ 4 x ≠1
at x 2,=
= f(2) 3 Domain x ∈ R − {1, 4}
x2 ≥ 0 (iv)Domain 4 − x2 ≥ 0
f(0) =−1 = 4 ≥ x2
f(1)= 0= f( −1) x2 ≤ 4
f(2)= 3= f( −2) x ∈ [ −2,2]
f(3)= 8= f( −3) Range u(2) = 0
So the range will be { −1,0,3,8} u(0) = 2
y ∈ [0,2]
Sol 19: (i) Domain x ∈ R
Range y ∈ R Sol 22: (i) f : R → R : f(x) =
logx
(ii) Domain x ∈ R Not a function
Range y ∈ R Because f(x) is not defined for negative values of x.
(iii) Domain x ∈ R (ii) g : R → R : g(x) =
x
x2 ≥ 0 Not a function
So the range y ∈ [ −1, ∞ ) As f(x) is not defined for negative values.
(iv) Domain x ∈ R 1
(iii) h : A → R : h(x) ; A = R − { −2,2}
2
2
x ≥0 x −4
h(x) is defined for all values of x in A set.
So the range y ∈ [2, ∞ )
So it is a function.
(v) Domain x − 1 ≥ 0
x ≥1
1 3 . 7 6 | Relations and Functions
x –4 –3 –2 –1 0 1 2 3 4
Sol 24: f = a × +b (Suppose)
f(x) = x 16 9 4 1 0 1 4 9 16
2
f(1)=1=a+b …(i)
f(2)=3= 2a + b …(ii) Sol 28:
a = 2 b = −1
x –2 –15 –1 –0.5 0 0.5 1 1.5 2
f(x)
= 2x − 1
f(x) = 1/x –1/2 –2/3 –1 –2 X 2 1 2/3 1/2
g(x) = ×
f + g = 3x − 1 Domain R–{0}
at x = 1 (f + g)(1) =
2 Range ( −∞ ,0) ∪ (0, ∞ )
(f + g)(2) =
5
Sol 29: f(x + 3) = x2 − 1 = (x + 3)2 − 9 − 6x =
(f + g)(0) =
−1
(x + 3)2 − 6(x + 3) + 8 − 10
(f + g)( −1) =4
So f + g [(1,2)(2,5)(0, −1)( −1, −4)}
=
1
Sol 25: (i) f=
+g
x+4
+ (x + 4)3 x ≠ −4 Exercise 2
1 Sets and Relations
f=
−g − (x + 4)3 x ≠ −4
x+4
Single Correct Choice Type
f.g= (x + 4)2 ; x ≠ −4
Sol 1: (C) A = {1, 2, 3, 4}
f 1
= ; x ≠ −4 R = {(2, 2), (3, 3), (4, 4), (1, 2)}
g (x + 4)4
→ A = {1, 2, 3, 4}
(ii) f(x) cos
= = x g(x) ex
and (1, 1) ∉ R so R is not reflexive
g cos x + ex
f += → (1, 2) ∈ R but (2, 1) ∉ R
x
f −=
g cos x − e So R is not symmetric
f.g = ex (cos x) → (1,2) ∈ R, (2, 2) ∈ R
cos x (1, 2) ∈ R
f /g=
x
e So R is transitive.
αf =α cos x
Sol 2: (B) Void relation: A → A
Sol 26: f(x) = x g(x) = x
It is also called empty relation
If x ≥ 0 g(x) =
x
A relation R is void relation, if no element of set A is
If x < 0 g(x) =
−x related to any element of A.
(f + g)( −2) = x − x = 0 (x, y) ⇒ x is not related to y
(f − g)(2) = 2 (2) = 4 ∴ (y, x) ⇒ y is not related to x
M a them a ti cs | 13.77
Sol 16: (B) R = {(1, 2), (2, 3)} on the set A (assume)
Sol 13: (B) A set A = {2, 3, 4, 5}
So A = {1, 2, 3}
Given relation R={(2, 2), (3, 3), (4, 4), (5, 5), (2, 3),(3, 2),
(3, 5) (5, 3)} → Reflexive (x, x) ∈ R, x ∈ A
⇒ if (x, y) ∈ R and x = y (1, 1), (2, 2), (3, 3) must be added
(x, y) ∈ R
→ symmetric (x, y) ∈ R for (y, x) ∈ R
Here x ∈ A
x = 2, 3, 4, 5 and (2, 2), (3, 3), (4, 4), (5, 5) are in solution (2, 1), (3, 2) must be added
M a them a ti cs | 13.79
Now for symmetric pair (1, 3), pair (3, 1) must be added If sinx>0, cosx<0
7 {(1, 1)(2, 2)(3, 3)(2, 1)(3, 2)(1, 3) (3, 1)} If sinx<0, cosx<0 Time period = 2π
x2 − y 2 2 11x − 15
f(x, y) = = −
4 7 49x2 + 14x + 70
a π 1 3
f(x) f = + ≠4
b ab 3
2 2
π
f = 1
2
1 a f( π) =1
)f x ) b a b
1 3 . 8 0 | Relations and Functions
x 4 − x3 + 3x2 − 2x + 2 Sol 4: (A) Clearly, f(x) = x – [x] = {x} which has period 1
(A) f(x) =
Sol 7:= nx & g(x)
1
2x2 − 2x + 1 and sin , x cos x are non–periodic function.
x
x 4 − x3 + 3x2 − 2x + 2
fog(x) = n x2 − (a + b)x + ab
2x2 − 2x + 1
Sol 5: (D) Let y =
x−c
x 4 − x3 + 3x2 − 2x + 2 ⇒ yx – cy = x2 – (a + b)x + ab
= n
2x2 − 2x + 1 ⇒ x2 – (a + b + y)x + (ab + cy) = 0
2x2 − 2x + 1 ≠ 0 for real roots, D ≥ 0
1 1
2 ⇒ (a + b + y)2 – 4(ab + cy) ≥ 0
⇒ 2 x − + > 0
2 2 ⇒ (a + b)2 + y2 + 2(a + b) y – 4ab – 4cy ≥ 0
Sol 2: (D) Given, f(x) = cos(log x) Again let f(x) = sin x, g(x) = | x |
⇒1– 1 + 4log2 y ≤ 0
Sol 13: (C) There is no information about co-domain
But x ≥ 1 therefore f(x) is not necessarily onto.
m p p r
S , S
n q q s
=
Sol 20: (B) fk ( x )
1
k
(sink x + cosk x )
⇒ qm
= pn, ps= rq f4 ( x ) − f6 ( x=
) 1
4
( ) (
1
sin4 x + cos4 x − sin6 x + cos6 x
6
)
rn transitive.
⇒ ms =
1 1
S is an equivalence relation. = 1 − 2sin2 x cos2 x − 1 − 3sin2 x cos2 x
4 6
1 1 1
Sol 16: (B) = − =
4 6 12
1
⇒ x − x > 0 ⇒ x > x ⇒ x is negative
1
x −x Sol 21: (B) f ( x ) + 2f =
3x
x
x ∈ ( − ∞ ,0 ) S : f(x) = f(–x)
1
f ( x ) + 2f =
3x … (i)
Sol 17: (A) x
2x − 1 1 1 1 3
f ( x ) x cos =
= π x cos x − π x→ f + 2f ( x ) = … (ii)
2 2 x x x
∴ a ∈ ( −1, 0 ) ∪ ( 0, 1 )
M a them a ti cs | 13.83
Sol 5: Singleton set → also known as a unit set which is Sol 9: Na = {an: n ∈ N}
with exactly on element (i.e., {0}) {π} is singleton set
N6 ∩ N8 → those element which are common in both
sets N6 and N8
Sol 6: In a plane there are two points A and B such that
OA = OB, O is a fixed point It means elements which are divisible by 6 and 8 both.
C = {4, 3, 6} ⇒x=–1
and A ∪ (B ∩ C) x2 – 2x + 1 ≥ 0
1,2 3 -2 4 5,6
1 3 . 8 4 | Relations and Functions
Sol 11: Total no. of subsets of M is 56 more than the Then A ∪ B will have
total no. of subsets of N so we know no. of total subsets
→A∪B=A+B–A∩B
of any set A which have n element = 2n
so 2m – 2n = 56 n(A ∪ B) = n(A) + n(B) – n(A ∩ B) ‘
n(A) = 10
n(A) = 10
n(B) = 6 all are disjoint
n(B) = 15 n(C) = 5
M a them a ti cs | 13.85
X Y
S Sol 20:
XY
B
A
3 6
YX
Given n(A) = 3
X ∩ (Y ∪ X)C
n(B) = 6
so in (Y ∪ X)C, there is no elements of X
we know
so X ∩ (Y ∪ X)C is π
n(A ∪ B) = n(A) + n(B) – n(A ∩ B)
C’ = X – C = {1, 2}
Sol 27: n R m ↔ n is a factor or of m (i. e. n/m)
A ∩ C′ = {1, 2, 3} ∩ {1, 2} = {1, 2}
Same as exercise –III question III
Sol 23:
Sol 28: n(A) = m
A x (B C) n(B) = n and R: A → B
then total no. of relations form A to B is
(2m)n = 2mn
(AxB) (AxC)
Sol 29: L ⇒ set of all straight lines in a plane
A × (B ∪ C) ⇒ (A × B) ∪ (A × C)
Relation R → α R β ↔ α ^β, α, β ∈ L any line never
perpendicular to itself
Sol 24: A × (A ∩ C)
so (α, α) ∉ R α ⊥ α (false)
(A × B) ∩ (A × C)
R is not reflexive
Sol 25: A = {a, b, c, d} → if (α, β) ∈ R ⇒ α ⊥ b
B = {b, c, d , e} so β ⊥ a
for → (A × B) ∩ (B × A) → (β, α) ∈ R
A × B = {(ai, bi)} R is symmetric
= {(a, b)(a, c)(a, d)(a, e), (b, b)(b, c)(b, d) (b, e)(c, b)(c, c)
(c, d)(c, e) (d, b)(d, c)(d, d) (d, e)}
B × A = {(b,a) (b,b) (b,c) (b,d) (c,a) (c,b) (c,c) (c,d) (d,a)
(d,b) (d,c) (d,d) (e,a) (e,b) (e,c) (e,d)}
(A × B) ∩ (B × A) = {(b,b) (b,c) (b,d) (c,b) (c,c) (c,d) (d,b) → if (α, β) ∈ R ⇒ α ⊥ b
(d,c) (d,d)}
and (β, γ) ∈ R ⇒ β ⊥ g
total elements = 9
so a||γ ⇒ (α ⊥ γ) → false
Sol 26: Given A = {1, 2, 3, 4, 5} ⇒ (α, γ) ∉ R R is not transitive.
R = {x, y}|x, y ∈ A and x < y|
X < x always false
so (x, x) ∉ R
Functions 1 − 5x
(iv) y =
x−2 1−x 7− x − 7
Sol 1:=
(i) y +
x+2 1+x 1 − 5x
≥0
x + 2 ≠ 0 x ≠ −2 7− x − 7
5x & 7x are always greater than zero.
x + 1 ≠ 0 x ≠ −1
x−2 Case I 1 − 5x ≥ 0 & 7− x − 7 < 0
≥0
x+2
5x ≤ 1 & 7 − x > 7
Case I x − 2 ≥ 0 & x + 2 > 0
× ≤ 0 & × < −1
x ≥ 2 & x > −2 x ∈ [2, ∞]
x ∈ (–∞, –1)
Or Case II x − 2 ≤ 0 & X + 2 < 0
Or Case II 1 − 5x ≤ 0 & 7− x − 7 < 0
x ≤ 2 & x < −2
5x ≥ 1 & 7 − x < 7
x ∈ ( −∞ , −2)
x ≥ 0 & x ≥ −1
1−x
And ≥0
1+x x ∈ [ 0, ∞ )
Case II 1 − x ≥ 0 & 1 + x > 0 So the answer is x ∈ ( −∞ , −1) ∪ [0, ∞ )
x ≤ 1 & x > −1 ⇒ x ∈ ( −1,1] 1
(v) y
= + x+2
log10 (1 − x)
Or Case II (1 − x) ≤ 0 & 1 + x < 0
(1 − x) > 0 & 1 − x ≠ 1
x ≥ 1 & x > −1
x <1 x ≠0
So the range domain will be x ∈ ( −1,1] ∩ [2, ∞ )
And x + 2 > 0
x∈φ
x ≥ −2 x ∈ [ −2, ∞ )
1
(ii) y= x2 − 3x + 2 + So the domain x ∈ [ −2,1) − {0}
3 + 2x − x2
1 2log10 x + 1
y= (x − 2)(x − 1) + (vi) f(x) = log100x
−x
(3 + x)(x + 1)
100 x ≠ 0,1 100 x > 0
(x − 2)(x − 1) ≥ 0 ⇒ x ∈ ( −∞ ,1] ∪ [2, ∞ )
1
And (3 − x)(x + 1) > 0 ⇒×∈ ( −1,3) x ≠ 0, x >0
100
The domain x ∈ ( −1,1 ], ∪ [ 2,3 ) 2log10 x + 1
And >0
1 −x
(iii) y =x+ − log10 (2x − 3)
(x − 2)1/3 2log10 x + 1
<0
×≥0 x
x > 0 So
And x − 2 ≠ 0
2log10 x + 1 < 0
x≠2
1 1
And 2x − 3 > 0 log10 x < − ⇒ 0 <x<
2 10
3
x> 1 1
2 So domain x ∈ 0, −
3 10 10
So the domain x ∈ , ∞ − {2}
2
1 3 . 8 8 | Relations and Functions
x ∈( −∞ ,1) − {0}
1
x ∈ [nπ, n + π]
2
So domain [ −2,1) − {0}
cos x − 1 / 2
5x − x2 (xii) f(x) =
(viii) y = log 6 + 35x − 6x2
4
cos x − 1 / 2 −1
5x − x2 = x ≠ 6,
log ≥0 (6 − x)(6x + 1) 6
4
−1
5x − x2 Case I: Let’s say < x<6
≥1 6
4 1
5x − x2 − 4 ≥ 0 So cos x − >0
2
−1
x2 − 5x + 4 ≤ 0 Case II: x <
6
(x − 4)(x − 1) ≤ 0 −1
So cos x <0
2
x ∈ [1, 4]
1
cos x <
2 1 2
(ix) f(x)
= x= x +
9 − x2
9 − x2 > 0
x2 < 9
6
x ∈( −3 ,3) /3
And x2 − x ≥ 0 -1/6
1 + 2Kx
1 + x + x2 − 1 + x + x2 Case III: x > 6
1 − 2Kx
x ∈ [1, ∞ ) 1
cos x ≤
2
if x < 0 x (x + 1) ≥ 0
From group analysing solution
x ∈ ( −∞ , − 1]
−1 π 5π
, ∪ ,6 ∪
So the domain x ∈ ( −3, − 1] ∪ [1,3) 6 3 3
[x]2 ≤ 9 2
And x − 5x − 24 > 0
[x]∈ [ −3,3] x2 − 3x − 8x − 24 > 0
And [x]2 − 5 > 0 (x + 3)(x − 8) > 0
[x]2 > 5 ⇒ x ∈[ −∞ , −3] > 0
2 2
And [x] − 5 ≠ 1 [x] − 5 > 1 x ∈[ −∞ , −3] ∪ [8, ∞ )
2 2
[x] ≠ 6 [x] > 6 [x]= 3, − 3 Case I if x + 2 > 0 ⇒ x > −2
2 2
And [x] − 5 ≠ 4 [x] ≠ 9 The x2 − 5x − 24 > (x + 2)2
[x] ≠ ± 3 −5x − 24 > 4 + 4 x
1 1 −28
(xiv) f(x) = + log(2{x} −5) (x2 − 3x + 10) + x<
[x] 1− x 9
2{x} − 5 < 0 So log2{x} −5 (x2 − 3x + 10) is not defined. So Case II if x + 2 < 0 then always true in interval
x∈φ
x2 − 5x − 24 > x + 2
=(xv) f(x) log7 log5 log3 log2 (2x2 + 5x2 − 14x) x <− 2
log5 log3 log2 (2x3 + 5x2 − 14x) > 0 So the domain x ∈( −∞ , −3]
3 2
log3 log2 (2x + 5x − 14x) > 1 (xvii) y= log(1 − log10 (x2 − 5x + 16)
log2 (2x3 + 5x2 − 14x) > 3 1 − log10 (x2 − 5x + 16) > 0 AND x2 − 5x + 16 > 0
2x3 + 5x2 − 14x > 8
log10 (x2 − 5x + 16) < 1 AND is the always positive as its
2x3 + 5x2 − 14x − 8 > 0 −b 5
minimum value of= x = is positive.
2a 2
(x − 2)(2x2 + 9x + 4) > 0
x2 − 5x + 6 < 0
(x − 2)(2x + 4)(2x + 1) > 0
(x − 3)(x − 2) < 0
−1
x ∈ −4, ∪ (2, ∞ ) x ∈( 2,3 )
2
So the domain 2<x<3
(xvi) f(x)
= cos2x + 16 − x2
2 x +1
(xix) f(x) = log4 (2 − (x)1/ 4 −
2
cos2x > 0 and 16 − x ≥ 0 x+2
π π 2 x +1
2x ∈ 2nπ − ,2nπ + & x ∈ [ − 4, 4 ] 2 − (x)1/ 4 − >0
2 2 x+2
π π
x ∈ nπ − ,nπ + & x ∈ [ − 4 , 4] 2 x +1
4 4 (x)1/ 4 − >2
x+2
5π 3π π π 3π 5π
So domain ∈ − , − ∪ − , ∪ , (x)3/ 4 + 2(x)1/ 4 + 2(x)2/ 4 + 1 < 2(x)2/ 4 + 4
4 4 4 4 4 4
x ≥ 0 so put x = t 4
2
(xvii) =
f(x) n( x − 5x − 24 − x − 2)
t3 + 2t + 2t2 + 1 < 2t2 + 4
x2 − 5x − 24 − x − 2 > 0 t3 + 2t − 3 < 0
1 3 . 9 0 | Relations and Functions
−3 x 1
sin(x − 450 ) > so the domain x ∈ R for x < 0 =
f(x) =
2 1−x 1
−1
π x
Range
= f(x) log 5 2sin x − + 3 −1 < f(x) < 0
4
π So f(x) ∈( −1,1)
+ 1 ≤ 2sin x − + 3 ≤ 5
4
(v) y = 2−x + 1+x
π
log 5 (1) ≤ log 5 2sin x − + 3 ≤ log 5 (5)
4 z − x ≥ 0 and 1 + x ≥ 0
0 ≤ f(x) ≤ 2 x ≤ 2 and x ≥ −1
2x Domain x ∈ [ −1,2]
(ii) y =
2
1+x
Range 2 − x decreases with increment in x 1+x
Domain x ∈ R Increase with increment in x
2
Range y = Since both are linear function root, so we can say that
1 it’s maximum will be at middle point of boundary
x+
x defined.
1
x+ ≥ 2 for x ≥ 0 from arithmetic mean > geometric 1 1
x So f(x) ≤ 2 − + 1 +
mean 2
2
1 3 3
x+ ≤ −2 for x ≤ 0 from A.M >G.M f(x) ≤ + =6
x 2 2
So [-1,1]
f(x) ≥ 3 ( at boundary point )
x2 − 3x + 2
(iii) f(x) =
x2 + x − 6 3 ≤ f(x) ≤ 6
2
Domain x + x − 6 ≠ 0
(vi)
= f(x) log(cos ec x −1) (2 − [sinx] − [sinx]2 )
2
x + 3x − 2x − 6 ≠ 0
[sinx]
= 0 or − 1
(x + 3) (x − 2) ≠ 0
2 − [sinx] − [sinx]2 =
2
x ≠ 2, − 3
M a them a ti cs | 13.91
5
2
25 5
2
25
= x + + 9 − x + + 1 − 3 4
2 4 2 4
3 4
1 3 . 9 2 | Relations and Functions
Even
1 + x
(g) n = f( −x)
1 − x
3 4
1−x
f( −x) = − n − =−f(x)
1+x
x ∈ ( −∞ ,3) ∪ (4, ∞ )
Odd function
(b) x − (x − 2) > x − x − 2 (1 + 2x )2
(h) f(x) =
2 > x − x−2 2x
(2 + 1)2
x
f( −x)
= = f(x)
x − x−2 < 2 2x
If x > 2 not true Even
−x x
If 0 ≤ x ≤ 2 2x − 2 < 2 f( −x)
(i) = − + 1 ≠ f(x) ≠ −f(x)
1 2
−1
x ≤ 2 x ∈ [0,2] ex
If x < 0 − 2 < 2 always true So neither odd nor even
( −∞ ,2)
( j) f(x) = [(x + 1)2 ]1/3 + [(x − 1)2 ]1/3
= log −x + 1 + x2
Sol 7: (a) f( −x) 1/3 1/3
f( −x)= (1 − x)2 + ( −x − 1)2
= log 1 + x2 − x 1 + x2 +=
f( −x) + f(x) x 0 1/3 1/3
= (x − 1)2 + (1 + x)2 = f(x)
f(x) = −f(x) Even function
So odd function
f(x) sin4 x + cos4 x
Sol 8: (a)=
ax + 1
(b) f(x) =
ax − 1 (sin2 x)2 + ((cos2 )3 x)2 + 2sin2 x
f(x) =
1
+1 cos2 x − 2sin2 cos2 x
x 1 + ax
f( −x) =a = =−f(x)
( )
2
1 ax
− 1 f(x) =sin2 x + cos2 x − 2sin2 x cos2 x
−1
ax
(sin2x)2
f(x)= 1 −
Odd function 2
= x 4 − 2x2
(c) f(x) sin2x ⇒ Time period ⇒ π
Even π
So f(x) has Time period
2
(d) f(x)= x2 − | x |
f(x) | sinx | + | cos x |
(b)=
f( −x) = x2 − | x |
π
Case-I 0 < x <
Even function 2
3π 2
Case-IV < x < 2π f(x) − sinx + cos x Sol 11: f(x) = n x + x + 1
2
sin x cos x
y = n x + x2 + 1
ey − x= x2 + 1
Since combining graphs, we can see | sinx | + | cos x | e2y + x2 − 2ey (x) = x2 + 1
π
has Time period. (1 − e2y ) e2y − 1
2 x=
− =
2ey 2ey
3 2x
(c)
= f(x) cos x − sin
5 7 1 y 1
2π e − y
2 e
cos3x has time period
3
1 1 x 1
3x 10π f −= (x) e − x
cos has time period 2 e
5 3
g ( x ) = sinx
2π x
has time period × (7)
2 (b) f(x) = 2 x −1
10π L.C.M.(10π,7π) x
L.C.M. ,7=
= 70π log2 y =
3 H.C.F.(3,1) x −1
2nπ 1 x +1
= 3π ⇒ f −1 (x) = log
H.C.F.(5 − n2 ,5 + n2 ) 2 1 − x
1 3 . 9 4 | Relations and Functions
1
= (a − xn )1/n
Sol 12: f(x) Sol 15: f(x) = max x,
x
1/n
n
fof(x) =a − (a − x)n
1
1/n -1
n f(x)
=a − (a − xn ) 1
-1
to f(x) = x
If g is inverse of f then fog(x)=x from 1 1
f = f(x)
= max , x
x x
Above we can say f −1 (x) = f(x)
1
Sol 13: f(x) = x2 + x − 2 , 0< x <1
x
2 x, x >1
1 9 f(x)
y = x + − 1 ,
2 4 x < −1
x
x, 0 > x > −1
1 9
x =− + y−
2 4 1
f = f (x)
1 9 x
f −1 (x) =y =− + x−
2 4
1
f(x) = f −1 (x) if 0< x ≤1
g(x) x2
⇒ fof(x) = x x2 if x >1
x + 1, x < −1
2
Sol 14: f(x)
3
x − 1, x ≤ x < 0 x2 + 2x + c
Sol 16: f(x) =
(a) If f(x) is odd x2 + 4x + 3c
Bijective function
Sol 3: (A) (A)x R, y ↔ |x| = |y|
Sol 6: (C) R = {(3, 3), (6, 6), (9, 9),(12, 12), (6, 12), (3, 9), If (e, f) = (c, d)
(3, 12), (3, 6)} then (c, d) R (e, f) is always true
on the set A = {3, 6, 9, 12} (R is reflexive)
→ for set A so in equation (i) (c, d) → (e, f)
(3, 3), (6, 6), (9, 9), (12, 12) ∈ R af (b + e) = be (a + f)
R is reflexive so ((a, b), (e, f)) ∈ R
→ (6, 12) ∈ R but (12, 6) ∉ R ∴ R is transitive.
So R is not symmetric
Sol 9: (A) W = all wards in the English dictionary
→ (3, 6) ∈ R and (6, 12) ∈ R and also (3, 12) ∈ R
R = {(x, y) ∈ w × w the words x and y have at least one
R is transitive. letter in common}
→ a word have all letter common to itself R is reflexive
Sol 7: (C) R = {(1, 3), (4, 2), (2, 4), (2, 3), (3, 1)}
→ if x and y have one letter common soy and x is same
on the set A = {1, 2, 3, 4} condition
→ for set A (x, y) ∈ R → (y, x) ∈ R
(1, 1), (2, 2), (3, 3), (4, 4) ∉ A R is symmetric
R is not reflexive → if (x, y) ∈ R, (y, z) ∈ R
→ (2, 3) ∈ R , (3, 2) ∉ R x and y have one letter common
So R is not symmetric (1,3)∈ R, (3,1) ∈ R y and z have one letter common
But (1, 1) ∉ R its not mean that it is necessary to x and z have one
so R is not transitive. letter common
R is not transitive.
Sol 8: (D) R: N × N
(a, b) R (c, d) if ad(b + c) = bc(a + d) Sol 10: (C) R → real line
⇒ for (a, b)R(a, b) ⇒ ab(b+a) = ba(a+b) Given subset S = {(x, y): y = x + 1 and 0 < x < 2}
Which is true so R is reflexive and T = {(x, y): x – y is an integer}
→ for symmetric for S y = x + 1
(a, b) R (c, d) ⇒ (c, d) R (a, b) but x ≠ x + 1
ad(b + c) = bc(a + d) s is not reflexive
cb(d + a) = da(c + b) ... (ii) so s is not equivalence
ad(b + c) = bc(a + d)
M a them a ti cs | 13.97
Sol 2: (B)
Multiple Correct Choice Type
D C
Sol 11: (A, B, C) x = {1, 2, 3, 4, 5}, y= {1, 3, 5, 7, 9} N o
(A) R1 = {(x, y) | y = 2 + x , x ∈ X, y ∈ Y } P
B
R = {(1, 3) (3, 5)(5, 7)} A M
(B) R2 = {(1, 1), (2, 1)(3, 3), (4, 3), (5, 5)} AC = 2 2
is satisfied R: x → y BD = 2 2
1 1 AP = x
2 1
3 3 → a subset of y PC
= 2 2 − x ⇒ MN
= 2x
4 3 1
( ∆AMN)Area =× base × height
5 5 2
1
(C) R3 = {(1, 1), (1, 3), (3, 5), (3, 7), (5, 7)} =× (AP) × (MN)
2
All elements are from x 1
= x2 (for x < 2)
× (2x)(x) =
1 1 2
f( −1) =−1 f( −2) =−8 ∆AMN is maximum
1 3
3 5
f(x) =maximum= ( 2)2 = 2
3 7
5 7 So the range is (0,2)
all element are from y
(D) R4 = {(1, 3),(2, 5), (2, 4), (7, 2)}
1 3 . 9 8 | Relations and Functions
y 5(17) +=
= 1 86 Multiple Correct Choice Type
[x + 2y] =
189
Sol 9: (A, D)
= f(x) x f : I →R
= ax3 + ex
Sol 5: (D) f(x) F is not onto function.
'
f= (x) 2ax2 + ex But f is one-one function.
1 + sinx 1 − sinx x x − 2 x −1
+ 2sec x
= Sol 11: (B, C) y =
1 − sinx 1 + sinx
x −1 −1
2 x −1 ≥ 0 ⇒ x ≥ 1
= 2sec x
2
1 − sin x
And x −1 ≠ 1 ⇒ x ≠ 2
2
= 2sec x
cos2 x And x − 2 x − 1 ≥ 0
(t2 + 1) t2 + 1 − 2t x if x ∈ Q
f(x) = = t2 + 1 = x Sol 14: (B, C) f(x) =
t −1 1 − x if x ∉ Q
Putting for x<2, it is not always defined. x ∈ [ 0,1 ]
1
0 ≤ f(x) < 1 [x ∈ [0,1] − {1}]
Sol 12: (A, C, D) (A) f(x) = 2 x −1
Decreasing function At x=1, f(x) = 1
It is clear from the graph that y = tan x is one–one and We must have
onto, therefore (B) → (r)
( )
2 ( 8 − k ) + 2 ( 4 − 2k ) + k 2 − 16 =
0
Sol 7: A→ p; B → q; C→ q; D → p ⇒k=
2, 4 .
(x − 1)(x − 5) (C) Let f(x) = |x + 2| + |x + 1| + |x −1| + |x −2|
Given, f(x) =
(x − 2)(x − 3)
⇒ k can take value 2, 3, 4, 5.
The graph of f(x) is shown
dy
y (D) ∫ y + 1 = ∫ dx
⇒ f ( x ) =2ex − 1 ⇒ f (ln 2 ) =
3
y=1
Sol 9: A→ q, s; B→ p, r, s, t; C→ t; D→ r
x’ x
0 1 2 3 5
2sin2 θ + 4 sin2 θ cos2 θ = 2
y’ (
sin2 θ + 2sin2 θ 1 − sin2 θ =1)
1
3sin2 θ + 2sin2 θ − 1 = 0 ⇒ sin θ =± , ±1
(A) If – 1 < x < 1 ⇒ 0 < f(x) < 1 2
π π
(B) If 1 < x < 2 ⇒ f(x) < 0 ⇒θ= , .
4 2
(C) If 3 < x < 5 ⇒ f(x) > 0 3x
(B) Let y =
π
(D) If x > 5 ⇒ f(x) < 1 1 π
⇒ ≤ y ≤3∀ x∈ , π
2 6
Sol 8: A→ p; B → q, s; C→ (q, r, s, t); D → r
Now f(y) = [2y] cos[y]
(A) f ' ( x ) > 0, ∀ x ∈ ( 0, π 2 )
1 3
Critical points are=y = , y 1,
= y = ,y 3
f (0) < 0 and f ( π 2 ) > 0 2 2
so one solution. π π π
⇒ points of discontinuity , , , π .
6 3 2
-4x 4x 1 1 0
(C) 1 2 0 = π ⇒ volume of parallelepiped = π
1 1 π
4-2x 2x-4
6
(D) a + b =3
-2 -1 1 2
⇒ 2 + 2 cos α = 3
⇒ 2 + 2cos α =3
(B) Let (a, b, c) is direction ratio of the intersected line, π
⇒α = .
then 3
ak + 4b + c = 0 1
Sol 10: f (=
0 ) 1, f ' (=
x ) 3x2 + ex /2
2
4a + kb + 2c = 0
a b c
( )
⇒ f ' g ( x ) g' ( x ) =
1
= =
8 − k 4 − 2k k 2 − 16 1
0 ⇒ g' (1 ) = =
Put x = 2.
f ' (0)
1 3 . 1 0 2 | Relations and Functions
x
Sol 11: (B) e− x f ( x ) =
2 + ∫ t 4 + 1dt … (i) ( π π
)
⇒ g g ( x ) ∈ − , ∀ x ∈R
0 2 2
(
f f −1 ( x ) = x ) (( π π
))
Also, g g g ( x ) ∈ − , ∀ x ∈R
2 2
⇒f f ( −1
( x ))' ( f −1
)
(x) ' =
1 ⇒ f (2) ' =( 1
−1
)
f ' f (2)
−1
( ) 1 1
Hence, f (x) and f g ( x ) ∈ − , ( )
⇒ f (0) =
2⇒ f −1
(2) =
0 2 2
(f ( 2) ) ' = f ' 10
−1
( ) f (x)
1
3
sin g g ( x ) 1 g g ( x )
. 3
( ) ( )
lim = lim
x →0 g(x) x →0 1 g(x)
e− x ( f ' ( x ) − f ( x ) ) = x4 + 1
3
g g(x) ( )
Put x = 0
π
⇒ f ' (0) − 2 =1 ⇒ f ' (0) = 3 sin sin x
π 2 π
⇒ lim . =
(f −1
(2)) ' = 1 / 3 x →0 6 π
2
sinx
6
{x} , 2n − 1 ≤ x < 2n π 1 π 1
Sol 12: (D) f ( x ) = ( )
Range of g f ( x ) ∈ − sin , sin
1 − {x} , 2n ≤ x < 2n + 1 2 2 2 2
3x − 3x −2
⇒r = − 3 ˆi + 6 ˆj + 3 kˆ Case I: −1 ≤ 0
1 − 32x −2
r .b =3 + 6 =9
⇒
(3 − 1)(3 − 1) ≥ 0
x x −2
(x)
Sol 14: (A, B, C) Given g=
π
2
sinx ∀ x ∈R (3 − 1) 2x −2
1 ⇒ x ∈ ( − ∞ , 0 ∪ (1, ∞ )
f ( x ) = sin g g ( x )
3
( )
M a them a ti cs | 13.103
3x − 3x −2 (1 − x )
2
Case – II: + 1≥ 0 sin2 x = x2 − 2x + 1 + 1
2x
1−3 −2
=(1 − x ) + 1
2
⇒
(3 x −2
)(
− 1 3x + 1 )≥0 ⇒ (1 − x ) cos2 x =
2
−1
(3 .3
x x −2
−1 ) Which can never be possible
⇒ x ∈ ( − ∞ ,1 ) ∪ 2, ∞ )
P is not true
So, x ∈ ( − ∞ ,0 ) ∪ 2, ∞ ) .
⇒ Let H (=
x ) 2f ( x ) + 1 − 2x (1 + x )
(C) R1 → R1 + R 3
H (0) = 2 f (0) + 1 – 0 = 1
0 0 2
f ( θ ) =− tan θ 1 tan θ H (1) = 2 f (1) + 1 – 4 = -3
−1 − tan θ 1 ⇒ So H (x) has a solution
(
= 2 tan2 θ=
+ 1 2sec2 θ . ) So Q is true.
3 1/2
f '(x) ( x ) (3x − 10 ) + ( x ) × 3
3/2
= Sol 18: (A, B)
(D) 2
15 1/2 1 1
= ( x ) ( x − 2) cos 4 θ= ⇒ 2 cos2 2 θ − 1=
2 3 3
Increasing, when x ≥ 2 . 2 2
⇒ cos2 2 θ = ⇒ cos 2 θ = ±
3 3
Sol 16: (B) f ( x ) = 2x3 − 15x2 + 36x + 1 2 1 + cos 2 θ 1
Now f ( cos 4 θ ) = = =1+
2
2 − sec θ cos 2 θ cos 2 θ
f ' ( x ) = 6x2 − 30x + 36
(
= 6 x2 − 5x + 6 ) 1
⇒ f =
3
1±
3
2
=6 ( x − 2 )( x − 3 )
(
f f ( x )
2 1 ), x<0
f ( x ) + 2x = 2 1 + x ( 2
) f4 ( x ) =
( )
f2 f1 ( x ) − 1 , x ≥ 0
⇒ (1 − x ) sin2 x + x2 + 2x =2 + 2x2
2
x2 , x<0
=
2x
e − 1 , x ≥ 0
1 3 . 1 0 4 | Relations and Functions
π π
Let g (x) = sec x + tan x ∀ x ∈ − ,
2 2
π π
⇒
= g' ( x ) sec x ( sec x + tanx ) > 0 ∀ x ∈ − ,
2 2
⇒ g ( x ) is one-one function
( ))
3
Hence log g ( x )
e ( is one-one function.
π π
And g ( x )∈ ( 0, ∝ ) ∀ x ∈ − ,
2 2
( )
⇒ log g ( x ) ∈R . Hence f (x) is an onto function.
1
Sol
= 22: G (1 ) t f ( f ( t ) ) dt
∫= 0
−1
f (- x) = - f (x)
1
Given f (1 ) =
2
F ( x ) − F (1 )
F(x) x= −1 f (1 ) 1
lim
= lim =
x → 1 G ( x ) x → 1 G ( x ) − G (1 )
( )
f f (1 ) 14
x −1
1/2 1
⇒ =
f (1 / 2 ) 14
1
7.
⇒ f =
2
2017-18 100 &
op kers
Class 11 T
By E ran ulty
-JE Fac
IIT enior emier
S f Pr tes.
o titu
Ins
MATHEMATICS
FOR JEE MAIN & ADVANCED
SECOND
EDITION
Exhaustive Theory
(Now Revised)
Formula Sheet
9000+ Problems
based on latest JEE pattern
1. LIMITS
1.2 Meaning of x → a
Let x be a variable and a be constant. Since x is a variable; we can change its value at our pleasure. It can be
changed in such a way that its value comes nearer and nearer to a. Then we say that x approaches a and it is
denoted by x → a:
x a- a+ x
Figure 14.2
We know that |x – a| is the distance between x and a on the real number line and 0 < |x – a| if x ≠ a, “x tends to a”
means
(a) x ≠ a, i.e. 0 < |x – a|,
1 4 . 2 | Limits, Continuity and Differentiability
(b) x takes up values nearer and nearer to a, i.e. the distance |x – a| between x and a becomes smaller and smaller.
One may ask “how much smaller”? The answer is, as much as we please. It may be less than 0.1 or 0.00001
or 0.0000001 and so on. In fact, we may choose any positive number δ. However small it may be, |x – a| will
always be less than δ. The above discussion leads up to the following definition of x → a.
Let x be a variable and a be a constant.
Definition: Given a number δ > 0 however small, if x takes up values, such that 0 < |x – a| < δ. Then x is said to tend
to a, and is symbolically written as x → a
Note. If x approaches a from values less than a, i.e. from the left side of a, we write x → a–. If x approaches a from
values greater than a, i.e. from the right side of a, we write x → a+.
But x → a means both x → a– and x → a+. So x approaches or tends to a means x approaches a from both sides
right and left.
Neighbourhood of point a: The set of all real numbers lying between a – δ and a + δ is called the neighbourhood
of a. Neighbourhood of a = (a – δ, a + δ) ; x ∈ (a – δ, a + δ)
x2 − 4
Illustration 1: Consider the function f(x) = . We investigate the behaviour of f(x) at the point
x−2
x = 2 and near the point x =2. (JEE MAIN)
Sol: Here as f(2) = 0 , therefore try to evaluate the value of f(x) when x is very near to 2.
4−4 0
f(2)
= = , which is meaningless. Thus f(x) is not defined at x = 2.
2−2 0
Now we try to evaluate the value of f(x) when x is very near to 2 for some values of x less than 2 and then for x
greater than 2.
Figure 14.3
It is clear that as x gets nearer and nearer to 2 from either side, f(x) gets closer and closer to 4 from either side.
M a them a ti cs | 14.3
When x approaches 2 from the left hand side the function f(x) tends to a definite number 4. Thus we say that as x
tends to 2 the left hand limit of the function f exists and equals to the definite number 4. Similarly, as x approaches 2
from the right hand side, the function f(x) tends to a definite number, 4.
Again we say that as x approaches 2 from the right hand side of 2, the right hand limit of f exists and equals to 4.
−1,if x < 0
Illustration 2: Discuss the limit of the function f(x) = at x = 0 (JEE MAIN)
1, if x > 0
Sol: Sketch its graph when x is very near to 0 for range of x from – 1 to 1.
−1, x < 0
We have, f(x) =
+1, x > 0
(i) As x approaches zero from the left of zero, f(x) remains at –1. And we say that the left hand limit of f exists at
x = 0 and equals to –1. lim f(x) = −1
x →0 −
(ii) As x approaches zero from the right of zero, f(x) remains at 1. So we say that the right hand limit of f at x = 0
exists and equals to +1. lim f(x) = +1
x →0 +
(iii) Left hand limit of f(x) (at x = 0) ≠ Right hand limit of f(x) {at x = 0}.
So the lim f(x) does not exist.
x →0
1
Illustration 3: Discuss the limit of the function f(x) = at x = 0 and its graph (JEE MAIN)
x
Sol: Same as above illustration.
1 1
We have, f(x) = Let us draw the graph of the given function f(x) = .
x x
(i) As x approaches zero from the left of zero the graph never approaches a finite number so we say that the left
hand limit of f at x = 0 does not exist i.e. lim f(x) does
= −1 not exist
x →0 −
(ii) As x approaches zero from the right of zero, the graph again does not approach a finite number. Again we say
that the right hand limit of f at x = 0 does not exist. lim f(x) =does
+1 not exist
x →0 +
(iii) At x=0 left hand limit of f ≠ right hand limit of f
Hence, the limit of f(x) does not exist.
1 4 . 4 | Limits, Continuity and Differentiability
1
Y=
x
8
7
6
5
4
3
2
1
-8 -7 -6 -5 -4 -3 -2 -1
X’ X
0
11 2 3 4 5 6 7 8
2
3
4
5
6
7
8
Y’
Figure 14.4
PLANCESS CONCEPTS
(a) The fact that the limit of f(x) exists at x = a means that the graph of f(x) approaches the same value
from both sides of x = a.
(b) The fact that f(x) is continuous at x = a means that there is no break in the graph as x moves from
a– to a+.
Vaibhav Krishnan (JEE 2009, AIR 22)
Illustration 4: Discuss the limits of f(x) = |x| at x = 0 and draw its graph. (JEE MAIN)
Sol: We have f(x) = |x|, therefore f(x) is equals to x for x > 0 and – x for x < 0.
Increasing x Decreasing x
x -3 -2 -1 0 1 2 3
f(x) 3 2 1 0 1 2 3
Limit
Decreasing f(x) Decreasing f(x)
x, x > 0
We have f(x) = |x| Þ f(x) =
−x, x < 0
M a them a ti cs | 14.5
(-3, 3) (3, 3)
3
(-2, 2) 2 (2, 2)
(-1, 1) 1 (1, 1)
X’ X
0
-3 -2 -1 1 2 3
Figure 14.5
(i) As x approaches zero from left of zero, f(x) = 0. And we say that left hand limit of f(x) exists, and is equal to
zero.
lim− f(x) = 0
x →0
(ii) As x approaches zero from the right of zero f(x) is equal to zero. So we say that the right hand limit of f at x =
0 exists, and is equal to zero. lim f(x) = 0 Here =lim f(x) = lim f(x) 0
x →0 + x →0 − x →0 +
Hence we say that the limit of f (x) at x = 0 exists and equals to zero.
Increasing x Decreasing x
Limit
Increasing f(x) Decreasing f(x)
From the table we deduce that the value of f(x) at x = 3 should be greater than 5.999 and less than 6.001.
It is reasonable to assume that the value of function f(x) at x = 3 from the left of 3 is 5.999.
lim
= f(x) lim
= f(x) lim
= f(x) 6
x →3− x →3+ x →3
3x2 − x − 10
Illustration 6: Evaluate: lim (JEE MAIN)
x →2 x2 − 4
Increasing x Decreasing x
From the table we deduce that the value of f(x) at x = 2 should be greater than 2.7499 and less than 2.750.
It is reasonable to assume that the value of function f(x) at x = 3 from the left of 2 is 2.7499.
Similarly, when x-approaches x = 2 from the right f (x) should be 2.750. lim f(x) ≈ 2.750 ....(ii)
x → 2+
From (i) and (ii), we conclude that the limit is equal to 2.75. lim
= f(x) lim
= f(x) lim
= f(x) 2.75
x → 2− x → 2+ x →2
PLANCESS CONCEPTS
• If Lim f ( a − b )= Lim + ( a + b ) , then we can say that both the left hand limit and right hand limits exist
x→a x → a+
at x = a, and irrespective of the value of the function at a, i.e f(a), the function does not have a limit at
• If both the left hand limit and right hand limit of f(x) at x = a exist and at least one of them is not equal
to f(a), then the limit of f at x = a does not exist.
Shrikant Nagori (JEE 2009, AIR 30)
1.5 Working Rule for Evaluation of Left and Right Hand Limits
Step III. The value obtained in step 2 is the right hand limit of f(x) at x = a.
M a them a ti cs | 14.7
5x − 4, if 0 < x ≤ 1
f(x) = 2
4x − 3x, if 1 < x < 2
Sol: By taking left hand limit and right hand limit we can conclude that the given limit exist or not.
Left hand limit = lim f(x) = lim (5x − 4) [∵ f(x) = 5x – 4, if 0 < x ≤ 1]
x →1− x →1−
Right hand limit = lim f(x) = lim (4x2 − 3x) [∵ f(x) = 4x2 – 3x, if 1 < x < 2]
x →1+ x →1+
2
= lim[4(1 + h) − 3(1 + h)] [Put x = 1 + h]
h→0
∴ At x = 1, Left hand limit = Right hand limit ⇒ lim f(x) exists and it is equal to 1.
x →1
Illustration 8: Evaluate the left hand and right-hand limits of the following function at x = 1 (JEE MAIN)
1 + x2 , if 0 ≤ x ≤ 1
f(x) = does lim f(x) exist ?
2 − x, if x > 1 x →1
Sol: By putting x = 1 – h and 1 + h, we can conclude left hand limit and right hand limit respectively. If both are
equal then the given limit exist otherwise not exist.
Left hand limit = lim f(x)
x →1−
= lim (1 + x2 ) [∵ f(x) = 1 + x2, if 0 ≤ x ≤ 1]
x →1−
Therefore, At x = 1, Left hand limit ≠ Right hand limit ⇒ lim f(x) does not exist.
x →1
| x − 6 |
, x≠6
Illustration 9: Evaluate the right hand limit of the function f(x) = x − 6 at x = 6 (JEE MAIN)
0, x = 6
Sol: Here Right hand limit of the given function f(x) at x = 6 is lim f(6 + h) .
h→0
Right hand limit of f(x) at x = 6 = lim f(x) , = lim f(6 + h) ,
x →6 + h→0
|6+h−6 | |h| h
= lim , = lim = lim , = lim 1 = 1
h→0 6 + h − 6 h→0 h h→0 h h→0
1 4 . 8 | Limits, Continuity and Differentiability
| x − 4 |
, x≠4
Illustration 10: Evaluate the left hand limit of the function: f(x) = x − 4 at x = 4 (JEE MAIN)
0, x = 4
Sol: Here Left hand limit of the given function f(x) at x = 4 is lim f(4 − h) .
h→0
| 4 −h− 4 | | −h |
Left hand limit of f(x) at x = 4 = lim f(x) = =
lim f(4 − h) lim = lim
x→4 − h→ 0 h→ 0 4 −h− 4 h→ 0 −h
h
= lim =lim − 1 =−1
h→0 −h h→0
Illustration 11: Evaluate the left hand and right hand limits of the following function at x = 2: (JEE MAIN)
2x + 3, if x ≤ 2
f(x) = Does lim f(x) exist?
x + 5, if x > 2 x →2
= lim[2(2 − h) + 3] (Put x = 2 – h) = 4 + 3 = 7
h→0
= lim(2 + h + 5) = 7 (put x = 2 + h)
h→0
Therefore, the left hand limit = right hand limit [at x = 2] ⇒ lim f(x) exists and it is equal to 7.
x →2
mx2 + n, x < 0
Illustration 12: For what integers m and n does lim f(x) exist, if f(x)= nx + m, 0 ≤ x ≤ 1 (JEE ADVANCED)
x →0
2 x >1
nx + m,
Sol: As the given limit exist, therefore its Left hand limit must be equal to its Right hand limit.
lim f(x)
Limit at x = 0 ; = lim (mx2 + n) = lim m(0 − h)2 + n =
n
x →0 − x →0 −h h→0
lim =
f(x) lim (nx +=
m) lim[n(0 + h) + m] = m
x →0 + x →(0 +h) h→0
a + bx, x < 1
Illustration 13: Suppose
= f(x) = 4, x 1 and if lim f(x) = f(1) .What are possible values of a and b?
x →1
b − ax, x > 1 (JEE Advanced)
Sol: Here =
lim f(x) lim
= f(x) lim f(x) .
x →1− x →1+ x →1
a + bx, x < 1
We have,
= f(x) = 4, x 1 Left hand lim= f(x) lim [a + bx] = lim[a + b(1 − h)] =a + b
x →1− x →1− h→0
b − ax, x > 1
M a them a ti cs | 14.9
lim f(x)
= lim
= a 0,b
f(x) lim f(x) ⇒ a + b = b – a = 4 ⇒ = = 4
x →1− x →1+ x →1
x− | x |
, if x ≠ 0
Illustration 14: If f(x) = x show that lim f(x) does not exist (JEE ADVANCED)
2, if x = 0 x →0
Sol: Here if left hand limit is not equal to the right hand limit of the given function then the lim f(x) does not exist.
x →0
(0 − h)− | 0 − h |
Left hand limit of f at x = 0 = lim f(x) = lim f(0 − h) =
lim [Putting x = 0 – h]
x →0 − h→0 h→0 (0 − h)
−h − h −2h
= lim = lim = lim = 2 2
h→0 −h h→0 −h h→0
(0 + h)− | 0 + h |
Right hand limit of f at x = 0 = lim f(x) = lim f(0 + h) =
lim [Putting x = 0 + h]
x →0 + h→0 h→0 (0 + h)
h− | h | h−h 0
= lim = lim = lim
= lim= 0 0
h→0 h h→0 h h→0 h h→0
Here, left hand limit of f (at x = 0) ≠ right hand limit of f(at x = 0). Therefore lim f(x) does not exist at x = 0
h→0
PLANCESS CONCEPTS
Hence the limit doesn’t exist. Remember that the limit must be applied only after complete simplification.
Case I: lim f(x) and f(a) both exist but are not equal.
x →a
x2 − 1
, x ≠1 x2 − 1
Example: f(x) = x − 1 ; lim f(x)
= lim = lim(x + 1)
= 2 , f(x) exists at x = 1
x →1 x →1 x − 1 x →1
0, x =1
f(1) = 0, value of f also exists at x = 1. But lim f(x) ≠ f(1) .
x →1
Case II: lim f(x) and f(a) both exist and are equal.
x →a
1 4 . 1 0 | Limits, Continuity and Differentiability
2
Example: f(x) = x2 ; lim
= f(x) lim(x
= ) 1 limit exists, and f(1) = (1)2 = 1; ⇒ value of f also exists. ⇒ lim f(x) = f(1)
x →1 x →1 x →1
(d) lim(fg)(x)
= lim f(x) ⋅ lim g(x)
x →a x →a x →a
( )
n
lim ( f(x) ) = lim f(x)
n
(f)
x →a x →a
( )
lim g(x)
lim ( f(x) )
g(x)
(g) = lim f(x) x →a
x →a x →a
PLANCESS CONCEPTS
lim | f(x) | =
| |⇒
/ lim f(x) =
x →a x →a
f(x) f(x) 0
Let lim . If by substituting x = a, reduces to the form , then (x–a) is a common factor of f(x) and g(x).
x →a g(x) g(x) 0
So we first factorize f(x) and g(x) and then cancel out the common factor to evaluate the limit.
Working Rule:
f(x)
To find out lim , where lim f(x) = 0 and lim g(x) = 0 .
x →a g(x) x →a x →a
x3 − 1
Illustration 15: Evaluate lim . (JEE MAIN)
x →1 x − 1
x3 − 1 0
lim Form
x →1 x − 1 0
(x − 1)(x2 + x + 1)
= lim = lim(x2 + x + 1) [After cancelling (x–1)] = 12 + 1 + 1 =3
x →1 (x − 1) x →1
4x2 − 1
Illustration 16: Evaluate lim (JEE MAIN)
x → 2x − 1
1
2
Sol: Similar to above illustration, By factorizing numerator and denominator we can evaluate given limit.
1 4x2 − 1 0
If we put x = , the expression assumes the indeterminate from .
2 2x − 1 0
1
Therefore x − i.e. (2x – 1) is a common factor of (4x2 – 1) and (2x – 1). Factorising the numerator and denominator,
we have, 2
1
= lim(2x + 1) [After cancelling (2x – 1)]= 2 × +1 =1+1 = 2
x→
1 2
2
x2 − 9x + 20
Illustration 17: Evaluate lim . (JEE MAIN)
x →5 x2 − 6x + 5
Sol: Take (x – 5) common from numerator and denominator to solve this problem.
x2 − 9x + 20 0
If we put x = 5, the expression assumes the indeterminate form .
2
x − 6x + 5 0
Therefore (x – 5) is a common factor of the numerator and denominator both. Factorising the numerator and
1 4 . 1 2 | Limits, Continuity and Differentiability
0 (x − 4) (x − 5) (x − 4)
denominator, we have Form = lim = lim
0 x →5 (x − 1) (x − 5) x →5 (x − 1)
5−4 1
[After cancelling (x – 5)] = =
5 −1 4
x3 − 7x2 + 14x − 8
Illustration 18: Evaluate lim (JEE MAIN)
x →2 x2 + 2x − 8
x3 − 7x2 + 14x − 8 8 − 28 + 28 − 8 0
When x = 2 the expression assumes the form =
2
x + 2x − 8 4+ 4−8 0
x 4 − 3x3 + 2
Illustration 19: Evaluate lim (JEE MAIN)
x →1 x3 − 5x2 + 3x + 1
x 4 − 3x3 + 2 0
On putting x = 1 in , we get .
3 2
x − 5x + 3x + 1 0
It means (x – 1) is the common factor of the numerator and denominator. Factorising the numerator and denominator,
we get
(x − 1)(x3 − 2x2 − 2x − 2) (x3 − 2x2 − 2x − 2)
lim = lim [Cancellation of (x – 1)]
2
x →1 (x − 1)(x − 4x − 1) x →1 x2 − 4x − 1
13 − 2 × 12 − 2 × 1 − 2
= [Substitution method]
12 − 4 × 1 − 1
1 − 2 − 2 − 2 −5 5
= = =
1 − 4 −1 −4 4
Note: When the degree of the polynomial is higher, then it is difficult to factorize. So, we apply L’Hôpital’s rule
0 ∞
Note: Most common indeterminate forms are , ,0 x ∞ , ∞ − ∞ ,00 ,1∞ and ∞0
0 ∞
M a them a ti cs | 14.13
PLANCESS CONCEPTS
f(x)
Evaluation of limits using L’Hôpital’s rule is applicable only when becomes
g(x)
0 ∞
of the form or .
0 ∞
0 ∞ 0 ∞
If the form is not or simplify the given expression till it reduces to the form or and then
apply the rule. 0 ∞ 0 ∞
To apply L’Hôpital’s rule differentiate the numerator and the denominator separately.
What you may overlook is the fact that the LH rule is applicable only when the modified limit (obtained
by differentiating the numerator and denominator) also exists. Let’s consider an example to illustrate this
x2 + sinx ∞
point. Consider the limit lim . We see that the limit is of the indeterminate form . Applying
2 ∞
x
x →∞
x2 + sinx 2x + cos x ∞
the LH rule two times in succession, we obtain: lim = lim (again form)
x →∞ x 2 x →∞ 2x ∞
2 − sinx 1
= lim = 1 − lim sinx
x →∞ 2 2 x →∞
Which does not exist. However, only a few moments of consideration are required to conclude that the
limit must exist, because the numerator is x2 + sinx, and since x tends to infinity, the term sinx can be
ignored in comparison to x2 (as sinx only ranges from -1 to 1); the denominator is x2 and so the limit must
be 1. Why did the LH rule go wrong?
Ravi Vooda (JEE 2009, AIR 71)
1.10 Theorems
xn − an
1. Let n be any positive integer. Then. lim = nan−1
x →a x − a
xn − an (a + h)n − an 1
First Proof: Putting x = a + h, we get = = (a + h)n − an
x−a a+h−a h
1
= (an +n C1 an−1h + .... + hn ) − an [Using the Binomial theorem]
h
1 n
= [ C1 an–1 h + nC2 an–2 h2 + .... + hn] = nC1 an–1 + nC2 an–2 h + .... + hn–1
h
xn − an
∴ lim = lim nC1an−1 + nC2an−2h + ....hn−1 = nan–1 (as nC1 = n)
x →a x − a h→0
xm − am
Illustration 20: Evaluate: lim (JEE MAIN)
x →a xn − an
1 4 . 1 4 | Limits, Continuity and Differentiability
xm − am
We have, lim
x →a xn − an
0 xm − am x − a xm − am xn − an xm − am xn − an
form = xlim ⋅ = lim ÷ = lim ÷ lim
0 →a x − a x − a x →a x − a x − a x →a x − a x →a x − a
n n
x5 − 32
Illustration 21: Evaluate: lim (JEE MAIN)
x →2 x3 − 8
x5 − 32 0
When x = 2, the expression assumes the indeterminate form .
3
x −8 0
x5 − 32 x5 − 25 (x5 − 25 ) / (x − 2) x5 − 25 x3 − 23
Now lim = lim = lim = lim ÷ lim
x →2 x3 − 8 x →2 x 3 − 23 x →2 (x3 − 23 ) / (x − 2) x →2 x − 2 x →2 x − 2
xn − an
= 5 × 25−1 ÷ 3 × 23−1 lim = nan−1
x →a x − a
5 × 245 20
= 5 × 24 ÷ 3 × 22 = = × 22 =
3× 2 32 3
xn − 2n
Illustration 22: Find all possible values of n, if lim = 80 , n ∈ N (JEE MAIN)
x →2 x − 2
xn − 2n xn − an
We have lim = 80 ⇒ n ⋅ 2n–1 = 80 ∴ lim nan−1
=
x →2 x − 2
x →a x − a
⇒ n ⋅ 2n–1 = 5×25–1 ⇒ n = 5
5 5
(x + 2) 2 − (a + 2) 2
Illustration 23: lim (JEE ADVANCED)
x →a x−a
Sol: Put x + 2 = y and a + 2 = b and after that solve this by using L’hospital rule.
Putting x + 2 = y and a + 2 = b, we get
5 5
5 3
y 2 − b2 5 −1 5 xn − an
= lim = b 2 = b 2 lim = nan−1
y →b y − b 2 2 y →a x − a
3
5
= (a + 2) 2
2
sinx
Illustration 24: Prove that lim =1
x →0 x
M a them a ti cs | 14.15
Draw a circle of radius unity and with its centre at O. Let ∠AOB = x radians
D
C
Join AB. Draw AC ⊥ OA at A. Produce OB to meet AC at C. Draw BD ⊥ OA
From the figure tan x
sin x
Area of DOAB < Area of sector OAB < Area of DOAC
1 x 1
⇒ (OA)(BD) < π(OA)2 < (OA) ⋅ (AC)
x
A
2 2 π 2
B
1
1 1 1 BD AC
⇒ (OA)(OB)sinx < (OA)2 x < (OA) ⋅ (OA)tanx =
sinx,tanx
=
2 2 2 OB OA
1 1 1
⇒ sinx < x < tanx OA = OB
2 2 2
sinx
⇒ sin x < x < tanx ⇒ sinx < x <
cos x
x 1
⇒ 1< < (Dividing by sin x)
sinx cos x
x 1
⇒ lies between 1 and
sinx cos x
When x → 0, cos x = 1
x
∴ When x → 0, lies between 1 and 1
sinx
x sinx
∴ lim = 1 or lim = 1 Proved
x →0 sinx x →0 x
Proof by algebra
lim
sinx
= lim
x − (x3 / 3!) + (x5 / 5!) − ....
= lim
( )
x 1 − (x2 / 3!) + (x 4 / 5!).....
= lim 1 −
x2 x 4
+ ..... =
1
x →0 x x →0 x x →0 x x →0 3! 5!
sin2 5x
Illustration 25: Evaluate: lim (JEE MAIN)
x →0 x2
sin θ
Sol: As we know lim = 1 , Therefore we can reduce given equation by multiplying and dividing by 25.
x →0 θ
= 5(1) × 5(1) =
25
Sol: By using algebra we can reduce given limit in the form of lim f(x) + lim g(x) + lim h(x) , and then by solving we
x →0 x →0 x →0
will get the result.
2cos ( a + (x / 2) ) sin(x/ 2)
= a2 lim + lim 2asin(a + x) + lim x sin(a + x)
x →0 x x →0 x →0
x sin(x/ 2)
= a2 lim cos a + lim + lim 2asin(a + x) + lim x sin(a + x)
x →0 2 x → 0 (x / 2) x →0 x →0
= a2 cos a + 2 a sin a
cot 2x − cosec2x
Illustration 27: Evaluate: lim (JEE ADVANCED)
x →0 x
Sol: By using trigonometric formulae, we can evaluate this problem.
We have, lim
cot 2x − cosec2x
= lim
((cos2x) / (sin2x)) − (1 / (sin2x)) = lim cos2x − 1
x →0 x x →0 x x →0 x ⋅ sin2x
We have, lim
tanx − sinx
= lim
((sinx) / (cos x)) − sinx = lim
sinx − sinx cos x
3 3
x →0 x x →0 x x →0 x3 cos x
2
2 sin(x/ 2) 1 1 1 1
= 1 ⋅ lim ⋅ xlim = (1) =
x →0 4 (x / 2) →0 cos x 2
1 2
1 − cos x
Illustration 29: Evaluate: lim (JEE ADVANCED)
x →0 x2
sinθ
Sol: Reduce given equation in the form of by using trigonometric formula.
θ
2
1 − cos x 2sin2 (x / 2) 2 sin(x/ 2) 1
We have, lim = lim = lim =
x →0 x2 x →0 x2 4 x →0 (x / 2) 2
Illustration 30: Prove that: | sin x | ≤ | x |, holds for all x. (JEE ADVANCED)
π
Sol: We know that sin x < x < tanx therefore by applying the cases, 0 ≤ x ≤ 1 < , −1 ≤ x ≤ 0 and |x| ≥ 1, we can
prove this illustration. 2
M a them a ti cs | 14.17
sin( π − x)
Illustration 31: Evaluate: lim (JEE MAIN)
x →π π( π − x)
Sol: By replacing x by π + h .
sin( π − x) 1 sin( π − x) 1 sinh 1 x → π
lim
We have,= = lim lim =
x →π π( π − x) π x →π ( π − x) π h→ 0 h π ⇒ π − x → 0
1 + cos x
Illustration 32: Evaluate: lim (JEE MAIN)
x →π tan2 x
Sol: Simply replacing x by π + h and using trigonometric limit we can solve above problem.
1 + cos x 1 + cos( π + h)
lim = lim [Putting x = +h]
2
x →π tan x h →0 tan2 ( π + h)
1 − cosh 2sin2 (h / 2)
= lim [ tan ( + h) = tan h] = lim cos2 h
∴
2 2
h →0 tan h h →0 sin h
2sin2 (h / 2) cos2 h 1 ×1 1
= lim = cos2 h lim
= =
h →0
( 2 2
4 sin (h / 2)cos (h / 2) h → 0
) 2
2cos (h / 2) 2 ×(1) 2
sinx − sina
Illustration 33: Evaluate: lim (JEE ADVANCED)
x →a x− a
Sol: Same as above illustration replace x by a +h.
= 2cos a lim
h
1 sin(h / 2)
→0 2 (h / 2)
hlim
→0
( a+h + a ) h
h → 0 ⇒ → 0
2
2
1
= 2 cos a (1)[ a + 0 + a] = 2 a cosa
2
1 4 . 1 8 | Limits, Continuity and Differentiability
1 + cos(2 x)
Illustration 34: Evaluate: lim (JEE ADVANCED)
x→
π ( π − 2 x)2
2
π
Sol: Replace x by + h and then by using trigonometric formulae’s we can evaluate above problem.
2
1 + cos(2 x) 1 + cox2x 1 + cos2 ( ( π / 2) + h) 0 π
We have, lim lim = lim form Put x= + h
0 2
2 2 2
x→
π ( π − 2 x)
x → ( π − 2x)
π h→0
π − 2 ( ( π / 2) + h)
2 2
2
1 + cos( π + 2h) 1 − cos2h 0 2sin2 h 1 sinh 1 2 1
= lim = lim form = hlim = lim = (1) =
h→0 ( π − π − 2h)2 h→0 4h2 0 → 0 4h 2 2 h→ 0 h 2 2
cos x − cosa
Illustration 35: Evaluate: lim (JEE ADVANCED)
x →a cot x − cot a
2sin ( (a − a − h) / 2 ) sin ( (a + a + h) / 2 )
= lim sin(a + h)sina
h→0 cos(a + h)sina − cosa sin(a + h)
2 − 3 cos x − sinx
Illustration 36: Evaluate: lim (JEE ADVANCED)
x→
π (6x − π)2
6
π
Sol: Replace x by +h.
6
= lim
( ) (
2 − 3 ( 3 / 2)cosh− (1 / 2)sinh − (1 / 2)cosh+ ( 3 / 2)sinh )
h→0 36h2
2
2 − 2cosh 1 − cosh 2sin2 (h / 2) 1 sin(h/ 2)
= lim
= = lim = lim = lim
h→0 36h2 h→0 18h 2 h→0 18h 2 9 h→0 h
2
1 sin(h/ 2) 1
= hlim =
9x4 → 0 (h / 2) 36
sinx − cos x
Illustration 37: Evaluate: lim (JEE ADVANCED)
x→
π x − ( π / 4)
4
π
Sol: Replace x by + h.
4
cos ( ( π / 4) + h)
sin ( ( π / 4) + h) − cos ( ( π / 4) + h)
= lim = sin ( ( π / 4) + h + ( π / 2) )
h→0 ( π / 4) + h − ( π / 4)
= sin (h + (3π / 4) )
Note: All of the above illustrations can be solved using L’Hôpital’s rule.
x 2 x3 xn
(ii) ex = 1 + x + + + ..... + ......
2! 3! 3!
x2
(iii) ax =
1 + x(loga) + (loga)2 + ..........
2!
x 2 x3 x 4
(iv) log(1 + x) =x − + − + ........ (|x|<1)
2 3 4
x 2 x3 x 4
(v) log(1 − x) =−x − − − − ......
2 3 4
x3 x5 x7
(vi) sinx =x − + − + ......
3! 5! 7!
x2 x 4 x6
(vii) cos x =1 − + − + ......
2! 4! 6!
x3 2x5
(viii) tanx =x + + + ......
3 15
x2
(ix) ax = 1 + x(loge a) + (loge a)2 + ........
2!
1 x3 1 3 x5
(x) sin –1 x = x + + + ....( −1 < x < 1)
2 3 24 5
x3 x5
(xi) tan−1 x =x − + ..... (–1 < x < 1)
3 5
PLANCESS CONCEPTS
Using these expansions is helpful where the limits are in the indeterminate form. But selecting the
number of terms to use in the expansion varies with problems.
ex − 1
Theorem 1: Prove that lim =1
x →0 x
Proof: We know that
x x 2 x3 x 4 x x 2 x3 x 4
ex =1 + + + + + .... ⇒ ex − 1 = + + + + ....
1! 2! 3! 4! 1! 2! 3! 4!
ex − 1 x x 2 x3 ex − 1 x x 2 x3
⇒ =1 + + + + .... ⇒ lim = lim 1 + + + + .... = 1
x 2! 3! 4! x →0 x x →0 2! 3! 4!
Hence proved.
log(1 + x)
Theorem 2: Prove that lim =1
x →0 x
loge (1 + x)
Proof: Let = y . then,
x
exy − 1 exy − 1
⇒ loge(1 + x) = xy ⇒ 1 + x = exy ⇒ exy –1 = x ⇒ =1 ⇒ ⋅y =
1
x xy
M a them a ti cs | 14.21
Now taking limit, when x → 0
exy − 1
lim ⋅ lim y =
1 [since x → 0 ⇒ xy → 0]
xy →0 xy x →0
exy − 1
1 lim
⇒ 1 ⋅ lim y = = 1
x →0
xy →0 xy
loge (1 + x)
⇒ lim y = 1 ⇒ lim =1
x →0 xy →0 x
Hence proved
Note: If no base of log is mentioned, then it is taken for granted that the base is e.
∴ log a is same as loge a
ex − x − 1
Illustration 38: Evaluate: lim (JEE MAIN)
x →0 x
ex − 1 ex − 1
Sol: As we already prove that lim = 1 , Therefore by writing given limit as lim − 1 we can easily solve
above problem.
x →0 x x →0 x
ex − x − 1 ex − 1
lim = lim −1 = 1 −1 = 0
x →0 x x →0 x
ax − bx
Illustration 39: Evaluate: lim (JEE MAIN)
x →0 x
ax − bx (ax − 1) − (bx − 1)
lim = lim
x →0 x x →0 x
ax − 1 bx − 1 ax − 1 bx − 1 a
= lim − = lim − lim = loga − logb =
log
x →0
x x x → 0 x x → 0 x b
ex − e− x
Illustration 40: Evaluate: lim (JEE ADVANCED)
x →0 x
ex − 1
Sol: Reduce given limit in the form of lim .
x →0 x
1
ex −
ex − e− x ex = lim e
2x
−1 1 e2x − 1 1
lim = lim lim = 2 lim = 2(1) = 2
x →0 x x →0 x x →0 x x →0 e x x →0 2x e0
esin x − 1
Illustration 41: Evaluate lim (JEE ADVANCED)
x →0 x
PLANCESS CONCEPTS
A common mistake made by students pertains to indeterminate limits. Consider a function f(x) = g(x)
h(x). We are given that lim g(x) = 0 . What is the value of lim f(x) ? Many students would say that it
x →0 x →0
is 0. However, the actual answer depends on lim h(x) = 0 . If it is not finite, then the limit of f(x) is
x →0
indeterminate. The point we are trying to make is that in calculating the limit of a function which is the
product of two or more functions, if one of the function tends to 0, then that does not make it necessary
for the entire limit to be 0 as well. Similar, remarks hold for other indeterminate forms. For example, if f(x)
= g(x)h(x), and g(x) → 1 as x → a, it does not necessarily imply that f(x) → 1 as x → a, because if lim h(x)
x →a
is not finite, then the limit on f(x) is indeterminate.
2. CONTINUITY
2.1 Introduction
The word ‘continuous’ means without any break or gap. If the graph of a function has no break or gap or jump, then
it is said to be continuous. A function which is not continuous is called a discontinuous function.
Ex.
(i) (ii) (iii) (iv)
y y y y
y = [x]
y=x
x x x x
0 -2 -1 12 3
y = sin x f(x) = 1/x
(Continuous function) (Continuous function) (Discontinuous at x = 0) (Discontinuous) At Every integer
Figure 14.6
If lim
= f(x) lim
= f(x) f(a)
x →a+ x →a−
i.e. If right hand limit at ‘a’ = left hand limit at ‘a’ = value of the function at ‘a’.
If lim f(x) does not exist or lim f(x) ≠ f(a), then f(x) is said to be discontinuous at x = a
x →a x →a
(i) Left continuous at x = a if, lim f(x) = f(a) i.e. f(a−) = f(a)
x →a−
(ii) Right continuous at x = a if, lim f(x) = f(a) i.e. f(a+) = f(a)
x →a+
Thus, a function f(x) is continuous at a point x = a, if it is left continuous as well as right continuous at x = a.
1 4 . 2 4 | Limits, Continuity and Differentiability
x + a 2 sinx 0≤x<π/4
π π
Illustration =
42: f(x) 2x cot x + b ≤x≤ is continuous in [0, p]. Find a and b. (JEE MAIN)
4 2
π
acos2x − bsinx 2
<x≤π
π π
Sol: By checking left continuous and right continuous for x = and x = we can obtain value of a and b.
4 2
π π π− π π π π
f = +b; f = +a ⇒ + a = +b ⇒ a−b =
4 2 4 4 4 2 4
π π+ π π
f = b ; f =−a − b ⇒ a − b =b ⇒ 2b =−a ⇒ a = , b = −
2 2 6 12
2x2 + 2, x ≤ 2
Illustration 43: Examine the continuity of the function f(x) = , at the point x = 2 (JEE MAIN)
2x, x>0
Sol: By obtaining Left hand limit and right hand limit we will get to know that given function is continuous or not.
−
L.H.L. f(2= ) lim((2 − h)2 +=
1) 5 ... (ii)
h→0
+
R.H.L. f(2= ) lim 2(2 +=
h) 4 ... (iii)
h→0
x + λ, x < 3
Illustration 44:
= If f(x) = 4, x 3 , is continuous at x = 3, then the value of λ is (JEE MAIN)
3x − 5, x > 3
Sol: As given function is continuous at x = 3, hence its left hand limit will be equal to its right hand limit f(x) is
continuous at x = 3
f(3)
∴= lim
= f(x) lim f(x) ⇒ 4 = lim((3 − h) + λ ) = 3 + λ ⇒ λ = 1
x →3− x →3+ h→0
PLANCESS CONCEPTS
The fact that f(x) is differentiable at x = a means that the graph is smooth as x moves from a– to a to a+.
Derivative of an even function is always an odd function.
Anvit Tawar (JEE 2009, AIR 9)
PLANCESS CONCEPTS
• If a function is not differentiable but is continuous at a point, it geometrically implies there is a sharp
corner or a kink at that point.
• If a function is differentiable at a point, then it is also continuous at that point.
• If a function is continuous at point x = a, then nothing can be guaranteed about the differentiability
of that function at that point.
• If a function f(x) is not differentiable at x = a, then it may or may not be continuous at x = a
• If a function f(x) is not continuous at x = a, then it is not differentiable at x = a
• If the left hand derivative and the right hand derivative of f(x) at x = a are finite (they may or may not
be equal), then f(x) is continuous at x = a.
ax + b x ≤ −1
Illustration 45: f(x) = 3 is differentiable for xÎR find ‘a’ & ‘b (JEE ADVANCED)
ax + x + 2b x > −1
Sol: By equating left hand limit to its right hand limit we can obtain ‘a’ & ‘b.
h→0
(
= lim a h2 − 3h + 3 + 1 + )
( − a− 1 + b + a)